Author: Velasquez F.A.  

Tags: trigonometria  

Year: 2005

Text
                    Problemas de
Triücncnietna
Y CÓMO RESOLVERLOS


NUEVA COLECCIÓN RACSO Problemas de TRIGONOMETRÍA y cómo resolverlos RACSO EDITORES Edición Revisada y Corregida Por: Félix Aucallanchi Velásquez
Primera Edición en Español Copyright © 2005 por Félix Aucallanchi Velásquez Prohibida la reproducción total o parcial de esta obra por cualquier método de publicación y/o almacenamiento de información, tanto del texto como de logotipos y /o ilustraciones sin autoriza- ción escrita de los autores y el editor. Caso omiso se procederá a denunciar al infractor a INDECOPI de acuerdo a la Ley N° 13/14 y al artículo N° 221 del Código Penal vigente. Printed in Perú - Impreso en Perú. Imprenta MAQUETI E.I.R.L. Jr. Carlos Arrieta 1319 - Santa Beatriz - Lima 1
SERIE DE LIBROS Y COMPENDIOS CIENTÍFICOS COLECCIÓN RACSO Problemas de y cómo resolverlos Primera Edición
El libro Problemas de Trigonometría y cómo resolverlos, para estudiantes de nivel básico y supe- rior, es una obra colectiva que ha sido concebida, formulada y diseñada por el departamento de Ediciones de RACSO EDITORES, bajo la dirección de Félix Aucallanchi V. La realización de esta obra se encargó a: Juan Carlos Sandoval Peña Wilber Manuel Chilet Cama Mario Orea Chavarria Uriel Aspilcueta Pérez La realización gráfica del libro Problemas de Trigonometría y cómo Resolverlos ha sido efectuada por los siguientes especialistas: Rosario Esther Alpiste Pacheco Sandrita Harline Tarrillo Dávila Maribel Alpiste Pacheco Diagramadoras Paul Farromeque Alegre Diseño de Carátula Supervisión General: Dr. Juan Carlos Sandoval Peña Supervisión Técnica de la Edición Revisada y Corregida: Luis Cabanillas Dulanto Supervisión de la edición: Miguel Ángel Díaz Lorenzo Hecho el depósito legal en la Dirección de Derechos de Autor de INDECOPI, y amparado a la Ley N" 13714 y al Código Penal (Artículo 221) Primera edición en español Copyright © 2005 por Félix Aucallanchi Velásquez.
PRÓLOGO Colección RACSO es una reunión estructurada de textos orientados a satisfacer , parti- cularmente, al estudiante preuniversitario y, en general, a la comunidad educativa compues- ta por estudiantes del nivel básico y superior así como de docentes, quienes encuentran en estas obras un estupendo complemento a su tarea diaria de aprender temas de índole cientí- fico. Esta colección de libros fue publicada a finales del año 1993, siendo la primera de ellas el texto llamado Problemas de Física y cómo resolverlos, para luego de cerca de 11 años completarse con el último de la serie titulado Problemas de Trigonometría y cómo resolver- los. Parafraseando a los poetas, “ha corrido mucho agua bajo el puente”, es una sentencia que gráfica mejor este proceso, pues en efecto, la realización de cada una de las obras que constituye esta colección ha tomado tiempo para concebirla, madurarla y publicarla, al mar- gen de las dificultades a las que nos enfrentamos los que producimos libros nacionales: falta de financiamiento, dedicación a tiempo completo para la selección del material, la composi- ción de los textos y gráficos, el diseño y acabados concordantes con la naturaleza de la obra, la piratería,...etc. Nada nos ha detenido, el entusiasmo no se ha agotado, por el contrario hemos encon- trado la forma de mejorar nuestras obras, y la palabra clave ha sido capacitación, es decir, hemos continuado estudiando la especialidad a la que nos hemos dedicado desde hace más de 25 años para afinarla y actualizarla, además de perfeccionar nuestra didáctica con cursos de postgrado: Diplomados, Maestrías y Doctorados. Todo ello nos ha convertido en profesio- nales de la educación capaces de elaborar obras que respondan a la exigencia de una educa- ción de calidad por parte de los que utilizan nuestros textos. Sirvan estas líneas para agradecer la preferencia de los miles de lectores que confiaron en nuestro trabajo, y que por ello los recomendaron a las siguientes generaciones Muchos de aquellos lograron alcanzar sus objetivos y nos complace ser reconocidos en variados eventos académicos, por muchos profesionales, entre los cuales están quienes utilizaron nuestras obras. Para usted que lee ésta presentación, que es un lector actual, le hemos preparado una NUEVA COLECCION RACSO, renovada, actualizada, madurada y didácticamente superior a nuestra anterior colección. En efecto, hemos utilizado todos los recursos de los que dispone- mos actualmente en términos pedagógicos: mapas conceptuales, ayudas de comprensión teórica a través de resúmenes pertinentemente actualizados, estrategias de resolución de problemas, resoluciones argumentadas y una efectiva selección de problemas propuestos. Todo esto convierte a cada texto de la colección en una máquina de autoaprendizaje, aspi- rando a lograr con ello el desarrollo de la capacidad de aprender a aprender, que es un caro postulado del constructivismo moderno. Los autores de esta NUEVA COLECCIÓN RACSO, son profesionales de la educación que poseen un perfil definido y común: Son ingenieros o licenciados en ciencias, son además licenciados en educación matemática, física o química, son diplomados en didáctica de su especialidad, son magísteres en el área de investigación educativa y a la fecha la mayoría de ellos son doctorandos, además de ser docentes cuya experiencia laboral la han desarrollado en las mas prestigiosas instituciones educativas en tres niveles de educación: secundaria,
preuniversitario y universitario. Un equipo así consolida nuestra labor editorial y nos da la confianza de producir textos de calidad. Este equipo labora en coordinación permanente con nuestra casa editorial y es el grupo de profesionales que tiene a su cargo los talleres de capacitación que realizamos para tratar temas educativos y presentar nuestras obras. Dado que los problemas educativos de índole académico, conviven con nosotros y exis- te una comprob ida inoperancia de la mayoría de las universidades en hacer investigación en el campo educativo para identificar sus causas y proponer soluciones concretas, este equipo de profesionales, del cual formo parte, ha tomado la iniciativa de asumir tal tarea desde el sector privado y se ha constituido en el CENTRO POLYA DE INVESTIGACIÓN EN DIDÁCTICA- CEP1D, para quienes les aguarda la misión de contribuir a identificar y proponer soluciones a los principales problemas de tipo pedagogi :o que imposibilitan el adecuado aprendizaje de las ciencias. Los resultados de tales investigaciones se publicarán en esta casa editorial, con lo cual estamos asumiendo el compromiso de mejorar la calidad de la educación en nuestro país. No puedo pasar por alto el reconocimiento que merecen, de nuestra parte hacia ellos, los promotores de colegios, centros preuniversitarios privados y directores de los centros preuniversitarios de las distintas universidades a lo largo y ancho del Perú, quienes por su oportuna decisión optaron por revisar, primero, nuestras obras y luego emplearlas como ma- terial de clase o de complemento en sus bibliotecas. A ellos nuestro profundo y sincero agra- decimiento por tal oportunidad. Quiero anunciar también que nuestros lectores muy pronto podrán acceder a nuestra página web, que crearemos con el fin de complementar sus aprendizajes en algún área espe- cífica de su interés, poniéndonos a tono con el uso de las nuevas tecnologías de información y comunicación que venimos proponiendo en nuestras obras escolares. Para concluir, quiero manifestar mi complacencia por las iniciativas que vienen toman- do las universidades privadas en cuanto se refiere a redefinir el perfil del profesional que desean formar, lo que los ha conducido a redefinir el perfil del alumno que desean que ingre- sen en ellas, todo a la luz del proceso de certificación universitaria en que se encuentran trabajando. Esto a su vez ha obligado a modificar la estructura y contenido del examen de ingreso, en los que ahora las preguntas ya no sólo se elaboran para establecer el nivel de conocimientos de parte del postulante, sino, reconocer las habilidades que éstos poseen ante situaciones problémicas contextualizadas. Sin duda, este cambio contribuirá a mejorar el proceso de selección de los postulantes, así como a la propia preparación preuniversitaria. Espero que lo mismo ocurra con el resto de universidades nacionales. A los profesionales que se dedican a la preparación preuniversitaria les aguarda la tarea de adecuación, adaptación y superación ante estas nuevas condiciones académicas. Buena suerte. La NUEVA COLECCIÓN RACSO, ha contemplado estos cambios y cree también que éstos apuntan a mejorar la calidad de la educación en sus niveles básicos, pues, en ellos el examen de admisión es un referente para diseñar los procesos de enseñanza aprendizaje que se llevan a cabo en las aulas Por todo ello, aspiro que esta colección se encuentre a la altura de las nuevas exigencias. Hasta pronto. Félix Aucallanchi Velásquez
AL PROFESOR Problemas de Trigonometría y cómo resolverlos, es un texto elaborado con el propósito de que los docentes encuentren en él un complemento para la elaboración de su material educativo así como una guía para diseñar sus actividades de enseñanza aprendizaje. Las principales y permanentes dificultades que encuentran los docentes en la elabora- ción de sus materiales es la selección de problemas para la realización de sus clases, talleres, seminarios, prácticas calificadas, tareas, etc. El texto ha sido elaborado para que el docente encuentre aquí un oportuno y eficaz recurso. La trigonometría es una parte de la matemática cuya enseñanza requiere de saberes previos en Sistemas Numéricos, Algebra, Funciones y Geometría. Sin duda se trata de una ciencia cuya enseñanza demanda de mucha creatividad de parte del docente para diseñar sus actividades pedagógicas, lo cual a su vez requiere de un dominio de las ciencias afines indicadas y de modernas estrategias didácticas. En esta obra el docente encontrará que el texto se ha dividido en tres partes: La teoría y enunciados de problemas resueltos, La resolución de problemas y Los enunciados de proble- mas propuestos. Podría sorprender, a primera vista, que no hay razón para separar los enunciados de los problemas de sus respectivas resoluciones, sin embargo, sí las tenemos. Investigaciones educativas hechas sobre este aspecto demuestran que la mayoría de los Estudiantes que leen los problemas, no los intentan si la resolución se encuentra al pié, perdiéndose de este modo la posibilidad de desarrollar habilidades matemáticas en el estudiante. En trigonometría como en cualquier otra área de la matemática, es limitado el número de problemas que se proponen a los estudiantes y que tienen la característica de ser contextualizados, es decir, ser suficientemente reales. En este texto hemos dado una adecua- da cabida a este tema en situaciones problémicas que revelan la intención de plasmar constructivamente nuestra concepción pedagógica y la tendencia actual de la enseñanza de las ciencias. • En el texto se incluyen temas que van más allá de la formación e información escolar, sin embargo y si usted lo cree conveniente, puede desarrollarse una parte o el íntegro de los mismos sin poner en riesgo su comprensión y conexión con el resto de los temas. Todo esto pasa por la generosidad del tiempo que se le destine a su enseñanza. En trigonometría existen temas específicos en los que se requiere bastante claridad y precisión en los conceptos y definiciones, por lo que se constituyen muchas veces en aspec- tos que provocan controversia. En ellos hemos puesto especial énfasis y cuidado, y son: Cir- cunferencia Trigonométrica, Ecuaciones Trigonométricas, Inecuaciones Trigonométricas, Lí- mites y Derivadas Trigonométricos, entre otros. Los enunciados de los problemas han sido cuidadosamente redactados para poder iden- tificar claramente qué habilidad matemática se pretende desarrollar con cada uno de ellos: Calcular, Demostrar, Identificar, Visualizar, Resolver, Aproximar, Algoritmizar, Definir,...etc. En- señar matemática es sólo un pretexto para desarrollar habilidades y convertir a las personas en seres competentes, por ello proponemos aprender una matemática para la vida. Al res- pecto es conveniente revisar los trabajos del cubano Dr. Delgado Rubí del 1SPJAE. Las resoluciones se han elaborado trazando la estrategia mas adecuada, de este modo el estudiante puede reconocer qué pasos se han propuesto seguir para llegar a establecer la solución, y no jugamos con él al gato y al ratón, es decir, la resolución no le resultará inespe- rada, si no será una consecuencia lógica de un plan previamente diseñado. Esperamos que esta forma de presentación del texto logre satisfacer su exigente selec- ción de materiales educativos.
AL ESTUDIANTE Los libros de la Nueva Colección Racso, son una apuesta por el desarrollo de las cien- cias en nuestro país, por el desarrollo de nuestros pueblos y por el crecimiento cultural de nuestra nación. Postulamos que una educación de calidad pasa por muchos aspectos, entre otros una adecuada condición para el estudio, la selección de la institución educativa, de buenos profesores y de buenos materiales educativos. En este último rubro se encuentra nuestra propuesta bibliográfica. Problemas de Trigonometría y como resolverlos es el último de una serie de textos que se vienen publicando desde hace casi once años y con relativo éxito. Este texto presenta el curso de Trigonometría de un modo práctico, es decir, mostrando su aspecto aplicativo a través de situaciones problémicas concretas. ¿Qué requieres saber para aprender, comprender y aplicar los conceptos, definiciones y teoremas de la Trigonometría? Sería extenso recordarte todo lo que se supone has aprendido hasta ahora, pero en la medida que hayas desarrollado una cultura matemática se hará mas comprensible esta importante área de la ciencia, sin embargo, vale la pena puntualizar as- pectos que deben merecer una permanente atención y evocación y son: Sistemas Numéri- cos, Algebra, Funciones y Geometría. De lo primero necesitas recordar los campos numéricos y sus propiedades, de lo segun- do la capacidad de traducir situaciones concretas en expresiones matemáticas así como sus propiedades, de lo tercero debes recordar que el nivel de correspondencia entre dos o mas elementos se puede expresar por una regla y de lo último la capacidad de visualizar y modelizar los cuerpos a través de figuras. Puesto que nos asiste la autoridad intelectual y profesional, adquirida por nuestra capacitación y por el ejercicio de su aplicación, es que hemos creído conveniente dividir este texto, es decir todos los capítulos, en tres partes: 1. Teoría, P oblemas Modelos, Estrategias de Resolución y Enunciados de Problemas Resuel- tos. Aquí encontrarás, en cada capítulo, un resumen teórico de todo el tema, una adecuada selección de problemas modelos resueltos que te permitirá reconocer la forma de presen- tación de los mismos y cómo es que se plantean sus resoluciones. Asimismo te detallamos, en un cuadro aparte, las estrategias que recomendamos para la resolución de problemas del tema. Luego observarás los enunciados de una vasta selección de problemas para que los intentes por tu cuenta. Te lo repito, esta forma de presentación la encontrarás hasta terminar con el último capítulo. 2. Resolución de Problemas. En esta segunda parte encontrarás las resoluciones de cada uno de los problemas que leiste e intentaste en la primera parte. Nos interesa que desarrolles tu capacidad de argumentar, por ello las resoluciones se presentan bien fundamentadas, al punto que tú puedas continuar con la resolución si acaso el método no es el mismo que tu empleaste en tus intentos. 3. Enunciados de Problemas Propuestos. En esta última parte encontrarás una batería de pro- blemas de cada tema, seleccionados adecuadamente y en un nivel progresivo de dificultad. Cada una de estas partes las puedes identificar por el pié de página con los siguientes códigos: : Teoría, número del capítulo y número de página. A : Resolución de problemas, número del capítulo y número de página : Problemas Propuestos, número dél capítulo y número de página. Esperando que hayas comprendido el mensaje /te deseamos ¡Buena suerte!
CONTENIDO | Enunciados | i Resduaonesj 1 i Problemas , Propuestos 1 Sistemas de Medida Angular 12 258 774 2 Longitud de Arco 23 274 778 3 Razones Trigonométricas de Angulos Agudos 32 288 784 4 Resolución de Triángulos Rectángulos 41 302 788 5 Razones Trigonométricas en Situaciones Contextualizadas 50 315 794 6 Razones Trigonométricas de ángulos en el Plano Cartesiano 59 328 798 7 Circunferencia Trigonométrica (Razones Trigonométricas de Números Reales) 70 343 803 8 Identidades Trigonométricas 84 362 808 9 Identidades Trigonométricas de Arcos Compuestos 92 386 812 10 Reducción al Primer Cuadrante 101 407 816 11 Identidades Trigonométricas del Arco Doble 109 421 820 12 Identidades Trigonométricas del Arco Mitad 117 437 824 13 Identidades Trigonométricas del Arco Triple 123 447 829 14 Transformaciones de Sumas o Diferencias a Productos 130 462 833 15 Transformaciones de Producto a Sumas o Diferencias 137 478 838 16 Sucesiones y Series Trigonométricas / 144 494 842 17 Funciones Trigonométricas 151 514 846 18 Funciones Trigonométricas Inversas 166 542 856 19 Ecuaciones e Inecuaciones Trigonométricas 183 584 862 20 Resolución de Triángulos Oblicuángulos 206 653 868 21 Estudio de la Trigonometría con Números Complejos 224 691 872 22 Límites y Derivadas Trigonométricos 234 710 877
Esta obra está dedicada a los estudiantes que intentan forjarse un mejor destino en circunstancias que incluso no les son favorables.
PROBLEMAS DE TRIGONOMETRÍA V COMO RESOLVERLOS ENUNCIADOS DE PROBLEMAS RESUELTOS J¡ NIL1!' J'i1 Ji LJ' 1° [ÍN l| lR FF iíc ífe Fío
1.1. ÁNGULO TRIGONOMÉTRICO Definición.- Es aquel ángulo que se genera por la rota- ción de un rayo al rededor de un punto fijo llamado vértice (la rotación se realiza sobre un mismo plano) desde una posición inicial (lado inicial), hasta una posición final (lado final). Cuando la rotación se realiza en sentido antihorario (O), la medida del ángulo generado es de signo positivo, en cam- 110 cuando la rotación se realiza en sentido horario (0), la medida del ángulo es de signo negativo. En-general la medida del ángulo trigonométrico toma cualquier valor real. a ángulo de valor positivo (<t > 0) 0: ángulo de valor negativo (0 < 0) Nota.- Por convención al ángulo nulo se le considera ángulo trigonométrico, a pesar que no se genera de una rotación. l.IAÁnguIo de una vuelta Id) Es aquel ángulo trigonométrico en el cual el rayo vuelve a su posición inicial por primera vez 1.2. SISTEMAS ANGULARES_________________________________________________ Para la medición de ángulos, tenemos tres sistemas llamados: Sistema Sexagesimal (Inglés) ; Sistema Centesimal (Francés) Sistema Radial o Circular (Internacional) TI Problemas de Trigonometría y cómo resolverlos ^RACSO W^BDITOKBB
1.2A Sistema Sexagesimal (Sistema inglés) Su unidad de medida es el grado sexagesimal (Io), que se define como: 10 = => m Zlt = 360° Equivalencias: Io = 60minutos sexagesimales = 60' 1' = 60 segundos sexagesimales = 60" .-. Io = 3 600" El empleo de estas unidades se denota así: a° + b' + c" — a° b’c" 1.2B Sistema Centesimal (Sistema francés) Su unidad de medida es el grado centesimal (ls) , que se define: ,S = 'Z400<; m Z lc = 40°8 Equivalencias: ls = 100 minutos centesimales = 100ra lm = 100 segundos centesimales = 1005 ls = 10 000s El empleo de estas unidades se denota así: x® + y171 + zs = x®ymzs 1.2C Sistema Radial (Circular o Internacional) Tiene como unidad de medida al radián (1 rad), que se define así: 1 rad = m Z Ir = 2ítrad ¿n Interpretación geométrica del radián Geométricamente 1 rad, es la medida de un ángulo central, en el cual la longitud del arco subtendido es igual a la longitud del radio de la circunferencia, tal como se indica en la figura. CONCLUSIONES: i) 360° = 400® = 2 n rad i) 180° = 200® = tirad O bien: 9o = 10g tirad = 180° tirad = 200g (rt = 3,1416) 1.3. RELACIÓN NUMÉRICA ENTRE LOS TRES SISTEMAS___________ S = número de grados sexagesimales C = número de grados centesimales R = número de radianes Sistemas de Medida Angular
Siendo S, C y R los números que representan las medidas sexagesimal, centesimal y radial de un mismo ángulo, los se relacionan de la siguiente forma: Wo-¿-7 s’9‘ c=,°* Donde k es una constante de proporcionalidad. 1.4. CUADRO COMPARATIVO DE LOS TRES SISTEMAS |QJ| E mZit hJUMWCAS QUE LAMSXM m MEHr DE MINUTO* N-LIMERO DE tCXJlMXB RELACIONES NUMÉRICAS 360° "a *5 *0 II II II £88 o A s (grados sexagesimales) 60 S 3600S s _ c 180 200 R n 400® l8=100m lm = too5 1®= 10000* c (grados centesimales) 100C 10000C s _ c 9 - 10 S _ R 180” n z 2nrad — R — — c._ A 200 n EQUIVALENCIAS 15°= rad 45°= rad 4 75°=^-rad 180° =n rad 18°=^nrf 90°= 2 raá '225°= ^-rad 4 22°3(r- ^-rad O 60°-y rad 120°--y- rad 270°=-^ rad 30°= ^rad O 67° 3CT = rad O 135° = -^- rad 4 300°= -y rad 36° =^rad 3 71°= ^-rad 3 150° =~rad O 360° = 2nral T1 Problemas de Trigonometría y cómo resolverlos t^RACSO UtDITOlli
PROB. I Según la figura, expresar x en términos de a y p. RESOLUCIÓN A partir de esto se observa que: Graficamos los ángulos en sentido antihorario y construimos el ángulo a-x. (20° -x) - 2x + (40° -x)-3x + (60° -x) = 360° => - 8x = 240° x = -30° PROB. 3 Reducir la siguiente expresión: Este gráfico nos muestra que: (a-x)+(-P) = 180° x = a-p-180° 90°+^ rad+1008 E =---------------- 30°+50s +^rad RESOLUCIÓN Expresando todos los ángulos en radianes: PROB. 2 De la figura que se muestra, determinar el valor del ángulo x. E = ^rad+rad+rad £rad+%rad+£¡rad 6 4 12 PROB. 4 RESOLUCIÓN Siendo: S y C los números de grados sexagesimales y centesimales respectiva- mente y R el número de radianes de un mismo ángulo, reducir: Graficando los ángulos en un solo sentido (sentido antihorario), tendremos: ^+S-+i N_ .20*18 1X1 “ C-S + R E = 3 Sistemas de Medida Angular
RESOLUCIÓN 171"x' 12" = 171° 53'12' Se sabe que: S = 9/? ; C = 10/? y R = -xtt Finalmente: x = 53 Luego: 2 2 K N = —----------- 10/?-9/? + /? N = /? + /? /? + /? N = I PROB. 6 Se sabe que 25 grados en un sistema "N" equivalen a 60 grados sexagesimales ¿A cuántos radianes equivalen 5 grados "N"? RESOLUCIÓN PROB. 5 Calcular el valor de x en: ITFx’ 12" = 3rad 25 grados "N" < > 60° 5 grados "N” < > x RESOLUCIÓN Sacando quinta a la condición inicial, encontramos que: Sabemos que: 1 rad = 57° 17' 44" Luego: 171' a' 12" = 3 (57° 17'44") 5 grados "N" < > 12° A continuación, transformamos dicho ángulo a radianes, obteniendo: 17!°y 12" = 171°5r 132” Z+ÍZ A continuación procedemos a efectuar la suma de minutos, deduciéndose que : 12° 180° 15 Luego 5 grados N equivale a: 7t Í5rad ESTRATEGIAS DE RESOLUCIÓN 1) Ante situaciones problémicas en donde se presenten ángulos orientados (ángulos trigonométricos), éstos se deben graficar en un solo sentido, de preferencia en sentido antihorario (positivo) 2) Cuando los ángulos trigonométricos estén expresados en diferentes sistemas, se deben transformar todos a un sedo sistema. 3) Si la condición del problema incluye a los números S, C y R (convencionales), se recomienda reemplazarlos por las siguientes relaciones: S = 9k ; C= 10 k ; R= Problemas de Trigonometría y cómo resolverlos
Enunciados de Problemas —¿i z; con Resolución ÁNGULO TRIGONOMÉTRICO 01.- De la figura mostrada, expresar x en térmi- nos de 6. A)2n+6 B)2n Qti-0 D)0 E)-2ti-6 05.- Del gráfico mostrado, calcule: 02.- De la figura mostrada, determine “x” en términos de “a”. A)a+36O° B)a+18Cf C) 2a -360° D)360p-a E)18Cf-a 06.- De la figura mostrada determine: “x + y en radianes. A) n/3 B)7t/2 Qti/4 D) 371/4 E)7i/5 03.- De la figura mostrada, determinar “x” 07.- De la figura mostrada, calcular “x” A) 15° B)20° C)25° D)30° E)45° (5-llx)' 27x" A)-2 B)-l Q5 D)4 E)3 04.- De la figura mostrada, evaluar el ángulo “x"’. A) 40“ B)20° 0-20“ D)-50° E)-10° 08.- Del gráfico mostrado a qué es igual: lOx-9y A) 1 100 B)360 C)280 D)2400 E) 1800 Sistemas de Medida Angular
09.- En la figura mostrada, calcular (en rad) el valor del ángulo a para que el ángulo 6 sea máximo. A) 3,34 B)2,6 Q4.2832 D) 1,7431 E) 2,1406 14.- Al convertir 7i/50 rad al sistema sexagesi- mal se obtiene A° B'. B-2A Calcula: A) 7 B)5 C)U D)-2 E)-3 15.- Si se verifica que: rad < > x° y'z" ; (a, y, z e N). Calcular el complemento de: (x + y - z)°. A) 83° B)60° C)53° D)30° E)12° 10.- En la figura mostrada, si OB y OC trisecan al ángulo AOD entonces la expresión inco- rrecta es: ---g ---m 16.-Si se cumple que: 37,98° o AB B0 ; o 2 determinar el valor de: M = A“ - B CONVERSIONES 11.- Evaluar: A) 10a+90=0 B)180P-cat=0 C)2OOP+0n=O D)380P=7t(a-0) E)900P=71(96 +5a) A 1*+2*+3*+4*+...+ 2005* M- i“+2o+3o+4<>+... + 2005" ’ A)3 B)5 Q7 D)9 E)ll A) 10 B) 12 C) 14 D) 16 E) 18 17.- Sabiendo que: i i ka £ + ¿/ + 5 + e calcular: M =----------r— fl + c + 4 A) 1 B)2 C)^ D) | E)3 18.- Se tiene un ángulo en el sistema sexagesimal cuya medida es de 16°15'36". De- terminar su equivalente en el sistema radial considerando n = 3,14. 12.- Dadas las siguientes medidas angulares: a=0,5236raJ ; P=30s50m ; 0 = 27°25' ordenar de menor a mayor. Utilizar: ti = 3,1416. A) P < a < 6 B)6<P<a C)a<P<6 D)6<a<P E)a<6<P 13.-Convertir: 1" 15'a radianes. A) 188 rad B) 90 rad C) 144 rad A) 0,5298 B) 0,4326 Q 0,3524 D)0,2836 E)0,1620 19.- La medida de un ángulo en el sistema sexagesimal es de 36°15'45". Calcule dicho án- gulo en el sistema radial. Considere 7t =3,14. A) 0,8543 rad B) 0,7265 rad C) 0,6326 rad D) 0,5214 rad E) 0,4318 rad D) roorad 18 Problemas de Trigonometría y cómo resolverlos A S.ACSO M dITOU
20.-Si: a + b + c = 63. jfy'z" =a°b'c" + c°a’b" + b°ca" x-y entonces al calcular: W =------ se obtiene: A) 50 B)40 C)30 D)20 E) 10 MEDIDAS ANGULARES RELACIONADAS 21 .- Sabiendo que: SR = donde: S.Cy R son los números que representa la medida de un ángulo en los sistemas: sexagesimal, centesimal y radial; calcular: A)VR B)R C)2jR D)R2 E)2R 22 .- Los números S y C que representan la medida de un ángulo en grados sexagesimales y grados centesimales están relacionados por: n 7t 5 = x + — ; C = x+ — Calcule la medida de dicho ángulo en radianes. .. r. ,.\ 3?t 5?t 7 tí A 60 B)W C>80 D,1Ó E)Í00 23 .- Si S y C son los números que representan la medida de un ángulo en el sistema sexagesimal y centesimal, entonces al calcular. W = ~~13 yJc+Js Vc-Vs se obtiene: A)8 B)7 C)6 D)5 E)4 24 .- Si S y C son los números que representan la medida de un ángulo en los sistemas con- vencionales, éstos verifican: n / S = ( 1 + é)0 + cTí X1 + C+2 términos Calcular la medida del ángulo en el sistema sexagesimal. A)»” B>(i) o(to)’ D)(ior E)(^J 25 .- La mitad del número que expresa la medi- da en grados sexagesimales de un ángulo ex- cede en 52 al quíntuplo de su medida en radianes Calcule dicho ángulo en grados centesimales, considerando n = 22/7. A) 160 B) 150 C)140 D)130 E) 120 26 .- La media armónica de los números que representan la medida de un ángulo en grados sexagesimales y centesimales es igual a 36 ve- ces el cuadrado de la media geométrica de las mismas. Halle el ángulo en radianes que sa- tisface la condición dada. A) 6840 B) 5200 C) 4360 r., p x > 3820 ' 252Ó 27 .- El número que representa la medida de un ángulo en grados centesimales mas el triple del número que representa la medida del mis- mo en grados sexagesimales es 37/ti veces el cuadrado del número que representa su medi- da en radianes. ¿Cuál es la medida del ángulo no nulo en radianes?. A) 50 B)40 C)30 D)20 E) 10 28 .- Al sumar los números de ('9 y (m) que dan la medida de un ángulo se obtiene 367 400. Encontrar dicho ángulo en el sistema radial. A) 20 20 C) 20 D) 20 E) 20 29 .- Se tiene un ángulo trigonométrico positi- vo, tal que el producto de sus números de mi- nutos sexagesimales y centesimales es igual a: (o + b)2 + 8ab I8ab a. b € R ¿Cuál es el menor valor del ángulo en el siste- ma sexagesimal? A) 1" B) 18" C) 36" D) 18" E)54" Sistemas de Medida Angular
30.- Si a y b son valores que representan el número de (') y (s) de un ángulo respectiva- mente, entonces el valor de la expresión: f nC-20R [tiS-120R J 4a —16¿> W =----------- , es: calcule el valor de R. A) 350 B)200 Q150 D) 100 E)50 31.- La suma de los (") y ('") de un ángulo es 33400. Determinar dicho ángulo en el sistema radial. A) 1X12 B)ti73 C)7t/9 D)ti/90 E)7tZ30 A) 10 B) 20 C) 30 D) 40 E) 50 36.- La semidiferencia de los números que re- presentan la medida de un ángulo en grados centesimales y sexagesimales es a 7 veces su producto como su suma es a 133 veces el nú- mero que representa la medida de ese ángulo en radianes. Encontrar la medida de dicho án- gulo en el sistema radial. 32.- Si S, C, R son los números que represen- tan la medida de un ángulo en los sistemas convencionales, determinar: 36000 A)----— 7t 72000 *0 2 Q Tt2 3600 W = 60—VÍ9 71 Vc2-s2 D) 72000 E) 180 A)5 B)4 C)3 D)2 E) 1 33 .- Si S, C y R representan el número de gra- dos sexagesimales, centesimales y radianes que mide un ángulo, éstos verifican: 10S + 3C + R = 2403,1416 Si además: n= 3,1416, calcular la medida del ángulo dado en radianes. A)~ B)^ C)y D)2n E)n 34 .- Siendo S, C y R los números convencio- nales, y verificándose que: mS + n ,C=20R a 6m + 5n = 77t/12; calcular: 3 5 9 10 2 A>Í B)| C)^ D)^ E)j 35 .- Siendo S, Cy R los números convencio- nales, se sabe que estos verifican: 37.- Si “R" es el número de radianes de un ángulo, que verifica la siguiente igualdad: =2- 1 ; VR-1 calcular la medida de dicho ángulo en el siste- ma sexagesimal. 38.- Si los números SyCrepresentan las medi- das de un ángulo en los sistemas sexagesimal y centesimal respectivamente, y se verifica que: x2(C-S)=x4-x2+l , x>0 ; calcule en radianes el valor mínimo que puede tomar la medida de dicho ángulo. A>15 b>S c)^ e>íó 39.- Si S, C y R representan el número de gra- dos sexagesimales, centesimales y radianes Problemas de Trigonometría y cómo resolverlos ftk RACSO PaDiToaas
que mide un ángulo y que verifican: n 162SCR 12R(C-S) 5n calcular el ángulo en radianes. 5it 1571 277t 1371 2271 A) -y B>4 c)“y D)-6~ ^7 40.- La medida de un ángulo expresado por los números convencionales, verifica que: yS2=xC2 Calcule dicha medida, si además se cumple: A) 108 B)123 C) 120 D) 127 E) 130 44.- El triángulo ABC es equilátero donde AD y AE dividen el ángulo “A” en tres ángulos congruentes. Determine “a + P” en rad anes. 20RS? rcf 9n J +[ioj =X (1) A) 10° B)20° C)40° D)90° E)100P A)5n/4 B)4ti/5 C)4tt/3 D) 3ti/5 E) 5ti/6 ANGULOS EN FIGURAS GEOMÉTRICAS 41.- Determine la medida en radianes, del án- gulo desigual de un triángulo isósceles en el que cada uno de los ángulos de la base es cuatro veces la medida del ángulo desigual. A \ 71 n \ 71 71 T"\\ 7T I-71 A) 9 B)y O5 D) E)^ 42.-De la figura mostrada, se tiene que BD y CD son bisectrices del Z B y Z C respectiva- mente. Determine:mZDen radianes. A) 5ti/9 B)7ti/8 C)ti/3 D) 771/9 E)5ti/4 E)7/12 Sistemas de Medida Angular T1 21
47 .- En la figura mostrada determine el ángulo A en radianes A) 1171/18 B) 12n/59 C) 1371/36 D)3ti/31 E) 5ti/39 48 .- En la figura mostrada determine la medida del mayor ángulo interior en radianes. A)í* 49 .- Los valores que expresan las medidas de los ángulos internos de un cuadrilátero en el sistema M, están en progresión aritmética. Sa- biendo que el menor de ellos mide 5 grados M, encontrar la medida del mayor ángulo interno en dicho sistema, si se sabe que 50 grados centesimales equivale a 40 grados M. A) 140M B)145M C)150M D)155M E)160M 50 .- En un hexágono los ángulos interiores a, b, c. d, e,f están en progresión aritmética, tal quef<e<d<c<b<a. Si la medida del mayor es 125°, calcular la medida del menor ángulo en radianes. 11771 2371 _ 11971 ' 180 ' 36 J 180 17n 1217t D) 16 E180 | REGLA MNEM0TÉCN1CA SOBRE F1 (7t) j El numero n, que se obtiene como la razón de la longitud de la circunferencia al diámetro, tan familiar a todos los estudiantes, hace ya muchos años, ha sido calculado nada menos que con 707 cifras exactas. Esta hazaña de cálculo fue realizada por W. SHANKS (1 873), y aunque en la actualidad este número de cifras ha sido largamente superado, ocurre que estas 707 cifras figuran grabadas a lo largo del friso circular en que se apoya la cúpula del "Palais de la Decouverte". Para ninguna aplicación práctica con p son necesarias tantas cifras, bastando usualmente los valores aproximados 3,14 ; ó 3,1416 ; ó 22/7. De todos modos, como regla mnemotécnica para recordar las 32 primeras cifras, se puede acudir a los siguientes versos, originales del ingeniero R. Nieto París, de Colombia: Soy rt, lema y razón ingeniosa de hombre sabio, que serie preciosa valorando, enunció magistral Por su ley singular, bien medido el grande orbe por fin reducido fue al sistema ordinario usual. Si se sustituye cada palabra por el número de letras que la forma , obtendremos el siguiente desarrollo decimal para rt: ti = 3,1415926535898932384626433832795— Problemas de Trigonometría y cómo resolverlos RACSO Pbditom*b
Es una porción de círculo limitada por dos radios y un arco comprendido entre ellos. mÁB ... longitud de arco Z AOB ... ángulo central r = OA... longitud del radio 2.2. LONGITUD DEL ARCO (fí________________________ Un arco de circunferencia es una porción de ella que es subtendida por un ángulo central y cuya longitud depende directamente de la medida del ángulo que lo subtiende y del radio de la circunferencia a la que pertenece, así: 1 = 0.r 0 < 6<2n Fórmula Especial: De la fórmula anterior se deduce: 2.3. ÁREA DE UN SECTOR CIRCULAR (S) El área de una región circular se puede determinar, utilizando las siguientes fórmulas: c _ 6r2 _ l-r _ ¿ 2 “ 2 “ 20 Nota.- En las fórmulas mostradas, el ángulo central debe estar expresado en radianes. Longitud de Arco T2
2.4. ÁREA DE DIN TRAPECIO CIRCDLAR (Sr) 2.5. APLICACIONES MECÁNICAS 2.5A Número de vueltas (n) Si una rueda de radio r se desplaza, sin resbalar, una distanciad sobre una superficie, el número n de vueltas que habrá dado en dicho recorrido está dado por: d n — o _ Cuando la superficie es curva, el número de vueltas viene dado por: 2.5B1 Engranajes.- Las longitudes de arco, definidas por el contacto entre dos poleas o piñones, son iguales. Esto se denota así: 2.5B2 Poleas.- Las longitudes recorridas por cual- quier punto del borde de las poleas son iguales a la longitud recorrida por un punto de la faja. 2.5B3 Transmisión por un eje.- Los ángulos centrales barridos son iguales, es decir: Siendo: r = radio de la rueda que gira n = número de vueltas que da la rueda /c = longitud recorrida por el centro de la rueda 2.5B Transmisión de Movimientos Los sistemas mecánicos que permiten trans- mitir movimientos pueden ser debido a un contacto entre sus elementos o unidos a tra- vés de una faja o un eje. "RACSO ÍDITO1EI T2 24 Problemas de Trigonometría y cómo resolverlos
PROB. 1 Dado un sector circular de ángulo central 120° y radio 30 m, se pide: a) El valor de la longitud del arco que lo limita. b) El área del sector circular comprendido. RESOLUCIÓN Graficamos: hasta el punto C, calcule el número de vueltas que da la rueda durante su recorrido. RESOLUCIÓN Recordemos que en el juego mecánico de la montaña rusa, el recorrido se ha diseñado de manera que los coches nunca se de prendan del riel que los conduce, con lo que está garantizado que las ruedas puedan girar en contacto con aquél. El ángulo central (120°) lo expresamos en radianes, es decir: 120°. ’J""7 = ^rad =» 120°= ^rad loU o o Luego: a)x = -3- . 30/ti x = 20 mn b) Conociendo el radio y el ángulo central en radianes calculamos el area del sector circular: í|((30m)2 S = \S = 300 ron2 PROB. 2 La figura muestra la rueda de un coche de una montaña rusa, cuyo radio mide «r». Si éste es liberado en A, y de desplaza (sin resbalar) La línea de trazos muestra la trayectoria que describe el centro de la rueda en todo su recorrido, de manera que si solo hay rodadura, se deberá cumplir que el número de vueltas está dado por: "i = 2kÍ57 ; pero: 'ic=Í '5r Longitud de Arco T2 25
Perímetro = 8 m Del gráfico: Perímetro = 2R + L Donde deducimos que: L = 8 - 2R ... (1) Finalmente, el número total de vueltas que da la rueda es: — fl । + /?2 — 25 4 nT = 10 vueltas 40 4 PROB. 3 2 El área de un sector circular es de 4m , su perímetro es de 8 m. Calcular el radio del círculo. RESOLUCIÓN Usando la siguiente relación tendremos: Asc = = 4 => R.L = 8 ... (Z) Reemplazando (1) en (2) obtenemos: R(8 - 2/?) = 8 =$ 8R - 2R2 = 8 Luego : 2/?2 - 8R + 8 = 0 R2-4R + 4 = 0 /?X\-2 2 Area del sector circular: = 4m Finalmente: R = 2 m ESTRATEGIAS DE RESOLUCIÓN 1) Para calcular la longitud de un arco de un sector circular necesariamente el ángulo central debe estar expresado en radianes, de lo contrario, se tiene que convertir previamente. 2) Cuando se determina el área de un sector circular, el ángulo central debe estar expresado en radianes, y luego de acuerdo a los datos utilizar una de las fórmulas dadas del área. 3) El número de vueltas que da una rueda sobre una pista, se determina mediante el recorrido que realiza su centro respecto de la superficie sobre la cual realiza su rodadura. Esta relación sólo se verifica si la rueda no resbala sobre la superficie. Problemas de Trigonometría y cómo resolverlos RACSO *P*DlTOBaB
Enunciados de Problemas con Resolución LONGITUD DE ARCO encontrar “x” en 01.- Calcular la longitud del arco que subtiende un ángulo central de 17 Io 53' 12" en un sector circular cuyo radio mide 40 cm. (Considerar: 1 rad =51° 17'44"). A)688cm B)120cm D)240cw E) 480 cm C) 100cm 04.- De la figura mostrada términos de “a” y “b". b 02.- Del gráfico adjunto, se tiene un sector circular AOB siendo O, A y B centros de los arcos AB, ÓR y OQ respectivamente. Deter- minar: M = + Lg, si: AO = BO - 6 cm. A)^Í A) a-b D)^- a+b &)—h a+b C) 'a+b A) 7t/4 cm B) 7t/2 cm Q3n/2cm D)7t cm E)2tic/h 05.- En la figura mostrada, BOC y AOD son sectores circulares, OA = L¡¿, AB = La- De- terminar la medida del ángulo central (6 > 0) en radianes. 03.- En la figura mostrada, AOB es un sector circular, BDC es una semicircunferencia de centroOp OA ± DOi,mZ. AOB=45°,O1B=4. Calcule el perímetro de la región sombreada. A) 73/71 B)ti/4 C) 71-1/7 D)(75 -1)/2 E) 2>/5 06.- Con la ayuda de la siguiente figura: A)7t(3+V2) B)7t(5 + T2) C)7t(3-V2) D)7t(4-T2) E)7t(4 + V2)+4j2 Calcular: M = A) 1 B)-l Kz-yXy-Jt) I be C) 2 D) -2 x a E) 0 Longitud de Arco
07.- Un móvil se desplaza con movimiento uni- forme sobre un arco de circunferencia cuyo diámetro mide 100 m. Si en 20 s recorre un arco subtendido por un ángulo de 508, ¿cuál es su rapidez en m/sl A) 5ti/8 B)ti/3 C)2ti/5 D) 3ti/7 E) ti/4 ÁREA DEL SECTOR CIRCULAR 08.- En la figura mostrada, AOB y EOF son sectores circulares cuyas áreas están en la re- lación de 16 a 1. Determine en qué relación están las longitudes de los arcos EF y AB. A) 2 B) 1/4 C) 3 D) 2/3 E) 1/2 09.- En la figura mostrada, se cumple que: R(6R + L) = 80 Calcule el área del sector circular AOB. A) ti/40 B) 7t/20 C) Ti/10 D)ti/5 E) ti/3 12 .- Determine el perímetro de la región sombreada en la figura, donde “O” es el cen- tro del arco AB y “M” es el centro del arco ÑB. Además se sabe que: AN = MB = 2 -Jl. . A) 5^2 + 71+ —2~ 71 13 .- En cierta zona de un parque de diversio- nes se ha instalado una regadera a ras del piso; la cual tiene un alcance máximo de 6 m. Des- pués de girar 150°; se barre en la superficie, un sector circular cuya área (en ni") es: A) 10 B) 20 C) 30 D)40 E) 60 A) 371 B) 571 C) 971 D) 1271 E) 1571 14 .- En la figura AOB es un sector circular, AO ± OB, AO = OB = 8 crn; C, D y E son puntos de tangencia. Calcule el área de la re- gión sombreada en cm2. 10.- Se tiene un sector circular en el que si du- plicamos el ángulo central y el radio, obtene- mos un nuevo sector de área “A”. Si el área del primer sector circular es “B”, evalúe: A/B. A) 16 B) 8 C) 4 D) 2 E) 1 11.- En un sector circular de radio V36/7 , la medida del ángulo central es Calcule el área del sector. ¿>E(5fc)m (15¿>)m A) n| 64-¿2 -26] B) 7t[82V2-34] C) 7t[ 128 V2 -176] D) 7t[136 ¿2 +12] E) 71(15272-138] 15.- En la figura mostrada, AOB es un cuarto de circunferencia, siendo AB = 3 -J1 m. Cal- cular (en m ) el área de la región sombreada. ¿•RACSO WlDITOlEl T2 28 Problemas de Trigonometría y cómo resolverlos
m Z AOC = 2a m Z COD = 3a m Z DOB = 4a A) n/3 B) ti/4 C)2ti/3 D) 3ti/4 E) 5ti/2 16.- A partir de la figura, calcular “6” si se sabe que: 13S, = 7S2- Considerar ti = 22/7. A) 1/2 B) 1 C) 1/3 D) 1/4 E)ti/3 17.- Si el área del sector circular AOB es 3n, determine la longitud del arco CD. Además se sabe que; AC = BD = 2 A) 5tt/3 B) 4tt/3 C) n D) 2ti/3 E)ti/3 TRAPECIO CIRCULAR 18.- En la figura mostrada, AOB y OCD son sectores circulares, OA = 3OD. Calcule la re- lación entre el área del trapecio circular ABCD y el area del sector circular DOC. A) 12 B)10 C)8 D) 6 E)4 19.- Siendo “S", “C” y “R” los números que expresan la medida de un ángulo en los siste- mas sexagesimal, centesimal y rad al, se pide calcular: “6” A) Tt/10 B) 10/7t C)2/n D) 71 E) 4/ti 20.- De la figura mostrada, determinal el área de la región sombreada: A) 2 B)4 C)6 D) 8 E) 10 21 .- De la figura mostrada, calcule “0” (en rad), si el área del trapecio circular ABCD es de 5 of A) 1/4 rad B) 1/2 rad C) 1 rad D) 1/3 rad E) 1/5 rad 6 rad 22 .- De la figura mostrada, calcular el área de la región sombreada: A) 6r2 B) 26r2 C) 36r2 D) 46r2 E) 56r2 23 .- En la figura mostrada, determine el valor de “L" si la región sombreada tiene un área de 20 m2. Longitud de Arco 29
A) Im B)3m C) 5 m D)7m E)9m 28.- En la figura mostrada: a = 210-40x , t = 7x2-30x. Si el área del trapecio circular tiene valor mí- nimo, entonces la medida de su ángulo cen- tral en radianes es: 24.- En la figura mostrada se verifica que: 2.OA = AB y OC es bisectriz del Z BOD Determinar: S-JSj. B A) 4,25 B) 3.75 C) 3,15 D) 2,55 E) 1,35 APLICACIONES MECÁNICAS 29.- En la figura mostrada determinar el nú- mero de vueltas que da la rueda de radio r al desplazarse, sin resbalar, por el arco AB = L: A) 2 B) 4 C) 6 D) 8 E) 10 25.- En la figura mostrada, AOB y COD son sectores circulares, en donde: AC = BD = x, OC = OD = 2,I1fi = (x + 2) y I^ = (a- 1) Calcule el área del trapecio circular ABCD. A) 15/2 B) 17/2 C) 21/2 D) 23/2 E) 25/2 26.- El perímetro de ui^ sector circular mide 6 m y su área es de 2 m . Calcular (en rad) la medida del mayor ángulo central que verifica estas condiciones. 30.- Una bicicleta que tiene ruedas de radio “r” recorre una pista circular de radio “/?”, pla- na y horizontal; determinando sobre ésta un ángulo 6o. Determinar el número de vueltas que dará una de sus ruedas. A)^ _6R_ B) 360r o «L 180r A) 1 B) 2 C) 3 D) 4 E) 5 27.- Determinar el área máxima de un trape- cio circular cuyo perímetro es “p”. D)26R 7 nr E) TtR 2 2 2 2 B)^- C)^- D) y- E)p2 2 4 O lo 31.- Sobre una pista circular plana y horizon- tal se desplaza un atleta con una rapidez de 17,6 kmlh y recorre un arco que subtiende un ángulo de 56° en 36 segundos. Calcule (en m) el diámetro de la circunferencia, si: n = 22/7. T2 30 Problemas de Trigonometría y cómo resolverlos RACSO
A) 360 B) 300 C)270 D) 240 E)240 32 .- En el sistema mostrado, el disco A gira 90°. Asimismo se sabe que: rA = 3. rB = 5, r^ = 1. Calcule la medida del ángulo que gira el dis- co C. A) 18° B) 27° C) 36° D)54OE)62<’ 33 .- Los radios de las ruedas de una bicicleta, son entre si como 3 es a 4. Calcular el número de vueltas que da la rueda mayor cuando la menor gire 8n rad. A) 2 B) 3 C) 4 D) 5 E) 6 34 .- ¿Qué distancia recorre el bloque si se gira la manivela un ángulo de 6 rad. Se sabe tam- bién que: q = 6, r2 = 9 , r3 = 12. manivela A) 20 B) 40 C) 60 D) 80 E) 100 35 .- Dos ruedas de radios R y r, tal que: R>r, recorren la misma longitud L. Si la diferencia del número de vueltas de la menor y la mayor L , , r , . es , entonces al evaluar: p-, se obtiene: A) 3/4 B) 1/4 C) 1/2 D) 3/5 E) 1/6 36 .- Determine el número de vueltas que dara la rueda de radio 2 cm, al desplazarse desde “A” hasta tocar la pared vertical (7i = 22/7). 37 .- En la figura mostrada se sabe que n es el número de vueltas que da la rueda de radio r (r = 1 m) al ir del punto A hasta el punto E sobre la superficie indicada. Se pide determi- nar el valor de: 44 n. Asumir que: n = 22/7. A) 125 B) 175 C) 267 D) 295 E) 376 38 .- Los radios de las ruedas de una bicicleta son 20 cm y 70 cm respectivamente Calcular (en m) el espacio recorrido por dicha bicicleta, si se sabe además que la diferencia del número de vueltas que dieron cada una de las ruedas para recorrer el espacio anterior fue 100. (n = 22 / 7). A) 174 B) 175 C) 176 D) 177 E) 178 39 .- Si una rueda de radio "6a" se mantiene fija y otra rueda de radio "a", puede girar al- rededor de ella. ¿Cuántas vueltas dará la rue- da pequeña si parte y llega al mismo punto por primera vez? A) 3 B) 4 C) 5 D) 6 E) 7 40 .- Calcular el número de vueltas que da la rueda de radio Im al recorrer el perímetro de un triángulo si el perímetro de este es de 44 m. Considerar 7i = 22/7. A) 5 B) 6 C) 7 D) 8 E) 9 Longitud de Arco
3.1. DEFINICIONES DE LAS R.T. DE ÁNGULOS AGUDOS_________________ Sea a un ángulo agudo de un triángulo rectángulo, tal como se muestra en la figura, entonces se definen: Cal Opuesto Hipotenusa Cat. Adyacente Hipotenusa TANGENTE Cat. Opuesto Cat Adyacente Hipotenusa Cat. Opuesto SECANTE Hipotenusa Cat Adyacente COTANGENTE Cat Adyacente Cat Opuesto 2 2 2 ' a + b ~c Teorema de Pitagoras c > a ; c > b a sena = — b eos a = — c tana = b , b cota = - seca= ¿ c csc a = — a 3.2. PROPIEDADES_______________________________________ 3.2APropiedad Fundamental Los valores de las Razones Trigonométricas (R.T.) de los ángulos agudos no dependen de la longitud de los lados que la forman, sino de la medida del ángulo definido por ellos. 3.2B Razones Trigonométricas Recíprocas Si a es un ángulo agudo, se cumple que: sen a. csc a = 1 => csc a = I I sena vt — csc a eos a. sec a = I => sec a = 1 eos a = 1 cosa V seca tana, cota =1 => <’ot«=taK(r v tana=<7T<T T3 32 Problemas de Trigonometría y cómo resolverlos A* RACSO WlDITOlll
3.2C Razones Trigonométricas de Ángulos Complementarías (Co-razones) Si ay P son dos ángulos complementarios (a + p = 90°), entonces se cumplirá que: sen a = eos p tan a = cot P sec a = esc p 3.2D Razones Trigonométricas de 30°, 60°, 45°, 37° y 53°. Se obtienen a partir de los siguientes triángulos notables en donde: k g R+ Razón Trigonométrica Notación 30° 37° 45° 53° 60° seno sen 1/2 3/5 <2/2 4/5 <3/2 coseno eos ^3/2 4/5 <2/2 3/5 1/2 tangente tan V3/3 3/4 1 4/3 <3 cotangente cot <3 4/3 1 3/4 <3/3 secante sec 2<3/3 5/4 <2 5/3 2 cosecante CSC 2 5/3 <2 5/4 2<3/3 Sobre ha base de los triángulos anteriores se pueden construir otros, de relativa importancia, para obtener de ellas sus Razones Trigonométricas. 24 k V2 + >/2Jt Razones Trigonométricas de Angulos Agudos
PROB. ] Si a es un ángulo agudo, tal que: tan a = 0,75 , obtenga los valores de: a) Todas las razones trigonométricas de a. b) tan a/2 c) tan 2a RESOLUCIÓN ****♦***♦**♦****♦* a) Si tan a = 0,75 = Entonces por Pitágoras: x2 = (4)2 + (3)2 a 1 tan 2 - 3 c) En base a) primer triángulo construimos un triángulo en su interior donde figure el ángulo 2a. Aplicando Pitágoras en el triángulo rectángulo sombreado. x2 = (3)Z + (4-x)2 => x2 = 9 + 16 + x2-8x Luego: 3 sen ct = 5 . 3 tan « = 4 5 sec a = 4 eos a = . 4 cot a = 2 5 esc a = -3 => 8x=25 => x — ~ O 3 3 Luego: tan 2a = /¡_x = ------25 4-_íT 24 .-. tan 2a = -y- b) A partir del triángulo anterior construimos un nuevo triángulo en donde figure a/2. Entonces se tendrá que: PROB. 2 Si se cumple que: sen (x - 40°) sec (2x + 10o) = 1 tan (3y + 10°) cot (2y + 30°) = 1 sen(x-10°) + cos(y + 40°) cacue. E tan(x + 50) + cot(y + 250) T3 Problemas de Trigonometría y cómo resolverlos 2* RACSO RDlTOBia
RESOLUCIÓN tan (3y + 10°) = tan (2y + 30°) De la primera relación se obtiene; senfjf- 40°). = 1 => senfx - 40°) = eos (2x + 10°) Esta igualdad se verifica cuando los ángulos son complementarios, luego: (x- 40°) + (2x + 10°) = 90° => 3x=120° => x = 40° De la segunda relación se desprende que: tan (3y + 10°) . tarl(2y+30'’) = 1 Esta igualdad se verifica si los ángulos son iguales, luego: 3y+10° = 2y + 30° => Finalmente: E = sen 30°+eos 60° tan45°+cot45° 11 r- 2 2 E= -í—r y =20° ESTRATEGIAS DE RESOLUCIÓN 1) Cuando ui ángulo es agudo, y se conoce una de sus 6 razones trigonométricas, es inmediato el cálculo de los valores de las razones trigonométricas restantes, simplemente construyendo un triángulo rectángulo ubicando a continuación uno de los ángulos agudos, e identificando sus lados, de acuerdo con la razón trigonométrica dada, y finalmente aplicamos el teorema de Pitágoras. 2) Cuando una razón trigonométrica es igual a su re pectiva co-razón trigono- métrica, inmediatamente se debe asumir que la suma de sus ángulos es 90°. 3) Toda vez que una razón trigonométrica de cierto ángulo es igual a la misma razón trigonométrica de otro ángulo, entonces se debe afirmar que dichos ángulos son iguales (pero esto ocurre solo cuando se trata de ángulos agudos) 4) Ante la presencia de las razones trigonométricas de los ángulos de 30°, 60°, 45°, 37° y 53°, debemos utilizar sus respectivos triángulos notables de dichos ángulos. 5) Si se tiene el valor de una razón trigonométrica de un ángulo agudo, y se desea calcular los valores de las razones trigonométricas del ángulo mitad o del ángulo doble, se procede a realizar construcciones geométricas adecuadas. Ra&nes Trigonométricas de Angulos Agudos T3
RAZONES TRIGONOMÉTRICAS DIRECTAS 01.- De un triángulo rectángulo ABC, se cumple: tan A + tan C = 2. Calcular el valor de: M = esc A . esc C A) 1/2 B)1 Q2 D)l/3 E) 3 02.- En el triángulo rectángulo ABC (A=90°), se sabe que: cot C + cot B = 4; entonces al calcular F = 16 sen B.sen C.cos B eos C se obtiene: A) 1/4 B)l/2 C)1 D)2 E)4 03.- Se tiene un ángulo agudo “6” tal que: 21 tane= — 1 Calcular el valor de: M = — sen 6 + 4 eos 6 A) 1 B)2 03 D)4 E)5 04.- En un triángulo rectángulo ABC se sabe que. ni Z. ABC = 90°. En este triángulo se veri- fica que: 2 sen A = esc C 2. Calcular. W = tan C - A) 1/8 B)l/4 Ql/2 D)1 E)2 06.- En la figura mostrada, ni Z ABC = 90°, m/CAl!=a,mZCDB = 6,DB = 3,CB=¿¡. Ademas : tan a + tan 6 = 77. Encontrar el valor de a: A) 124 B)142 C)168 D)186 E)210 05.- Calcula la secante del mayor ángulo agu- do de un triángulo rectángulo sabiendo que sus lados están en progresión aritmética. A)5/3 B)5 C)4/3 D)3/4 E) Jí T3 36 07.- En la figura mostrada m Z ACB=90°, AC = b, BC = a, AB = 2 -Job ,a<b. Calcule: tan 6. A) 72 + 75 B)2- 75 C) 73 - 72 D)2+ 73 E)3- 72 08.- A partir de la figura mostrada, determine el valor de: M = cot a - tan p, si: AB = CD. A) 1/2 B)2 C)3 D)1 E)l/3 Problemas de Trigonometría y cómo resolverlos RACSO
09.- El perímetro de un rectángulo es de 492 m y su diagonal forma con la base un ángulo cuya cotangente es 1,05. Calcule dicha diago- nal (en m). 13.- El perímetro de un triángulo rectángulo es 132 u, y suma de los cuadros de sus lados es 6 050n~. Calcule la tangente del menor án- gulo agudo. A) 140 B)158 C)166 D) 174 E) 182 10.- A partir de la figura, calcule el valor de: 3 5 R 1 A)i C) 3 D)h e>3 2.sen6 11.- En el gráfico mostrado, se sabe que: AD = CD = a; AB = £>. T. Í6\ ' - . Exprese cot 1^1 en términos de «a» y «£»>. 14.- En la figura mostrada, ABCD es un cuadra- do de lado L y E es punto medio de AD. Calcu- le la longitud del arco BC aproximadamente. A)7tL/17 m 7tiL B) 25 Q 9 71717 25tiL > 53 p. 53-J5 7tL } 360 b + Jab 15.- En la figura mostrada m Z. ABC = 90°. Si: 12x tan A = y 10(6c - b) = b-c A) y \2ah-b2 D) b + ^la.b a + -Jbja entonces al calcular x se obtiene: 12.- En un paralelogramo los lados adyacen- tes miden 8 ni y 16 m. Si el ángulo comprendi- do entre dichos lados mide 60°, determinar la longitud de la menor diagonal. A) 2^ B)4£ D)873 E)16-j3 C)673 R.T. RECÍPROCAS 16 .- Calcular el valor de: |(4cos3íF+9señ54“)sec3(?^ M= V cotl8<’.cot72<> A) 13 B)JÍ3 C)5 D) J5 E)3 Razones Trigonométricas de Angulos Agudos T3 37
17 .- Calcular el valor de “x” que verifica: sec(3x—15°) _ senlO°+sen2O°+...+ sen8O° ’Z cosl0°+cos20°+...+ cos80° siendo x un ángulo agudo. A) 15“ B) 12,5° C)16° D)37° E)25° 18 .- Si se verifica que: sen(50°+x) -cos(40°-x)+tan(x+ 10°)-tan(.r+40°)=l 2 3X Determinar: M = sec 3x + cot — A) 1 B)2 Q3 D)4 E)5 19.- Siendo x, y, z ángulos agudos q ue se rela- cionan así: sen(A + j)-cos(85°-j-z) = 0 ... (1) tan 2x. tan 3z = 1 ... (2) Calcular: M = tan(2x + 11°) - tan (a + z) A) 3/4 B)l/5 C)7/9 D)1 E)7/12 20.- Sabiendo que: A = tan 1°. tan 2°. tan3°... tan 45° B = tan 46°. tan 47°. tan 48°... tan 89° Calcular: M = (A. B)2. tan ( A.B.^J A) 3 B)2 Ql/2 D) 1/3 E)1 a£>(senlO°—l) + a2 —£>2cos80° abl cos80°+1)+a2 + ¿2sen 10° 1 + M Simplificar: -—~ A)bfa B)2 C)2a!b D)3b/a E)alb 22 .- En el triángulo rectángulo ABC, recto en B, se verifica que: (sen A)(cosC) + (eos C)(sen A) = Calcu ar el valor de sen A. A) 3/5 B)j2/2 01/5 D) 75/2 E) 1/3 23 .- A part r de la figura mostrada, calcule “a” si: AD = DC y sen (39° - 6) = eos (14° + 36). E)27 24 .- Sabiendo que 6 es un ángulo agudo y que: csc (6 + 20°) = 2 tan 10°.sen 20°.sec 70°. tan 80° calcular: M = eos 66 + tan (56 - 5°) A) 1/2 B)1 C)3/2 D)2 E)5/2 25 .- Sabiendo que se verifican las siguientes relaciones: sen (5a + 2b + c) = eos (20° - 3a) ..(1) eos (4J + e) = eos (40° + e) ... (2) a + b=~ rad ---(3) o 3 Calcular: D = tan (10° + 2c + y J) A) 75 B) 1 C) 75 /3 D) 1/2 E) 1/3 R.T. DE ÁNGULOS NOTABLES 26 .- De la figura mostrada, calcular: “tan 6”, si se sabe que: mZOBC=6 a mZOCB = 37° Además: O es centro de la circunferencia. Problemas de Trigonometría y cómo resolverlos RACSO DITO1II
A) 1/3 B)2/5 C)3/7 D)5/4 E)2/3 27 .- De la figura mostrada, calcular: tan 6. A)1 B)3/4 C) 4/3 D)73/5 E)2/3 28 .- Si ABCD es un cuadrado, calcular: “tan jc” A) 11/19 B) 21/25 C) 13/16 D) 14/19 E)5/12 29.-Calcular el valor de “a” si: 2a (sec 45° - sen 45°)sec 60 = 4 eos 60° - a A) 1 B)-2 C)3 D)-3 E)5 30.- Si se cumple que: sec 0 = tan 60° + sen 30°, donde 0 es agudo Calcular: M = J5 .tan 0 + A)4 B)8 06 D) 10 E)12 31.- Al calcular: Jcot330°-6.sen60° +csc245° -----------------F--------Q cot45o.sen"60°+v3.csc60ü— Se obtiene: A)5 B)4 C)3 D)2 E)1 32 .- Si x es igual a 15°, entonces al calcular: . i* -n tan 4x + cos 3a 2. . - W =---------=-----+ cot 4a, se obtiene un sen 2x número de la forma . Evaluar a + 3 A) 25 B)18 C) 6 D)22 E)35 33 .- De la figura mostrada, calcular el valor de: M = tan(20 - 30°). cot 0 ^loXX^ \ _____________________eX A 14 C A) 1 B)2 C)V3/3 D) E)3/4 34 .- En la figura mostrada ABC es un triángu- lo rectángulo isósceles, donde D es punto medio de BE . Calcule: cot a. Razones Trigonométricas de Angulos Agudos
MISCELÁNEA 43.-SÍ; tan(fl-¿)= 1 a tan (a + b) = -J3 35 .- Los lados de un triángulo rectángulo es- tán representados por tres números en pro- gresión aritmética. Calcular el coseno del me- nor ángulo agudo. A) 0,6 B) 0,5 C) 0,4 D) 0,7 E) 0,8 36 .- En un triángulo isósceles ABC (AB = AC) se sabe que: eos A = 0,6. Calcular tan B. A) 1 B)2 C)3 D) 1/2 E) 1/3 37 .- Si: sen 3x=eos 75°, calcular "v" (agudo) A) 10° B)15° C)20° D)5° E)30° 38 .- Si: tan (2a + 25°) = cot (5a - 5°). Determinar "x" (agudo) A) 10° B)20° C)30° D)40° E)45° 39 .- Calcularx en: tan (jc+ 41°). tan (2a - 3 Io) = 1 A) 26.3° B)26°30’ C)26°40’ D)30°40’ E)30° „ . Í5x-96O\ 1 40 .-Si: cot I--5—I =------, . , , 1 2 ’ “•(*) entonces el valor de x es: A) 36° B)30° C)45° D)20° E)35° 41.- Si: cot 2a - tan 3y = 0 , y , 2a - y — 10° El valor del mayor ángulo agudo es: A) 15° B) 20° C) 25° D) 35° E) 45° 42.-Calcular: a + P ,si: sen a - eos 2P = 0 a sen P. esc 4a = 1 A)20° B)30° C)40° D)50° E)60° Calcular: a±b A) 1 B)3 C)5 D)7 E)9 44 .- En la figura: ¿ Cuál es el valor de ?. 4—2 <3—4 A)4>/3 B)6 06 D) 12 E) 12^3 45 .- ¿Cuál es la distancia x en la figura? A) 240 B)300/-j3 C)400/^3 D)250^- E)175 46 .- El valor de la expresión: cot226+sec26 n -,„n E =----sec20----- ’Para6 = 30 es: A) 10/3 B) 3/2 C)5/6 D) 1/2 E)2/3 Problemas de Trigonometría y cómo resolverlos nACSl'
4.1. DEFINICIÓN______________________________________________ Resolver un triángulo rectángulo es determinar la longitud de sus lados y la medida de sus ángulos agudos, para lo cual deben ser conocidos al menos un lado y un ángulo agudo. En la resolución de triángulos rectángulos se presentan tres casos los que se resuelven por medio de un determinado grupo de teoremas. 4.2. TEOREMAS 4.2A TEOREMA 1. Conocida la Hipotenusa (m) y un ángulo agudo (a). Fig. (a) 4.2B TEOREMA 2. Conocido un ángulo agudo (a) y su cateto adyacente (m). Fig. (a) 4.2C TEOREMA 3. Conocido un ángulo agudo (a) y su cateto opuesto (m). Fig. (a) En general.- En cualquier triángulo rectángulo, se tiene: Incógnita = (Dato) • R.T. (Z) Donde: Incógnita Dato = R.T. (Z) Siendo: Incógnita: El lado del triángulo rectángulo que se desea calcular. Dato: Es el lado del triángulo rectángulo que se conoce. R.T.: Es la razón trigonométrica que corresponde al dividir la incógnita entre el dato. Resolución de Triángulos Rectángulos T4 41
4.3. AREA DE UNA REGIÓN TRIANGULAR Está determinado por el semiproducto de dos de sus lados, multiplicado por el seno del ángulo comprendido por dichos lados. PROB. 1 mn S = serI a => 3 sen 0 + sen 0 = 3 eos 0 + 2 sen 0 1.- Si ABCD es un cuadrado, calcule: En el nuevo gráfico: AD = PQ 2 sen 0 = 3 eos 0 => tan<¡>= sen<¡> 3 cos0 — 2 , A 2 => cot 0 = 3 Luego: tan 0 + cot 0 = “7“ PROB. 2 En la figura: ABCD es un cuadrado, demuestre que: p + r = q + I. RESOLUCIÓN, En la nueva figura, se observa que: UkAC.v D1TOMI i 42 | Problemas de Trigonometría y cómo resolverlos
PR = QS RESOLUCIÓN Entonces: p sen a + r sen a = q sen a +1 sen a Sean m y n los lados del triángulo y a el ángulo comprendido. En el triángulo sombreado: h . sen a = — => h = m sen a m Luego: Área A = basexaltura 2 PROB. 3 Dos lados de un triángulo rectángulo miden myn. Calcula el área de su región triangular. Si además se sabe que el ángulo comprendido entre dichos lados es «a». Área A = nxmsena Finalmente: Área A = sen a ESTRATEGIAS DE RESOLUCIÓN 1) Tratar de buscar triángulos rectángulos en los cuales se conozca mínimamente uno de sus tres lados y uno de sus ángulos agudos. 2) Una vez Que identificamos al triángulo rectángulo con un par conocido: lado y ángulo, se aplica la siguiente técnica. Incógnita = dato(lado) x R.T. agudo) Resolución de Triángulos Rectángulos
Enunciados de Problemas con Resolución TEOREMA 1 OI.- Dada un banderín, como muestra la figu- ra, calcular <«». A) b + a eos a B)b - a eos a C) b + a sen a D) b - a sen a E) b + 2« sen a 02.- En un triángulo rectángulo se conoce uno de los catetos «m» y el ángulo opuesto «6». Calcular la altura relativa a la hipotenusa. A) m sen 6 D) ///(sen 6 + eos 0) B) m eos 0 E) 2/n sen 0 C) ///(sen 0 - eos 0) 03.- Calcular «a» en la figura: A) a - b sen 0 D) a sen 0 + b eos 0 B) a eos 0 + b sen 0 E) a sen 0 - b eos 0 C) a eos 0 - b sen 0 04.- En la siguiente figura, G es el baricentro del triángulo ABC; AD = BD y 3 sen o. - eos a = 3. Calcular la tangente del ángulo DCG. A) 3 B)2/3 C) 1/3 D)3/2 E)l/2 05.- La figura muestra un cuadrado cuya área es 64 //i2 y tal que PC = BP. Calcular AM si AP =6/n. 16 12 /r C) m D) -y V5 m E) 12-73/// 06.- En la figura la longitud del segmento PS y RT es L y la segmento TS es k. El valor de k está dado por: 441 Problemas de Trigonometría y cómo resolverlos ARACSO WPIDITOKXB
A) L (sen 0 - sen a) B) L (sen a + sen 0) C) L (sen a. sen 0) D) L (sen a - sen 0) E) L (sen a + sen 0) pq eos a A) p-gsena pq sen a B) q+ p eos a pq eos a C) q — p sen a pq sen a D) p + q eos a P9 E) psen ct+q eos a 07.- En la circunferencia de radio R se ha ins- crito el triángulo ABC con AB = AC. Si la me- dida del ángulo BAC es 6, entonces la longi- tud del lado BC es: A) R sen 6 B) R sen 0/2 C) 2R eos 0 D) R eos 0/2 E) 2R sen 0 08.- En la figura mostrada, calcular el valor de “a". Si: AC = 4 y m Z BPC = 53°. A) 3 eos a + 4 sen a D) 3 eos a - 4 sen a B)4cosa+3 sen a E)4cota-3seca C) 4 eos a - 3 sen a 09.- De la figura mostrada, m Z ABC = 90°, m Z CBD = a ; AB = p ; BC = a ; BD - q. Calcule a. 10.- En la figura mostrada se cumple: AB = BC=R y sen~a + eos a = M. determinar: PQ. ABC y PBD son sectores circulares concéntricos. A) RM B) R/M C) R(M - 1) D)R(M+1) E)RM2 11.- Si ABCD es un cuadrado m Z EBA = 53°. ni Z DCE = a, m Z BEA = 90°, calcular: W = 5 V10 . eos a. A) 18 B) 15 C)12 D)9 E)6 TEOREMA 2 12.- Las bases de un trapecio isósceles son B y b. Si los lados no paralelos forman con la base mayor un ángulo 0, hallar el área del tra- pecio. .. / B+ ¿A n A) I—2~ I - tan 0 B) cosO Resolución de Triángulos Rectángulos T4 45
Bfe „ C) . sen 6 D) (B-~4 fc2) tan 6 A) V2 -1 B)2j2+1 C) Jl + 1 D)2V2 -1 E) J2 +2 B.b E) 4 tan 6 13.- En un triángulo ABC, recto en B, la me- diana CM y el cateto B A forman un ángulo 6, entonces tan 6 es: A) 2 tan Á B) 2 cot Á C) 2 tan C D) tan Á + tan C E) 2(tan C + cot Á) 14.- La altura de un cono circular recto es h y el ángulo de abertura es 2a. Hallar en función de h y de a, el radio de la esfera circunscrita. A) 0,5 h sen2 a B) 0,5 h eos* a C) 0,5 h tan2 a D) 0.5 h sec2 a E) 0,5 h csc2 a 15.- En la figura mostrada se cumple: AB = CD, m Z BAD = P y /n Z ACD — a, calcular: cot a - tan P 17.- En la figura mostrada se verifica que: m Z ABC = m Z AEB = 90°, m Z CBE = mZ DCE = P , m Z DAE = 0 Calcule tan 0 en términos de alguna razón tri- gonométrica de p. A)sen3p B)cos2p C) tan3p D)cot2p E)sec4p 18.- Si ABCD es un cuadrado, m Z DFA = a, y además E es punto medio de BC; calcular el valor de: sec a. A) 2/3 B) 710 C) D)2/2 E)/5 C E B D A 19.- En la figura mostrada ABCD es un cua- drado. Determinar el valor de: 16.- En la figura mostrada, m Z ABC = 90°, m Z DCB = m Z CAB = a, AD = 2 BC. Calcu- le: tan a. R = tan jc - 2 tan (x - y) A) 0,5 B)1 C)l,5 D)2 E)2,5 Problemas de Trigonometría y cómo resolverlos
TEOREMA 3 20.- En la figura, halle AB en términos de R y 0. A) R tan 0 (esc 0+1) B) R cot 0 (esc 0+1) C) R tan 0 (sec 0+1) D) R cot 0 (sec 0+1) E) R tan 0 (esc 0 - 1) 21.- En la siguiente figura: AB = a, 2 AB = IX?. Calcular el área del triángulo EFG. A) yg tan a B) cot a C) -q¡t tan a D) yz- (tan a+cot a) E) (tan a - cot a) 22.- En la figura mostrada, determinar1 jc”, si: NC = a, ni Z ABM = a ymZ. MCN = p. «secP ccscP cot P-tan a D>cota-tanP (l CSC P B> tan a-cot P E) «<cot a + tan P> «secP tan a-cot P 23.- Una circunferencia con centro en O está inscrita en un triángulo ABC. Si la distancia de O al vértice A del triángulo es la media pro- porcional entre las distancias del mismo pun- to O a los otros dos vértices, entonces la rela- ción que se establece entre los ángulos del triángulo es: 2 A B C A) sen = sen - sen 2 A B C B) sen = cos-^ - sen 2 A B C C) sen ~2 = eos . eos ™ 2 B A C D) sen = sen . sen „ 2 C A B E) sen = sen - sen 24.- De la figura, calcular “a”, si: AC = CD. A) a tan 0 B) 2a tan 0 C) a cot 0 D) 2a cot 0 E) 2a sec 0 25 .- En la figura mostrada BDEF es un cuadra- do. Si además: m Z DBA = CL,mZ BCA = P; calcule: cot P- 26 .- En la figura mostrada determinar “x” en términos de “r” y “0” Resolución de Triángulos Rectángulos
csc 6 + 1 _rcot0_ l> CSC0-1 E) r tan 6 2 A) k csc a B) y eos a C) ksec2 a k D) y sen-a E) k tan2 a 27 .- En la figura mostrada se cumple que: OB - AB = OC = CD. Calcular: “cot 6” A) V3/3 8)2^3-1 C) J3 -1 D) V3 +1 E) J3 30 .- A partir de la figura mostrada, se pide de- terminar M, si: 31 .- De la figura, calcular “0”, si se sabe que: S = área de una región triangular. 28 .- En la figura se muestra un arco de circun- ferencia. donde: AM = BN. Determine el valor de: M = 2 eos 0 + cot 6. A)1 B)2 03 D)4 E)5 AREA DE UNA REGIÓN TRIANGULAR 29 .- De la figura mostrada se sabe que: m Z BC A = m Z ADC = 90°; m Z ABC = a Si además el área de la región triangular ADC es k, calcule el área de la región triangular ABC. A) 45° B)37° C)30° D)60° E)53° 32.- En la figura mostrada, evaluar el área de la región triangular AOB en términos de 6 A) 4 sen 0 B) 8 sen 26 C) 2 eos 26 D) 5 sen 6 E) 3 eos 26 33.- Si ABCD es un cuadrado donde: CD=3ED y además: m Z BEA = 0 ; calcular csc 0. Problemas de Trigonometría y cómo resolverlos RACSO Pbditokii
36.- En la figura mostrada. ABCD es un rectán- gulo. Si: AD = 4CD, CE = CD, m Z BFA = a; calcule: W = V3 + 7tana A) 3 B)2 C)1 D) 1/2 E)l/3 34. - En la figura mostrada, m Z. ABC = 90°, m Z BCA = m Z DAB = a. Asimismo se sabe que el área de las regiones triangulares ABD y ADC son equivalentes. Calcular el valor de: 37.- Determinar el área de la región triangular de la figura: A) 5 B)4 03 D)2 E)1 35.- En la figura mostrada se sabe que: _____fe2_____ A) 2(cota+cotP) fe2 3(cota-cotP) ______fe2_____ E) 2(csca-cscP) fe2 ) 3(cota+cotP) ______fe2_____ D) 6(csca-cotP) m Z ABD = m Z AED = m Z BCE = 90° ; roZBDC = 6 ; AB = fe ; BD = « Calcule el área de la región sombreada. A) y «fe eos" 6 B) ~ ah sen O.cos 6 C) 2 t b sen2 6. eos 6 D) ~ ah sen S.cos2 6 E) -y fe sen2 6 38.- En la figura ABCD es un cuadrado, M y N son puntos medios. Determinar «cot 6». A) 2 B)1 C)3 D) 1/2 E)l/3 Resohición de Triángulos Rectángulos
i _ -í - 4 +r - a -V 1 JazonesN ngon^metricas 1 eTT tíTa c i dtíesrcXi téx t u al izad a s -AWfcr i i/‘ / , i CAP ®1 5.1. ÁNGULOS VERTICALES__________________________________ Se denominan así a aquellos ángulos agudos, uno de cuyos lados se ubica sobre la línea horizontal mientras que el otro se localiza en el mismo plano vertical, por encima o por debajo de aquella, llamándose: ángulo de elevación y ángulo de depresión, respectivamente. a: es la medida del ángulo de elevación 5.2. ÁNGULO DE OBSERVACIÓN____________________________ Es el ángulo formado por dos líneas visuales que definen un campo de observación respecto de un observador. Este ángulo puede ubicarse en cualquier plano: en un plano verti- cal (P.V.), en un plano horizontal (P.H.), y en cualquier otra orientación. Ó: ángulo de observación en el P.H. 6: ángulo de observación en el P.V. Nota: El ángulo de observación está comprendido entre 0o y 180° T5 Problemas de Trigonometría y cómo resolverlos '^4 RACSO
5.3. ÁNGULOS HORIZONTALES Son aquellos ángulos que se encuentran en un mismo plano horizontal. Estos ángulos están constituidos por las denominadas direcciones cardinales: este (E), oeste (O), norte (N) y sur (S). |N TS ___E Punto de referencia 5.3A Rumbo o Dirección Es la dirección considerada o trazada en el plano horizontal, y principalmente cualquiera de las comprendidas en la rosa náutica. Para definir un rumbo o dirección de movimiento se toma como referencia cualquiera de los puntos cardinales. 5.3B Direcciones NE, NO, SE y SO (Ñor Este, Ñor Oeste, Sur este y Sur Oeste respectivamente) 5.3C Rosa Náutica o Rosa de los Vientos También conocida como rosa de la aguja, fue antes de la generalización de las brújulas magnéticas, una excelente referencia en las cartas marinas en la que se mostraba la dirección de los ocho vientos principales. Las más antiguas rosas de los vientos de las que se tiene noticias son las que aparecen en Jas cartas de navegación del siglo X11J manejadas por los navegantes españoles e italianos. En ellas, los ocho puntos cardinales aparecían marcados con las iniciales de los principales vientos, si bien en ocasiones —como puede observarse en la rosa que aparece en la imagen— el punto cardinal Este aparecía señalado con una cruz, en tanto que el Norte Jo hacía con una flor de lis. A partir de la expansión del uso de la brújula, la rosa de Jos vientos pasó a convertirse en una herramienta auxiliar de aquélla. [Enciclopedia Encarta, Madrid, 2004]. Razones Trigonométricas en Situaciones Contextualizadas
PROB. 1 Desde un punto en tierra se observa la cumbre de una montaña con un ángulo de elevación a Si la distancia del punto de observación a la falda de la montaña es «d», exprese la altura de la montana respecto al nivel del suelo en términos de a, P y d, siendo P la pendiente de la falda de la montaña. RESOLUCIÓN Se observa que: x cot a = d + x cot P =s x cot a-xcot P = d =s x(cot a - cot P) = d - x=________<1____ cota-cotp PROB. 2 Desde un avión que vuela horizontalmente y en línea recta, a una altura «h» km, es ubicado en tierra un punto bajo un ángulo de depresión 0. Luego de «/» horas, este punto es visto nuevamente con un ángulo de depresión igual al complemento del anterior. Si el avión no ha pasado por encima del punto mencionado, exprese la velocidad del avión en km/h en términos de h, t y 0. Se sabe que la velocidad del avión es constante. RESOLUCIÓN Elaboramos el gráfico correspondiente: Se sabe que: distancia = velocidad x tiempo x = v.l ---(1) Pero: x = h cot 0 - h tan 0 =s x = h (cot 0 - tan 0) ... (2) Reemplazando (1) en (2): => ñ(cot 0 - tan 0) = v. t v = y (cot 6 - tan 6) Problemas de Trigonometría y cómo resolverlos RACSO IDITOKll
PROB. 3 Un reflector situado al ras del suelo ilumina un monumento bajo un ángulo de 30°. Se traslada el reflector a 2 m más cerca del monumento y éste se ve bajo un ángu o de 45°. ¿Cuál es la distancia del monumento al segundo lugar de iluminación? Racionalizando, obtendremos: ‘¿43 3+45 = 2(3^5+3) X 3-45'3 + 45 =* x 6 x = V3 + 1 PROB. 4 La elevación de una torre desde un punto A al Oeste de ella es 60° y desde un punto B al sur de A, la elevación es de 30°. Si la torre tiene 75 m de altura, calcular la distancia comprendida entre A y B. Trasladando los datos a un gráfico como el mostrado, reconocemos que: AP = 75cot60° = aBP = 75cot30° =7545 45 Aplicando el teorema de Pitágoras en el triángulo rectángulo sombreado obtendremos: AB2 + AP2 = BP2 => x2 + = 752(3) x* = 7S*(3-±) jr2 = 752(|) x = 50j6 ESTRATEGIAS DE RESOLUCIÓN 1) Lea e interprete adecuadamente el enunciado del problema. 2) Elabore un gráfico indicando en él los datos reconocidos del enunciado 3) Centralice su trabajo en triángulos rectángulos. 4) Resuelva dichos triángulos rectángulos Razones Trigonométricas en Situaciones Contextualizadas
Enunciados de Problemas „ con Resolución ÁNGULOS VERTICALES 01.- Desde la parte superior e inferior del se- gundo piso de un edificio de cuatro pisos igua- les, se observa una piedra en el suelo y a una distancia de 9 m con ángulos de depresión «P» y «6» respectivamente. Desde la parte más alta del edificio la depresión angular para la piedra es «a». Si se conoce que: tan a - tan P - tan 6=1/4 La altura del edifico es: C)-^= 1-J3 A)6/n B)10zn Q9m D)8/n E)4m 02.- Un observador aprecia dos puntos que están en una misma vertical bajo ángulos de elevación y de depresión de 30° y 15o respec- tivamente. Si la distancia del observador al no . - ' u 4J3 cambiar mas alto es ni. ¿Cuál es la distancia del observador al otro punto? A)2(Jó + V2 ) D) Jó - J2 04.- En la figura mostrada. ¿A qué distancia se encuentra el globo respecto del lago? A) Heos 2a aobo B)Hsen2a H" \ ¡ C)Hese2a * F *-----------,'|La-° < D) H sec 2a \ í E) H cot 2a £ Imagen 05.- Desde un punto a nivel del suelo un ob- servador divisa una estatua con su pedestal de 5 m y 4 m respectivamente. El ángulo de elevación de la cabeza de la estatua es el do- ble del ángulo a la parte superior del pedestal o pie de la estatua. ¿Cuál es el valor de la tan- gente del mayor ángulo de elevación? B)2(Jó - J5) C) Jó + -J2 E)2(j2 +Jó) A). 1/2 B)3/4 C)2/3 D) 1/5 E)5/6 03.- Los ángulos de elevación de la cúspide de una torre, vistos desde 2 puntos situados en línea recta con el pie de la torre son de 45° y 30° respectivamente, si la distancia entre los puntos de observación es de 60 m, la altura de la torre (en m). es: 06.- El diámetro aparente (ángulo de observa- ción) del sol es aproximadamente 32’. ¿A qué distancia del ojo debe colocarse una moneda de 30 mm de diámetro para poder tapar exacta- mente al sol?. Considere que tan 16’ = 0,00465. A) 3,844 m B) 3,223 m C) 3,448 m D) 4,483 m E) faltan datos T5 Problemas de Trigonometría y cómo resolverlos
07.- Un lugar de la provincia de Santiago tiene una latitud 30”. ¿A qué distancia respecto del eje de la tierra se encuentra, si el radio terres- tre mide 6 370 km? A)3 185A:/n B)951,lfan C)961,lA/n D) 917 km E)5516fan 08.- Desde un globo que está en la vertical que cae sobre un camino recto, los ángulos de depresión de dos piedras consecutivas, indicadoras de los kilómetros, miden 45“ y 60°. Calcular la altura del globo. A) 2 300 m B) 2 320 m C) 2 340 m D) 2 366 ni E) 2 360 ni 09.- Una mosca que está en el suelo observa un pajarillo con un ángulo de elevación de 45°. Para llegar donde está el pajarillo, la mos- ca describe una trayectoria curva de un cuar- to de circunferencia de modo que en un punto de su recorrido observa al pajarillo con un ángulo de elevación de 37°. ¿A qué altura (en ni) se encuentra la mosca en dicho punto?. El pajarillo está a una altura de 2,5 ni respecto del piso. A) 0,5 B)O,55 C)0,6 D)0,4 E)0,7 10 .- Desde un avión se observa un punto en tierra con un ángulo de depresión “a”. Cuan- do el avión se desplaza horizontalmente una distancia igual al triple de la altura a la que se encuentra, el nuevo ángulo de depresión para el mismo punto es “ 90° - a”. Calcular: M = tan"a + cot"a A) II B) 12 C)13 D) 14 E) 15 11 .- Desde un punto en tierra se observa la altura de una torre con un ángulo de eleva- ción de 37”. Si la visual en dicho lugar mide 20 ni, ¿qué distancia horizontal (en m) deberá acercarse un observador, hacia la torre, para que el nuevo ángulo de elevación tenga por tangente 2? A) 13 B)ll C)10 D)9 E) 5 12 .- Un alumno de -J5 m de altura, observa la parte superior de una torre de alta tensión con un ángulo de elevación de 60°. ¿Cuánto deberá retroceder el alumno para observar la m sma to- rre pero con un ángulo de elevación de 30", si la altura de la torre es de 6 m?. A) 8ni B) lOm C) 12ni D)14m E)16m 13 .- Un pescador situado a 600 ni sobre el ni- vel del mar observa una lancha con un ángulo de depresión “a”. Seis miniaos después ob- serva en la misma dirección a la lancha pero ahora con un ángulo de depresión “P”. Calcu- lar la rapidez de la lancha en km/h, si: tan a = -Jí + 1 y tan P = V3 - I A) 5 B)6 C)7 D)8 E)9 14 .- El asta de una bandera está colocado ver- ticalmentc en lo alto de una vivienda. A 36 m de distancia de la parte baja de la vivienda se observa la punta del asta y la parte superior de la vivienda con ángulos de elevación de 53" y 37" respectivamente. Calcule (aproxima- damente) la longitud del asta (en ni). A) 13 . 13)18 C)19 D)2I E)23 15 .- Un avión que vuela en línea recta y hori- zontalmente antes de pasar sobre 2 puntos en tierra “A” y “B” los observa con ángulos de depresión “a” y “P” respectivamente. Cuan- do está sobre “A” es visto desde “B” con un ángulo de elevación “6”. Si: cot a = 1/3 y cot P = 1/2, determinar “tan 6”. A) 2 B)4 C)6 D)8 E)I0 16 .- Una antena de radio de 15 ni de longitud se encuentra en la azotea de un edificio. Desde un punto del plano horizontal que pasa por la base del edificio las elevaciones angula- res de la parte superior e inferior de la antena son a y P respectivamente. Si: tan a = 0,76 y tan P = 0,19, determinar (en m) la altura del edificio. A)4 B)5 C)6 D)7 E)8 Razones Trigonométricas en Situaciones Contextualizadas T5 55
YL- Un poste de longitud “x”, está inclinado 60° respecto a la vertical. El foco del poste es observado por 2 personas que se encuentran ubicadas a los dos lados de éste y lo obser- van con ángulos de elevación “6” y “ 90° - 6”. Si la separación entre dichas personas es de 16 m, calcular “x” en términos de “6”. Sugerencia: 2 sen 6 eos 6 = sen 26. A) sen 26 B) 2 sen 26 C) 4 sen 26 D) 8 sen 26 E) 16 sen 26 18 .- Una persona sube una cuesta y cuando llega a la cúspide, se da cuenta que la altura a la cual se encuentra es la mitad del camino recorrido. Calculare! ángulo de elevación con el cual se observa a la cúspide, desde la base de la cuesta. A) 15° B)30° C)45° D)60° E)75° 19 .- Una persona observa un objeto que está en caída libre vertical, con un ángulo de eleva- ción de 66°. Luego de un momento lo vuelve a observar con un ángulo de elevación de 36°. Si en la primera observación, el objeto, se en- contraba a 60 m de altura, ¿a qué altura (en m) se encontraba en la segunda observación? A) 10 B)20 C)30 D)40 E)50 ÁNGULOS HORIZONTALES 20 .- Una persona en A se encuentra al este de la otra persona en B, si la persona en B se desplaza en la dirección N NE y la persona en A en la dirección NO, se encuentran en el punto P. Calcular ¿cuánto mide el ángulo APB?. A) 45° B)ll°15’ C)47°15’ D)56°15’ E)15°15’ 21 .- Si desde un carrousel observas a un ami- go en la dirección oeste en la situación más cercana y al cabo de 0,15 segu dos los obser- vas en dirección NO. Calcular el número de vueltas que da el carrousel po cada minuto A)30RPM B)40RPM Q50RPM D)60RPM E)70RPM 22 .- Un niño sale de su casa y hace el siguien- te recorrido: 20 m al norte, 40 m al este y una cierta distancia al SO, hasta ubicarse al este de su casa. ¿A qué distancia (en m) de ella se encuentra? A) 60 B)50 C)40 D)30 E)20 23 .- Un auto recorre 40 km en la dirección N53°O, luego recorre 40km en la direc- ción SO y finalmente recorre 60 km en la direc- ción este. Determine en qué dirección y a qué distancia (en km) se encuentra el auto respecto a su posición de partida (aproximadamente). A)S37°O;20 B)S53°O;20 C)S45°O;10 D)S36°O;20 L)S45l’O,20>/2 24 .- Desde un faro se observa dos barcos con direcciones ESE y NNE. Si la distancia que hay entre los dos barcos es aproximadamente 10 (J2 + V2 ) km, y desde el segundo barco se observa al primero en la dirección SE, calcular (aproximadamente en km) la diferencia de las distancias del faro a los 2 barcos. A) 10 B)15 Q17 D)20 E)25 25 .- Desde una estación de control A se ob- serva a otra estación B en la dirección N6E a una distancia L. Desde la estación B se obser- va una tercera estación C en la dirección N26E a una distancia también igual a L..¿En que di- rección se encuentra la estación A respecto de la estación C? T5 56 Problemas de Trigonometría y cómo resolverlos JRACSO DbDITOIBI
A)SjO B)s4^O C)S®O D)O®N E)O^N 26 .- Dos embarcaciones parten de un puerto con movimientos rectilíneos, el primero con di- rección NPE y el segundo con rumbo S2PE. Cuando el primero recorre 4 km. el segundo recorre 4,2 km. La distancia que los separa es de 5,8 km. Encontrar el ángulo P en radianes. 71 71 71 71 71 A>12 B)- Qj D)- E)^ 27 .- Una lancha sale de un puerto con movi- miento rectilíneo en la dirección SE. Luego de un t empo “r” de recorrido se desvía y conti- nua rectilíneamente por la dirección N15°O, hasta equidistar del puerto y del punto de des- vío. ¿En qué dirección se encuentra la lancha respecto del puerto?. A) SSE B) S75°E C) E4mS D)N60°E E)ESE 28 .- Dos personas se ubican una frente a otra en la línea Este - Oeste. Si éstas se desplazan en rectilíneamente en las direcciones N70°E y O10°N respectivamente hasta encontrarse, de- termine el menor ángulo formado por las di- recciones de sus movimientos. A) 45° B)30“ C)70° D)80“ E)150° 29 .- Jorge y Giovanna, cogidos de las manos, se encontraban conversando. Al despedirse Jorge se dirige en la dirección Oeste caminan- do rectilíneamente 200 m, pero Giovanna lo hace rectilíneamente en la dirección N 63° 30’E una distancia de 100 -Js »i. Ella olvidó dar un mensaje a Jorge, por lo cual decide darle el encuentro caminando rectilíneamente en la di- rección S(90° - 6)0 hasta encontrarlo. Se pide determinar cot 6, aproximadamente. A) 2 B) 1/3 C)4 D) 1/4 E) 1/2 30 .- ¿Cuál es la dirección opuesta a la direc- 1 ciónNE “E? 4 1 1 A)N~NE B)SE-S 1 1 D)NE-N E)N-NO C)SO^O 31 .- Al calcular el mayor ángulo formado por las direcciones: 1 1 SE~SyN~NE se obtiene: 4 4 A) 135° B)225° C)45° D)60° 11)120" 32 .- Para las siguientes proposiciones deter- mine la verdad (V) o falsedad (F): a) El mayor ángulo formado por las direccio- nes SO y SSE es 305° b) El menor ángulo formado por las direccio- nes ENE y ONO es 135°. c) El menor ángulo formado por las direccio- nes ESE y NNO es 90° A)FVF B)WF C)FFF D)FFV E)FVV SITUACIONES TRIDIMENSIONALES 33 .- Desde lo alto de un acantilado se observa en la dirección 0(90°- cz)S a una boya bajo un ángulo de depresión de 45° y en la dirección EaS a un bote con un ángulo de depresión de 30°. Si la distancia que separa a la boya y el bote es 80 m, determinar (en m) la altura del acantilado. A) 20 B)13 C)40 D) 14 E)25 Razones Trigonométricas en Situaciones Contextualizadas
34.- Un observador se encuentra ubicado al sur de una torre de alta tensión y visualiza el extremo superior de aquella con un ángulo de elevación a. Asimismo otro observador se en- cuentra en la dirección OcxN respecto a la an- terior y al Oeste de la torre visualizando el ex- tremo superior de ésta con un ángulo de ele- vación p. Calcule cot P en términos de una razón trigonométrica de a. ra que la proyección de su sombra tiene una longitud de 7,20 m. Si camina rectilíneamente hacia el Oeste, de tal manera que su distancia hacia la parte inferior del poste es de “v" ni y la proyección de su sombra tiene como medida 9 ni; se pide determinar: a/v. A) 1/2 B)4/5 C)2/3 D) 1/3 E)l/5. A)sen2 a B) tan2 a C)2seca D) cot" a E) 2 csc a 35.- Un avión desciende rectilíneamente con una inclinación a en la dirección Este - Oeste. Si un observador ve al avión primero hacia el NE y luego hacia el N con ángulos de eleva- ción iguales al complemento de a. entonces al calcular sec a se obtiene: 38.- Desde 2 puntos A y B ubicados al sur u este de un edificio, se observa la parte supe- rior del mismo con ángulos de elevación de 45° y 53° respectivamente. Hallar la distancia entre el punto B y el edificio, si la distancia entre el punto A y la parte superior del edificio es 60 Jí ni. A) 45 ni B)40ni D) 50 m I) 43 ni C) 43 ni A) í/3 D) B)2 J2 E) í/2 C)2^3 36.- Dos edificios ÁB y CD tienen la misma altura Una persona de ubica entre dichos edi- ficios de tal manera que su posición está en la línea recta AC que une sus bases. Dicha per- sona < bserva el extremo B con un ángulo de elevación de 60°, si luego de caminar rectilínea- mente 3-Jl m en una dirección perpendicular a la recta AC observa los extremos B y D con ángulos de elevación de 45° y 30° respectiva- mente, entonces la distancia de su posición ini- cial al extremo C es: 39.- Un niño de 1 ni de estatura observa un foco de luz que se encuentra sobre un poste en la dirección N18UE. con un ángulo de ele- vación cuya tangente es 2/3; luego se des- plaza 20 ni en la dirección N71°E y vuelve a observar al poste ahora en la dirección N72nO. Hallar la longitud de la sombra del niño en su posición final. A) 5 ni B) 4 ni C) 3 m D) 2 ni E) 1 ni A) 3/7 B)5j2 C)7j3 D)2?/5 E)2/7 37.- Una persona de 1,80 ni de altura se en- cuentra al sur de un poste luminoso de altura “H” m y a una distancia de “a” ni. de tal mane- 40.- Una avión cae con un ángulo de inclina- ción “a” debajo de la horizontal en la direc- ción EO. Calcular el valor de la tan a para que un observador vea el avión primero hacia el NE y luego hacia el norte con el mismo ángulo de elevación “6” A)(1 - /Zj.tanO B)(-j2 + Ij.tanO 0(^2 D.tanO D)(3-j2 - D.tanO F)(2-j2 - D-tanO Problemas de Trigonometría y cómo resolverlos A --ACSO HTOlll
6.1. SISTEMA DE COORDENADAS RECTANGULARES Es aquel sistema formado por dos rectas numéricas una horizontal llamada eje de abscisas (x) y la otra vertical llama- da eje de ordenadas (y), que se cortan perpendicularmente, en un punto llamado origen de coordenadas, formando un plano en el que cualquier punto está plenamente ubicado por un conjunto llamado par ordenado. Par Ordenado.- Es el conjunto formado por dos números reales de la forma (x; y), caracterizados por el orden de sus elementos: x es el primer elemento (abscisa) e y es el segundo elemento (ordenada) . En el plano se identifican cuatro cuadrantes denotados y ubicados según como se muestra en la figura. Axioma. A cada punto del plano cartesiano le corresponde uno y solo un par ordenado. 6.1 A Ubicación de un punto en el sistema de coordenadas Un punto se ubica en el plano cartesiano cuando se conoce su respectivo par ordenado. 6.IB Distancia entre dos puntos en el plano cartesiano Es la longitud del segmento de recta que existe entre dos puntos de un plano cartesiano, y está dada por: d= + (y2 -y¡? d > 0 6.1C Radio Vector (r) Es la distancia que hay desde el origen de coordenadas hasta un punto (x; y) cualquiera del plano. r= J(x2+y2 o bien r2=x2+y2 ;r>0 La denominación de radio vector, se debe a que este elemento matemático tiene origen (0; 0), dirección (del origen al punto) y módulo (la distancia OP = r). Razones Trigonométricas de Angulos en el Plano Cartesiano T6 59
6.ID División de un segmento en una razón dada (r). am Sea: vtít = r, donde «r» es el valor de la razón MB (comparación) de las longitudes de dos segmentos ubicados en una misma recta. Si A y B son conocidos, las coordenadas del punto M vienen dadas por: X] +rx2 I + r _ yi+o’z ~ l+r 6.1E Coordenadas del baricentro de un triángulo Conocidas las coordenadas (X] ¡y^, (x2 ;y2), (*3 ¡>3) de los vértices de un triángulo, se puede determinar las coordenadas del baricentro de dicho triángulo, a partir de: 3 y~ 3 6.1F Area de una región triangular Conocidas las coordenadas de los vértices de un triángulo, el área de la región triangular limitada por ella se determina a partir del siguiente determinante: S = l^Kxpyj + x2.y3 + x^) - (xj.y3 + x3.y2 + x2.y,)] Nota.- Queda claro que el valor asociado a esta matriz (el determinante), es el área de la región triangular. Esta técnica nos permite determinar el área de una región poligonal 6.2. ÁNGULOS EN UN SISTEMA DE COORDENADAS RECTANGULARES QJ 6.2 A Ángulo en posición normal Es aquel cuyos elementos están plenamente determinados en un plano cartesiano, de modo que: i) Su vértice es el origen del sistema de coordenadas. ii) Su lado inicial es el semi eje positivo de las abscisas. iii) Su lado terminal nos indicará el cuadrante al cual pertenece el ángulo Problemas de Trigonometría y cómo resolverlos *¡S1RACSO WiDiroiia
a e IC ; a < O 6.2B Ángulo Cuadranta Es aquel ángulo en posición normal cuyo lado final se encuentra sobre un semieje. Por convención se ha establecido que los ángulos cuadrantales no pertenecen a ningún cuadrante. Por ejemplo: En general: Z cuadranta] = 90° n ; V n e Z kit o bien Z cuadrantal = -y ; V k e Z 6.2C Ángulos Coterminales Son aquellos ángulos en posición normal que tienen el mismo lado inicial y el mismo lado final sin considerar sus correspondientes sentidos de rotación ni su medida. Razones Trigonométricas de Angulos en el Plano Cartesiano T6
6.3. DEFINICIONES DE LAS R.T. DE ÁNGULOS EN POSICIÓN NORMAL Sea a un ángulo en posición normal tal que su lado final pasa por el punto Pfx ; y). Se definen las razones trigonométricas de a, de la siguiente manera: sena = ordenada y cota = abscisa _ X ~ y radio vector ordenada eos a = abscisa __ radio vector X sec a = radio vector abscisa . _L "" X tana = ordenada _ y esc a = radio vector _ r abscisa X ordenada y 6.4. PROPIEDADES________________________________________ todas , lasR.T.VJ (+) CSC _______□ tan , . cot <+> eos , sec <+> 6.4A Signos de las Razones Trigonométricas IC: Todas las Razones Trigonométricas son (+) IIC: Seno y Cosecante son (+), las demás son (-) 111C: Tangente y Cotangente son (+), las demás son (-) IVC: Coseno y Secante son (+), las demás son (-) 6.4B Valores de las R.T. de los ángulos uadrantales A partir de las definiciones anteriores se pueden deducir los valores del siguiente cuadro: ÁNGULO CUADRANTAL R.T. 0o 90° 180° 270° 360° sen 0 1 0 -1 0 eos 1 0 -1 0 1 tan 0 ND 0 ND 0 cot ND 0 ND 0 ND sec 1 ND -1 ND 1 CSC ND 1 ND -1 ND ND = valor no definido 6.4C Propiedades de los Ángulos Coterminales Sean ay P dos ángulos coterminales, entonces se cumple que: i) a - P = n x 360°; V n e Z La diferencia de dos ángulos coterminales es un número entero de vueltas de 360° . ii) R.T.(a) = R.T.(P) Si dos ángulos son coterminales sus razones trigonométricas serán iguales. T6 62 Problemas de Trigonometría y cómo resolverlos 31RACSO
PROB. 1 Dado los puntosA(-2;7)yB(10; 1), siP(x;y)es el punto medio del segmento AB, por donde pasa el lado final de un ángulo en posición normal 6, se pide: a) Ubicar el punto P(x ; y) en el sistema de coordenadas rectangulares. b) Calcule los valores de las 6 R.T. del ángulo 6. RESOLUCIÓN Por la fórmula del punto medio: -2 + 10 . -7 + 1 = =4 y = — = -3 b) A continuación calculamos la longitud del / 2 2 radio vector. r= ^x + y => r = J(4)2+(-3)z r = 5 Del gráfico deducimos: „ y -3 n x 4 r 5 r 5 tan 0 = — = cot0 = — = x 4 y -3 3 sec 0 = — = 4 csc 0 = - = -Ar = x 4 y -3 3 PROB. 2 Obtener el sen 0, a partir de la siguiente figura: RESOLUCIÓN Dado que el ángulo 0 no se encuentra en posición [vértice del ángulo (5 ; 0)], trasladaremos su vértice u(5 ; 0) al origen de coordenadas. La traslación del segmento VP se realiza en forma paralela así: Razones Trigonométricas de Angulos en el Plano Cartesiano
PROB. 4 De la figura que se muestra, calcule cot 6. RESOLUCIÓN ****************** Dado que el ángulo 6 no está en posición normal, lo hacemos rotar 90° en sentido horario, hasta que su lado inicial coincida con el eje de abscisas. En estas condiciones se obtiene el gráfico inferior: Como el ángulo 6 no se encuentra en posición normal, tenemos que expresarlo en posición normal, haciendo rotar 180° en sentido horario el radio vector OP. Como el ángulo ya está en posición normal, entonces: cot 0 = ESTRATEGIAS DE RESOLUCIÓN: 1) Para determinar el cálculo de cualquier razón trigonométrica en un sistema de coordenadas cartesianas debemos conocer las coordenadas de un punto del lado final del ángulo en posición normal, (de este modo el lado final queda definida por su origen y dicho punto) 2) Es necesario tomar una escala adecuada para ubicar los puntos en el sistema cartesiano. 3) Para el cálculo de las razones trigonométricas necesariamente el ángulo tiene que estar en posición normal. 4) Cuando el ángulo no se encuentra en posición normal, hay tres posibilidades: . Rotar 90°. . Rotar 180°. . Trasladar el vértice del ángulo al origen del sistema de coordenadas cartesianas. T6 64 I Problemas de Trigonometría y cómo resolverlos -JÉ RACSO jP BDITOMia
Enunciados de Problemas u R.T. DE UN ÁNGULO EN P. NORMAL 01.- Si el lado final de un ángulo en posición normal “0” pasa por el punto M(6; -1), calcu- lar el valor de: E = -JyÍ esc 0 + cot 0 06.- Si “a” es un ángulo en posición normal del cuarto cuadrante, el cual verifica: -----Vena 7“^ picosa j — (sec a) ; calcular: M = 7 eos a + 3 cot a A)40 B)-15 C)22 D)-35 E)-43 A)-3 B)2 C)-l D)-2 E)0 02.- Si el lado final de un ángulo en posición normal cuya medida es a pasa por el punto (-2; 3), calcular: JÍ3 seca W = -----------p= csca -Jl 3 11 9 7 5 A)y B)f C) ' D)j E)2 i n 03.- Si: 9 tan“a - 16 = 0 ; y: 7~ < a < 47t; entonces al calcular W = esc a + cot a , se obtiene: A) 1 B)0,75 C)0,50 D)0,45 E)-2 3n 04.- Si: — <x <271 ; y: 3 -0,25 = sen x +, + sen x + ... ; calcular: W = (sec x + cot x). A) 3,5 B)4,5 C)5,5 D)6.5 E)7.5 05.-Si:sen0 = -^ - — y cos0<O; "n "términos calcular el valor de: M = (tan © ' sec ®) A)-l B)1 C)2 D)l/2 E) SITUACIONES GRÁFICAS 07.- Las ecuaciones de las rectas mostradas en la figura son: L¡:x+3y=-7 a L2: 5x+2y=4. 2 Determinar el valor de: W = tan a + sec a A) 1,75 B)l,50 0)2,25 D)0,75 5) 1,25 08.- En la figura mostrada OA = OB, BC = CD. Determinar: M = 5 tan 0-6 cot 0, si el punto D es (-5;-4). A) 3 B)4 C)1 D)5 E)2 Razones Trigonométricas de Ángulos en el Plano Cartesiano
09.- En la figura mostrada, si AB = BC = CD, encontrar el valor de: W = tan 0 - tan <[). A) 1,0 B)2,5 C)3,0 D)4,0 E)l,5 10 .- En la figura mostrada se tiene la gráfica de: y = -2| jc|; donde a y 0 son ángulos en posición normal. Calculad valor de: A)2 B)-3 Q-2 D)3 E)4 11 .- En la figura mostrada “O” es el centro de la circunferencia y además: OA = AB = BC. Determinan M = cot 0 + VÍO tan <}> I) tan a < 0 12 .- De la figura mostrada, calcular “cot 0”; si: DP=PC. A) 1/3 B)2 Q-2/3 D) 1/4 E)3/2 PROBLEMAS CONDICIONALES 13 .- El lado final de un ángulo a en posición normal pasa por el baricentro del triángulo PQR cuyos vértices son: P(-l; -5), Q (4; 3), R(6; -10). Calcule: W = 5(sen a - eos tí) A)-ll B)-9 C)-7 D)3 E)5 14 .- En la figura mostrada se sabe que: BC=CD, 37° m Z OAB = y las coordenadas del punto B son (-3; - 6). Si además: m Z AOD = a, sien- do a un ángulo en posición normal, calcular: W = tan a +cot tí A) 17/23 B)-85/42 C) 15/29 D) 12/31 E)-23/27 15 .- Dadas las siguientes condiciones: II) cosa>0 III) 22csca-(0,25) *'25 = 0 Calculan W = 5 eos a - 4 cot tí Problemas de Trigonometría y cómo resolverlos ’liRACSO
A) 1 B)2 C)4 D)6 E)8 16 .- A partir de las siguientes condiciones: I) tan 0 > 0 II) sen0<O III) |cos 0|> 2/3 Calcular “sen 6” cuando eos 0 sea máximo. A)^ B)^ C)1 D)—^- E)-l 17 .- De acuerdo con las siguientes condiciones: I) |sen 0| = sen 0 II) |cos 0| = - eos 0 III) |tan0]=l,3 Determinar el valor de M = csc 0 + cot 0 A) 1/2 B)-l C)-2 D)-l/2 E)l/3 18 .- Sabiendo que: I) |cos 0| = - eos 0 II) |cot0| = cot0 III) |sec0| = 2 Calcular: W = sen 0. tan 0. A)-2,0 B)-l,5 C)-l,0 D)2,0 E)2,5 19 .- Si se verifica que: I) |csca| = -csca II) |seca| = seca III) |csc a - cot a| = k Calcular, sec a. (a es positivo y menor que una vuelta) 1- tanl 2f - o l 2 Sugerencias: eos a =--------7—t l + tan2í — I 2 a tan — = csc a - cot a D) l-*2 1 + it2 1-Jt2 B)-p- k2 A) M E>^ SIGNOS DE LAS R.T. 20.- Si <J> y 0 representan la medida de dos ángulos en posición normal positivos y me- nores que una vuelta, cuyos lados finales se ubican en diferentes cuadrantes, tal que: I) 7t/2<0<<J> II) sen 0- ^cos <}>—sen 0 >0 Determinar el signo de W, si: W = tan 0 + cot <J>. A) Es siempre positivo. B) Es siempre negativo. C) No es posible determinar el signo. D) Falta mayor información. E) W es nulo. 21.- Si a e ( 0; 7t); 0 e ( 7t; 27t), determine el f27t+a) 0 signo de: M = tanl —-— 1+ csc — A)() B)0 C)± D)(+) E)N.A 22.- A partir de las siguientes condiciones: I) 0e(O°;36O°> U) |sen 0| = - sen 0 O) csc 0. eos 0 > 0 Determine el signo de M, si: Ramones Trigonométricas de Ángulos en el Plano Cartesiano
M = sen j . cot . sec (W) A)(+) B)0 C)+ D)(-) E)N.A 23.- Si “0” es un ángulo positivo y menor a una vuelta tal que: sen 0 < 0 y cot 0 < 0; deter- mine el signo de: A)-l B)0 C)1 D)2 E)-2 28 .- Siendo “a” y “0” ángulos cuadrantales positivos y menores de una vuelta que verifi- can: sen a = tan 0 + 1; calcular el valor de: sen2a+sen3a tan(0/4) a) sen 0/2 b) tan 20/3 c) sec 0/4 A)+;+;+ B)-;+;+ C)+;-; + D) + ; + ;- E)-; + ;- A)2 B)-l Q-l/2 D) 1/2 E)3Z2 29 .- Siendo a , 0 y 0 ángulos cuadrantales; calcular: M = A + B + C, donde: (<x, 0 y 0 G [0 ;2n» 24 .- Si los puntos P y Q son simétricos res- pecto al eje Y exprese: H = |cot 0| + |cot a|, en términos de a y b. K)a+b!b fi)b/u C)aJb D)bf2a E)2«/fc 25 .-SÍ: V-sec0 <0< V-sen0 ¡determinare! signo de: M = tan 0 + cot 0 . eos 0. (0: es no cuadrantal) A) (+) B)0 C)± D)() E)N.A R.T DE ÁNGULOS CUADRANTALES 26 .- Siendo a, 0, 0 ángulos cuadrantales dis- tintos, mayores o iguales que 0o pero menores o iguales que 270° y además cumplen: cos0 = JsenO-Jsena Calculan W = cos(a + 0 + 0) A) 2 B) 1 C)0 D)-l E)-2 27 .- Si la expresión: M = VO-2 + V4-6 es real, calcular: R=sen 0 + tan 0 + eos 0; cuando “0” es un ángulo cuadrantal. A = Jtanct+sen0+cos0 , B = ^cotp-cosa C= ^/cosa + cos0+señ0 A) 1 B)-l C)2 D)-2 E)3 R.T. DE ÁNGULOS COTERMINALES 30 .- En la figura se cumple que: sen a = -15/17. Determinar: M = tan a + tan 0 + tan (a - 0) A) 15/4 B)7/2 C)5/4 D)3/4 E)2 31 .- Determinar el menor de 2 ángulos coter- minales, si la suma de ellos es 1320° y el mayor está comprendido entre 900° y 1200°. A) 240° B)260° C)300° D)320“ E)340° 32 .- Sean a y 0 dos ángulos coterminales tal que: a > 0. Si además el doble del menor es a la suma de ellos como 13 es a 23, calcule la medida del mayor si está comprendida entre 1 100° y 1300°. Problemas de Trigonometría y cómo resolverlos ''A RACSO Pbditokbb
A) 1288° B) 1198° C) 1 188° D) 1298° E) 1260° 33 .- Si la medida de dos ángulos coterminales positivos son proporcionales a los números 2 y 7, y además la diferencia de sus medidas está comprendida entre 1 200° y 1 500°; calcu- lar la medida del menor. A) 634° B)603° C)576° D)428° E)415° 34 .- Si la medida de dos ángulos coterminales negativos son proporcionales a los números 7 y 5; y además la diferencia de sus medidas está comprendida entre 540° y 900°, determi- nar la medida del mayor. A)-1800° B)-1700° C)-1600o D)-1500° E)-1400° A) 83Jó3 D)--7= 3-J63 B) B) 63 El E)’63>/63 38.- Calcular: a2 sen + 2ahcos0 - b2 sen R =------~ (a-fe)2 eos720°+ 4ah A) 0 B) 0,5 C) 0,25 D) 1 E) 1,25 39.-Si: siendo: 3n/ 2 < a < 2rt. El valor de cot a - eos a es: MISCELÁNEA 35.- Si tan 0 = 1,5 , siendo 0 un ángulo del 3er cuadrante, el valor de la expresión: a)-y J7 B)-4^ J7 C)-27 D)'27 7 M = (sec P " csc P) • •S B) c> - 'i E> Jó 36.- Si a es un ángulo ubicado en el primer cuadrante y sen a = 0,25. ¿Cuál es el valor de csc a + cot a ? A) 15 B) C)‘^ D) E)19 37.- Si a. es un ángulo del tercer cuadrante, tal que: -^l + cot2c = 8, calcular (8 sec a)3 40.- Del gráfico mostrado, calcular: "sen2 a + eos2 0" A)0 B) 1 C)3/4 D) 1/2 E)2 Razones Trigonométricas de Angulos en el Plano Cartesiano T6 69
El estudio de las R.T. de números reales se efectúa a partir de las características que posee la circunferencia unitaria, más conocida como circunferencia trigonométrica (C.T.) 7.1. CIRCUNFERENCIA TRIGONOMÉTRICA (C.T.) (*;y) CT. (0:01 Es una circunferencia que posee las siguientes características: i) Su centro es el origen del sistema de coordenadas. ii) Su radio es igual a la unidad* (r = 1) En consecuencia su ecuación será: .2 . ..2 * En genera] las circunferencias cuyo radio posee una longitud determinada, pueden convertirse en circunferencias trigonométricas si dicha longitud se constituye en un equivalente unitario. Por ejemplo una circunferencia de 3,5 m de radio se convierte en una circunferencia trigonométrica si hacemos: 3,5 m = 1 u, donde u es una nueva unidad de longitud. 7.2. ELEMENTOS DE UNA CIRCUNFERENCIA TRIGONOMÉTRICA Toda circunferencia trigonométrica tiene los siguientes elem A(1 ; 0): origen de Arcos A'(-l ; 0): origen de Suplementos B(0U) y = t B(0; 1): origen de Complementos Z P(x;y): extremo del Arco ct A(~l» entos: y . eje de tangentes eje de cotangentes aV 0 x = 1: eje de Tangentes \ P(x;y)\ y = 1: eje de Cotangentes B’((pií 7.3. LOS NÚMEROS REALES SOBRE LA C.T. A(1; 0) x = 1 En matemática es importante desarrollar la habilidad de aproximar. Empecemos esta tarea haciendo las siguientes aproximaciones: n = 3,14, entonces: 2it = 6,28; rt/2 = 1,57; 3n/2 = 4,71. A continuación nos proponemos elaborar un gráfico en el que estos números aproximados sean ubicados sobre la circunferencia trigonométrica: n Problemas de Trigonometría y cómo resolverlos ARACSO Pbditobbi
Si asumimos válidas nuestras aproximaciones, observamos que ambos gráficos son equivalentes . En forma análoga se pueden ubicar los números reales negativos sobre la C.T. Si ahora tomamos como referencia los números reales: 0; 1,57; 3,14; 4,71 ; 6,28 , entonces se puede predecir la ubicación aproximada de los números enteros 1; 2; 3; 4; etc. Es necesario reconocer que cualquier número real ubicado sobre la C.T., define sobre ésta un arco en radianes. 7.4. REPRESENTACIÓN DE LAS R.T. CON SEGMENTOS DIRIGIDOS EN LA C.T. 7.4A Seno El seno de cualquier número real (o arco) en la C.T. está representado por la ordenada del extremo del arco. 7.4B Coseno El coseno de cualquier número real (arco) en la C.T. está representado por la abscisa del extremo del arco. Si: a g R => -1 £ eos a < 1 V \ mínimo máximo Circunferencia Trigonométrica cosa t -oo y-x IX mínimo máximo
7.4C Tangente La tangente de cualquier número real (arco) en la C.T. está representada por la ordenada del punto de intersección del eje de tangentes con la prolongación del radio que pasa por el extremo del arco. 7.4D Cotangente ae R-(2n + 1)n/2 ;V neZ => tan a e R La tangente no tiene máximo ni mínimo La cotangente de un número real (arco) en la C.T. está representado por la abscisa del punto de intersección del eje de cotangentes con la prolongación del radio que pasa por el extremo del arco. a g R - (nn) ; V n e Z => cot a e R La cotangente no tiene máximo ni mínimo La secante de un número real (arco) está representada por la abscisa del punto de intersección del eje x con la tangente geométrica trazada por el extremo del arco. ag R-(2n + 1)ti/2 ; V ne Z => sec aS-1 v sec a 2: 1 seca seca -co - co Máximo Relativo Mínimo Relativo 7.4F Cosecante La cosecante de un número real (arco) está representada por la ordenada del punto de intersección del eje y con la recta tangente trazada por el extremo del arco. Problemas de Trigonometría y cómo resolverlos fas RACSO Pbditobbi
ct e R - (n ir) ; V n e Z i => csc ai -1 v csc a2:1 csc a csc a -co - a> Máximo Mínimo Relativo Relativo 7.5. RELACIONES AUXILIARES (seno verso, coseno verso y exsecante) 7.5A) Seno verso o verso Matemáticamente el versó de un número real (arco) se define así: vers a = 1 - eos a En la circunferencia trigonométrica está representado mediante el segmento dirigido desde el pie del seno hasta el origen de arcos. ct e R => 0 < vers a < 2 vers a jO 2^ a> mínimo máximo 7.5B) Coseno Verso o Coverso Matemáticamente el coverso de un número real (arco) se define así: - ¡ cov a = 1 - sen a En la circunferencia trigonométrica esta representado como el segmento dirigido desde el pie del coseno hasta el origen de complementos. V a e R => 0 i cov a S 2 cova J) 2^ mínimo máximo 7.5C Exsecante o External Matemáticamente la exsecante de un número real (arco) se define así: exsec a = sec a -1 Circunferencia Trigonométrica
En la circunferencia trigonométrica está representada como el segmento dirigido desde el origen de arcos hasta el extremo de la secante. máximo relativo mínimo relativo PROB. 1 De la circunferencia trigonométrica mostrada, se pide expresar en términos de 0: a) La longitud del segmento OM. b) El área de la región triangular AOP. Donde: |sen0|=sen0 ; |cos0|=-cos0 RESOLUCIÓN sen0 1-COS0 Aplicando las definiciones de senq y coseno en la C.T., tendremos: a) En el gráfico se puede reconocer que: k AOM - kAHR r--. = t--.-1—Ki |sen0| l + |cos0| b) Área = (brise) (altura) 2 (l)(|sen0|) =* 2 S = sen 0 n Problemas de Trigonometría y cómo resolverlos RACSO IDITOlll
PROB. 2 PROB. 3 En la circunferencia trigonométrica mostrada, descubra una relación para el área de la región sombreada en términos de <¡> Ordenar en forma descendente los siguientes números reales: sen 6; sen 2 ; cos(3,5) ; tan(l) RESOLUCIÓN RESOLUCIÓN Aprovechando la habilidad de las apro- ximaciones, lo que haremos es ubicar los números reales: 6 ; 2 ; (3,5) y (1) sobre la circunferencia trigonométrica. Es recién a partir de dichas ubicaciones que trazamos los segmentos dirigidos que corresponden a cada R.T. de la lista dada. Resolveremos recurriendo a la identificación de las coordenada de cada vértice del triángulo. Empezaremos reconociendo que dos de los vértices están ubicados en los propios ejes coordenados: A’ y B’. En vista que no son fácilmente identificables una base y su altura correspondiente, el cálculo del área se hará aplicando el método de las coordenadas, cuyo algoritmo es como sigue: S = — 3 2- -1 X° eos 4> sen 4* 0 >^-1 -1 x0 S = ¿ J(-sen 0 - eos 0 + 0) - (1 +0 + 0)] Veamos: En atención a la longitud y signo de cada segmento, se puede establecer las siguientes relaciones de orden: tan 1,5 > sen 2 > sen 6 > eos 3,5 Y ordenando en forma ascendente será: cos(3,5) ; sen 6 ; sen 2 ; tan (1,5) PROB. 4 Si eos 0 e ¡ , determine el intervalo S = -77.(1 + sen 4* + cos$) Circunferencia Trigonométrica de valores que puede asumir el sen 0. 17
RESOLUCIÓN Ubicamos los arcos cuyo coseno es -1/2, de este modo reconocemos que existen hasta cuatro posibles valores para 0. Recordando que el radio de la C.T. es uno, los valores de los segmentos dirigidos que representan a los senos de dichos arcos se deducen por aplicación del teorema de Pitágoras. De este análisis se puede establecer que: < sen 0 < 1 ESTRATEGIAS DE RESOLUCIÓN: 1) Graficar la línea trigonométrica correspondiente respetando su ubicación en el cuadrante asignado así mismo su magnitud angular debe obedecer a la información dada. 2) Para calcular longitudes y áreas en la circunferencia trigonométrica se debe considerar las medidas de sus magnitudes como segmentos positivos, en cuyo caso se recomienda trabajar con sus valores absolutos. 3) Un método para el cálculo de una región sombreado limitada por un polígono de «n» lados es el «método de las coordenadas» que corresponde al siguiente esquema: Área sombreada = (M-N) Problemas de Trigonometría y cómo resolverlos A* RACSO 1DITO1II N M
Enunciados de Problemas con Resolución LÍNEA TRIGONOMÉTRICA SENO 01.- Si 0 e IIIC, determinar la variación de: v2 -1 < eos x < ---; entonces la variación de 2 5-3sen0 W = ~T~ es: A)(l;2) B)(|;2) C)(l;3) D)(0;l) E)(-l;2) 02.-Si: 30° < 0 < 150°. calcular la variación de: M = 2 sen 0+3 A)[l;4] B)[l;2] C)[0;l] D)[l;3] E) <-l; 1> A)[1;31 B)[1;2] C)[0;2]D)[2;5] E) [4;5] jc-2 a+1 03.- Sabiendo que: sen 0 = —~~; don- de 0 e IVC, determinar el intervalo de valores que puede asumir “a”. 07.- Determinar la variación de: W = - sen x - Vi-2 sen acosa - cosa si: aG <n;5n/4) A) <0; 43 > B)<O;J2> C)(0;3> D)<0;3/2> E)<0;V2/2> 08.- Sabiendo que: 0 G 71 7ü\ T; "7/ .determinar 4’4/ A)(-l;|) B)(-l;3) C)(0;l) D)(0;2) E)(l;3) |~ \ 04.-Teniendo en cuenta que: .ve se pide calcular el valor mínimo de: W = 1 + sen |2a| el rango de valores que toma “M”. si: M = sen (2 tan 0+1) A)[senl;2] B)[-senl;0] C)[-senl;l] D)[senl;3] E)[sen2;j2] A)1 B)2 05.- Sabiendo que: C)0 D)3 E)-3 M=4sen"x-1; se pide 09.- Dadas las siguientes condiciones: . í ’t a) sen — + a 6 a —42 calcular la suma del máximo y mínimo valor / 7t 7t que puede tomar M. si: a e — A)-l B)0 C)2 D)-3 E)-2/5 T571 1 b)xe — ;3n Determinar el máximo valor de 06.- Si: a e Y además se verifica que: A) 42 D) 43/2 B) 43 E) 43 +42 C)-V3 Circunferencia Trigonométrica T14
LÍNEA TRIGONOMÉTRICA COSENO 10 .- Calcular el intervalo de valores para M: M = 1 + eos x + cos“> A) [ 1/4; 2] B) [-3/4; 0] C) [-3/4; 2] D)[3/5;2] E)[3/4;3] 11 .- Determine el valor de "a + b" si se verifica la siguiente desigualdad: 7cos0-3 , , a < --------- < b 2 A)-l B)0 C)2 D)3 E)-3 12 .- Identifica los valores de verdad o false- dad de las siguientes proposiciones: I. eos 1 > eos 3 n. |cos4| >cos5 HI. eos 2 < eos 3 15.- Sabiendo que x es un arco cuyos valores está definidos así: x e . se pide determinar el intervalo de variación de W: A)WF B)FVF C)VFF D)VW EJFFV 13.- Determinar el intervalo de variación de W: 3—2cos0 w= 5 si se sabe que: 0 e IVC. W = |2 eos (|.r| - )| A)[l;2] B)[j2;2] C)[-l;2] D)[-J2;l] E)[V2;41 16.- Determine algún mínimo valor de la expre- sión: W — cos( sen2© + 2 sen 0) A) eos 3-1 B) 2cos(-3) C) -eos 3 + 2/3 D) eos 3 E) eos 3 + 1/3 17.- Determine algún intervalo de variación de la expresión: cosa+ 3 M = -z------? 2cosa-l A)[8; + ~> B)[7; + ~> C)<-~;-l] D)[l;3) E)[4; + oo) LIN. TRIG.: TAN - COT- SEC - CSC 18.- Si: 0 G [0; 7t/4], calcular la variación de “m” en: 4m - 3 sec 0= —~— A) 5.2V2+3 4’ 4 B) 1 . v2 4’ 2 C) 1.V2 3’ 3 14.- Teniendo en cuenta que: 0 G 1C, se pide determinar el intervalo de variación de W, si: cosO + 3 W=----------- cosfi +1 A) < 0;l> B) ( -2;0> C) < 1 ;2> D) <3; 4) E) < 2; 3> D) 0;\2 E) Í-V3 3* 3 19.- Determine la variación de *'n”, si se sabe que: 0 G 7t 7t 4'"2 3 v csc~0 =-----r J n + 1 Problemas de Trigonometría y cómo resolverlos RACSO BDITOtlI
A)[0;2] D)[o;£| B)L2’ . C) [4;3] A) B) c> 20.- Si |2 - J3 tan x| < 1, entonces todos los valores de “x" en ( 0; 7t) que verifican la desi- gualdad se encuentran comprendidas en: t D) 4’3j 3 ’ 2 J F——1 Ló ’ 3J E)[0;6] 21.- Si sec x = 2/n + l , —-—, entonces todos los valores de “»i” que no verifican la igualdad se encuentran comprendidos en: 25.- Encontrar todos los valores de “x” que no verifican la igualdad. 3x+2 CSC0= 2775 A)(0;l> B)(0;3> C)(-2;0> D) <-!;!> E)<-!;0> 26.- Si a y P son las medidas de dos ángulos independientes, entonces se pide calcular la suma del máximo y mínimo valor de la siguien- te expresión: W = 5 vers(a) - 4 cov(P) A)(-2;4> B)(-3;l> D)<-2;1> E)<-1;1> 22.- Calcular el intervalo de “x” para que se verifique la igualdad: x + 3 sec 6 —------ , donde: 0 G IIC. x + 2 23.-Si.xe (^p271) entonces la variación de C)<-2;3> A) 2 B)-2 C)3 D)-3 E)1 27.- A partir de la siguientes condiciones: i) exsecx = .J-covy ii) 7<y<9 iii)5<x<7, se pide calcular el valor de: V = x+2y. A)37t B)5n C)4ti D)5n E)7n ÁREAS EN LA C. T. 28.- En la circunferencia trigonométrica mos- trada, /ziAbF = a, PQ J. A'A . Calcular el área de la región sombreada BPA'Q. W= 1-2 2x sec— 3 es: A)(l;3> B)<-3;-l> C)<-2;-l> D)(-l;0> E)<-4;-2> senO + 2 24.- Si: 0 G IIC y csc ó =---—7; determinar J , senO +1 la variación de: “ csc" <J>”. 2 A)eos a B)sen a C)sen "a 7 D) sen'a. cosa E) 2cos2a Circunferencia Trigonométrica
29.- En la C.T. mostrada, se pide calcular el área de la región sombreada, si: tn ABP - 0. 30.- En la circunferencia trigonométrica mos- trada determinar el área de la región sombreada, si m A BP = 0. A) -(sen2 0) B) (sen 0 . eos 0) C) -(sen 0 . eos 0) D) -(eos" 0) E) (sen20.cos0) 31.- En la circunferencia trigonométrica mos- trada se cumple: mABP =0 Determinar el área de la región sombreada. 33.- En la circunferencia trigonométrica mos- trada se cumple m A B A'P = 0. Calcular el area de la región sombreada. A) -sen 0 eos 0 B) sen 0 eos 0 C) 2sen 0 eos 0 D)sen3 0 E) 3cos2 0 34.- En la circunferencia trigonométrica se sabe que: wí? = 0, PQ ± RS, ÁQ ± A'A . De- terminar el área de la región sombreada. Suge- 2 2 rencia: sen 0 + eos 0=1 A) y cos0 (1 + sen0) B) sen0 (1 + cos0) C) cos0 (1 + sen0) D) sen0 (1 + cos0) E) ^cos0(l -sen0) 32.- En la circunferencia trigonométrica mos- trada se cumple m A P =0. Determinar el área de la región sombreada. 3 A)-— csc0 B)-^ sec20 sen0+cos0 „ sen0+cos0 „ sen0 A)---------- B)------o---- C} 2~ 2 2 1 7 C) *2 sec 0csc0 E) 2sec2 0 csc0 J3 7 D) sec 0 csc0 Problemas de Trigonometría y cómo resolverlos RACSO 1DITOBII
35.- En la circunferencia trigonométrica se cum- ple mABP = 0, calcular la variación del área E)[2;4] 36.- En la circunferencia t gonométnca se sabe que: mÁP' = 0, QB’ _L BB’; AT _L A'A . Evaluar el área de la región triangular ORQ. A) 2/7 B) 1/3 C)2/5 D)3/4 E) 1/4 37 .- En la circunferencia trigonométrica mos- trada, = a, AR -L A'A . Calcule el área de la región sombreada RTO. 38 .- En la circunferencia trigonométrica mos- trada se cumple que: m A BP =0. Calcular el área de la región sombreada si se sabe que “P” es un punto de tangencia. A) (tan 0 - cos0) B) (cot0 + sen0) C) (cot 0 - cos0) D) (tan 0 - cos0) E) (tan 0 + sen0) 39 .- En la circunferencia trigonométrica mos- trada,nABM' = a, PM ± MR; PB ± BR . Evaluar el área de la región triangular POR: .. 1 2 A)-eos a. esc a 2 B) eos a. esc a C) sen a. tan a 1 2 D) - tan a csc a 1 2 E) —cota.sec a 40.- En la circunferencia trigonométrica mos- trada,= 0,PQ_LAA'; A'R=RB. Si el área de la región triangular RQB' es de la forma S = M + N eos 0 + T sen 0, entonces el valor de M +'N + T es: A) 2/7 B) 1/4 C)3/7 D)5/4 E)3/8 Circunferencia Trigonométrica
41.- En la circunferencia trigonométrica mostra- da se tiene: mÁP' = a, PQ ± OA; SM ± OU, SU -L OU, m Z_ POT=90°. Calcular el área de 42.- Analice la verdad (V) o falsedad (F) de las siguientes proposiciones: I. eos 25° < sen 25° II. csc 40° < sec 40° III. tan 27° > cot 27° A)FW B)WF QVW D)FFF E)FVF 43.- Identifique la veracidad o falsedad de las siguientes proposiciones: I. sen 3 > sen 4 II. sen 4 > sen 5 III. sen 3 > sen 6 A)VW B)WF QVFV D)FW E)FFV 44.- Indique verdadero (V) o falso (F) según como corresponda: I. |cos2|>|cos3| II. sen 290° + eos 290° > 0 III. |sen 1| < |sen 3| A)VFF B)FFV C)FFF D)FVF E)FW 45.- Siendo: 7t <x( <x-, < 3rt/2, señalar la ver- dad (V) ó falsedad (F) 3e las siguientes propo- siciones: I. sec x, < sec x2 II csc > csc x2 III. IsecxJ^JsecxJ A)VW B)FVF C)FFV D)FFF E)VFF 46.- En una circunferencia trigonométrica se 7t verifica que: ~ < Jtj <x2<n. Indicar la verdad (V) ó falsedad (F) de las si- guientes proposiciones: I. tan xt > tan x2 II. Icosxjclcosxj III. cot x, < cot x2 A)FFF B1FFV C)FVF D)VFF E)FW 47 .- De la figura mostrada calcular: 48 .- En la circunferencia trigonométrica mos- trada se cumple, m A P =0. Determinar la lon- gitud del segmento BQ. Sugerencia: sen(-0) = -sen 0; cos(-0) = eos 0 A) 2 sen 0 B)2cos0 D) 1 +sen0 cos0 E) 1 - sen 0 2cos0 Problemas de Trigonometría y cómo resolverlos Mi RACSO P IDITOBII
49 .- En la circunferencia trigonométrica mostrada se sabe que: tn A P = a. además se verifica que: OQ = Q A Se pide calcul ar: 50 .- En la circunferencia trigonométrica mos- trada, calcular el valor de OM si: m A B P =0 51 .- En la circunferencia trigonométrica mos- trada se pide calcular Jl PM,si: m ABP =0. A) 1 + sen 0 + eos 0 B) 1 - sen 0 + eos 0 C) 1 + sen 0 - eos 0 D) 1 - sen 0 - eos 0 E) 1 - 2 sen 0 + eos 0 52.- En la circunferencia trigonométrica mos- trada se cumple que: m A B P =0. Calcular las coordenadas del punto “M”. A) (-eos 0 ; -eos 0) B) (-sen 0 ; sen 0) C) (-sen 0 ; -eos 0) cos0 A) l-cos0 m cos0 2cos0 l-sen0 l+sen0 D) (sen 0 ; -eos 0) E) (sen 0 ; eos 0) 2cos0 D) l-3sen0 p. sen0 ’ l-cos0 Circunferencia Trigonométrica
8.1. DEFINICIÓN_____________________________________________________ Son igualdades entre expresiones trigonométricas, las cuales se verifican para todo valor admisible de la variable angular. Las Identidades Trigonométricas fundamentales se agrupan en identidades pitagóricas, por cocientes, recíprocas y auxiliares. En términos operativos, las identidades expresan igualdades entre razones trigonométricas que permiten reducir expresiones trigonométricas. Las identidades trigonométricas son múltiples, y se utilizan adecuadamente según el tipo de situación problémica que se enfrente. Aquí presentamos un resumen de las identidades trigonométricas fundamentales: i) Identidades Pitagóricas iijldentidades por cociente üi) Identidades Recíprocas iv) Identidades Auxiliares PITAGÓRICAS COCIENTE RECÍPROCAS AUXILIARES sen2 a + eos2 a =1 sec2 a - tan2a =1 esc2 a - cot2a =1 sena tan a = cosa eos a cota- sena l csca= sena seca" cosa cota" tana tan a + cot a = sec a esc a sec2 a + esc2 a — sec2 a esc2 a sen4 a + cos4a = 1 -2 sen2 a eos2 a sen6a + cos6a = l -3sen2acos2a (1 ± sen a ± eos a)2 = 2(1 ± sen a)(l ± eos a) Nota.- 2 2 2 Si: asenx + b cosx = c a a +b =c Entonces: sen x = — ; eos x = — ; tan x = 77 c c b T8 Problemas de Trigonometría y cómo resolverlos RACSO DITOIIt
PROB. 1 Luego de simplificar y reducir, tendremos: Demostrar que: 2 (senx + cosx) -1 2 —-------------- = 2 tan x cotx-sen xcosx cosx eos x sen x 1 senx k = esex k = RESOLUCIÓN *-****<r<r***i***i*** Transformamos el primer miembro para obtener el segundo miembro: PROB. 3 Si se cumple que: 2 2 sen x+cos x + 2senxcosx-l eos x —-senxcosx senx l + 2senxcosx-l 2 - cosx-sen xcosx senx 2sen xcosx. senx _ cosx(l-senZx) 2senZx .2 ----2— = 2 tan x eos x 1+senx 1-cosx = X |cotx + esex + 11 calcula el valor de X. RESOLUCIÓN Multiplicando dentro del radicando al numerador y denominador por (1 + eos x), se tiene: (1 +sen x)(l+cosx) _ .. .. (l-cosx)(l + cosx) “ Á|COIX + cscx + 11 PROB. 2 1(1+sen x)(l+cosx) ,. . . .. ------= X cotx + esex + 1 (1-cos2x) 1 Reducir la expresión: secx + cscx 1+tanx 2(l+senx)(l+cosx) . ,, —----------------- = X| cotx + esex +11 2.sen x RESOLUCIÓN Expresando en senos y cosenos: 1 2 (l + senx + cosx) 2senZx = X|cotx + esex + 11 1 1 (senx + cosx) cosx senx _ cosx senx । senx (cosx + senx) cosa cosx 1 |l+senx + cosx| = X| cscx , j /2 I senx | 1 1 Identidades Trigonométricas T8
1 t cosa + senxl _ senx senx senxj X|cotx+cscx + 1 |cscx + cotx + 11 = X |cscx + cotx + 11 1 12 ’2 2 PROB. 4 Sabiendo que se cumple la siguiente condición: J7 eos 6 + 1 = tan2 6 , obtenga el valor de: k = tan6 6 - tan4 6 - tan2 6 RESOLUCIÓN ****************** Del dato despejamos: J7 eos 6 = tan2 6 -1 Elevando al cuadrado: 7 eos2 6 = tan4 6-2 tan20 + 1 7 = —U- (tan4 0 - 2 tan2 6+1) eos 0 => 7 = sec2 0 (tan4 0 - 2 tan2 0 + 1) 7 = (1 + tan2 6)(tan4 6-2 tan2 6+1) Efectuando y ordenando como sigue, tendremos: 7 = tan46-2tan% + 1 + tan66-2 tan40 + tan20 Finalmente: tan6 6-tan4 6-tan2 0 = 6 ~k k = 6 ESTRATEGIAS DE RESOLUCIÓN 1) Para efectuar una demostración, se parte del miembro más complicado para llegar al segundo miembro. 2) En muchos problemas es recomendable expresar en función de senos y cosenos, pero no significa que siempre se realice dichos cambios. 3) Es conveniente realizar artificios, de tal manera que se obtengan identidades conocidas, para beneficio propio del problema. 4) En el caso de problemas condicionales se sugiere trabajar tanto en la parte condicional (dato) como en la parte problémica (incógnita) Problemas de Trigonometría y cómo resolverlos RACSO dDITOIll
Enunciados de Problemas. con Resolución DEMOSTRACIONES 01.- Demostrar que: 1 + senx + eos 2 *x ------------= 2-senx 1 + senx 02.- Demostrar que: 10.- Demostrar que: 2 9 2 *» cot x.(l+cos"x)+tan x.(l+sen~x) = - 2 2 - = 2sec x.csc x-5 SIMPLIFICACIONES 11.-Simplifican (senx—cosx +1 )(senx + cosx -1) ----------:------------ = 2 eos x 1—cosx 03.- Demostrar que: 2 (1-senx+cosx) = 2(1-senx)(l + cosx) 04.- Demostrar que: .4 2 2 2 2,6 cot X . CSC X - cot X. CSC X + CSC x - 1 = cot X _ senx I cosa 05.- Demostrar: ------=---------- 1 + cosx senx / . \2 z ( secx.cscx—tanx i ( secx.cscx —cotx M = --------------- + ---------------- esex j secx A) 1 B)sec2xC) csc2x D)-l E)0 12.- Reducir: z. x-2 , - x-2 ( tanx + secx ) I cotx + esex ) M = -------+ ---------------r~ senx+1 j cosx+1 j A) tan2x B)cot2x C)cos2x D)2 E)1 2 2 9 2 2 06.- Demostrar: tan x - sen“x = tan x .sen x 13.-Simplificar: 07.- Demostrar q ue: 8 . 8,-2 2 — 44 sen x + cos x= 1 -4sen x.cos x+ 2sen xcos x 08.- Demostrar que: 1 1 ,, 2 x 2 T7-----+ i-------- - (1 + sen a) - 2 tan a= 1+senc 1-senc y ' 2 — eos a 09.- Demostrar que: 1—senx 1 + senx . 2 - , . , =4sec x-2 1 + senx 1 — senx sec2x + 2tanx csc2x—2cotx M =----------.— +-------------— tanx +1 cotx — 1 2 A) sen x.cos x B) senx C) eos x D)tanx.cscx E) secx.cscx 14.- Identificar la expresión mas reducida para: M = cosx+senx.tanx senx.secx A\ * 2 A) tan x.csc x B)secx C)cscx 2 D) sen x.cot x E)cotx Identidades Trigonométricas
15.-Simplificar: (1 —cosx).cscx 1 — cosx .. cosx tanx M=---------;-------------- tanx—senx A)-l B)2 C)1 D)-2 E)0 16.- Reducir: M = secx(cscx- l) + cosx(sec x-cscx) A) cot x B) tan x C) cot x. sec x 3 D) sen x. sec'x E) tanx 17.- Simplificarla expresión: sec2x+cos2x—2 sec2x+cos2x+l W=------------------- .--------------- secx + cosx — 2 secx + cosx +1 A) 2 B)1 C)4 D) 1/3 E)3 18.- S'mplificar la expresión: cot6x + 3csc2x.cot2x +1 M — í 7 ñ tan x+3sec x.tan x + 1 A) tan4x B) sen x. cot5x C) sec x. cot3x D) sec5x E) cot6x 19.- Simplificar la expresión: W = sec6x - 3tan2x.sec2x - tan6x + 2 tan x.(sec x- tanx).(cscx+ 1) A) 2 B)4 C)5 D)-l E)3 .20.-Simplificar: tan4x + sen4x—tan4x.sen4x M = ~--------------------~ (tanx + senx)(tanx—senx) A) 1 B)0 C)3 D)4 E)2 21.- Simplificar: vers(x).[3 - exsec x] + 2 + exsec x W=-------------3^2^-------------- A) 1 B)0 C)-l D)-2 E)2 22.- Reducir: 1 £ f. 1 j 2 2 M = — (sen x + eos x) - — (eos x - sen x) 13 1 11 A> 6 B>8 D’í E>ñ 23.- Reducir: 1+tanx + secx 1 —sen2x M = ~------------ +--------------- 1 + cotx + esex cosx + senx.cosx A) tan x B) csc x C) sen3x 2 D)cosx E)secx 24.- Reducir: , , cosx versx versx—covx M - ------- +---=— +--------ó-- 1 + senx cos2x eos x A)2cotx B)3cosx C)2sen3x D) cos2x E) tan3x 25.- Reducir: 2 2 M = vers x + cov x + 2(sen x + eos x) A) 1 B)2 04 D)-l E)3 PROBLEMIZACIÓN CONDICIONAL 26.- Si: sen20 + sen 0=1 determinar: M = cos40 + cos20 A) 2 B)3 04 D)5 E)1 3 27.- Si: sen'x + sen x = m 3 eos x + eos x = n calcular: M = m csc x + n sec x A)-2 B)-4 05 D)-l E)3 T8 Problemas de Trigonometría y cómo resolverlos RACSO DITOIII
determinar: M = 1 - sen x - eos x 34.-Si: sec x = a + csc x 2-y3 z->\ 42 a)T B) k C) k D> f E) > 29.- Si: sen4x + cos4x = k , entonces calcular: 3 2 2 3 2 W=(senx+cos x) +(senx-cosx) +(cosx+sen x) 3 A)3(14) B)-(1+Jt) C)^(l+¿) 3 1 D)|(l-¿) E)|(l-¿) 30.- Si: sen x + eos x = 42 , -.44 determinar: M = sen x + eos x 3 5 1 2 5 A)| B) | C)| D)-y E)~3 31.-Si: 2sec~x - csc'j = 1, entonces calcular: 2 2 W = 2sec y - csc x A)0 B) 1 C)3 D)4 E)2 cos4x—sen4x 32.-Si se cumple: -g----g- =m ; eos x—sen x k calcular: M = 1 + sen6x + cos6x A>^ B) L~~ C)~^— ' tn ' 2m sen x-versx , 33.- Si:----:----- - k, sen x+vera x entonces calcular: W = k - A1 A)-2cotx B)-2senx C)3tanx D)-2tanx E)tanx determinar: M = tan x + cot x - 1 Además se sabe que: x G IIC, a > 1 A) 7a2+1 B) V3c2 + 1 C) Vc2-1 D) V2«2-l E)-7o2+1 35.- Si: tan a - csc a = 1 , calcular: W = (csc a - eos a)( 1 + cot a) A) 2 B)3 C)4 D)-5 E)1 I 1 o 36.- Si: k > 0, se verifica que: k+— > 2 , cal- cular el mínimo valor de: 2 2 W = sec x + csc x + 2 sec x . csc x A) 3 B)4 C)5 D)9 E)8 37.- Si: tan a- sen a= 1, calcular: W = sec a. csc a - sen a + eos a A)-2 B)2 C)4 D)-l E)-3 2 3 38.- Si: sen x - eos x + sen x = 0, entonces calcular: 3 W - csc x + sen x A)0 B) 1 C)2 D)8 E)-2 2 2 n 39 .- Si: sen x + csc x = 7, entonces calcular: W = eos x. cot x + 2 sen x A)-2 B)±2 C)4 D)-5 E)±3 40 .-SÍ: cotx + cosx=l, 2 calcular: W = cot x + csc x A) 42 B)5V3 C)2V2 D) 1 E)3V2 41 .- Si: sen4a = 2 cos4a + a. csc4a ... (1) b sec2a = cos6a + 4 sen2a ... (2) Identidades Trigonométricas T8
calcular -.a+b. 47.- Determinar “ni' en la siguiente identidad:. A)9 B)2 04 D)3 E)1 42 .- Sabiendo que se verifica: tan r+cot v+m _ sec3x+csc3x tan x+cot x+2 (secx+esex)3 ’ calcule el valor de m. A)2 B)3 Q-l D)5 E)4 43 .- Si se cumple que: (2-cos2aXl+sec2a) 1 5 = 2 + , l + 2tan-ct * entonces calcule el valor de k. A) 3csc"a B) - csc'a C) 6tan a — 2 3 D)-4sec a E)4cot“a eos x(3+tan r— 2 sec~r) 1—wi 2tanx + l ~ secx A) tan x B) 2cos x C) 5sen r D) -cot x E) 3tan v 48.- Si: exsec x + tan x + 1 = a , a * 0. a * ± I calculancot x A) 2 «2-l 2o B) 2 । a -1 D) 3(n-l) 2« 49.- Calcular “x”, a partir de: lo 7 (x + cov 0)- + vers-0 = (exsec 0)"; x > O a e IC A) 2sen 0-1 B) cot 0 - 1 C) tan O - 1 D) 2tan 0+1 E) sec 0+1 entonces calcular: eos r A) cot n . tan b B) 2cos a . sec b C) tan b D) sen a . tan b E) eos a . tan b 45.- Sabiendo que: tan _x cot2x --------- --------- = 4 + sec x , secx — 1 esex — 1 calcule: sen x + esc x. 3 5 5 5 2 A) j B) 3 O-5 D) - E)-| 46.- Calcule el valor de “ ni ” si: secx-tanx secx + tanx in --------- + ------- — = ni + ni . cot x. seci + tan r secx — tan r A)3 B)4 C)2 D)-2 E) 1 l + sen4r 50.-Si: ------7— =tan r 1 + cos x calcular: M = sen x-sen3x+sen5x—sen7x eos x-cos3x+cos5x—cos7x A) 2 B)3 C)1 D)-4 E)5 ELIMINACIÓN DE ARCOS 51.- Determinar una expresión independiente de la variable angular “x”, si se sabe que se verifican las siguientes condiciones: a b ----=------- ..(1) senx cosx 1 sen 1. eos x — ~ . (2) A) ab2= a-b B) «2 + ¿>"= 3«£> C)a~b = 1 D) lab = a2 + b2 E) ab = a + b Problemas de Trigonometría y cómo resolverlos Jk RACSO PBDITOKBfl
52 .- Si se sabe que se verifican las siguientes condiciones: 1 + sen4* = a eos2*. sen2*... (1) 2 - eos4* = b sen2*. eos2*... (2) 2 determinar el valor de: F = (b - a) A) 9 B)3 C)25 0)4 E)16 sen* eos* tan* 53 .- Si:- =---- =------; abe se pide identificar la expresión equivalente de: V = n"(«2 + ¿>2 ) A)flV B)/?c2 C)2ab2c D)4 E)3 54 .- Si se verifican las siguientes condiciones: «(1 + tan"*) = (1 - tan*) ---(1) (1 - cot*)2 = (b - 1)(1 + cot"*) ... (2) se pide calcular el valor de: A = b - a A) 1 B)-2 Q3 D)4 E)1 55 .- A partir de las condiciones: p eos2*. sen*= q... (1) 2 p sen *. eos * = r... (2) encontrar una expresión independiente de «*» A)p q r =(r +q ) 2 2 z 2 2» 3 B)r +q =(p +q ) O r -q =(2r +p ) D) p q =(p +r ) E) 4pqr — (p2 + 2q2)3 56 .-Si: tan* .(1+cos*) = 4p ---(I) tan*. (1- cos*) = 4iy ...(2) determinar una expresión equivalente para: N = p? - q2 A) Vp v/fl bWp «i O Jpq E)2^p’ j 57 .- Sabiendo que: ni sen a h eos i = nn (1) n sen * + ni eos * = n -t I (2) determine el valor de: E = i» + n + I A) 2 B)0 03 E)-2 58 .- Si: sen"* - eos"* = a .. . (1) tan4i - col4* = b . . (2 ) determinar el equivalente de: S = iía(a + 1) A) a( 1 - fe2)2 B) ¿>( 1 - n2)2 C) (1 - b2)2 D)<i(l+¿>2)2 E)(l-o2)2 3 5 8 59 .-SÍ: tan’*+tan* = /ri -..(1) 3 5 8 cor* + cor * = /i ... (2) 2 2 identificar el equi valente de: S—m + n A.) 2nm B) ni + n O 2 D) ni5 n5 E) m n 60 .- Si se sabe que: * e (o; , y además: 1+sen* m=~-------- ..(1) 1 — sen* n = (tan* + sec*- l)2 ...(2) calcular el valor de: A = -Jñi —Jñ A)2 B)3 05 D)0 E)1 Identidades Trigonométricas T8 91
Ijd^ñti d a d |s Tr ¡ g¡ó ñom é t ri cas fccdeÁrcos“C^mptí estos ¡Cíe i—U.-- .1. - I 9.1. R.T. DE LA SUMA DE DOS ARCOS___________________ sen (ct + P) = sen a. eos P + eos a. sen p eos (a + P) = eos a. eos p - sen a. sen p tan (a + P) = tan a + tan P 1-tana.tanP 9.2. R.T. DE LA DIFERENCIA DE DOS ARCOS sen (a - P) = sen a. eos P - eos a. sen P eos (a - P) = eos a. eos P + sen a. sen p tan (a - P) = tan a - tan p 1 + tana.tanp 9.3. RAZONES TRIGONOMÉTRICAS PARA LA SUMA DE TRES ARCOS sen (a + P + 6) = sen cucos P-cos 6 + sen p.cos cucos 6 + sen O.cos cucos P - sen cusen p.sen 6 eos (a + P + 6) = eos cucos p.cos 6 - eos cusen p.sen 6 - eos p.sen cusen 6 - eos 0.sen cusen p tancc + tanP+tanO-tanatanptanO an ct + p + I-(tana.tanP + tanatan6 + tanptan6) 9.4. IDENTIDADES AUXILIARES_______________________________ 1) Si: ct + P = 0 => tan a + tan P + tan 0 tan a tan P = tan 0 2 2 2 2 2) sen (a + P) sen (ct - P) = sen a - sen p 3) eos (a + P) eos (ct - P) = eos a - sen P « n sen(a + P) „ n sen(ct-P) 4) tan ct + tan P = cosacosp 5) tan ct - tan p = cosacosp T9 92 Problemas de Trigonometría y cómo resolverlos RACSO 1CITQ111
6) 1 + tan a tan p = cos(ct-P) cosacosP cos(P-a) 8) tan a + cot p = ------- senp cosa / 2 2~ 10) a senx +b cosx = va +b sen(x + <[>) / 2 2 / 2 2 11)-va +b <a senx ± fe cosx< Va +fe eos (a + P) 7)1- tanatanp= ^¿¿¿“p cos(P + a) 9) cot p -tan a = senpcosa Siendo: tan ó = — v a sen(a + P) 12) cot a + cot p = senasenp sen(p-a) 13) cot a-cot p= senasenp 14) tan a - tan p - tan(a - p) tan a tan p = tan (a - P) 9.5. IDENTIDADES CONDICIONALES__________________________________________ i) Si: a + P = 45° => tan a + tan P + tan a tan P = 1 ii) sen x± cosx = J2 sen (x + 45°) iii) sen x ± -J3 eos x = 2 sen (x + 60°) iv) SÉ x + y + z = kn V k e Z tan x + tan y + tan z = tan x tan y tan z cotx. cot y + cotx. cot z + cot y. cot z — 1 Identidades Trigonométricas de Arcos Compuestos
PROB. I PROB. 2 Calcula el máximo valor de: k = 5 sen(x + 37°) + J2 cos(x - 45°) Simplifique la siguiente expresión: E = tan x + tan 2x + tan 3x tan x tan 2x RESOLUCIÓN RESOLUCION Desarrollando como sigue: k = 5(senxcos37° + cosx sen 37°) + J2 (eos x eos 45° + sen x sen 45°) Se observa que: (x + 2x) = 3x Tomando tangente en ambos miembros, obtendremos: 4 3 »- ¿2 k = 5 senx.-5 + 5 cosx.+ ¿2 cosx-^- + tan (x + 2x) = tan 3x Simplificamos, obteniendo: k = 4 sen x + 3 eos x + eos x + sen x => k = 5 sen x + 4 eos x A continuación dividimos ambos miembros b —¡= = cose senx + sen 6 cosx -¿41 Luego: -j= = sen (6 + x) Finalmente: k = -J41 sen (6 + x) Su máximo valor será: h = V51 tanx + tan^x =tan3x l-tanxtan2x Luego efectuamos como sigue: => tanx + tan 2x = tan 3x-tan 3xtanxtan2x Finalmente: => tan x+tan 2x +tan 3x tanx tan 2x E~ E = tan 3x PROB. 3 En la figura que se muestra, calcule tan x, siendo M y N puntos medios de lados AB y BC respectivamente. Si: mZACB = 37° T9 Problemas de Trigonometría y cómo resolverlos RACSO
=> 25 tan x = 18 tanx = RESOLUCIÓN Se observa que: 6 = x + y aplicando propiedad. => tanx + tan y + tan 6 tan x tan y = tan 6 3 Pero: tan y = g . o 3 tan 6 = 3 3 3 3 Luego: tanx+ g + -^tanx g = 18 25 PROB. 4 Calcula el valor de: sen(x-y) sen(y-z) sen(z-x) cosxcosy + cosycosz coszcosx RESOLUCIÓN ★ iot^iifiii****** sen(a-P) Se sabe que: cosacosp = tan a - tan p Haciendo un trabajo análogo tendremos: k = tan x - tan y + tan y - tan z + tan z - tan x => 16 tanx + 6 + 9 tanx = 24 k = 0 ESTRATEGIAS DE RESOLUCIÓN 1) Para hallar el valor de alguna razón trigonométrica, de un determinado ángulo se debe descomponer en una suma o diferencia de otros dos ángulos cuyas razones trigonométricas sean conocidas. 2) En figuras geométricas se debe buscar un ángulo exterior en un triángulo, para luego aplicar la propiedad siguiente: Si: x + y = z => tan x + tan y + tan z tan x tan y = tan z 3) Para encontrar máximos y mínimos de expresiones de la forma: k = a sen x ± b eos x Debemos recordar que: = Ja2 + b2 a km(n = - Ja2 + b2 4) Cuando se tienen sumatorias de expresiones trigonométricas, se debe reemplazar por diferencias, para que se simplifiquen, para ello se debe tener en cuenta que: tan x - tan y = sen(x—y) eosxeos y cot x - cot y = sen(y—x) sen x sen y Identidades Trigonométricas de Arcos Compuestos T9 95
Enunciados de Problemas con Resolución SENO, COSENO DE ANCOS COMPUESTOS 01.-Si: 7? sen x + eos x =- 8 Determinar el valor de: M = 16 sen j A) 1 B)2 C)3 D) 1/2 E)3/2 02.-Calcular: M = (sen 18° + cos 12°)" + (sen 12° + cos 18°)" A)3 B)2 C)1 D)0 E)0,5 A) 1/2 B)2 C)4/3 D) 1/4 E)3/4 07.- Eliminar los arcos “x" e ‘V’de las condiciones: sen(jc + y) . sen(x - y) = a ... (1) cos(jc + y). cos(jc - y) = b ... (2) eos x. sen y = c ... (3) A)fl2 = h2-c2 B)n" = Z?" + c' C)n2 = hc D) (1 + n)2 = b" + 4c2 E) (1 - a)- = b2 + 4c" 08.- Eliminar los arcos “x” e “y” de las condiciones: 03.-Reducir: j3sen50" cos50" sen25°-cos25° sen x + sen y = m ...(1) eos x + eos y = n ... (2) A)0 B)0,5 C)-72 D) 72 E) 73 04.-Reducir: 2 M=^en (n + b) - 2 sen(a + h).cos n.sen b + sen 6 A) sen a B) sen b C) sen"« D) sen2í> E) 1 05.-Simplificar. eos "(a + p) - 2cos a.cos p cos(a + p) + cos2p A) sen a B) sen2a C) eos a D) cos2a E) eos2 p cos(x-y)=p ...(3) 7 3 i 3 A) 2( 1 + p) = m + n~ D) 2p = m~ - n~ B) 2p=mn E) 2( 1 - p) = m~ + n~ C)2p — m-n TAN, COT DE ARCOS COMPUESTOS 09.- Si: x + y = Calcular: } 4 tanx + ta ny cot a + coty A) 1 B)0 C)0,5 D) 1,5 E)2 10.- Simplificar: W = tan 2a + tan a + 06.- Si: x + y = 30°, entonces al calcular W = sen2y + eos2* - sen y .eos x. se obtiene: tan 3a eos 2a T9 96 A) tan a D) 2 tan 3a B) tan 2a C) tan 3a E) cot a Problemas de Trigonometría y cómo resolverlos a4RACSO WiDiToaas
11 .-Simplificar: 2tanA + 3tanB — tanC - tanA.tanBtanC W=--------------------------------------- tanA + 4tanB - 2tanC — 2tanAtanBtanC con la condición A + C = B A)1 B)0 C)0,5 D)tanA E)tanB 12 .- Si: 4 tan (x - y) + 3 sec jccsc x = 4 tan x, entonces al calcular cot x.cot y se obtiene: A) 1/2 B)l/3 C) 1/4 D)3/2 E)3/4 2 2 2 2 13 .- Si: tan x + 2 tan x. tan y = 1 + sec y , tan (x - y) = 3 Determine: M = tan(x + y) A) 1/3 B) 1/2 C)3/2 D)2/3 E)3/4 14 .-Calcule: W=-j3(l tan 5o. tan 10o)+ (1 + tan 10o). tan5°+ (1 + tan 5°). tan 10° A)0 B)0.5 C)1 D) 1,5 E)2 15 .- Calcular: 41 — J3cot65°+cot800 cot800.cot65° 1 +cot65°—J3cot80° cot65°—J3cot80° A)0 B) 1 C)1.5 D)2 E)2,5 16.-Calcule: W=cot254°(l -tan290)+cot81<’(tan90+4tan360) A) 1 B)l,5 C)2 D)2,5 E)3 a+b 17.-Si:tan(x + y+z)=----- Atan(x-y-z)- 1 a —b Determinar: “tan 2x” A) a B)-a/b C)b D)b/a E)«/í> / A 1 18.- Si: tan--------a = —, calcular [14 J 2 A) 1 B)2 C)3 D)4 E)5 19.- Si: sen(a - p) = 3 cos(p — a); cot(a + p) = 0,5; entonces al calcular tan(2a) se obtiene: A)0 B) 1/2 C)-l/2 D) 1 E)-l 20.- Sean tan p y tan q las soluciones de la ecuación (jc- 1)(A1 2 jc+1) = 2A , Ae R-{-l;OJ Calcule: tan(p + q) en términos de “k”. 1 k +1 2 L -1 A)I B)T C)^ D)r E)^ 21.-SÍ: N = tan21°+ tan24°+ tan2l°.tan24° M = tan 63° - tan 63° - 41 tan 63°.tan 63° Calculare! valor de: “N.M2” A) 1 B)2 C)3 D)4 E)6 IDENTIDADES ESPECIALES 22.- Calcule el valor mínimo de la expresión: W = n(sen x - eos x) + fe(sen x + eos x) A) ab B) a + b C) -Ja2 + b2 D) 1/2 ab E)-j2(a2+b2) 23.-Simplificar: Ía + b A 4 a b\ ---- —sen ------- 2 2 ----L-------k----1 cos(« - b) — cos(n + b) A) 1 B)2 C)-l/2 D) 1/4 E) 1/2 24.- Reducir: M = senz38°-sen28o cosz38<’-sen28<’ k 7 .tan 44° A) B) 42 D)1 E)^ Identidades Trigonométricas de Arcos Compuestos
25.- Calcular el valor aproximado de: D) sec 19°. sec 33‘ 2senl0°+2-j3cosl0°+3cos70o M=-----------5---------5-------- sen255°-sen~18° A)| B)^ C)¿ D)*f E)^ 26.- Simplificar: (taño + tan¿).(cota + cot¿>) W — (taño - cota) + (taní> — cotí>) A) tan a B) tan b C) tan(a + b) D) tan(a - b) E) -tan(a + b) 27.- Si: tan x = cosfrz + b)\ tan y = cos(a - b), 2cot(jc+ y).cosacosb calcular: W =---------5-----7----- sen a + sen b A) 1 B)0 C)0,5 D)2 E)2,5 28,-Si: tan x + tan y = m. sen Cx + y) . .(1) tany+tanz = H. sen(y + z) ...(2) tanz + tanx = p. sen(z + jr) ...(3) Determinar: M = eos x. eos y. eos z A) (mnp) B)mp C)mnp D) (mnp)112 E) tnn 29 .- Si cot 76° = a [cot 38° - cot 52o], entonces el valor de “a” es : A) 1 B) 1/2 C)0 D)2 E)l/4 30 .- Exprese W como un producto, donde: W=tan 19°+tan 33°+tan 38o- tan 19° tan 33°. tan 38° A) esc 19° ese 33°. esc 38° B) esc 33°.csc 38° C) sec 33°. sec 38° E) sec 19°.sec 33°. sec 38° 31 .- En una circunferencia, con centro en el ori- gen del sistema y de radio «z»; se cumple que: xsen0+ycos0 = z Determine: M = -Jl • sen (0+^) A)f C)i^ E)i±2 32 .- Sabiendo que: sen(x-y) sen(y+z) --------- + — --------= 2 sen (x + z) cosxcosy cosy.cosz ’ Calcular: M = eos x. eos z A)0 B)1 C)l/3 D) 1/2 E) 1/4 SITUACIONES GRÁFICAS 33 .- De la figura mostrada, calcule “x” Si:BD=3,ED = 5,CE=4 34 .- De la figura mostrada, determinar: “tan 0”, si: AD = BD = EC = 1 y BE = 2 A) 1 B)l/3 C) 1/4 D)l/2 E) 1/6 Problemas de Trigonometría y cómo resolverlos fcj RACSO «OiTCltl
35.- El triángulo ABC es rectángulo isósceles. Calcular "tan 0”, Si: AM = MN = NB. A) 4/3 B)7/3 C)3/4 D)3/7 E)4/5 A) 1/11 B12/11 Qll/3 D)3/ll E)ll/2 39.- En la figura mostrada se tienen tres dis- cos tangentes exteriormente de radios r, = lu, r-, = 9 m, r3 — 4 u. Calcular: sen 0. 36.- Si el triángulo ABC es isósceles (AB = BC). Determine el máximo valor de "0” si: AM=MB. A) 20° B)45“ C)25° D)40° E)30° a\ 25 o- 7 ™ 61 p. 49 g» 63 A) 24 B) 65 65 J 65 ’ 65 37.- De la figura mostrada, PQRS es un rectán- gulo, PT = TQ = 5QU = | UR, ni Z STU = 0. Determine, tan 0. A)-4/3 B)-10/9 Q-2/3 D)-5/9 E)-20/9 40.- En la figura mostrada, AB = 2, BC = 6, CD - , ni Z CED = a. Calcular: cot a. 41.- En la figura mostrada, ABCD es un cua- drado BE= |EF.BF = FC,mZEAF=0. Calcular: sec 0. B E F C 38.- En la figura mostrada, ABCD es un cua- drado, BC es el diámetro de la semicircunferen- cia, OE es el diámetro de la circunferencia ins- crita en la semicircunferencia, ni Z BDF = a. Determinar: cot a A D Identidades Trigonométricas de Arcos Compuestos T9
42 .- En la figura mostrada, BC = 1. CD = 2, DE = 3. m Z BAC = m Z DAE; m ¿ ABE = 90°. Calcule: AB. A) 1 B)2^ C)L5 D)2 E)3 43 .- En la figura mostrada, BC = 1, CD = 2, DE = 3,0=/nZCAD=»iZAED,/nZ ABE=90°. Calcular: sen 0. A) B) C) D) E) IDENT. TRIG. DE SUMA DE 3 ARCOS 44 .- Sabiendo que: a+y + z=90°. Calcular: cos(a-j) cos(y-z) cos(a-z) M ------------ + ----------+ ------------ cosx.cosy cosy.cosz cosx.cosz A) 1 B)2 C)3 D)3,5 E)4 45 .-Si: A + B + C = 90° Calcule: W = tan A.tan B + tan B.tan C + tan A.tan C A)0 B)0.5 C)1 D)2 E)3 46 .- Si:A + B+ C= l 80°. Calcular: W = tan A + tan B + tan C - tan A.tanB.tan C A)0 B)0,5 C)1 D)1.5 E)2 47 .- Si: a+y + z — 7t/2 cot a - tan y—m, col y - tan z = n, cot z - tan x—p Determinar: M -- tn tan a + n tan y + p tan z A) 1 B)2 Q3 D)4 E)/n 48 .-Si: a + v + z= 180°, además: sen a + eos y. eos z = 0. Calcular. M = tan a . tan y. tan z, en termino de “tan a” A) tan a B) 2 tan a: C) -tan a D)tanA:+l E)tanA--l 49 .- Si: tan a — 2; tan y=4 ; tan z=3, entonces al calcular: W =-------------- se obtiene. cos(a - y + z) A)0 B)1 Q2 D)-2 E)-l 50 .- En un A ABC, si tan A + tan B = k-, tan A + tan C = 1; tan B + tan C = m-, k, 1 y m G R, entonces al calcular sec A.sec B.sec C se ob- tiene: A) -^-7 B)A/m C) . fd,m m + l ’ ' k+l+tn 0,5 klm 2klm k+l+m k + l + m 51 .- Si: A + B + C = 71, calcular: sen 2(B + C)—cos2B - cos2C cos(B + C).cosB.cosC A) 3 B)2 Q1 D)-l E)-2 T9 100 Problemas de Trigonometría y cómo resolverlos Í^RACSO IDITOUI
Son técnicas que se realiza para obtener los valores de las R.T. de cualquier ángulo por otros equivalentes pero cuyos ángulos se ubiquen en el primer cuadrante. Para el estudio de reducción al primer cuadrante, se presentan los siguientes casos: 10.1. CASO (I): ÁNGULOS POSITIVOS MENORES QUE 360° QJ PROPIEDADES R.T. (90° ó 270° ± a) = ± COR.T (a) R.T. (180° + a) = ± R.T (a) R.T. (360° - a) = + R.T (a) Tener en cuenta que los signos ± del segundo miembro se eligen de acuerdo al cuadrante donde se encuentre el ángulo que se está reduciendo y la función trigonométrica a este se le aplique. Considerar a ángulo agudo con el fin de ubicar con facilidad el cuadrante. 10.2. CASO (11): ÁNGULOS MAYORES QUE 360°____________________________ Es suficiente con dividir el ángulo que se desea reducir entre 360°. A continuación se toma la misma función trigonométrica al residuo, así: R.T.(n x 36(1° ± a) = R.T. ± (a); n e Z 10.3. CASO (111): ÁNGULOS NEGATIVOS fP] Es suficiente convertir el ángulo negativo en positivo siguiendo los teoremas siguientes: Supongamos que: a > 0 => - a < 0 sen (-a) = -sen a eos (-a) = eos a tan (-a) = -tan a cot (-a) = -cot a sec (-a) = sec a esc (-a) = -esc a Reducción al Primer Cuadrante T10 101
10.4. CUANDO EL ANGULO SE EXPRESA EN RADIANES La reducción se procede aplicando las reglas siguientes: R.T. (*Tt ± ct) = ± R.T. (a) R.T. i(2?t + l)a/2 ± a] = ± COR.T(a) V *e Z En general: R.T.(2é7t ± a) = R.T.(± a) Recuerde lo siguiente (V k e Z) 10.5. ÁNGULOS RELACIONADOS ENTRE SÍ_______________________________ 10.5A Ángulos Complementarios Si: x + y = rt/2 , se cumple; R.T.(x) = co-R.T. (y) 10.5B Ángulos Suplementarios Si: x + y = n , se cumple: R.T.(x) = ± R.T. (y) RACSO 1DI7O1II T10 p!021 Problemas de Trigonometría y cómo resolverlos
PROB. 1 Calcular N = RESOLUCIÓN eos300°+eos 120°+ sen 150° sen 330°+ eos240°+ tan 135° ********************************************* Aplicando reducción al primer cuadrante: K, cos(360o-60°) + cos(180°- 60°) + sen (90°+ 60°) K, cos60°-cos60°+cos60° sen(360°-300) + cos(1800 + 600) + tan(900+ 45°) =» 1X1 - _sen30o_cos60o_cot45o Luego de reemplazar los valores notables, tendremos: 1 _L N = --------- => N = -2- N = - — -2 " 4 2 2 PROB. 2 sen750°+cosl 500°+tanl 665° Calcular: E— sen(-150°)-cos(-1200) + cot(-7650) RESOLUQÓN *****♦***♦*♦♦♦****♦♦**♦♦**♦*♦♦*♦♦♦**♦*♦*♦*♦** Reduciendo al primer cuadrante: sen(2x360°+30°) +cos(4x360°+60°) + tan(4x360°+225°) h- -senl50°-cosl20°-cot765° Al aplicar los casos II y 111, tendremos: E _________sen300+cos600+tan45°_______ - sen30°-(-cos60o) - cot(2x 360+ 45°) 2 E = ~ .-. E = -2 1 + 1 + 1 2 2 -1+1-1 2 2 finalmente: PROB. 3 Calcule: S = eos 10o + eos 20° + eos 30° + ... + eos 160° + eos 170° RESOLUQÓN E = Se observa que los ángulos que están equidistantes suman 180° y se sabe que si: Reducción al Primer Cuadrante
Luego: x+y = 180° cosx = -eos y S = cos/T0° + cq/20° + eos30° + ...-CQ8z0°-cos-H0o S = 0 PROB. 4 sen(57i - x)+eos + x)+sen(87i - x) Reducir: R = ----------------X——4----------------- tan(457i + x) - cotí — x I+tan(487i+x) RESOLUCIÓN Escribiendo apropiadamente «R», tendremos: sen(47t + ti - x)+eos ^4ti - ^+x j+ sen(8?i - x) tan(447t+ti+x)—cot (12ti+^ - x j+tan(48rt+x) Al aplicar el caso 111, en el paso anterior, nos queda: sen( jt - x) + eos] - ~ + x 1+ sen(-x) R =-------------------------4--------- tan(7i + x) - cotí y - x 1+ tanx Finalmente aplicamos el primer caso, obteniendo: tanx - tanx + tanx senx tanx = senx. cosx senx R = eos jr ESTRATEGIAS DE RESOLUCIÓN R = R 1) Cuando un ángulo no es agudo, se recomienda aplicar reducción al primer cuadrante para determinar R.T. de ángulos agudos. 2) Si observamos que dos ángulos suman 180®, significa que la suma de sus cosenos es igual a cero o bien sus senos son iguales. 3) Cuando un ángulo se expresa en radianes, se busca un valor par de n, de tal manera que se puede aplicar el siguiente teorema: R.T.(2 kit±a) - R.T.(±a) ; Vke Z Problemas de Trigonometría y cómo resolverlos RACSO DITOtlI
Enunciados de Problemas con Resolución CASOS DE REDUCCIÓN 01.-Si: csc (90o- A) - a eos A cot (90o- A) = sen (90o- A) El valor de a es: A) cot A B) -cot A C) tan A D) -tan A E) sec A 02.-Calcular: eos Io+eos 2o+eos 3°+...+eos 179° + cos 180° A)-2 B)-l C) 1 D)2 E)3 03.- El valor de la siguiente expresión: “(é) c“® Es igual a: A)0 B)1 Q-l D)2 E)-2 04.- Hallar el signo de las expresiones trigonométricas, en el orden dado: 52n 2571 32ti 22ti sen —y . eos —y ; sen -y . cot —y ; /-205ti\ 73ti sen O/ cot 10 A) (+) (+) (-) B) (-) (+) (-) C) (-) (+) (+) D) (-)(-)(+) E) (+)(-)(+) 05.- Sea R = eos 810° + cot 425° S = (sen 450°) (tan 785°) El valor del producto R.S es: A) 1 + cot 65° B) 1 + tan 65° C) (tan 65o)2 D) cot 75° E)1 06.- Reducir: A - B siendo: A = sen (ll^ + xj-sen ^33^+jj B = eos (55^ + a) - eos ^77+ y j A) sen (a + y) B) eos (a + y) C) -eos (a + j) D) -eos (a - _v) E) -sen (a - y) 07.-Simplifican W = sen(it+a).cos +a)+cos(a - ?t).sen (y+a ) csc(tc+A).sec(y + A j - tan ( A - j.cot - A C) tan2A A) tan a D) cot2A B)cotA E) -tan2A 08.-Simplificar: tan (~y - «j.sec(37t - a).sen p-y - a W _ cos(—40n+ct) A) sen a B)coscc C)tancc D) cot a csc a 09.-Simplificar: sen(n—A).cot (A— y j.cos( a—36tt) tan(l 3tt + A)tan (a+iy j.cos2 (~y^ + x j Reducción al Primer Cuadrante
A) O B)O,5 Qtf D)-l E)1 15.- Reducir: 10.- Simplificar cot (1995 n - 0).cos 1239-^ + 6 W =-----------------)--í—? sec(0 - 804n).senl0 -161^1 A) sen 0 B) tan 6 C) cot 6 sen( 180°- a).cos(a-90°).tan(1260°+ a) ~ cos(540°- cc).tan(360°+ a)sen(450°+a) A) tan a B) cot a C) cot2 a D) -cot2 a E) -tan2cc D) -eos 6 E) eos 6 11 .-Si: tan ^37^ + a j .cot( 1 75tc - a).sen(809n + a) cot(72ít — a).sen ^91 —a j .sec + a j 2 9 además: ccg IIC, Calcule: M = 2-j5 cotcc+csccc. A) 30 B)31 Q-30 D)28 E)-31 12 .-Simplificar: cov(a-27n) very(-10n+jr) W„(-I + 23|) ™.(-a+I3í) A)1 B)2 C)-2 D)-l E) 0 13 .- Si: n G Z, calcular el valor de: M = senpm-(-l)" .esc pin + (-l)n.^j A)0 B)1 C)0.5 D)2 E)-l 14 .- Si “a” e “y" son complementarios. 6 G ( —7t ; - n!2) y se cumple: t sen( a + 2 y ).tan(2x + 3 y) cos(2a + y).tan(4jr+ 3y) Determinar el valor de “6”: Ai —— Rt —ct ru A}- 3 B)- 6 C)- 4 D)- 1() E)-n 16.-Si: k.senp5^-6j .cos(77-^ + 6j = 5 2 Calcular: W = (sen 6 + eos 6) en términos de k. k + 1 A)k B)Á“ C)-k D)k-1 E)-*- 17.- Calcule: W = cos(159^) +sec^242yj -Senp25^j A)0 B)1 C)2 D)-l E)-2 18 g. cos(3000°) - cos(2000°) _ cos(300°)—cos(200°) Calcular: W = términos de k. cos(3000°) + cos(2000°) cos(300°) + cos(200°) A)k B) -A C)*2 D)jy E) | 19.- Si los ángulos internos de un triángulo ABC están en progresión aritmética (A < B < C) Reducir: sen( A + 3B + 2C) cos(B + 2A + 3C) sen(B - C) + cos(B - C) A)0,5 B)1 Q2 D)l,5 E)0 20.- Determinar el valor de: í sen420°.cos240°.tan405° A2 M= [sen210°.cos225°.tan570° J 3 5 9 A)- B)1 C)0 D)| E)^ Problemas de Trigonometría y cómo resolverlos TafcF.ACSO
21.- Simplificar: 27.- Calcular el valor de: sen(2540°) + 2cos3(1910°) W = cos(2680°) + 2sen ’(2630°) A) tan 10o B) tan 30° C) cot 10o D) cot 20° E) tan 20° n 3ti 5tt 7n M = eos — + eos— + eos — + eos — 000 o A) 0,5 B)1 C)l,5 D)-0,5 E)0 28.-Si:A+y= 180° a y + z = 270° CASOS PARTICULARES DE REDUCCIÓN 22.-Calcular el valor de: eos 10° + eos 30° + eos 50° +... + eos 170° A) 1/2 B)0 0^3/2 D) 1 E)2 23.-SÍ: A + B + C= 180°. Calcular: senA + cosy seny — senz +(tanx- tanyXcoty + tan z) A) 1 B)2 O-l D)0 E)-3 29.- Simplificar la expresión: Hallar: -—r-----73—-— tanA + tanB + tanC A) sen A.sen B.sen C D) cot A.cot B.cot C B)cos A.cosB.cosC E)sec A.secB.secC C) tan A.tan B.tan C 24 .- Qué relación existe entre a y b; sabiendo que: /2«-3fe\ /6n+3o-2¿\ „ tan (—g— ) + cot (---4-----) = 0 A) 1/2 B) 1/3 C) 1/4 D) 1/5 E) 1/6 25 .- En un triángulo ABC. Simplificar: sen( A + B) M =-------z + tan(A + B + 2C). cot (A + B) senC A)0.5 B)1 C)-l D)l,5 E)0 26 .- Si: a+y = 7t, simplificar 2 tan -í n - M _ sen a _______2_ 3 sen 2a “ sen y y sen 2y A)0,5 B)1 Q1.5 D)-0.5 E)0 tan(230°+A) + tan(50°+A) M cot(40°-A) A)0 B)0,5 C)1 D)l,5 E)2 2m + 1 ni + 2 30 .- Si: csc a = “-7 ; csc B =-7 2m -1 m — 1 Determinar el valor de “m” que hace que a y sean suplementarios A)| B)^ C)^ D)0 E)| 31 .- Reducir: sen 120°+sen 140°+sen 160°+...+sen260° M=-------------------------------------- cos20°+cos300+cos40°+... + cosl70° A)0 B)0.5 01.5 D)2 E) 1 32 .- Si: sen a = eos (P + 6), «a» y «P + 6» son ángulos agudos tan(cc+P+26) Reducir: M= cot(2a + 2p + 36) A)0 B)1 C)2 D)-2 E)-l 33 .- ¿En qué tipo de triángulo ABC se cumple: sec(A + 2B + 2C) = csc(2A + 2C + 3B) ? Reducción al Primer Cuadrante
A) escaleno B) isósceles C) equilátero D) obtusángulo E) rectángulo 34.- Si A, B y C son los ángulos de un triángu- lo. Simplificar: cos(A + B) cos(B + C) cos( A + C) M =------------+------------+------------ cosC cosA cosB AJO BJ1 Q-2 DJ-1 EJ-3 AJO BJ0,5 Q-l D)2 EJ1 SITUACIONES GRÁFICAS 35.- Determinar BM de la figura en términos 38.- De la figura mostrada, calcule: de R y 0: A)R BJRsenO CJR eos 6 D) R sec 6 EJRtanO W = a tan 6 - 2b. 36.- De la figura mostrada, calcular tan 6. A)^ BJ1 C)2yÍ5 D)3>/5 E) AJ a BJ¿ C)-a D)-ab E)-b 39.- De la figura mostrada, determinar: 13 sen a. AJI BJ3 Q4 DJ2 EJ5 *4racso niDitoiii T10 108 Problemas de Trigonometría y cómo resolverlos
En el capítulo anterior se estudiaron las identidades de las razones trigonométricas de la suma y diferencia de dos arcos, así por ejemplo: sen (a + 0) = sen a eos 6 + eos a sen 0 A partir de aquí, haciendo el cambio de 6 por a es decir (0 = a), se obtienen las razones de los arcos dobles, así: sen(a + 6) = sen(a + a) = sen a eos a + eos a sen a => sen 2a = 2 sen a eos a Otra forma interesante de deducir las relaciones de los arcos dobles es aplicando números complejos (fórmula de Abraham de Moivre 1 667-1 754), que se verá en el Cap. 22: (eos a + i sen a)n = eos na + i sen na Si: n = 2 => (eos a + i sen a)2 = eos 2a + i sen 2a Desarrollando el binomio, e igualando las partes reales y partes imaginarias se obtienen: 2 2 sen 2a = 2 sen a eos a a eos 2a = eos a - eos a 11.1. RELACIONES FUNDAMENTALES_______________________________________________ 2 2 2 íTft n fx sen 2a = 2 sen a.cos a eos 2a = eos a - sen a tan 2a = --------------------g— 1 - tan a 11.2. RELACIONES AUXILIARES Se obtienen a partir de las razones fundamentales con la ayuda de las identidades trigonométricas, así: 2 2 sen a = 1 - eos 2a 2 2 eos a = 1 + eos 2a oCl 1 mIX ““ 1 + tan'a , . 2 •»„ _ 1-tan a eos 2a = ------=— 1 + tan a Identidades Trigonométricas del Arco Doble
tan a + cot a = 2 csc 2 a cot a - tan a = 2 cot 2a 4 4 3 1 sen a + eos a = -r- + -r- eos 4a 4 4 8 sen4 a = 3 - 4 eos 2a + eos 4a 6 6 5 3 sen a + eos a = -g + -g- eos 4a 8 eos4 a = 3 + 4 eos 2a + eos 4a PROB. 1 Sabiendo que tanx = 1/2, determina: a) sen ¿x b) eos 2x c) tan 2x RESOLUCIÓN Utilizando el triángulo adjunto, se tiene: 3 eos 2x = -f- = •= 5 5 4 c) tan 2x = -2^-* = 1-tan x PROB. 2 tan 2x = Determine el valor de x en la figura: _______o.. 2tanx iz] a) sen ¿x - -----5— = —i 1+tan x i.f_l] l2 J sen 2x = —U- = 4- 1+\ b) eos 2x =~ l talY 1+tan x RESOLUCIÓN De la figura propuesta se tiene: . Q 6 tan 2a = — x 2 tan a , • 2 1-tan a 6. x tana , . 2 1-tan a 3 2 — , pero: tan a = — A ACSO PBDITOBtt T11 110 Problemas de Trigonometría y cómo resolverlos
Luego: 2x1 2 * = 2 x(x2 -4) x 2x2 = 3x2-12 2x2 x2-4 = 3 12 = x2 Finalmente: x = 2^3 PROB. 3 Calcular el máximo valor de: 2 2 E = sen x + 6 eos x + 12 sen x eos x RESOLUCIÓN Utilizando: 2 sen2x = 1 - eos 2x ... (*) 2 cos2x = 1 + eos 2x... (*) 2 sen x eos x = sen 2x ... (*) Se tiene, luego de multiplicar «2» a cada miembro: 2E = 2 sen2x + 6.2 cos2x + 12.2 senx cosx => 2E = 1 - eos 2x + 6(1 + eos 2x) + 12 (sen 2x) 2E = 1 - eos 2x + 6 + 6 eos 2x + 12 sen 2x => 2E = 12 sen 2x + 5 eos 2x + 7 Pero: - J(12)2 +(5)2< 12 sen2x + 5cos 2x< J(12)2+(5)2 Puesto que: I 2 2" / ? 2~ -va +b <a senx±b cosx< va +b => -13 < 12 sen 2x + 5 eos 2x< 13 => -13 + 7 < 12sen 2x+5 cos2x+7 <13+7 2E -6<2E<20=> ^<£<10 mínimo máximo = I» ESTRATEGIAS DE RESOLUCIÓN 1) Es recomendable expresar los ángulos doble ¡ en función del ángulo simple. 2) Si se tiene el valor de la tangente del ángulo simple, y se desea calcular los valores de las razones trigonométricas de sus respectivos ángulos dobles, se utiliza el triángulo del ángulo doble. 3 ) Cuando se tiene senos o cosenos a la potencia cuadrada o potencia cuarta (sen2 x ; eos x; sen’ x; eos' x), se debe degradar (bajar la potencia a la unidad), utilizando las siguientes identidades. 2 sen x = I - eos 2x ; 2 co2 x = l + eos 2x 8 sen4 x = 3 -4 eos 2x + eos 4x 8 eos4 x = 3 + 4 eos 2x + eos 4x Identidades Trigonométricas del Arco Doble
Enunciados de Problemas con Resolución RELACIONES FUNDAMENTALES 01.- Simplificar: W = 3 + eos 4x - 8 sen4x A) eos x B) eos 2x C) 2cos 2x D) 3cos 2x E) 4 eos 2x 02.- Reducir: M = 2(cos4x - sen4x)“ - 1 A) eos 4x B) eos 2x C) eos2 2x D) eos2 4x E) 2 eos 4a 03.- Reducir: sen3x + cos3A i M =-------------- + 77 sen 2x senx+cosa A) 0 B) 1 C) 0.5 D) -1 E) 2 1 + cos2a 04.- Simplificar: VV = ; — l-cos4a 1 2 A) . csc a B) 4 sec2 a C) 4 cot" a H- H- D) 7.sen2a H- E)| 05.- Reducir: 2 4 2 4 M = eos x - sen x - eos x.sen x + sen x.cos x A)eos x B) 2 eos a C) eos 2a D) 2 eos 2x E)-eos 2x 06.- Reducir: senx cosx M=------------— - -----------5— secx(l + tan‘a) cscx(1 + cot a) A) eos a B) -eos a C) -eos 2x D) eos 2a E) eos" 2x l + tan2í^-2aj 07.- Simplificar: W =-----------7-----r A) sen 2a B) sen 4a C) csc 2a D) sen"a E) csc 4a 08.- Simplificar: .. 2 tan3x 2 - kit . _ „ M =------5— - sec x. sen 2x, x * sec'a - A) sen x B) -sen 2x C) sen 2a D) eos 2x E) -eos 2x 09.- Reducir: tanA cotx M=----------5-^5- +-------3— (1 + tan ‘a)‘ (1 + cot ~x)~ A) sen 2x B) sen 2x C) sen 2x D) sen" 2x E) . sen 2xe CAMBIAR SOLUCION 10Í Simplificar: W = cot 7o - 2 cot 14%> A) tan 14° B) cot 14° C) cot 7o D) tan 7o E) 2 tan 7o 11.- Si: sen x < eos x, simplificar: W = eos 2x - (sen x + eos a). Vi — sen2x T11 112 Problemas de Trigonometría y cómo resolverlos JÉRACSO WBDITOlll
A) 1 B) -1 C) 2 D) -2 E) 0 12.- Escribir la siguiente expresión en térmi- nos de eos 0. M = tan-'- ,0 7+ - 2sen" — senO 2 k > 20 .eos A) cosO B) ^cosO C) ^cosO D) ^cos“t E) c^O 13.- Si: W = 1 + sen x + eos x + tan x, enton- ces una expresión equivalente de factores para W será: A) 2 V2 eos (7^) .sec x B)2-j2 cosí— ].csc x \2/ A) 0 B) 1 C) 2 D) 3 E) 4 16 .- Si: --7—v - A.cosn(^J, calcule el l-cos(f) 'S/ valor de : A) 2 B) 1 C)0.5 D) 1,5 E)4 17 .- Si: 2 AsenZr + Bcos2x + C=p.tan x+^tanx+r=O 2A halle el equivalente de: W - ——— B + C A) p!r B) q!r C) r!q D) ríp E) p.q 18 .- Si se sabe que: eos 2x — 1 - 8 cos~-£ + 8 eos4-? 2 4 Calcular: M = eos 2x + 3 PROBLEMAS CONDICIONALES 14.- Si: 4 4 7sen ct + eos ct = ni + n eos 2ct + p eos 4ct Calcule: m -n- p A) 1 B) 3 C) 5 D) 0 E) 2 A) 0 B) 0,5 C) 1 D) 3 E) 2 19.- Si: 4 sen a - 3 J2 eos a = 5, entonces calcular: W = eos 2a - 12 -J2 sen 2a A) 8 B)6 C)4 D) 2 E) 0,5 20.- Si: eos x - sen j secx l'= Calcular: esc 4x A) ni+1 B> 2m-l C) D) ’ 2171 + 1 E) ",2 ' 2/n 1 15.- Si: (1 + sec 2x)( 1 + sec 4x)(l + sec 8x) = A. taníBx). cot(Cx) Calcular: -------; siendo: B > 0 a C > 0 A + C 1 21.- Si: sen 2x = ~t= , calcular: V3 W = sen6x + eos6x Identidades Trigonométricas del Arco Doble
3 3 1 13 A) 2 ») 4 C) 2 D)| E)j cosx senx 22.- Si: -=------, a qué es igual: m n E = m eos 2x + n sen 2x A)h B)-h C)-/n D)/n2 E)m l + sen2a 23.- Si: k = ----— , calcular: 1—cos2a 2tan2a W=-----------T (l + tana)‘ K)k B)-Á C)-| I))pr E)| 24.- Si: eos x. eos y ~ sen a sen x . sen y = eos a Calcular: sen (x- y) en términos de a. A) sen a B) eos a C) sen 2a D) eos 2a E) -sen 2a 25.- Si: sen f—+xl = ~, calcular: l17 ) 3 W = cosÍ2x-15— l 17 A)| B)-j C)-| D) l E)| í 7t xA 26.- Si: eos — + — I = k. calcular ^4 5j f 2xA W = senl — I A)Á2 B)Jl2-l C)*2+l I))2F E)-2A2+1 27.- Si: tan 2x = 8 cos'x - cot x Calcule: M = 2 sen 4x - 1 A) 0,5 B)1 C)l,5 D)2 E)0 28.-Si. tan -x) - tan (3n + x) = 2k, calcular: W = sec 4x - eos 4x Al k B> o » A) ¿2+l rrx 412 D) *2-l 29.- Si: sec2a - sec20 = 2 tana Calcular: M =-----— sen2a tanp sen2p A)0 B)0,5 C)l,5 D)2 E) 1 2 30.- Si se cumple: 2 - sec x — 3 tan x Calcular: cot 4x 13 13 5 12 5 A> 12 013 D> 13 31 .- Si: 14x=ti , entonces al calcular: W 1 2-j2cosx • 1 1 + -J2senx sen2 x 1—-J2senx Se obtiene: A)-72 B)2-72 C)-2-72 D)3-72 E)4-72 32 .- Calcular el valor aproximado de: Ícos22° sen22°V sen8° cos8° J A) 1578 B) 2345 C) 3497 D) 2453 E) 1875 T11 114 Problemas de Trigonometría y cómo resolverlos RACSO DITOlll
33 .- Calcule el valor de la expresión: rt 7»t 5»t llrt W - lan — - lan — + tan— - tan — A) 0 B) 0,5 C) 2 D) 4 E) 8 34 .- Calcular el valor de la expresión: 4 n 4 3n 1 3rr W = sen — + sen — + —eos— + 16 16 2 8 1 n 2COS8 3 1 3 A) 3 B) C)| D) 1 E) J 39.- ¿Cuál es la variación de la expresión: 1 —cos4x ? 1 - cos2x A) [0 ; 2> B) [0 ;3> C) <0 ; 2] D) <0 ; 4] E) [0 ; 4> 40 .- ¿Cuál es la variación de la expresión: W = tan 0 - tan 20 + tan“0.tan 20, si 0 e A)<0;l> B)<-l;0> C)f-l;O] 35 .- Calcular el valor de la expresión: D)[-l;l] E)<-1;1> M = (l+cos^)(l+cos^)(l+cos^)(l+cos^j IDENTIDADES AUXILIARES A)1 1 B) 2 c>i D>i E>iK 41.- Si: csc a = 4, calcular 36 .- Calcular el valor de la expresión: 'J1 + COS40'1' sec45° . sec 20° M = A) 0 B) 0,5 C) 2 D) 3 E) 1 37 .- Calcular el valor de la expresión: M = eos 5o. sen 5o -(1 + sen 40°)( 1 - sen 40°) A)0 B) | C) 1 D) -l E)-| VARIACIÓN DE EXPRESIONES 38 .- Calcula el valor máximo de la expresión: ... 5 5 W = eos x . sen x - sen x. eos x A) j B) | C) | D) E) J „n<.(«+S)+cos«(|+S)_5 A) 0 B) 1 C) | D) J E) | 42 .- Si: tan + tan (^8) = i< l 2Í4n\ calcular: W = ^ . sen I 9 I A) * B) | C) f D) E) 43 .- Si: tan2x + cot2x = ni, a G ( 0 ; 7t/2) Calcular: -Jni + 2 .sen 2a A) 0 B) 0,5 C) 1 D) 1.5 E)2 Identidades Trigonométricas del Arco Doble
,A co/x-sen8x 44 .- Si:--------= A + B eos 4x cos2x x*(2¿+l)v 4 A) 1 B) 2 C) 3 D) 4 E) 6 48.- Si los lados de un rectángulo son ay b (a > b), entonces al calcular la tangente del ángulo agudo que forman sus diagonales se obtiene: Calcular: “A + B” A) 0 B) 0,5 45.- Si se cumple: Calcular: C)-l D)-0,5 cos4x—sen4x s s~ = m eos x—sen x M = 3 - eos 4x E)1 1 2 3 4 A)m B)~ m C) — D) — E) m m m Al a lab lab A) b B) a+b C) a2+b‘ ab lab 2 .2 a -b E) a2-fc2 49.- Si en un triángulo rectángulo sus catetos tienen por medida sen 20° y 1 + eos 20°, en- tonces la medida de sus ángulos agudos se- rán: 46.- En la siguiente identidad halle: A - B 8(sen6x + cos6x) — A + B eos 4x A)0 B)0,5 C) 1 D) 1,5 E)2 SITUACIONES GRÁFICAS 47.- En la figura mostrada: AD = Jí , DC = J3 , m Z BCE = m 7 ECD, m Z EAD = 45°, m Z ADB = 90° y m Z ABD = x. Calcular: cot x. A) 20° y 70° B) 30° y 60° C) 40° y 50° D) 45° y 45° E) 10° y 80° 50.- En la figura mostrada, AB = 3 cm, CD = 7 cm, m Z BAC = 2a, m 7 ADB = a, m Z BAD = m Z BCD = 90°. Calcular: BD. A) 4 cm B) 5 cm C) 6 cm D) 7 cm E) 9 cm Problemas de Trigonometría y cómo resolverlos RACSO P1DITOBBI
Utilizando las identidades de los arcos dobles tendremos: 2sen26 = 1 - eos 26 ; 2cos26 = 1 + eos 26 De tal manera que si hacemos: 26 = a, entonces: o 2 a , o 2 a . , 2 sen = 1 - eos a ; 2 eos 2 = + cos a (X ex Si se despeja sen a cos se obtienen las relaciones de los arcos mitad. 12.1. RELACIONES FUNDAMENTALES sen f 11 - cosa 11 2 a _ + /I + eos 2 tan ~ 1-cosa 1+cosa cot = ± J 1+cos a 1 - eos a ex El signo ± depende del cuadrante donde se encuentre y la función que a este se le aplique. 12.2. FÓRMULAS RACIONALIZADAS [QJ ex ex sen^ eos— ex 2 ex 2 Teniendo en cuenta que tan -y = ---- y cot -y =---— , se obtienen: cosy z seny tan = esc a - cot a cot = esc a + cot a Identidades Trigonométricas del Arco Mitad
PROB. 1 Calcular: sen -g- y eos RESOLUCIÓN *) sen -g- = sen l-cos^ 4 PROB. 2 1 3ti . Si: eos 2 9 ~2~ <x <^71^ calcule: a) sen y b) eos y c) tan RESOLUCIÓN 71 Escogemos el signo (+) dado que -=• g 1 C o sen 8 - Escogemos el signo (+) dado que -x e IC O cos 8 ” De los resultados anteriores, deducimos el triángulo notable mostrado en la figura. Primero tenemos que ubicar el cuadrante donde se encuentra x/2. 3rt 2 <x < 2n . 3?t 2L - „ . * nr 4 2 n 2eI1C fi----- IÑf x - /1-cosx _ J_2 a) sen 2 _ V 2 12 T12 118 Problemas de Trigonometría y cómo resolverlos RACSO DITOlll
c) tan y cosx + COSX 4 . Tí Tí 71 [ñ i d) tan g- = csc - cot = V2 - 1 PROB. 3 tan y Simplifica: PROB. 2 E = cot-g 4-csc-g +csc jg + csc gg +cscg^ Calcule: RESOLUCIÓN a)tan 15° b) cot 22° 30’ Utilizando la fórmula racionalizada tendremos: c) tan 18° 30’ d) tan 7t/8 cot 4 + CSC77 + CSC T7 + CSC 77 + CSC 77 8 8 16 áz b4 RESOLUCIÓN ro,Í6tC“Í6 Se sabe que: tan y = cscx-cotx “•^+CSC32 Luego: cot = esc x + cot x cot -£7 + csc ~ 64 64 a) tan 15° = csc 30° - cot 30° = 2 - -J3 cot-~- 128 b) cot 22° 30’ = csc 45° + cot 45° = -J2 + 1 Finalmente: E = cot c) tan 18° 30’ = csc 37° - col 37° = ESTRATEGIAS DE RESOLUCIÓN 1) Para aplicar las fórmulas del ángulo mitad, primero tenemos que ubicar el cuadrante al cual pertenece el ángulo mitad, para que de esa manera podamos asignar su signo. 2) En ángulos mitad es recomendable utilizar las fórmulas racionalizadas para evitamos de los radicales, estas fórmulas racionalizadas son: x tan 2 = csc x - cot x x cot 2 = esc x + cot x Identidades Trigonométricas del Arco Mitad T12 119
Enunciados de Problemas con Resolución RELACIONES FUNDAMENTALES 1 Ol.-Si: cosx = — ; xG IC, O A) a B)fc C)^ D)afe E) 06.- Dada la siguiente identidad: calcular: 4 cos A)0 B)0,5 C)1 D)l,5 E)3 02.- Reducir: lsecx-1 lsecx+1 M= J-------- + J----7 Vsecx+1 isecx-1 A) sec x B) esc x C) 2 sec x D)senx E)2cscx 03.- Reducir: x „ ? x cot—.cosx+2cos—.tanx M =----2------------------ secx.cot— 2 A)1 B)0,5 Q1.5 D)2 E)2,5 1 04.-Si: csc2x-cot2x= — .calcular: W = esc 4x + cot 4x 2 3 3 4 1 A)^ B)| D)| E)| lab 05.-Si: scnx = ——-j ; a> b ; xG IC, a~ + b calcule: tan bO|i-f 2 (1+senx+cosx) +(1 - senx+cos x)“=W.cot calcula el equivalente de W. A) sen x B) 2 sen x C) 4 sen x D) cos x E) 2 cos x 07.-Si: cotx=0,75 , xG IIIC, x calcular: W = cos 2x - tan — 3 4 7 43 23 A)| B)^ C)¿ D)g E)-§ 08.- Calcular: 2sen .(cos^.tan x - sen^j+ l,parax=30°. A)>/3 B)2>/3 C)f D)^ FÓRMULAS RACIONALIZADAS 09.- Simplificar: W = esc 2x + 2 cotx- 3 tanx A) 5 cot 2x D) cot x 10.- Reducir: B) cot 2x C) 3 cot 2x E) 2 cot 2x tan20°+cot40° M= cot200-cot40° T12 120 Problemas de Trigonometría y cómo resolverlos RACSO DITOBBB
A) 1 B)0,5 C)0,25 D)-0,5 E)-0,25 11.-Simplificar: W = cot - 2 cos2^ . cot x A)cox B)senx C)1 D)0 E)cotx x 2x 12.-Simplificar: M = tan — +2sen — .cotx A) cosx B)cotx C)secx D) cscx E) sen x 13 .- Simplificar: ... . W . o ( cosx l+cosx\ W = tan (2) - tan 2x. A)0 B)0,5 C)1 D)2 E)l,5 1 x cscx---tan— 2 2 14 .-Simplificar: M= -j------------- —tan—+ cotx 2 2 A)0 B)0,5 C)l,5 D)1 E)-l 15 .- Simplificar: fu xY, . tari-------------------L(l + senx) 1 4 2 j W= —k---------2-------- cosx A)0 B)1 Q0.5 D)l,5 E)2 16.- Reducir: x x x x M = CSCX + CSC~ +CSC~ +CSC~ +CSC — Z 4 o lo X X A)cot32 -cotx B)coty^ C)cót D)cot^ E)cot^ 17.-Reducir: . . 2- í 2 1 M = eos 2x - --------- l tan x+cot x J A) eos x B) eos 2x C) eos 3x D) eos 8x E) eos 4x 18.- Simplificar: W = csc 2x + cot 4x + csc 4x 2 2 A) tan x B) tan x C) cot~x D) cot x E) 2 cot x 19.-Simplificar: cot—-tan— • W=-------------— csc2x + cot2x A)0 B)2 Q0.5 D)1 E)l,5 20 .- Calcula: tan 7°3O’ A)V6+^ + a/2 + 2 B)V6 + ^-a/2+2 C)V6-^ + a/2-2 D)Vó->/3-a/2+2 E)>/6 + >/3+V2-2 21 .- Calcule el valor aproximado de: tan(9°) A)>/5- 1+75+2V5 C)>/5 + l-V5 + 2>/5 B)>/5 + l+V5+>/5 D)>/5- l+V5-2^ E)>/5 +1+^5-245 PROBLEMAS CONDICIONALES X 22 .- Si se cumple: tan — + tan ~ = 2 csc ~ 4 o Z X X calcule: M = cos~r +sec~ 4 4 A) 2¿ B)0,5 C)l,5 D)2 E)3 23 .- Calcule el valor de la expresión: cotlCf—tanlO” W=---------------- csc40°+cot40° Identidades Trigonométricas del Arco Mitad T12 121
A) O B)O,5 C)1 D)l,5 E)2 24 .- Si: tan(45° - x) — tn, calcular: E = sec 2x + tan 2x A)m B)-^ C)m~ ^7^2 E)2zn 25.- Si: csc x - cot x = sen 6, sec2 ~+ eos2 6 calcular: M =-------------- coty CscG + 1 A) 0.5 B)2 C)1 D)l,5 E)3 26 .- Determine el valor de: M = (tan 10° + 2 cot 20°)(sec 70° - cot 20°) A) 1 B)0,5 Q1.5 D)2 E)2,5 27 .- Si: sen x + m eos x = m, x calcular: cot — 1? 12 A)— B)m C)m D) E)— m m- ' m 2 28 .- Si la ecuación: x - (2 csc a)x +1=0, calcule una de sus raíces. A) tan y B)seny C)cosy D) sec y E) csc y 29.-Si: eos 8x.cos óx.sec 2x = csc(Ax) - csc (Bx), B calcule: — A A) 2 B)0 C)0,5 D)1 E)l,5 30.- Expresar: tan (^4^) en función de: tan A ) -tan y + sec y D) -tan y - sec B ) tan y + sec y E) 2 (tan y + sec y ) Q ^(tan y + sec y) 0 0 31 .- Si: k sen y = eos y , siendo sen 6 > 0; „ |l+sen6 p = 2 |-= cscO sera y sen20 A) 7íaM-2) b) * + Q k - c1 díJa+a1 ejTTjF SITUACIONES GRÁFICAS 32 .- En la figura mostrada, AB = 2 CD, m Z CAD=in ZCDB = 6, zhZACD=90p. Determi- ne la medida del ángulo “6” A) y rad B)^rad C)^rad ty^rad E) rad O 33 .- Si sen 4x=0,6, siendo 0 < x < -g, calcular tanx A) 1 B) VÍ0 C) VIÓ +3 D) VVÍÓ+3 E) V¡6 -3 T12 122 Problemas de Trigonometría y cómo resolverlos RACSO DITOlll
Al igual que en arco doble o arco mitad, en arco triple también las relaciones fundamentales se obtienen aplicando las identidades de los arcos compuestos o bien con la formula de De Moivre en números complejos así: sen 3a = sen (2a + a) = sen 2a cos a + cos a sen a sen 3a = 2sen a cos a. cos a + (1 - 2 sen a) sen a sen 3a = 2sen a (1 - sen2 a) + sen a - 2 sen3 4a sen 3a = 2sen a - 2sen a + sen a - 2 sen a sen 3a = 3sen a - 4sen a Por complejos: (cos a + i sen a)3 = cos 3a + i sen 3 a Desarrollando el binomio al cubo, luego igualando las partes reales e imaginarias se obtiene: 3 3 eos 3a = 4 cos a - 3 cos a sen 3a = 3 sen a - 4 sen a 13.1. RELACIONES FUNDAMENTALES IfTlI Se tienen las 3 relaciones principales: 3 3 - sen 3a = 3 sen a - 4 sen a cos 3a = 4 cos a - 3 cos a 3 „___3 tan a - tan a tan 3a = ---------~—— 1 — 3 tan a 13.2. RELACIONES AUXILIARES______________________________________________[X] Se obtienen a partir de las relaciones fundamentales con ayuda de arcos dobles e identidades 4 sen a = 3 sen a - sen 3a 4 cos a = 3 cos a + cos 3a sen 3a = sen a (2 cos 2a + 1) cos 3a = cos a (2 cos 2a - 1) sen 3a = 4 sen a.sen(60° - a), sen (60° + a) cos 3a = 4 cos a.cos(60° - a), cos (60° + a) tan 3a = tan a.tan(60° - a), tan (60° + a) Identidades Trigonométricas del Arco Triple T13 123
sen 18° = "JS. 1 4 Nota: eos 36° = PROB. 1 Calcule: a) eos 11 Io RESOLUCIÓN a) 4E = 4 senlO" sen(60°- 10°)sen (60° + 10°) b) sen 159° 4E = sen 3 (10°) = sen 30° a)cos 111" = eos 3 (37°) = 4 cos337°- 3cos 37° 4E=| E=| b) 4P = 4 eos 20° eos (60° - 20°) eos (60° + 20°) 4P = eos 3(20°) = cos60° eos 111° = 256 300 125 " 125 4P = — P = — 2 " 8 eos 111" = 44 125 b) sen 159° = sen 3(53°) = 3sen 53° - 4sen3 53° c) Q = tan 5° tan (60° - 5o) tan(60" + 5o) Q = tan 3(5°)= tan 15" Q = 2- V3 ,3 sen 159° = 12 256 5 125 sen 159° = 300 256 125 ' 125 PROB. 3 Determine el valor de: sen 159° = 44 125 sen310o+cos3 20° senl0° + cos20° PROB. 2 Calcular: RESOLUCIÓN Multiplicando numerador y denominador por 4, así: a) E =sen 10° sen 50° sen 70° b) P =cos 20° eos 40° eos 80° c) Q = tan 5° tan 55° tan 65° K = 4sen310o+4cos320° 4(sen 10° + eos 20°) A continuación aplicamos una de las relaciones auxiliares: KACSO IDITO1B1 T13 124 Problemas de Trigonometría y cómo resolverlos
K _ 3sen 10°-sen30°+ 3cos20° +cos60° 4(sen 10°+cos 20°) K _ 3senl0°+ 3cos20° 4(sen 10°+cos 20°) Finalmente simplificamos, obteniendo: „ _ 3(sen 10°+cos 20°) _ k — — 4(senl0° + cos20°) " K“ 4 PROB. 4 De la figura que se muestra calcule la medida del ángulo x. => a cot 0 cot 20° cot 40° = a cot 10° tanO = cot 20° cot 40° cot 10° En el Es AHB: Escribiendo en términos de tangentes: tan 0 = tan 10° tan 50° tan 70° Finalmente tiene la forma: tan 0 = tan 10° tan (60°-10°) tan (60° + 10°) tan 0 = tan 3 (10°) => tan 0 = tan30° „ . _ a cot 10° acot0cot40° 0 = 30° x = 20° ESTRATEGIAS DE RESOLUCIÓN 1) Al igual que ángulos dobles, cuando se tenga potenciación cúbica, se debe degradar, es decir bajar al exponente de 3 a 1, aplicando las identidades siguientes: 4 sen3x =3sen x - sen 3x 4 eos3 x — 3cos x + cos 3x 2) Utilizar las fórmulas anteriores en forma apropiada. 3 ) Aplicar cada una de las relaciones auxiliares presentes a situaciones problémicas específicas. Identidades Trigonométricas del Arco Triple
Enunciados de Problemas con Resolución RELACIONES FUNDAMENTALES 01.-Simplificar: ' _ cos3x-cos3x sen3x+sen 3x — eos x + sen x A) 1 B)2 C)3 D)4 E)6 02.- Simplificar: W = 4 eos x. cos(60° + x). cos(60° - x) A) eos 3x B) eos 2x C) eos 3x D) eos 4x E) eos 6x 03.-Simplificar: sen3x.cscx 3cos r + cos3x W=-------------— +---------------- 0,75 - sen ~x 3senx - sen 3x A) 4 + cot3x B) cot3x C) 2 cot3x D) 3 cot3x E) 4 cot3x 04.- Simplificar la expresión: A) 1 B)2 C)-2 D)-l E)3 06.- Simplificar: 1 + secx+cos2x.secx W =-------------------------- tanx + 2senv + sen3x.secx A) 0,5 csc x B) csc x C) 2 csc x D) 3 csc x E) 6 csc x V5 07.-Si: sen x + eos x = , 2 calcular: M= lósenóx A) 10 B)12 Q15 D)ll E)16 08.-Si: 1 —cos2x . „ n =4,0<x < —, l + cos2x calcular: tan 3x 1 „ 3 5 7 2 A>n B>n C)n D)n E)n 09.-Si: W = 4 - 8 sen29°- 3 sec 18°, A) tan x B) tan 2x C) cot x D) tan 3x E) -tan 3x 05.-Simplificar: ,,, sec3x sec2x 8 tan x W =------5— +------x— - -—x— sec 3x sec 2r tan 2v entonces una expresión equivalente para W será: A) tan 9o B) tan 18o C) 2 tan 18° D) 2 tan 9° E) tan 36° 3 3 10.- Si: 3 tan" v + 6 tan x -1 = 2 tan x. calcular: tan 6x A) 4/3 B)3/4 C)l/2 D)-l/2 E)2/3 Al sO IDITOIII T13 126 Problemas de Trigonometría y cómo resolverlos
RELACIONES AUXILIARES A) cot x B) cot i C) 2 cot x 11.- Calcular el valor aproximado de la expresión: W = tan 9o + tan 27° + tan 63° + tan 81° D)tan E)2tan A)4j5 B)j5 C)2j5 D)3j5 E)6-j5 18.- En la siguiente identidad, determine el va- lor de M: 12.- Calcule el valor aproximado de la expresión: sen3x cos3x kn Ian72o+tan36“ w= cot72°+eot36° M + = , x * , k G Z senx cosx 2 A)V2 B)73 C)2-j3 D)2-j5 E) J5 A)0 B)1 C)-2 D)2 E)1 19.- Calcule el valor de la expresión: 13.- Calcule el valor de: W = tan 10“(3 cos 10° - 2 sen 10“. cos 70°) W = cos380“.cos 140°.cos260° A) 1 B) 1/4 C) 1/6 D) 1/8 E) 1/2 A) 1/8 B) 1/2 C) 1/4 D) 1/8 E) 1/16 14.- Calcule el valor de: PROBLEMAS CONDICIONALES M = 3 sec210“. sec250°. csc"20° 20.- En la siguiente igualdad, se tiene una iden- tidad trigonométrica: A) 16 B)18 C)42 D)36 E)64 Asen4x+Bcos2v _ Q . = sen 3x.cot x + cos 3a. tan x 15.- Calcular el valor de: ovllA 1 Calcula: A+B. _ sen 22°.sen 82°.sen 38° cos412°-sen412° A) 1 B)2 C)3 D)6 E)4 A) 1/4 B) 1/2 C) 1/8 D) 1/16 E) 1/32 sen3x—senx 21.-Si se cumple: sec2x- "" , r senx 16.-Simplificar: calcular: cos 4x í 2cos6x +1 'l W - 1 , 1. tan 3x ^2cos6x-l j A) 1 B) 1/2 C)V2/2 D)0 E)-j3/3 A) tan x B) tan 3x C) tan 6x sen3x 1 , . . 22.- Si: = —, al calcular W = cos 4x, se D) tan 8x E) tan 9x senx 3 obtiene: 17.- Simplificar: A) 1/3 B)-l/3 Q-2/3 D)2/3 E)-7/9 4senx Í3x3 W- + 3cot — l+2cosx V 2 7 23.- Si: a esc x = 3 - 4 sen'x 9 ? 2„ b sec x = 4 cos'x - 3, calcular: “a~+ b Identidades Trigonométricas del Arco Triple Il27|
A)-2 B)0 Q0.5 D)-l E)2 30.- Si: cot x = A.cot 3x, sen3x4-sen3x 1 24.-Si: , calcular: tan x cos3a-cos x 2 esex calcular: W - • - csc3x A) 1 B)2 C)V3 D)-l E)-V3 k k A>* B)I+1 C>ÁH 1 25.- Si: sen(60° - x) = — , m 2A_ T-Á k D)A-1 E) 2Á -1 calcular: W = -eos 6x 31.-Si: tanx=(2-n Jí). tan^jj. 41 5 329 63 P1 A>57 B)A C)|g D) 63 E)1U calcular sen(x 4- 30°). l-cos9a i , 26.-Si: -(x -3x+l) , l-cos3a A> B)^ entonces el valor de “x” es: A) 2 eos a B) eos a C) sen a D>25 E)^W2 D) 2 sen a E) 3 sen a 32,-Si: 3 sen 2x4-2cos2x = 2;0<x< , __ ... sen3x cos3x 27.- Si: 4- — m, secx-cosx esex —senx calcule: tan 3x calcula: “cot2x” A)-^ B)¿ A)mf2 B)„i/3 C)m/4 D)/n/6 E)2//n 7 7 28.- Si: sen x 4- eos x = k, D) 43 E) ’43 calcular: W = eos 3x - sen 3x 7 33.- Si: tan(x + 15°) - , calcular: ran 3a A) A B)3A-2A3 Q2A-3A3 D)3A-A3 E)3A-4A3 A) 37 B) C) || 29.- Si: tan <}> - V5 tan 0 -1 = 0, calcular: tan 6<t> D>37 E>37 A>f B)f 34.- Calcule el valor de la expresión: W = (Vi -6senl0°). sec 80° O)^ E)^ A)0 B)0,5 C)1 D)1.5 E)-2 1281 Problemas de Trigonometría y cómo resol verlos 'MK.'ÍÍ.ÍS
35.- Calcule el valor de la expresión: W = 2 eos 160°.(2 sen 20° -1)(2 sen 20° + 1) A) 1 B)0.5 C)l,5 D)2 E) ^2 36.- Calcular el valor de: (3cos65°—4scn325°) sen270°—sen220° sen 50° ®> 2 a calcule: — b A) tan 120° B) tan 240° C) tan 30° D) tan 54° E) tan 21° D)— 25 E)V6±V2 41.- En la figura mostrada, calcular: “x” 37 .- Calcular el valor de: 4cosl8°-3secl8° M= tanl8° A) 1 B)2 C)l,5 D)2,5 E)3 38 .-Calcular el valor de: Vl + 6cos20° M= 2cos20° Si: m Z BAE = 54 , m Z EAD = 27 , m ZBCE=x wiZCED = 57+x , A) 1 B)0 C)0,5 D) 1.5 E)3 39 .- Si a * fe, eliminar el arco “x” de: (a + fe)tan x — (a - fe) tan 3x ...(1) a sen*x = b eos 6x ... (2) A) fe* - a~ = ab D) 8fe* - 2a* = ab B)8fe* -a~=ab E)6a~-&b~ — ab C)8fe* -2ab = a2 mZ ADE = 90 A) 10° B)20° C)15° D)25° E)30° 42.- De la figura mostrada, BC = 3, CD = 4, aiZBAC=»iZCAD=/nZDAE,WiZABE=90. Determine el área de la región triangular ABE. SITUACIONES GRÁFICAS 40.- En la figura mostrada: mZEAD = 7 , m ZECB = 53 , m ZABE = 67 . AC ± BD , ED=a , EC = fe . A) 61 a2 B)61.15u2 C) 63,25 w* D) 54,16 a2 E) 59,53h2 Identidades Trigonométricas del Arco Triple
r • H -- »-- \ I / ^TTairsformaciorfes de Sumas ^^Diferencias aProductos Es frecuente encontrar situaciones que involucran razones trigonométricas senos y/o cosenos de ángulos (arcos) diferentes, cuya simplificación o tratamiento, requiere de la habilidad de saber transformar las expresiones de sumas o restas a producto. Las relaciones que permiten realizar dichas transformaciones provienen de: sen (a + £) = sen a cos £ + cos a sen £ sen (a - £) = sen a cos £ - cos a sen £ cos (a + £) = cos a cos £ - sen a sen £ cos (a - £) — cos a cos £ + sen a sen Si sumamos y/o restamos miembro a miembro sen(a + £) con sen (a - £) y cos (a - £) con (a + £) y haciendo cambios de variables convenientes, como: a + £ = A a-£ = B De manera que: a = a £ = - ... (*) Por ejemplo si sumamos sen (a + £) + sen (a - £) , tendremos: sen (a + £) + sen (a - £) = sen a cos £ + cos a sen £ + sen a cos £ - cos a sen £ sen (a + £) + sen (a - £) = 2sen a cos £ ... (**) Sustituyendo (*) en (**), obtenemos: _ „ /A+B) /A-B) sen A + sen B = 2sen I—2— j cos I—2— / En forma análoga se obtienen las otras relaciones. Si A y B son dos ángulos cualesquiera, se cumplen las siguientes transformaciones: sen A + sen B = 2 sen | . cosí | l Z J \ 7 / A 1 r A í A — B 1 sen A - sen B = 2 cos —=— . sen —5— .A RACSO W^BDiTO*aa T14 130 Problemas de Trigonometría y cómo resolverlos
< . do í A + B ) (A-B eos A + eos B — 2 eos —=— L eos —«— k “ / k “ eos A - eos B = -2 sen ]• 8enl ^2^ 14.1. RELACIONES DE TRANSFORMACIÓN CONDICIONADAS En un triángulo ABC se cumple aplicando las relaciones de transformaciones, y teniendo en cuenta que A + B + C = 180° se obtiene: sen A + sen B + sen C = 4 eos eos . eos y ABC eos A + eos B + eos C = 4 sen % • sen . sen y + I sen 2A + sen 2B + sen 2C = 4 sen A. sen S. sen C eos 2A + eos 2B + eos 2C = -4 eos A. eos B. eos C -1 PROB. 1 Si: 36 0 = n; calcule: K = 2 eos 60 Pero como: 360 = n K _ eos 20 +eos 40 +eos 80+eos 100 cos40 + cos20 =* 6e= K b RESOLUCIÓN *★***<***##***#<★* Luego K = 2cos = 2 Agrupando convenientemente, para luego transformara producto. K= 5/3 K = => K = K = (eos 100+eos 20)+(eos 80 + eos 40) (cos40+cos20) 2cos60cos40+2cos6Ocos20 (cos40+cos2ü) PROB. 2 Encuentre el valor de: _ sen40°+sen20° | sen50°+senl0° ~ cos40°+cos20° cos50°+cos!0° Transformaciones de Sumas o Diferencias a Productos TU 131
RESOLUCION Transformando a producto _ 2sen30°eos 10° 2sen30°cos20' — 2 eos 30° eos 10° 2cos30°cos20‘ Al simplificar obtenemos: => N = tan 30° + tan 30° => N = 2tan30‘ Finalmente: p _ cos4Osen20 cos!20sen2O — sen40sen20 cosl20cos20 Al simplificar obtenemos: => P = cot 40 + tan 20 Utilizando arco mitad => P = cot 40 + csc 40 - cot 40 P = csc 40 /3 3 N = 1N=^ 3 Pero: 0=^ 40 = PROB. 3 40=18° => P = csc,8°=^iF Si 0==~ , determinar el valor de: 40 p _ sen60-sen20 senl40-senl00 — cos20-cos60 cosl40+coslO0 Reemplazando su valor numérico tendremos: RESOLUCIÓN Transformando a producto: 4(75 + 1) (75-l)(75 + l) 2cos40sen20 2sen20cosl20 -2sen40sen(-20) 2cosl20cos20 P= 75 + 1 ESTRATEGIAS DE RESOLUCIÓN 1) Cuando se tiene una suma o una diferencia de dos senos o dos cosenos, se debe transformar a producto. 2) Cuando se tiene sumas o diferencias de varios senos o cosenos, se agrupa convenientemente, para luego transformar a producto. 3) Para transformar a productos no interesa el orden de los ángulos (si uno es mayor o menor que el otro), es suficiente con aplicar las propiedades de transformación. T14 132 Problemas de Trigonometría y cómo resolverlos ÜRACSO
Enunciados de Problemas con Resolución TRANSFORMACIONES A PRODUCTO 01.- Reducir: cosl5x + 10cosl0x+cos5x M=-------------------------- sen 15x +1 Osen 1 Ox+sen5x A) cot x B) cot 2x C) cot 4x D) cot 5x E) cot 10 x 02.- Reducir: cos(45°+x) -cos(45°-x) M sen( 120°+x) - sen( 120o—x) A)^2 B) 1 C)0 D)0,5 E)2 03.- Simplificar: _______l,5 + -/3sen2x (cos2x+i j.tan(x+30°) A) 1 B)j2 C)2 D)l,5 E)>/3 04.- Reducir: 2 W = cos 10x + cos8x+6cos 2x+3cos2x-3 - 8 cos 3x.cosx A)0 B)1 C)0,5 D)2 E)-0,5 05.- Reducir: _________2(sen 2x + sen 2y)____ ~ 1+cos 2x + cos 2_y+ cos(2x-2_y) A) tan x B) tan y C) tan x - tan y D) tanx + tan y E) tanx. tan y 06.- Simplificar (tan 2x+cot x-4 coszx)(sen 3x—sen x) W * "5 [2 sen x(sen x+cos x)—1] A) cosx B) 2 cosx C)senx D) 2 sen x E)cos 2x 07.- Transformar a producto: M = sen x + sen 7x + 2 sen 3x + 2 sen 5x A) 2 sen 2x cot x. sen 3x B) 2 sen 2x. cot 3x. sen 6x C) 2 sen 4x. cot x. cos 3x D) 2 sen 4x. cot 2x. sen 3x E) 2 sen 4x cot x. sen 3x , ... cosl2°+sen30°+cos84" 08.-Calcular: W= E-0s24o+sen42o A)1 B)| C)-| D)0 E)2 09.-Calcule: W — eos210° + sen220° - sen 20°.cos 10” 3 3 1 A) 1 B)| I))' E)3/4 10 .- Calcule el valor de la expresión: 2sen50°-l W=----------------- tan35°+cot35” A) 1/4 B)l/2 C)1 D)0 E)-l/4 Transformaciones de Sumas o Diferen^ ;as a Productos
11 .- Halle “m” de la identidad: sen4x + sen6x _ sen mx sen2 r - sen x A) 1 B)5 C)3 D)2 E)0,5 12 .- Calcular el valor de: M = eos 20° + eos 100° + eos 140° A)0 8)0,5 C)1 D)-l E)-0,5 13 .- Determinar el valor de “M” si: sen5x + senóx + sen7x -------------------— - M. eos 4x + senx + sen2x+sen3x A) 1 B)2 C)3 D)4 E)03 14 .- Calcular el valor de: M= sena + sen5a+sen9a _„ra„= 10g cosa+cos5a + cos9(i P A) 2 B)-2 C)-l D)0 E)1 PROBLEMAS CONDICIONALES 15 .-Si a= —, D) -2 sen 2A. sen 2B . sen 2C E) 4 eos 2A. eos 2B . eos 2C 1 18 .-Si: tanx= — .reducir: sen5x+sen3x M =-----------~ cos5x + cos3x A>¿ B>í O< D>| cos8x—cos7x 19 .-Si: — -------— =Pcos5x + Q, cos3x — cos2x calcularde:P + Q A)0 B)1 02 D)3 E)3,5 20 .- Si: sen x + sen y = a eos x + eos y = b, calcular: sen (x + y) A)fc B)« C)^ n, ab r. 2ab ^a2*# E)b2 + a2 calcule: W = sen 23a-sen 7a sen 2a+sen 14a 21.-Si: sen5x sen3x = m, calcular: tan4x tanx A)0 B)1 C)-l l))0,5 E)-05 m c- eos a+sen ¿> , , , (a — b\ 16 .-Si: ——— -r =k, calcular: tanl—5—I seno+coso \ 2 / A>‘ B>í o| D’|íí 17 .- Si: A + B + C = 71, expresar como producto: W = sen 4A + sen 4B + sen 4C A) 4 sen 2A . sen 2B . sen 2C B) 2 sen 2A. sen 2B . sen 3C C) -4 sen 2A. sen 2B . sen 2C A)— B)— Q™-1 D)^-} E)"^ m 'm ' m ' m + l ' m — 1 22.-Si: R 2 P. cos(Qx). eos (Sx) = eos 8x+eos 4x - 4 sen x + 2, determine el valor de: (P + Q) - (R + S) A)0 B)03 0-1 D)-Q5 E)1 c- cos47°-cos73° v3 , , , 23*"Sl- sen43<’+senl7° “ 3 k -calcilIar; W = tan 32° A>‘ «i Oí D>|zí Problemas de Trigonometría y cómo resolverías •'jií RACSO
24.- Si: A + B + C = 7t, calcular “M” de: A B sen A + sen B - sen C = M . sen~. sen — C COS2 A) 2 B)4 C)1 D)8 E)0,5 25.- Si: A + B + C = 7t, simplifica: A A senA.sec— + (senB + senC).tan— W =----------------------------— senB-senC p /B -C\ A) cot y B)coty Qcot^ 2 j D) cot E) cot 26.- Si: senx + sen y = m\ cosx + cosy = n; m * 0, n 0. Calculan W = cos(x + y) 3O.-3Si: sen 5x + sen 3x + sen x = M sen5x + N sen x + P sen x. calcular: W = P - N - 2M A)0 B)05 C)-0,5 D)1 E)-l 31.-Si: W = senfx + 20°) + sen(x- 50°) ,xG [0; 360o], determine un valor de “x” que maximice dicha expresión. A) 105° B) 120“ C) 110° D) 108° E) 115° EXPRESIONES EQUIVALENTES A)n B)^ C)^ D) n2+m2 n2 —m2 E) n2 + m2 27.-Si: tan 0 = 2cos20°-cos40° sen40° determinar: sen 0 , si 0 G IIIC J3 -J? A)2 B)-2 C)f D)f E>-4 28.- Si:^ec(x + a) + sec(x - a) = 2 sec x. calcu- lar: cos"x en termino de “o” 32.- Si: W = 4 sen x + sec x, determine una expresión equivalente de W en factores. A) 4 cosx sen pe 4--^). cos(v- B) 4 csc x sen pe + yE j. eos Px - yE j C)4cotx . senpt +. cos(x--pEj D)4secx.senpc + ^j .eospe— E)4secx. senpr + y^j.cos(x-jE) 33.- Si: W = 4 sen 40° - V5, determine una expresión equivalente de W. A) eos B) 2 eos C) sen" 2 a 2 a D)cos E)2cos — 29.- Si “0” y 0 son las raíces de la ecuación: ( 6+0 a sen x + b. eos x = c. calcular W = tan 2 A} a B)¿> C)- D)^ E) v a b b A) tan 10° B) tan 20° C) tan 30° D) tan 35° E) tan 40° 34.- Expresa como un monomio: M = 1 + 4 eos 20° A) >/3 B) ^3 cot 10° C) y¡3 cot 40° D)-j3cot40° E)^3. cot 20° 35.-Si:W = 3+ V5 , determine una expresión trigonométrica equivalente para W Transformaciones de Sumas o Diferencias a Productos
A) Jó sen 75° B) 2 Jó sen 15° C) Jó sen 15° D) J3 sen75° E) 2 Jó sen 75° 36.- Si: W = 1 + cos 2x + cosx, determine una expresión equivalente de W en factores: .. . x A) 4 cos x cos 2 X B) 4 senx cos 2 C) 4cos x cos D) 37.- Factoriza: 1 + sen 14° + cos 14° A) 2 Jicos 38° D) 2 J3 cos 38° cos 7o B) Jicos 38’ cos7o E)2 JÍ . cos 38°. cos 7o C) J5 cos 38° cos 7° 38.- Eliminare! arco “x" de: sen5x sen3x sear abe A) c(a + c) = b(b + c) D) c(o - c) = + c) B) c{a + b) = b(a - c) E) c(a + c) = h(b - c) C) c(c + a) = b(b + c) 39.- Elimine el arco “x” de: sen(x+-^j =/w + senx .. (1) cos (x + = n - cos x ... (2) A)JÍm = »i B)(2 + JÍ)n = ni C)(2-JÍ)r/=m D)(2 + j3)« = m E) (2 - J5)« = m SITUACIONES GRÁFICAS 40.- En la figura mostrada, AB = CD, AC = 3. BD = 4, m Z EAB = a, m Z ECD = 6. 41.- De la figura mostrada, DC = 2 CB, calcular: W = A) 1 B) ‘ Q | D) | E) | 42.- En la figura mostrada, ABCD es un cuadri- látero. »iZDAE=x. »iZ EAB =3x. BE ±CA; DC ± C A; AB = AD = k. Determine el área de la región sombreada, en términos de «A» y «a». 7-1 o q D) 2k~ sen 2 v E) Á-.sen 2x Problemas de Trigonometría y cómo resolverlos ’A RACSO Wbditoqbb
¡J'WsfSr m ici on ésde Producto ’ÜSumas'O Diferencias Al igual que en las transformaciones de sumas o diferencias a productos, también es importante las transformaciones de productos a sumas o diferencias, se obtienen a partir de: sen (a + P) = sen a eos P + eos a sen p sen (a - P) = sen a eos P - eos a sen p eos (a + P) = eos a eos P - sen a sen P eos (a - P) = eos a eos P - sen a sen P Se obtienen sumando o restando senos con senos y cosenos con cosenos, así por ejemplo: sen (a + p) + sen (a - P) = sen a eos p - eos a sen p Es decir 2 sen a eos P = sen (a + P) + sen (a - P) En forma similar se deducen las otras relaciones de transformación: Seanx ey dos ángulos cualesquiera, se cumple las siguientes transformaciones: 2 sen x. eos y = sen (x + y) + sen (x -y) 2 eosx. seny = sen (x + y) - sen (x -y) 2 cosx. cosy = eos (x +y) + eos (x -y) 2 senx. seny = eos (x-y) - eos (x +y) Transformaciones de Producto a Sumas o Diferencias T15 137
PROB. 1 Reducir a su mínima expresión: K = cos 3x sen x - cos 4x sen 2x + cos 5x sen x RESOLUCIÓN Multiplicando por 2 ambos miembros, luego transformando a sumas o diferencias: 2K = 2 cos 3x senx - 2cos 4x sen 2x + 2 cos 5x sen x 2K = sen 4x- sen 2x - (sen 6x - sen 2x) + sen 6x - sen 4x Al efectuar como sigue tendremos: 2K = sen 4x - sen 2x - sen 6x + sen 2x + sen 6x - sen 4x 2K = 0 Finalmente: K = 0 PROB. 2 Calcule: W = cos^ + cos^ + cos — 7 7 7 RESOLUCIÓN Multiplicando por 2 sen y a ambos miembros: 2 sen y ,W = 2seny cosy + 2cos ^y sen-y + 2cos-y^ sen y 2seny .W = sen^y + sen4y -sen^y + sen^y -sen4y 2 sen y .W = sen ^y Pero: ti 6n 7 7 n 6n sen y = sen-y Problemas de Trigonometría y cómo resolverlos JktKACSO 2DITO11I
Luego tendremos: 2 sen y. W = sen y Finalmente: W = y PROB. 3 senfPx) De la siguiente igualdad: 4 eos x.cos 3x + 1 = --—- : calcule el valor de "P" s senx RESOLUCIÓN ♦ **fc*t*t**O**fctt**O***Ot**fc*Ot**t**tt**** Multiplicando a ambos miembros por senx 2.2senxcosx eos 3x + senx = sen (Px) sen2x Luego: 2. sen 2x eos 3x + sen x = sen (Px) Transformando a sumas: sen 5x - sen x + sen x = sen (Px) sen 5x = sen Px Al comparar obtenemos: P — 5 ESTRATEGIAS DE RESOLUCIÓN 1) Cuando hay productos de 2 senos o 2 cosenos o un seno con un coseno, se debe transformar a sumas o diferencias. 2) Necesariamente para realizar la transformación, debe llevar un coeficiente 2; Si no tiene hay que multiplicar. 3) En las transformaciones a sumas o diferencias, no interesa cual de los ángulos es el mayor o menor, es suficiente con aplicar convenientemente las fórmulas de transformación de producto a suma o diferencia. Transformaciones de Producto a Sumas o Diferencias
Enunciados de Problemas . con Resolución APLICACIONES DIRECTAS 01.-Simplificar: 2cos3x—cos2x.cosx + cosx W=-------------------------- 8cos4x-6cos2x A) sec 3x B) csc 3x C) sec 2x D) csc 2x E) sec 4x 02.- Reducir a un monomio la siguiente expre- sión: W=sen 3x(3 sen 3x- sen 9x + 4 sen 3x.cos23x) A)sen3x B)4sen23x C)sen23x D) sen2 x E) 4 sen2 x 03.- Reducir a un monomio la siguiente expre- sión: W=2cosx.(eos5x + 5eos3x + lOcos x) A)sen63x B)4sen4x C)8cos4x D) 16 cos6x E) 32 cos6x 2senl5x.senx 04.- Simplificar: M = —-—------- + eos 6x 2cos 10x4-1 A) eos x B) eos 2x C) eos 3x D) eos 4x E) eos 6x 05.-Reducir: M = 2 eos 6x+2 eos 4x 4- 2 eos 2x - sen 7x.csc x A)-l B)1 C)0 D)0,5 E)-0,5 06.-Simplificar: 1 sen7x W = —----------- eos 2x - 2 eos 5x.cos x 2 senx A)0 B)-l/2 C)l/2 D)1 E)-l 07.- Encontrar el valor de: “a”, si: eos 12°. eos 60°.sec 72o- eos 24° = sen2ot.cosot sen3a A) 2o B)3° C)4° D)5° E)6° PROBLEMAS CONDICIONALES 08.- Si: eos A = eos B. eos C . ................../A + Bi . (A-B\ reducir: W = tan I—%—I . tan I—I A) tan2 (y j B) tan2 C) tan D)tan(yj E)tan2(yj 09.- Si 60x=771, calcular el valor aproximado de: . secx.seny W = tan(x 4- y)--------— cos(x4- y) A> 44 17 44 117 „ 131 A)Tl7 B) 44 C) 17 D) ~44 ^53 10.- Si: sen 7x = 2 sen x, calcular: W = eos 2x 4- eos 4x 4- eos 6x A)0 B)-l C)1 D) 1/2 E)-l/2 T15 140 Problemas de Trigonometría y cómo resolverlos RACSO 1DITO1II
11.- Si: tan(a + 6) + tañí a - 6) = 2, calcular: 18-Si: M = sen 4a - sen220 32sen6x-M + Ncos2a + Pcos4a + Qcos6x, A) 1 B)-l C)0,5 D) 0,5 E)0 entonces el valor de: M + N + P + Q es: 12.- Si en un triángulo ABC se cumple: A)0 B) 1 C)-l D)0,5 E)-0,5 sen B. sen C = cos 1 2 1, entonces dicho trián- gulo es: 19.- Si: M sen a + N sen 3a + P sen 5x = 8 sen x. entonces el valor de: M + N + P es: A)0 B)1 C)2 D)3 E)4 A) isósceles B) equilátero C) rectángulo 20- Si; W = 2 cos 10°.sec 20° - J3 , entonces D) acutángulo E) oblicuángulo una expresión equivalente de W será.- 13.-Si: A) tan 10o B) tan 15° C) tan 20° cos . tan + 2 sen y = sen y (W - 1), D) tan 25° E) tan 35° entonces W es igual a: 21- Si: P sen25° - Q sen 21° = cos 16°.cos 13° - A) 1 B)2 C) 4 D)0 E)0.5 cos 59°.cos 20°, entonces al calcular: P + Q se obtiene: 14.- Si: tan 3x = 2, calcular A)0 B)1 Q-l/2 D) 1/2 1$ 1 W = (cos 3x + 2 cos x)(sen 3x - 2 sen a) 22- Si:\V = 4 cos 3x . cos x + 1, entonces una A)0 B)l/5 C)1 D)2/5 E)-l expresión equivalente de W en factores será: tanx l+cos2jc 15-Si: — = j-, tana 1 + sen x A) sen 5jc.sec a D) sen 3a.scc x B) sen 5a.csc x E) sen 5A.sen a calcular: W = sen(3x + a). csc(a - a): C) sen 3x.csc x A) 3/2 B)5/2 C)7 D)3 E)5 VALOR NUMÉRICO 16- Si: cos a . cosy. cos z = nr.x + y + z = 2n, 2 2 2 entonces al calcular \V = sen x + sen y + sen z, se obtiene: 23- Calcular: W = cot 19° - -J2 sen 26° esc 19° A)/m B)2mi C)2/n-l A)0 B)-l C)l/2 D)-l/2 E)1 D) 2(1+mi) E) 2(1-mi) 24.- Calcule el valor de: sear+seny y _ y 17-S,: sen(A+y) "3’tan2 ~Rtan 2 "Q’ entonces al calcular: P.Q se obtiene: 2sen40° (sen50°—2sen 10°.cos40°) M“ sen80°—sen20° A) 1 B)-l C)0 D)0.5 E)-0,5 A)0 B)-l/2 C)l/2 D)1 E)-l Transformaciones de Producto a Sumas o Diferencias
25 .- Calcular el valor de: M = .Jsenl5x.sen6x+-i-cos21x , 7t para: x = — A) 1/2 B)l/3 C)-l/2 D)-l/3 E)3 26 .- Calcule el valor de: 1 M = — . sec 80° - 2 sen 70° A)0 B)-2 C)1 D)2 E)-l 27 .- Calcule el valor de la expresión: W = sen 50°.sen 10° + sen 610°.sen 130° - sen 430°.cos280° A) 1 B)3/4 Q-l/4 D)-3/4 E) 1/4 28 .- Determine el mínimo valor de la expresión M definida por: M = 3 eos 4x + 4 sen 3x. sen x B)l/2 C)-l/2 D)0 E)3/2 J3cos20°—sen40° tan x =-----;---—;--------- , 1+senlCF calcule un valor de “x” A) 10° B)18° C)20° D)32° E)40> MISCELÁNEA 30 .- Una expresión en términos de sen A y sen B, equivalente a sen (A + B) sen (A - B) es: 7 7 7 *7 A) sen A - sen”B D) -(sen A + sen”B) 7 7 B) sen"A + sen'B E) -(sen A - sen B) C) sen2B - sen2A A)-3/2 29.-Si: 31.- Simplificar: sen 2a.sena+sen 4a.sena+sen la.sen 2a sena.eos 2a+sen 2a.eos 5a + sena.cos 8a A) tan a B) tan 3a C) tan 2a D) tan 5a E) tan 7a 32.- Reducir: sen 7 a senfl - 2 eos 2a - 2 eos 4a - 2 eos 6a A) sen a B) csc a C) tan a D)1 E)-l 33. Calcular: eos 6o. eos 42°. eos 66°. eos 78° A)0 B)1 C)-l D)l/16 E) 1/8 34.- Al simplificar la expresión: E = sen 6o. sen 54°. sen 66°, obtenemos: A) sen 12° B) 2 sen 6° C) sen 18° D) 2.sen 12" 35 .-Hallan sen 40°.sen 80° + sen 80°.sen 160° + sen 160° .sen 320° A) 1 B)3/2 C)3/4 D)3/8 E)0 36 .- Simplificar: senx sen3x sen2x sen4x sen x sen x sen x sen2x ^^’sen2x $en4x senx senx ^'sen4x senx 37 .- Si: a - b = 15°, ¿cuál de las siguientes expre- siones es equivalente a: eos b (sen a + eos a) Problemas de Trigonometría y cómo resolverlo -’TgRACSO
•Jl r~ A) (eos 2¿ + v3sen2¿) B) (yfí cos2í> + sen 2b + -j3) ~ /7i „. sen 2b , C) -y 3 eos 2b+------i-l I 2 D) (eos 2¿> + -j3 sen 2b) ..Fi r- E) (sen 2¿> + V3cos 2b) A) B) V5 - 1 D) E) C) V5 1 39.- Reducir: -Jsen a (sen 3a +senx) A) |senx| B) |sen2A| C) |sen3A| D) |cos2a| E) |cos3a| 40.-Si: eos 6 = 0,75 38.- Determine el valor de la siguiente ex- presión: N = 4sen6 sen46 sen66 + 2 sen6 i i 36 6 calcular 32 sen — . sen - A) 6 B) 8 C) 10 D) 12 E)ll Cuando: 0 = 7t 30 PROSTAFAIRESIS Cuando Neper se encontraba enfrascado en el estudio de las sucesiones para luego deducir sus famosos logaritmos, le visitó John Craig, médico del rey Jaime VI de Escocia, y le habló del uso que se hacía en Dinamarca del método de prostafairesis. Por aquella época comenzaron a aparecer diversos tipos de identidades trigonométricas por toda Europa, como resultado de un menor énfasis en los cálculos de resolución de triángulos, y mayor en cambio en las relaciones funcionales de tipo analítico. Entre estas identida- des estaba un grupo de fórmulas conocidas como las "Reglas de Prostafairesis", es decir, fóríhulas que permitían convertir un producto de funciones circulares en una suma o una diferencia (de donde les venía el nombre de prosthaphaeresis, palabra griega que significa suma y resta). Transformaciones de Producto a Sumas o Diferencias T15 143
l^íteesjonesty :S,e n es' ^^Tri giwWétn cásT fi®O£tííU- -<-i -' El trabajo con sumas de números es frecuente en múltiples problemas que deben enfrentar a diario los especialistas de diversas ramas del conocimiento, y para su determinación se trabaja desde el punto de vista teórico en la obtención de expresiones compactas, no obstante las facilidades que brindan las aplicaciones de ia O fanática (Automatización, mediante sistemas electrónicos, de las comunicaciones y procesos administrativos en las oficinas ), con vistas a evitar errores provenientes de la captación de datos. Tomando en cuenta el amplio espectro de aplicaciones que pueden ser beneficiadas con este tipo de resultado, en el presente trabajo se realiza una recopilación de las propiedades de las sumatorias que figuran en la literatura, posteriora lo cual se proponen y demuestran otro conjunto particularmente relevante cuando se trabaja con funciones de variable discreta cuyos intervalos de variación son uniformes en todo el dominio de la función. En muchas situaciones problémicas donde se presenta una serie de sumas o productorias, éstas deben reducirse en otras equivalentes más sencillas, para lo cual es recomendable tener en cuenta dos sumatorias y productorias de senos y de cosenos, como las más importantes. 16.1. SUMATORIAS Siendo : n = Número de términos r = Razón aritmética del ángulo x = Primer ángulo u = Último ángulo Verificándose que: u=x+(n-l)r => n = + 1 > así: eos o 71 . + eos . + eos . +... + eos | |si = i 2m+1 2m+1 2m+1 ^2m+1J 2 2n , 4n , 6 ti ( 2n _ 1 cos 2m+1 +cos2m+1 +cos2m+1 +--- + cos I 2n+l 1’2 T16 144 Problemas de Trigonometría y cómo resolverlos RACSO JDITOII1
16.2. PRODUCTORIAS 71 2ti 371 nn 1 2m+1 €Os2m+1 €Os2m + 1 ...eos 2w+1 2" 71 2ti 3tt nn V2n+1 2m+1 2m+1 2m+1 ...Afc» 2m + 1 2" PROB. 01 Calcular el valor de: „ 2ít 4ít 6ít E = eos— + eos— + eos — RESOLUCIÓN iiiiii***ii**i*i * * al Analicemos el numerador de la fracción: N = sen x + sen 3x + sen 5x +__+ sen (2n - I )x Aplicando la fórmula de sumatorias, tal que: Multiplicando ambos miembros por H) 2sen-J.E= 2sen —.eos—+ 2sen—. eos— 1 7 7 7 7 n = n ; u = (2n - l)x => r = 2 x + 2sen—.eos— 7 7 3n n 5n 3n 5n =sen-y -seny +sen-y -sen-y +senn-señ- Esta expresión tiene la forma de una serie telescópica, por lo cual se reduce a: „ n „ n „ 1 2sen—. E = - sen— E = - „ ti PROB. 02 Calcular el equivalente de: senx + sen3x + sen5x + sen (2n - l)x K — cosx + cos3xcos5x + + cos (2n - l)x sen(nx) => N =-------- . sen (nx) senx b) Ahora analicemos el denominador de la fracción: D = cos x + cos 3x + cos 5x +...+ cos(2n - l)x Aplicando la fórmula de sumatorias, tal que: n = n ; u = (2n - l)x => r = 2x .eos |~x+(2n-l)xj „ sen(nx) r . D = -----i—- .eos (nx) senx Sucesiones y Series Trigonométricas T16 145
senínx) „ _ /V v _ senx__________________ - D - sen(nx).cos(nx) senx K = tan nx 03.- Hallar la suma de la serie: S = csc 6+ csc 26 + csc 40 +.+csc 2"’1 6 RESOLUCIÓN ★★★★★★★★★★★★★★★★★★ Generamos una serie telescópica, tal que: csc 6 = 1 sen6 sen| sen(e-|) 6 6 sen— sen6.sen — 2 2 Desarrollando el numerador de la expresión y simplificando, encontramos que: 0 csc 6 = cot - cot 6 csc 26 = cot 6 - cot 26 csc 46 = cot 26 - cot 46 csc 21”1 6 = cot 2" 2 6 - cot 21”’ 6 S = cot|-cot2nl 6 ESTRATEGIAS DE RESOLUCIÓN 1) Para calcular la suma: Sn = a¡ + a2 +...........+ an no existe un método general, pero uno de los muy pocos casos en que es posible calcular su valor es el siguiente. Si an se puede expresar en la forma: an = b)l + ¡ - bn Entonces: S = a, + a2 + a? +...........+ a = b2 - b¡ + b3 - b2 + b4 - b3 +......+ bn +J - bn “ «i i I — a esta serie se le denomina «Serie telescópica» 2) Para simplificar una productoria como: gerir la siguiente estrategia. P = a¡ . a2 . a3.an; podemos su- Haciendo: an = IU1 Un _ Ü2 U3 U4 LUi ut • u2 • u3... un p = U„+1 3) Cuando se tenga una productoria de tangentes de ángulos consecutivos, se aplicará lo siguiente: . tan^j . tan^r . . . tan^ = ^ñ±¡. 2n+l 2n+l 2n+l 2n+l jn donde: n = número de términos 4^racso WlDITOlli T16 146 Problemas de Trigonometría y cómo resolverlos
Enunciados de Problemas con Resolución SUMATORIAS 01.- Reducir: W = sen jc + sen 3x + sen 5x + ...+ sen 35x 7 7 A)sen"(12x) B) 12 esc a C)sen (18x) D) sen2(18x). esex E)sen2(12jr).cscjc 02.- Indicar una expresión equivalente de: W = cosx + cos 3x+ cos5x+ ... + cos45x A) y. sen 46a . esc x B) 2 sen 23x. esc x C) 2 sen 24x. esc 4x D)8sen21x.csc6jc E) . sen 48x. esc 12x 03.-Calcular el valor de: W = sen 2° + sen 4° + sen 6o + ... + sen 540° A) tan 9o B) cot Io C) sen4 9o D)3sen49° E)-4cot29° 04.- Determinar el valor de: ... n 5n 9it 33n W = sen -¡y + sen yy + sen '¡y + . . + sen “¡y . VT3 fíj5-yf5 V10-2V5 A) 4 B) 2 C) 4 D) -J4+-J3 V5-V3 05.- Identifica la expresión equivalente de: n 2n 3n W = sen— +2sen— +3sen— + IOtt lln 12n lOsenyy + llsenyy+ 12senyy A) 2cot B) 6tan C) 12cot D) 8tan $ E) 4c cot o Z Zv) 06.- Reducir la siguiente suma: t ít t 3n , M = cos“— +cos"— +cos — + ... n n n 2 71 ... + cos (2n - 1) — n ax n m 3" r<x 5'1 rxx 11 ex n A) 12 B) T C> ~T D) 2 E) 4 07.- Calcular el valor de M, si: 2n 4n 2n 6n M = cos~.cos~ +cos~-cos~ + 4n 6n + COS — -COS — A) 3 B)-2 C)-3 D) 1/3 E)-l/2 08.- Calcular el valor de: > n 7 2n 2 M = sen~— + sen'— + sen — 7 7 7 A) 3/4 B)7/4 C)7/6 D)4/9 E)5/9 Sucesiones y Series Trigonométricas
09.- Determinar el valor de W, si: 4 ir 4 2n 4 3n W = sen — + sen — + sen —- 7 7 7 A) 21/16 B) 1/6 C)2/7 D)3/5 E)2/9 10.- Evaluar la suma de la serie finita: N = sec a. sec 2a + sec 2a. sec 3a + ... ... + sec na. sec(n + l)a A. . tan(n —l)a A)cos(n+l)a-------- B) sen(n + l)a - tan(n + l)a } sena tan(n + 2)a ' cosa tan(,7-l)a ' cosa tan(n - l)a sena tan a sen a tan(n — l)a sena tan a sen a 11.- Encontrar la suma de la serie finita: R = tan^ + |tan^- + | tan~- + ....+ 2n-i tan 2n A) ¿T tan^ +2 tanx B) ^TCot-^-Zcotx 1 JC C) 2-iitan 2,1+1 -2" tanx D) cot ^¡7-2" cot x 1 X E) ¿¡Tcot^T -2tanx 12.- Calcular: 2n 4n 6tr W = sen — + sen — - sen — A) — B) — C) — D) — E) — 2 2 4 6 4 13.- Determinar el valor de M: 2n 4n 6n M = sec— + sec — + sec — 7 7 7 A)-l B)3 C)-3 D)5 E)-4 14.- Hallar la suma de la serie: S = csc 6 + csc 2 6 + csc 40 + ... + csc 2” ’* 1 6 A) cot 2'1 6 + cot 20 B)cot2’e-cot2nl6 C)cot 31 0-cot 3"‘1 0 D)cot2’* 6-COS2"'1 6 E) cot 2"'16 - cot 2"'1 6 15 .- Calcule el valor de: R = tan x + tan 2x + tan 4x cuando: 7 - x = n A)0 B)-l C)(V7) D)-(V2) E)(V7) 16 .- Calcular la suma de la siguiente serie: sen x + sen 3 x + sen 5 x +... + sen (2 n - 1) x sen2 nx sen2 nx eos2 nx cosx senx senx eos2 nx sennx cosx E' senx 17 .- Calcular el valor de: eos 10° + eos 30° + eos 50° + ... + eos 170° A) 1/2 B)0 C) 1 D)1 E) 3 Problemas de Trigonometría y cómo resolverlos RACSO P RD1TO1II
18 .- Hallar el valor de: E = sen2 1 ° + sen2 2o + sen2 3o + — + sen2 90° A)22,5 B)30 C)45 D)45,5 E)60 19 .-El valor de: _ 2/1 f z(2\° 2Í180o\ E = cos 1^1 +cos K) +...+ cos ^,es: A)0 B)90 C)45 D)89,5 E)91 . 2ít 4ít 671 20 .- Calcular: eos — + eos — + eos A)0 B)1 C)j D)-2 E>4 21 .- Determinar: 2 ji 2 2 eos — + eos -y + eos — 15 7 A)0 B)1 C)‘ D) ¿ E) 22 .- Simplifica: „ sen/Uj-UoJ rl— ------------ eos Uj .cosU0 A) tan U20 - tan Uo C) tan U19 - tan Uj E) tan U20 23.- Calcular: 2tc A eos -jr- + eos -77- + eos — + eos 7. y y y y । sen(U29 U19) eos U20 -eos U19 B) tan U19 - tan Uo D) tan U20- tan Uj 6ít 8rt A) 1 B) \ C)-^ D)-l E) | 24.- Reducir: 3(2+eos 3x) (2 + cos3x) A1 4 4 „ 3(2+eos3a) p (3+cos3x) 4 4 _ (3 + cos2x) J 4 25 .- Calcular: 2tt „ 4n - 6jt „ 2(hr eos -¡y + 2cos jí + 3cos -j-j- +... +10 eos -jp A)-11/2 B)-10/2 C)-ll/3 D)-13/2 E)-ll/5 26 .- Una semicircunferencia de radio a se divi- de en n arcos iguales, calcular la suma de las distancias de los distintos puntos de sección a cada uno de los extremos del diámetro. A>“(“"ío2) d>«(c«J-2) B>«(cot¿-1) E)2„(co(i-1) C>4«£-3) 27 .- Calcular la siguiente sumatoria: eos x + eos (120° - x) + eos (120° + x) A)0 B) 1 C) 1 D) 1/2 E)-l/2 28 .- Calcular la siguiente sumatoria: E = eos x. eos (120° - x) + eos x. eos (120° + x) + eos (120° - x) eos (120° + x) A) 3/4 B)4/3 C)l/4 D) -3/4 E) 1 PRODUCTORIAS 29 .- Determine el valor del producto: (1- tan2x)(l- tan"2x)(l-tan24x).(l-tan22"’,x) 3 X cosn(x) + cosr(120°-x) + cos,1(120o+x) n=l A) 211 tan x gx 2I1+I tan x q tanx tan2nx tan2n+lx tan2n,x Sucesiones y Series Trigonométricas T16 149
22n tan x p. 2°+1 tan x tan2njr tan2nIx -J2 V2 -J2 J? V2 A)y B)^- D)^y E)y3 30 .- Calcular el valor de W, si: 36.- Reducir el siguiente producto finito: 1 rt 2n 3rr 4n W = - .tan — .tan — .tan —. tan — A)3 B)6 Q12 D)1 E)0 31.- Determinar el valor de: n 4n 5n W = sen y. sen— . sen — COS ~2 • eos . COS -g cos A)cos nx D. sen x B) sen nx C)-------- z z 2r D) SCnX- 2Dsen^ 2" „ sen x E)------~ 2ncos— 2n v'3 V7 A) 3 B) — C)5 D)6 E)y— 3 o 32.- Calcular el valor de: M= (1 + COS-^y) (1+COS^y) (1 + COS^y) A) 1/8 B)2/3 Ql/6 D)3/8 E)5/6 37.- Calcule; A) 1 B) 1/2 P - cos y C) 1/4 38.- Calculare! valor de: .COS-y^ -COSy^ D) 1/8 E) 1/16 jr 2tt Ott P — sen j .sen .sen । —.... sen j'q 33.-Evaluar la expresión: n 3n 5n M= cosy .eos—.cos~ A)2/5 B)-l/8 Q3/5 D)l/7 E)-2/7 A) V19 B>^ D)^r V19 E) 2« 39.- Calcular: 34.- Determinar una expresión equivalente de: K = (2cosx- l)(2cos2x- l)(2cos4x- 1) 8cos4x+l 2cosx-l „. 4cos4x + l 2 2cos x +1 F 2cos 3x +1 ' 2cosjt-1 _. 3cos3x-l 2cosx-l n 2cos8x + l ' 2cosx+l 35.- Calcular el valor de: - 3n 5n sen 2^ «sen -sen 13n , sen 3 l + cos(2n-l)5 n=I l+COS^y^ A) 1 B) 3 C)7 D)5 E)2 40.- Indica el equivalente de: n 2n 3n 4n 5n COSy . COS-y-. COS-y. COS-y . COS-y A)64sen7 B)60sen7 C)64sen5 I~.. 1 ít 1 tt D) sen y E) ^4 sen y ^áRACSO BpBDITOftMB T16 150 Problemas de Trigonometría y cómo resolverlos
CAP 17 ^•^Bii^TrigOTOrrigtr; óas1 1 En el estudio de las funciones trigonométricas, la característica que más resalta es la periodicidad, es decir, la característica fundamental de las funciones trigonométricas es que todas ellas son periódicas, pues se repiten de idéntica forma en intervalos iguales. Muchos fenómenos de la naturaleza son periódicos, como por ejemplo: Las estaciones del año, las 24 horas del día, los latidos del corazón, etc. Se debe tener en cuenta que toda función trigonométricas tiene una representación gráfica especifica. Asimismo se debe tener en cuenta que toda función tiene gráfica, pero que no toda gráfica representa a una función. 17.1. FUNCIÓN TRIGONOMÉTRICA___________________________ Es un conjunto no vacío de pares ordenados (x; y), tal que para todo valor de la primera componente (corresponde un valor angular expresado en radianes}, le corresponde un único valor obtenido mediante una evaluación de la función, que se denota así: y = F.T(x) Esto es: F = {(x; y)/x, ye R ; y = F.T.(x)} 17.1 A Dominio de una ET. (Df) Son los valores que toma la primera componente en una función. (Se encuentra sobre el eje x). 17.1B Rango de un E T. (Rf) Son los valores que toma la segunda componente en una función. (Se encuentra sobre el eje y). 17.1C Función Par y, L Una función fes par, si V x e Df: i) -x e Df ii)f(-x) = F(x) Nota: La gráfica de toda función par es simétrica respecto al eje y. 17.ID Función Impar Una función f es impar, si V x e Df. i) -xeDf — ii)f(-x)=-f(x) Nota : La gráfica de toda función impar es simétrica respecto al origen del sistema de coordenadas. 17.1E) Función Creciente Una función f es creciente en un intervalo 1 de su dominio, si V xt , x2 e I se cumple: f (Xj) < f (x2) , siempre queX] < x2 Funciones Trigonométricas
17.1F) Función Decreciente Una función f es decreciente en un intervalo I de su dominio, si V x, , x2 e I se cumple: F(X|) > f(x,) , siempre queX] < x2 17.1G) Funciones Periódicas Una función f cuya regla de correspon- dencia es y = f(x), se dice que es periódica, si: i) Vx a (x + T) e Df i¡K(x + T) = f(x);T*O Donde “T" es el periodo de la función. (El menor valor). 17.2 FUNCIÓN SENO F = {(x ; y)/y = sen x; x e R} 17.2A) Representación gráfica 17.3 FUNCION COSENO 17.2B) Análisis de la gráfica Dominio: Dfe R a Rango: Rze (-1; 11 Periodo: T = 2n Es función impar en todo su dominio, Crecien- te en el 1C y IVC, Decreciente en el IIC y I1IC Si: f = sen(/?x) => Tf = j-rr I K I F = {(x;y)/y = eosx;xe R} I7.3A) Representación gráfica 17.3B) Análisis de la gráfica Dominio: Dfe R a Rango: fye [-1 ; l] Periodo:T = 2rt Es función par en todo su dominio, creciente en el IIIC y IVC, decreciente en el IC y IIC Si: F = eos (ftx) => T>= I K I T17 152 Problemas de Trigonometría y cómo resolverlos 4RACSO
17.4 FUNCIÓN TANGENTE F = {(*;y)/y = tanx;xe R - (2n + l)n/2 ;ne Z} 17.4A) Representación gráfica 17.4B) Análisis de la gráfica Dominio: Df& R -(2n + l)n/2;ne Z Rango: Rz e R a Periodo:T = n Es una función impar en todo su dominio, es creciente por intervalos. Si: F = tan (ftx) => ¡/ = j^j 17.5 FUNCIÓN COTANGENTE F = {(x;y)/y = cotx;xe R -(m);ne Z} 17.5 A) Representación gráfica 17.5B) Análisis de la gráfica Dominio: Dfe R - (nít) ; n e Z Rango: R/e R a Periodo: T = n Es una función impar en todo su dominio, es decreciente por intervalos. Si: F = cot (fer) 17.6 FUNCIÓN SECANTE F = {(x; y)/y = sec x; x e R - (2n + 1 )k/2 ; n e Z} 17.6A) Representación gráfica 17.6B) Análisis de la gráfica Dominio: Dfe R - (2n + I )tV2 ; n g Z Rango: R¿gR-(-1;1) Periodo: T = 2ti Es una función par en todo su dominio, es creciente en el IC y IIC, es decreciente en el HlCylVC. Si: F = sec (ftx) - _ 2n ‘ 1*1 Funciones Trigonométricas
17.7 FUNCIÓN COSECANTE F = {(x; y)fy = escx; x g R - (nn) ; n g Z} 17.7A) Representación gráfica 17.7B) Análisis de la gráfica Dominio: Dfe R - (nít) ; n g Z Rango: R¿gR-(-1;1) Periodo: T = 2ít Es una función impar en todo su dominio, es creciente en el IIC y I1IC, es decreciente en el IC y IVC. Si: f = esc (fex) => T, = — ' R PROB. 1 Pero-1 <cosjc<1 Arrufen => -1 ccosxd Dada la función f, cuya regla , . sen2x correspondencia es: f\x) = senJf de => -2<2 cosx< 2 => -2 < f (x) < 2 a) Determine su dominio b) Evalúe su rango c) Elabore su gráfico d) Determine su periodo e) Discriminar si es par o impar. RESOLUCIÓN ****************** R,g <-2,2> Para que la función esté definida en todos los Reales, necesariamente sen x * 0, entonces: senx^O => x*kn; Vfe g Z, luego: a) Df g R - {fe rt}; V fe g Z b) f(x) = 2>efí'xcosx >eííx => f (x) = 2cos x d) Su periodo es T =2n x . , x sen2x . .. . e) La gráfica f (x)= seriJf es simétrica res- pecto el eje Y, luego es una función par (pues toda función par es simétrica respecto al eje y, mientras que las funciones impares son simétricas respecto al origen. ftkHACSO U DlTOIKI T17 154 Problemas de Trigonometría y cómo resolverlos
PROB. 2 Dada la función f, cuya regla de corres- pondencia es: f(xj = tan |x| + cot |x|. a) Calcule los puntos donde / no está definida. b) ¿Es periódica la función?, si lo fuera determine dicho periodo. c) Obtenga el Rango de la función d) Elaborar su gráfica. RESOLUCIÓN En primer lugar analicemos |x|: 1) Si x > 0 => /(x) = tanx +cotx => /(x) = 2csc2x 2)Six<0 => f(xj = tanx- cotx => f (x) = -2csc 2x 3) Si x = 0 => f (x) no está definida. Para dar respuesta a las preguntas, elaboramos la gráfica: a) Se observa que, x no está definida en: 0,-|,n;37i/2;2ji;..= ™ Los puntos donde/(x) no está definida son en: b) Sí es periódica, de periodo: T = 2n c) Viendo la figura Rf e R - (-2; 2) ESTRATEGIAS DE RESOLUCIÓN I) Analice la existencia de la función 2) Obtenga los valores que no definen a la función 3) Elabore el gráfico de la función 4) Vea si la gráfica es Simétrica respecto al eje "y" o es Simétrica respecto al origen para asegurar si es la función par o impar respectivamente. 5) Cuando la variable de la función trigonométrica está afectada por valor abso- luto ( I x I), analice qué ocurre cuando x>0,x = 0ax<0 6) Para determinar el dominio se recomienda analizar todas las restricciones posi- bles de la expresión dada. 7) Para determinar el rango se recomienda transformar la expresión dada hasta quedar en términos de una F.T. Luego reconstruir la función a partir del dominio de la F.T. obtenida. Funciones Trigonométricas
Enunciados de Problemas con Resolución DOMINIO DE LAS F. T. 01.- Sea la función fdefinida por: senx + cos.r+ tanx |senx| (13.- Sea la función/definida por: cosx — cos5x /(x) =---------~ cosx-cos3x Determine su dominio. determine el dominio de f A)R-y,VIeZ QR-y.VJteZ D)R-y.V*eZ E)R-y, V ke Z 02.- Sea la función/ definida por: > 1 isen'x---- 4 determine el Dominio de la función f. A) n . 5n , —+KJI,— + KJI 6 6 , V *6 Z B) [| + lt7t,^ + k7t], V ke Z C) I, V ke Z 1_6 6 J Djfl+fcn.^+jtnj,v *e z E) j+fot,^ + fc7tj, V ke Z A)R-y,AeZ B)R-y,*eZ C)R-y,JtGZ D)R-y,ieZ E)R- y, ke Z 04.- Determine el dominio de la función /defi- nida por: /(x) = 3 tan ^3x + j + cos 2x A)R-(3áí+ l)y ,V ke Z B)R-(3áí+ 1)|,V ke Z C)E-(3A+l)y V ke Z D)R-(3A.+ l)y,V AgZ E)R-(3A + 1) j.V ke Z 05.- Determine el dominio de la función/defi- nida por: ftx)= Jtanx-1 + J-JJ tanx T17 156 Problemas de Trigonometría y cómo resolverlos H RACSO ÍDITD1II
A) ^ + kn;j + kn B) ^ + kn;^ + kn C) ^ + kn;^ + kn L5 6 D) ^ + kn;^+kn ,VkeZ ,VkeZ , V*gZ ,VkeZ C) O D) <2 \ 2/ \ 2 / \ 6/ \ o / E)/o;^\ O \ 4/ \ 4 / 09.- Determine el dominio de la función/defini- da por: J jsecjq — |cscx| E) ^ + kn ;^+kn 6 2 ,VieZ 06.- Delimitar el dominio de la función /defini- da por: /*) = 3tan5x-2 A)R-(2A + 1)|,V ke Z B)R-(2A: + 1)^,V Ag Z C)R-(2* + l)yj,V Ag Z D)R-(5A+1)^,V AgZ E)R-(2¿ + ke Z 07.- Evalúe el dominio de la función /definida por: ftx) = Vsenx + Veos* + -Jtanx , si: x e [0; 2tt] A)K- V ke Z C)K- V ke Z E)K- V ke Z B)IR- V Ae Z D)K- V ke Z 10.- Evalúe el dominio de la función/definida poc /(*)= 75 sec4 (3* + ^) A)R- I*/* = ^(2k +l);JtG z| B) R -{*/* = ^ (2¿ +1);A G z} C) R-1*/* = ^(2k + l),k e z| D)R- {jr/x = y(2¿+!);£G zj E) R-{x/x = y(2¿+l);A:Gz} A) 0;^ B) 0;^ Q 0,| RANGO DE LAS F.T. D) E) 08.- Encontrar el dominio de la función /defi- nida por: /*) = J2(tanx + cotx) - 4 , si x e (0; 27t) A)^0;^ B) (0:5) u 11.- Determine el rango de la función/ defini- da por: Funciones Trigonométricas T17 157
12.- Si el dominio de la función f definida por: 5 + 2sen* /a) =---------es determine el rango de/: A*(2’Í B)(2;l Q(2;| D)(5;3 13.- Sea f una función definida por: seiu-1 encontrar el rango de f. A) {-1} B){f} C){n}D) {1} E){0} 14.- Sea /una función definida por: f(x) = sen-* + 3|sen *|, determine el rango de/: A)[O;1] B)[0;2] C) [0; 3] D)[0;4] E)[0;5] 15.- Determine el rango de la función /defini- da por: cosa + 2 cosa+1 A)|J;+~) B)[|;+~) C) 3. 3’ +oo 7 D>|_5;+~ E) 16.- Evaluare! rango de la función /definida por: flx) = eos2* |senv|-l A) (-4;-3] B)(-5;-l] C)<-2;-l] D)(-2;-7] E)(-6;-3] 17 .- Encontrar el rango de la función /definida por: fíx) = eos2* + 4 eos x + 7 A)[4;l] B)[5;12] C)[7;3] D)[2;10] E)[4;12] 18 .- Sea la función /definida por: cos2 x flx) = ^osx—sen* . determine el rango de/ AX-V1 ; yfi) B)(-v/3;>/3> C)<-j2;j2> D)<-j5; J5> E)<-7?; y/7) 19 .- Sea la función /definida por: fix) = 4 cos2(ttx) - 4 cos(tdc), x e ; 3 determine el rango de/*) A)[-l;2> B)[-l;8) C)[-l;l> D)[-l;3> E)l-1;6) 20 .- Sea la función /definida por: 4 2 2 4 /*) = sen *. eos * + sen *. eos *, evaluar el rango de la función: A) [O; l/4> B) [O; 1/2] C) [O; 1/3] D)[0;l/4] E)[0;l/5] 21 .- Sea/ la función definida por: ,, sen*+sen2* + sen3* /w=-------------Q. sen2* encontrar el rango de la función/: Problemas de Trigonometría y cómo resolverlos RACSO IDITO1I1
A)<-1; 1) <J <1 ;7> B) <-l; 2> o <1; 4> C)(-l;3> O (1;6> D)(-l ; 1) u <1 ;5> E)(—1 ; 1> O <1 ; 3) 22.- Si: x G (n; 37t/2), determine el rango de la funciónf definida por: fl*) = (1 + tanx)2 1 + tan2x + 3 26.- Delimitar el rango de la función/definida por: 2 / (x) = (tan x - cot x) A)[0; + oo) B)[l; + oo> C)[2; + °°> D)[3; + «>) E)[4; + °o) 27.- Encontrar el rango de la función/defini- da por: A)(4;5] B)[4;5] C)[4;5> D)(2;5] E)(2;4> 23.- Sea f una función definida por: '5n , 4 /(x) = tan x + cos x, si x G 3 / n /x) = |tan x| + |sen x|, si: |x| < — , determine el rango de la función. A) 0;^) B) 0;^ C) 0;^ /2-V7.2^-l\ /2->/4.2^-l\ 2 ’ 4 / J\ 2 ’ 10 / „/2->/2,2>/3-l\ E)\ 2 ' 2 / 28.- Evaluar el rango de la función f definida pon D) 0;—7— E) 0 ;^ 24.- Evaluar el rango de la función f definida por: /(x) = tan4 x + 4 tan2x + 7 A) [9; + °°) B) [3; + °°) C) [2 ; + <») D) [7; + «>) E) [5 ; + <») 25 .- Determine el rango de la función/defini- da por: /(x) = 2senx-3tanx,sixG \ 6 4 / A)(->/4 -3;- Vs -1) B)(->/2 -3;- sfi -1) C)(-7ó -3;- >/3 -1) D)(->/5 -3;- -Jl -1) E)(-a/2-3;- >/4 -l) A)[2;4> B)(2;4] C)(2;4> D)[2;8] E)[2;4] 29 .- Sea la función/definida por: ‘ /x) = cos x. tan x, determine Dy-n Ry. A)(-4;l) B)(-3;l> C)(-2;l> D)(-l;l> E)(-0;l> 30 .- Determine el rango para la función/defi- nida por: fix) = |sen x|. esc x + |csc x| sen x A){-3;2} B){-2;1} C)(-2;2) D){-5;5} E){-2;7} Funciones Trigonométricas, T17 159
31 .- Para la función definida por: f{x) = sen x + cot x + -»/senx-l + cscx, determine el rango. A)Rf={l} B)Rf={2} QR,= {3} D)Rf={4} E)Rf={5} 32 .- Sean las funciones f y g definidos por: fix) = sen( -Jx2 l ) y g(x) = |ex - sec x|, determine: Df o Rg. A)[5; + °o) B)[4; + °o> C)[3; + °o> D)[2; + °o> E) + VALORES MÁXIMOS Y MÍNIMOS DE LAS F.T. 33 .- Determine el valor máximo de la función/ definida por: senx-2 X) l+|senx—2| A>-i B>-f «-i H 34 .- Sea /la función definida por: /x) = 2 sen 7x. eos 3x - sen 4x - 5, encontrar: f^ + 2/^. A)-15 B)-16 C)-17 D)-18 E)-19 35 .- Identificar el máximo valor de la función/ definida por: /x) = (3 - sen x)(3 + sen x) +1 A) 14 B)13 C)12 D) 11 E)10 36 .- Si *7n” y “M” son los valores mínimo y máximo respectivamente de la función /defini- da por: a . 4 4 j(x) = sen x + eos x, encontrar el valor de ‘7n + M” A) 3/2 B)4/2 C)5/2 D)6/2 E)7/2 37 .- Sea la función /definida como: /» = 35-tan2x + 4tanx,xG [0°;90°]. identificar la medida del ángulo “x”, para ob- tener el valor máximo de la función. A) x=65°30’ B)x=64°30’ C)xs63°30’ D)xa62°30’ E)xa61°30’ 38 .- Determine el valor máximo de la función/ definida por: fix) = (4 - tan x)(2 + tan x) A) 10 B)9 C)8 D)7 E)6 39 .- Sea la función /definida por: fix) = tan x + 2 tan x, determine el valor mínimo de/ A)-4 B)-3 C)-2 D)-l E)0 40.- Calcule el valor mínimo de la función / definida por: fix) = tan x - cot x, si x G A) 4 B)3 C)2 D)1 E)0 41 .- Encontrar el valor mínimo de la función/ definida por: fix) = senx + tan x, si x G A)-2 B)-l C)0 D)1 E)2 42 .- Determine el valor mínimo de la función/ definida por: fix) = sec x - csc x, si x G „.5n . * 4 Aíracso ^oiToua T17 160 Problemas de Trigonometría y cómo resolverlos
A) 1 B)5 C)2 D)0 E)3 PERÍODO DE F.T. 43 .- Determine el período mínimo de la fun- ción/definida por: X X X fix) = sen — + sen ~ + sen ~ A)2n B)3n C)6n D)12n E)24n 44 .- Sea la función /definida por: 6x fix) = 3 sen 3x+ eos — + 1 evaluar el período mínimo de/: A)^ B)^ Q| D)^ E)-^ 45 .- Determine “a + b" si el periodo mínimo de las funciones: b A fix) = eos (ax + bn) a g(x) = sen (bx-ait) son respectivamente: — y - Además “a y b por números impares” A) 16 B)18 C)22 D)24 E)26 46 .- Sea/ la función definida por: fix) = cos( 7t eos x), evaluar el período mínimo de la función/ A)y B)JÍ C)| D)§ E)n 47 .- Calcule el período de las siguientes fun- ciones: 15f 2x) a)/x) = tan 1 g I b) g(x) = sec10(4x) 3jc A c)/í(jc) = csc I “y I Funciones Trigonométricas .. 7tt ti 14ít r,.9n ti 117t A)T;4;“T B)T;4;^" 5?t 7t 12n J 2 ’ 4 ’ 3 } 2 ’ 3 ’ 3 ... 9n n 14tt E)T: 4 :-3~ 48 .- Calcule el período mínimo de la función/ definida por: f(x) - sen a.cos x.tan jc.cot x.sec jc.csc x A)~ B)7ü 07? D)7t E)y 49 .- Calcule el período mínimo de la función/ definida por: 2 2 1 fix) = sec x + sec jc.csc x + csc x + — A)g B)y C)^ E)7t GRÁFICAS DE LAS F. T. 50.- Esbozar el gráfico de la función/definida pon f(x)= Vsen2x—sen4x T17 161
51.- El punto P(xf ; 2¿z + b) pertenece al sinus- oide y el punto Q(x> ; a - b) pertenece al n cosinusoide. Si además: jc, + x, = ~, calcular: A) 1/3 B)2/5 C)3/5 D)2/3 E)4/5 52.- Esbozar la gráfica de la función/defini- da por: sen2x cosx f(x) = |tan x. cos x] - 53 .- Indicar el gráfico correspondiente a la fun- ción /definida por: /*)= sen3x—senx senx 54 .- ¿En cuántos puntos la gráfica de la fun- ción/definida por: /x)= Jx -4|cosx], intersecta al eje X si x e (0; 3rt)? A)6 B)7 C)8 D)9 E)10 T17 162 Problemas de Trigonometría y cómo resolverlos RACSO DITC11I
55 .- Determine en cuántos puntos, la gráfica de la función /definida por: fix) = 2 |cos jc| - |sec x|, x G (0; 2¡t) intersecta al eje X. A) 2 puntos B) 3 puntos C) 4 puntos D) 5 puntos E) 6 puntos 56 .- Grafique la función/definida por: /jc) = |sen jc| + sen x 57 .- Determine la regla de correspondencia del gráfico mostrado: C)4seny + 2 D)4sen^ + 2 E)4sen^- + 2 o 59 .- Determine el área de la región sombreada del gráfico adjunto: A) 8 m? B) 6 nú2 C) 4 7tw2 7 7 D) 3 itu~ E) 5 un Funciones Trigonométricas T17 163
E) y V3 w MISCELÁNEA 61.- Sea/ la función definida por: fix) = sen jc.cos x, indicar verdadero con (V) o falso con (F) de las siguientes proposiciones: I) Es creciente enxe (0; — \ 4, D) Ry= ~¡± Vxg(0;ti/4> C) 63.- Determine los valores de “x” para los cuales la función/no se encuentra definida, donde: /(jr) = sec^2jr+^j + csc^2x + yj A)^(2n+l);neZ D)~(2n± l);n€ Z □ o B)^(3n±l);neZ E)y(2n±l);neZ C)y (2n± l);ne Z 64.- Dada la función/ definida por: fix) = (eos pe]) |sen jc.csc jc| determine su rango. A)(-l;5> B)(-l;3> C)(-l;6> D)(-l;l> E)(-l;8> HD Posee periodo mínimo igual a 2rt A)FW B)VFF C)WF D)FFV E)VFV 62.- Para la función /definida por: /jc) = tanjc.|cotjc|, determine la gráfica. 65.- A partir de la función/ definida por: fi*) = cosxcotx |cotx| ’ determine su rango. A)(-l;l)u(0;2> D)(-l;0>u(0;2> B)(-l;0>V>(0;l> C)(-1;O)<J(O;5> E)(-l; l>u(0;3> T17 164 Problemas de Trigonometría y cómo resolverlos "RACSO
66.- ¿Para qué valores de “x” en función /definida por: \ 2’7/ la 7 fix) = tan“x - 3, es no negativa? 69.- Sea la función/definida por: fix) = |cscx| - |sec x|, determine todos los posibles valores de x G (0; Tt) para el cual la función no es negativa E)(- B)(- n.n 2’3. u FtI .Tt\ L3’2/ Tt.Tt 4’5. u Tt. Tt\ .3’2/ Tt.Tt 2’3. u Tt.Tt\ .2’6/ Tt.Tt 2’3. u Tt.Tt \ .7’8/ Tt.Tt' 7’9. u Tt.Tt\ .3’2/ 67.- ¿Para qué valores de “x” en función/ definida por: fix) = -J| senx | — | cosx | no está definida? A)xe^;^ D)xe[f;^ B)xe[^;^] E)xg[^;^] \ 4 4 / 68.- Sea la función/definida por: fix) = cot x - tan x, determine todos los posibles valores de x G (0;—) para el cua] la función siempre es positiva. A)xg^0;^ B)xe ^0;^ C)xe D)xg(o^ E)(-1;ti> n 7n 4’ 4 la A)ag^ u 37t._\ 3 ’ 7 B)xg 0 3rt.„\ . 6 ’ / C)xg ^0;| 0 D)xg (o; j 0 E)xg (o£ \ 4 0 70.- Sea la función/definida por: fix) — 3 tan x - 2 sen 2x, determine todos los posibles valores de x G (0; Tt). Identificar los valores de x G (0; Tt) para los cuales /es siempre positiva: I. ití6 n. 571/6 . A) Solo I B) Solo II D) I y III E) Todas C) Solo III Funciones Trigonométricas
En el capítulo anterior se estableció que una función asigna a cada elemento del dominio, una y solamente una imagen, que desde luego puede ser común a varios o a todos los elementos del dominio. Si la función tiene además la propiedad de que la imagen es exclusiva, o sea, que cada imagen en el recorrido es de un solo elemento del dominio, se dice entonces que está función establece una correspondencia biunívoca o biyectiva entre los elementos del dominio y del recorrido. Cuando tal es el caso, se puede definir una nueva función conocida como función inversa, respecto a la función original, cuyo recorrido sea el dominio de la primera y viceversa. 18.1. CONCEPTOS PRELIMINARES CD 18.1A) Función Inyectiva Una función f se llama inyectiva si y solo si V x,, x2 e Dom f, se cumple: Axj) = flx2) => X! = x2 Es un conjunto de pares ordenados donde existe una correspondencia biunívoca entre las primeras y segundas componentes (Relación uno a uno) 18.1B Características de la Gráfica de una Función Inyectiva Toda gráfica de una función inyectiva es cortada en un solo punto por cualquier recta horizontal (recta paralela al eje x) 18.1C Función Inversa Si una función f es biyectiva, es decir, es univalente y suryectiva, entonces existe su función inversa y se denota por f* o F1. 18.1D Obtención de una Función Inversa Dada una función f biyectiva, la función inversa f * se obtiene intercambiando x por y a y porx, verificándose que: Df« — Rf Rp = Dj 18.I E Obtención de la Gráfica de una Función Inversa Dada la gráfica de una función f, para obtener la gráfica de su respectiva función inversa f* se procede de la siguiente manera. T18 166 Problemas de Trigonometría y cómo resolverlos RACSO IDITOIBI
i) Se traza la recta y = x (recta que es eje de simetría entre fyf*). ii) Se refleja la gráfica / respecto al eje de simetría. iii) La gráfica reflejada es la gráfica de la función f*. Las funciones trigonométricas por ser periódicas no son univalentes, sin embargo al restringir sus dominios, se logra que sean inyectivas. Dichas restricciones se muestran a continuación: FUNCIÓN DOMINIO RANGO y — senx [-jí/2;k/2] 1-1; 11 y = cosx [0;«l 1-1; 11 y = tanx <-íi/2;k/2> R y = cotx <0;«> R y = secx [0; n/2) u (n/2; n] R-<-l ;1> y = esex [-n/2 ; 0) u <0; k/21 R-<-l; 1> 18.2 NOTACIÓN PARA UNA FUNCIÓN TRIGONOMÉTRICA INVERSA______________________ Sea: Ar(O) = n => 6 = are f.t(n) v 0 = f.f’(n) Entonces: Si: y = f.t(x) => x = are f.t (y) Luego: y = are f.t.(x) 18.3 FUNCIÓN ARCO SENO y = are sen x Dominio: Dfe [-1; 1] D _ F n-n Rango: Rfe l-g.-g Función Creciente Funciones Trigonométricas Inversas T18 167
18.4 FUNCIÓN ARCO COSENO y — are eos x Dominio: Dfe í-1; 1] Rango: Rf e [0 ; ti] Función Decreciente 18.5 FUNCIÓN ARCO TANGENTE y = are tan x Dominio: DfeK Rango: Función Creciente 18.6 FUNCIÓN ARCO COTANGENTE y = arccotx Dominio: Dfe R Rango: Rfe (0;7t) Función Decreciente 18.7 FUNCIÓN ARCO SECANTE y = are secx Dominio: Df e R - (-1 ; 1) Rango: Rf e (0; n/2) vj (n/2; nj Función Creciente: V xe C-oo¡ -1) Función Creciente: V x e [ 1; iARACSO Pbditdk*! T18 168 Problemas de Trigonometría y cómo resolverlos
18.8. FUNCIÓN ARCO COSECANTE________________________________ y = are esex Dominio: Dfe R -<-l ; 1) Rango: Rf e [- n/2 ; 0) kj (0; n/21 Función Decreciente: V x e ; -11 Función Decreciente: V x e [ 1; 18.9. PROPIEDADES_______________________ 18.9A F.T.(O) = n => are F.T.(n) = 0 18.9B F.T. [are F.T.(n)l = n <=> n e DFTJ 18.9C are ET.[F.T.(e)l = 6 <=> 6 e RFTJ Es decir: sentare sen x) = x ; si- x e [-1 ; 1] cos(arc eos x) = x ; si: x e [-1 ; 1] tan(arctanx) =x ; si:xe R cotfarc cot x) = x ; si: x e R sec(arc sec x) = x ; si: x e R - <-i; O esc (are esex) = x ; si: x e R - <-i ; i> are sen (senx) = x ; si: x e [-n/2 ; n/2] are eos (eos x) = x ; si: x e [0; n] are tan (tan x) = x ; si: x e <-n/2 ; n/2) are cot (cotx) = x ; si: x e (0 ; n) are sec (secx) = x ; si: x e [0 ; n] - {n/2} are csc (csc x) = x ; si: x e [-n/2 ; n/2} - {0} are cot (- x) = n - are cotx are sec (- x) = n - are secx are csc (-x) = - are esex are sen x = are csc (1/x); -I <x < 1 - {0} are eos x = are sec (1/x) ; -I <x < 1 - {0} arctanx = arecot (1/x) ;x >0 are tanx = are cot (1/x) - Tt;x <0 18.9D FT1 de Variables Negativas are sen (-x) = - are sen x are eos (-x) — ti - are eos x are tan (-x) = - are tanx 18.9D Identidades Aditivas y Recíprocas de FT.l. are sen x + are eos x = n/2 ; x e [-1 ; 1 ] are tan x + are cotx = n/2 ;x e R aresec x + are esex = n/2 ;x e R - [-1 ; 1] Funciones Trigonométricas Inversas
18.9F Identidad Aditiva Especial are tanx + are tan y = are tan Donde: k = 0; si xy < 1 k = 1 ; si xy > 1 a x > 0; y > 0 /?=-!;sixy>l AX<0;y<0 PROB. 2 PROB. 1 Resuelva la siguiente ecuación: are senx + are eos (x2 -1) = Resuelva la ecuación: are sen 2x = are cosx RESOLUCIÓN RESOLUCIÓN ir-*-***-*--*-**** ***** ** Aplicando la identidad aditiva, se puede afirmar que: x-=x2-l => x2 -x -1 = 0 prc sen 2x = are eos x => a = p sen a =2x a eos P =x sen a = 2x a eos a = x 1 + 71-(-1)4 _ 1 + 75 2(1) =* X 2 Pero: sen2 a + eos2 a = 1 (2x)2 + (x)2 = 1 4x2+x2 = 1 => 5x2 = 1 La única solución que satisface es: 1 5 Puestoque: -1 <x< 1 a-1 <x2-1 < 1 Esdecir: -1 <x< 1 a0< x2<2 -1 <x< 1 a (-72 < x<Ovx< 72) Para ver cuál es la solución, lo hacemos a través de sus gráficas: RACSO y|>DITOBBB T18 170 Problemas de Trigonometría y cómo resolverlos
Se observa que hay solo un punto de intersección, y está en la parte positiva del i- & ejex Luego la solución es x= -¡z- RESOLUCIÓN Aplicando la identidad aditiva especial: N = tan are tan ( 1+_L 7 24 1-lx-7- 7 24 => N = tan arcti + kn Para determinar el valor de k aplicamos la regla: Como: 7X24~ 24<' =>ft = 0 Luego: N = tan are ti PROB. 3 N = tan Calcula: N = tan are tan 4; + are tan -J- 7 24 ESTRATEGIAS DE RESOLUCIÓN ["clan™ ” 161 + kn 1) Hacer cambios de variables a los arcos, enseguida trazar triángulos rectángulos y aplicar el teorema de Pitágoras. 2) Tener en cuenta los rangos y los dominios de las funciones trigonométricas inversas, de forma que estén adecuadamente definidas. 3) Hacer las gráficas correspondientes, para ver los puntos de intersecciones, y de esa manera ver las soluciones. 4) Siempre que sea pertinente, sustituya are tan x + are tg y, reemplace por su equivalente: Íx + y 4 . ~ + kti l—xy I Funciones Trigonométricas Inversas
Enunciados de Problemas con Resolución DOMINIO DE LAS F.T. INVERSAS 01.- Sea f una función definida por: 1 .---- n f(x) = ~. are sen v3jc — 1 + ~ , determine el dominio de la función. 1 2 3’3 02.- Sea la función/definida por: Jto = J áreseme—arecosx 71 I delimite el dominio de la función: A) 0;^ C) 04.- ¿Para qué valor de “a” la función /defini- da por: 71 — + aresenx 9 /x) = —---------- , no está definida? ----arecosx 3 A) 2/3 B) 1/3 C)2/5 D) 1/2 E)3/4 05.- Sea/una función definida por: ( 3.x — 1 71 /jt) = 3 are sen I ~ I + are tan(jt - 2) + y, determine el dominio de la función: A) [-1; 5/3] B) [ 1/4; 5/3] C) [ 1; -5/3] D)[5/3;l] E)[l;l/3] 06.- Encontrar el dominio de la función /defi- nida por: n 3 fix) - — + — are sec (4x-3) B) -^;2j2 / 2 D) \ J2 ; 2 03.- Sea/una función definida por: í x~ 0 ( 2jt + l'] fix) = 2 are sen I I + 3 are eos I I, encuentra el dominio de la función: A)[l;l] B)<-1; + 1) C) [-2; 1] D)[-2;2] E)[2;-3] B) u43;+°o) C)(— ; v [2; +-> D) ^-oo • A] lj[1 ;+«>) E) j u [ I: +<») A RACSO P«DiToaaa T18 172 Problemas de Trigonometría y cómo resolverlos
07.- Sea la función/definida por: ( 1 fix) = 2 are sec I x + ~ delimite el dominio de la función: 2 08.- Identifica el dominio de la función/defini- da por: _______ f(x) = are sec ( Vfix2 — 1) 09.- Determine el dominio de la función/defi- nida por: f(x) = are csc (2x2 - 7) A)(-°o;-3] O|-V3 ; V3 |LJ[2;+co> B)(-<»;-2> O[-V3 ; + V3]lj(2;+oo> C)(-~;-3] u[-y¡3 ; ^3 |l43;+o°> D) <-«>;-2] U [-V3 ; V3 ] u [2 ; +~> E)<-°°;-5] u[->/3 ; V3]u[3;+oo> 10 .- Delimite el dominio de la función/definida por: /(x) = are sec (8x + 3) + are csc(5x - 2) 11 .- Encuentra el dominio de la función/defi- nida por: f(x) = are csc x + are sen x + are tan x A) {-!;!) B){1;2} C){-2;-l} D){-3;1} E){-1; 3} RANGO DE LAS F. T. INVERSAS 12 .- Sea/una función definida por: f(x) = 2 are sen(2x - 1) + 3 are sen (3x + 1), determine el rango de la función. A){f} B>{f} C){?} D){^} E) {5} 13 .- Sea la función / definida por: f(x) = (are sen x)~ + 4 ¡are sen x| - 2tt, determine el rango de la función. Funciones Trigonométricas Inversas T18 173
n2 n2 A) 27i;^ B) -2n;-y C) -27t;y 4 4 n2 D) 7i;-- E) -271; J 18.- Determine el rango de la función/defini- da por: 4 f(x) = are cot (sen x + cos x) 14.- Sea f una función definida por: ir f(x) = 2 are senx + are cos x + delimitar el rango de la función. A) n. 12571 ] 3 ’ 360 J Fn. 1277t ] F7t. 127ti] |_4’ 365 J U |_4 ’ 360 J A) n.3rc] rJ-Í-ZE] r^F-—-l 2’ 2 J DJ[ 2’2] JL 2’2] r 7t. 120tl] r 7t.l257i: D)|_4’ 360 J fc)L4’ 365 19.- Delimitar el rango de la función/defini- da por: D) [3tt . tl] 2 ’ 3] E) 15.- Sea f una función definida por: n r~ f(x) = ~ + 2 are cos -Jx , identifique el rango de la función. /(x) = are cot 1 + x2 /3n 4n] / ¡t 3n\ D)\V’TJ E)\2;T/ 20.- Identificar el rango de la función/defini- da por: /(x) = are cos x + are cot x 16.- Sea la función/definida por: f(x) = 2 are tan(x2 + 2x + 2), encontrar el rango de la función. 17.- Sea / una función definida por: «xF^-HI mí"-5"! r^Fn-5TEl A) [7’4] B)L4’TJ C)L2’Tj F7t . 7t] F 7C 7tE1 DH4’3j E)L4’TJ 21.- Determine el rango de la función/defini- da por: /(x) = («re tanx )(arc cot x) , si: x> 1 7t f(x) = — + 2 are tan x.xG (0; 1J determine el rango de la función. ry. / 7i 2 D)\ °°’16 J p,. / 2n \ Problemas de Trigonometría y cómo resolverlos ^RAeso
22 .- Encontrar el rango de la función/defini- da por: f{x) = are sec(sen x + eos x) 23 .- Hallar el rango de la función /definida por: /(x) = csc {are sec x) A)[l;+oo) B)(-l; + l> C)[-l;+l> D)(-l;l> E)(l;+oo) DOMINIO Y RANGO DE LAS F.T. INVERSAS 24 .- Determine el dominio y rango de la fun- ción/definida por: /(*) = |-|xzrcsen(^^)+^| A)[-l;4]y [(); y] D) [-4 ; 1] y [o; B)[l;4]y [O; E)[-3;+2]y [o; j ] C)[-l ; 3] y [0;f ] 25 .- Sea la función/definida por: 2 7T /(x) = 3 arc sen(3x - 2) + 4 , determine el dominio y rango de la función. 26 .- Sea la función /definida por: 1 f2 + x) 3n /(x)= - .«recosí 4 ] + §. determine el dominio y rango de la función. A)[l ;2]y [f;^l D)[-6;2]yfe^l L o o j L o J B)[-3;2]y [jíy1] E) [-3 ;2] y [|;|] C)[-6;-2]y \ o o / 27 .- Determine el dominio y rango de la fun- ción /definida por: /(x)=|«rccot(x- 1) -^| A)Ry[0;^} D)Ry[o;^} B)Ry[();^ E)Ry [(); y) r 5tl\ C)Ry 28 .- Establezca el dominio y rango de la fun- ción/definida por: f{x) = arc sen(-x) + arc cos(-x) + arc csc (-x) + arc sec(-x) A){ l;l}y{7T} D){-l;l}y{37t} B){-3;l}y{n} E) {-2; 1} y {2n} C){l;3)y{n} Funciones Trigonométricas Inversas T18 175
29 .- Determine el dominio y rango de la fun- ción /definida por: f(x) = are sen x + are sec x + 1 A){l;l}y{^ +3} Ín 1 3 + 1) (71 2 +1J {tt 2 +3I E) {-1; 1} y +2| 30.- Determine el dominio y rango de la fun- ción/definida por: 1 /(x) = — are csc (3x) + n 36.- Calcular el valor de: 32.- Calcular el valor de: M = eos [are tan(--Jí) + are sen 1] -J2 -J2 -J5 -J3 A) 2 B)^ C)^ D>Y 33 .- Determine el valor de: f 1 M = tan I n + 2 are tan — A) 2/5 B)3/4 C)4/3 D) 1/2 E)l/5 34 .- Hallare! valor de: M = are sen (sen 5) + are cos(cos 6) A) 1 B)-2 C)3 D)2 E)-l 35 .- Evaluar M, si se sabe que: M= creeos sen 2k T .. n t,a 3n ..5/1 5tl A> 6 B> 2 C) 6 3 A) 1 A) 2 f 1 M = cot I ~ are tan B)2 C)3 37.- Determine el valor de: 5 12 D)4 M = 3 cos(2 are eos V2/3 ) B) 1 C)3 D)4 38.- Determine el valor de: E)5 E) 5 PROPIEDADES DE LAS F.T. INVERSAS 31.- Determine el valor de: crcsení — |+crctan( 1) M =-------\ ---------- arccosf—1) A)A B)15 C)9 D) 3 E)ll M = cot | — are tan cos(2 are sen -J7/30 ) A) 1 B)2 C)3 D)4 E)5 39.- Encontrar el valor de: 2 M = sen are eos, A) 1 B)2 C)3 + cos'l are sen. D)4 E)5 T18 176 Problemas de Trigonometría y cómo resolverlos ‘^TtACSO
40.- Evaluar M si se sabe que: M = are cos are tan A) 1 B)l/2 C)l/3 D)2/3 E)3 46.- Determine el valor de M, si: 2 tan2 are sen— -3 sen2! are tan -y- M — 4 eos2 (are eos ) A) 3/5 B)5/4 C)2/3 D) 1/4 E) 1/3 41.- Encontrar el valor de: 47.- Calcular el valor de W, si: M = sen[2 are tan(cos (2 are tan -Js ))] 12 -13 -9 -12 5 A>Í3 B> 15 C>10 D> 13- E)l3 42.- Calcular el valor de: f 12 A f 8 A 13senl arelan ~ I + 17cos I arccot ~ I A) 10 B)20 C)30 D)40 E)50 43.- Identificar el valor de: ( ! 1 1 A M = cos I are sen— + are cos — - are tan — I Js J3 Js a) b) y C) D>V E)^ W = are tan V2-1 7 - are tan A) f B) y Q-J D)y E)J ore sen x a 48.-Si: ------------=—, are cos x b determine una expresión para are sen x. A) - 2 (a+¿)2 B) ——— ’ 2(a+b') C) - — (* + ¿) D) an 2(a—b) E) an (a-b)2 49.- Determine el valor de: 44.- Evaluar M, si: M = eos4 1 3 are cos — 4 -sen 1 3 are sen — A) 1 B)3 C)0 D)2 E)4 , tan(2arc sen x + are cos x) M ---------------------------- tan(2arc sen x + 3arc cos x) A) 1 B)2 C)3 D)4 E)5 50.- Si se cumple que: are tan x + are tan _v + are tan z = n. determinar el valor de M: x+ y + x+2xyz 45.- Reduc ir la siguiente expresión: A) 3 B)2 C)5 D) 1 E)0 71 51.- Si: 0 <x < —, reducir la expresión: (1—sen2x—cos2x A M = are tan ----------— 1 - sen2x+cos2x j A) 2 B)3 C)1 D)4 E)5 A)x B)2x C)-x D)-2x E)3x Funciones Trigonométricas Inversas 1177
52.- Determine el valor de: M = Jn2-In-arc eos x + .larc sen x- V V 2 A)2n B)7i/2 C)3tc D)7t/3 E)n cot(3arc eos x + 2arc sen x) cot(3orc eos x + 4crc sen x) A) 2 B)-3 C)1 D)5 E)-l 58.- Evaluar W, si: 53.- Calcular el valor de M: 7 _ i n — ¿nxirc eos x+ Jare sen x---- W = sec2 3 arc cot ~ - arc cot 3 are tan x + arc sen(x — 1) A) M 120 A) 1 B)2 C)3 D)4 E)5 54.- Determine el valor de 6, si: 1 4^3 6 = are eos ~ + creeos- 7 7 A) | B)f C)| D)~ E)^ 127 E>M121 59.- Determine la expresión equivalente de: W = arc cos(8x4 - 8x2 +1), y su correspondiente restricción. 55.- Si se verifica que: 1 1 x — arc tan 1 + arc tan — + arc tan — A) 4 arc sen x, si x G °’ 2 determine el valor de: “sen x” B) 2 arc sen x, si x G A) 1 B)2 C)3 D)4 E)5 56.- Determine la siguiente suma: 1 1 1 M - are tan ~ + arc tan — + arc tan — 1 ore tan j . n +/1 + I C) 4 arc sen x, si x G D) 2 arc sen x, si x G E) 3 arc sen x, si x G A) arc tan I—rv ' vi+1 D) arc tan B) arc tan n n2 + 2 E) arc tan C) are tan \n-2j «4 0; 2 60.- Determine la expresión equivalente de: W = 2 arc eos x asimismo su respectiva restricción. 7 A) arc cos(x - 1) 7 B)crccos(2x +1) . sixG [0; 1] ,sixG [0; 1] 57.- Calcular: C) arc cos(2x -1) , si x G [ 1; 0] T18 178 Problemas de Trigonometría y cómo resolverlos *MRACSO
C)arccos(2x- 1) , sixG [l;0] D)arecos(2x + 1) ,sixG [1; I] E)orccos(2x~- 1) , sixe [O; 1] 61.- Encontrar la expresión equivalente de: W = 3 are tan x, asimismo su respectiva restricción. 65.- Evaluar W, si: 1 W=are cot (-1) + yzzrc sec(- v2 ) + 3 are csc(2) a\ 8tt 1 Itt 13tt -p|x 8tl r\ 1 3tc A)b B) 3 C)3" D)T E)“F 66.- Reducir la expresión: A) are tan ) >s* 1*1< ( 2 r - /í B) are tan -—, si Ixl < ^l + 3x‘J 3 C) are tan , si |x| < D) are tan [ |, si Ixl < ~- ^l-3x2J 11 3 c. . | 3x + x2 ] -ij -73 E) are tan I I ’sl 1*1 < jF 62.- Calcular el valor de a: ( cot2-tan2A a = are cotí--------I A)3-n B)4-tt C)4-2tt (0 = 5 eos are tan 7 24 + cot are sen 5 13 + -76 sen(¿zrc sec (-5)) ax 24 20 24 22 24 A) g “) 5 O -j D) -] E) 67.- Determinar el valor de: 0 = tan(3 are tan 5) - cot(3 are cot 5) A) 1 B)0 C)3 D)2 5)4 68.- Si se sabe que: a = are sec el valor de a es: D)4 + n E)4+2tt 63.- Encontrar el valor de 6, si: 6 = are csc (csc 8) - are sec(sec 2) A)n-10 B)3n-I0 C)2n-10 D)3n+ 10 E)tt+9 64.- Determinar el valor de P: P = are cot (cot 4) + are sec(sec 6) A)2n-2 B)3j+2 C)n-2 D)3tt-2 E)2h+2 Funciones Trigonométricas Inversas 'sec4 -16sec2 + 20 A)f B)f C)j D) E)^ 69.- Determinar W, si: W = are cot(3) + are cot(7) + are cot( 13) A) are tan C) «re tan /3\ D) are tan 1 I E) are tan 70.- Simplificar M, si: 2 5 B) are tan 4 M=--------yy czrctaru — U5 A) 1/3 B)3/2 Ql/2 D) 1/4 E) 1/5
SITUACIONES GRÁFICAS 71.- Bosquejar el gráfico de: x n y = 4 . are cos ~ ~ 2 4 Ti.- Resolver la inecuación: 72.- Identificar el gráfico de: I n f(x) = jare tan x+ — A)xg are tan(x) - are cot(x) < 0 A)xg(-°°;1) D)xg(-oo;1] B)xg(-°°;2) E)xg(-°°;3) C)xg(o°;3] 75.- Determinar el conjunto solución parax: are sec(x) - are csc(x) > 0 A) x G ; -1] o [ Jt. ; +°°) B)xg (-°°;-l]o(j2 ;+°°) C) x G (-“; J2 J o f 1; +°°) D) x G ; 1] kJ [ -Jt. ; +°°) E) x G (-0°; -2] O [ J2 ; +<») 76.- Resolver la inecuación: n are sec(2x) > ~ 1 ' 2 . J3 2 73.- De la figura mostrada determine el área de la región sombreada: B)xg 1 2 . 2 Ji C)xG 1 '2 4 J1 2 E)xG A)37tw2 B)9rtH2 C)4tih2 D)5tih2 E)2íih2 77.- Del gráfico mostrado determine la regla 'de correspondencia para «y». Problemas de Trigonometría y cómo resolverlos RACSO DITOMI
B) 2-flZtcos (2'2 2 C) -3 . are cos D) 5n + 4 fx H 5n \2 2/ ' 4 3 íx 1\ 5rt 2 ” cos I 2 2 / — 4 3 E) -^.árceos 79.- Sea la función/definida por: B) are sen í^x + ^j + 77 o \5 5/ lo „ 3 /8 1 \ 5n C) g.«resen (-x + -)-- ™ 7 /8 1\ 3n D)-.«resen ^-x + -)-^ ex 7 /8 1\ 5n E)g.«resen^x + -)-^ n f(x) = 2arc sen(2x- 1) + y, determine el punto en el cual la gráfica de/ interceda al ejex. A> (|;o) D>(^') c>(|;°) 78.- Del gráfico mostrado determine la regla de correspondencia. A) 3 2 - are cos x j_\ 5n 2 2/4 £)(!;) MISCELÁNEA 80.- Sea la función/definida por: /(x) = arctan 2x ) 1 + x2 J determine el máximo valor de/ A)7t/3 B)27t/4 Qn/4 D)7t/2 E)2n/3 81.- Determine el valor máximo de la función/ definida por: 2 2 /(x) = are cot(tan x + cot x - 1) A)7t/2 B)7i/3 Qíi/6 D)íi/5 E)íi/4 82.-Resolver: árceos (V3x) + arc cosx= — A) 1/3 B)3/4 C) 1/5 D)2/3 E) 1/2 Funciones Trigonométricas Inversas T18 181
83.- Determine el valor de “x” en la igualdad: .— n are tan (cot[arcsen -J2x ]} = T o A) 3/8 B)5/3 C)3/4 D)8/3 E)4/5 84.- Determine el valor de “x” si: I 1 are tan x + are tan — + are tan =arc sen x + are eos x A)2 B)1 C)4 D)5 E)3 85.- Resolver la ecuación: A) 1 B)2 Q3 D)4 E)5 86.- Resolver para x: are cos(x) = n. csc (y) A) 1 B)2 Q-l D)-2 E)3 89 .-Si se cumple que: are sen{cos(arctan[cot(arc sec (esex))])} = calcular: W = are sen(cos x+eos 2x) are tan A)1 B)2 C)0 D)3 E)4 90 .- Resolver: are csc|x| > are sec|x| A)xG (—-J2 ;-l] u [1 ; 72> B)xG<-72 ;-l] U [1;72> C)xG<^2 ;1] u[1;72> D)xG <-72 ;-l] O [-J¿ ;1> E)xg(-72 ;1]o[1; V?> 91 .-Sí:xg [-1; 1J, determine el máximo valor de: 87.- Calcular el valor de “x”, si: 3 are eos ~ + 2 are cot 2 = are csc x 25 ...24 20 ...23 „. 25 A) 24 B) 25 C) 24 D) 21 E) 21 88.- Si se verifica la igualdad: are cot cotx.cot = 3x. entonces, ¿qué valores puede tomar “x”?. n y = 3 are tan x - — 4 A)7t/2 B)7i/3 Qíi/6 D)7t/5 E)n/4 92 .- Resolver la siguiente inecuación: 3ti 2 are tan x + are cot x < — 4 A)xg<-o°;1] D)xg<-o°;2> B) x G (-°°; 2] E) x G ; 3] C)xg(-o°;1> B)xg(-^;0) E)xg (3;n> C)xg <2;n) T18 182 Problemas de Trigonometría y cómo resolverlos &RACSO
- i‘GTt aciones e Inecuaciones ‘ jrJT r i gon o rifétri cas Si se tienen dos funciones fyg con una misma variable x, tal que: f(x) = g(x), se denomina ecuación con una incógnita, donde x es la variable llamada incógnita, y los valores que toma x se denomina solución de la ecuación. Una ecuación se llama algebraica, si cada una de las funcionas contenidas en f(x) yg(x) son algebraicas (racional o irracional), donde además una de estas funciones puede ser constante. Una ecuación se llama trascendente, si por lo menos una de las funciones contenidas f(x) og(x) no es algebraica. Una ecuación trigonométrica es de tipo trascendente, cuando cada una de las funciones contenidas enf(x) og(x) son funciones trigonométricas de la forma f./. (ox + b), en donde los términos a, b e R, (a * 0). Si se tienen desigualdades de la forma Af^x) > f.t2(x) v f.tt(x) < f.t2(x) ; se denominan inecuaciones trigonométricas. Para que esta relación se constituya en ecuación o inecuación trigonométrica, la variable debe encontrarse afectada por algún operador trigonométrico. Las ecuaciones trigonométricas son igualdades de expresiones trigonométricas, que se verifican para ciertos valores de su variable angular. 19.1. ECUACIONES TRIGONOMÉTRICAS ELEMENTALES (E.T.E.) QJ Toda ecuación trigonométrica de la forma: ET.(ox + b) =c Se denomina ecuación trigonométrica elemental, donde: i) a, b, c e R ; a * 0 ; c e Ran F.T. ii) (ox + b): arco; x: la variable angular 19.2. RESOLUCIÓN DE LAS E.T.E._______________________________________________ i) sen (ox + b) = c => (ox + b) = kn + (-l)k.arc sen (c) ; k e Z ii) eos (ox + b) = c => (ox + b) = 2 kn ± arc eos (c) ; k e Z iii) tan (ox + b) = c => (ox + b) = kn + arc tan (c) ; k e Z Ecuaciones e Enecuaciones Trigonométricas
19.3. SOLUCIÓN PRINCIPAL Es la menor solución no negativa que satisface a la ecuación trigonométrica. 19.4. RESOLUCIÓN DE LAS ECUAC. TRIGON. EN SU FORMA GENERAL Resolver una ecuación trigonométrica, consiste en reducirla , hasta obtener otra de la forma de una ecuación trigonométrica elemental y luego solucionarla. 19.5. INECUACIONES TRIGONOMÉTRICAS_____________________________ Son desigualdades de la forma: i) f.t](x) + f.t2(x) > 0 v f.t,(x) + f.t2(x) < 0 También pueden ser de la forma: ii) f.t](x) + f.t2(x) > 0 v f.t](x) + f.t2(x) < 0 19.6. RESOLUCIÓN DE LAS INECUACIONES TRIGONOMÉTRICAS QJ Para resolver una inecuación trigonométrica, primero se debe calcular las coordenadas de los puntos de intersección igualando dichas funciones. En el gráfico mostrado se observa que: i) En los puntos A, B, C: f = g ii) En el intervalo {a ; b): f <g iii)En el intervalo (b ; c): f > g Problemas de Trigonometría y cómo resolverlos 454 RACSO wp DITOKBfl
PROB. 1 Resuelva la ecuación: 2 sen x + cot x = csc x PROB. 2 Resolver la ecuación: , x x n sec2 "3 - 2 tan "3=0 RESOLUCION RESOLUCION En primer lugar hay que analizar si la ecuación está o no definida para algunos valores de la variable angular. Esto lo determinamos transformando a senos y cosenos: 2senx4- cosx = —-— senx senx De esta relación está claro que: senx/ 0 => x*feít;feE Z (*) Efectuando operaciones, se tiene: 2 sen2x + eos x = 1 => 2(1 -cos2x) + cosx = 1 => 2 - 2cos2x + eos x = 1 => 2cos2 x + eos x + 1 = 0 Resolviendo se obtiene: cosx =-1/2 v cosx=l 2íi => x = 2kn ± “3“ ¡ x = 2 fot; .V/? e Z Pero de acuerdo con (*) se sabe que:x * kn => x = 2kn, no es solución En primer lugar las funciones sec y tan deben estar definidas, lo cual requiere que: COS *0 => y ?t(2£ 4-1)71/2 -> x^(2k+ l)"y ; Vfce Z Luego, en la ecuación dada, aplicamos la identidad pitagórica de la sec x, así: 1 4- tan2^ -2 tan =0 =* (tanf-l)2=0 => tan - 1 = 0 => tan = 1 => = fot 4- ti/4 x = 3 kn 4- PROB. 3 Calcule la suma de soluciones de la siguiente ecuación: cos6x= p + sen6x\ csc2y; yXe [0;Tt] \ sec2x / Ecuaciones e Enecuaciones Trigonométricas T19 185
RESOLUCIÓN RESOLUCIÓN Expresamos todo en términos de seno y coseno y tendremos: Transformando a producto cada miembro de la inecuación, se obtiene: cos6x = l + sen6x 1 cos2x 1 sen 2x _ 3x x _ 3x x 2sen-2~ cos y > 2cos -^“cosy Sr x 3x x sen-^- cosy > cos-^- cos'2’ De aquí se deduce que admitidos son: los valores no De acuerdo con la condición, se tiene que: cos 2x^0 sen 2x^0 2 2 Es decir: kit 2 x*^p, VfeeZ , 3x xZ 3x X Luego: sen-^-co^y > cos-^.co?^ „ „ cos2x(l + sen6x) Entonces: cos ox = -------5---------- sen2x 3x 3x sen~2~ > cos~2~ => sen 2xcos 6x = cos 2x+ sen 6x cos 2x Graneando las funciones de cada miembro: sen 2x cos 6x - cos 2x sen 6x = cos 2x /(x) = sen~2~ A SM = cos-^- sen(2x - 6x) = cos 2x yi sen(-4x) = cos 2x -sen 4x = cos 2x -2sen 2x cos2x = cos 2x Como: cos 2x * 0 => sen2x = -1/2 -1 1 0 Kx) = sen-^ «tó =j “sy Dadoque: xe[0;ir] => 2xe [0;2n] , . . . 7n Un Luego las soluciones son: A Según la figura, la solución es (x|;x2) • calculemos: x, a x2. Esto requiere hacer: PROB. 4 _ 7n llrt Suma- 12 + 12 3n 2 3x 3x sen 2 — cos 3x , tan “2“ = 1 3x , n ~2~=fíJc+ 4 3x = 2£jt+ y Resuelva la siguiente inecuación: sen 2x+sen x> cos 2x +cosx; Vxe [0;n] 2bn n 3 + 6 T19 11861 Problemas de Trigonometría y cómo resolverlos RACSO
7t 5ít => X1 = 6 A x2 = g ••• PROB. 5 Resolver la siguiente ecuación trigonométrica: 2 3cot x + 5 = 7 cscx PROB. 6 Determinar x en el sistema: x + y=f ..(1) 2 sen2 = eos 2x .. (2) RESORCIÓN RESOLUCIÓN Como intervienen las funciones cotang 'i le y cosecante las restricciones son: cotxa cscx => x*kn,k e Z Como en el problema anterior, nuestra estrategia consistirá en elaborar una expresión en términos del seno, por lo cual apelaremos a las identidades trigonométricas. Veamos: 3cos x 5sen x _ 7senx 2 + 2 2 sen x sen x sen x 2 Como: sen x * 0, podemos simplificarlo: 2 2 3 eos x + 5 sen x = 7 senx 2 2 Pero: eos x = 1 - sen x => 3(1 - senzx) + 5 sen2x = 7 sen x Efectuando: 2 sen2x - 7 sen x + 3 = 0 Factorizando: (sen x - 3)( 2 sen x -1) = 0 i) senx-3 = 0 => sen x = 3 (No puede ser) ii) 2 sen x -1 — 0 => sen x = =* ®P= 6 x = kn +(-l)k ,VéeZ b Nuestra estrategia consistirá en obtener una ecuac on en términos del arco «y», para lo cual haremos las siguientes transformaciones: De la ecuacióii (1), se deduce que: 71 x + y = 2 => 2x + 2y = n Por reducción al 1C se puede establecer que: eos 2x = - eos 2y (3) Reemplazando (3) en (2): 2¿ y A 2 sen I 2 I = ' cos => 1 - cos y = - cos 2y Pero: cos 2y = 2 cos y -1 => 1- cos y = - (2 cos2y -1) => 2 cos y - cos y = 0 Factorizando nos queda: cosy (2 cosy -1) = 0 Igualando a cero cada factor, analizamos teniendo en cuenta la condición dada: i) cos y = 0 => y = 2kn ± n/2 Luego:x =7; -y => x = => x = -2kn ó x = n - 2kn Ecuaciones e Enecuaciones Trigonométricas
ii) 2 eos y -1=0 => eos y = => y = 2kn ± • - -(*) Reemplazamos (*) en (1): x= 2 ’ (2fel±f) => x = v - 2*it ó x = ^? - 2Ztrt o O PROB. 7 Resolver la'inecuación trigonométrica: sen 3x - sen x > 0, si: x e (0; n) RESOLUCIÓN Del arco triple tenemos: senx(2 eos 2x + 1) - sen x > 0 => 2 sen x eos 2x + sen x - sen x > 0 => senx. eos2x > 0 ... (1) Por condición: 0 < x < n => 0 < sen x < 1 Reemplazando en (1) tenemos: eos 2x > 0 De la C.T. se observa que: ESTRATEGIAS DE RESOLUCIÓN 1) La ecuación tiene que estar definida en el campo de los números reales. 2) Obtenga los valores donde la variable no está definida, para no tomarlos en el conjunto solución. 3) Reducir a su mínima expresión a la ecuación hasta obtener de la forma: f.t (ax + b) = c 4) Si se trata de una inecuación es necesario graficar las funciones para visualizar can facilidad el conjunto solución. Problemas de Trigonometría y cómo resolverlos RACSO 1DITC11I
Enunciados de Problemas con Resolución ECUACIONES TRIGONOMÉTRICAS 01.- Determine el conjunto solución de: 7 X 4 sen + 3 tan x = 2 A)2ta + (-l)k-T D) O J 3 B)2ta+(-l)ky E)ta+(-l)ky C)ta + (-l)ky 02.- Determine el conjunto de solución de la ecuación trigonométrica: cos 7.x - 1 + tan x = 0 \ 4/ o \ 4/ 3 + E)pta-^juta C) íta+—luta \ 4/ 03.- Determine un conjunto solución de: 7 7 3 cos^x - sen 2x - sen~x = 0, si: tan x > 0 A)ta+^ B)ta-j C)ta-^ D)tay E)ta + ^ 04.- Resolver la siguiente ecuación trigono- métrica: 1 > 1 + cos x = ~ . sen".* A)| B)^ C)J D)ti E)k^ 05.-. Resolver la siguiente ecuación trigono- métrica: 3 3 1 sen x. cos x - sen x. cos x = ~ 4 A) (4tal)^ B)(3A-l)g C)(2A-l)y D)(2¿+1)£ E)(4Á-1)^ O o 06.- Resolver la ecuación trigonométrica: cos3x cos2x i senx tanx y determine la mayor solución negativa. A)-3 B)-^ C)-6 D)-g E)-^ 07.- Determine un conjunto solución de: 7 sen 5x + sen 3x + sen x + 4 sen"x -3 = 0 A)(4m+l)g B)(4/n+l)J C)(4W-1)^ D)(2/n+l)j E)(4/n-l)^ 08.- Determine el conjunto solución de: sen x. sen 3x - sen 2x. sen 4x = 0 A) -y B) C) y 2kn ... 3kn Ecuaciones e Enecuaciones Trigonométricas
09.- Resolver la ecuación trigonométrica: sen3x cos3x +------= 4 cos 4x senx-----------cosx 217t D. kn ™ kn 3fat kn A)— B)3 Q-t D)~5 E)4 10 .- Determine el conjunto solución de la ecua- ción trigonométrica: senóx cosóx —— = csc2x + —~ senzx cos2x a x kn . .. k 7T tx\ kn . . .k 7T A)T + í1)12 D) T + (1)-6 B)^+ (-!)“. J E)^ + (-l)k^ C)^ + (-l)k.^ 11 .- Determine el conjunto solución de: cos4x+ 2sen2x + 3 = 0 A)^ + J(-l)k 4 4 J(-Dk D) An . ^(-i)k 2 4 E)^ +J(-l)k 12 .- Determine el conjunto de solución de la ecuación trigonométrica: 5 sen x + 6 sen~x . cos x + 5 cos x = 4 . kll . ,.k 11 A) -y + í-l) -4 D) -y + (-ir-g B)y + (-l)k.| E)2^ +(-Dk-| Qy + (-l)k-f 14.- Determine el conjunto solución de: (sen 2x + -»/3 cos 2x)2 - 5 = cos - 2. A) * (41 + 3) D)~^(12k+7) B)|(41-3) E) ^(121-7) C) ^(121 + 7) 15.- Determine el con junto solución de: |sen 2x| + 2 = 2(|sen x| + |cos x|) * x 2171 kn kn 2kn kn A)_3~ B)T C)-4 D) 5 E)T 16 .- Determine el conjunto de solución de la ecuación: J3 senx + cosx= -J2 A)2kn± ~ B)kn± 7 + ? 4 3 C)2¿ti±^ D)¿7r±7 + O J E)2^±^ + f A)(2Á+1)J B)(l+l)| C)(¿+1)J D)(2¿-l)g E)(2¿-1)J 17 .- Resolver la ecuación trigonométrica: 2 _ I &J2 cos2x — V1+V2 13.- Determine el conjunto solución de: cos A)¿7T±£ o D)2kn±^ B)2Ati±7 O E)kn±^ C)kn±^ T19 190 Problemas de Trigonometría y cómo resolverlos RACSO PBDITOtai
18 .- Resolver la ecuación trigonométrica: 4 4 . n eos x - sen x - 1 = 0 A) C)^ D) ^y E) kit 19 .- Resolver la siguiente función trigonométrica: 1 2 1 - sen x = . eos x A)(¿-1)^ B)(4¿+1)^ C)(4¿-1)J D)(4¿-1)£ E)(4¿-l)y 20 .- Determine el conjunto solución: 3+75 sec x + tan x =---- 2 A) ¿Tt + are sen j B) k + tire eos C) y + are sec i) D) ¿7t + are tan 71 E)k~^ + are cot O 21 .- Determine el conjunto solución de: 1+cotx 1-tanx = eos 371 .. ¿71 371 n. ¿71 371 ¿71 371 A>T + T B)T-T Qy-y D) y - y E) y + y 22 .- Determine la suma de las soluciones en: xe[O;27t), de la ecuación: senx = 73 cosx - 1 a \ 7t ... 571 7t 371 —. 7t A>2 B) y Og D) y E) g 23 .- Determine la suma del menor ángulo posi- tivo y el mayor ángulo negativo que verifica: eos 2x - eos x - sen x = 0 a \ 3tt ti \ Tt ti 4rt ... 571 A)y B)- C)2 D) y E) y 24 .- Determinar la menor solución positiva de la ecuación trigonométrica: sen 2x + 2 eos 2x. sen x = 0 A)| B) C)§ D)J E)^ 25 .- Resolver la ecuación trigonométrica: 2 eos 4x. eos 3 a +2 sen 5x. sen 2x=eos 4x+cos 2x y determine la suma de las dos menores solu- ciones positivas. a \ ^7t nx 271 2rt p.. 97t p. 371 A) 5 B) 5 C) 5 D) w E) 1() 26 .- Resolver la ecuación trigonométrica: sen3x tt 2rt 3rt 4rt ------- + 1 = tan — -tan — .tan — .tan — senx---9 9 9 9 y determine la suma de las soluciones com- prendidas en [0; 2it]. 71 A)47t B)3n C)27t D)tt E) 27 .- Determine la suma de las 2 menores solu- ciones positivas de la ecuación: eos x = 72 eos 5x - eos 9x A.37t n 87t „ 771 117t 1371 A)y B)y C) y D) E) 28 .- Determine la suma de soluciones en [0; TtJ de la ecuación: x 2 sen x = tan a x 571 7t 7t i 271 371 A)y B) — O2 D) y E) y Ecuaciones e Enecuaciones Trigonométricas T19 191
29 .- Resolver la ecuación trigonométrica: o 2 6 cot“x - 4cos x=1 y determinar la suma de soluciones compren- didas en ( 0; 7t) A) 7t B)2ti C)3ti D)47i E)5ti 30 .- Resolver la ecuación trigonométrica: 3tanx+tan -2 = 0 y determine el producto de sus soluciones comprendidas en (0; 7t). —L_ p\ 3tt 5n2 A) lg B) C) 36 47T 71^ D> > 31 .- Resolver la ecuación trigonométrica: tan 3x + tan x — 4 sen x y determine la dife- rencia de soluciones en el intervalo de (0; 7i). 35 .- Determine el número de raíces que se ob- tienen al resolver la ecuación trigonométrica: 1 + cos 12x= 2cos 8x, síxg [0;7t/2] A)0 B)2 C)4 D)6 E)7 36 .- Determine el número de soluciones si x G ( 0; 7t) en la ecuación trigonométrica: 2sen2jc+ -J? sen^x+^j- cosx= sen 3x A)0 B)1 C)2 D)3 E)4 37 .- Resolver la ecuación trigonométrica: sen3x. sen 3x + cos3x . cos 3x = cos 2x y determine el número de soluciones compren- didas en [0; 7t] A) 1 B)2 C)3 D)4 E)5 38 .- Si: x G ( -7t; 7t), determine el número de raíces que se obtiene al resolver la siguiente ecuación trigonométrica. tan 2x + tan x = sen 3x. sec x 4n Stt 2n o-h A)V B)f C)f D)- E)f A) 3 B)5 C)2 D)1 E)4 32 .- Determine el número de soluciones en ^0; y j de la ecuación: sen5x+cos2x=senx-2sen2x + 1 A) 3 B)2 C)4 D)0 E)1 33 .- Resolver e indicar el número de solucio- nes para xg de 2C 2 . cos 5x - sen x = cos 4x A)1 B)2 C)3 D)4 E)5 34 .- Determine el número de soluciones si x g [0,47i] en ecuación: 3tanx=cotx 39 .- Determine el número de raíces que se ob- tiene al resolver la ecuación trigonométrica cos2x = 2sen2^-xj;sixG [0;7t] A)1 B)0 C)3 D)4 E)5 40 .- Si x G (0; 7t), determine el número de raí- ces que se obtienen al resolver la ecuación trigonométrica: vers(x) + ex - sec(x) = cos x - 2 sen x A) 2 B)1 C)0 D)4 E)6 41 .- Determine el número de soluciones que se obtienen al resolver la ecuación trigonomé- trica: sen 3x = 8 sen3x, si: x e (0; 2ti) A) 1 B)8 C)3 D)9 E)5 A) 3 B)4 C)6 D)5 E)7 T19 192 Problemas de Trigonometría y cómo resolverlos RACSO iDITOtlI
42 .- Determine el número de soluciones que se obtiene al resolver la ecuación trigonométrica: tan 3x + cot x = tan x + cot 3x, si: x e (71; 2n) A) 1 B)2 C)3 D)4 E)5 43 .- Resolver la ecuación trigonométrica tan ^-xj + tan 2x = 2 y determine el número de soluciones compren- didas en (0; 2n) A) 3 B)5 C)6 D)4 E)7 44 .-Si: re ^0; ción trigonométrica: resolver la siguiente ecua- (1 + cosx). covx= (1 + senx) . versx. A)^ B) j C)| D)g E)§ 45 .- Determine las soluciones en ( 0; 7t) de la siguiente ecuación trigonométrica: 2 sen3x - -Jl sen2x = 2 sen x - -Jl — Jít.471.5711 E)l2’T’T/ 46 .- Resolver la ecuación: . c i . 1371 tan 5x. tan 3x = tan —— 4 y determine la menor solución positiva. A X D X /“'X TXX 371 T7X 27T A)12 B)6 C)16 D)T E) 15 47 .- Determine las soluciones positivas y me- nores de una vuelta de la ecuación: (x/3 + 1) senx +(-Jl -1) cosx= V? 71.271 D 71.271 371.271 A) 6’ 5 B 16’15 C) 17’13 7t . 371 —. 71.371 D)12’T E)6’4 48 .- Resolver la ecuación trigonométrica: sen 2x + cos2x - -1 y determine la menor solución positiva. A)- B)j Qj D)* E)^ 49 .- Resolver la ecuación trigonométrica: 2 3 3 sen x(2 cos x + sen x) = sen x - cos x y determine el cociente entre la mayor y menor solución si xe [0; 2ti]. A) 3 B) 1/2 C)4 D) 1/3 E)5/2 SISTEMAS DE ECUACIONES 50 .- Resolver el sistema de ecuaciones: 71 z.. x+y=2 -(l) sen x = sen y. . (2) A)^-Ati B)^+Ati C)J-*7t D)^-to E)^-to 51 .- Resolver el sistema de ecuaciones: n x + y=- ...(1) sen x + cos x — 1 ... (2) Ecuaciones e Enecuaciones Trigonométricas T19 193
. 71 J 71 l~x\ 1 n 3 71 A) y -kit ±2 D) j -kn ± y ... 7t i 37l 1—X 7t —j Tt B) — + kn ± E) ~n - 2kn ± -r jo Z j f 71 C) J +kn ± ~ 52 .- Resolver el sistema: Tt x-y=~ --(I) a tan x = (2 + -J3 ) . tan y... (2) A)T+G1) 6 + 3 B)^+(-l)k-f QT+(1)k’6+¥ D)^+(-l)kJ+£ T7\ , z i TT n n)5+(1) y + 2 53 .- Resolver el sistema: 3n x-y=Y -(1) 3 tanx.cos_v= 2 ---(2) Indica el conjunto solución de «y». A)17t + (l)k( -|) D)21rt+(-l)k(y) B)21rt+(-l)k ( f) E)317t+(l)k(y ) C)21rt+(-l)k(|) 54 .- Resolver el sistema: x~>=2 ... (1) 1 sen x . sen y = y ... (2) A)x = (l-l)J, y = (l+l)J B)x = (311)y = (21+l)J C)a = (213)^, y = (21-l)J D)x = (41+3)y y = (41+l)J E x =(41-3) y = (41-l)J 55 .- Resolver el sistema: x-y=y (0 senx ^=5 - <2) e indique un conjunto solución de “y": A) y = kn + are sen Tt 2 B) y = kit + are cot C) y = kn + are tan — \5l + 2 D) y = kn + are eos E) y = kn + are tan 56 .- Resolver el sistema: Tt A-y=--...(l) tan x + tan y = 2 ... (2) Indica el conjunto solución de «y». a x 7t rn 2171 n r \ kn n A) 4 ’ g B) 5 '8 C)T'8 p\\ ti p» kn ti D) 5 - 8 E) 3 ' 8 T19 «Tr RACSO DITOBLBB 194 Problemas de Trigonometría y cómo resolverlos
57.- Determine un conjunto solución para “x” del sistema: 2x-3j=~ ...(1) sen 3j. cos 2x =----- ... (2) ? +(Dk-5 2 o o B) 4 +16+(1) 12 q Á5 _ZL + r_nk 5 J 3 8 1 ‘ 6 D) —- — +r_nk u' 4 16 1 ’ 6 kn n . ,.k 7t E)T i+(-!) -j2 58 .- Resolver el sistema 7t x + j=- ...(1) sen x + sen y = 1... (2), indicar un conjunto de solución para «x»: A)2*7t+-j C)3Ati-^ D)¿7t+^ E)fcrc-^ 59 .- Resolver el sistema: x-y= 3 -(I) cosx 1 ----= — - . (2) cosy 2 Indicar el conjunto solución para «y». A)2kn B)^ C)^ D)“ E)Ati 60 .- Resolver el sistema: cos x + cos y = 1 - - - (2) Da como respuesta el conjunto solución de «x». A)j-2Ati B) J +2kn C) | +2kn D) +2kn E) v +^kn O J 61 .- Resuelva e indique un conjunto de solu- ción para “x” del sistema: 2n x + y=~ ... (1) cos 2x + cos 2y = 0... (2) A)(1-1)J-^ D)(3¿+l)J-§ B)(21+1)J+^ E)(31+1)J-^ C) (k+ 1) 4 - 12 62.- Determine los valores que puede tomar “x”del sistema: x+y = 2?t... (1) 2 2 sen x + sen y = 0 ... (2) A)4Ati B)2to C)17t D)^ E)^p 63.- Dadas las siguientes ecuaciones, deter- mine un conjunto solución para “x + y”: Jó sen x + sen y = ... (1) J2 cosx + cosy — ... (2) ., 21:71 71 21:71 71 _. . 71 A) —+ - B) -Q*n+4 2tt ?7T 0)2*71+^ Ejtot-^y Ecuaciones e Enecuaciones Trigonométricas T19 195
64.- Determine un conjunto de solución para “x” del sistema: 44, sen x-cos y = 1 . . (1) T 7 sen\x + cos“y =1 . (2) A)(2Jt+l)^ B)(Jt+l)g C)(Jt-l)-y D)(2A+l)y E)(2¿-1)| 65.- Dado el sistema: senx + seny=l ..(1) 4 cos x . cos y = 3 ... (2) , 68.- Resolver el sistema: tan x +cot y = 2 . . (1) cot x + tan y = 2 ... (2) , e indicar un conjunto solución para “x” D)fcrt+£ O 69.- Dado el siguiente sistema de ecuaciones: 7t x+2y=y - (1) determine la expresión general de arcos (x+y) que verifican el sistema. A)2Á7t±5 B)2¿7t±v C)2Jbi±^ 8 4 3 D)Z:7t± E)te±J 66 .- Resoh er el sistema: tanx+tany=l ---(1) 4 tan(x + y)=y ..(2), indicar un conjunto solución para y. A) 2Ati -are tan QJ B) kn + are tan j C) kn. -are tan j D) 2kn -are tan E) kn + are tan j 67 .- Resolver el sistema: tanx + tany = 1 + -J3 ...(1) cot x. tan y— J3 ... (2) A)Ati-^ B)A7t+~r C) D)*7t + y E)frt-y cos x =---. sen v 2 -..(2) , n tal que: 0 <y < —; determine: (x + y) 71 yj'l 71 71 A) - arc cos B) + are sen 71 V7 C) + are cos tí D) 2 + arc sen ^4 ít v7 E) - arc cos 70.- Si: x, ye (0;—) , resolver el sistema: \ 2/ A-y=^--.(l) tan ‘ x) = tan y... (2) .. 71 771 371 71 A)a“24;j-24 B)jr- 25 ;j-20 4tt 3tt 5tt 3ti c)x-23’j“23 d)*~ 26 ’y~ 26 „ 571 71 E)x- 24’^-24 feí RACSO 1DITO11I T19 196 Problemas de Trigonometría y cómo resolverlos
71.- En el siguiente sistema de ecuaciones, determine la primera solución positiva dex. sen2x + sen2y = ... (2) A) | B) | C) | D) | E) | 72.- Dado el sistema de ecuaciones: 75.- Resolver el sistema: 1 senx.seny = ~ ...(1) 1 cosx. cosy = ~ ... (2), si:xe ye \ 2/ z \ 2 / . . 71 71 A)x = --;y= - B)x- 5 ;y—5 x + y = 0...(l) 2 2 sen x + sen y = 1... (2), , indique el valor de “x’ 0 71 0 71 B) 2 + 4 C) 3 + 4 donde 6, x e A)—-~ 2 4 D) - - — ' 3 4 0 ti C)x=6’>=6 ex TI 71 E)x = -3;y=3 —. TI 71 D)x= ;y=- 76.- Si:x,y e resolver el sistema: 7 1 sen"y - sen x = —... (1) 73.- Si: x, ye (0;—), resolver el sistema: \ 2 / sen2x + sen2y = -J?. ...(1) sen(x+y)=l ...(2) Indica el valor de «y». A) 4 B)5 C)3 D)8 E)6 1 2 cosy - sen x - ~ - - - (2) A) x = 45°, y = 45° B)x= 30°,y = 60° C)x=37o,y=53° D)x=16°,y = 74<’ E)x=24<’,y = 66o 77.-Resol ver el sistema: determinar “y” del si- 74.- Si x, ye guiente sistema: 2 sen x. cosy = — ... (1) 3 eos x. sen y = ~ ... (2) tanx. tan y = !...(!) 1 sen x. sen y = ~... (2) n determinar “y”, si: 0 < y < x < ~ a x Dx Tt 71 Tt 71 A) 8 B)6 C)12 D)3 E)4 A \ ti A) - - are sen J3 5 B) ? + are sen 4 71 C) -¿7 + are sen 8 rx, 71 D) + are sen 78.- Si: x , y e sistema e indicar 0;— , resolver el siguiente L 2j o „ como respuesta: “2x + y . 2 1-x 7t E) 77 + are sen o 3 sen x . eos 2y = a~ + 1 ... (1) eos x . sen 2y = a ... (2) Ecuaciones e Enecuaciones Trigonométricas 197
A)a=^;> = 0 B)x=-~-y= ¿ 82.- En el siguiente sistema de ecuaciones: C)x = O;y = -| 71 E)x = ^;y = O 71 7T D)x-- 3 ,y —3 tanx V3 + 1 tany = ^3-1 7t 6 - (2). 79.- Si: x, y e ^0; y y, calcular el valor de “y” en el siguiente sistt ma: senx. sec y = 1... (1) sen x. sen y = ^ ... (2), . . 71 571 D 771 271 —. 71 571 A) 6 V 6 B) 9 v 9 C) 12 V 12 71 lt „ 71 571 D> ¿ *4 E) 5 vjj 80.- Si: x, ye (0;7t/2), resolver el sistema: 2 sen x. sen y = ... (1) J2 2 cosx. cosy =...(2) a\ 71 71 nx 71 71 A)x=-,y=- B)x= g-,y= — 7T 71 i~\\ 7C 71 C)*- 6 .y- 3 D)x— 3 ,y— 4 E)a- 12 .y- 6 81.-Si: x, ye ^0,^, resolver el sistema: 2senx+3tany = 4-»/3 ...(1) 6 sen x - tan y = 2 -J3 ... (2) A)x= 37°. y=53° D)x= 30° , y=60° B)x=60° . y=60° E)x= 30° , y = 74° C)x= 45° , y=30° 7t — <x + y<n, ¿a que es igual “xly”? A)f B)2 C)2 d>2 E)f 83.- Si: x, y, z son ángulos positivos que perte- necen al intervalo 0;^ ,determine:x+y+z,si: 2 sen x. sen y = . (1) sen y. sen z = ...(2) sen x. sen z = ... (3) A)145° B)125° C)105° D)165° E)155‘ INECUACIONES TRIGONOMÉTRICAS 84.- Determine el conjunto solución de: sen 6x < 5 sen 3x .. /kn. 7t , 2A7t h-4+t B)xe / 3kn n. 2kn C>xe \"5“’5+^ ^7t. 37t kn 3’4 3 D)xe JtaRACSO W^BDITOBLBa T19 198 Problemas de Trigonometría y cómo resolverlos
85.- Determine el conjunto solución de: sen 3x + sen 7x < 2 sen 5x l2kit.it , 2¿7t\ A>xe \~’6+~/ ¡kit. 71 , 5klt \ lklt.2lt , 2A:7t\ D^e\T’T+-5“/ ¡2kit.it , 2¿7t\ E>xe \~T'5+~/ 86 .- Resolver la inecuación trigonométrica: 4 sen2* + 2(^3 - ^2 ) sen*- -Jó >0, si:xe (0;2ti) ¡it.3it\ /4it.5n\ A) x g y > 7 y u y 7 >7 y fit.it\ /it.5it\ E)xe \4 ’2/° \3 ’ 3 / ¡it.3it\ /47t.57t\ c>*e U’TMT’T/ l2it.3rt\ /47t.67t\ D)xe^y,yyo ^-7-’—/ /27t.47t\ ¡4it.5it\ E)xe^ 3 ’ 3/u \ 3 ’ 3 / 87 .- Resolver la inecuación trigonométrica: 2 2 sen 6x + 2 sen 3x > 2, C)xe 21-21 .6’4. 71.571^ .2’12/ D)xe 21-21 .9’3. 571.27? .9’3j E)xe 71 .71 .12’4. u 571.77? .12’12/ 88 .- Determinar el conjunto solución de: |sen x + v/3 cos x| < 1 A) ((2¿-l)-^;(6¿-l)-£) \ 2 0/ B) (^2k-l) -^;(3¿-l) X 4 0/ D) ((2k\)~-,(6k + l) E)((2¿-l)-f;(6¿+ !)--£) \ o iz/ 89.- Resolver la inecuación trigonométrica: 2 eos x - 2 cos x < 0, si: x e [-271; 0] A)xe -27t; -3^\ 2/ u 1 1 0 1 B)xe o..- r71’!/ u1 22-2L1 C4’4J C)xe f 371. 71 V 2 ’ 2. VJ __ 371* . 5 ' D)xe 771. 371 .5’2 )c 7---.0 \ 2 E)xe 771. 37? 5 ’' 2 i u / íi. \ 2’ 3711 7 J 90.- Resolver la inecuación trigonométrica: cos7x<cos 3x, si:xe (0;5 \ Ecuaciones e Enecuaciones Trigonométricas T19 199
91 .- Determine el conjunto solución de: 2tt eos 2.x-eos— <0 A)xe kit; ^?+A:7tl L5 6 J B)xe |jj+/c7t; kñ^ C)xe ^-kn; + kit] D)xe + E)xe 92 .- Determine el conjunto solución de sen3x senx cos3x -----<2 cosx A)xe + + kn,+ kn) B)xe {~kn;^ + kn\ {~kn;^ + kn\ \o 2 / \Z 6 / C)xG ^|-far;^+2far^tj + kn:,+ kn) D)xe ^ + far;^ + far^tj -kn;+ kn) E)xg + ^+far;^ + far^ 93 .- Determine el conjunto solución de: sen 3x. sen3x - eos 3x. cos3x > A. kn n kn n 2kn n a)T±3 B)t±z C)-g-±8 D)ta±^ E>ÍT±4 94 .- Determine el conjunto solución de: 2 1 sec x - 2tan x < — 4 A)xe {arctan±+kn;arctan^ + kn^ B)xe {are tan ~~kn; are tan-|+Á:7t^ C) xe {arecoti-kn;arecot^+kn) D) xe {arccot^ + kn;arccut^ + kn) E)xg {arecot^+kn;arecot— + fat^ 95.- Determine el conjunto solución de: tan^3x+^j > 1 A)xe + B)xg o«e D)xe «•&) E)XG \^’6+Í2/ 96.- Determine el conjunto solución de: tan4x+8tan3x+2sec2x-8tanx- 1 <Q Aire /2L + kn.5n 3fat\ A)XG \12+ 2’12+ 5 / RACSO DITOlll T19 200 Problemas de Trigonometría y cómo resolverlos
D, - ¡ n .kn 5it . kn\ B)aG fe+T’T8+T/ Q__ / it kn _n . kn\ xG \20 2 ’4+ 2 / n> _ / ti , kn.Sn . kn\ D)XG \24 + T’24+T/ 5fct.5ít fat\ E)XG \24"ÍT’24+T/ 97 .- Resolver la inecuación trigonométrica: cosx+cotx --------<0. si:xG<0;27t) cscx A)xe - {27t} C>« (M)-if I D)XG E)xe(X&)-|f) 98 .- Resolver la inecuación trigonométrica: / TC\ cot2x + tanx<2,si:xG v’^/’ 99 .- Determine los posible valores de para que la función: /(*) = r 1 9 . ^|senx|- - |x| se encuentre definida, sixG [-7t; 7t] A)xg (-$;0)u(0;f) B)xg C)XG D)xg (-4po) E)xg H’-t) 100 .- Determine los posibles valores de “x” para que la función: / (x) = sen x - sen 2x + sen 3x sea siempre positiva, si x G (0; 7t) E)« 101.- Si: f (x) = |sen x| y g(x) = 1 + cos x ; en donde: x G (0; 27t), entonces, ¿ para qué valores de “x” se verifica: /(x)>g(x)? A)xg /n.37t' \3’ 2¡ \ - {7t| B)xG I 37t\ _{71j \2 2/ 1 C)xG ln.5n\ \3’ 3 1 > -{f }D)XG \T 47' E)xg ln.9n\ \4'~5/ Ecuaciones e Enecuaciones Trigonométricas T19 201
102.- Resolver la inecuación trigonométricas: sen2x + cos2x<0,si:xG [0;7t] A)xG n. 5n\ 6’ 6/ B)xg n.4n\ >3’ 7/ QxG 7C.57r\ 3’ 6/ D)xg (g. g E)xg \ 9’12j I03.-Resolver la inecuación trigonométrica e indicar la afirmación conecta: cos 2x - 2 sen' >0;si: xG [0;27t] »«(^) IV) xG {271} 106.- Resolver la inecuación trigonométrica e indicar la afirmación correcta: 2 7 sen x + cos~2x> 1,xg fO;7t] 10xG{0;7t} IB)xg[p^ A)IIyIII B) lyll C) IIIyIV D) Ninguna E) Todas menos II A)SóloI B)IyII C) IIIyIV D) Ninguna E) Todas menos II 104.-Si: xG ( 0; 27t), resolver la inecuación trigonométrica e indicar la afirmación correcta: 4 sen2x + 2(^/3 - 1). sen x - -J3 < 0 l)xe !">« [$;*) A)I, IlylV D) Ninguna 107.- Resolver la inecuación trigonométrica e indicar el intervalo solución; cos j - sen (ti/2 - x) > 0. si: xG ( 27t; 47t) /5n.7n\ fin. llitX /77t.I5rt\ A) \ 2 ’ 2 / B) \T’T/ C) \ 2 ’ 4 / __/Stt.IItA _. /?7t. 15ti\ D)\T’~r/ e*\T’^7 108.- Resolver la inecuación trigonométrica: sen4x> cos4x, si xG [-27t; -7t] IV) XG 2rt) B) lyll C) IIIyIV E) Todas menos II 105.- Resolver la inecuación trigonométrica: sen 2x + cos 2x + sen x - cos x < 1, / 7?t. 5ti\ E)*e VT"4/ si:xG‘/— - —\ \4 4/ 109.-Si: x G (0,?) resolver la inecuación \ 2/ trigonométrica e indicar el intervalo solución: 3 3 5 cos x . cos 3x - sen x. sen3x < ~, O T19 202 Problemas de Trigonometría y cómo resolverlos RACSO
Jt.2jt\ / 7t . 5n\ „ /n.3n\ 5’9/ 1;\12’12/ '“'\5’7/ «(H) 110.- Resolver la inecuación trigonométrica: 113.- Si: x G (0; 2ti), resolver la inecuación trigonométrica e indicar la afirmación correcta: (sen x+eos x- 111.- Resolver la inecuación trigonométrica: |sen^| <|cosx|, si:xG (-7t;27t) / 71 . 711 I /37T.77t\ A) XG \ 3’3j ) \T’T/ / 7t\ /5n.3n\ B) XG \ 9/ u \ 6 ’ 2 / C) XG / \ 3’3^ D) XG \ 3) 1 O 2ti^ f Tt \ /5ti .9ti\ E)xG < ¿ 6/ 1 J \ 3 ’ 5 / III)xG 1I)XG (ojj) !V)xg(^;^) A)SóloI B) lyll C) IIIyIV D) Ninguna E) Todas menos III 114.- Si: x G [0; 27t], resolver la siguiente inecuación trigonométrica: sen(3x) - cos(3x) + 2 Jlsenx-l A)xG C)xG E)xG B)xg D)xg n. 3n\ 4’ 4 / Tt. 3ti\ 4’ 4 / (W) 115.- Si: x G (0; 27t), resolver la inecuación trigonométrica e indica el intervalo que define el conjunto solución: sen3x—-J5 cos2x+3cosx+2 112.- Si: x G (0;^), resolver la inecuación \ 4 / trigonométrica e indica el intervalo que define el conjunto solución: sen(2nx) - cos(2tix) > 0, A)^;27t) - {ti} C)(f^)-{7t} E) /3n.7n\ , B)\ 5 ’ 5 / I71! Ecuaciones e Enecuaciones Trigonométricas T19 203
116.- Indique el intervalo solución de la si- guiente inecuación trigonométrica: sen3x ------— < 0, si: x G [0; 2n] l-cos2x A) < n; 2n> B) < 7t; 2n> C) (y; 117 .- Resolver la inecuación trigonométrica e indicare! intervalo solución: |tanx|> l,si:xG (0;7t) A|«[H]-{?| B)xe {*} 118 .- Si: x G ( 0; 7t), resolver la inecuación trigonométrica e indica el intervalo que define el conjunto solución: tan2x-(.j3 + l).tanx + <0 A)(M) D>{W 119 .- Si: xe ( 0; 27t), al resolver la inecuación trigonométrica, se afirma que: 7 7 (tanx- 1) (tan x-3)<0, A)SóloI B) II y IV QinylV D) Ninguna E) Todas menos IV 120 .- Si: x G , resolver la inecuación trigonométrica e indica el intervalo que define el conjunto solución: tan3x ---+l<0 tanx A) (0;t) d>M 121 .- Resolver la inecuación trigonométrica: tan > 1 -cotx,si:xG <0;n) A)»e - {|} B|lf (0;n>- {f} Qxe - {f} D)xe {*.%) - {|} B)- (i?) |1} 122 .- Al resolver la inecuación trigonométrica: cot(j-x) -tan2x>0,si:xG (0;n) se afirma que: I)xe(0;j) njxe^n) HI)xg III) xg (y;2”) >V)« fcí) Son verdaderas: Son verdaderas: A) Sólo I B) lyll D) Ninguna E) Todas C) inyiv T19 204 Problemas de Trigonometría y cómo resolverlos ^RACSO
123 .- Al resolver la inecuación trigonométrica: 5 71 tanx + cotx<tan ,sí:xg (0;27t) se afirma que: I)xG<0;7t) II)xG^;27tJ Iü)x€(f;rt) Son verdaderas: A)lyll B) SoloII C) I y III D) Ninguna E) Todas A)xG r57t.ii7ti rii7t.i37fi E)xG g ' y J 127.- Resolver la inecuación trigonométrica: 1 + tan x < 4 cos~x, si: x G (0; 27t) e indicar la afirmación correcta: 124.- Al resolver la inecuación trigonométrica: JJ sec2x- (Jó+2) .secjr+2j2 <0, si: x e (0; 2ti), se afirma que: I)xG<0;7t) III)xG ;2n) IV) XG (o; -5} Son verdaderas: ”>«(? 5) A)SóloI B) lyll C) IIIyIV D) Ninguna E) Todas menos I 128.- Si: x G (0; 2ti), al resolver la inecuación trigonométrica, se afirma que: A)IyII D) Ninguna B) Todas E) lylll C) lyll -> 1 tan“x--- ___________3 Jó — senx + 2cosx <0 125.- Resolver la inecuación trigonométrica, e indicar el conjunto solución: 2 2** sec x - esc x >-~, si: x e. <0; tt) versAx I)xG ^i^;27t] IIDxg|o;|) Son verdaderas: ">'<= (?•?) A)SóloI B) II y IV C) Eli y III D) Ninguna E) Todas menos II 129.- Si: x G [0; 27t], resolver la inecuación trigonométrica e indicar el intervalo solución: secx + cosx< 0, si: x G [0.; 27t] 126.- Resolver la inecuación trigonométrica: / 37t senx. tanx+cosx< 2 , si:xg (—;27t \ 2 D>(0;í) Ecuaciones e Enecuaciones Trigonométricas T19 205
En este capítulo desarrollaremos la capacidad de resolver triángulos, pero no desde el punto de vista geométrico, si no mas bien utilizando las propiedades trigonométricas que estas poseen. La razón de este tratamiento se debe a que existen muchas situaciones problemicas que se solucionan con menos complejidad aplicando las definiciones trigonométricas, lo cual nos invita a descubrir y establecer nuevos teoremas para nuevos campos de aplicación. Es bastante conocida la aplicación del teorema de senos o ley de senos, así como también el teorema de cosenos o ley de cosenos para triángulos, es a partir de estos teoremas y con la ayuda de los temas anteriormente vistos como: identidades trigonométricas, transfomiaciones trigonométricas,... etc, se deducen nuevas relaciones de gran importancia, los que nos permitirán resolver no solamente triángulos si no, en general, todo tipo de polígonos planos. Sea el triángulo ABC, donde: A, B, C: medida de los ángulos intemos del triángulo ABC. a, b, c: longitudes de los lados del triángulo ABC. 2p: perímetro del triángulo ABC. 2p = a + b + c p-, semiperímetro a + b + c P = R: circunradio del triángulo ABC r. inradio del triángulo ABC ra: exradio relativo al lado a rb: exradio relativo a! lado b r' exradio relativo al lado c 20.1. LEY DE SENOS (TEOREMA DE SENOS)________________ En todo triángulo ABC se verifica que «sus lados son directamente proporcionales a los senos de sus respectivos ángulos opuestos y la constante de proporcionalidad es igual al diámetro de la circunferencia que circunscribe a dicho triángulo». T20 2061 Problemas de Trigonometría y cómo resolverlos
a _ b _ c sen A sen B sen C . . « b c o bien ------, - - -----¡g- =----= 2R sen A sen B sen C De donde: a = 2R sen A A b = 2R sen B A c = 2R sen C 20.2. LEY DE COSENOS (TEOREMA DE COSENOS) En todo triángulo se cumple: “El cuadrado de uno de los lados es igual a la suma de los cuadrados de los otros dos, menos el doble del producto de éstos multiplicado por el coseno del ángulo opuesto al primeriado”. a2 =b2 +c2 - 2bc. eos A b2 = a2 + c2 - 2ac. eos B c2 = a2 + b2 - 2ab. eos C De donde: .2 2 2 ___._ b + c - a eos A =------------ 2 be 2 2.2 cosB= c - - 2ac 2ab eos C = 2 20.3. LEY DE PROYECCIONES (TEOREMA DE LAS PROYECCIONES) En todo triángulo se cumple que: “Uno de los lados es igual a la suma de las proyecciones de los otros dos respet to al primer lado” a = b eos C + c eos B b = a eos C + c eos B c = a eos B + b eos A 20.4. LEY DE TANGENTES (TEOREMAS DE TANGENTES)___________ En todo triángulo se cumple que: “La diferencia de dos de los lados es a su suma, como la tangente de la semidiferenc ia de sus respectivos ángulos opuestos es a la tangente de la semisuma de los mismos” Resolución de Triángulos Oblicuángulos
20.5. R.T. DE LOS SEMIANGULOS scn|| = seny C sen 2 = Kp-b)(p-c) V be cosf = i Ip(p-a) V be tany=i l(p-b)(p-c) y p(p-a) Kp-aXp-c) V ac c°s^ = 1 Ip(p-b) J ac tanl’=^ Kp-a)(p-c) | p(p-b) l(p-aXp-b) c Ip(p-c) tan^=, 2 1 l(p-a)(p-b) V ab 2 1 V ab y p(p-c) 20.6. SEMIPERÍMETRO (p), INRADIO (r) Y EXRADIO (1?) A B C p = 4 R cos "2 cos £ «os y . _ A B C r = 4 R sen sen y sen y . „ A B C ra = 4 R sen eos eos . D B A C = 4 R sen ^' cos ^ eos C A B rc = 4 R sen eos cos-^ De estas relaciones se deducen los inradios: ra=ptany r = (p - á) tan y , B rb=ptan 2 r = (p - b) tan % „ C rc =ptany r = (p-c)tany 20.7. MEDIANA DE UN TRIANGULO ABC 4 m* 2 * 4 = b2 + c2 + 2 be cos A 2 2 2 4 wib = a + c +2 ac cos B 4 mi?; = a2 + b2 + 2 ab cos C 20.8. BISECTRIZ INTERIOR DE UN TRIANGULO T20
20.9. BISECTRIZ EXTERIOR DE UN TRIÁNGULO . _ 2 be „ A “ |b-c| Se 2 2ac rb = TTT7 senf 20.10. ÁREA DE UNA REGIÓN TRIANGULAR c be . S = “2" sen A S = sen C S = 2R2 sen A sen B sen C _ _ abe 4R S = Jp(p - a)(p - fc)(p- c) 9 A R C S = r cot „ cot “ cot^y L L 2. A. B. C S=p tan tan -y tan L L S =p(p-a)tan S = p(p - fc) tan S =p(p-c~) tan y S=ra(p-a) S=rb(p-fc) S = rc (p - c) _ 2ab । • sen _ S = “ sen B S = 7^a^bAcR sen 45" Siendo: «S» Área de la región triangular 20.11. ÁREA DE REGIONES CUADRANGULARES _ (AC)(BD) S = -——- sen a 4 C __ , B + D ______ . 2 — 9 v 2 — Si: => S = ^(p-a)(p-b)(p-c)(p-d)-abcd cos2<|> Resolución de Triángulos Oblicuángulos T20 209
20.12. PARA CUADRILÁTEROS INSCRIPTIBLES Si se cumple que: A + C=180° ó B + D = 180° => <> = 90° Luego: S = - aXp-b)(p-c)(p-d) En este tipo de cuadriláteros se cumple que: «El producto de sus diagonales es igual a la suma de los productos de sus lados opuestos», así: (AC) (BD) = ac + bd (Teorema de Ptolomeo) 20.13. PARA CUADRILÁTEROS CIRCUNSCRIPTIBLES Se cumple el Teorema de Pitot: a + c — b+d => p = a+c 6 p = b + d Luego: S = -Jabcd sen $ 20.14. PARA CUADRILÁTEROS INSCRIPTIBLES Y CIRCUNSCRIPTIBLES Para éstos cuadriláteros = 90°, y se llaman Bicéntricos. Luego: S = -Jabcd Siendo: «S» Área de la región cuadrangular '.4RACSO IfiDlTDlll T20 210 Problemas de Trigonometría y cómo resolverlos
PROB. 1 a) Las medidas de los tres lados. Se tiene un triángulo ABC, en el que su lado mayor es el doble del menor, y su menor ángulo es la tercera parte del mayor. a) Calcular la medida de los tres ángulos del triángulo ABC. b) ¿Qué clase de triángulo es? b) El valor del coseno del mayor ángulo. c) El área de la región triangular. d) La medida del inradio e) La medida del circunradio. RESOLUCIÓN RESOLUCIÓN Elaboramos una gráfica: C 2a 3a (n + 1) Por ley de senos: (n + 1) _ n-1 sen2a sena serra B Apliquemos la ley de senos: a _ 2a sena sen3a " + l = n-l => cosa = 2cosa Pero por ley de cosenos: " f*) 2(n-l) J sen 3a = 2 sen a (n - l)2 = (n+1)2 +(n)2 - 2 (n+1) (n) cosa => sena (2cos2a +1) = 2sena 2cos2a +1=2 => 2cos2a = 1 2n(n+l)cos a = (n+1)2 - (n -l)2 + n2 2n(n+ l)cos a = 4n + n2 ...(**) cos2a = 1/2 a = 30* Reemplazando (*) en (**), tendremos: Luego: a) A = 60°; B = 30°y C = 90° ^(n + OJí7P0 =4n+n2 b) El triángulo es rectángulo. <n + l)2 (n-1) (n + 1)2 t .. fcü =(n+4) PROB. 2 Los lados de un triangulo son números enteros y consecutivos. Si su mayor ángulo es el doble del menor, determine: (n+1)2 = (ñ + 4) (n -1) n2 + 2n + l=n2 + 3n-4 => n = 5 Resolución de Triángulos Oblicuángulos T20 211
Luego el triángulo es así: RESOLUCIÓN a)a = 5;b = 6;c = 4 b) (6)2 = (4)2 + (5)2 - 2 (4)(5) cos B 40 cos B = 16+25-36 => 40 cos B = 5 cos B = 1/8 c) Cálculo del área (S) Aplicando la ley de senos (la proporcio- nalidad en dos triángulos que tienen los mismos lados). m m s = 7p(p - o)ÍP - ¿>Xp - c) ÍP = sen 2a _ sen 5a sena sen 2a 2senacosa _ sen5a sena sen2a => 2 sen 2a cos a = sen 5a /15 5 3 7 Y 2 2'2'2 Transformando a sumas el primer miembro: sen 3a + sen a = sen 5a => sen a = sen 5a - sen 3a Transformando a producto el 2do. miembro sena = 2cos 4a sena => 1 = 2cos 4a cos 4a = 1/2 => 4a = 60° a = 15° d) Se sabe que: S = p r e) Se sabe que: S = ~¡^ PROB. 4 AI resolver el siguiente triángulo donde AM es mediana; el ángulox es: 15 = 5 + 6 + 4 4 4/? => 15R = 5x6x4 => 15R=15x8 PROB. 3 A partir de la figura que se muestra, calcule la medida del ángtho a. BMC Problemas de Trigonometría y cómo resolverlos RACSO JDITD1II R = 8
m n RESOLUCIÓN sen 15° senx En el AABM aplicamos el Teorema de los Senos: Igualando (1) y (II), tendremos: sen30° _ sen!5° sen(45°+x) ~ senx sen(45 + x) _ sen30' senx ~ senl5* Desarrollando el sen de (45° + x) y efectuando como sigue, deducimos: m =n sen 30° sen(135°-x) J2 /cosx + senx) _ [ 2 \ senx I I 4 m _ sen 30° n sen(45°+x) -CO => cot x +1 = -J3 + 1 AAMC: aplicamos el Teorema de los Senos: Finalmente: cot x = J3 m _ n senl5° — senx De donde: x = 30° ESTRATEGIAS DE RESOLUCIÓN 1) Cuando en un triángulo se conocen sus tres lados y se desea calcular sus ángulos, se aplica ley de cosenos. 2) En un triángulo cuando se conoce la relación de lados y ángulos, se aplica la ley de senos. 3) Si se tienen dos triángulos de lados iguales, los senos de sus respectivos ángulos son proporcionales. 4) En cualquier triángulo, tanto sus lados, perímetro, inradio; área, se expresan en función del circunradio y los ángulos. Así: a = 2R senA ; b = 2R senB ; c = 2R senC ¿r, A B C p = 4K cos ~2 eos eos r = 4Rsen-y B sen7 C sen T at} A B C ra= 4Ksen-^ eos eos -j ZD B rb= 4R sen ^2 A cos~2 C COSJ- C A C rb = 4K sen-? cos~2 cos ~2 S = 2R2 sen A sen B sen C Resolución de Triángulos Oblicuángulos
Enunciados de Problemas con Resolución LEY DE SENOS OI.- En un AABC se cumple: BC = a , AC = b . AB = c; a senA calcula: M = ~ -----— b senB B>T 3 4 D)< E)| 05.- En un AABC se cumple: BC = a , AC = b y AB = c, reducir: M = «Z?c.sen C.ícot A + cot B) A) ab B) ac C) c3 D) ¿>3 E) fi3 A)0 B)1 C)2 D)3 E)4 02.- En la figura mostrada determine el valor deje: 06.- En un AABC se cumple: BC = a, AC = b, AB = c, reducir: Z?senB — csenC M=--------------- 2o.sen(B—C) A) 1/2 B) 1 C)2 D) 1/3 E) 1/4 07.- En un AABC. se cumple: BC = a. AC = b, AB = c, mZB - m ZC = 96° y b + c = a -Jl , determinar: m ZA. .. 1 1 rJ I D) are sen E) •arc cos A) 30° B)37° C)60° D)45° E)53° 03.- Dos de los ángulos interiores de un trián- gulo miden 26 y 66 los lados opuestos miden 4 y 8 respectivamente, determine el valor de 6 A) 12° B) 14° C) 15° D)26° E)30° 04.- De la figura mostrada, determine el valor 08.- En un AABC, de lados: BC = a, AC = Z? y AB = c, reducir. ZtcosB + c.cosC M= cos(B-C) A) Z? B) c C) a+b D) a E) a-b 09.- En un A ABC, se cumple que: BC = 26, AC = 36 y m Z A = 37°. Determine “ cos 2B”. JX XL 81 100 .50 M 50 nx J1 r-' 19 D) 100 E) 100 Problemas de Trigonometría y cómo resolverlos -a TACSO vp BDITOHI
10 .- Dado un triángulo ABC, si el ángulo “B” es el triple del ángulo “C” y AB = 5, AC = 7, determine: “ eos 2C” A) 1/2 B) 1/3 C) 1/4 D) 1/5 E) 1/6 11 .- ¿En qué tipo de triángulo ABC, de lados BC — a, AC - b, AB = c, se cumple que: a b c = =---- ? cosA------------------------------cosB-cosC A) 15° B)18° C) 13° D)20“ E)1(F 16 .- En un AABC se cumple: BC = a, AC = b y AB = c, reducir: M = (a - b) sen C + (b - c) sen A + (c - a) sen B A)-l B) 1 C)0 D)2 E)-2 LEY DE COSENOS A) isósceles C) obtusángulo B) rectángulo D) acutángulo E) equilátero 17.- En un A ABC se cumple: AC = 2 -Js , AB = 3 y mZ A = 60°, determine el lado BC. A)j35 B)>/7 C)J19 D)J17 E)VH 12 .- En un AABC, se cumple: BC = a, AC = b, AB = c y además: eos A cosB cosC c a + b + c ~ a.b ' determine la medida del ángulo C. A) 45° B)37° C)6(r D)53° E)90° 13 .- En un AABC se cumple BC = a, AC = b, AB = c, reducir: senA + senB c—a senB+senC b+c A) 1 B) 1/2 C)2 D) 1/3 E)3 14 .- En un AABC, se cumple: BC = a. AC = b, AB = c y además R. tan A. tan B . tan C =2, donde R es el circunradio, determine: M = a.sec A+ ¿.sec B + c.sec C 18 .- En un A ABC se cumple AB = eos 30, AC = eos 0 y m ZA = 40, determine BC. A) sen 0 B) sen 20 C) sen 40 D) sen 30 E) sen 50 19 .- En un AABC de lados a, b, ye se cumple: 3a = 7c y 3¿ = 8c, determine la medida del án- gulo A. A) 30° B)45° C)80° D) 120° E)60° 20 .- En un AABC se cumple: BC = a, AC = b, 2 7 7 2 AB = c y además: a -b~-c~= ~ be, determine A el valor de tan —. A) 1 B)2 C)3 D)V3 E)>/2 A) 1/2 B)5 C)2 D)4 E) 1/4 15 .- De la figura mostrada determine el valor de “6”, siAB = CD 21.- En un AABC se cumple: BC = a,AC = ¿,AB = c y además: 3( a2 - ¿>2 - c2 ) — 2bc, 2 A determine: eos ~ A) 1/3 B) 1/2 C) 1/4 D) 1/5 E) 1/6 Resolución de Triángulos Oblicuángulos T20 215
22.- En un AABC, se cumple: BC = a, AC = b, lab AC = c, además: (a + b + c)(a+ b-c)= ——, determine el valor de: “cos C” A) 1/2 B) 1/3 C) 1/4 D) 1/6 E) 1/8 A) 45° B)60? C)90° D) 120° E) 135° 29.- Los lados en un triángulo son 3 números consecutivos y el ángulo mayor es el doble del menor, determinar el perímetro de dicho triángulo. 23.- En un AABC, se cumple: AC = fc, BC = a, AB = c, determinar: M = a(b cos C - c cos B), en términos de los lados “£>” y “c” 7 7 7 A) be B)2b~ C)c~-b¿ D) fc2 + c2 E) fc2 - c“ 24 .- Determine el mayor ángulo de un triángu- lo cuyos lados son proporcionales a 7,8 y 13. A) 60? B)105° C) 150° D)90° E) 120? 25 .- De la figura mostrada, determine el valor de“x”. A) 9 B)12 C)18 D) 15 E)21 30 .- En un AABC se cumple: BC = a, AC = ¿>, zí zf /— AB = c y a . cos I 2 I + cos I 2 1 ~ ’ determine el perímetro de dicho triángulo A) >/2 C) V5 B) V5 D)2>/2 E)2V3 LEY DE TANGENTES 31 .- En un AABC se cumple: BC = a, AC = b, A B 1 AB = c además: tan ~ 1 y tan ~ ~ , deter- mine el valor de: M =---~ a + b A) 3/5 B)9/25 C)4/5 D) 16/25 E)9/16 A) 2 B)1 C)3 D)4 E)04 32 .- En un AABC se cumple: BC = a, AC = b. 26.- En un AABC se cumple: BC = a, AC = b, AB — c y se cumple que: a + b = J?. c, deter- minar el valor de “ 1 + cos C” A) be le2 B)^- ab C) — ac f A + B AB = c, a = 4¿> además: tan I I + í A —B'l C tanl 2 I =8, determine el valor de: cot A) 3 B)5 C)7 D) 1 E)2 d)¿ 2ab E) — ac 33 .- En un A ABC se cumple BC = a, AC = b, AB = c, además: b = Se. Reducir: 27.- En un A ABC se conocen: BC = 8, AC = 7 y m Z_ B = 60°, determine el valor de: cot C. A)5/ll B)ll/5 Qll/3 D)3/ll E)ll 28.- En un AABC, se cumple: BC = a, AC = b, AD J - 4 . t4 . 4 o 2,,Z . 2. AB - c y ademas: a + b + c =2a {b + c ), determinar la medida del ángulo A. A)-1/3 B) 1/4 C)l/2 D)l/5 E) 1/7 if^RACSO WBDtTOIBI T20 216 Problemas de Trigonometría y cómo resolverlos
34.- En un AABC se cumple: BC = a, AC = b, AB = c y además: b . cot B = (2c - b) cot A, determine: M = A)2 B)3 C)5 D)7 E)4 40.- En un A ABC acutángulo se cumple: BC-a , AC = b , AB = c ; simplificar: A) 1 B)2 C) D) V5 E) Vi M = ¿c.Vl+cos 2A 4- ac. Vi + cos 2B + ab. 35.- En un AABC, se cumple: BC = a, AC = b, AB = c, m ZC = 60° y a = 3b, determinar: 71 + cos2C A) V2 (a2 - ¿>2 + c2) r~ 7 7 7 D) V3 (a~ + b~ - c ) tan(A- B). A)V2 B)2-j2 D)3V2 E)4V3 C)3VS B) V2 (a2 + ¿>2 + c2) C) V3 (a + b~ + c~) E) -J3 (cZ + b2 + c") ÁREA DE UNA REGIÓN TRIANGULAR LEY DE PROYECCIONES 36 .- En un AABC, se cumple: BC = a, AC = b, AB = c, reducir: í a—c.cosB A M =-------;---- . sec C l b I A) 1 B)2 C)0,5 D)3 E) 1/3 37 .- En un AABC, de lados BC = a, AC = b y AB=c, reducir: a—c.cosB c-fl.cosB .secC M = A) sec B B) sec C C) csc A 41 .- En un AABC se cumple: los ángulos inte- riores de dicho triángulo son proporcionales a los números 1,2,3 y la diferencia de longitudes entre el lado mayor y el lado menor es “21:”, determine el área de la región triangular ABC. A) k2 D) V3 .k2 B) V> .k2 E)2V2 Jl2 C)2.-<]3 J? 42 .- En un AABC se cumple: el área (S) de dicha región triangular es 90 -J3 cm~ y los se- nos de los ángulos A, B, C son proporciona- les a los números 5; 7; 8, determine la medida del ángulo B. D) csc B E) sen A A) 60° B)3CT C)90° D)45° E)12(F 38 .- En un AABC, se cumple: BC = a, AC = b y AB = c, reducir: c.cos( A + C) + Acos( A + B) M=--------------------------- a A)-l B)l C)05 D)-04 E)0 39 .- En un AABC, Se cumple BC = a, AC = b, AB = c y además: a~ + b~ + c" = 10, calcular: M = be. cos A + ac . cos B + ab cos C 43 .- En un A ABC se cumple: los lados son tres números impares consecutivos y uno de sus ángulos mide 120°, determine el área de la región triangular en p . A) -y u2 R, -J3 2 B)~4~u ~ 15V3 2 E)— 3-J3 2 O-5-« Resolución de Triángulos Oblicuángulos no 217
44 .- En un AABC se cumple: 4S = (p-«)(p-fc) + p . (p- c) donde S es el área de la región triangular ABC y p es el semiperímetro, determine la medida del ángulo C. A) 60° ó 120° C) 20° ó 160° B)45°ó 135° D)90° E) 30° ó 170° 45 .- En un AABC se cumple: C 4S.tan ~2 =tib, donde S es el área de la región triangular ABC, C determinar el valor de: tan . A)2 B)3+V3 C)2-V2 D)2-V3 E)2+ -J3 46 .- En un AABC, calcule: 7 2 7 W = a~b c~.sen A sen B sen C en términos del área (S) de la región triangular ABC. A)S3 B)2S3 Q4S3 D)8S3 E)04S3 47 .- En un AABC, calcule: f 1 1 IV W = —r4";—1------en términos del l ab be ac j circunradio (R) y el inradio (r), donde a, b, y c son los lados de dicho triángulo. A)2R.r B)Rr D)3Rr E)0,2R.r C)0,5Rr 48 .- En un AABC, se cumple la longitud del >Í3 circunradio (R) es 13 -y an y la media geométrica de sus lados a, b y c es 2 V5T, determine el área de la región triangular. A) 7-J3 en? B) 14en? C) 16 ^3 en? 0)12^3 en? E)lóV2 ctrT 49 .- En un AABC; calcule: _ «2 b2 c2 W =-------+-------+-----.., tan A tan B tan C en términos de los lados a, b y c de dicho trián- gulo si además el circunradio (R) mide 1 cm A) abe B)a C)b D)c E)a+¿+c 50 .- En un AABC, calcule: (b2 — <?2).senB.senC W =----------——tt-----. en términos del área 2sen(B-C) (S) de la región triangular ABC. A)0,5S B)2S C)3S D)4S E)S 51 .- En un AABC, calcule: W = 2S(cot B + cot C) , en términos del lado “a”, donde S es el área de la región triangular ABC. A) 2a2 B)3a2 C)4a D)a2 E)6a2 52 .- En un AABC se cumple: S = a eos A + b eos B+c. eos C, donde S es el área de la región triangular ABC, determine el circunradio (R) respecto a dicho triángulo. A)04 B)1 C)14 D)2 E)24 53 .- En un AABC se cumple: 2 a eos A + b eos B+c. eos C = — , R. donde R es el circunradio, determine el área de la región triangular en p". T20 218 Problemas de Trigonometría y cómo resolverlos ^RACSO
A) 0,5 B)0¿5 C)3 D)2 E)l/C 54 .- En un AABC, calcule: A B B C A C tan—.tan— + tan—.tan— + tan—.tan— 2 2 2 2 2 2 W- ABC tan—.tan—.tan— 2 2 2 si el perímetro es igual a cuatro veces el radio de la circunferencia inscrita a dicho triángulo. A) 2 B) 1 C) 1,5 D)2,5 E)3 55 .- En un AABC se cumple: S = p . (p - a), donde es el área de la región triangular ABC y p es el semiperímctro de dicho triángulo, de- termine la medida del ángulo A. A) 60° B)45° C)75° D)30° E)90° 56 .- En un AABC se cumple: a sen B + b sen A=c, donde a,byc son las longitudes de los lados de dicho triángulo, dcterm ne la longitud del lado “c" en función del área (S) de la región triangular ABC. A)2-^ B)>/S C)0.5>/S D)3-Js E)4«Js 57 .-En un AABC, calcule: a. sec A + AsecB W=---------------- , csc2A.csc2B en términos del área (S) de la región triangular ABC y el circunradio (R). A)t B)^ C)“íf D)r E)5R 58.- En un AABC los ángulos de dicho trián- gulo miden ti 2n 4n — , ~ y — asimismo el circunradio mide 4 m, determine el área de la región triangular. A)V7/n2 6)2^7 w2 m2 V)0¿Jín2 E)4-/7m2 59 .- En un AABC se cumple: a2.cot A + b~. cot B + c2. cot C = 4, donde a.b, y c son las longitudes de los lados de dicho triángulo, determine el área de la re- gión triangular. A)2 B)4 C)5 D)3 E)1 60 .- En un AABC se cumple: 3 cot A + cot B — 1 ; cos C = — ; el circunradio R = 10 m, determine el área de la región triangular en ABC. A)16w2 B)32m2 C)128»i2 D)64w2 E)68»i2 61 .- En un A ABC se cumple: las cotangentes de sus ángulos interiores son proporcionales a 3.5,7 y dicho triángulo está inscrito en una circunferencia cuya región circular tiene un área de 90 n cm', determinar el área de la re- gión triangular ABC. ..13 ^213 A) 16 B)j7 Q 16 D) V71 en 2 E) -JTl en2 62.- En un A ABC, calcule: ABC W = tan~ . tany . tan — , en términos del semiperímctro p y el área S de la región triangular ABC. A)S.p B)S.p2 C)2S.p D)S. p'2 E)S.p"’ Resolución de Triángulos Oblicuángulos T20 219
63.- En un AABC, calcule: ,,, a+b + c w =-----A----B C ’ cot^.cot’cot-^ 2 2 2 en términos del inradio (r) A)0> B)r C) l,5r D)2r E)2,5r 1 2 2 3 1 A)7 B)y C)^- D)y E) 68.- En un AABC. calcule: (tan2 ~ +tan2 B +tan2 j.S2 W (fcc)2.sen4~+(nc)2.sen4 B+(ch)2.sen4^ ’ 64 .- En un AABC se cumple: ABC r— cot—+cot —+cot—= ky el inradio r= -J3 p, determine el área de la región triangular ABC enp2. A)2A B)3Ap2 C)k D)4k E)0,5Á 65 .- En un AABC se cumple: b+ c = 2a, donde a, b, c son las medidas de los lados de dicho triángulo, determine el área de la región triangular ABC en términos del lado “a" y la medida de su ángulo opuesto A, res- pecto a dicho lado. 2 2 o 2 a)t * A) B) t C)~ 3fl2 A p., 3c2 A D)—7—.cot-^- E) —.tan-^ o z Z z 66 .- En un AABC se cumple: 4A-2 V rb + rb- rc + ra- rc= k>G- dondera. rb, rc son las longitudes de los radios de las circunferencias exinscritas y r el inradio, de- termine el área de la región triangular ABC. A) 0,5* B)* C)2* D)3* E)2,5* 67 .- En un AABC, calcule: términos del inradio (r). donde S es el área de la región triangular. A) 1 B)l,5 C)2 D)2,5 I¿)3 69.- En un AABC, se cumple: el lado BC es igual a 2 -Jl cm, las mediatrices de los lados AB y AC son PS y SM , deter- mine el área de la región triangular PSM en términos de los ángulos A, B, y C. A) senB ' cosA cosB m senA ' cosB.cosC cosB.cosC 1 cosA cos(B-C) ' senA cosB.cosC E) senA 70.- En un AABC, calcule: W = —a , ~, 2bc donde ra, rb , rQ son las longitudes de los ra- dios de las circunferencias exinscritas y r el inradio respecto a dicho triángulo. A) 2 B)l/3 C)3 D) 1/2 E)4 71.- En un AABC se cumple: (rbraXr|-ra) = 2rbrc’ donde ra, rb y rc son las longitudes de los ra- dios de las circunferencias exinscritas, deter- mine que tipo de triángulo es: A) acutángulo B) rectángulo C) equilátero D) isósceles E) escaleno Problemas de Trigonometría y cómo resolverlos .4 kacso WfDITCtll
72.- En un AABC, calcule: ra -r it, -r W =-------+ —, donde r„ rh son las longi- a b a n tudes de los radios de las circunferencias exinscritas y r la longitud del inradio; además rc = 2 cm ye = 4 cm. A)0,5 B)2 C)1 D)3 EJO.25 73.- En un AABC, calcule: r - rb + 2R W = ---------- en términos de alguna razón 2K trigonométrica, si res el inradio, rb es el exradio y R el circunradio. A) eos B B) eos C C) eos A D) sen A E) sen B LÍNEAS NOTABLES 74.- En un AABC, calcule: 2 2 2 ma + mb+mc W = —5—tí-----5~ , donde zn„, mh, zn_ son las a¿ + b¿ + c¿ abe longitudes de las medianas de dicho triángulo. A) 3/4 B)4/3 C)l/2 D)2/3 E)3/2 75 .- En un AABC se cumple: el lado mayor y menor miden 26 cm y 10 cm, además sus ángu- los están en progresión aritmética, determine una de las medianas de dichp triángulo. A) 14 cm B) 14,09 cm C) 15 cm D) 15,09 cm E) 16,09 an 76 .- En un AABC se cumple: 2sen-2 = - determine la longitud de la mediana relativo al lado “o” (ma) en términos de los lados “b” y “c”. 77 .- En un AABC se cumple: la longitud de la mediana relativa al lado “a” es media proporcional entre las longitudes de 2 A los lados “í>” y “c”, determine W = sen en función de los lados de dicho triángulo A) be + be } 2bc D)^Sr" 4bc E)^Í J 4cb 78 .- En un AABC se cumple: 2 3 R .sen A + 2S.cos A = sen A , donde S es el área de la región triangular ABC y R es el circunradio, determine la longitud de la mediana relativa (ma) al lado “a”. A)0,5 B) 1 C)l,5 D)2 E) V2 79 .- En un AABC se cumple: 1 A 1 A Icosy + seny = 13, donde k y q son las longitudes de la bisectriz interior y exterior respectivamente del ángulo A, determine el lado “c” si b > c. A) 1 B)3 C)5 D)7 E)9 80 .- En un AABC se cumple: ... ! Al B 1 C W=I.c°sy + 9.cos 2 +;cos2 • en términos de los lados a, b y c de dicho triángulo, si k, q, t son las longitudes de las bisectrices interiores para sus respectivos ángulos A, B, y C. «x 1 1 A)«+Z> B)-+- ' a c C) 1 + - b c A) b + c B) b - c C) 2bc V)2-Jbc E)-Jbc ^111 E>«+F + e .. a + b + c J abe Resolución de Triángulos Oblicuángulos T2P 221
81 .- En un AABC se cumple: la bisectriz ex- terior relativa al lado “c” mide J2 + 1 cm,m Z A - m Z_ B = 45°, determine el valor de la bisectriz interior relativa al mismo lado en cm. A)0,5 B)2 C)1 D) 1,5 E)3 82 .- En un AABC se cumple: la medida del án- gulo A es 60° y la longitud de la bisectriz inte- rior (Va) es media proporcional entre los seg- mentos que determina sobre el lado BC, deter- mine la medida de los ángulos B y C. A) 100° y 20° B)90°y30° QlKFylO0 D) 108°y 120° E) 105°y 15° 83.- En un AABC, calcule: 2 2 W = 2R. r sen A + r . esc A + r . cot A , en términos del área (S) de la región triangular ABC, donde Res el circunradio y res el inradio respecto a dicho triángulo. A)2S B)0^S C)3S D)2,5S E)S 84 .- En un AABC, calcule: abc.ra R 5 W= S.(p-fe)(p-c) ’ s,: 7 = 2 y además a, b, c son las longitudes de los lados, ra es la longitud del exradio y S es el área de la región triangular. A) 10 B)20 C)15 D)8 E)4 85 .- En un AABC, calcule: W = p[r.cot A + ra.cot B] - a.ra.cot B, en términos de la longitud del lado “c”, donde p es el semiperímctro r la longitud del inradio y ra es la longitud del exradio. 2 A) 4c2 B)3c2 C)2,5c2 D)c2 E) 86 .- En un AABC, calcule: A „ r. cot ^7 = 2a, a z donde ra es el exradio, determine el valor de: r esc A - — a A) sen A B) cos A C) tan A D) sec A E) cot A 87 .- En un AABC se cumple: rb = 3r, donde rb es exradio y r es el inradio, determine cuál es la relación entre los lados a,b y c de dicho triángulo. A)a + fe = 2c B')a + c — 2b C)b-c—2a D)a-b = 2c E)b + c = 2a 88 .- En un AABC, calcule: [A CT tan -y + tan y , en términos de las longitudes de los lados de dicho triángulo, donde rb es la longitud del ex- radio respecto al lado b. A) a B)£> C)c D)2a E)2í> 89 .- En un A ABC, calcule: w= ra(rb + rc)cscA 'b-0'a + rc) donde ra, rb , rc son las longitudes de los ra- dios de las circunferencias exinscritas. A) sec B B) esc C C) sen C D) esc B E) esc A CUADRILATEROS 90.- Sea ABCD un cuadrilátero, O el punto de intersección de las diagonales AC y BD. Si las áreas de las regiones triangulares AOB , BOC RACSO DITOIII T20 222 Problemas de Trigonometría y cómo resolverlos
2 y COD son 1,2 y 4 m respectivamente, deter- mine el área de la región triangular AOD. A) l»i2 B)3m2 C)2m D)4m2 E)5m2 91.- En un cuadrilátero ABCD se cumple: AB = lu, AC =20 u y AD=25 u. Si AD es diámetro de la circunferencia, determine la medida del ladoBC. A) 10 B)12 C)18 D)20 E)15 A tan~ = 2 (p-a)(p-d) (p-fc)(p-c) 97.- Si en un cuadrilátero inscriptible ABCD de lados AB = a, BC = b, CD = c y AD = d, se cumple: a + c = b + d, determine el valor de: •Jabcd M = —r (ab + oc)sen A 92.- Los lados de un cuadrilátero inscriptible son 3,5,6 y 8 cm, determine: “29 sen a”, sien- do “a” el menor ángulo agudo que forman las diagonales. A) 13 B)13a/5 C)75 D)6 E)6^ A) 1/2 B)2 C)1 D) 1/4 E)4 98.- Se tiene un cuadrilátero circunscriptible ABCD de lados AB = a, BC = b, CD = c y AD = d, demostrar que: 93 .- Los lados de un cuadrilátero circuns- criptible ABCD miden AB = 12 u, BC = 25 u y CD=52 u, determine cos (A + C) sabiendo que el área de la región cuadrangular es 650 m2. 5 5 7 7 9 A) 13 B)Í3 C)Í8 D) 18 E) 13 94 .- Se tiene un cuadrilátero inscriptible de a = 1, b = 2, c = 3 y d = 4, determine el coseno del ángulo formado por los menores lados. A) 3/5 B)-3/5 C)5/79 D)3/ll E)-5/7 95 .- Se tiene un cuadrilátero bicéntrico ABCD de lados AB = sen 0, CD = tan 0, BC = cos 0 y AD = cot 0, determine el área de dicha región cuadrangular. .— B D í/fe.sen — = dcd sen — 2 2 99 .- Si ABCD es un cuadrilátero circunscrip- tible que cumple AB = a, BC = b, CD— dy AD = d, demostrar que: B D ÍB+D] S = afe.sen— .csc — .sen z ,donde \ J S es el área de la región cuadrangular. 100 .- En un paralelogramo se cumple la medida de sus diagonales son 2a y 2b y la medida de uno de sus ángulos agudos es “a”, determine el área de dicho paralelogramo, si a >b. A)V2u2 B)x5 u D)4-J2m2 A) ab B) ab cos a C) ab sen a D) ab tan a E) (a2 + b2). tan a 96.- Se tiene un cuadrilátero inscriptible ABCD de lados AB = a, BC = b, CD = c y AD = d, demostrar que: Resolución de Triángulos Oblicuángulos
deja Trigonometría CAP 21 21.1. DEFINICIÓN_______________________________________________________ C = {(x ;y)/z = x + iy ; x,y e R ; i = J-i } Donde: Re(z) = x: parte real del complejo z lm(z) = y: parte imaginaria del complejo z. i = unidad imaginaria que satisface la propiedad. í2 = (o; 1) (0; 1) = (-1 ; 0) v i2 = -1 z = x-iy: conjugando de z, y que verifica la propiedad: z. z . = x2 + y2 21.2. REPRESENTACIÓN POLAR O TRIGONOMÉTRICA DE UN NÚMERO COMPLEJO fXl Si: z = x + iy = (x; y), puede representarse como un vector en el plano complejo (o plano de Gaüss). Del gráfico se observa que: . —2 |z| = |x + iy| = -Jx +y : módulo del complejoz x = r eos 6; y = r sen 0, luego: z = r (eos 0 + i sen 0) Representación polar de z (z * 0) y/k Im(z) (eje imaginario) r: módulo o magnitud de z 0 = argumento de z (Arg z) 0 < Arg z < 2n v -ti < Arg z < n 0 = arg z = Arg z + 2kn ;kez La representación polar o trigonométrica se suele denotar así: eos 0 + i sen 0 = cis 0 T21 Problemas de Trigonometría y cómo resolverlos ¿1RACSO
21.3. REPRESENTACIÓN EXPONENCIAL DEL COMPLEJO z z =re ¡e 21.4. EXPONENCIAL COMPLEJA____________________________ ez = ex + iy = ex(cosy + i seny) Está notación conduce a: c,y = cosy + i seny ; |c,y | = 1 21.5. TEOREMA DE DEMOIVRÉ (cos 6 + i sen 6)n = cos nO + i sen n6 ; n e Z Fórmulas de Euler senz = cosz = De Moivre fue francés de nacimiento. Al trasladarse a Inglaterra, hizo amistad con Nevvton y con Halley, y se dedicó a dar clases particula- res de matemáticas. En 1697 fue elegido miembro de la Royal Society, y poco después de las Academias de París y Berlín. Su trabajo tuvo gran importancia en el desarrollo de las matemáticas actuales y en la aplicación del recién creado Cálculo Infinitesimal y el .Cálculo de Probabilidades. A pesar de su reconocido talento nunca pudo ingre- sara la cátedra universitaria. Su obra titulada Miscellanea analytica es importante tanto para la teoría de las probabilidades como para la Trigonometría, proponiendo su conocido teorema: (cos 6 +sen6)r = cosnO +ísenn6 Y asi mismo propuso: , 2Art±O , . 2Art±O (cosB±sen0)n = COS------ ± /.sen----- Estudio de la Trigonometría con Números Complejos 225
PROB. 1 Dado el complejo z/z = x + iy 2 2 sen + eos z = 4 Demuestre que: a) sen2z + cos2z = I 4(eizXeizJ = 1 2 2 sen + eos z = 4 2 2 sen z + eos z = 1 b) sen 2z = 2 sen z eos z c) sen 3z = 3 sen z - 4 sen3z b) sen 2z = Í2z e -e 2i RESOLUCIÓN sen 2z = (efZ)2-(e ,z)2 Si: z = x + iy, podemos expresar en la forma polar y forma exponencial respectivamente. z = r(cos 6 + i sen 6) ; z = re® sen 2z = 2/ g ¡Z —¡ZIZ —ÍZ "x (e +e )(e -e ) 2/ Puesto que: z = r(cos G - i sen G) ; z = re® sen 2z = De donde: iz -iz e -e 2i e__ 2 —iz (2) z + z = 2reos6 a z + z = r(e,e+ e',e) sen 2z = (sen z) (eos z) (2) osea: 2rcosG = r(e® + sen 2z = 2 sen z eos z e¡& -íe eos G = —— c) e3fz-e-3fz _ (eíz)3-(e fz)3 sendz- 2/ 2i En forma análoga se deduce que: fe'z-t sen3z= <e -<() senG= — 2i „ e'z-e lz sen 3z =---%--- Luego: sen z = ;cosz = x2 2 2 sen + eos z = 2 2 a) sen + eos z = iz e -e 2i iz -iz e -e 2i sen 3z = sen 3z = e'z -e~'z 2i etz -é 2i |(e'z-e,z)2+ 31 iz -iz e —e 2i 2 (2í)2 +3 T21 226 Problemas de Trigonometría y cómo resolverlos
sen 3z = e ¿ [(senz)2(-4) + 3) Z] + z2 = 2 , T — /9 Co'7^4 Z>^41 2 sen 3z = (sen z) (3 - 4 sen z) sen 3z = 3 sen z - 4 sen z PROB. 2 Sean los complejos: Zj = 1 + i a z2 = -Jz e"”174, calcule: a)zt + z2 b)Z]-z2 C) Z, . z2 d) Zj/z2 e) z2 + z3 RESOLUCIÓN Primeramente expresemos Zj a z2 en sus formas cartesianas y exponencial. 2| = l+z => z.= JZeM z2 = => z2 = 1-í a) Para sumar o restar complejos es conve- niente que deben estar expresados en su forma cartesiana o binómica, luego: zx + z2 = (1 + 0 + (1 - í) => Z] + z2 = 2 b) z1 - z2 = (1 + i) + (1 - 0 => z1 - z2 = 2i Sin embargo también pueden estar expresados en su forma exponencial, así: Z] - z2 = JZ (sen n/4)(2z) z, - z2= J2(J2/2)(2D Z] - z2 = (20 c) z1.z2 = (l+í)(l-í) 2 2r z2 = 1_f' z ।. ^2 ~ 2 • z1.z2 = (j2e";4)(j2e^/4) Z].z2 = 2 2 j-\ _ I 14~f í 14~f 0 °JZ2 - [1-f “ U/2 zi = l + f2+2f = 0+2/ z2 2 2 z2 = z2 J2e~in/4 — = cos + i sen = 0 + i z2 2 2 =f z2 e)z?.z3 =(1 +02.(1-03 ¿-z23 =(1 +02(l-02(l-0 ¿Z¡ = [(1 +O(1-O12(1-O Estudio de la Trigonometría con Números Complejos T21 227
z\.zl =(1-í2)2(1-0 = 4(1-0 o también: z1 = J2 e^4 => z2 = 2e"t/2 z2=j2e™ => z¿s=2V2e'-w4 Multiplicado: z2. zf = 4j2 e™2.^™4 z2.z23=4j2e^4 Z]-z3 = 4j2 [cos(-n/4) + i sen(-n/4)] Z2.Z23 = 4J2 = 4(1 -o 03.- Resolver: cosx = 2 RESOLUCIÓN De hecho esta es una ecuación que no pertenece al campo de los números reales, puesto que en dicho campo se cumple que: -1 <cosx< 1 Luego, la validez de la ecuación dada se encuentra en el campo de los números complejos ademas se sabe que: iz . -iz iz -iz cos z = ——— a sen z = ——— 2 2i Luego: e‘* tf" = 2 => eix + éix = 4 (e/jr)2+ 1 = 4(í 'x) => (e“)2- 4(e“) +1=0 => (e“J2- 4(e/x) + 1+4 = 4 => (e"- 2)2 = 3 =* eix - 2 = ± V3 => e“ = 2 ± ix = Ln(2 ± -J3 )=> x = -i Ln (2 ± -J3 ) Es decir: xj = -i Ln (2 + J3 ) x2 = -i Ln (2 - J3 ) ESTRATEGIAS DE RESOLUCIÓN 1) Para realizar sumas de números complejos, éstos deben estar expresados en su forma cartesiana. 2) Para realizar productos o divisiones de números complejos, estos deben estar expresados en su forma exponencial. 3) Si se realizan potencias de números complejos, éstos deben estar expresados en su forma exponencial. 4) Para demostrar cualquier identidad trigonométrica la teoría de los números complejos es una nueva vía para su ejecución. 5) Toda ecuación trigonométrica que no está en el campo de los reales, es solucionado por complejos, teniendo en cuenta las relaciones siguientes: ¡6 -i0 te G_ e +e . o _ e -e — ----ñ---- a sen ti — ------ z zi Problemas de Trigonometría y cómo resolverlos RACSO WlDITOlEI
Enunciados de Problemas con Resolución FORMA POLAR 01.- Determine el valor de: arg(Z) , si: 71 < arg(Z) < — A)1 B)2 C)3 D)4 E)5 02.- Si;Z = 1 + i yfj , determine el valor de Z6. A>T B)^ C)f D)f E)^ 06.-Sean: 7l-2 + i a Z2 = 3 + i. determine: Z,.Z2 en su forma Polar. A) 62 B)70 C)64 D)68 E)65 03.- Sea el número complejo: Z = 6 + V2 2 6-V2 2 determine: M = —• (- -Ji . 7? + 8Z4) 64 A) 5 V3 .cis ? 4 i— 71 B)3<2.cis~ 4 D)5j2.cis~ E)2 JI.cis^ A)5 B)4 C)3 D)2 E)1 Li I t-* |Ct 07.- Si se cumple: ----— = e , determine: a-bt “tan a” 04.- Dada las condiciones: J3.Z.Z Z+Z = -l (1) 71 71 Z = cos 20 + i sen 20 ; 4 <«< : 2 ’ -(2) determine el valor de “0”. A) 4^ crlr 7- 2 crb rx 5ab J a2-b2 8o6 2íz¿ 2 r2 a -b R 2 >2 a -b FORMA EXPONENCIAL * x yxx 17t f i ^7t i?x 7tl A)4~ B)^- C)-^ D)-g- •) 12 08.- Determine el mayor valor de la expresión: M = f 05.- Sea: Z. = i 1 + J3f 1-V3Í 7 determine: Re(Z3 + Z2 + Z) +1 2cos0 - 10 si: Z = e A) 1 B)2 C)3 D)4 E)5 Estudio de la Trigonometría con Números Complejos T21 229
09.- Sea el número complejo: Z = eos 5" - i sen 5" , determine: M ~ -4= (Z"7 + Z 27) •J2 A)-l B)2 C)-2 D) 1 E)3 10.- Reducir: Z = VI 3n . 3n 1 e 8 sen— +icos— (8 8 ) + £1 A) 1/2 B) 1/3 C)1 D) 1/4 E)2 11.-Si se cumple: Z = 2sen0 + 2-?3 cos0-4ísen(o-^j, determine: |Z|. A) 1 B)3 C)4 D)2 E)5 12.- Sea el número complejo: Z = e'21B (1 - e8l°). 1 15 .-Sea: Z+ — = 2 eos 0, determine: M=fz4- -Mi l Z4 J A) -2 sen 40 B) -2 sen 20 C) -3 eos 40 D) sen 40 E) -4 sen 20 16 .- ¿Cuál será el número complejo en forma exponencial que al multiplicar a (i - 1 )4 dé otro número complejo tal que su módulo sea 4 y el argumento 180°. A) | .í ‘2rJ3 B) 4 .e1”73 C) | .ei7t/3 D) ± .e'"73 E) 4 .í’ir23 17 .-Sea: (sen3x + icos 3x) +(icos x-senx) n cosjc + cos3jt Re(Z) determine: M = _ . Ini(Z) A) -tan 30 B) tan 20 C) -tan 50 D) tan 30 E) -tan 20 z —10 1 \ 13.- Sea: W = — - , - n < 0 < 0, determi- 0 ne: |W| .4 0 0 0 _ 2 0 A) g .sen B) 2 ’sen 2 0 'CSC 2 2 0 ^2 0 D) g .tan 2 E) g .sen 14.- Si se cumple: í -•o „12 ?0 . „ 1 2cos~2 4-tsenO 1 =Mcos-2 cisO, determine del valor de “M”: A) 1 B)2 C)3 D)4 E)5 a G ^0; . determine: |Z| A) cscnjr B) secnjr C) tan*jc D) sen”* E) cosnjr 18 .- Sea: Z = sen 82" + i eos 82°, 15 1 determine: W = Z + + 1 A)0 B) 1 C)2 D)3 E)4 19 .- Simplificar: _ [y3(cos31<’+lscn3l°)|'<.[V5(cosl2"+lsenl2")|s M [VTSícosir+isenll0)]3 A)-l 0-1-4 c>-h4 «-í Problemas de Trigonometría y cómo resolverlos AIracso SpiDiioaifl
20 .- Sea: M =--------- + 1, determine la parte real de M. A)-1/2 B)l/2 C) 1/3 D)-l/3 E)2 21 .- Resolver la ecuación: |e"e- 1| = 2,0<0<27t A)7t/2 B)27t Q37t D)n E)ti/3 22 .- Sea el número complejo: Z = sen x - i cos jr, 71 — <x <71, determine: «rg(Z +1) c>f-f D)f+J E)f-j eM + 2eao 7*1 Qm-7- + 1 j - f zJ.-bea .Z- ¡2G |0 e -Le 1 , determine la par- te imaginaria de Z. T A A) -2 cos~0. cot D) -2 cot~0. eos ® A B) l cos~0. cot n E) 2 cos 0. cot C) -2 cos 0. cos B 24.- Si se cumple: (1 + cos 0+isenO)2 - b cos 0. determine el cos 0 +isenO valor de: “2a + b". A) 1 B)2 C)3 D)4 E)5 25.- Si se verifica: 2 + Jí + i=(-Ja + iÍB )cis 0, A> B. determi- ne el valor de: ; (0 < 6 < n/2) A) 2/n B)4/7t C)5/7t D)9/7t E)7/n 26 .- Sea: Z = cos 20 + i sen 20, determine el equivalente de: A) 5 fi+zY M = I j y I + cot“0 B)4 C)3 D)2 EJO senO + icosO 27 - Sea: Z=----—;----- .determine: rzref-Z) senO-tcosO ° A) 30 B)-50 C)-20 D)-20 E)-70 28 .-Sea: Z= sen20 + icos20 y 0 g (0;7t/2), determine el módulo de (Z + 1) A)cos(^-e) D)2cos^-0 B)2csc(j-o] E)3cos(^-ej C) 2 cos -o) 29 .- Si se cumple: t’a+lb — + i, determine el valor de A)//i2 B)//í4 C) -/n2 D)/«3 E) -/n3 30 .- Sea el número complejo: l+cos 20—i sen 20 z = ;----tót7—™ n < G < 371/2, 1-eos 20+1 sen 20 determine: |Z| A) esc 0 B) tan 0 C) cot 0 D)sen 0 E)cos 0 31 .- Si: W = e,Z y Z = re16, determine: |W| A. rsen-0 r sen 26 -r^ cos 26 A) e B) e C) e rseii6 -r.senZG D)e E)e Estudio de la Trigonometría con Números Complejos
32.-Sea: W = e(iZ)2, donde: Z= J5 e° , 6 = are tan ~. Determine: |W| A) e2 B) e’3 C) e5 D) e E) e3 33.- Sea: , . e o l + isen—+ cos— q 9 o 37t Z= g g . — <0<27t , 1 + isen--cos— 2 2 determine: |Z| Ajeos® B)cot® C)cot® n. 6 T7X 6 DJesCg EJseny 34.-Simplificar: 36.- Si se cumple que: cos(4a + p) —tsen(4a + p) cos 2a—isen2a cos(4a+p) + tsen(4a+p) cos 2a + isen2a determine: “A + B” = A.cos(Ba+p), A)5 B)4 C)3 D)2 E)1 37.- Si se cumple: (l + ei2e)4 ^¡46 determine: = A + B cos 26 + C.cos 46, M = B-C A A) 5 B)4 C)3 D)2 E)1 38.- Sea: Z = l+^e + ei4e + fi6e e120 + l ,6<6< 7t 4 ’ A)M = csc6 ; Be <0;Tt72> B)M = cot6 ; Be (0;7t/6> C)M = csc6 ; Be <6;n/9> D)M = cos6 Be <B;7t/4> E)M = csc6 ; Be <B;7t/5) sen46-ícos46 35.-Sea: Z=--------——:----— , sen46+icos46 determine: |Z| A) 5 cos 36 B) 4 cos 46 C) 2 csc 36 D) 2 cos 26 E) 3 cot 26 Uc" 3x) 39.- Sea: Z = i(_6x)—-, determine la parte real de Z. si: arg(Z)=^, determine el valor de “6”. A)^ b>Ít C)íb D)ii E>f A)-1/2 B)-l/4 C)-l D)-5/2 E) 1 REGIONES SOMBREADAS 40.- Determine el área de la región R del plano complejo definido por: R= |ze C/Z. Z < 1 /\0<arg(Z)< y | A) y «2 B) y u2 C) u2 D) f «2 E) u- < ACSO wp KDITOBLBB T21 232 Problemas de Trigonometría y cómo resolverlos
41 .- Determine el área de la región sombreada R, definida por: R=|zeC/|Z|€[2;4]Aar?(Z?)e 0;^l 9 7 7 71 7 37T t A)2w“ B)7tw‘C)27nr D) y w* E) 42 .- Determine el área de la región sombreada por: R={Ze C/l <|Z|<2,0<org(Z3)<37t} .. 5n 2 A) u 2 B) ~2 11 2 C) -y w ... 3n 2 6íi 2 D) y w E) y u 43 .- ¿A qué conjuntos de números complejos corresponde la región sombreada R.? , Im(Z) A) |Zg C/]Z|<2<2, <arg(Z)<7t| B) |zg C/|Z+2|<2, y <arg(Z)<3y | C) |zg C/|Z + 2|<2,-| <arg(Z)<y | D) |zgC/|Z + 4|<2, y <arg(Z)<y | E) |zg C/|Z + 2|<2, y <arg(Z)<7t| 44 .- Determine el área de la región R del plano complejo definido por: R={Zg C/(Z+1)(Z + 1)<1 a 2 ji <arg (iZ2)<37tJ 45 .- Determine el área de la región triangular potencial cuyos vértices son las raíces cúbicas del número complejo: Z = 4-j2 +Í.4-J2 A)2V3 u~ B) 3 V3 u~ C) V3 u~ D)-2w2 E)2y¡2u2 46 .- Determine el área de la región sombreada de la región: R= (Ze C/¡Re(Z)| + |bn(Z)| < 1} A)9m2 B)6m2 C)4«2 D)2m2 E)3h2 47 .- Determine el área de la región triangular cuyos vértices son: Zj = 2e‘,íl/3 , Z2 = 4e'"76 , Z3 = eml2; ubicados en el plano complejo. A)(7 + 2V3)w2 D)y(7+2V3)w2 B) |(2V3 +5)u E)|“2 C)y(2 + 7V3)«2 Estudio de la Trigonometría con Números Complejos
22.1. LÍMITES TRIGONOMÉTRICOS_________________________ 22.1 A Teorema de la función intermedia o teorema del sandwich Si las funciones f, g y h están definidas en algún intervalo abierto 1, donde está contenido el número c, excepto posiblemente el c mismo, y que: f(x)<g(x)<h(x) V xe 1, para lo cual x^c, entonces: Si: entonces: lím f(x) = lím h (x) = l, X >C x—>c lím g(x) = l 22.IB Límites Trigonométricos Fundamentales lím senx x->0 x De donde: 22.1C Teorema lím I™* = 1 x>0 x lím arc senx = 1 x»0 X lím arctan^ = , x >0 X Vxe R diferente de cero, se verifica: I. senpx Zún —-— =p x-»0 x II. tanpx x = P x—>0 x 22.ID Propiedades 1. lím |Z(x) + g(x)] = lím f(x) + lím g(x) x—>a x^>a x^>a II. lím |í(x) . g(x)] = lím fM - lím gW x^>a x—>a x—>a III. lím x—>a f(x) sM lím f(x) f . ; s(x)/0 hm g(x) * x-»a IV. lím c = c, siendo «c» una constante x-»a V. lím |f(x)]n x—>a -n lím f(x) ; V n e Z+ x-»a VI. lím = ni lím f{x) \Vn^Z+;(n>2) x—>a Vx^a RACSO WiDiroiBi T22 234 Problemas de Trigonometría y cómo resolverlos
Las propiedades V y VI tienen la condición de que si n es par, lím f (x) > 0 x—>a 22.1 E El número e Sean las funciones fyg cuyas reglas de correspondencias son: f(x) = (1 + x),/x a g(x) = (1 + l/x)x Para que fyg estén definidas en el campo de las reales, las bases de dichas potencias deben ser positivas y diferentes de la unidad, es decir: 1+x>0a x # 0 ; 1 + 1/x > 0 a x * 0 Bajo estas condiciones los dominios de fyg serán: Df = (—1; 0> u (0;+~) ; Dg = (-~;-l) u (0;+~) Y sus correspondientes gráficas son: x-»0 Luego podemos afirmar que: lím (l+l/x)x =e El número e es irracional, y su valor aproximado es: e = 2,7182 22.2. DERIVADAS TRIGONOMÉTRICAS________________________ Observemos la gráfica de la función f, asimismo observemos la recta tangente (T) trazada por el punto A(x ; f (x)). De la figura se observa que: tanp= n Si hacemos que h tienda a cero (f? —> 0), en el límite ocurre lo siguiente: , f(x + h')-f(x') tana= hrri —------¿ h-+o h Límites y Derivadas Trigonométricas
Si éste existe, diremos que lím es ]a derivada déla función f (x) respecto h-»0 n a x,y se denota como f '(x) , es decir: Si:y = f(x) f(x) = lím h >0 n Nota.- La derivada de una función es otra función, siempre que la función sea derivable. La interpretación geométrica de la derivada es que su valor coincide con el valor de la pendiente de la recta tangente en un punto de una curva. En la gráfica, si f(x) es la función y T es la recta tangente en el punto (c;f(c)), cuya pendiente es mT en dicho lugar, se verificará que: mT = f’(c) 22.2A Teorema Si una función fes diferenciable en c, entonces fes continua en c. Nota: Las funciones f (x) = sen x a g(x) = eos x, son continuas V x e R 22.2B Teoremas y = f(x) (función) f'(x) (derivada) f (x) = c f(x) = 0; V c constante f (x) = xn f’(x) = nxn'1 ; Vx e Racionales f(x) = c.g(x) f’(x) = c.g’ (x); V c constante f (x) = u(x).v(x) f’(x) = u’(x) + f(x) =u(x).u(x) f’(x) = u'.v + v’.u = ;»W*o f(x)=ex f’(x) = ex f(x) =Lnx f’(x) = 1/x ^RACSO WlCITOlM T22 236 Problemas de Trigonometría y < otro resolverlos
22.2C Derivada de las funciones trigono- métricas 22.2D Derivada de las funciones trigono- métricas inversas y=fj(x) y’ = f.f(x) ffy) = senx f’(x) = cosx f (x) = cosx f’(x) — -senx f(x) = tanx f’(x) = sec2x f (x) = cotx f’(x) = -csc2x f(x) = secx f’(x) = secx tanx f (x) = cscx f’(x) = -cscx cotx f (x) = arcf.tfx) f’M f (x) = arc sen (x) f (x) = árceos (x) f (x) = arc tan (x) f(x) = arc cot (x) f(x) = arc sec (x) • f(x) = arc csc (x) rbí)-^ rw- 1 + x2 f’(x) *== |x| Vx2- 1 f(x) = *= |x| -Vx2- 1 22.2E Teorema 1 Si f es una función inyectiva y diferenciable, y f * es su correspondiente función inversa, entonces f* será diferenciable, siempre que f no tome el valor de cero, y se verificará que: 1 22.2F Teorema 2 Sig es diferenciable enx, y fes diferenciable eng(x), la composición (f og) es diferenciable enx, entonces se verifica: -g(x) 22.2G Teorema 3 Si fes una función diferenciable de u, y u es una función diferenciable en x, tenemos que: 4- ~ «X «U «X Límites y Derivadas Trigonométricas
22.3. REGLA DE LA CADENA PARA LAS FUNCIONES TRIGONOMÉTRICAS Sea f(x) = f ./n(u(x)) una función derivable, entonces se verifica que: f’Cx) = n f.tn,(u(xJ) . f.t ’(u(x)) - Según esta regla se tendrá que: (sen ti)' COS ILU’ (cos ti)' -sen u.u' (tanu)’ sec2u.u’ (cot u)’ -csc2u.u’ (sec tí)’ secu.tanu.u’ (cscu)’ -csc u.cot u.u’ (arc sen ti)’ 1 Vl -u2 U (arc cos tí)’ - —* -tz* Vl-U2 (arc tan u)’ 1 t u2 U (arc cot tí)' (arc sec u)’ —! -12' |u|^u2-l (arc csc ti)’ —! u’ |uHu2-1 22.4. LA DIFERENCIAL Según la figura, para «ñ» muy pequeño (f? —> 0) se puede afirmar que: f(_x + tí) - l\x) = h.f ’(*•) Nota. La expresión f (x + h) - f (x) recibe el nombre de incremento o variación de f desde x hasta (x + tí), y se denota por A f: &f=f(x + tí)-f(x) T22 238 P oblemos de Trigonometría y cómo resolverlos A* "IACSO
El producto h. f (x) se denomina diferencial en x con incremento h y se denota por df. df=hf (x) También se suele denotar h = A x, luego: A f = f(x + A x) - f(x) ; df = f' (x)A x Cuando t>x es muy pequeño (Ax = 0), A fy de df son aproximadamente iguales t±f ~ df. f(x + Ax) = f(x) + f(x) Ax S22.4. REGLA DE L’ HOSPITAL£E] 0 00 Se aplica para determinar límites, siempre y cuando éstos sean de las formas q ; ~ llamadas formas indeterminadas. x" gM X™ g'W x‘2 g"(x) 22.5. MÁXIMOS Y MÍNIMOS DE UNA FUNCIÓN________________ Sea f una función cuya gráfica es como la que se muestra, observándose que: En A, B, C, D, E, F y G las pendiente son nulas, es decir f = 0 En B, D y F hay máximos En A, C, E y G hay mínimos En B y D los máximos son relativos En F el máximo es absoluto En A, C y E los mínimos son relativos En G el mínimo es absoluto 22.6. FUNCIONES CRECIENTES Y DECRECIENTES Teorema: Si fes una función derivable en {a ; b), entonces la función fes estrictamente creciente en (a; b), si: Límites y Derivadas Trigonométricos
Calcule el límite de: RESOLUCIÓN c) lím x-»0 b) lím x-»0 x,, sen3x a) lim —-— sen rx l-cos2x ,3 = lim x-»0 _ 4 lím x—>0 Cuando se tienen límites de la forma 77 ; — ; U 00 “0a0.“; también se puedenaplicar derivadas sucesivas (regla de L’Hospithal). Veamos: = 4 lím x->o ( x = 4[7t]3 = 4n3 ->3 senrtx ítr I 4sen3 Ttx „3 PROB. 1 3sennx-senSrtx Si se evalúa cada expresión en x = 0, se obtienen indeterminaciones de la forma 0/0. a) lím x-»0 sen3x o .. —— = 3 hm x x—>0 sen3x „ o.. ; lim < > lim •XX x—>0 3x—>0 a) lím sen3x evaiuaníio -Q _ x->0 X 0 Derivando tanto al numerador como al denominador, se tendrá: sen3x =3 lím 3x->0 ^^ = 3(1) = 3 lím x-»0 3c°s3* = 3(1) = 3 senux b) lím *->0 l-cos2x 2 1 u 0 , evaluando: -g . Derivamoa al numerador y denominador: 0+2sen2x 2x 1 j o evaluando: g sen2x Nuevamente derivamos al numerador y denominador: .. 2cos2x lim ------- x—>0 f = 2 lím cos 2x — 2(1) = 2 1 V-fcí) = lím x-»0 3sennx-(3sen7tx-4sen37tx c) lím x-»0 3sennx-sen3nx 3 .3 RACSO WeDITOKBI T22 [240| Problemas de Trigonometría y cómo resolverlos
c , . o Evaluando: q función f: f’(x), y resolver la ecuación f ’(x) = 0. Derivando: lím x-»0 ÍTtCOSTtX -z^TtCOS37U ti lím x-íO c , . 0 Evaluando: . COS7U-COS37tX Nuevamente derivando: „ í -Ttsenrtx+37tsen3Ttx \ l 2i I n2 lím ( 3 sen 3tu-sen tu 2^ x-»o1 * J Evaluando: . Volvemos a derivar ti2 lím (9rtcos37tx -neostu \ _ ~2 X~M \ 1 / ¡p l(m Í9cOS3tUj2COS7U ~2 x^ \ 1 Evaluándose tiene: 4(90)-»=4(8)=4^ PROB. 2 Calcule los valores de x, donde la función f es máximo y mínimo, usando el criterio de la segunda derivada: f(x} = senx cos x RESOLUCIÓN ****************** f(x) =senxcosx = 2senxcosx 2 f(x) = sen 2x Calculamos los valores críticos de la función A, para lo calculamos la primera derivada de 1 f'(x) = cos 2x = 0 => 2x = (2¿ + 1) => x = (2k + l)|/¡e Z Tt.37t.57t.7Tt. 1 4 ’ 4 ' 4 ’ 4 ’ ”J Para obtener los máximos o mínimos relativos, se debe calcular la segunda derivada de la función f: f ” W, y luego resolver la ecuación: 7”(x) = - 2 sen 2x Apliquemos ahora el criterio de la segunda derivada como sigue: x = = -2 < 0 => fí-vl es máximo 4 (4J i. 4 ) 3rt 3rt^ ,f37t^ - - x = <1 4 I = 2 > 0 => f I “4“ 1 es mínimo 5rt (Srt^ _ X Srt^ , . x = “4"' I I = -2 < 0=> rl 1 es máximo Entonces se deduce que: a) f es máximo en: x= ^;...;(4ft + 1)£ ;*e Z 4’4’4’ v ' 4 b) f es mínimo en: * = ;-y; ;-.-;(4* + 3)^ ;*eZ Su gráfica sería: mínimo mínima Límites y Derivadas Trigonométricos
ESTRATEGIAS DE RESOLUCIÓN 1) En el cálculo de límites (especialmente límites trigonométricos), se debe tener en cuenta que: Ifrn senx = ¡ A senmx = m x x x—>0 x -> 0 2) En el cálculo de límites, si al evaluar se obtienen de la forma 77 , — : se J (J oo deben aplicar las derivadas sucesivas, evaluando en cada derivada que se realice hasta obtener un número real. 3) Si se tiene una función: y = f (x) al calcular la primera derivada f (x) , y luego resolver la ecuación: f’(x) = O lo que estamos obteniendo son los posibles máximos o posibles mínimos. 4) El criterio de la primera derivada nos indica lo siguiente: i) Si f tiene máximo o mínimo, entonces: f’(c) = O o f ’(c) no existe ii) Si f cambia de signo de (-) a (+) en c, entonces f(c) es un mínimo relativo de f. iü) Si f cambia de signo (+) a (~) en c, entonces f(c) es un máximo relativo de f. iv) Si f ’ no cambia de signo en c, f(c) no es máximo ni mínimo relativo de f. v) f es estrictamente creciente en el intervedo (a ; b) Si: f ’ (x) > 0 ; para a < x < b ni) f es estrictamente decreciente en el intervalo (a ; b) si f ’(x) < 0 ; para a < x < b 5) El criterio de la segunda derivada f ’(x) > 0 i) La gráfica de una función f es cóncava hacia arriba en cualquier intervalo 1, si f ’(x) > 0. ii) La gráfica de una función f es cóncava hacia abajo en cualquier intervalo I, si f’ (x) < 0. iii) Sif’ (x) = 0, significa que los valores de x que solucionan la ecuación son los puntos (x ; f(x)) donde la gráfica cambia de curvatura (llamados puntos de inflexi ón) 6) Si f es una función tal que f(c) = 0, y tal que su segunda derivada f'(x) exista en un intervalo abierto que contiene a «c», entonces: i) Si f’(x) > 0, entonces f(c) es un mínimo relativo. ii) Si f’(c) < 0, entonces f(c) es un máximo relativo. 7) Para graficar a una función seguir los pasos siguiente: a) Calcule los puntos críticos, para ello resuelva la ecuación: f ’(x) = 0. b) Calcule los puntos de inflexión, para ello resuelva f’(x) = 0. c) Calcule el signo de f y f’ en cada uno de los intervalos, así como también ver si la función es creciente o decreciente. d) Con toda información de a, b y c realice la gráfica de f en el sistema de coordenadas cartesianas (sistema xy) T22 242 Problemas de Trigonometría y cómo resolverlos RACSO 1DITO11I
Enunciados de Problemas con Resolución LÍMITES TRIGONOMÉTRICOS sen(2cos0) 01.- Calcule: lím - „ 0 COS B A) 1/2 B) 5 C) ti D) 3 E)2 sen(x+/;)-senx 02.- Calcule: lím ~ fi->o h A) sen x B) cos x C) h D) esc a E) 2 cos x n, , - . sen2x—sen2fl 03.- Calcule; hm ~5 A->í/ x^—a Al Sen° FU cot2fl C1 sen2o 2 2a 2a 2 1A. seníz c sen D) ~2a E) "2T ‘ „ ^ + 1 04.- Calcule: lím ,, 2. a > i senil-jT) 3 1 3 A)-j B) 1 C)| D) 2 E)| i . senC¿C + 4-2) LO.— calcule. hiti *-*> X2 4 11 1 A)j B)| C){ D)| CN|*t Wk-Calcule: tó„ AtíSíizA x->0 x -senx A)1 B)2 C) 3 D) 4 E)5 07.- Calcule: 1-sen— lím 2_ X-MT 7t-X A) 1 B) 4 C)6 D)0 E) 2 08.- Calcular: sen(x—2) fe x>-8 A>i B>| C)| D)X E)X 09.- Calcular: (sen3x)(sen5x) «0 (x-xj“ A)0 B)15 C) 5 D) 10 E) 3 10.- Calcular: sen3x Imi -> a-m) 5x”-2x 3 A)-l B)| 3 2 O-i D)-f 3 11.- Calcular: .. sen3x senx-sen2x A) 3 B) 0 C) 2 D) - 3 E)-2 12.- Calcule: , x—sen2x llltl _ x->o x+sen3x A)-| B) 2 1 1 I C) 1 D) 4 E)-| 13.- Calcular: (0 + 4).sen(7t0) lím e->4 0--16 A) 6 B) 2tt : C) O2 D) 7t/2 E)7T Límites y Derivadas Trigonométricas
3sen(nx) - sen(37tx) 14.- Calcule; lím ---------------------- x-»0 X x->0 X A) 4 B) 3 C) 0 D) 1 E)2 n -3x 22.- Calcular: lím -——----- n 1 - 2COSX 3 A) 73 B)V7 C)-V5 D)Vs E)V2 cos(mx) - cos(/ix) 15.- Calcule: lím 2 x—>0 X A) ”l2 n n2-mz O 2+/nz B) 2 2 D) w2~"2 16.- Calcule: 3 A) y B) n + m E)~ l-cos3x : lím , . x—»o l-cos4x 3 9 1 C>í¿ D) 1_ 16 3 E>F6 17.- Calcule: A) 0 B) lím , i x—>0 1 —VCOSX 1 C) 2 D) 3 E) 4 cosx-cos2x 23 .- Calcular; lím —:-------- x->o 1 - cosx A) 4 B) 0 C) 2 D) 1 E) 3 1—cos(l—cosx) 24 .- Calcular; lím -----4— x—>0 X A)| B)| C)| D)i E)| b.senO 25 .- Calcular: lím “ tT e-»o 1—eos v A) 4 B) 3 C) 2 D)1 E) 0 , (2-Veos x-eos v) 18.- Calcule: Inri ----z-------- x—>0 X~ A) | B) | C) | D) | E) j l + cos(m) 19.-Calcular: lún ~ o .. *—»i x^ — 2x +1 A) y B) c)-y D)y E) - Í7L^ — -~L A) y B) Tt C) 2 D) 0 E) 2n 21.- Calcule: lím ----- x—>o 1 —cosóx A>s B)s c>5 D>§ E>¿ 26.- Calcular: A)| B)J 27.- Calcule: 1 2 A)¿ B)^ 6 J 28.- Calcule: A)4 B)1 29.- Calcular; A) -3 B) -2 30.- Calcule: A) -4 B) -3 lím x * 2 1 +senx l + cos2x D>l E>{ lím x-»0 1 —Vcosx x.senx C>3 e4 lím x-»0 1 — cos8x sen8x C) 0 D)2 E) 3 lím r—>0 1 +senx—cosx 1 —senx—cosx C)-l D) 1 E)2 sen2x hm -----------— n COSX + COS3X X— 0-2 D)-l E) 1 Problemas de Trigonometría y cómo resolverlos RfcRACSO Uiditoii
31.- Calcule: (1-senx)3 (l + cos2x)3 A) 32 B) 64 C>ií D>3 E»t 32.- Calcule: Un. sen2x + 2sen2x — 2sen.r l 9 0 COSX COS ~X A) 0 B) 1 C)2 D)3 E) 4 33.- Calcule: x.sen(sen2x) l-cos(sen4x) 1 3 A)| B)- c>í D>l E)1 34.- Calcule: sen(A + B).cosB - sen(A + C).cosC sen(B -C) A) cos(A + 2C) D) cos(A + B) B) cos(A + 2C) E) cos(A + B + 2C) C) cos(A + C) 35.- Calcule: lím x—>0 •J3 —V 2 +cosx sen23x A) Y B)| C)- /3 J3 -J2 2 D)1 E)f(l 36.- Calcule: lím senx — cosx 1 - tanx A’-y B) r~ 41 -41 0 D)-l E) V2 4 37.- Calcule: lím A~>0 tanx—senx x3 A) | B) jL ( D)| E)| l-cos3x jo.- calcule. lím 1 x-»o tan“x 5 4 ^3 1 . 3 A) | B) C)| D)| E)-^ 39.- Calcule: 71 ( TTxA uní —.tan — *-»0 X 2 J A) T B) y C>6 D) 4 E) 2 40.- Calcule: lím (sec x - tanx) A) 4 B) 3 C) 2 D) 1 E) 0 41.- Calcule: lím -seg2 ^-2.tanx v n 1 + COS 4x 4 A>4 B>2 O i D) | E) 1 3sen2x 42.- Calcule: lím *->o x.tan4x A>i B>§ 1 3 2 C) \ D) j E) j tan(l + cosx) 43.- Calcule: cos(tanx)-! A) -3 B) -2 C) -1 D) 1 E) 2 tanx+ tan2x 44.- Calcule: ti cosx + cos2x 3 J3 J3 ,.8 A)V B) C) D)^ E) (tanx+ cotx — cscx)2 45.- Calcule: lím x->o secx—1 2 1 3 2 1 A) - B) C) | D) J E) | |245| Límites y Derivadas Trigonométricas
tan(ox) 46.- Calcule: lím ,, , , , „ x—»o (l-cos(ox) + x)(sec(ox)) A) 0 B) a C) D) 2o E) | x.tanx.secx ... 2x 3 i x- >o (1 + x ).sec x—1 A)f B)7 C)-l D) 2 E) 1 48.- Calcule: X Jt-*o Zflrccosx—71 A)| B)-f C)1 D) E)j 49.- Calcule: 2tanx - nrcsenx lun a->o senx A)1 B) 2 C) 3 D) 4 E) 5 50.- Calcule: lint (1 -x).tan(^) A) - ' 71 B) - 71 C) - D) - E) - 7t 71 7t DERIVADAS DE FUNCIONES TRIGONOMÉTRICAS dv > 51.- Calcule: — , si.- y = sen (3x~ + 2x + 1) dx A) sen 3x2 . (6x + 2) B) cos (3x + 2x + 1) . (6x + 2) C) cos (3x" + 2x + 1). (6x + 2) D) cos( 3x2 + 2x + 1 ).( 6x + 4) E) sen(2x + 1) . (6x + 2) x 71 52 .- Sea: y = sen(e - sen v) + cos~. Calcule: -j- dx A) cos ex- senx. e~ - cosx B) (cos <?x - sen x). (ex - cos x) C) (cos x - sen x).( ex + cos x) D) (ex - cos x) E) cos(ex). (ex - cos x) 53 .- Sea: y — 3 sen4(sen x + 5) + 1, , dy determine: — dx A) 6 sen4(sen x + 5) . (cos x)) 3 B) 12 sen(sen x + 5). (cos x) 3 C) 7 sen (sen x + 5 . cos x) 3 D) 12 sen (sen x + 5). cos(sen x + 5)(cos x) E) 10 sen4(sen x+ 5). (cos x) rfy 54 .- Sea: y = sen(sen (senx)); evalúa: A) cos(sen (sen x) . cos (sen x) . cos x B) cos(sen (sen x). cos (cos x) C) sen(sen (sen x) . cos (sen x). cos x D) cos(sen (cos x). cos (sen(cos x)) E) cos(sen(sen x)). cos (sen x). cos x dy t 55 .- Determine: — , si: y = sen“(3x + 1) - 3 dx A) 2 sen (3x +2) D) 3 sen (6x - 3) B) 3 sen' (6x + 5) E) 3 sen (6x + 2) C) -2 sen (3x + 2) Problemas de Trigonometría y cómo resolverlos RACSO IDITCltl
56 .-Sea: y = cosí — + sen2 x) — . \4 / 8 j ¿y determinar: -r~ dx A) -sen (+ sen2 x). sen 2a B) -sen + sen2 xj. sen 2x C) eos + sen2 xj. sen 2a D) sen + sen 2a) . sen2 x E) -sen +sen2 x). sen 2x 57 .- Sea: y = 4 cos3(2a + 5) - 7, determine: dy dx A) - 24 cos2(2x + 6) . sen 2x + 5 B) 24 cos3(2x + 5) . sen (2x + 4) C) - 24 cos2(2x + 5). sen (2a + 5) D) 24 cos2(2a + 5) . sen (2a + 2) E) -24 cos(x -2). sen (2a- 5) dy 58 .- Determine: —, si: y = tan (sen x) + 1 dx 2 2 A) sec ((sen x).(cos x) D) sec (eos x). sen x 2 2 B) sec (sen x).cos x E) sec (sen x. eos x) 2 7 C) sec (sen x). cos~ x , 71 59.- Sea: y = tan (In (x")) + csc'l r/y Calcula: -j- 2 9 4 7 A) - . sec (In (x )) B) - tan (In (x )) 7 7 7 7 C) seC(ln (x-)) . 2x D) — . sec (In (x-)) 1 7 7 E) . cscZ(//i (x )) 2 60.- Sea: y — 3 tan(x + sen x) - 1, , dy determine: — dx 2 2 A) (3 sec x(x + sen x))(2a + eos x) 2 B) (x + sen x)(2a + eos x)) C) (3 tan2(x2 + sen 2x))(x + sen x) D) (x + sen x)(x + eos x) E) (3 tan2x)(2x + eos x) 61.- Sea: y = (3x4 - 7). cot (x2 + 1) - 5; , dy determinar: -r- dx A) 12a. tan(3x2 + 1) - 2x(x4 - 7). csc2 (x2 + 1) B) 2a3. (cot(x2+ 1) - 2x (3x4 - 7)). csc2 (x2+ 1) C) 12a3. sec(x2+ 1) - 2x (3x4 - 7) . cot2 (x2+ 1) D) 2a (3x4- 7) . csc2 (x*"+ 1) - 12x3. cot(x2+ 1) E) 12a3. cot(x2+ 1) - 2a (3x4 - 7). esc2 (x2 + 1) TT 62.- Sea: y = sec(2x + tan x) - ~ O dv determine: ~r- dx A) sec(2 + sec"x) . tan(2x+tan x) . (x+tan x) B) sec(2x+tan x) . tan(2x+tan x) (2 + sec2x) C) sec(2x + sec2x). tan(2x+tan x). (2 + sec2x) Límites y Derivadas Trigonométricas T22 247
D) sec~(2x+tan x). tan(2x+tan x) . (1 + secx) E) sec(2 + tan x) . tan(2 + tan x). (2 + sec2x) > dy 63.- Sea: y=ese (sen x - x") + 1 ;determine: — dx A) -csc(sen x- x“). cot(sen x~ - x).( cos x - 2) B) -csc(sen x-x2). cot(sen x - x~). (sen x - 2x) C) -esefsen x- x2). cot(sen x - x~). (cos x - 2x) D) csc(sen x~- x). cot(sen x-x ). ( sen x - 2) E) -csc(sen x- x"). cot(sen x - x~). ( cos x - 2) 64.- Determine: ab . si: /(x) — a cos x + b. además se cumple: -V3 + x— x.arc cos x A) (1-x2)3'2 x/5 —x2 + x.arc cosx B) (i. *2)3/2 -(8 - x2 ) + x.arc cos x O3------F=5ñ-------- -x/1 —X2 + X.fl/~CCOSX D) (1-x2)3'2 „ --J2 — x2 + xzarcos x E) -------------------- -2 í tr'l í tr'l 3 \3j =2 A J = 2 7 21 311 A)A B)-^ C)-3 D)-J E) y 67.- Sea: y = arc sen (ln x) + arc sen dv determine: -7- dx K)—/ - - ~ B)-------/ > x.vl - /n2x x.Vl - Zn2x 65.- Sea la función/, definida por: T x /(x) = sen"x- —, determine los valores de x G ( 0; 7t) de modo <lueAx) = ° D>{?} E){íTu} 3 4 C) 1------— D) / = . x yjl-ln~x yjl-(lnx)2 E)----, Xy¡l-ln2x 68.- Sea: y = cos (are sen Jx )- l,xG (0; 1), dv determine: -7- dx A)t^- B)-~t= C)t^- D)~y= E)^f= l-X ' y/x 1* Vx Vx DERIVADA DE F. T. INVERSAS 69.- Sea: y = arccos( Vl-x2 ) + y n ---a resen x 66.-Sea: y= 2 .------=—. xG (-1; 1), vi — x2 dv determine: — dx A> ~r=^ B> —7= D) E) 1-V1-X2 V2-X2 C) -2 Problemas de Trigonometría y cómo resolverlos RACSO 1DITO1II
70.- Sea: y = arc cos -Jx - arc tan (3), x G (0; 1), 74.- Sea: y = arc tan ( 74x2 —1) determine: — dx A)-y== B)^= O— 7x-x2 2-Jx 7x-x2 D)~TT E)— 2-Jx2 2-Jx-x2 71.- Sea: y = arc cos (ex) + arc cos 71-e2x , A) '------ B)—' = C)— 274x2 -1 jc.v4x2 -1 x.yJAx2 +1 D) / E)-r== X.V4X-1 v4x2-l 75.- Sea: y - arc tan (3 sen x) - cos —, dy determine: — dx determine: dy dx A) 0 B) 1 C) 2 D) 3 E) 4 c ^arc sen 3x 72.- Sea: y = 2 8 + arc sec — ax cos~x } l + 9senx B) 3 cosx l + 3sen2x dy determine: — dx 3 cosx l + 9sen2 x cosx D) l + (senx)2 px 3 cosx l+9sen2x (//i3).(2arcsen3j) (//i2).2arcsen3x A) 71-9x2 B) 71 3x2 76.- Sea: y = tan (z 3n orccos(x)-—1+tan — /n4.(2arcsen3x) C) r. 7 (/n2).2arcscn3* ' 71 -9x2 71 — 9x2 xg determine: dy dx ln2 2arcsen3x E) 71 + 9x2 73.- Sea: y = arc tan (2a - 5) - arc csc (4), rfy determine: —— dx -1 (1-x2)3'2 ---—---- O — (l-x)3/2 1- (1-x2) 77.- Sea: y = arc cot( 72X2 -1 ) - ¿r , O A)2x-10+13 B)2x-10x-13 , dy determine: — dx C)2x2-10x + 13 D)2x2-10x+26 A)-*----- B) /------- O- J------- V2x2-1 x.-Jx2-l x.V2x2-1 El-----=----- 24x2-10x+13 D)—pj— E) - -1 x.V2x“ +1 xv x2 +1 Límites y Derivadas Trigonométricas T22 249
78.- Sea f la función definida por f\x) = x . are sen *, determine el valor de:/'(1/2j. D\ 4- ~ — B)3" + 6 C) 3 ' 2 y.cos.r A)—'--------- 7 sen x- sen y -y.cos x C)—--------- 7 sen*-seny -y.cos* El ---’------- ’ sen * + sen y -y.cos* B) sen x - eos y ---eos*--- ' sen*-sen y D>f+ ? 71 6 DERIVACION IMPLICITA 79.- Sea la ecuación: sen y + y - 1 = 0 , y (3k + 1 )7t, k e Z, . . dy determine: —— dx A) 4 B) 3 C) 2 D) 1 E) 0 80.- Sea la ecuación: sen2y - r~ + 1 = 0, deter- dy mine: — dx 4* 3* 7* A) sen y sen2 y sen 2 y 5* 5y D) sen y sen 2* 81.- Sea la ecuación: y. sen * - * eos y + 1 = 0, . dy determine: — dx cosy —ycos* sen *+ sen y ycos*—eos y o----------- 7 sen * + *sen y eos y—sen* E)----z------- 7 sen* + *cosy eos y — veos* B)---:—1----- sen* + *sen y eos y-eos* D)----1----- 7 sen * + sen y 82.- Sea la ecuación: y sen * + eos y = 0, de- . dy termine: ~ dx 2 83.- Sea la ecuación: y = cos(*v) + y , I . dy determine: — ax A) -y.sen(xy) 1 + *sen(*y) — 2 y -y.sen(*y) B) 1- *sen(*y) — 2y —sen(.ry) D) 1 + sen(xy) - 2 y -y.sen(*y) l + *sen(*y) —y.sen(*y) ' 1 — *sen(*y) + 2y 84.- Sea la ecuaciómcos (r*-) - sen y + y = 0, dy determine: — dx 2*. sen y A)1 + eos y 2*.sen(*2) C) 4-eosy 3*2.sen(*2) E) ~----------- ’ 1 + eos y 2*.sen(*) b)—-------- 3 + eos y* 2*.sen(*2) D)-¡----- 1 1 + eos y 85.- Sea la ecuación: tan(* + y) + eos y - 1=0, dy determine: -i~ dx -cos2(*+ y) cos2(*+ y) + cosy -cos2(* + y) eos2 (* + y) - sen y Problemas de Trigonometría y cómo resolverlos ¿iRACSO W^BDITOKEB
COS2 (x + y) eos2 (a - y) - sen y -eos2(a - y) D) eos2 (a + y) - sen y -cos(A + y) E) cos(a + y) - sen y 2 86.- Sea la ecuación: sec (a + v) - y +1=0. , dy determine: — dx - sec(A + y). tan(A + y) secA+ y.tan(A+ y)-2y sec(A + y). tan(A + y) sec(A + y).tan(A + y) - 2 - sec( a + y). tan( a + y) C) sec(A+ y).tan(A — y) -sec(A+ y2).tan(A+ y) sec(A+ y2).tan(A + y)-2y , dy determine: — dx A) -2y •Jl-A2 E) J2-A2 B) E) 2y V2a-1 -2y 7T7 C) 2y V1-2a 89.- Sea la ecuación: arc cos y - x + 1 = 0, , dy determine: — ax D)y 71-y2 B)-714V Ej-Ví^y 90.- Sea la ecuación: arc sen (Ay) + yx = 0, . d(yx) determine: —;— dx A) 4 B) 3 C) 2 D) 1 E) 0 REGLA DEL’HOSPITAL - sec( a + y). tan(A + y) E) ---------'-----------— sec(A+y).tan(A+y)-2y 87.- Sea la ecuación: tan (Ay) - y = 0, dy determine: — ax —y.sec2 (Ay) —y3.sec2 (Ay) A.sec2(Ay) —3y2 A.sec2ÍAy) —3y2 -y. sec2 (Ay) _ —y2.sec2 (Ay) C) i, x t i ’ a. sec-(Ay) - 3 y- D) A.sec2(Ay) -3y2 _ —y-sec2 (Ay) E) A.sec2(Ay) + 3y2 88.- Sea la ecuación: arc cos a - -Jy = 0, x — sen(7tx) 91.-Calcule: lím A + sen(7tA) A) ! + 7T B)!^ C)-¡-^— 1-71 1 + 71 ’ 1 + 71 D)t-^~ E) - 1 + 71 1-71 92.- Calcule: lím sen(2cos0) 2cos0 A) 3 B) 4 C) 5 D)1 E)2 5a2 — 2 a 93.- Calcule: lím i->0 sen3A .2 1 2 1 . 2 3 B>-3 O-f D)^ E)-s Límites y Derivadas Trigonométricas
1 - senx — cosx 94.- Calcule: hm '-»o 1 + senx — cosx A) -5 B) - 4 C) -3 D) -2 E) - 1 95.-Calcule: lím —n(* A ) <>i r+1 102.- Calcule: lint “—------ n 1 — ZCOSA 3 A)V2 B)x/ó C)-x/3 D) 2 E) -x/7 1 13 2 1 A)| B>-^ C)| D)j E) 2 96.- Calcule: lím j->0 3x - 2sen4 v 6x — 7sen5x A)29 B,Í9 C)35 D) 29 E) 39 97.- Calcule: lím 775—77—'7. 0—>4 (0 + 4) - scn(TtO) 2 1 7T 1 4 A)— B) - C)7 D)7 E) - ti ti 4 ’ 2 7 71 7 1-^ 98.-Calcule: lím ---- i-wr senx 103.- Calcule: lím x->i 1 - Jx A) 4ti B) 7t C) 2 7t D) 371 E) 571 l-cos8x 104.- Calcule: lím ----— x-íO senox A) 2 B) 5 C) 3 D) 1 E)0 x A)^ 71 I3)f C)| D)f E)- 71 senx - sen2x 99.- Calcule: lím - A_»o sen3x 9 S 3 1 1 A>4 B)-| C)- D)5 • E)-3 100.- A)l A” Calcule: lím ¡ *_>0 sen(J x2 + 4 — 2) B) 2 C)3 D)4 E)5 106.- Calcule: lím -------- x->5 senx —cosx 4 A)-V? B) -yf5 C)-2>/2 D)->/3 E)-Vó 107.- Calcule: lím n 3 cos2x + cosx tan2x + tanx .... r. . . Vl+senr—Jl-sen 101.-C a 1cule: hm -------------- 1—0 a A) 1 B) 2 < >3 D) 4 E) 5 B)^ E>^ C>T 108.- Calcule: tan(flx) lím r->o |1+ x-cos(flx)][sec(flx)] 2521 Problemas de Trigonometría y cómo resolverlos Jfc7.ACSO P*DITOMB>
A) 0 B) | C) a2 D) 3a arcsen5x 109.- Calcule: lím x—>o arctanx E) a A) 3 B) 4 C) 5 D) 6 1 1- 2tanx—aresenx 110.-Calcule: hm x—>0 senx E)7 A) -l B) 1 C)2 D) 3 ,,, 1- Tt —2arccosx 111.- Calcule: hm x->0 x E)4 A) 5 B)4 C) 3 D)2 sen2x—sen(x2) 112.- Calcule: lím 2 x->0 X E)1 A) 0 B) 1 C) 3 D) 4 1—-Jcosx 113.- Calcule: lím 2 x->0 X E) 5 3 112 A>Z C)z E> x2 —2x + l 114.-Calcule: hm 1+cos(7tx) 3 8 1 6 4 3 9 A>rf B)^ Q-f D> E>^ 11S-Calcóte: Um x—>0 X □ o A)" f -trP' B)- „ n2-m2 v; 3 D) ”2 6'"2 n2 - m2 E) 8 116.- Calcule: 1—cos4x hm ; o x—>0 1 —cos3x 123.- Calcule: A)y B) 2 C)^ D)5 E) 2 cosx —eos x 117.- Calcule: Zí>n -i-------------------- x—»o sen2x + 2sen~x —2senx 112 1 3 A)^ B)^ C)f D)^ E) 118.- Calcule: J3 —1/2 + cosx lím ------------ x—>0 serT3x A)is¡ V3 C)1Ó8 D>ÍS E) J 128 x.sen(sen2x) 119.- Calcule: lím ; ", 7~r x yQ 1 cos(sen4jc) A)| B)-| C)| D)-| E)| 120.- Calcule: lím x->o x.senx A)| B)^ C) -| D) 3 E)| 1—cos3x 121.- Calcule: lím ----2— x->o tan x A) 2 B) 1 C)| D)| E)| 122.- Calcule: 3sen~x lím ------- x->o x.tan4x 3 13 2 A)^ B)^ C)-| D) 3 E)| , sec~x — 2tanx hm -------------- 1 + cos4x 2 5 13 A>3 C)í D)Í E)i Límites y Derivadas Trigonométricas T22 253
tan(l + cosx) 124 .- Calcule: lím n . 7 x-jn cos(tanx)-l A)-6 B) -5 C) -4 D)-2 E) -1 (x-x3)2 125 .- Calcule: lím , „ c , x^o (sen3A)(sen5x) A>Í7> ">15 C>13 D>n E> 18 2cosx—2+xz 126 .- Calcule: lím---t-t- ji—>0 3x 11 12 A)¿ B)i C)1 D)^ E)^ x —tanx 127 .- Calcule: lím -- x—»o x — senx A)-2 B)-l C)0 D) 1 E)2 128 .- Calcule: lím tanv-senv x—>0 X A)| B)| C)| D)^ E)^ nn r i , , l-COS(l-COSx) 129 .- Calcule: lun --H------ x—>0 X4 A)-| B)i C)í D)| E)-| 130.- Calcule: lím (1 + cos x) . (1 + cot x) X—>71 A) -l B) 0 C) 1 D) 2 E) 3 131.- Calcule: A) 4 B)3 132.- Calcule: e2x —e3x /wn----7,----T“ x—>o sen2x-sen3x C)2 D)1 E)0 tanx /wi , r.— x_,o 1-vl+tanx A) -l B) -2 C) -3 D)-4 E)-5 133.- Calcule: lím cot 2x. cot | - x | x X) \2 I 2 13 5 1 A)f B)| O| D)| E)| 134 .- Calcule: lím (1 + cosx)3secx _n 2 A) e2 B) e C) e3 D) e E) e1'2 135 .- Calcule: lím (1 + 3 tan~x)cot x x—>0 A) e B) e5 C) e2 D) e3 E) e3/4 APLICACIONES DE LA DERIVADA 136 .- Sea f la función definida por: /(x) = sen"x - " 1’ / 7T \ determine el valor de x en (0;—J para que/ tome su mínimo valor. A)^ B)y C)^ D)| E)y 137.- Determine la ecuación de la recta tangente a la curva: y = sen x + 2 en el punto (0; 2) A) x - 2 = y B)x + 4 = y C)x + 3= y D) x + 9 = y E) x + 2 = y 138.- Determine la ecuación de la recta tan- 7t gente a la curva- y = arc tan x - — en el punto: d;0) A ACSO WlDITOlll T22 254 Problemas de Trigonometría y cómo resolverlos
..11 nx 1 1 A)4*-4=y B)^x-^=y 1 1 .,.11 C)6A-6=J D)^-j=y E) j x- -1 = y 139.- Determine la ecuación de la recta normal a la curva: y = 2 sen a - 3 en el punto: (0; -3) A)-|x-3 = y B)-|x-3=y C) -jx-2-y D)-|a- 3 = y E) -|a- 3 = y 140 .- Determine la ecuación de la recta nor- 71 mal a la curva: y = arc tan (a +1) -~en el punto: (0; 0) A) - 5a = y B) - 4a = y C)- 3a = y D)-2a = > E)-^A=y 141 .- El perímetro de un sector circular es Su, determine el valor de su radio para que el área del sector circular sea máximo. A) 4u B) 3m C) 2m D) lu E) Ou 142 .- Un bloque de peso “P” se cuelga de una cuerda que pasa por una polea, tal como se muestra en la figura y a una altura de 20 ni, sobre la superficie del suelo. El otro extremo es tirado por un montacargas que está a una altura de 2 m del suelo. Si el montacargas se aleja a una velocidad de 9 m/s; ¿a qué veloci- dad sube el peso cuando está a una altura de 6 m del suelo? 9>/3 , o, ó73 , 3-^3 , A) m/s B) m/s C) m/s , ,, 27? , D) m/s E) m/s 143.- Se desea cercar un lote rectangular que tiene 4 000 m" de superficie, uno de cuyos la- dos está definido por la ribera de un río recto. Si no se necesita cercar por el lado que da al río; ¿qué dimensiones requiere la menor can- tidad de cerca? A) 1175 D)5>/5 B)9>/5 C)4>/5 E) 20 75 144.- Una estatua de 6 m de altura tiene su base 2 m arriba del nivel del ojo de un obser- vador. ¿A qué distancia de la estatua debe co- locarse el observador para que el ángulo subtendido desde su ojo por la estatua sea máxima?. A) 75 u B)7T3h C)273 u V)-i4Íu E)573m Límites y Derivadas Trigonométricas
LA TRIGONOMETRIA NUESTRA NATURALEZA CONSTRUCCIÓN DE UNA COLMENA Las abejas construyen sus colmenas con cera pura, que solo pueden producir las obre- ras. Ambos lados de la colmena constan de celdas hexagonales geométricamente per- fectas y se construyen de forma tal que la cantidad de material requerida es la mínima posible. Hay unos 5 millones de colonias de abejas en los Estados Unidos que producen en el orden de 125 millones de kilogramos de miel cada año. Las abejas construyen sus compartimentos de almacenamiento en for- ma de celdillas hexagonales. Cada celda tiene una base hexagonal y tres caras superiores rómbicas que se cortan con la altura de la celda formando un ángulo 6. El volumen de cada celda viene dado por la siguiente expresión: i/ 3^3 2. V = —2~ s h y es independiente del ángulo 6. Por otra parte, el área superficial de la celda, vie- ne dada por: S = 6ñ.s+-~-l V3-cos6 senS dependiente del ángulo 6. El ángulo que hace mínima el área (y, por tanto, la cantidad de material requerida) se determina resol- viendo la siguiente ecuac ón obtenida luego de derivar la expresión anterior: dS 3s í l-VscosB^ q de 2 I sen6 La solución de esta ecuación es: eos 6= 1/73 o sea 6 54, 74° Problemas de Trigonometría y cómo resolverlos •iRACSO DITOBBt
PRORlCMflS D€ TRIGONOMETRÍA V COMO RESOLVERLOS
«Oh1 Sistemas de Medida < Angular 01.- Graficamos los ángulos en sentido antihorario. De esta nueva figura podemos establecer que: -0 = 2n + x x = -271 - 0 02.- Graficamos los ángulos en sentido antihorario. De esta nueva figura podemos establecer que: a + a - x = 360° Despejando "x": x = 2a - 360° 03.- Graficando los ángulos en sentido antihorario, podemos establecer que: 3x + 20° - (x - 40°) = 90° => 3x + 20° - x + 40° = 90° 2x = 30° x = 15° 04.- Graficando los ángulos en un mismo sentido (antihorario), se propone que: 10° + 40° - 5x - 10° = 90° => -5x = 50° x = -10° Problemas de Trigonometría y cómo resolverlos RACSO
05.- De la figura: (90 - 3x)° - 5y8 = 180° , y además 9° < > 108 06.- Siendo AC una línea recta, graficamos todos los ángulos en un mismo sentido. Se observa que: Pero: y-(-x) = 60° => y+x = 60° 180° = Tirad > x + y = 60°. n rad 180° x + y = rad 07.- Al graficar los ángulos en sentido antihorario tendremos: 27x° - (5 - llx)8 = 180° .(1) Pero: 9° o 108 => (5 - llx)8 = (5 -llx)8 . — .. .(2) "(5 108 _______ 9° (2) en (1): 27x° - (5 - llx) . = 180° Efectuando: 30x° - 5 + llx° = 200° => 41x = 205 .-. x = 5 2ti 08.- De la figura: x° - y8 + — rad = 360° ...(*) 9° f9y\ Pero: 9° < > 108 => ys = ye- 1()g = 271 Tirad < > 180° => 3rad = 120° f 9yA 10x-9y En (*) : x° - I + 120° = 360° => ---= 240 lOx - 9y = 2 400 Sistemas de Medida Angular R1 259
09.- Como: 6 = x-x2 => x2 - x + 0 = 0 Para que la ecuación tenga solución, el discriminante de la solución general debe ser tal que: 1 - 40 > 0 1 n __ _____ Dmáx “ 4 X = 2 Luego de la figura original se establece que: rad = 2nrad Y para que 0 sea máximo sustituimos los valores encontrados: => a + — - — + 2 = 2n => a = 2n - 2 4 4 a = 4,2832 rad 10 .- Los números a, -6 y 0 que representan la medida de un mismo ángulo en los sistemas sexagesimal, centesimal y radial, y en el mismo sentido se relacionan por: a 9 -0 200 10 = n a Por lo tanto: ~ 9 10 => 10a + 90 = 0 . (A) correcto a 9 _ 200 7t => 1800 - are = 0 (B) correcto -0 10 200 - 7t => 2000 + 0n = 0 (C) correcto De(B) + (C): 3800 -an + 0 n = 0 => 3800 = n(a + 0 ) ... (D) correcto • 9000 = n(90 + 5a)... (E) incorrecto CONVERSIONES 11 .- Recordemos: 9o o 108 => M = ^j+2+—+ 2005)° • "i(y M = 9 12 .- Convirtiendo los ángulos a y 0 a grados sexagesimales tendremos: „ - n rqqa 180° _ í0,5236.180 V _ „„o R _ _ „ng_91 Rf1m 1* 60' a -0,5236 rad. 31416 ) - 30 0 - 30 50 - 30 10.c 50 • wo- 10k - r 0 = 27°27' 0 = 27°25' Por lo tanto se observa que: 0 < 0 < a ‘.i RACSO W 1IDJTOBBB R1 260 Problemas de Trigonometría y cómo resolverlos
13.- ri5' = i° + 1)’= (ií Luego: 5\" (5\° n rad n 4/ “ \4/ • 180° - 144 rad V15' < > 144 rad 14.- Por dato: "¿77 rad < > A°B' □(J Pero: . 1cnc. nrad 180° ,0 nrad = 180 => 5Q = = 3+0,6 Pero: Io < > 60' => 0,6° = 36' Luego: rad = 3o + 36' = 3°36' = A°B' => A = 3 a B = 36 Luego: _ 36-2(3) _ 30 36-10(3) - 6 7T 15.- Por dato: rad < > x^y'z,' Pero: nrad < > 180‘ n . _ 180° => 69 rad - 69 - 2 + Pero: = 36' + => rad = 2° + 36' + Pero: 1' o 60" => = 31 Luego: rad = 2°36'31" = x°y'z‘ x = 2, y = 36, z = 31 Io =7° M = 5 .’. Complemento de (x + y - z) = 90°-7° = 83° 16 .-Nuestra estrategia consistirá en convertir los grados a minutos y segundos sexagesimales. Para tal efecto hacemos: 37,98° = 36° +1,98° Como: 1° o 60' => 1,98° < > (l,98)(60') < >118,8' => 37,98° = 36° + 108' + 10,8' Asimismo: 1' < > 60" => 10,8' < > (10,8)(60") < > 648" Luego: 37,98° = 36° + 108' + 648" Sistemas de Medida Angular
También: 9o < > 108 27' < > 50m 80" < > 250s 36° < > 408 108' < > 200m 648" < > 2000s => 37,98° = 408 + 200^, + 200(f 2« 20"' De donde reconocemos que: 37,98° < > 42g20m < > AB* B0'" A=4 a B=2 M = 12 17 .- Nuestra estrategia consistirá en convertir todos los grados en minutos sexagesimales. i „ m Í1°21’Y /2°15’\ (1°3'\ .. f 1"+21'Y/2°+15'\/1°+3'\ Luego. M = ¡-y-J (— J (—) => M = (-gr-J (-^7-) (-3^) Como: 2° < > 120' => M = fér) = 27°27'21" ^3 J 3 J \ ¿ / Pero: 9° < > 108 => 27° < > 308 Asimismo: 27' < > 50m a 21 "< > 65s => 27°27'21" = 30g50m65s => 27°27'21" = 308 + 50m + 65s .. _e —m s => 27°27'21" o 30g50 65s = a0 be ele a = 3,b = 5,c = 0,d = 6,e = 5 Luego: M= " M = 3 18 .- La estrategia que utilizaremos consiste en transformar minutos y segundos a grados sexagesimales. Para ello requerimos establecer las siguientes equivalencias: 60' 1° 1“ = 60' => 15' = — = — => 1° = 3600" 4 4 3600" 1° 36 =-------= ----- 100 100 Entonces: 0 = 16°15'36" = 16° + 15' + 36" => 0 = 16° + Í-) + f 1 ) ^4) + [100J 0 = (16 + 0,25 + 0,01)° => 0 = 16,26° Pero también: Tt rad = 180° 0 = 16,26°. Tt rad 180° „ í 16,26x3,14 \ , e = I 180 —) rad 6 = 0,2836 Problemas de Trigonometría y cómo resolverlos Aíracso n DITOIlt
i° 11 r i° 19 .- Del ejercicio anterior se sabe que: 15'. = I 41 => 45". 3^00 " = í 1V ( 1 Entonces: 36O15'45" = 36° + 15' + 45" => 36°15'45" = 36° + - + — => 36'15'45" 2880+ 20 + 1 80 => 36°15'45" 2901 Y 80 J Convirtiendo grados sexagesimales a radianes, por el criterio del factor unidad tenemos: 36'15'45' 2901Y nrad _ (967)(3,14) 80 J ’ 180° 4800 raü 36°15'45" = 0,6326 rad 20 .- De la condición: x°y'z" = a^h’c" + c°a'b” + Ifc'a" => x°y'z" = (a + b + c)°(c + b + c)'(c + b + c)" Y según la condición: x°y'z" = 63°63'63" Asimismo recordamos que: Io = 60'; 1' = 60" ; Io = 3600" Entonces: xy z" = 63° + 63' + 6CT + 3" => x°y'z" = 63° + 60’ + 4' + 3' x”y'z" = 64°4'3" => x = 64 ; y=4 ; z = 3 64-4 w = -- - W = 20 MEDIDAS DE ÁNGULOS RELACIONADOS 180R 200R 21 .- Recordemos: S =--- a C =-------- 7t 7t R C Reemplazando en la condición: S = R => Ordenando las potencias convenientemente y simplificando: (122) . (r“t* ) Sistemas de Medida Angular
Se observa potencias iguales, entonces: 180 =R^ n Luego: M = R 22 .- Los números S y C están relacionados por: S C 9 ~ 10 De la condición tenemos: 7t X + — 4 9 + n 2 . „ 5 „ 9n JQ- => 10* += 9x + “2“ => x = 2n C 971 S = 2n+ 4 =7r Convirtiendo los grados sexagesimales a radianes, tendremos: Í9n\ Tirad \4) 180° 23 .- Recordando la identidad algebraica: (c + b)2 + (c - b)2 = 2(c2 + b2) (o + b)(a -b) = a2-b2 Entonces: W = -13 En donde al aplicar la identidad, obtenemos: W = •(*) Además se sabe que: 2(C + S) C-S S = 9k C = 10k S = C 9 — 10 Reemplazando en (*): W = ^^^—^^13 W= J2(19)-13=725 W=5 24 .- Efectuando las operaciones en cada paréntesis, tendremos: Problemas de Trigonometría y cómo resolverlos W^BDITOREÚ
n C+n De donde obtenemos: ~ ~— Pero: S = 9k, C = lOk => tí—------— l 9k lOk H 1 n--- S C S = (9k)° 25 .- Los números que representan la medida de un ángulo en los sistemas sexagesimal, 9C ttC centesimal y radial, se relacionan por: S = -jp-; R = - - (1) De la condición: q | = 5R + 52 ... (2) Reemplazando (1) en (2) y simplificando: 19 TtC 9 1 22 — ~ 5 ---- + 52 => — C - — — C = 52 2'10 200 20 40 ’ 7 _ (52)(280) C=14° 126-22 280 C = 52 26 .- Sean S y C los números que representan los grados de un ángulo en el sistema sexagesimal y centesimal, entonces se tendrá que: Media armónica: Ma = -j—p = g + ^-. S+C Media geométrica: Mg = VSC Y según condición del problema: Ma = 36(Mg)2 Reemplazando y simplificando: g+g = 36. (VSC J => S + C = -jg ... (1) Ademas se sabe que: S = 180— ; C = 200— ... (2) Tt Tt R R 1 ti Reemplazando (2) en (1): 180—+ 200— = — => R= R= ^840 ra^ 27 .- Se sabe que los números que representan los grados en los tres sistemas de medición angular se relacionan por: 180 R „ 200R “ Tt ' “ Tt ' ' ' ( / Sistemas de Medida Angular R1 265
Según condición del problema: C + 3S = R2 ... (2) 7t Reemplazando (1) en (2): 200+ 3 ^R2 7t Simplificando: 740^ = 3Z^ 7t 7t R = 20 28.- Para un mismo ángulo, representamos: 3 600 . S ... (i) - El número de segundos sexagesimales: - El número de minutos centesimales: 100 . C ... (ii) En los cuales: S: Número de grados sexagesimales C: Número de grados centesimales y se relacionan: S 9 _ C ~ 10 (S = 9.c a C = 10.fl) (*) Reemplazamos según el dato: 3 600 S + 100.C = 367 400 Simplificando: 36.S + C = 3 674 => 36(9o) +- (10a) = 3 674 Resolviendo: o = 11 => S = 99 (*) En la fórmula general de conversión: S R 180 ~ 7t Se obtiene: R- — 180 K 20 29.- Como en el caso anterior, representamos: - El número de minutos sexagesimales: 60.S ...(i) - En número de minutos centesimales: 100.C • • • (ü) - Relación entre S y C: c _ s 10 - 9 - (üi) (a + b)2 + 8ab En el dato reemplazamos (i) y (ii): (60.S) . (100.C) = -\&ab- -4 P A eso UlDITOkll R1 266 Problemas de Trigonometría y cómo resolverlos
Efectuando: 6 000.S. = 2 2 a +b +2ab+8ab 18.flb S2 - 120^00 (f+c+1°) Siendo: a > 0 /\b> 0 => ~ >2 De los cual: 120 000 (f +1 + w) > j^oVoO <12> s2- 10 000 ' <s>°) C 1 , Io 3 600" => Stmn- 100 , X«»«- 100 - 100 - 36 30.- Según el dato: a = 60.S a b = C En los cuales: S es el número de grados sexagesimales, C es el número de grados centesimales. S C Que se relacionan: = iq A (S = 9 . fi a C = 10 . fi) Reemplazando en lo que se pide: 4.(60.S)-16(C) C 240(9c)-16(10c) W~ (10.a) Efectuando: W = 200 31.- Según dato: 3 600 . S + 100 . C = 33 400 . .. (i) S C R De la fórmula general de conversión: jgp = 2qq = — ... (ii). De ii, reemplazamos en i: 3 600.(180.^- ) + 100^200.^) = 33 400 Simplificando «100», se obtiene: 6 480 R + 200 R = 334.7t => 6 680.R = 334.n Efectuando: R= Sistemas de Medida Angular R1 267
S C 20R 32.- Los números S, C y R se relacionan por: — = — =------- = k => J r 9 10 Tt 60 j— /— V71 9 3kJ\9 Reemplazando en W tenemos: W = j=> W = S = 9k C = 10fr R~2Ó 33.- Reemplazando las relaciones entre S, C y R del ejercicio anterior en el dato se tiene: ziJc 10(9k) + 3(10k) + — = 2400 + Tt Ttk => 120k + — = 2 400 + Tt Por lo tanto: k(2 400 + Tt) = 20(2 400 + Tt) k = 20 R = ~ (20) .’. R = Tt W = 3 34 .- Reemplazando las relaciones entre S, C y R del ejercicio anterior en el dato se tiene: mS + nC = 20R => 9m + lOn = Tt .. (1) 7tt 7tt 6m +5n= ~ => 12»; + lOn = ~ .. (2) 12 6 Tt Tt Restando (2) - (1), obtenemos: 3»í = ~ => m= ~ o lo [ Tt ] Tt Reemplazando en (1): ^1 18 I + ~ n =* ” = 20 „ . . , m Tt/18 m 10 De este modo se tendrá que: — = ------- — = — n Tt/20 n 9 35 .- Reemplazando las relaciones entre S, C y R del ejercicio anterior en el dato se tiene: Por analogía: =3 => k = =>R=^fc R = ‘álRACSO W^BDiToaas R1 2681 Problemas de Trigonometría y cómo resolverlos
36 .- Nuestra estrategia consistirá en expresar matemáticamente las condiciones del pro- blema. Luego: C-S 2 = C + S _ C-S 14SC 7SC “ 133R C + S ~ 133R 7T Reemplazando S = 9k, C — 10/c, R = ~ k en (*) obtenemos: 1 _ 14(90k2) k___n_ 19 = 13/^d ' = 3600 \20 ) Finalmente se tendrá que: R = k R= ^OCG 37 .- Recordemos que todo número real positivo verifica las siguientes relaciones: 1 Si : x > 0 => x + — >2 x 1 => x+ - =2 <=> x= 1 x ...(») De la condición del problema: R-l + 1 Jr-i = 2 ...(«) Reemplazamos (*) en (**): =1 R = 2 Sea: "6" el ángulo => 6 = 2rad 6 = 2rad. 180° 7t rad n 38 .- Sustituyendo las relaciones conocidas: S = 9k, C = lOk, R = ~ k, en la condición dada: x2(C - S) = x4-x2 + 1 => k = x2 + - 1 x Empleando la propiedad (*) deducida en el ejercicio anterior, diremos que: x2+-^->2,x*0 => x2 + \ - 1 > 1 => k>l => kmin = 1 R- -5- R“ 20 39 .- Sustituyendo las relaciones conocidas: S = 9k, C = lOk, R = — k, en la condición dada: 162(9k)(10k^ 12(S)10fc~9^ ti 5n Sistemas de Medida Angular R1 269
Simplificando: flI 3k 9k = ------ 25 k= 75 Luego podemos afirmar que: = 20 „ 1571 , R= —rad 71 40 .- Sustituyendo las relaciones conocidas S = 9k, C = lOk, R = ~ k, en cada ecuación: En(1,: + -x => 2k6’x En(2): (ído*10*)'+ ' ” a8=i/ Y reemplazando estos resultados en la condición dada: i/S2 = xC2 2 k6. S2 = 2k6C2 => 2 ke . (9k)2 = 2k6(10k)2 1(] « 10 -> 81k = 100/c => k = — => S° = (9k)° S° = 10° FIGURAS GEOMÉTRICAS 41 .- Elaboramos un gráfico con los datos del problema: De acuerdo con la propiedad de los ángulos interiores de un triángulo, tendremos: xrad + 4xrad + 4xrad = Tirad => 9xrad = Tirad -> x= x = 77 rad 42 .- De acuerdo con la propiedad del ángulo formado por dos bisectrices interiores, planteamos que: ni Z D = 90° + —2 m Z D = 90° + 50° = 140° . ti rad 7tt « m Z D = 140°. 18qO = -g- rad RACSO IDITOtlI Problemas de Trigonometría y cómo resolverlos
43 .- Tratándose de un triángulo se verifica que: A + B + C = 180°... (*) „ . * . 180° ,no g 9° /9x\' Donde: m Z C = ~ rad - "3" = 60 ; m Z B = r. ^7 = I -jp I Io m Z A = 9°18' = 9°+ 18'. — = 9° + 0,3° = 9,3° En (*) : 9,3° + + 60° = 180° => = 110,7° .’. x = 123 44.- Como el AABC es equilátero, se debe cumplir que: 45 .- Nuestra estrategia consistirá en transformar cada una de las medidas de los ángu- los al sistema sexagesimal. Luego: 50 „ 9° 7nx 180° A = — x8. = 5x° ; B = 8x° ; C = rad.---------------7 = 7x” 9 10B 180 Tirad Por tratarse de un triángulo se cumple: m Z A + m ZB + m ZC = 180° Reemplazando las equivalencias en esta relación, tendremos: => 5x° + 8x” + 7x” = 180° => 20x” = 180° .-. x = 9 46 .- De acuerdo con la información que brinda la figura, podemos establecer que: 30 = x°... (1) ; 56 = yg...(2) , „ m 3 x° 3 x° 10g (1)*(2): - =— => - = — .— 5 5 9 x 27 Eliminando las unidades y despejando: — = — ... (3) Sistemas de Medida Angular R1 271
Transformando la condición dada: M = y Reemplazamos (3) en (4): 27\ 50 J -1 M=i 47 .- Convirtiendo las medidas de los ángulos dados al sistema radial, tendremos ___________ Tirad 22n , , Tirad 2n m/B = 88 . = -7^-rad => mZD = 80g. — = —rad 180 45 200® 5 Como la figura ABCD es un cuadrilátero, sus ángulos interiores verifican: A + B + C + D = 2nrad 22n 3ti 2tt Reemplazando las equivalencias: A + rad + — rad + -y rad = 2n . 13n . A=~36 rad 48 .- Convirtiendo los ángulos a radianes: ,40 g Tirad x , mZB=y/. 20()j; = Tirad „ Tirad XX tn Z C = 10x^ .------ = — rad 180° 18 „ „ nrad xn tn Z D = 6x . ---- = — rad 180° 30 Por ser la figura un cuadrilátero : tn Z A + tn ZB + tn ZC + tn ZD = 2nrad 2tix x nx => rad + th xrad + ~ rad + 45 15 18 TIX — rd = 2rt rad Tt ¡r X = 271 => X = 10 Reconocemos que el mayor ángulo: mZB=y 49 .- Sean: a, P, 0, y 8 los ángulos internos de un cuadrilátero Y según condición del problema: a = x-2r P = x-r 6 = x 8 = x + r (Progresión Aritmética) M = 2 , „ 1071 J m Z B = -jij- rad Se sabe que: a + P + 0 + 8 = 360° = 4008 => x - 2r + 3x = 400g__(1) R1 Problemas de Trigonometría y cómo resolverlos RACSO D1TOIIB
Estableciendo las equivalencias entre el sistema centesimal y el sistema M, se tendrá: 508 < > 40M => 400g < > 320M ... (2) Por dato: x - 2r = 5M ... (3) (2) y (3) en (1): 5M + 3x = 320M => x = 105M ...(4) (4) en (3): 105M-2r = 5M => r = 50M Luego el mayor ángulo será: 8 = x + r = 155M 50.- Sean los siguientes las medidas de los ángulos del hexágono: a ; b=a-r ; c = a-2r ; d=a-3r ; e = «-4r ; f=a-5r Donde r es la razón de la progresión aritmética. Asimismo en un hexágono se cumple: Suma ángulos interiores = 180°(n - 2) , n = #de lados , donde: n = 6 => a + (a-r) + (a-3r) + (a- 3r) + (« - 4r) + (a - 5r) = 180°(4) => 6a - 15r = 720°... (*) Pero por dato: a - 125°; luego de (*) se puede despejar r, tal que: 6«-720° r= 15 Luego, el menor ángulo es: 6(125°)-720° 15 = 2° f= a - 5r = 125° - 5(2°) = 115‘ rt rad 180° f 23n f=~3€ rad Sistemas de Medida Angular R1 273
CAP.?- Longitud de Arco LONGITUD DE ARCO 01.- Para determinar la longitud del arco necesitamos convertir el ángulo dado en radianes, para ello utilizaremos la equivalencia dada y una regla de tres simple: Irad 57°17'44" 0 171°53'12" _ 1 rfld.l71°53'12" _ 1 rfld(171°51'+120"4-12") b - 57°1Z44" =* b “ 57°17'44" n 1 rad(3)(57° 17'44") „ o , => 0 - 57°17'44" 0-3 rad Como: L = 0.R > L = (3)(40 cm) L = 120 cm 02.- Unimos O con Q y Q con B, logrando formar el triángulo equilátero OQB, de donde: m Z OBQ = 60°. Además: m Z OBA = 45°, de este modo: m Z ABQ = 15°. Como: 15° < > rad => LfS = • (6 cm) = cm T 71 — 2 cm M = n cm RACSO DITOlll 274 Problemas de Trigonometría y cómo resolverlos
=> Preg. son*. = k»+Ui+ L„= n( J2 + 1) + 3n + 4 ¿2 preg somb= 7l( Vz +4) -» 4 ^2 ) 04.- Recordando la fórmula especial: L.L. e=-v- La aplicamos dos veces en el caso dado: x—a b-x a2+b2 6 = ~¡T~ = •• x = —.i. b a a+ b 05.- Nuestra estrategia consistirá en atribuir a cada arco una medida según las condicio- nes establecidas: y — AB — Lj¡¿ x = O A = L£ ; Haciendo un gráfico y aplicando la propiedad del problema anterior, se tendrá: 6 = *-y y x 0= - -1 y Donde: y = x0 Pero: 0 > 0 => 02 +0-1=0 => Longitud de Arco
06.- La expresión "M" se puede escribir así: M = Aplicando la propiedad de los ejercicios anteriores en el gráfico dado se tendrá: ^=6...(1) ; ^=0...(2) ; De (1), (2) y (3) en (*) , la expresión queda como: m= 7ae-e=Je2-e . m = o - = e . (3) a ' ' 07.- Elaboramos un gráfico según el enunciado del problema: Dato: 2R = 100 m => R = 50 m 50 m Luego: 0 = 50g. = ^rad => 6= ^rad O<^50* \ /L-e De este modo la distancia recorrida será: e = . 50 = m J Y recordando la fórmula de la rapidez: v~ t 25tt/2 m v= 20s e 5n , v = ÁREA DEL SECTOR CIRCULAR 08.- Sea OA = R ; OE = r; m Z AOB = 0. Entonces de la condición del problema se tiene: Problemas de Trigonometría y cómo resolverlos RACSO Diroin
09.- Nuestra estrategia consistirá en transformar el dato en una expresión cuyos térmi- nos podamos reconocer como áreas de regiones circulares: R26 + RL = 80 1 i RL Multiplicando por 1/2: —R26 + = 40 2 1 2 1 Donde: Area sector circular AOB = ~ R 0 = — RL => 2A = 40 A = 20 m 10.- Haciendo el cálculo de las áreas de cada sector circular según las condiciones del 11.- En el sector circular AOB: OB = OA = R= ^cm m Z AOB = 6 bg(5b)m (15b)m ... (lg=100m) (10Qb)m + (5b)m ( 105bY e z, Tirad _ Z2Lrad (15b)m ” [ 15b J =* ” ’ 180° “ 180 ™ Luego el área del sector será: R26 _ í ¡36 f 7n_ 2 “ 2 V 7 180 \ 7 1 36 7rt 2‘ 7 ’ 180 C Jt 2 S- wem Longitud de Arco R2
AREA DE UN SECTOR CIRCULAR A 3tt 12.- Recordemos que: 135° = — rad 311 . nz 3y/2. => Ua=—(2V2)=—nm 4 2 También: Lj¡¡ = y (2 + 2 V2) => = Tt + V2 Tt Luego el perímetro de la región sombreada es: 3V2 L= AN+ Lfs+Ljíi = 2^2 + 7t + J2n + n 2^2 N 45°'\ 2J2 \ 45W*50 1 O 2 M 2<2 B 2 L = 2V2 + 71 + ^p-71 13.- Graficamos según el enunciado del problema: Donde: R = 6 m ; 6 = 150° Tirad 5rt - 6 = 150°=>6=—rad r. „ 6R2 5ti 62 2 c Como: S = -- => S = — . — m S = 15rt 2 6 2 14.- Reconocemos que: OjC = OC = OjE = r => LOjCO es Isósceles. Por el teorema de Pitágoras: OjO = r-¿2 También: OE = O1O + O1E => OE = r-j2 +r Pero:AO = OE => r(V2+l) = 8 => r = 8(j2-l)cm Luego: Área sector _ Área sector Área sombreado — circular AOB círculo S= -JaO2- 7ir2 S = 16ti -7t[192- 128 ^2] => S=^.82- ti[8(V2-1)]2 S = 7t[128 ^2 - 1761 cm2 Problemas de Trigonometría y cómo resolverlos RACSO BD1TOBBI
15.- De la figura tenemos: 71 7t 2a + 3a + 4a = — => a = — 2 lo " Según dato: AB = 3-J2 => R = 3 1 2 Area región sombreada: S = ~ R (3a) ” s=í<9>(3#) S=T” 16.- De la figura: A = „ ti ( Ri 2 Luego: S2 + A = y ------ v > R2 Asimismo (*): Sj + A = Según condición: 13Sj = 7S2 => 13. ——(1-6) = 7. —[--el 2 2 1^2 J 7tt => 13 - 136 -----76 => 66 = 13 - — 2 2 1 Pero: 7tt = 22 => 66 = 2 6 = 17.- De acuerdo con el dato se tiene: $aob ~ 3rt => - - 3tt => r - 6 O Z 71 Luego: L¿g = 6. OC = — (8) O i 4tc La - 3 Longitud de Arco
TRAPECIO CIRCULAR 19.- Aplicando la fórmula especial para el ángulo central, tendremos: C-S Sustituyendo los datos: 6 = 6 = Lj-C Del Cap. 1 recordamos que: C = lOJc; S = 9k; R = — k Al reemplazar en (*): 6 = ~ ió‘ 20.- En la figura mostrada se puede reconocer que: 21.- Del gráfico se puede establecer que el área del trapecio está dado por: Problemas de Trigonometría y cómo resolverlos RACSO DITC1II
22.- Nuestra estrategia consistirá en determinar el área de las regiones sombreadas en función de las magnitudes 6 y r. Veamos: s=< (*) 2 => Sj = 2©/ => Sj = 2©/ - = 3S Análogamente calculamos las demás regiones, tal como se muestra en la figura. Luego la región sombreada es: A = IOS ( Gr2 ? Yde(*): A = lOl —I A = 5©^ 23.- Sustituyendo los datos en la figura, reconocemos que: *Oaob: OA = Lxa => © = 1 rad => OD = L® = L => L/j = L - 4 Luego usando la formula del área del trapecio circular: íL+L-4^ S=l-----¿---l4=2(2L-4) O Pero: S = 20 2(2L-4) = 20 L = 7 L 24.- Puesto que la figura dada es simétrica respecto de OC, podemos establecer que: (2©) r2 ©r2 25!=*-^— S1=— ...(1) Y del trapecio circular ABDE: 2S2 = (26r266r)2r S2 = 4 ©A .. (2) O 30r C 30r S. S. S2 Dividiendo (2) -e- (1): g = 8 25.- Sea © la medida del ángulo central. AO = 2 + x; AB = x + 2 .'. 6 = 1 rad En el sector COD: 2 = x - 1=> x = 3 ci - c ii c ÍCD+AB El area SD calcular: bD = I-2--- D Longitud de Arco
(x + 2)-(x-l) 3 Aplicando la fórmula especial se cumple: 6 =--------- => 6 = — - (1) Y del sector OCD, se verifica que: x-1 e=— ...(2) X X — 1 2 De (1) y (2) tenemos: — = => x - x - 6 = 0 Como(l) = (2): f =♦ x2-x-6 = 0 => (x-3)(x + 2) = 0 => Finalmente el área del trapecio circular viene dado por: x = -2 (absurdo) x = 3 r(x + 2)+(x-l)' 2 26.-De la condición del problema tenemos: 6-2R 2R + R6 = 6 => 6 = ... (1) De la figura mostrada, el área del sector circular está dado por: 1 2 2 R 6 = 2 - - (2) 1 •> (6-2R) Reemplazando (1) en (2) — R“. -—— = 2 6R - 2R2 = 4 =» R2 - 3R + 2 = 0 => (R - 2)(R - 1) = 0 =» R = 2 ni o R = Im 6 = 6-2(2) 6-2(1) ------- = 1 rad ó 6 = ------------ = 4 rad El mayor ángulo central mide 4 rad. 27.- Por dato : LT + + 2a = P Transformando esta expresión, multiplicando por cada miembro tendremos: Esta ecuación tendrá solución en los reales, si: discriminante > 0 => P2 - 16S > 0 => P2 16 P2 Q - 2____ ^máx 26 R2 282 Problemas de Trigonometría y cómo resolverlos RACSO UlDITOlll
28.- Nuestra estrategia consistirá en determinar el valor mínimo que tiene el área del trapecio circular. Luego determinaremos los valores de x que lo hacen posible, a conti- nuación calcularemos los valores de a y b que se corresponden con dicho valor y final- mente se determinará la medida del ángulo central 0. En el gráfico mostrado calculamos las longitudes de arco: a = R0 a b = (R + 4)0 = R0 + 40 fí b — a = t> b = a+46 =* 0 = —— ... (1) A continuación calculamos el área S del trapecio circular: Sustituyendo los datos para a y b, se obtiene: S = 2(210 - 40x + 7x2 - 30x] => 14x2 - 140x + 420 - S = 0 ...(*) Puesto que el discriminante debe ser > 0 , tendremos: (-140)2 - 4(14)(420 - S) > 0 => 350 > 420-S =» S>70 Smln = 70 Sustituyendo este valor en (*), descubriremos el valor de x: 14x2 - 140x + 350 = 0 => x2-10x + 25 = 0 =» (x-5)2 = 0 => x = 5 Con este valor encontramos: a = 210 - 40(5) = 10 a b = 7(5)2 - 30(5) = 25 Finalmente reemplazamos en (1) y obtenemos: 0 = 25-10 4 APLICACIONES MECÁNICAS 29.- En el sector OAB, se verifica que: En el sector C^OC^: L1 = 0(R-r) => L1=^(R-r) 0 = 3,75 L L = 0R => 0 = — donde: LCR = longitud recorrida por el centro de la rueda. Y: p = perímetro de la circunferencia del móvil L(R-r) Li = R . n = UR-H 2nr 2w 2nRr Longitud de Arco
30 .- Convertimos 0 a radianes y a continuación determinamos el número de vueltas de la rueda para lo cual reconocemos que su centro recorre una trayectoria idéntica a la cir- cunferencia del piso. „ „ Tirad 6n 6 = 6. ----- = 7577 rad 180° 180 0Tt L Luego: L = .R => n = —— 2nr Bn p 180 0R ' 2nr 11 ~ 360 r 31 .- Sea s la longitud de arco recorrido por el atleta. Como: s = vt => s = (17,6 . ).36s => s = 176 tn tirad 14rt Sea: 0 = tn Z AOB = 56°. 7777 = --------rad Pero recordando que: s = 6R => R = 180/n 176 44 45 _ £ _ 176 tn “ 6 ~ 14 22 45’ 7 El diámetro de la circunferencia es igual a: 2R = 360 m 32 .- Los discos A y B tienen un punto de contacto, entonces se cumple : f Q ®B'rB = ®A'A => eB=eA-^ => eB = 90°.| Los discos B y C tienen un eje común de giro (O), entonces se cumple : 3 6c = 6b = 90°. I =54° .-. 0C = 54° 33 .- Rueda menor gira Bnrad Iv____2n rad —> «j = 4 vueltas nT____8ti rad Entre las ruedas se verifica que: nA. Rj = n2 - R2 Donde: Rt = 3r a R2 = 4r Luego: (4r)(3r) =n2(4r) .-. n2 = 3v Problemas de Trigonometría y cómo resolverlos RACSO 1DITOUI
34.- Por estar en contacto las poleas 1 y 2 verifican: Lj = L2 => ©i - ri = ®zr2 2 => e(6) = e2(9) => e2=-e 2 Entre las poleas (2) y (3) que tienen eje común, se cumple: 02 = 63 => 03 = — 6 2 Como: L = 0.r, tendremos: L3 = 03 . r3 => L3 = ~ 0(12) = 80 Finalmente la distancia que recorre el bloque es igual a la longitud de la cuerda que se envuelve en la polea 3: L3 = 80 35.- Desde que las ruedas recorren la misma longitud, se verificará que el número de vueltas que estas dan, se encuentran relacionadas del siguiente modo: L L Rueda menor: n, = 7— a Rueda mayor: n, = _ “ 1 2nr 1 2ítR L L L L r 3 De la condición: 11, - h, = 7— => 77— - 77777 = 77— = — 1 2 &7tr 2nr 2nR 8nr R 4 36 .- Se oberva que el centro de la rueda recorre la distancia: e = 90 - 2 = 88 cm. Luego aplicando la relación conocida para el número de vueltas dado: 88 - 2nr 2^)(2) e = 8&cm n = 7 37 .- Nuestra estrategia consiste en determinar la longitud «L» que recorre el centro de la rueda y que de acuerdo con los datos se puede determinar a partir de la relación conocida: Luego: —7— = 267 .’. 44m = 267 Longitud de Arco
38 .- Datos: R = 70 cm r = 20 cm Incógnita: x = ? Debemos reconocer que la relación entre el número de vueltas y los radios es inversamente proporcional, luego: VUELTAS RADIOS a ................. 70 a + 100 .............. 20 =» 70» = 20(«+100) Y efectuando operaciones, obtendremos: a = 40 A partir de la siguiente relación sabemos que: espaciorecorrido * vueltas = 27t(radio) De donde : Espacio recorrido = 2rt (radio). # vueltas Utilizando "cualquiera" de las ruedas obtendremos el espacio recorrido solicitado: Im x = 2n(70cm) .40. => x = 2n. 28m Haciendo uso de la aproximación para n dada en la condición, tendremos : x = 2^~y-J28m => x = 176 m Finalmente: espacio recorrido = 176 m , espacio recorrido 39 .-Se sabe que: # vueltas=~—, --------?— Perímetro de una vuelta Luego en relación al disco móvil diremos que, cuando este retorna por primera vez al punto P, su centro habrá recorrido la longitud de circunferencia de radio (R + r) 2n(R + r) . # vueltas = ——-~O Pero, como:R = 6« y r = a Luego al reemplazar en (* *) RACSO IDITOIII Problemas de Trigonometría y cómo resolverlos
Se tendrá que: # de vueltas = # de vueltas = 7 40.- Para determinar el número de vueltas solicitado calcuremos primero las vueltas que da sobre los lados del triángulo espacio perímetro 44 # vueltas 3 = = 27t(l) = 2n # vueltas = Ahora calcularemos las vueltas que da la rueda en los vértices del triángulo . Para ello basta con apreciar la figura adjunta en donde se nota que la rueda da un nú- mero de vueltas que viene dado por la suma de los giros que ella experimenta cuando pasa dp un lado a otro, teniendo en los vértices a su centro de giro. n = 7i-a + 7i-P + 7i-0 = 3n-(a + P +0) n = 3n - ít = 2ít rad n = 1 vuelta Finalmente, el número de vueltas que en total da la rueda el recorrer el perímetro del triángulo, será: # vueltas = 8 Método abreviado Bajo el supuesto que lo anterior se ha comprendido, te sugiero resolver lo mismo del sgte. modo: #V = e 2nr Elados+Ee #V=---------- (fl + b+c) + 2nr #V= i------------ 2ítr #V = (a+b+c) 2ítr + 1 #v=4F+1 2.^.1 #V = 8 Longitud de Arco
«W.3 Razones Trigonométricas ios Agudos 01.- En base a la figura construida, reemplazamos en la condición dada: tan A + tan C = 2 => — + — =2 => í?2 + c2 = 2ac ... (1) c a ' ’ Pero: b2 = fl2 + c2 .. - (2) 2 r-, m ,,x ,2 - . b b b 2ac De (2) en (1): b = 2ac => — . — = — = ' ' ' ’ a c ac ac Luegi esc A . esc C = 2 02.- Sean : BC = a, AC = b, BA = c . Sustituyendo en la condición dada: .o a b c a b2+c2 a >i( cotC + cot B = 4 => - + ^=4 =^> bc = 4 ... (*) / 2 2 2 B Por el teorema de Pitágoras: a = b + c / 2 s _________E En (*): t- = 4 =» a2 = 4ab => a4 = 16b2? B c A ' ’ bc Calculando y reemplazando a4 en: F = 16 sen B. sen C. cos B. cos C _ 16b2c2 f=_7“ 16b2c2 16b2c2 F=1 F = 21 03.- Construimos un triángulo rectángulo en base al dato de: tan 0 = — Donde: x2 = 212 + 202 x = 29 Reemplazando los valores de las RT en la expresión para M, tendremos: 21 M= - . 3 I .29 + 4Í— 29 21 20 M = 3 R3 288 Problemas de Trigonometría y cómo resolverlos RACSO
2 04.- Se pide: W = tan C - co* . (1) Graficamos el L con los datos: 2 2 2 Teorema de Pitágoras: a + c - b ... (*) 2,2 2 a = b -c Reemplazando en: 2 sen A = csc C, 2. => 2.í?.c = b2 ... (2) 2 2 En(l>: =2^-^.de(2) w_ b2-c2 _ a2 2.a2 ~2.«2 05.- Sean a, b y c los lados del triángulo rectángulo en progresión aritmética, tal que: a = x-r , b = x y c=x+r B Como: b > a => B > A / c/ También se cumple que: c = a + b / 2 2 2 2 _________EZ => (x + r) = (x - r) + x => 4xr = x => x = 4r A b C , o x+r 5r 5 Luego: sec B = — = ^ .-. sec B = 06.- Sustituyendo los datos en el gráfico original podemos reconocer que: En el LABC: a tan a = — 8 EnelLCBD: Sustituyendo en a tan 0 = — la condición dada: a a tan a + tan 6 = 77 => — + — 8 3 a = 168 07.- Sustituyendo los datos en el gráfico original podemos reconocer que: En el ACB tenemos: tan 6 = — < 1 b Por el teorema de Pitágoras se cumple que: í?2 + b2 = (2 Vflb )2 => a2 + b2 = 4ab Razones Trigonométricas de Angulos Agudos
(a\ a a l — l +l=4v; pero tan 6 = 7- l b 1 b r b => tan2© - 4 tan 6 + 1=0 Resolviendo: tan 6 = 4±-J(-4)2 — 4(1)(1) 2(1) tan 6 = 4 + 2J3 2 tan 6 — 2 + a/3 ó tan 6 = 2--J3 tan 6 = 2 - V3 >2 08.-Sustituyendo los datos en el gráfico original podemos reconocer que: m + a m cot a =--- a tan B = — a a m + a m Luego M -------------— a a M = 1 09.- Construyendo el rectángulo ABCD y trazando su diagonal AC, nos muestra que: AB = DC = o , BC = AD = b, m/CAD-(í , , I® 21 21A Según condición: cot a = — = 1,05 =------ = — = —- a 100 20 20k El perímetro es: 2a + 2b = 492 => 2 (20 A) +2 (21 A) = 492 => 82A = 492 => k = 6 >a-20k = 20(6) = 120 m a b 21A = 21(6) = 126 m Por el teorema de Pitágoras en el LADC tenemos: d = 174 m diagonal: d = Ja2 + b2 = V1202 +1262 => d = ^30276 290 R3 Problemas de Trigonometría y cómo resolverlos JUACSO WlDlTOlll
11.- Completamos los lados del L, con los datos: En el L DBCA hallamos BC: (BC)2 = («)2 - (b - a)2 BC= Jiab-b2 En el L ABC, hallamos AC: (AC)2 = (BC)2 + b2 12.- Sea el paralelogramo ABCD: AB = 8 ffl , AD = 16 m. BD es la longitud menor. En el LAEB: se traza BE ± AD , además para ángulos notables de 30° y 60° se cumple: AE = 4 m , BE = 4 V3 m ED = AD - AE = 16 - 4 = 12 En el L\BED tenemos: Por el teorema de Pitágoras: BD = VbE2+ ED2 ^(4^3 )2 BD = JÓ4(3) m BD = 8 - J3 m 13.- Sea el L al que se refiere el enunciado, de casos a,byc. Según datos: a + b + c = 132 ... (i) a2 + b2 + c2 = 6050... (ii) 2 2 2 Por el teorema de Pitágoras: a + b = c ... (iii) De (iii) en (ii): c2 + c2 = 6050, de lo cual: c = 55 El problema se reduce a resolver: a + b = 77 , de (i) a2 + b2 = 3025 ..., (iii) Haciendo: a = 77 - b, reemplazando: (77 - b)2 + b2 = 3025 Resolviendo: b = 44 => a = 33 => a, es el menor ángulo agudo. . 33 3 tan a = 44 = 4 Razones Trigonométricas de Angulos Agudos R3 291
14.- Reconocemos que el LBAE es de los notables: 53° m ZABE = = m Z. ECD Por el teorema de Pitágoras: Sea a la medida del ángulo central: Tirad 53íi => a = 2m Z ABE = 53° => a = 53°. “ = rad loU loU En el sector circular BEC: BE = EC (radio del arco BC) Lg = BE . cz = FE L 53ti 5 2 ‘ 180 53^571 360 15.- En el LABC dado se puede identificar que: . a . a 12 11 a tan A - c => c - n x => x - 12 . c ... ( ) De la otra condición: 10(6c - b) = b - c => 60c - 10b = b - c c ii c 11b 61c - 11b —. = b 61 b 61b De donde deducimos que: c = 11b a b = 61b Por el teorema de Pitágoras: b2 = c2 + c2 => (61b)2 = o2 + (11b)2 a = 73721b2 -121b2 => « = 60b Reemplazando en (*) tendremos: 11 60b 12 ’ 11b x = 5 R.T. RECÍPROCAS 16.-Efectuando la multiplicación indicada en la expresión original, tendremos: '4 cos36°. sec 36° +9 sen 54° .sec 36' cotl8°.cot72° Por la propiedad de las razones recíprocas se tiene que: cos 36°. sec 36° = 1 Problemas de Trigonometría y cómo resolverlos JS RACSO paDiTOBBa
Por la propiedad de las Co-razones trigonométricas: sec 36° = csc 54' => sen 54°. sec 36° = sen 54° . csc 54° = 1 cot 18°. cot 72° = cot 18°. tan 18° = 1 a cot 72° = tan 18° Sustituyendo estos productos en (*): 4(1)+ 9(1) M=V^— M= V13 17 .- Aplicando la propiedad de las Co-razones, se tendrá que: senl0°= cos80° sen20°= cos70° (+) sen70 = cos20 sen80 = cosl0 sen 10° + sen 20° + ... + sen 80 = eos 10° + eos 20° + ... + eos 80° senlO° sen20°+—. + sen80° cosl0°+cos20°+... + cos80° 1 sec(3x-15°) Reemplazando en la condición del problema: --------- = 1 => sec (3x - 15°) = 2 2 Pero: sec 60° = 2 => 3x - 15° = 60° x = 25° 18 .- Tenemos: sen (50° + x) - eos (40° -x) + tan (x + 10°). tan (x + 40°) = 1 ---(*) Por la propiedad de las Co-razones: sen (50° + x) = eos (40° - x) Transponiendo términos: sen (50° + x) - eos (40° - x) = 0 Reemplazando en (*) obtenemos: tan (x + 10°). tan (x + 40°) = 1 Transponiendo términos: Por razones recíprocas: Reemplazando en (**): tan (x +10°) = 1 tan(x + 40°) 1 tan(x + 40°) = cot (x + 40°) tan(x + 10°) = cot(x + 40°) - • (”) Por co-razones trigonométricas: x + 10° + x +40° = 90° x = 20° ,3(20°) , Luego en: M = sec (3.20°) + cor 2 = 2 + (V3 ) M = 5 Razones Trigonométricas de Angulos Agudos R3 293
19 .- De la condición (1) despejamos: sen (x + y) = cos (85° - y - z) Por la propiedad de co-razones: x + y + 85° - y - z = 90° => x - z = 5o... (3) De la condición (2) despejamos: tan 2x = Por razones recíprocas: tan 2x = cot 3z Luego por la propiedad de co-razones: 2x + 3z = 90°... (4) 3x-3z = 15°l Hacemos: 3(3) + (4): „ 1 + => 5x = 105° => x = 21° ' ' 2x + 3z = 90 J Sustituyendo en (3): 21° - z = 5o z = 16° Luego: M = tan [2(21°) + 11o] - tan (21° + 16°) 4 3 7 => M = tan53°- tan37° = | | .-. M= 12 20 .- Por la propiedad de Co - razones tenemos: tan 89°= cot 91° tan 88°= cot 92° tan87°=cot93° (multiplicando) => B = cot l°.cot 2°.cot 3o.... cot 44° tan 46°= cot 44° Luego: AB = (tanl°.cotl°).(tan2°cot2°) (tan3°.cot3°).. - (tan44°cot44°).(tan45°) => AB = 1 i i i i i 2 Finalmente tendremos que: M = (1) . tan — M = 1 21 .- Según las propiedades de las co-razones: sen 10° = cos 80° Esto nos permite expresar M en términos del seno, así tendremos: «b(senl0°—l) + fl2 -b2senl0° absenlCP—ab+a2 —b2senl0° flb(senl0°+l) + fl + b senl0° ¿?feenl0°+flb + fl +h2senl0° Problemas de Trigonometría y cómo resolverlos ÍJ1P.ACSO PaDlTOBBI
Usando la propiedad de las proporciones, tendremos: 9 1+M loteen 10°+2íi 1-M 2ab + 2b2senl0° 1+M 2fl(bsen46°T:flJ 1-M = 2tefl+hseR-TC*J 1+M _ a 1-M “ b 22.- Como: m Z B = 90° mZA + mZC = 90" Luego por la propiedad de las co-razones: sen A = eos C 2 2 3^ Reemplazando en (*): (sen A)(sen A) + (sen A)(senA) = ' => 2(sen A)(senA, = 2.33 => (senA)(senA) Finalmente por analogía deducimos que: sen A = 1/3 23.- Como: sen (39° - 6) = eos (14° + 36) 39° - 6 + 14° + 36 = 90° 26 = 37° Luego en el LDBC (notable) Se observa: 5k = 36 =» k=6 => x = 4k .-. x = 24 24.- Por co-razones trigonométricas: tan 80° = cot 10° a sec 70° = csc 20° Entonces en la condición del problema: csc(6 + 26°) = 2tanl6°.cot20° gen26°.csc26° 1 => CSC (6 + 26°) =2 =$• 6 = 16° , 1 3 Luego: M = eos 60° + tan 45° —> M = — +1 .-. M = 25 .- De la condición (1): 5a +2b + c + 26° -3a = 96° 2a + 2b + c = 7G° => 2(o + b) + c = 76° Sustituyendo en (*) la condición (3): 2(36°) + c = 76° => c = 16° De la condición (2): 4d + e = 40° + e => d - 16° Razones Trigonométricas de Angulos Agudos
Finalmente remplazamos estos valores en la expresión dada: D = tan [(10° + 2(10°) + |(10°)] = tan 45° D = 1 fí.T. DE ÁNGULOS NOTABLES 26 .- Colocando los datos en la figura original, reconocemos que: LODA es notable. => OD = 4k a OA = 5k También se observa que: OD = BE = 4/c OA = DE = 5k Finalmente en el LOEB: tan 6 = 27 .- Reconociendo que el L ABC es notable anotamos los valores de sus lados y ángulos AB = 4k , BC = 3k Por dato: BC = CD => CD - 3k Luego L ABC: tan 6 = — 6k „ 2 tan 6 = 28 .- Del LADE Notable, afirmamos que. En el cuadrado ABCD: CD = AD = 4k => De la figura: BF = 4k - k tan 37° 3 13 => BF = 4A> ~k= —k 4 4 I3A . BF 4 Luego: E\ ABF: tan x = - = —— 6 AB 4k DE = 3k a AD = 4k R3 296 Problemas de Trigonometría y cómo resolverlos XRACSO WdiTOJII
29 .- Reemplazando los valores notables en la ecuación dada tendremos: Efectuando obtenemos: x = 2 - x 30 .- Recordando que: tan 60° = -J3 , sen 30° = 1 /2, reemplazamos en la expresión dada: sec 6 = tan260° + sen 30° => sec 0 = (Jsf + — Tk/' sec 0 = 7- = ("0" es agudo) c Pero: x2 = 4912 - 4k2 => x2 = 4512 => x = 3-j5k x 3J5 t- (3^5 | 1 15 1 Luego: tan®=2k=-^~ => M = -J5 I l+ ~ ~ M = 8 t 31 .- Recordando las R.T de los ángulos de 30o- 45° - 60°, reem lazamos en la expresión dada: W = 2 32.- Reemplazando el valor de x = 15°, tenemos : w=tan2(60°)+cos2(45o)+a3t260l sen30° Recordando que: 1 11 cos 45° = -7= ; cot 60° = —¡= ; sen 30° = — V2 J3 2 Calculando: tan 60° = -J3 W = 3 + — ____2 1 2 3 = 7 + 2 3 22 3 2 Según condición: W = g = -y => a = 22 n + 3 = 25 Razones Trigonométricas de Angulos Agudos
34 .- En el gráfico: Haciendo: BD = DE = M => AE = 2n => LADE es notable: DAE = coo 070 Calculamos: a = 45° - -7- = “7“ cot cot = 3 35 .- Sean los números: (x - r), x, y, (x + r) los lados del triángulo rectángulo- A partir de la fi- gura podemos reconocer que el menor ángulo es el indicado con a, dado que él se opone al cateto de menor longitud, luego: cosa = 777---C) A continuación aplicamos Pitágoras, para reconocer una relación entre x y r: (x - r)2 + x2 = (x + r )2 => x2 + r2 - 2xr + x2 = x2 + r2 + 2xr => x2 = 4rr => x = 4r...(**) Finalmente reemplazamos (**) en (*): cos a = cos a = 0,8 Problemas de Trigonometría y cómo resolverlos «JíFlACSO BOITOIII
36 .- Luego de graficar el enunciado del problema tendremos: eos A = 0,6 = Jq Dado que no aparece un triángulo rectángulo don- de intervenga el ángulo A, trazamos la altura CH . En consecuencia por "la propiedad fundamental" de las R.T. de un ángulo agudo. Podemos decir que: cosA = AH 6 AC " 10 - > AH = 6k a AC = lOk Graficando una vez más el problema y reemplazan- do los valores obtenidos, concluimos que: HC =8k a BH = 4k Finalmente: tan P = HC 8k BH ~ 4k tanP = 2 37 .- De acuerdo con el item 3.2.3 se sabe que: sen 3x = eos 75° <=> 3x + 75° = 90° Luego: 3x = 15° .-. x = 5o 38 .- Al igual que en el problema anterior, se cumple que: 2x + 25° + 5x - 5° = 90° => 7x + 20° = 90° Finalmente: 7x = 70° .-. x = 10° 39 .- Trabajando con la condición dada : tan (x + 41°). tan (2x - 31°) =1 => tan (x + 41°) = tan(2x-3i°) (*) Por lo visto y explicado en el item 3.2.1 sobre razones recíprocas, podemos afirmar que: tan(2x-31°) = cot (2x ' 31 Luego al reemplazar en (*) tendremos: tan (x + 41°) = cot (2x - 31°) Razones Trigonométricas de Ángulos Agudos R3 299
Dado que esta relación corresponde a una razón y su co-razón,diremos que los ángulos dados son complementarios, por lo tanto: (x + 41°) + (2x - 31°) = 90° => 3x + 10°= 90° => 3x = 80° x = 26,6° x = 26° 40' 40.- Utilizando el mismo procedimiento del problema anterior, tendremos que: 1 = tan 4x\ /por razones ~3 / \ recíprocas Luego: , /5x-96°\ /4x\ cot (—2—) = tan U 1 5x-96° 4x Entonces: ---2--- + = 90 => 15x - 288° + 8x = 540° => 23x = 828° 41.- De la condición dada: Luego por ser co-razones: Y por condición del problema: Restando (1) - (2) m.a.m. Y reemplazando en (2): 2x - 20° = 10° => 2x = 30° .’. x = 15° Finalmente: el mayor ángulo es: 20° (por tratarse de co-razones) => 23x = 540° + 288° x = 36° cot 2x = tan 3y 2x + 3y = 90° ... (1) 2x - y = 10° ... (2) 4u = 80° .-. y = 20° 42.- Se sabe que : sen a - cos 2P = 0 => sen a = cos 2P Luego: a + 2P = 90° ... (1) Análogamente, si: sen p.csc 4a = 1 deducimos que: P = 4a ... (2) A continuación trabajando con (1) y (2) obtendremos: a + 2(4a) = 90° => 9a = 90° Entonces: a = 10° y P = 40° Finalmente: a + P = 50° Problemas de Trigonometría y cómo resolverlos MíRACSO W^BDITOBBB
43 .- Por razones trigonométricas de ángulos notables, tendremos: tan (« - b) = 1 <=> a-b = 45° tan (a + b) = -J3 <=> a + b = 60° Luego, resolviendo el sistema anterior, obtendremos: a = 52° 30' y b = 7° 30' , a 52°30' a _ Finalmente: = yogg- fe = 7 44 .- En el triángulo rectángulo BAD: tan 60° = —= ^3 2^/3 b = 6 Finalmente, en el triángulo rectángulo CAD: sen 30° = ~ = i a 2 => a = 2b a = 2 (6) .'. a = 12 45 .- En el triángulo rectángulo ABC: 100 i sen 30° = .. = -x => y - 200 En el triángulo rectángulo CAD: eos 30° Luego: 200 _ x 2 400 46 .- Reemplazando el valor de "0"en la expresión tendremos: Íi-\2 / i-\2 V3 | J 2V3 | 3 A 4(3) 3 3 Q + _Q- “ sec60° 1 => E = 2 3 + 12 => E = —~— —> p — — • E — — => r. 2 =* 18 • 11 “ 6 1 Razones Trigonométricas de Ángulos Agudos R3 301
Resolución de Triángulos TEOREMA 1 01.- Con ayuda del siguiente gráfico, podemos deducir el valor de «x», así: x = b - m Pero: m = a cos a (usando el teorema 1) Finalmente: x = b - a cos a 02.- Graficando el enunciado del problema ten- dremos: Z ACH = 0 (Por ser complemento del Z A) Finalmente aplicamos el primer teorema en el triángulo sombreado, obteniendo: h = m cos 0 B 03.- Con ayuda del gráfico siguiente, podremos trabajar en dos triángulos rectángulos parciales, así: Problemas de Trigonometría y cómo resolverlos ía RACSO «P ICIYD1II
04.- Reconociendo que: AD = DC = BD = 3a Encontramos: BG = 2a y GD = a En el IXGHD: tan(ZDCG) = tan 0 = .. . (1) En el triángulo GHD: HD = a cos a y GH = a sen a „ . ... . «sena sena Entonces en (1): tan 0 = wcosa + 3f/ = o7saT3 B (2) Por dato: cos a + 3 = 3 sen a; luego al reemplazar en (2) tendremos: _ sena . _ 1 tane = 3^7E =» lan e = 3 Observación.- Demuestra que a es un ángulo obtuso 05.- Area del cuadrado = (6 +a)2 = 64 => 6 + a = 8 => a = 2 Asimismo se observa que 2m = &J2 => m = 4^2 r> j k run r\ /lím HC PC HC 2 _ X Reconociendo que: IX CHP ~ IX AMP: => = & => HC = 3 A continuación recurriremos a la variable auxiliar 0, la que nos permitirá relacionar aún más los lados conocidos con el que buscamos. En el triángulo AMO: x = m sen 0 ... (1) En el triángulo BP'O: = m cos 0 ... (2) Dividiendo (1) -s- (2) tendremos: tan 0 = 3 = 3/1 Finalmente: en (1) x = m sen 6 12y[5 5 06.- En el triángulo TQR: TQ = L sen a (Teorema 1) En el triángulo PQS: QS = L sen P (Teorema 1) De la figura: TS = TQ -QS k = L sen a - L sen P k = L (sen a - sen P) Resolución de Triángulos Rectángulos
07.- De acuerdo con la teoría de los ángulos inscritos vistas en Geometría, diremos que: m Z BOC = 2 m Z A = 20 A continuación, aplicando el teorema «1» en el trián- gulo OMC obtendremos que: MC = R sen 0 Luego: BC = 2 MC (MC =BM) BC = 2 Rsen G 08.- Colocando los datos en la figura dada, aplicamos los teoremas de resolución en el triángulo rectángulo ABC: (*) 4k = 4 sen a => (*) x + 3k = 4 cos a => De (1) en (2) tendremos: k = sen a ... (1) x = 4 cos a - 3k ... (2) x = 4 cos a - 3 sen a 09.- Tracemos DE _L AB En el LBED: BE = q sen a; ED = q cos a Asimismo: AE = BA - BE = p - q sen a Por semejanza de triángulos (ABC y AED): BC = AB x _ P ED AE qcosa p-qsena pqcosa p-qsena 10.- Aplicamos los teoremas en el L BQC y el L CDQ, estableciéndose que: R cos a - PQ = R - R sen2a 2 PQ = R(sen a + cos a - 1) 2 Pero: sen a + cos a = M PQ = R(M -1) R4 304 Problemas de Trigonometría y cómo resolverlos ^Mracso S OITOUI
11.- Sustituyendo los datos en la figura original, identificamos: El L BEA, donde: BE - 3k => BF = 3k eos 53° = — 5 12k En el L BFE: EF = 3k sen 53° = 5 En el L CHE: HE = HF - EF , 12* 13* Luego: HE = 5k- —— =---- 55 B 5k A 9* Además se observa que: CH = BF = — — | +í~^—| => CE = kVÍ0 5 J < 5 J CH 5 eos a = —— = . ’, CE Vio* => 5 VÍ0 . eos a = 9 W = 9 TEOREMA 2 12 .- Graficando el enunciado del problema, tendremos que: Area Trapecio: Por el teorema «2»: Pero: 2m + b = B => h = m tan 0... (2) 2tn = B-b m =^...(3) A= (M)h (1) A continuación reeplazamos (3) en (2): h = tan® Finalmente sustituiremos el resultado en (1): A = (^2^)' 2 tan ® . ÍB2-b2\ A = I—£— I tan 0 13 .- Graficando el enunciado del problema, consideremos que: m Z CMB = 0, luego en el t\ MBC aplicamos el teorema 2. Razones Trigonométricas de Angulos Agudos
BC = a tan 6 Y del ts. ABC: t BC atanG . . tanO tanA = AB =* tanA= 2- tan 0 = 2 tan Á 14 .- En el ts. ACD: Aplicamos el teorema 1 AC = 2R cos a En el ts. AOC: Aplicamos el teorema 2 AC = h sec a Luego: 2R cos a = h sec a 2R eos a . sec q = h sec a i 1 9 R = ^hsec a R = 0,5 h sec2 a 15 .- Aplicando los teoremas de resolución de triángulos en el triángulo rectángulo ABD y a continuación en el triángulo rectángulo ABC, tendremos: a + atanP cot a =--------- a Luego de simplificar, tendremos: cot a - tan P = 1 16 .- Sea BC = a => AD = 2a En el LCBD: BD = a tan a En el L ABC: BA = 2a + a tan a En el L ABC: cot a BA _ 2a + atana BC ~ a 1 9 ----- = 2 + tan a => tan a + 2 tan a - 1 = 0 tana Resolviendo la ecuación cuadrática: tan a = -1 ± V2 Pero a es agudo, entonces: tan <x= 42 - 1 > 0 Problemas de Trigonometría y cómo resolverlos RACSO IDITOII1
17 .- En el L BEC hacemos: BE = a => EC = a tan P En el L DEC: DE = EC . tan P = a tan p. tan P => DE = a tan2p En el L AEB: m Z BAE = P => AE = a . cot P DE atan2p En el L AED: tan 6 = A _ =------- AE acotp tan 6 = tan3p B A E C 18 .- En la gráfica, ubicamos «6» en el LDCE, en el cual por la relación 1 a 2 de los catetos se reco- noce: CQO a = 45° + Se pide: sec a; que por de arcos complementarios: sec a = csc (90° - a) = csc De la figura: seca= VÍÓ 19 .- Considerando que el lado del cuadrado mide 2L e identificando los ángulos más adecuados, la figura dada queda según como se muestra: En el L MPE: 2Ltan(x—y)+L tan x - ------------ tan x - 2 tan (x - y) = 1 Resolución de Triángulos Rectángulos
TEOREMA 3 20 .- En ts. CMO aplicamos el teorema 3. OC = R csc e Luego: AC= R csc 6 + R AC = R (csc6 + 1). -. (1) Ahora, en el tx BAC aplicamos el teorema 2. AB = AC. tan G ... (2) Finalmente reemplazamos (1) en (2): AB = R (csc 6+1) tan 6 2L- Recordemos primero la propiedada geométrica, en donde conocidos ay b se podrá calcular x a partir de: ab x ~ a + b _3 3 De la figura: => GH = Area A EFG = Area tx BFE - Área BGF ^cota ~a ^-Tjcota . _ 3_____3 o 3 aaefg “2’2 a2 a2 aaefg = V cota’Í5 cota = 5»2 cota - 3fl2 cota 45 aaefg - 45 cot a Problemas de Trigonometría y cómo resolverlos >^RXCSO MlDITOlIi
22 .- Aplicando los teoremas de resolución de triángu- los rectángulos en el L BAM y el t\MNC, tendremos: x cot P = xtan a + a sec P => x(cot P - tan a) = a sec P risecp Finalmente: x = cotp.tana 23 .- Por dato se sabe que O es centro del A ABC. Por resolución de triángulos rectángulos se tiene: AO = r. csc ; OC = r. csc ; OB = r. csc Por ser AO media proporcional de OC y OB: r.csc^ = ^r.csc^.r.csc^ => csc2^ = csc J|.cscy i ü j 2 A B C Invirtiendo: sen = sen y - sen 24 .- Desde que AC = CD, podemos reconocer que el AACD es un triángulo isósceles. Empleando los teoremas de resolución de triángu- los rectángulos, en el L ABC y trazando la altura del AACD, se puede establecer que: x = 2a cot 6 25 .- Instalando los datos y haciendo que: BD = ED = a En el L DAB: DA = DB sen a = a sen a AB = DB cos a = a cos a En el L DEC: m Z EDC = ni Z ABD = a CD = ED. sec a = a sec a CA = CD + DA = a sec a + a sen a _ .. „ AC flseca + nsena En el L CBA: cot P = — AB F «cosa cot P = sec2a + tan a Resolución de Triángulos Rectángulos
26.- De acuerdo con la propiedad de las tangentes geométricas, se verifica que: CT = CB. 27.- De acuerdo con los datos: AOD es un cuarto de circunferencia. Asumiendo que el radio de la cir- cunferencia es 2r, procedemos a identificar los la- dos conocidos. De este modo podemos establecer que: cot 6 = rh/3-D r cot e = V3 -1 28.- Completando una semicircunferencia y prolongando MN y BO encontramos que Z OBN y Z OMP son congruentes. En el L PNB: NB = 2R cos 6 En el L POM: OM = R cot 6 Luego, en la figura: AM + OM = AO 2R cos 6 + R cot 6 = R => 2 cos 6 + cot 6 = 1 Por consiguiente: M = 1 ÁREA DE UNA REGIÓN TRIANGULAR 29.- Reemplazando datos y aplicando los teoremas de resolución en la figura dada, se puede establecer que: m Z AED = m Z ABC = a , sea: AC = a EnelLADC: AD = csena a CD = ccosa Si: k = área de la región triangular ADC , (AD)(CD) 1 , => k =------2-----= 2 (fl sen a' cos a' R4 310 Problemas de Trigonometría y cómo resolverlos RACSO D1TOBIB
, CT ÍT k k=- sen a. eos a => - = sen-acos¿ En el L ACE: CB = AC. cot a = a cot a Sea: S = área de la región triangular ADC _ (AC)(CB) _ flflcot a _ fl2.cota _ k cosa - 2 - 2 2 senacosa ' sena S = fc csc2a 30.- Nuestra estrategia consistirá en determinar los valores de la tangente y la cotangen- te de los ángulos dados por medio del cálculo de áreas de cada triángulo dado. De la figura: Además: cota S=— ---(1) 4S=») (2) Dividiendo las relaciones (2) + (1): 9cotp cota cotp _ 4 cota 9 De donde, obtenemos: cot p = 4* => tan P = — 4* 1 a cot a = 9k => tan a = — 9k Luego: 9.4*-1 2jk 4.9*-1 36*-1 2jk 36*-1 M=| 31.- Observando la relación de áreas, podemos afirmar que: AD = 2DB. Si hacemos: BD = b => AD = 2b a BC = h bh AD => S = — => 2S= — b En el L DBC: tan 6 = ~ h h En el L ABC: tan 0 = — => AD = 2b ...(1) --(2) 2 1 1 Multiplicando (1) x (2): tan 0 = ~ => tan 0 = 6 = 30° Resolución de Triángulos Rectángulos
32.-Construimos el LAHB, podemos establecer que el área de la región triangular AOB. Luego: 4.4sen20 S= 2 S = 8 sen 20 A 4 OH 33.- Tal como se ha hecho en los ejercicios anteriores, aprovecharemos la fórmula del área de una región triangular para determinar el valor de la razón trigonométrica pedi- da. Con este propósito trazamos la altura EH del triángulo ABE y, colocando los datos en el gráfico se establece que: ED = k => CD = 3k 3k(3k) 9 , Area A ABE = —- = ~k2 ... (1) 1 Pero: Área AABE = ~ (EB)(EA). sen 0 ... (2) Por el Teorema de Pitágoras, tenemos: En el L ECB => EB = J(3kf +(2k)2 = -J13 .k En el L EDA => EA = -jk1+(3k)2 = JÍO 1 i— i— 9 , Igualando (2) y (1) tenemos: — (<13 íc)( <10 k) sen 0 = — k J130 . sen 0 = 9 csce=^ 34. - Sea AB = a ; DE = h y tracemos FD ± CA En el ABC: AC = a . esc a En el L AED: AD = h . esc a. En el L ABC: m Z FA’3 = 90° - a ... (1) En el L AFD: m Z FAD + a = m Z FAB... (2) Reemplazando (1) en (2): m Z FAD = 90° - 2a => m Z FDA = 2a De este modo: FD = AD . cos 2a RACSO DITO1IB Problemas de Trigonometría y cómo resolverlos
FD = h . csc a. cos 2a Según condición: Área región AABD = Área región A ADC (acsca)(hcsca.cos2a) ah , -------------------- = — => cos2a. csc a = 1 2 2 _ 1 1 Área L ECB = -^ab sen 0 36 .-Sea CD = k => BC = 4k AD = 4/c Trazamos AG//BD y prolongamos CD hasta que se intersecten en el punto G. Siendo ángulos opuestos por el vértice, se observa que m Z BFA = tn Z EFD = a Siendo ángulos correspondientes, se ob- serva que: tn Z G AE = m Z EFD - a En el L EDA: EA = 2 Víí k En el L ADG: AG = JÍ7 k Calcularemos el valor de la tan a a partir del cálculo del área de la región triangu- lar AEG: Resolución de Triángulos Rectángulos
37 .- De la figura: hcot a + /icot 0 = b Factorizando: h(cot a + cot 0) = b b cota + cot0 Luego el área de la región triangular (S) será. bh b2 S = 2 ’' S = 2(cot a + cot 0) 38 .- Irücialmente, calculamos el área del triángulo MDN, así: . base altura a-a a2 aamdn _ 2 “2 “2 .........* * Y de acuerdo con lo visto, tendremos: A^mdn = sene.....(**) Luego, igualando (**) con (*): a sen 6 = ~~ => sen0=^Jj A partir de este resultado construiremos un triángulo auxiliar en donde la hipotenusa sea -JÍO y el cateto opuesto a 0 sea 1. Allí resultará evidente que el otro cateto es por Pitágoras, igual a 3. 3 Finalmente: cot 0 = y cot 0 = 3 Problemas de Trigonometría y cómo resolverlos »¡<RACSO UiDITOHI
R. T. en Situaciones Contextualízadas ÁNGULOS VERTICALES 01.- Con la ayuda del gráfico y la propiedad de los ángulos altemos internos, la condi- ción del problema queda así: tan a - tan 0 - tan 6 = 4c 2c c _ 1 9 ' 9 ' 9 “ 4 el 9 9-4 => a~ 4 Finalmente la altura del edificio será: Altura = 4c = 4j = 9 m 02.- Del triángulo superior, tenemos: cos 30° = y 4^3 -Jl 4>/3 3 ’ 2 3 => y = 2 Finalmente en el triángulo inferior tenemos: y eos 15' cos 15° = x = - -J2) m 03.- Finalmente: - h = G0m 4--------------------------4 Razones Trigonométricas en Situaciones Contextualízadas
04.- Elaboramos un gráfico en donde indi- camos que el objeto y su imagen equidistan del lago, quien hace las veces de un espe- jo. Luego en el triángulo sombreado, ten- dremos: sec 2a = p x = H sec 2a 05.- Haciendo un gráfico adecuado como el mos que es posible aplicar allí el teorema a x a = 5k 5 = 4 =* x = 4Jt A continuación aplicamos el teorema de Pitágoras en el triángulo rectángulo ma- yor así: (5k)2 = (4k)2 + 92 => 25k2 - 16k2 = 81 Luego: 9k - 81 => k2 = 9 > 9 De este modo: x - 4(3) = 12 => tan2a = — mostrado, nota- 3 tan2a=^ 07.- Con la ayuda del siguiente gráfico, observamos que resulta conveniente trasladar el ángulo dado al trián- gulo rectángulo SAO. cos 30° = => r = R cos 30° 1\ r = 5516 km 316 Problemas de Trigonometría y cómo resolverlos
08.- En el triángulo notable de 45° a 45°, tendremos: Finalmente: h = 2,366 tn x 1 000rn 1 km h = 2 366 tn 09.- Construimos el gráfico, según el enunciado. En el triángulo rectángulo sombreado se cumple: (2,5)2 = (4k)2 + (2,5 - 3/c)2 => (2,5)2 = 16k2 + (2,5)2 - 15k + 9k2 Reduciendo: 25k2 = 15/c => 3 k= 5 5 5 9 Luego: x = ~ -3k = ~ 7 45° 3k r-2^-3k n ♦ Mosca Pajarito x = 0,7 tn ~ 10 m 10.- Elaboramos una gráfica, en ella se observa: h cot a = 3h + h tan a => cot a - tan a = 3 Elevando al cuadrado: (cot a - tan a) = 3 2 2 => cot a + tan a - 2 tan a. cot a = 9 2 2 tan a + cot a = 11 Razones Trigonométricas en Situaciones Contextualizadas
11.- En el gráfico elaborado se debe verificar que: tan a = 2 Asimismo: 5k = 20 m => k = 4 m También: x = 4k - 3k cot a = 4k - 3k. - => x = 4(4) - 3(4). ~ = 16 - 6 x = 10 nt 12.-En el LRST: RS = ST cot 60* pq EnelLPST: cot 30° = = 5 x= 10 nt 13.- Luego de elaborar el gráfico, reconocemos que: e = 600(cot 0 - cot a); e = v.t => e = 600 m , 600nz 60niin Ikm 11 1000,» v = 6 14.- Al elaborar el gráfico según el enunciado, se pueden reconocer tres triángulos. En el LPRS: RS = PR tan 37° => RS = 36 En el LPRT: cot 53 = = RS+gT 3 _ 36 =* 4 — 27 + h => 27 + h = 48 .’. h = 21 ni R5 318 Problemas de Trigonometría y cómo resolverlos í» RACSO Pbditobbs
15.- Suponiendo que el avión se mueve en el mismo plano vertical que contiene a los puntos A y B, se tendrá que: h cot 6 = h cot P - h cot a 1 cot 6 = — 1 1_ 3 = 6 tan 0 = 6 16.- En el gráfico elaborado la antena está representada por CD En el L ABC: h = d tan P En el L ABD: h + 15 = d tan a (2) : (1): h + 15 _ tana h ~ tanP Pero: tan a = 0,76 y h+15 ~r=* h = 5 nt 17 .- En el L ADB se verifica que: x 2^ = 16 cos 0 . sen 0 => x - 16.2 sen 0 . cos 0 x = 16 sen 20 18 .- Elaborando un gráfico correspondiente del contexto en el que se desarrolla la situa- ción, podemos reconocer que el triángulo rectángulo formado es un triángulo rectángu- lo notable. Veamos: Punto máximo 0 = 30° Razones Trigonométricas en Situaciones Contextualizadas
19 .- Sea A el punto desde que se soltó el objeto y B el punto en el que se volvió a visualizar. Entonces elabo- ramos el gráfico correspondiente, tal como el que se Objeto,oA muestra al lado, dende: • > h = 60 . cot 60° . tan 30° I* Obtenido a partir de la resolución de triángulos rectángulos y' => h = 60. ----- ---- m = 20 m s . i lsj(3j :-----di ----------------------------------------------60. cot 60°-----1 h = 20 60 tn ÁNGULOS HORIZONTALES 20 .- Sabemos que: N 1/4 NE = N11°15’NE e Luego, en la figura: ÁPB = 45° + 0 = 45° + 11° 15' ÁPB = 56°15' .-. m ÁPB = 56°15' 21 .- Por Física sabemos que: W=7 w--Velocidad angular 0 - - Angulo en radianes t - - tiempo Siendo A la posición del amigo observado y P y P' las posiciones del observador o lrad- => CÜ=T =—------ i 0,15 15 1 vuelta 2n rad. 60 x Imin. => (ü= 15x2 ~ mm. vueltas (á = 5°-^ 100 El carrusel da 50 revoluciones en un minuto. 50 RPM E 45° Problemas de Trigonometría y cómo resolverlos £jÍRACSO WlDITOlIl
22 .- Tratándose de desplazamientos rectilíneos, lo conveniente es elaborar un gráfico de todos ellos, tal como el que se muestra al lado: x + 20 tn = 40 m x = 20 m 23 .- En el LBCA reconocemos que: CA = 32 km, CB = 24 km Asimismo, en el LBID: DI = BI = 40 km => IC = 16 km En el tXAHF: AF = J(FH)2+(HA)2 AF = + 162 = 20 km cu- . o FH También: tan 0 =- HA 12 16 37° Finalmente el auto se encuentra en la dirección S 37° O respecto del punto de partida y a una dis- N <X Posición partida 3o ¡24 (A) o !16 20 n 60 km ®!16 I F H Posición "* final O1) tancia de 20 km. 24 .- Elaborando el gráfico correspondiente a los desplazamientos efectuados, logramos identifi- car que: 1) x = 1G^2 + j2 sen 22° 30' x = 10 J2 + J2 . = 5^2 2 2) y = 10^2 + 72 - cos 22°30' y=10-j2 + V2 . =5(2+ Jí) y - x = 10 ktn Faro N NNE Razones Trigonométricas en Situaciones Contextualizadas R5 321
25.- Elaboramos un esquema de acuerdo con los datos proporcionados en el que se pue- den establecer las siguientes observaciones: 1) El AABC es isósceles dado que: AB = BC 2) Asimismo, por ángulos alternos internos: 6 0 + a = 20 - a => 2a = 0 => a = ~ Por lo tanto la estación A respecto de C se encuentra en la dirección: S(20 - a)O sfo 26.- Uniendo el punto de partida con los puntos de llegada de cada móvil, se forma un triángulo en el que son conocidas las medidas de sus lados. Asimismo se puede reconocer que estos valores verifican la siguien- te relación: (5,8)2 = (4)2 + (4,2)2 Dicho triángulo es recto en P. 7t P + 2P=- .’. P=|r«d 27.- Graficando el enunciado, siendo: P el punto de partida R el punto de desvío T el punto final de la lancha. Dato: PT = RT => A PTR es isósceles => TPR = PRT = 30° Así: a = 15° La dirección es: S75°E N >E S Problemas de Trigonometría y cómo resolverlos ARACSO paDiTOiaa
28.- De acuerdo con el enunciado, las perso- nas ubicadas en A y B se desplazan rectilíneamente hasta encontrarse en C. Se observa que sus direcciones forman los án- gulos de medida 150° y 30°. Por lo tanto el menor ángulo formado por ellos mide 30°. N 29.- Al hacer la construcción gráfica, reconoce- mos la presencia de un triángulo rectángulo no- table uno de cuyos ángulos agudos es: 53°/2. Aplicando la proporcionalidad de los lados: JO = 200 ; IG = 100 J5. Y en el L IHG: O<— E > /'53°'\ r- 2 IH = IG.cosI — = 100 V5 . -j= l 2 I J5 IH = 200 í 53° i r~ 1 GH = IG.sen ---- = 100 V5 . —¡= => GH = 100 l 2 I J5 '%6°30'-53°/2 _ -fc "->-----s I E H S Finalmente en el LJHG: JH _ JQ + IH _ 200 + 200 GH ” GH ” 100 cot 0 = 4 30.- Este tipo de situaciones problémicas requieren de una buena dosis de paciencia. Ya que la estrategia consistirá en trazar, sobre un esquema, las direcciones más usuales, cuyas prolongaciones nos indicarán sus respectivas direcciones opuestas. La dirección opuesta al NE~ E es la dirección: SO O Razones Trigonométricas en Situaciones Contextualizadas
31.- Se sabe que a = 11°15' es el ángulo que re- presenta la medida de un ángulo en el sistema sexagesimal entre dos direcciones contiguas. De la figura: El mayor ángulo "P" será: P = 180° + 4a = 180° + 4(11°15') P = 180° + 44° 60’ .-. P = 225° 32.- Tal como hicimos en el problema 20, elaboramos las direcciones correspondientes para cada proposición. Se verifica que el ángulo formado por dos direcciones sucesivas es: a = 11°15'. a) El mayor ángulo es: 360° - 6a = 360° - 6(11°15') = 292,5° b) El menor ángulo es: 180° - 4a = 180° - 4(11°15') = 135° c) El menor ángulo es: 90° + 4a = 90° + 4(11°15') = 135° Por lo tanto tenemos: a) F b)V c)F PROBLEMAS TRIDIMENSIONALES 33 .- Situaciones problémicas de este tipo exigen la elaboración de un esquema gráfico tridimensional, en el que se puedan colocar los datos correspondientes. En nuestro caso el pla- no horizontal es el lugar donde se hacen los tra- zos de las direcciones náuticas. En el LPQR: (Ii ¿3 )2 + (/i)2 = 802 => 4h2 = 6400 => h2 = 1600 80 R O-»,-' Boya S h = 40 m Problemas de Trigonometría y cómo resolverlos RACSO
34 .- Sea QR = a En el LQRP: PR = QR . tan a = a tan a En el LPRT: TR = PR tan a = a tan2a En el LQRT: cot P = QR TR cotP = a 2 otan a cot P = cot2a 35 .- En la figura: m Z IHD = m Z DHF = Asimismo reconocemos que el ADFH es i guio e isósceles. En el LHFB: HF = hi. tan a => HF = FD = BC En el LHDA: HD = h2 tan a En el L HFD: Por teorema de Pitágoras (HD)2= (HF)2 + (FD)2 2 2 2 2 2 2 => h2 tan « = hi tan a + fq . tan a => h2= En el BCA: tan a = -——— => tan2a tana => tan a = : que permite elaboi el gráfico adjunto. seca= $2 36 .-Sea PQ = 3j2m En el LBAQ es isósceles => AQ = AB = /z. h EnelLBAP=> AP = ABcot60°=> AP = Razones Trigonométricas en Situaciones Contextualizadas R5
En el L APQ: Por el teorema de Pitágoras. (ÁQ)2 = (ÁP)2 + (PQ)2 2 2h2 r~ => ---- -18 => /i = 3V3m 3 EnelLQCD: QC = CD cot 30° = h . J3 = 3 V5. V5 = 2 Aplicando el teorema de Pitágoras : (PQ = PC = V63 PC = 3V7wi 37 .- En la figura: AR = x tn ; DR = y tn Según datos: AB = 7,20 tn ; CD = 9 m En el LGDI y el LGRF tenemos: 9(7,20) + 7,20y = 9(7,20) + 9x JfACSO BDITOBia Problemas de Trigonometría y cómo resolverlos
39.- En el L AMB: Hacemos: MB = 4k AM = 3fc => HR = 3k a PR = 2k n . 2 Pues: tan a = L PRH' = IXH'BQ 2k _ _L 4k ~ BQ BQ = 2m La sombra mide: 2 m 40.- Sea: CD = h; AG = H EnLiDC => FD = /icotO En L FGA => FG = H cot 0 En L FDG => es isósceles => FD = FGsen45° 1 h cot 0 = H cot 0 . .-. H= 42 h. En t\CBA: tan a = AB AB AB CB DG FD H-ft h(42-l) tana= ~~ = —----------- /icotO Jicote = (J2-I).tane Razones Trigonométricas en Situaciones Contextualízadas R5 327
CAP.6 R. T. de Ángulos en el Plano cartesiano RAZONES TRIGONOMÉTRICAS DE UN ÁNGULO EN POSICION NORMAL 01.- Graficando según los datos del problema, reconocemos que: Luego: x=6 ; y = -l r= ^62+(-lf = 437 Finalmente: E = V37 6 ’ (-D E = -43 02.- Elaboramos el gráfico correspondiente indicando el ángulo en posición normal a. En el triángulo rectángulo PHO se verifica que: r = OP = ^(—2)2+32 = ^13 , r J13 r JA3 Luego: esc a = — = —o- a sec a = — = —— J/p J *p ~2 Reemplazando en W obtenemos: 3 2 w= - 2 2 4 4 03.- De la condición: 9 tan a -16 = 0 => tan a = a tan a = - 7t -4 Dado que: 7— < a < 4ti => ae IVC => tan a < 0 => tan a = — ... (*) Sea P = (xP ; yP) un punto que pertenece al lado final del ángulo a en posición normal. Luego al aplicar la R.T. de (*), se puede establecer que: Vp Vp —4 tana=— =* - = ^ Vp = 4 xp=3 Haciendo: yP = -4 ; xp = +3 R6 328 Problemas de Trigonometría y cómo resolverlos ¿RACSO
Luego: r = OP = -^(-4)2 +(3)2 = 5 r 5 CSC Vp - 4 ; cot a = ~ xp -4 -3 4 5 Finalmente, obtenemos: W = — 4 - - — - -2 4 - 4 - 2 W = -2 04.- Efectuando como sigue: - 0,25 = sen x + 2 sen x + 1 9 ' , — = sen x [1 + sen x + sen x + sen x +... J => 4 Como: <x<2n re IVC Por el teorema de Pitágoras: p (y?)2 => xP = ^32-(-i)2 = 2J2 Luego- W = 12 (sec x - cot x) 3 2^2 2V2 -1 3 + 8 2 |_2j2_ = 11 2 1 4 sen3x +.. 2 2 3 +15 25 "n"tér minos 05.- Escribiendo como sigue la condición: sen0 = - — 2 2 Luego: 1 1 sen 0 = - — 2 111 3 + 3 ' 5 1 5 1 7 1 1 2n-l ' 2ii + l sen 0 = — 2 1 1- 2n + l -n sen 0 = -. 2w +1 Como: sen 0 < 0 y eos 0 < 0 => 0 e IIIC Luego de (*): y = -n , r = 2n + 1 , x <0, siendo (x; y) un punto del lado final de «0» y r su radio vector. x2 = (2n + l)2 - n2 = (2n + l)(n + 1) => x = - J3ii +1 . J11-I Vn + 1 Luego: M = J—. -n (2h+1) M = „--7 = 1 3m + 1 Razones Trigonométricas de Angulos en el Plano Cartesiano 329
íai----Vena 7 -4® 06.- Como: (vcosaj = (sec a) sena 1 1 (cosa) 3 = (seca)7 = (cosa) 7 Luego: . sena _ 3 — 7 3 sena = - ~ => x2 = T1 - (-3)2 = 40 => x = 2y[W Finalmente, reemplazando en M tendremos: M = 7 7 7 2J10 -3 M = 0 SITUACIONES GRÁFICAS 07.- Nuestra estrategia consistirá en determinar las coordenadas del punto de intersec- ción P; resolviendo el sistema de ecuaciones dado: x + 3y = -7... (1) 5x+2y = 4 ...(2) V Resolviendo (1) y (2): x = 2 ; y = -3 Al graficar, se deduce que: r = J13 Finalmente: 7 W = tan a + sec a = ... -3 2 13 7 W = —. — + — — 2 2 4 4 W = 1,75 08.- Utilizando la ordenada del punto «D» y haciendo que: 4k = 4 R6 330 Problemas de Trigonometría y cómo resolverlos *%’ACSO p BDITOKBB
09.- Sea OA = OD = R, radio de la circunferencia. Reconociendo que el triángulo AOD es isósceles, trazamos EO -L AD . De este modo se evidencia que EO es altura, mediana, mediatriz y bisectriz interior del AAOD. Hagamos que: En el L COB => EnelLCFD => En el L AGB => AB = BC = CD = k OC = OB = 2 CF = FD = 2 „A ™ GA = GB =-- 2 De este modo podemos reconocer que las coordenadas de A y D son: A = (*a ' Va) = a D = (rD; yD) = W2 Finalmente: W =Jan 6 - tan <J> => W = 1,5 10.- Sean P y Q dos puntos que pertenecen a los lados finales de los ángulos P y a. Tenien- do en cuenta que la gráfica corresponde a la función: y = -21 x |, evaluaremos asi: Razones Trigonométricas de Angulos en el Plano Cartesiano R6 331
11.- Nuestra estrategia consistirá en determinar las coordenadas de los punto de intersección entre los lados finales de los ángulos dados y la circunferencia. Para este propósito asumiremos que el radio de ésta mide 3a, con lo cual se puede utilizar la relación dada entre los segmentos OA, AB y BC. Luego se tendrá: Punto P: = -J5 Punto Q: M =0 Luego: M = -V5 + J5 12.- En el L OCD: r2 = (r - a)2 + (2a)2 => r2 = r2 - lar + a2 + 4fl2 => Zar = 5a2 5 5 3 Luego: r=~a => r-a-~ a-a=~ a 3 De este modo: P(-a; ~ a) —a 7 Luego: cot 6 = « /. cot G = ó PROBLEMAS CONDICIONALES 13.- Aplicando la propiedad dada, las coordenadas del baricentro B será: Elaborando un gráfico de B y sus coordenadas, aplicamos el teorema de Pitágoras: r= ^32+(-4)2 =5 De este modo se tendrá que: W = 5(sen a - cos a) => W = 5 ["y" 5] W = -7 Problemas de Trigonometría y cómo resolverlos RACSO Wdiioiii
14.- Sobre el gráfico dado, indicamos el ángulo trigonométrico «a» reconocemos que el Z HAB es notable, nuestra estrate- gia consistirá en hallar las coordenadas del punto D. En el L DHC: HC = ó.tan 3J‘= 2 Por congruencia entre LCPD y LCHB: HC = PC = 2 Se pueden hallar las coordenadas de «D»: D = (-7;6) Calculamos: tan a = 6 -6 -7 ~ 7 A cot Efectuando: W = =6 +zZ 7 + 6 1 “ 42 15.- Si tan a < 0 => o. e IIC ó IV C .... (1) Como las bases son no nulas e iguales, la igualdad se verifica si los exponentes son iguales. Luego: -5 2 esc o. = — => -5 5 esc a = - — = — ... (4) De (3) y (4), aplicamos el teorema de Pitágoras y determinamos que: xr = 3 W = 5 cos a - 4 cot a Í3 W = 5 - 5 W = 3 + 3 W = 6 Razones Trigonométricas de Angulos en el Plano Cartesiano 3331
16.- Como: i) tan 6 > 0 => G G IC ó IIIC ii) sen 6 < 0 => 6 G IIIC ó IV C De (i) y (ii): G 6 111C => eos G < 0 2 2 => | eos G | = - eos G => - eos G > ~ => eos G < - — 2 El máximo valor de eos G es: eos G = => x = -2, r = 3 Por Pitágoras: y2 — 32 - (- 2)2 = 5 Pero: y < 0 => y = - V5 .-. sen G = 17.- Analizando cada proposición tendremos: i) | sen G | = sen 6 => sen 6 > 0 => GgICó IIC ii) |cosG| =-eos G=> eos 6 < 0 => GgIICóIIIC De (i) y (ii) concluimos que: G G IIC -4 4 => | tan G | = - tan G = 1,3 => tan G = - - -i 3 ¿ 5 => y = 4 , _r = -3 => r =5 => csc G = -r j 4 Luego: M = y + M = 18.- Si: | eos P | = - eos P => eos P<0 => Pg IICóIIIC ...(1) Si: | cot P | = cot P cotp>0 => pcICóIIIC ...(2) De (1) y (2) se concluye que: Pg IIIC 2 Si: | sec P | = 2 => - sec P = 2 => sec p = Por el teorema de Pitágoras: yp = - J3 => W = sen P . tan p = V 3 -V3 2 -1 3 Finalmente: W = - — W = -1,5 Problemas de Trigonometría y cómo resolverlos ^RACSO
19.- Nuestra estrategia consistirá en utilizar las condiciones dadas para determinar el valor y signo de tan (ct/2). De este modo sec a se determinará por medio de la relación dada. Veamos: Si: |csc a| = -csc a => csc a < 0 => ae IIC ó IVC... (1) Si: |seca| = sec a => seca>0 => ogICóIVC ... (2) De (1) y (2) se desprende que: a e IVC De la 3ra condición: [ csc a - cot a | = k Sustituyendo el primer miembro: = siaelVC => 3^ <a<2n => 3^ < y <7t; g IIC => |tan^ =-tan^ a (a A En (3): -tan — = k => tanl — I = -k De la sugerencia: secct = l + tan2í — l 2 l-tan2f— sec a = 1+k2 1-k2 SIGNOS DELAS RAZONES TRIGONOMÉTRICAS 20.- Del dato: sen 6 > ^cos<¡>-sen6 a) Por definición la raíz cuadrada es mayor o igual a cero, luego: sen6>0 => OgICóIIC => ~ < 6 < rt b) De esta misma condición se puede afirmar que: cos <J> - sen 6 > 0 => cos <J> > sen 6 Y por el paso (a), podemos afirmar que: cos <J> > 0 => <J> 6 IC o IVC 3ti c) Según condición: <J> > 6 => -y- < <}> < 2n W = tan 6 + cot<i> OeIICxO ¿elVCxO => W<0 W es siempre negativo Razones Trigonométricas de Ángulos en el Plano Cartesiano
21.- De la condición: Luego: De la otra condición : 0 2 nc ti 6 2 < 2 <7t 6 esc — es (+) 0< a <7t Sumando 2n en cada miembro: 2ti < 2ti + a <3n , , „ 2rt + a 3ti 2rt + a Dividiendo entre 2: 7t < < : — => 6 IIIC 2 2 2 ( 2rt+a 1 es (+) Entonces: tan l 2 Finalmente el signo de M será: M = (+) + (+)= (+) 22.- Por definición de valor absoluto: | sen 01 = - sen 0 => sen 0 > 0 => sen 0 < 0 => 0 e IIIC ó 0 e IV C... (*) De esta conclusión: esc 0 < 0 Luego de la condición: esc 0 - cos 0 > 0 => COS0<O => 06 IICÓ06 IIIC...(**) De (*) a (**) concluimos que: 0 6 IIIC => 180° < 0 < 270° 0 0 0 Dividiendo por 3 se tiene. 60° < ~ < 90° => —e IC => sen— es (+) 0 0 0 Asimismo: 45°< ~ < 67,5° 4 => -elC => cot- es(+) 0 0 Del mismo modo: 90° < ~ < 135° => 135° < - + 45° < 180° => (f+45°) G IIC => sec (®+45°) es(-) Finalmente el signo de M será: M = (+). (+). (-) M = <-) 23.- Como: i) sen 0 < 0 => 0 6 IIIC v IV C ii) cot 0 < 0 => 0 6 IIC v IV C 3ti De (i) y (ii) se puede concluir que: 0 6 IVC => — < 0 < 2rt ...(*) 3n 0 0 6 a) Dividiendo (*) por 2, se tiene: ~q~ < — < n => — 6 IIC => sen — = (+) R6 336 Problemas de Trigonometría y cómo resolverlos ACSO IDITOlll
26 4n b) Multiplicando (*) por 2/3, se obtiene: n < — < — 37t 6 TI c) Dividiendo (*) por 8, se obtiene: 16 < 4^ < 4^ 26 26 => ~3 e IIIC => tan — = (+) e 6 - g IC => sec - = (+) 24.- Colocándolos ángulos en posición normal para ello trasladamos el origen del sistema. ac IIC 6 g IIIC Se dice: => |cota| =-cota => | cot 61 = cot 6 & P’ / H = cot 6 + (-cot a); aplicando definición de R.T.: IT -n (n-2a\ n-n + 2a H= -b b )= b rl 2a H- b 25.- Nuestra estrategia consistirá en analizar radicandos que figuran en los términos de la desigualdad dada: a/-sec 6 < 0 < -J— sen6 i) -sen 6 > 0 => sen 6 < 6 => 6 g IIIC ó IV C ii)-sec6 <6 => sec6 > 6 => 6gIC6IVC De (i) y (ii): 6 6 IVC Luego el signo será: M = (- ) + (- ) (+) = (- ) + (- ) M = (-) RAZONES TRIGONOMÉTRICAS DE ÁNGULOS CUADRANTALES 26.- Los fundamentos en los que nos basaremos, para la resolución, provienen de la definición de raíz cuadrada. Así tendremos que: Si: cos P = Jsen6-jsena => cos P > 0 a sen a > 6 a sen 6 > 0 ... (*) También: sen 6 > sen a... (**) Puesto que: a, P, 6 son ángulos cuadrantales ubicados en el intervalo [0° ; 270°], las relaciones de (*) y (*•) se verifican si: sen 6=1; sen a = 0; cos P = +1 6 = 90° , a = 180° , P = 0o Razones Trigonométricas de Angulos en el Plano Cartesiano
Luego: eos (a + P + G) = cos(180° + 0° + 90°) = eos 270° = 0 eos (a + P + 6) = 0 27.- Analizando los radicandos de la expresión "M" , se deduce que: a) 0-2>O b) 4-0>O e>2 e<4 2<G<4 El menor ángulo cuadrantal que pertenece a [2 ; 4] es: Tirad ~ 3,1416 rad => R = sen Tt + tan rt + eos Tt => R = 0 + 0 + (-l) R = -l 28.- Como: sen a = tan G + 1 a a y G son cuadrantales, podemos inferir que: a) sen a = 1 => a~ 2 b) tan 0 = 0 => G = 0 ó Tt Dado que 0 es positivo: G = Tt , ., senn+sen3Tt/2 0-1 Luego: M =----------------= tan— 4 M = -l 29.- Al inspeccionar la expresión "M", se puede reconocer que: a) a * — ; — => a = 0 ó a = Tt => eos a = +1 v eos a = -1 2 2 b) P*0, Tt, 2rt => P = — ó P = 3~=> cot P = 0 a eos P = 0 A continuación inspeccionaremos cada término por separado: B= Jcotp-cosa => cosa = -l => a=Tt => B=1 ---------------- Tt C =^/cosa-cosp + senG => senG = l => G = — => C = 0 .--------------- Tt A = Jtana + senp + cosG => senp = l => P=~ =* A=1 Finalmente: M=l+0 + l M = 2 Problemas de Trigonometría y cómo resolverías ^íbracso PlDITOMBa
RAZONES TRIGONOMÉTRICAS DE ÁNGULOS COTERMINALES 30.- Como: sen a = - — y a g IIIC a) De la definición del seno: y = - 15 a r = 17 => x2 = 172 - (-15)2 => *2 = 8 => x = -8(x<0) b) Luego como "ct" y "0" son coterminales: „ -15 15 => tan a = tan 6 =---- = — -8 8 c) De la figura: a - 6 = 2rt => tan (a — 6) = 0 Finalmente: M = — + — +0 .*. M = — 8 8 4 31.- Sean "a" y "P", (a > P), los ángulos coterminales dados, nuestra estrategia consistirá en establecer las relaciones existentes entre estos ángulos que nos permita determinar el número (n) exacto de vueltas que los diferencia, para que sea posible identificar al menor de ellos a partir de sus valores. Veamos: a) De la Ira condición: a + P = 1320° .. (1) b) De la 2da-condición: 900° < ct < 1200° _(2) c) Por teoría se sabe que: a - P = 360°n , n e Z ... (3) Efectuando (1) + (2) y simplificando: ct = 180° n + 660° .. (4) Reemplazando (4) en (2): 900° < 660° + 180° n < 1200° 1,3 < n < 3 n = 2 Reemplazando en (4): a = 180° (2) + 660° = 1020° Reemplazando en (1): P = 1320° - 1020° = 300° .’. «300°: es el menor» 32 .- Decodificando la proporción establecida en la condición del problema, tendremos: 26 13 13 —E- = => 46P = 13a + 13P => 33P = 13a P=— a (1) a+P z¿s 33 Y según la otra condición, se debe cumplir que: a - P = /c(360°) ... (2) Razones Trigonométricas de Ángulos en el Plano Cartesiano
Sustituyendo (1) en (2): a - — a = k (360°) 20 — a = *(360°) a = *(594°), keZ De (1) reconocemos que a es el mayor, luego según condición del problema: 1100° < a < 1300° 1 100° < A(5940) < 1 300° 1,85 < k < 2,18 k = 2 Finalmente el mayor ángulo mide: o. = 2(594°) a = 1 188° 33 .- Procediendo como en el ejercicio anterior, sean a y P los ángulos coterminales posi- tivos, tal que a > P , entonces se cumplirá: a- P -A(360"), fceZ De la condición: a 7 P " 2 7 - P - P = A(360°) 5 2P = A(360°) P = ¿(144°) .. (*) Pero también de la condición: 1 200° < a — P < 1 500° => 1 200° < fc(360°) < 1 500° => 3,33 < k < 4,167 K = 4g Z En(*): P = 4(144°) ct = 576° 34 .- Sean a y P los ángulos coterminales negativos, tal que a > P de la condición: p = 7k y a = 5k, lo que por ser coterminales verifican: 7k - 5k - 360°h —> k = 180°n Además se sabe que: 540° < a - P < 900° => 540° < 7k - 5k < 900° => 540° < - 2k < 900° => 270° < - 180°n < 450° => 3 < -2n < 5 De donde reconocemos que: n - -1 (único entero que verifica la desigualdad) Finalmente el mayor de los ángulos mide: a = 5k = 5(180°) (-1) a = -900° Problemas de Trigonometría y cómo resolverlos "A Racso
MISCELÁNEA 35.- A partir del gráfico notamos que los valores de x e y son negativos; por ello al utilizar el dato de la tangente y expresarse bajo la forma de una razón, tendremos: tan p — 1,5 — iQ — ~ De donde: secP=2^,y, cscP^y- 1 IV13 V13 -3+2 6 25 1 36.- Por dato: sen a = 0,25 = Como: a G IQ, podemos reconocer que el valor de x es positivo, tal como se indica en el gráfico. Luego apli- cando Pitágoras encontramos: J . ,2 /2 2 , , ,, 15 2 4 Finalmente: esc a + cot a = j + .2 = 4 + 15 2 esc a + cot a = 19 37.- Del dato: -Jl + cot2a =8 => 1 + cot2 a = 64 cot2 a = 63 => cota = ±V63 Como: a g IIIQ, entonces cot a es positiva Luego: cota= <63 Asimismo: r2 = i + 63 = 64 r = 8 Finalmente: (8 sec a)3 = 8. (8 sec a)" ----j= 63V63 X Razones Trigonométricas de Angulos en el Plano Cartesiano Re 341
38.- Haciendo uso de los valores de las razones trigonométricas de los ángulos cuadrantales, tendremos: «2sen^ + 2flbcos0 - b2sen^ R =-------*--------------------- (a-b)2 cos720° + 4ab ,,, . „ a2+2ab+b2 Efectuando, obtendremos: R = ~—rrj—“7 (a-b) +4ab R=______________ n2 + b2 -2ab+4ab R= + a2 +b2 +2ab R=<^ (a + b) 39.- Efectuando, las potencias de potencias, tendremos: (eos a)1/8 = (eos a)-5*31 a Dado que trabajamos en el campo de los números reales diremos que : Si las bases son iguales, los exponentes también lo serán: 1 3n => sen <x = - g , y puesto que: -£- < a < 2n => aelVQ A partir del gráfico : x2 + (- l)2 = 82 => x2 = 63 x = Vó3 =3-j7 Finalmente: cot a - eos a = 3-J7 3J7 -1 ' 8 cot a - eos a = - 40.- Del gráfico podemos calcular "R", para lo cual aplicaremos la distancia entre dos puntos; así tendremos : R2 = (2a)2 + a2 = (-4)2 + (-a)2 => 4a2 = 16 Luego: a = 2, y puesto que : R2 = 5a2 => R = 2 ^5 Ahora por definición tendremos que: y -a -2 o x 2a 4 sena=R=y'CosG=R=T=i^ 2 2. 4 16 20 2 2 Finalmente: sen a + eos 6=™ + ™ = ™= l sen a + eos 6 = 1 R6 342 Problemas de Trigonometría y cómo resolverlos *j*RACSO WlDITOl I R = 1
Circunferencia Triaonométrica LÍNEA TRIGONOMÉTRICA SENO 01.- De la C.T. se observa que para el rango de variación de 0: n < 0 < , la linea del seno varía de modo que: '1 02.- Como: 30° < 0 < 150°, en este rango de valo- res la linea del seno varía de modo que: 1 — < sen 0 < 1 => 1 < 2 senO < 2 4 < 2 sen 0 + 3 < 5 M G [4; 5] 03.- De la C. T. se observa que para el rango de valores de 0, la linea del seno varía de modo que: => -1 < sen 0 < 0, es creciente Sustituyendo el seno por la expresión dada: 4(x-2) + 3(x+l) - 12 < 7x - 5 < 0 M Circunferencia Trigonométrica
- 7 < 7x < 5 04.- Nuestra estrategia consistirá en determinar el rango de valores de 12x | y a partir de éste reconocer el intervalo de valores correspondientes del seno Veamos: 7t 71 05.- Para este caso procederemos como en el ejercicio anterior. Como: - ~ < x < ~, la linea del seno varía de modo que: 2 1 0 < sen x < — - 1 <4 sen2x -1 < 1 T ’—M------- T min máx Mmáx + Mmín = ° 06.- A partir de la condición dada para los valores del coseno se puede deducir el rango de valores del arco x correspondiente, que trasladado a la C.T., es como se muestra: Problemas de Trigonometría y cómo resolverlos RACSO DITO1II
De este modo se concluye que: También de la C.T. se observa que la variación del seno para este nuevo arco varía de modo que: 0 < sen We [0;l] 07.- Nuestra estrategia consistirá en transformar la expresión para W, de modo que sea posible analizar su variación a partir de una expresión equivalente. Veamos: Circunferencia Trigonométrica R7 345
En la figura (2) se muestran los segmentos dirigidos que señalan la variación del seno cuyo arco se encuentra definido en el intervalo dado por la expresión (*). Luego: sen(-l) < sen (2 tan 6 + 1) <1 M M G [- sen 1; 1] 09.- Trabajando con el intervalo de definición del arco se tiene: LÍNEA TRIGONOMÉTRICA COSENO 10.- Según los datos tenemos: Completando cuadrados obtenemos- 2 M = 1 + eos x + eos x M= (cosx+lj Sabemos por la variación de la línea coseno: -1 < eos x < 1 ; Vxe R 2 4 Sumando 1/2 a cada miembro: Elevando al cuadrado: 1 1 3 2 -cosx+ 2"2 0<(cosx+l)2 <| Sumando 3/4 cada miembro: M .-. M G [3/4; 3] R7 346 Problemas de Trigonometría y cómo resolverlos Mracso
11.- Por la variación de la línea coseno sabemos que: - 1 < cos 6 < 1 , V 6 e R Multiplicando por "7"a toda la expresión: Sumando -3 a cada miembro: - 7 < 7 cos 6 < 7 - 10 < 7 cos 0 - 3 < 4 Dividiendo por 2 a toda la expresión: 7cos0-3 2 Comparando esta expresión con la propuesta: 7cos0-3 2 Concluimos que: 12.- Sea la circunferencia trigonométrica. I. De la figura (1) se observa: cos 1 > cos 3, es correcto. II.De la figura (2) se observa: | cos 41 > cos 5 es correcto III. De la figura (3) se observa: cos 2 > cos 3, es Incorrecto 13.- De acuerdo con la condición del problema: 3 ~ < G < 2n a = - 5 /\ b = 2 a + b — - 3 Y de la C.T. se observa que en este intervalo la variación del coseno está dada por: 0 < cos G < 1 (creciente) => 0 > - 2 cos G > -2 => 3 > 3 - 2 cos G > 1 1 3-2cosG 3 /i 3\ => z<—í—<z ••• 3 3 3 \3 3/ Circunferencia Trigonométrica
14.- De la coiuii. »ón dada efectuamos la división indicada, y obtenemos: W=------- cos6+1 + !...(!) Asimismo, si: 6e IC 0 < cos 6 < 1 Sumando 2 a cada término, se tendrá: 1 < cos G + 1 < 2 Elevando a la -1 toda la desigualdad: ~ ~~ < 1 ° 2 cosG+l 2 1 T <2 cosG+l 2 Reemplazando (1) en (2): 2< W<3 W G ( 2; 3> 15.-Procederemos según como se hizo en el Prob.7. Empezaremos determinando el in- tervalo de valores del arco (|*|yjpara el coseno. Para ello, utilizaremos el intervalo dado para x en la condición del problema. Veamos: 5n T <X< 71 5ti T Ahora evaluamos el coseno en este intervalo, para lo cual empleamos el recurso gráfico de la C T., de donde reconocemos que: - -y- >cos >-l (decreciente) -2<2cos(¡x|-j) J2 < 2cos(|x|-^)| <2 We [V2;2] 16.-Reconocemos que la expresión W, se puede transformar completando cuadrados: W - cos((sen 6 + l)2 - 1) Se sabe que si: G g IR 1 < sen G < 1 => 0 < sen G + 1 < 2 => - 1 < (sen G + l)2 - 1 < 3 <- Jz Problemas de Trigonometr a y cómo resolverlos RACSO DITO1II
De la C.T. se observa que al evaluar el coseno en el intervalo correspondiente a su arco, encontramos que sus valores están comprendidos entre los siguientes límites: eos 3 < eos [(sen 6 + l)2 - 1] < 1 => Wg [eos 3; 1] Wmín = eos 3 17.-Nuestra estrategia consistirá en encontrar una expresión equivalente a la propuesta para M, la cual sea posible de construir a partir de las condiciones establecidas por la R.T. que en ella se encuentra. Veamos: 1 a) Analizando el denominador de M, se plantea que: 2 eos a - 1 * 0 => eos a * ~ b) Para poder efectuar la división indicada, transformamos «M» como sigue: 1 7 7 |(2cos(X-l)+| 2 I 7 1 M =----¿,------- => M = --------------r + ~ => M = 777 77 + ~ 2coscc-l 2coscc-l 2 2(2cosa-l) 2 c) Sabemos que: |-1 < eos a < 1J - | 1 1 — 00 < ------s - — 2cosa-l 3 7 7 => -~< 2(2cosa-l) 6 7 2 -°°< 2(2cosa-l) + 2 ~~ 3 => - 3 < 2 eos a - 1 < 1 - (OJ 1 v 1 <----------- <. + 00 2cosa-l 7 7 v 2 - 2(2cosa-l) <+“ 7 1 v 4- 2(2cosa-l) + 2 < + / 2\ M g \ A [4; + °° > LÍNEAS TRIGONOMÉTRICAS: TAN - COT - SEC - CSC 18.- Como: 0 < G < —, trazando la línea secante 4 en la C.T., tenemos: 1 < sec G < -J2 . Luego: 4>h-3 sec G = —-— 4m — 3 Multiplicando por "2": 2<4»íz-3 < 2 V2 Sumando 3: 5<4m < 2V2 +3 Dividiendo por 4: 5 2^2+3 , — < m<------ 4 4 2 2 Circunferencia Trigonométrica
19.- Reconociendo que: 6 G 7t 7t 7'7 trazamos la linea cosecante en la C.T., observándose que ésta tiene valores comprendidos entre los siguientes límites: Elevando al cuadrado: Por condición: csc 6 = Elevando a la -1: Multiplicando por 3: Sumando -1: 20.- Para determinar el intervalo de valores del arco x, que verifica la relación dada en la condición, nuestra estrategia consistirá en graficar en la C.T. el resultado del análisis que hagamos de la condición establecida. Veamos: -1 <2-J3 tanx< 1 => -3 <-J3 tanx<-1 3> J3 tanx>l De la C.T. se observa que los valores de x, en el intervalo de 0 a n, que verifican esta relación están comprendidos en el siguiente intervalo : L6'3J 21.- La sec x no toma valores comprendidos en ( -1; 1), por lo tanto para que no se verifique la igualdad se debe cumplir que: 2m + l -1 < secx <1 => - 1 < —-— <1 => - 3 <2tn + 1 < 3 => - 4 <2m < 2 => - 2 < tn < 1 .-. m e ( -2; 1) R7 350 Problemas de Trigonometría y cómo resolverlos ^íracso ^pBDITOBBB X G
22 .- Si 6 g IIC, se verifica que los valores de la secante en dicho cuadrante están compren- didos en el siguiente intervalo: 23 .- Tal como se vienen resolviendo los casos anteriores, construiremos la expresión de W a partir del intervalo de valores para el que está definido el arco x. Veamos: Por condición del problema: — < x < 2n 2x 4ti 3 < 3 Graficando este arco en la C.T., vemos que la secante tiene valores comprendidos entre los siguientes límites: -2>-2 => -1 > -3 2x sec— 2x sec— 3 <2 2x 3 >-3 W G <-3 ; -1> 1 < 3 24 .- Este modelo de ejercicio se resolverá aplicando la misma estrategia que empleamos para los problemas 14 y 17. Esto lo iniciamos analizando la extensión del arco 6 y desde este intervalo reconocer el intervalo de valores del seno, lo cual nos permitirá llegar a la expresión dada. Veamos: Como 6 g IIC 0 < sen 6 < 1 ...(*) Transformamos la expresión dada efectuado la división indicada: sen© + 2 1 ----Z—7 = 1 +--------ñ-T = esc <1>... (1) sen6+l sen6+l Circunferencia Trigonométrica
Ahora en (*): 1 < sen 6 + 1 < 2 => 1_ 2 <-----~ < 1 sen© +1 3 2 <1+ senB+1 <2 Debido a la relación (1): 3 — < csc <J> < 2 n => 4 < csc <¡) < 4 1 1 2 / Q CSC <¡> G ( 4 Z 4 25 .- Sabemos que csc 6 no toma valores en el intervalo ( -1; 1). Luego, para determinar los valores de x, que no verifican la relación dada para la cosecante, nuestra estrategia consistirá en evaluar a dicha R.T. en el intervalo que no la define. Veamos: - 1 < csc 6 < 1 => -1 < 2*+3 < 1 restamos a cada miembro 5 -5/2 ¿ 1 1 2 < 2x + 3 <- 2 5 < 2x + 3 < 1 < 2x + 3 < 5 -1 < x < 1 x G ( -1; 1> 26 .- En este caso, lo conveniente es determinar el intervalo en el que se encuentra defini- do W, de este modo sus valores extremos se constituyen en los valores mínimos y máxi- mos que puede tomar. Nuestro procedimiento se iniciará expresando las relaciones auxiliares del seno verso y coseno verso en términos del seno y coseno, luego evaluamos los valores extremos en base a los que toman éstos en la recta numérica. Veamos: W = 5(1 - cos a) - 4(1 - sen 0) =» Wmáx=l-5(-l) + 4(l)=10 =* Wmáx + WmIn=10 + (-8) W = 1 - 5 cos a+ 4 sen 0 a WmIn=l-5(l) + 4(-l) = -8 WmáA + VVmfn = 2 27 .- Empezaremos nuestra solución analizando el radicando de la Ira condición, sus conclusiones serán utilizadas en el análisis de las s'guientes condiciones. Veamos: i) - cov y > 0 => cov y < 0 => 1 - sen y < 0 Esta relación solo se verifica para: sen y = 1 ii) Como: 7 < y < 9 5n y- 2 > sen y > 1 rt 5n 9rt 2' ~2 ' 2"" iii) De las conclusiones anteriores, se desprende que: exsec x = 0 => sec x -1 = 0 sec x = 1 x = 0; 2n; 4rt;... Problemas de Trigonometría y cómo resolverlos RACSO PlDITOBIS
Dado que: 5 < x < 7 x = 2n Finalmente podemos concluir que: x + 2y = 2rt + 2 5rt 2 x + 2y = 7ft ÁREAS EN EL CÍRCULO TRIGONOMÉTRICO 28.- Nuestra estrategia consistirá en seccionar la región dada en dos triángulos y deter- minar las áreas de éstos determinando las longitudes de sus bases y alturas correspon- dientes. De la C.T. reconocemos que: PH = sen a a PR = | eos a | = - eos a (ag IIC,cosa<0) A'H = AO-HO= 1- PR => A'H = 1 + eos a (HO=PR) Los triángulos son: APA'Q y APBQ. La base y altura del primero son PQ y A'H; para el segundo son PQ y PR respectivamente. Luego las áreas son: PQ.AH PQ.PR $ = $APA'Q + $APBQ => S = 2 + 2 (2sena)(l+cosa) (2sena)(-cosa) =* S= — + — => S = sen a + sen a eos a - sen a eos a S - sen a 29.- En base al procedimiento seguido en el qercicio anterior, podemos calcular el área de la región triangular indicada reconociendo que su base y altura coinciden con las líneas del seno y coseno respectivamente. Luego: S |sen6||cos6| 2 Como: 6 G IIC sen 6 > 0 eos 6 < 0 S (-senB.cos8 i 2 / *) Aunque la expresión obtemd para el área presenta un signo menos (-) adelante, esto no debe significar que ésta sea negativa. El signo del área es positivo y esto se confirma porque en la expresión dada, el seno es positivo en el 2do. cuadrante mientras que el coseno es negativo en el mismo. Circunferencia Trigonométrica
30.- Este ejercicio es una variante del anterior, por tal motivo determinamos la base del triángulo y su correspondiente altura. De la figura: Base: 12sen 01 a Altura: | cos 01 Luego la altura estará dada por: |2sen0|.|cos0| 2 Como: 0 e IIC => sen 0 > 0 a cos 0 < 0 S = -(sen 0 . cos 6)u2 31.- Reconocemos que los triángulos tienen de base común a | cos 61. Asimismo pode- mos reconocer que existen dos triángulos rectángulos congruentes cuyas alturas miden sen 6. Luego de la figura se puede establecer que: S = Sj + S2 c |cos6| |cos6|.|sen6| Si = -y- .1 a S2 =---------------- Como: 6 G IIC => sen 6 > 0 a cos 6 < 0 , „ (-cos6)+(-cos6)sen6 Luego: S= --------------------- -cos 6 -i > S = —~+ 5611 ®) = 2 cos® + sen®) 32.- La estrategia consiste en seccionar el gráfico original, reconociéndose que las partes son triángulos que tienen por base al radio de la C.T., es decir, de medida: 1. Luego se establece que: ^3*RACSO UíDITClBI R7 354 Problemas de Trigonometría y cómo resolverlos
33.- De la figura: | cosB | (21 serte |) 2 Como: 6 G IVC => sen 6 < 0 a eos 6 > 0 Luego: S = (- sen 6 . eos 6) S = sen 6 eos 0 34.- De la figura tenemos: a) Como: 6 g IIIC, sen 6 < 0 => PT = | sen 61 = - sen 6 ; OR = - csc 6 b) Como: 6 G IIIC, eos 6 < 0 a tan 6 > 0 => PU = | eos 61 = - eos 6 ; SO = - sec 6 También: AQ = tan 6 ; QV = 1 Circunferencia Trigonométrica
36.- A partir de la figura reconocemos que: AT = tan 6 sena 37.- De la figura tenemos: AR = tan a = - ° cosa Además: PU = |cos a| = - cos a (ae IIIC =>cosa <0) También: PH = UO = | sen a | = - sen a (ae IIIC => sen a < 0) y En los triángulos rectángulos BOT y los trián- TO PU gulos rectángulos BUP: tan 6 = TO = BO.PU BU l(-cosa) 1 + (-sena) $arto ~ TO.AR 2 1 -cosa sena 2 (1-sena) cosa c _ 1 sena ^arto - 2 ’ sena-1 R 38.- Trazando la línea cosecante y coseno, se pue- den identificar la base y la altura del la región triangular sombreada cuya área estará dada por: (1+|csc6|)|cos6| 2 Donde: | cos 61 > 0 g_ (1-csc6)2.(-cos6) 2 R7 cot6-cos6 2 1 S = 2 (cot G " c°s6) -J&RACSO idito i 356 Problemas de Trigonometría y cómo resolverlos
39.- De la figura tenemos: a) OR = | esc a | = - esc a (ae IIIC esc a < 0) b) BP = cot a (ae IIIC Luego: (OR)(BP) •’apor _ 2 (-csca)(cota) 2 1 1 ' 2 ' sena ' cosa sena 1 2 S = ”2 eos a. esc a 40.- Hallaremos el área de la región identificando las coordenadas de sus vértices y aplicando el or- denamiento propuesto en el resumen, de este modo: S = 0 cos 6 1 ’2 0 -1 -sen0 1 2 -1 3 =>S= ^cosO- 1 ”4' = [jcose+^-^sene+cose] 1 n 1 4 sen6+4 N = 3/4; M = 1/4 M (0;i) 2 (cos 0; sen 0) B M + N + T = 3/4 Q(cos6;-sen6) 41.- Podemos reconocer las coordenadas de los vértices: P (cos a ; sen a), Q (cos a; - sen a), T (sen a; - cos a), U (sen a; cos a) y S (esc a; 0). Luego el área de la región triangular QSU R7 Circunferencia Trigonométrica 357
2 2 eos a-senacsca + sen a-csca cosa -csca.cosa 2 1 'cosa 2 sena i 1 > S - — cot a S = - 2 cota MISCELÁNEA 42.- Apoyados en los gráficos de la C.T. se puede establecer que: I. eos 25° > sen 25° => 1. F n. csc 40° > sec 40° => II. V ni. cot 27° > tan 27° => III. F 44.- En base al gráfico elaborado, podemos establecer que I. De la figura: | eos 21 < | eos 31 => F n. De la figura: | eos 290° | < | sen 290° | Pero: 290° G IVC eos 290° > 0 sen 290° < 0 R7 358 Problemas de Trigonometría y cómo resolverlos j^RACSO WBDITO1BI
cos 290° < - sen 290° => cos 290° + sen 290 < 0 => II F 111. De la figura: | sen 11 > | sen 31 > III F 46.- Para resolver este ejercicio graficaremos las líneas del coseno y la tangente de los arcos co- rrespondientes y según la condición del proble- ma. De esta figura tenemos: I. AP = tan x2 AQ = tan Xj tan x2 > tan xx .... FALSO II. TV = cos Xj a UW = cos x2 VM = | cos xt | a WN = | cos x21 WN = | cos x21 .-. | cos x21 > | cos Xj | .... FALSO IB. BS = cot Xj a BR = cot x2 .’. cot Xj > cot x2 .... FALSO Circunferencia Trigonométrica
VJ.- De la figura: a = cos 0 d = esc 0 b = sen 0 e = tan 0 c = sec 0 f= cot 0 Luego, sustituimos en la expresión dada: ,, cos0.sec0+sen0.csc0+l M ----------------------- tan0.cot0 M = 3 48.- Debemos notar que: «-0» es ángulo agudo. 49 .- Podemos reconocer que existe una congruencia entre dos triángulos: L PHB ~ LQOB: |cosct| l+|sena| =* 1/2 - r- => 2cosa = 1- sena Transformando, se tiene: => 2 = seca - tana seca - tana = 2 Problemas de Trigonometría y cómo resolverlos RACSO P >DITOKB«
50 .- De la figura por semejanza de triángulos rectángulos: Como: d _ |sen6| 1 |cos0|+l 6e IIC sen 0 > 0 a eos 0 < 0 , _ sen6 d - 1-cosfi 51 .- Luego de trazar las líneas del seno y coseno, hacemos: PM = x. Al disponer de las líneas se logra visualizar un triángulo rectángulo isósce- les. Es así que se puede establecer la siguiente relación: x-Ji + | sen 01 =1+|eos 01 Como: 0g IIC sen 0 > 0 a eos 0 < 0 x ^2 + sen 0=1- eos 0 72PM = 1 - sen 0 - eos 6 52 .- La identificación de las coordenadas se hará en base al gráfico, en el que se puede reconocer que los pares ordenados verifican una disposición simétrica respecto de los ejes cartesianos. Veamos: De la figura, las coordenadas de M son: (- sen 0; - eos 6) Circunferencia Trigonométrica
DEMOSTRACIONES 01.- Trabajemos con el primer miembro, para lo cual debemos reconocer que: 2 2 2 eos x = 1 - sen x => cos x = (1 + sen x)(l- sen x) (1 + senx)+ (l + senx)(l-senx) X13=-serrfJ[(l + l-senx)] => 3-----------—1------—------------ = --------------ZZZv7----- = 2 - sen x (1 + senx) (ljtserrr)T 02.- Sea: N = (sen x - cos x + l)(sen x + cos x - 1) => N = [sen x -(eos x - 1)] [sen x + (cos x -1)] 2 2 2 2 Por diferencia de cuadrados: N = sen x - (cos x - 1) - (1 - cos x) - (1 - cos x) => N = (1 - cos x)(l + cos x) - (1 - cos x)2 => N = (1 - cos x)[l + cos x - (1 - cos x)] Factorizando: N = (1- cos x)(2 cos x)= (1- cos x). 2cos x Luego reemplazando en el primer miembro de la expresión: (l^>eeSxJ-2 cos x ----.——------- = 2 cos x 2 03.- Sea: M = (1 - sen x + cos x) Desarrollando el trinomio al cuadrado: M = 1 + sen x + cos x - 2sen x + 2 cosx - 2 sen x . cos x Agrupando convenientemente: M = 2(1 - sen x) + 2 cos x(l - sen x) Volviendo a factorizar: M = 2(1 - sen x)(l + cos x) ___cot2x 04.- Partiendo del primer miembro: V = cot4x . csc2x - cot2x . csc2x + esc2 x -1 Factorizando: V = cot4x. csc2x - cot2x(csc2x - 1) => V = cot4x . csc2x - cot4x cot2x Finalmente factorizando, tendremos: V = cot4x(csc2x - 1) = cot6x RACSO WlEITOIEl 362 Problemas de Trigonometría y cómo resolverlos
2 2 05.- Recordemos que: sen x = 1 - cos x = (1 + cos x)(l - cos x) Luego, efectuando en el primer miembro como sigue, tendremos: senx 1 + cosx 1 + cosx ’ 1-cosx seríx(l-cosx) sen x Simplificando, obtenemos: 1-cosx s e nx O6.-Nuestra estrategia consistirá en transformar el primer miembro haciendo las si- guientes operaciones: 2„ KT . 2 2 KT . 2 sen X 2 N = tan x - sen x => N = tan x-------=— .eos x eos x 2 2 2 2 => N - tan x (1 - cos x) => N= tan x . sen x . 2 2.2 2 tan x - sen x = tan x. sen x 07.- Se sabe que: sen4x + cos4x = 1 - 2sen2xcos2x Elevando al cuadrado a ambos miembros de la igualdad tendremos: A *7 *7 9 *7 (sen x + cos x) = (1 - 2sen xcos x) sen8x + 2sen4xcos4x + cos8x =1- 4sen2xcos2x + 4sen4x cos4x g g 2 2 4 4 sen x+cos x = l-4sen x.cos x+2sen xcos x 08.- Efectuando la suma indicada en el primer miembro de la igualdad tendremos 1 —senn +1 + senn (1 + serw)(l - serw) 2 2 - 1 - sen a - 2 tan a 2 9 9 E =-------1 - sen a - 2 tan o 1—sen a T7 "1 2 q - 2 E = ----— - 1 - sen a - 2 tan a COS2fl 9 9 9 E = 2sec a - 1 - sen a - 2 tan a Y recordando que: sec2fl - tan2¿? = 1 => E — 2(sec2fl - tan2fl) - 1 - sen2fl 2 2 => E = 1 - sen a /. E = cos a Identidades Trigonométricas
09.- Efectuando la adición indicada en el primer miembro de la igualdad tendremos 2 2 (1-senx) + (l+senx) (1 + senx)(l - senx) 2 2 2 2 Aplicando las identidades algebraicas: (c + h) + (a-b) = 2(a + b ) (a + b)(a-b) = az-b2 2(1+sen2*) J 1-sen x 1 + sen x , 2 1-sen x , , 2 1 + 1—eos x 2 eos X ,2 2 — eos2 x 2 eos x 1 2 Recordando que: -----2~ = sec x eos x S = 2(2. .sec x - 1) S = 4 sec2x - 2 S = 2 = 2 10.- Efectuando las operaciones indicadas del primer miembro de la igualdad tenemos: 2 2 2 2 2 2 S = cot x + cot x . eos x + tan x + tan x . sen x Utilizando las identidades pitagóricas y las de división, se obtiene: „ 2 , eos2* 2 2 sen2* 2 S = csc x -1+-----cos *+sec *-l + -----5— . sen x Efectuando y agrupando: S = Sec A + esc A - z. -t- -o- + 2— sen x cos x sen x eos x 4 4 2 „ cos x sen x 2 2 S = sec x . csc x - 2 + 6 . 6_, cos x+sen x 2 2 sen x.cos x 6 6 2 2 Recordando que: cos x + sen x = 1 - 3sen *.cos x 2 2 1-3sen2*.cos2* => S = sec x . csc x - 2 + ------5---5--- sen x.cos x Efectuando la división indicada: 2 2 2 2 2 2 S = sec x . csc x - 2 + sec x . csc x - 3 S = 2 sec x . csc x - 5 SIMPLIFICACIONES 11.- Recordando que sec x . csc x = tan x + cot x 21364 Problemas de Trigonometría y cómo resolverlos Mracso
Reemplazamos en "M": M= ' tanx + cot x-tan x)2 /tanx + cotx-cotx i cscx / \ secx Reduciendo la expresión : cotx cscx tanx secx cosx senx Expresando a términos de sen x y cos x: Simplificando la expresión : ££nx 1 COSX 1 cosx M = coszx + sen2x L senx ,2. M = M = M = 1 12.- Expresando en términos de sen x y cos x, se obtiene: senx M = M = senx senx cosx + 1 cosx 1 cosx cosx 1 + senx Simplificando: Aplicando la definición de exponentes negativos para expresiones reales, tendremos: 2 2 M = cos x + sen x (Identidad) .-. M = 1 2 2 13.- Recordemos que: sec x = 1 + tan x Sumando 2tan x a ambos miembros se obtiene: 2 2 sec x + 2tan x = 1 + tan x + 2 tan x En donde el 2do. miembro concuerda con el desarrollo de un binomio al cuadrado: n 7 => sec x + 2 tan x = (tan x + 1) ... (1) 2 2 También recordemos: csc x = 1 + cot x Identidades Trigonométricas
Restando a ambos membros 2cot x, se tiene: 2 2 2 esc x - 2 cot x = 1+ cot x - 2 cot x Cuyo 2do. miembro es también el desarrollo de un binomio al cuadrado: 7 7 esc x - 2 cot x = (cot x - 1) ... (2) Reemplazando (1) a (2) en la expresión original de M, obtenemos: (tanx+1)2 (cotx-1)2 M =---------+ -------- tanx+1 cotx-1 Simplificando: M = tan x + 1 cot x - 1 M = tan x + cot x = sec x.csc x 14 .- Transformamos la expresión original escribiendo en términos de seno y coseno, obteniéndose: M = senx cosx + senx.---- _____________cosx 1 senx.---- cosx Efectuando las operaciones indicadas y simplificando: M = Sustituyendo el numerador por la identidad pitagórica: M = 2 2 sen x + cos x. senx 1 senx M = esc x 15 .- Transformamos el numerador en términos de seno y coseno y, el denominador lo transformamos adecuadamente para factorizar la tangente. Veamos: (1-cosx) (l-cosx),coszx .. senx.CQS*_____senx,cosx__ M = senx tanx-^L—.cosx cosx Factorizando en el numerador y denominador: (1-COSX) senx. cosx [(l-coszx)] tanx[l-cosx] M = R8 366 Problemas de Trigonometría y cómo resolverlos RACSO IDITOkll
sen2x Simplificando y transformando a seno y coseno: M = cosx M = 1 16 .- Efectuamos las operaciones indicadas y obtenemos: 9 M = sec x.csc x - sec x + cos x.sec x - cos x.csc x tanx+cotx secx cotx Haciendo las sustituciones indicadas, la expresión queda así: M = tan x + cot x - sec x + sec x - cot x Reduciendo términos nos queda: M = tan x 17 .- Sabiendo que: 2 sec x . cos x = 2 , nuestro siguiente paso consistirá en completar cuadrados en los numeradores de la expresión dada: w_ (secx + cosx)2-4 (secx + cosx)2-1 secx + cosx-2 secx + cosx+1 2 2 Aplicando la identidad algebraica: a - b =(a- b)(a + b), tendremos: W _ (secxjhcesxi::r2)(secx + cosx + 2) _(seGX-f-eosrmj(secx + cosx -1) Luego de efectuar las simplificaciones indicadas, nos queda: W = sec x + cos x + 2 - sec x - cos x + 1 Finalmente, reducimos términos y obtenemos: W = 3 18.- Nuestra estrategia consistirá en sustituir sec x y csc x para poder obtener una ex- presión en términos de tangente y cotangente. Para ello es necesario recordar que: 2 2 2 2 1 + tan x = sec x a 1 + cot x = csc x cot6x + 3(l + cot2x).cot2x + l cot6 x + 3cot2 x + 3cot4 x +1 tan6x + 3(l + tan2x).tan2x+l tan x + 3tan x + 3tan x+1 Logramos reconocer que los términos de la fracción obtenida corresponden a los de la identidad algebraica del binomio al cubo: (o + b)3 = a3 + 3a2b + 3ab2 + b3 Identidades Trigonométricas
w = (cot2x + l)3 (tan2x + l)3 (secx) 6V W= sec x Transformando esta expresión a seno y coseno, obtenemos: 6V COS X sen x W = W = cot6» 19 .- Nuestra estrategia de resolución consistirá en investigar toda la expresión como si estuviese constituida por dos términos, los cuales serán trabajados de un modo inde- pendiente. Veamos: M = sec x - tan x - 3 sec x tan x + 2tan x.(sec x - tan x).(csc x + 1) (A) (B) a) Determinemos a qué se reduce el término (A), para lo cual vale recordar que: 2 2 sec x - tan x = 1 . . (*) Luego de elevar al cuadrado, se obtiene: sec4x + tan4x = 1+2 sec2x tan2x Y elevando (*) al cubo se obtiene: sec x - tan x = 1 + 3 sec x tan x Esto nos permite reconocer que la expresión (A), se reduce a: A = 1 + 3 sec2x tan2x - 3 sec^x tan2x => A = 1 ,. (1) b) Trabajamos ahora con la expresión (B), en donde todo lo transformamos a seno y coseno: 2serr? cosx 2cos2x eos2 x B= 2 ... (2). B = Finalmente de (1) y (2), obtenemos: W = 1 + 2 .-. VV = 3 20 .- Efectuamos la multipl cación indicada en el denominador de la expresión dada: M _ (tan4 x + sen4 x - tan4 x.sen4 x) (tan2 x-sen2 x) Ahora transformamos el denominador para poder factorizar tan2x: tan 4 x + sen4x - tan 4 x.sen 4x => M =-----------------5----------- 2 sen x 2 tan x------.eos x eos x *, tan4 x + sen4 x — tan4 x.sen4 x M = ---------5-------5------- tan x.Q-cos x) sen x Problemas de Trigonometría y cómo resolverlos faüACSO Pkditokem
Después de efectuar la sustitución indicada, efectuamos la división y obtenemos: 2 2 2 2 2 2 M = tan x . esc x + cot x . sen x - tan x . sen x Transformando convenientemente a seno y coseno: 2 cos X 1 ^22 x - tan x . sen x seczx-l M = — cos x 2 2 2 2 2 2 2 2 => M = sec x + cos x - (sec x - l)sen x => M = sec x + cos x - tan x + sen x 2 2 2 2 Y reconociendo que: M = eos x + sen x + gec x-tan x M = 2 i 1 21 .-Nuestra estrategia consistirá en sustituir las relaciones auxiliares según sus respec- ta as definiciones. Veamos: (1 -cosx)[3-(secx-l)] + 2 + (secx-l) 3-2cosx n . .. ... (l-cosx)(4-secx)+l+secx Ki unciendo términos obtenemos: W =---------5—-------------- 3-2cosx =1 Ete< triando las operaciones indicadas: W = 4-4cosr- s&e< + >e<^x ..cogx +1 + séc< 3-2cosx Reduciendo términos: W = 2 22 .- Efectuando la potencia indicada, se obtiene: 1 2 2 1 4 4 22 M = — (1 - 3sen x . cos x) - — (sen x + cos x - 2sen x. cos x) Efectuamos la multiplicación en el primer término, mientras que en el segundo aplica- mos la identidad auxiliar para la suma de las cuartas potencias del seno y coseno: M = — - sen2x . cos2x - — (1- 4sen2x. cos2x) Efectuando la multiplicación indicada, se obtiene: 1 M=- _________r" sen—rTcos x M=i Identidades Trigonométricas
23 .- Trabajando en términos de senos y cosenos obtenemos: cosx senx 1 _ cosx cosx cosx (1 + senx)(l-senx) senx cosx + 1 eos x(l + senx) senx senx senx Simplificando en ambos sumandos, nos queda: senx 1 —senx 1 M =--------+-------- => M —------------- .-. M = sec x cosx cosx cosx 24 .- Nuestra estrategia consistirá en sustituir las relaciones auxiliares por sus corres- pondientes definiciones, para lo cual recordamos que: vers x = 1- cos x a cov x = 1 - sen x M, cosx M=-------- 1 +senx 1 — cosx senx-cosx 2 3 COS X COS X Transformamos el término indicado, multiplicándolo y dividiéndolo por: 1 - sen x 1—senx 1—cosx senx —cosx M=--------+---- COSX COS X cos3x Efectuando las divisiones indicadas: ,, 1 senx . 1 cosx . senx cosx ==> Ni — - + 7 - n + o - o COSX COSX COS x COS X COS x COS X ✓ \ 2 2 2 => M = serTx - tan x + sec^x -séc x + tan x. sec x - sec^x Simplificando y ordenando nos queda: 2 3 M = tan x (sec x - 1) M = tan x tan2x 25.- Sustituyendo cada relación auxiliar por sus correspondientes definiciones, nos da: 2 2 M = (1 - cos x) + (1 - sen x) + 2(sen x + cos x) 2 2 => M = 1 + cos x - 2cos x + 1 + sen x - 2sen x + 2sen x + 2 cos x => M = 2 + (sen x + cos x) M = 3 i Problemas de Trigonometría y cómo resolverlos ¿&RACSO
PROBLEMIZACIÓN CONDICIONAL 26.- Nuestra estrategia consistirá en aprovechar la condición dada y a partir de ella obtener una relación que nos permita reducir o valuar la expresión dada. Veamos: De la condición: sen20 + sen 0 = 1 => sen 0 = 1- sen2© > sen 0 = eos2© > sen2© = eos4© Luego, sustituimos (*) en la expresión dada: M = sen2© + eos2© .'. M = 1 27.- Tal como procedimos en el problema anterior, empezaremos nuestra resolución por las condiciones dadas. Veamos: 3 De la condición: sen x + sen x = m Dividiendo por sen x: sen x + 1 = m csc x . . (1) 3 De la condición: cos x + cos x = n Dividiendo por cos x: cos x + 1 = n sec x ... (2) 2 2 Ahora conviene efectuar (1) + (2): sen x + cos x + 2 = ni csc x + n sec x 1 M .-. M = 3 28.- Efectuando las operaciones indicadas en la condición, tendremos: (1 + senx)(l - senx) + (1 + cosx)(l - cosx) _ 1 — Ir (1 - cosx)(l - senx) 2 2 1—sen x + 1—cos x 9 1 => ~ ~ = k => (1 - cos x)(l - sen x) = —y . . (1) (l-cosx)(l-senx) ' ' Jt2 Para poder aprovechar esta nueva relación, será conveniente elevar al cuadrado a la expresión a calcular, esto es: M2 = (1 - sen x - cos x)2 => M2 = 2(1 - sen x). (1 - cos x). . . (2) Reemplazando (1) en (2), obtenemos: K,2 2 ™ M = —y .. M =~¡~ k2 * 29.- Desarrollando las potencias indicadas, obtenemos: 3 6 2 2 2 3 6 W = sen x+ 2sen x.cos x+ cos x + sen x -2sen x.cos x + cos x + cos x +2 cos x sen x + sen x Identidades Trigonométricas
Agrupamos convenientemente y factorizamos: 2 2 2 2 W = 2(sen x+cos x)+2sen x. cos x(sen x + cos x) + sen x + cos x - 2 sen x cos x Sustituimos las expresiones entre paréntesis por 1, y nos queda: VV = 2 + 2_sen-x-cósx + sen6x + cos6x- 2_sen^rcos3r Reemplazamos la expresión indicada por la identidad auxiliar correspondiente: W = 2 + 1 - 3 sen1 2 *x.cos2x => W = 3 - 3 sen2 x cos2x ... (1) 2 2 De la condición: 1-2 sen x cos x = k 2 2 1 sen x cos x =---- 2 - - (2) Reemplazando (2) en (1): ’l-Jt* W = 3-3 W= J(l + fc) 30.- Nuestra estrategia consistirá en transformar, de un modo controlado, la expresión dada para «M» tal que sea posible identificar en ella un término que pueda ser obtenido de la condición planteada. Veamos: M = sen4x + cos4x => M = 1 - 2 sen2x cos2x M = 1 - 2 sen2x. cos2x ... (1) De la condición dada trataremos de obtener el equivalente de la expresión indicada: sen x + cos x = -J2 => (sen x + cos x)2 = (-Jl )2 => 1+2 sen x. cos x = 2 1 Finalmente despejamos y obtenemos: sen x. cos x = — - - (2) Luego reemplazamos (2) en (1): M = 1 - ± M=| 31.- Nuestro procedimiento consistirá en transformar la expresión dada en términos de sec y y esc x. Para ello es necesario recordar que: 2 2 2 2 1 + tan a = sec a a 1 + cot a = esc a Aplicando estas identidades en la condición dada: 2(1 + tan2x) - (1 + cot2y) = 1 => 2 + 2tan2x - 1 - cot2y = 1 R8 372 Problemas de Trigonometría y cómo resolverlos RACSO P tDITOKBN
2 2 2 2 => 2 tan x = cot y => 2 tan y = cotx Aplicando por 2da vez las mismas identidades en cada miembro, tendremos: 2(sec2y - 1) = csc2x -1 => 2sec2y - csc2x = 1 .-. W = 1 32 .- Procediendo como en la resolución del problema 30, transformamos de manera controlada a la condición dada. Veamos: _ . eos4 x-sen4* Se tiene que: ----8------= m cos *—sen x Transformamos el denominador: --. ——i-------— = m -feos^^sénx)(cos x + sen x) Despejando lo que queda: — = cos4x + sen4x Aplicando la identidad auxiliar correspondiente: — = 1 - 2 sen2x . cos2x 2 2 De lo cual podemos deducir que: sen x . cos x = . (1) Ahora trabajamos con la expresión «M», en donde aplicamos la identidad auxiliar: 2 2 2 2 M = 1 + 1 - 3 sen x . eos x => M = 2 - 3 sen x. eos x . . (2) (m-1) »m+3 Reemplazando (1) en (2), tendremos: M = 2 - 3—M - - 33 .- En este caso resulta conveniente sustituir la condición dada en la expresión solici- tada, obteniéndose: ... sen x—vers x senx + versx W =------------ ----------- sen x+ vers x senx- versx y J r [senx-vers x] -[senx+uersxj W = -----------2------2--------- sen x-vers x 2 2 Apliquemos, en el numerador, la identidad algebraica: (a - b) - (a + b) = - 4ab y En el denominador sustituiremos el sen x por su identidad pitagórica asi como reem- plazaremos la relación auxiliar versx por su correspondiente definición. Veamos: W______-4.sen x.vers x 1 —cos2x—[1 —cosx]2 _______—4-senx.uers x______ W- i-COs2x-[1-2cosx+cos2x] Eliminado los corchetes y reduciendo: -4.senxx%rs~x~~ W = 2 cos x.n^cosxl W = - 2 tan x Identidades Trigonométricas
34 .- Nuestra estrategia consistirá en transformar la condición dada de modo que de ella' se obtenga la expresión «M». Veamos: 2 2 sec x - csc x = a => (sec x - csc x) = a 2 2 2 2 2 2 sec x + csc x - 2sec x . csc x = a => sec x. csc x - 2 sec x . csc x + 1 = a +1 => (sec x. csc x - l)2 = o2 + 1 Aplicando la identidad auxiliar en el primer miembro: (tan x + cot x - 1) = a +1 ---(*) Identificamos en el 1er miembro a la expresión solicitada pero afectada del exponente 2. Lo pertinente será extraer raíz cuadrada a ambos miembros, lo cual requiere analizar previamente el signo de la expresión buscada. Veamos: Como:xg IIC => tanx + cotx<-2 => tanx + cotx-1 <-3 Extrayendo raíz cuadrada a (*): | tan r + cot x - 11 = Ja2 +1 (-) Aplicando la definición de valor absoluto: tan x + cot x - 1 = - Vfl2 + 1 M = - 7fl2+l 35.- Expresemos W en términos de senos y cosenos, y tendremos: ... í 1 cosa') W =--------cosa 1+-------- ^sena sena J (1 — senacosa)(sena + cosa) vv =------------------------- 2 sen a A continuación transformamos la condición dada en términos de senos y cosenos con el propósito de obtener expresiones equivalentes los términos que forman a «W»: sena 1 2 -.2 ------------=1 => sen a - cos a = sen a cos a => 1 - cos a - cos a = sen a cos a cosa sena 2 2 => 1 - sen a cos a = cos a + cos a... (1) => 1 - cos a = sen a cos a + cos a... (2) Reemplazando (1) en (*) tendremos: 2 (eos a+cosa)(sena+eos a) W =-------------ñ--------- 1—cos a cosa(cosa + l)(sena + cosa) (l + cosa)(l - cosa) Reemplazando (2) en (*), tendremos 2 senacosa + cos a 1-cosa 1-cosa W = ------ 1-cosa W = 1 •iRACSO VlDlTOtll 3741 Problemas de Trigonometría y cómo resolverlos
36.- Lo que haremos es transformar la expresión dada para «W», en términos de tan x y cot x. Para ello aplicaremos las identidades correspondientes: 7 7 W = 1 + tanx + 1 + cotx + 2(tan x + cot x) Aqui es conveniente que la expresión quede solo en términos de la tan x, veamos: 2 i i i W = 2 + tan x + -----ñ- + 2 tanx +-------- tan x tanx (*) (**) Debemos señalar aquí que la condición dada es en realidad una propiedad general de los números reales y positivos. De este modo queda claro que: 2 i í 11 tan x + --j— >2 a tanx+------------- >2 tan x tanx) Operando estas desigualdades podemos establecer que: W > 2 + 2 + 2(2) => W > 8 VV - = 8 ' / BuAlfllO 37.- Nuestra estrategia consistirá en aplicar la condición en la expresión dada. Veamos: De la condición despejamos: tan a = 1 + sen a ___(1) Identificamos en W la expresión (*): W = sec a .esc a - sen a + cos a O Sustituimos (*) por la identidad : W = tan a + cot a - sen a + cos a. (2) Sustituimos (1) en (2), tenemos: W = 1 + sen a + cot a - sen a + cos a cos ex Expresando en términos de senos v cosenos: W = 1 +------ + cos a K J sena W = 1 + cos a (—-— +1) => W = 1 + cot a (1 + sen a)... (3) \sena / Reemplazando (1) en (3) tenemos: VV = 1 + cot a. tan a .-. VV = 2 i 38.- Transformando la expresión dada en términos del seno, tendremos: 1 3 VV =----- + sen x senx 1 + sen4x senx W = ...(1) 2 En la condición sustituimos cos x por su equivalente y tendremos: Identidades Trigonométricas
2 3 sen x - 1 - sen x + sen x = O 2 3 . sen x + sen x + sen x = 1 Multiplicando por sen x, nos queda: Despejando, tendremos que: Reemplazando (2) en (1) tendremos: Aplicando la identidad pitagórica: De la condición inicial despejamos: Reemplazando (4) en (3): W = 2 3 4 sen x + sen x + sen x = sen x sen4x = sen x - sen2x - sen3x .. (2) ,íT 1 + senx-sen2x-sen 3x W = --------------------- senx 2 3 senx + cos x-sen x W — ... seru cos2x - sen3x = sen x — (4) senx + senx ---------- W = 2 senx 39 - Completamos cuadrados en la condición dada y recordando que las razones seno y cosecante son recíprocas, tendremos: 2 2 sen x + 2sen x. csc x + csc x = 7 + 2 2 Sustituimos el 1er miembro por un cuadrado perfecto: (sen x + csc x) = 9 Extrayendo raíz cuadrada a ambos miembros: sen x + csc x = ± 3 ... (1) Expresando W en términos de senos y cosenos: cosx W = cos x. ----+ 2 sen x senx Efectuando las operaciones indicadas: 2 2 cos x + 2sen x W =------------- senx 2 2 2 cos x + sen x+sen r W =---------------- o l + sen x senx senx W = A continuación realizamos la división indicada: W = csc x + sen x ... (2) Finalmente reemplazamos (1) en (2): W = ± 3 40.- Nuestra estrategia consistirá en transformar la expresión dada e identificar en ella una relación que sea obtenible de la condición establecida. Veamos: 2 W = CSC X - 1 + CSC X => W = CSC x(csc X+l)-l ... (1) De la condición despejamos convenientemente: Problemas de Trigonometría y cómo resolverlos RACSO ID1TO1II
cos x ----+1 1=1 => csc x + 1 = sec x ^senx j => sec x - csc x = 1 ...(*) v sec x = csc x + 1 .. (”) Reemplazando (**) en (1): W - sec x. csc x - 1 (2) Nos concentraremos ahora en determinar el valor de la expresión indicada. Para ello nos apoyaremos en las relaciones obtenidas de la condición dada. Veamos: 2 2 Elevando (*) al cuadrado tenemos- sec x + csc x - 2(sec x. csc x) = 1 2 Por la identidad auxiliar nos queda: (sec x. csc x) - 2(sec x. csc x) - 1 = 0 2±J(-2)2-4(l)(-l) Resolviendo la ecuación cuadrática: sec x. csc x =---------------- 2±2-j2 => sec x. csc x = “ => sec x . csc x = 1 ± -J2 => sec x. csc x = 1- 42 v sec x . csc x = 1 + 42 Debemos recordar que: sec x. csc x < - 2 v sec x . csc x > 2 sec x. csc x = 1 + 42 Reemplazando en (2): W = (1 + 42 ) - 1 VV = 42 41.- Nuestra estrategia consistirá en despejar «a» a) Despejando "a" de la condición (1), tenemos: y «b» de las condiciones dadas, sen 4a— 2 eos4 ex a = csc4 a Aplicando la identidad recíproca para la csc x: 4 4 4 2 a = sen a. sen a - 2sen a. cos a Efectuando operaciones, nos queda: 8 4 4 a = sen a - 2 sen a . cos a • • (1) De: pejando "b" de la condición (2) tenemos: , cos6<x + 4sen2a b = 2 sec a Aplicando la identidad recíproca para la sec x: b = cos6a. cos2a + 4sen2a. cos2a Efectuando operaciones, nos queda: 8 2 2 b = cos a + 4sen a cos a - - - (2) Identidades Trigonométricas 377
8 8 4 4 2 2 Sumando (1)+ (2): a+b = sen a + cos a - 2sen a cos a + 4sen a cos a ...(*) 8 8 2 2 4 4 Del Prob. 7, recordamos que: sen a + cos a = 1- 4sen a cos a + 2sen acos a Sustituyendo en (*), obtenemos: a + b = 1 42 .- Lo que haremos es transformar el 2do. miembro de la expresión dada de modo que adquiera la forma del 1er. miembro, para luego determinar «ni» por simple compara- ción. Veamos: En el 2do. miembro aplicamos la identidad algebraica: o3 + b3 = (a + b)(«2 -ab + bz): secx.cscx+ m ^(seex^Fcscx)(sec2 x - secx.cscx + esc2 x) secx.cscx+ 2 _(seex^Fcscx)(secx + cscx)2 2 2 2 2 Aplicando la identidad auxiliar: sec x + esc x = sec x . esc x, tendremos: 2 2 secx.cscx + m (sec x.csc x-secx.cscx) secx.cscx+ 2 (secx + cscx) secx.cscx + m Factorizando, nos queda: -----------~ n secx.cscx+ 2 _see¿er5cx(secx. cscx -1) _seexrcScx(secx.cscx + 2) Simplificando se obtiene: secx.cscx + m secx.cscx+ 2 secx.cscx—1 secx.cscx + 2 Wl=-1 43.-Expresando el primer miembro de la igualdad en términos de senos y cosenos tenemos: (2-cos2a)í 1+—K— _________( cos a 2 l + 2.sena eos a 7 7 (2-cos a)(cos a + 1) 2 2 „ 1 cos a+2gen a =2+ Aplicando la identidad pitagórica en la expresión indicada, tendremos: +cos2 a) i 12:<-os2cx) =2+k Nuevamente aplicamos la identidad pitagórica: 1 + cos2a = 2 + 2 1 2 - sen a = 2 + 2 Aplicando la identidad recíproca para el sen x: 2 +------- — 2 + — -csc2q ~ * Finalmente por comparación obtenemos que: 2 k = - esc a R8 378 Problemas de Trigonometría y cómo resolverlos ¿1KACSO BeCITCIII
44 .- Nuestra estrategia consistirá en transformar la expresión original con el propósito de reducirla y poder despejar el cos x. Veamos: Aplicando la identidad por cociente para la tan x, la expresión se transforma en: (tana-tanb.cosx)2 2 . 2, -----------=---— = tan a - tan b (senx) 2 2 Teniendo en cuenta que: sen x = 1 - cos x, transponemos términos: 2 2 2 2 (tan a - tan b.cos x) = (1 - cos x)(tan a - tan b) Desarrollando las operaciones indicadas, tendremos: 2 2 --—2' 2 2 2 2 2 ___~ tarfs^- 2tan a . tan b. cos x +Jan-ircós x = tahs^ - tan b - cos x. tan a + cDS-xrtan b Transponiendo todos los términos al primer miembro, tendremos: tan b - 2tan a . tan b. cos x + cos x. tan a = 0 Reconociendo que el 1er membro es el desarrollo de un binomio al cuadrado, se tiene: (tan b - cos x . tan a) = 0 cos x = cot a. tan b 45 .- Utilizando las identidades pitagóricas en los numeradores tendremos: 2 2 sec x-1 csc x —1 +----— = 4 + sec x secx-1------------cscx-1 Aplicando la identidad algebraica de la diferencia de cuadrados, se obtiene: j(seex-='TJ'(secx + 1) -(escx^^TÚcscx +1) ----Zseex-'T'--- +--------o9rrr"T"---- = 4 + sec x Al simplificar, la expresión se convierte ere / / 1 stjex + 1 + csc x + 1 = 4 + sp<f x => csc x = 2 => sen x = ~ 5 sen x + csc x = 46 .- Procederemos tal como se hizo en el Prob. 42, es decir, transformaremos la expresión dada en el 1er. miembro de modo que adquiera la forma del 2do. miembro. Luego el valor de «m» se determinará por una simple comparación de términos. Empezaremos reali- zando la adición indicada: (secx - tanx)2 + (secx+tanx)2 2 2 sec x —tan x = tn + tnm. (cotx)1" Identidades Trigonométricas
Aplicando la identidad algebraica de Legendre en el numerador del 1er. miembro: l+tan2x 2 2 2( sec X + tan x) m , . »-m Q . 2 v m , . x-m ----------------- = m + tn . (cot x) => 2(1 + 2 tan x) = m + m . (cot x) Transformando esta última expresión en términos de cot x, nos queda: 2(1 + 2[cot-1x]2 ) = m + mm. (cot x)'m => 2 + 22.(cot x)'2= m + tnm. (cot x)’m m = 2 47 .- La estrategia será la misma que el del problema anterior. Empezaremos transfor- mando el 1er. miembro sustituyendo la sec2x por su identidad pitagórica. Veamos: l+tan2x 2 cosx(3 + tanx-2sec x) 1-tn 2 tanx+ 1 secx cosx(l + tanx-2tan2x) 1-m 2tanx+l secx Factorizando el numerador del 1er. miembro, tendremos: cosx(l- tanx)X24arrxny 1-m -(¿lanxrTJ secx cosx. (1- tanx) = 1-m secx Finalmente por la identidad recíproca: 1-tan x 1—tn secx secx m = tan x 48 .- Sustituimos en la condición dada la relación auxiliar exsec x por su correspondien- te definición. De este modo nos queda: sec x -1 + tan x + 1= a => sec x + tan x = a ... (1) De la identidad pitagórica para la tan x y sec x se puede deducir que: 2 2 1 secx - tan x = 1 => (sec x - tan x)(sec x + tan x ) = 1 => sec x - tan x =------ sec x +tanx 1 Y de (1), tendremos que: sec x - tan x = ~ ... (2) 1 n2-l Restando (1) - (2): 2tanx = fl-— => tanx=—-— cotx= , íl2-l Problemas de Trigonometría y cómo resolverlos *J* RACSO W^BDITOBSa
49.-En este caso conviene expresar todos los términos de la expresión original en base a seno y coseno, para lo cual sustituiremos las relaciones auxiliares por sus correspon- dientes definiciones. Veamos: (x + I - sen 6)2 + (I - cos 6)2 = => (x + 1 - sen 6)2 = -(I-cos6)2 cos 6 cos 6 Ahora podemos factorizar el 2do. miembro, obteniéndose: (x +1 - sen 6)2 = (1 - cos 6)2 (I-cos26) eos2 6 => (x + 1 - sen 6)2 = (1 - cos 6)2 sen2 6 .eos2 6 Extrayendo raíz cuadrada a ambos miembros, tendremos: | x + 1 - sen 61 = 11 - cos 61 . | tan 61 Siendo: x > 0 a 6 e IC => | x + 1 - sen 61 = x + 1 - sen 6 ; 11 - cos 61 = 1 - cos 6 , | tan 61 = tan 0 x + 1 - sen 6 = (1 - cos 6) sen6 cos6 => x + 1 - seríÚ = tan 6 - serrtí .-. x = tan 0-1 50.- Nuestra estrategia consistirá en transformar, de forma controlada, la expresión dada para «M», hasta que sea posible identificar entres sus términos a la condición dada. Veamos: .. senx+sen5x-sen3x-sen7x senx(l+sen4x)-sen3xfl + sen4x) M =----------é----q-----7— => M = ~ 4~ 4 r COSX + COS X-COS X-COS X COSX(1+COS x)-cos X(1 + COS x) Factorizamos en ambos términos de la fracción y obtenemos: 4 2 4 7 senx(l + sen x)(l-sen x) senx (1 + sen x) cos~x M “ cosx(l + eos4 x)(l - eos2 x) =* cosx (l + cos4x) sen2x Reemplazando la condición dada en esta última expresión, tendremos: 2 2 2 M = tan x . tan x . cot x => M = tan x . cot x M = 1 ELIMINACIÓN DE ARCOS 51.- Este tipo de ejercicios se caracterizan por que la tarea consiste en obtener tina expre- sión trigonométrica independiente de la variable angular. Esto se logra apelando a las identidades conocidas y a la transformación de las condiciones dadas.Veamos: De la condición (1): senx _ a cosx b a tan x = - ...(*) b Identidades Trigonométricas
De la condición (2) se deduce que secx csc x = 3 Por la identidad auxiliar la expresión queda así: tan x + cot x = 3 ... (**) Si ahora reemplazamos (*) en (**) obtenemos una expresión independiente de «x»: - + - -3 b + a Efectuando y transponiendo nos queda: a' + o = 3ab 52.- Se pide: F = (b - a)2 * Por ello restamos convenientemente miembro a miembro: (2) - (1) Obteniéndose: (2 - eos4 x) - (1 + sen4x) = b sen2 x. cos2x - a cos2x sen2x 1 - (sen4 x + cos4x) = (b - a) sen2x.cos2x Por identidades auxiliares: 1 - (1 - 2 sen2x.cos2x) = (b - a) sen2x . cos2x 2 = (b-a) .-. F = (2)2 = 4 53 .- En primer lugar de la condición se puede deducir que: senx cosx senx a a b cosx b a => tan x = — b Nuestro problema se puede resolver fácilmente si ahora aplicamos en la condición dada una de las propiedades de las proporciones. Veamos: 2 2 2 sen x cos x tan x Aplicando la propiedad de razones y proporciones en la primera igualdad: sen2x+cos2x tan2x «2 + b2 ~ c2 Reemplazamos (*) en (**) y además aplicamos la identidad pitagórica en el 1er. miembro: Problemas de Trigonometría y cómo resolverlos *¿ÍRACSO r DiTOMks
a2 +b I fl2 a2+b2 b2.c2 2. 2 >2. ,2 2 a (a +b ) = b c 54 .- Nuestra estrategia consistirá en transformar las condiciones dadas hasta que sea posible independizarnos de la variable angular. Veamos: Transformamos la condición (2), expresándolo en términos de la tan x: (1--—)2 = (fe -1) a-*-—V-) tanx tarTx (tanx-1K T (1 +tarTx) 2 2 Luego e simplificar nos queda la expresión: (b - 1)(1 + tan x) = (1 - tanx) ...(*) Ahora es conveniente dividir (1) (*) para liberamos de la variable angular: a = b- 1 55.- Dividiendo (2) + (1): sen X-jxtéi r cós2x.s£r.x 9 f tan x = — 9 Utilizando el siguiente triángulo, podemos escribir la condición (2) en términos de la función «tan x», así: tanx p sen2 x . cos x = r => p. 2 tan x I 1 + tan x Vi + tan2 x = r ...(«) Sustituyendo (*) en (**) y elevando al cuadrado: P2- l+-y 9 Efectuando y simplificando obtenemos: 56.- Transformaremos las condiciones dadas de modo que sea posible establecer con estas las relaciones necesarias para poder eliminar la variable angular. Veamos: a b^í =1 b - a = 1 ...(») Identidades Trigonométricas
De (1): tan x + tan x . cos x = 4p => tan x + sen x = 4p ... (3) De (2): tan x - tan x cos x = 4q => tan x - sen x = 4q .. (4) De (3) + (4) y despejando: tan x = 2(p + q) .. (5) De (3) - (4) y despejando: sen x = 2(p - q) ... (6) De (1) x (2) tenemos: tan2x .(1 - cos2x) = 16 pq tan x . sen x = 16 pq . - (7) Reemplazando (5) y (6) en (7): 4(p + q)2.4(p - q)2 = 16 pq .-. p2V=^ 57 .- Por tratarse de condiciones que tienen entre sus términos al seno y coseno lo conve- niente es elevarlas al cuadrado, para luego agrupar convenientemente e independizarnos de la variable angular aplicando las identidades conocidas. Veamos: 2 2 2 De la condición (1) : (m sen x-n cos x) = (m + 1) , => m2 sen2x + n2cos2x - 2nin sen x . cosx = m2+ 2m + 1 ... (3) 2 2 2 De la condición (2) : (n sen x + m cos x) = (n + 1) 2 2 2 2 2 => m cos x + n sen x + 2mn sen x . cos x = n + 2n+ 1 _(4) Efectuamos la adición de (3) + (4) y factorizamos convenientemente: 22 2 22 2 22 m (sen x + cos x) + n (sen x + cos x) = m + n +2m +2n +2 ' i i .-. m + n ♦ 1 = 0 2 58.- Si dividimos ambos miembros de la condición (1), por cos x, tendremos: 2 2 sen x-cos x a 2 = 2 cos x cos x 2 2 2 2 => tan x - 1 = a sec x => tan x - 1 = fl(l + tan x) 2 + 1 Despejando: tan x = 2 l-« También : cot x = 7 a+1 ... (3) Reemplazando (3) en la condición(2): a + 1 1-fl lfl+1 J Problemas de Trigonometría y cómo resolverlos ¿A RACSO V ülTOIia b
De donde al efectuar se obtiene: 8fl(«2 + 1) = Ixl- h2)2 59.- Lo conveniente es transformar las condiciones para obtener relaciones adecuadas que logren expresarse independientemente de la variable angular. Observa: De (1): tan3x (1 h f tan2x) = m8 => 3 2 8 tan x .sec x = tn - - (3) De (2): cot3x(l i- cot2x) = n8 => 3 2 8 cot x . csc x = n --(4) Multiplicando (3) . (4): 2 2 8 8 sec x. csc x = m . n --(5) 2 8 8 Por la identidad auxiliar se tendrá: (tan x + cot x) = m . n Dividiendo (3) (4) y aplicando las identidades recíprocas: tan3x.----5— _______cos x 1 1 3 ' 2 tan x sen x o m tan x = —<r tr m tan x = — n n a cot x = — .. (*) ni ' z Reemplazando (*) en (5): 2 2 5 5 tn + tt = tn .tt 60.- Transformaremos las condiciones hasta que logremos aislar a «m» y «n». Veamos: De la condición (1): (1 + senx) (I+senx) (1 - senx) ’ (1 + senx) (1 + senx) m =------5— 1 —sen x tn - í 1 +senx cosx => -Jm = sec x + tan x .. .(3) /— 2 Reemplazando (3) en la condición (2), tendremos: n = (-Jm - 1) n = -Jm -1 ’m - >¡n = 1 Identidades Trigonométricas R8 385
, f A D O 9 identidades Trigonométricas A. de Arcos Qompuestos SENO, COSENO DE ARCOS COMPUESTOS 01.- Recordemos: sen x + cos x = f— I 71 ’2 sen x +~ 4 J2 8 l 71 '2 senl x +~ sen 71 4 2 = 8 Luego: M = 16 M = 2 02.- Desarrollando los binomios y ordenando como sigue obtenemos: M = sen218° + cos212° + 2 sen 18° cos 12° + sen212° + cos218° + 2 sen 12° . cos 18° M = (sen218° +eos218°) + (sen212° +eos212°) + 2(sen 18°. cos 12° + cos 18°. sen 12°) 1 1 sen(18°+12<') Finalmente: M = 2 + 2 sen 30° = 2 + 2^|) .-. M = 3 03.- Escribiendo la expresión en forma apropiada obtenemos: —— sen50°—-.cos50° 2 2 M = —A------------A i/2j —^=sen25°—y=.cos25° I V2 V2 M 2(sen50°. cos30°—cos50°.sen30°) iÍ2 (sen25°. cos45°— cos25°.sen45°) Luego aplicando las fórmulas de los arcos compuestos: 2 sen(50°-30°) M“ V2 ’ sen(25°-45°) xa- sen 20° V2 ’ (-sen20°) Finalmente al simplificar se obtendrá: M = -j2 RACSO DITORBB R9 386 Problemas de Trigonometría y cómo resolverlos
04.- Factorizamos: sen (a + b) en los dos primeros términos: M = sen (« + b) - 2 senffl + fe), cos a . sen fe + sen fe M = sen(fl + fe).[sen(fl + fe) - 2 cos a sen fe] + sen2fe A continuación desarrollamos el seno del arco compuesto, así: M = sen(fl + fe). [sen a . cos fe + cos a . sen fe - 2 cos a . sen fe] + sen fe Luego, observamos la presencia del desarrollo de un arco compuesto. M = sen(fl+ fe), [sen a. cos fe - cos a .sen fe] + sen fe Finalmente utilizamos una propiedad especial, obteniendo: 2 2 M = sen(fl + fe).sen(fl-fe) + sen fe M = sen a sen’a - sen2b 05.- Completando cuadrados, tenemos: 2 2 2 2 2 2 W = cos (a + P) - 2 cos a cos P cos(a + P) + cos a cos P - cos a cos P + cos P Luego de agrupar convenientemente obtenemos: W = [eos (a + P) - cos a cos P]2 + cos2P(l - cos2a) W = [cos a cos P - sen a sen p - cos a cos P]2 + ces2p. sen2a 2 2 2 2 A continuación al simplificar, tendremos: W = sen a sen P + cos P- sen a Finalmente al factorizar «sen a» obtendremos: W - sen2a(sen2P + cos2p) W = sen2a v 2 2 06.- Escribiendo apropiadamente como sigue: W = sen y-sen x + 1 - sen y cos x Utilizamos una propiedad que nos permite utilizar la condición x + y = 30°, así W = sen(y +x )sen (y - x ) + 1 - sen y cos x => W = y sen (y - x) + 1 - sen y cos x Luego multiplicamos por 2 ambos miembros y desarrollamos el arco compuesto: 2W = sen y cos x - cos y sen x + 2 - 2 sen y cos x => 2W = 2 - sen y cos x - cos y sen x Luego de agrupar convenientemente y aplicar el seno de un arco compuesto con su respectivo valor numérico obtenemos: 2W = 2 - sen (y + x) => 2W = 2 - | => 2W = 2 - | .-. W = | Identidades Trigonométricas de Arcos Compuestos
07.- Escribiendo las condiciones (1) y (2) de otra manera, al utilizar las propiedades especiales de arcos compuestos, así: 7 7 2 2 sen x - sen y = a ... (1) cos x - sen y = b .. (2) Sumando las condiciones (1) y (2): sen2 x + eos2 x - 2 sen2y = a + b i 2 1-a-b Al efectuar y despejar en la igualdad anterior, obtenemos: sen y = --- . (4) En (2) reemplazando (4), tenemos: cos2x - —- j = b => cos2x = ~- (5) La condición (3) elevando al cuadrado en ambos miemb ros de la igualdad: 2 2 2 eos x . sen y = c ... (6) Reemplazando (4) y (5) en (6), tenemos: 2 2~ 7 7 7 7 7 7 Al reducir, obtenemos: (1 - a) -b = 4c (1 - fl) = b + 4c 08.- Elevar al cuadrado en ambos miembros de la igualdad la condición (1) y (2) tenemos: 2 2 2 2 2 (sen x + sen y) = m => sen x + 2 sen x sen y + sen y = m .. (4) (eos x + cos y) = n => cos x + 2 cos x. cos y + cos y = n ... (5) Sumando (4) y (5), luego agrupando convenientemente y aplicando las propiedades tendremos: 2 2 2 2 2 2 sen x + cos x + 2[cos x cos y + sen x sen y] + sen y + cos y = m + n =1 cos(x-y)=p...de(3) =1 2 + 2p = m2 + n2 2(1 + p) = wi2 + m2 TANGENTE, COTANGENTE DE ARCOS COMPUESTOS 09.- Escribiendo la expresión «M» en términos de la función tangente, obtenemos: - tanx.tany tanx + tan y tanx + tan y 1-tanx. tan y 1 tanx + tany “ tan(x + y) 1 Finalmente: M =-------------- .-. M = 1 tan ti/4 RACSO WeDlTOlKI 388 Problemas de Trigonometría y cómo resolverlos
10.- Si utilizamos: tan A + tan B = sen(A +B) cosA. cosB En nuestro problema obtendremos: W sen(2a + a) sen 3a 1 " cos2a.cosa + cos3a cos2a Agrupando como sigue: sen3a cos2a 1 1 cosa cos3a ,,, sen3a W =------ cos 2a ( cos3a+cosa cos3a.cosá A continuación, utilizando arcos compuestos desarrollamos los equivalentes de: cos 3a y cos a. cos 3a = cos(2a + a) = cos 2a cos a - sen 2a sena Luego de sumar los resultados anteriores, obtenemos: cos a = cos(2a - a) = cos 2a cosa + sen 2a sena cos 3a + cos a = 2 cos 2a cos a ... (2) Reemplazando (2) en (1) tenemos: ... sen3a 2cos2o.cosa yy — --------——— cos2a ' cos3o.cosa Al simplificar tendremos: VV = 2 tan 3a „ c . • 2tanA + 3tanB-tanC(l + tanAtanB) 11.- Factonzamos como sigue: W =-------------------------------- .. (1) tan A + 4 tan B - 2 tanC(l + tanA tan B) A continuación operamos con el dato: „ tan B-tanA „ B - A = C => tan(B - A) = tan C => -~ - tan C ' 1 + tanBtanA Deduciendo: tan B - tan A = tan C(1 + tan A tan B) (2) Reemplazando lo obtenido en (2) en (1): 2 tan A + 3 tan B—(tan B - tan A) tan A + 4 tan B - 2(tan B - tanA) Luego de efectuar y reducir obtenemos: 3tanA + 2tanB 3tanA + 2tanB ...(1) W = 1 Identidades Trigonométricas de Arcos Compuestos
12.- Escribiendo como sigue .tendremos: 3 sec x . csc x = 4[tan (x) - tan (x - y)] Desarrollando ambos miembros, el primero en términos de senos y cosenos y en el segundo la fórmula especial: 3 „ senlx-(x-y)] „ , = 4 . -1'— => 3 cos (x - y) = 4 . sen y. sen x senx.cosx----------------------------cosx.cos(x—y) 3 cos x cos y + 3 sen x sen y = 4 sen x sen y Luego de efectuar y simplificar, obtenemos: 3 cosx . cosy = sen x . seny cotx.coty=^ 13.- Escribiendo en términos de la función tangente: t t t o 7 7 2 2 tan x + 2 tan x . tan y = 1 + sec y => tan x - tan y = 2(1 - tan x . tan y) l + tan2y => (tan x + tan y)(tan x - tan y) = 2(1 - tan x . tan y)(l + tan x . tan y) A continuación ordenamos apropiadamente, logrando: (tanx + tany | [ tanx-tany | 1 - tanx. tany I I 1 + tan x. tany I Finalmente al reemplazar la segunda condición, obtenemos: 2 tan(x+y). tan(x-y) =2 .-. tan(x + y)=g 3 14 .- Efectuando los productos indicados tenemos: W = -Js - -Js tan 5°.tan 10° + tan 5o + tan 5°.tan 10° + tan 10° + tan 5°.tan 10° A continuación agrupamos y ordenamos: W = tan 10° + tan 5o + (2 - 73 ).tan 10°.tan 5o + 73 Recordando que: tan (a + P) = tan ct + tan P + tan ct. tan P . tan (a + P) En el problema tendremos: W = tan (10° + 5o) + -J3 = tan 15° + -J% Fmalmente reemplazando el valor de la tan 15°, obtenemos: W = 2 - 73 + 73 .-. W = 2 Problemas de Trigonometría y cómo resolverlos íMracso K^bditokii
15 .- Luego de ordenar y agrupar apropiadamente, tendremos: w 73(1 - cot65°) + cot80°(l - cot 65°) (1 - cot65°)( 73 + cot80°) l+cot65°-73cot800(cot65°+l) ” (1 +cot65°)(l73 cot 80°) A continuación la expresión se expresa en función de tangentes: tan45°-tan25° tan60°+tanl0° W=---------------------------------- 1 +tan 45° tan 25° 1-tan 60°. tan 10° Utilizando fórmulas de la tangente de ángulos compuestos obtenemos. W = tan(45° - 25°).tan (60° + 10°) => W = tan 20°.tan 70° = tan 20°.cot 20° W = 1 16 .- Por ángulos complementarios: cot 54° = tan 36° a cot 81° = tan 9o Al reemplazar en W tenemos: W = tan236° - tan^.tanV + tanV + 4 tan 36°.tan 9o Escribiendo apropiadamente, obtenemos: W = tan236° + 2 tan 36° tan 9o + tan29" - tan^.tanV + 2 tan 36°.tan 9o W = (tan 36° + tan 9o)2 - tan^.tanV + 2 tan 36°.tan 9o ... (1) Relacionando los ángulos 45° = 36° + 9o y recordando: tan(a + P) = tan a + tan P + tan a .tan p. tan (a + P) En el problema, tendremos: tan(36° + 9o) = tan 36° + tan 9o + tan 36°.tan 9°.tan (36° + 9o) tan 36° + tan 9o = 1 - tan 36°. tan 9o ... (2) Reemplazando (2) en (1): W = (1 - tan 36°.tan 9o)2 - tan236° tan2 9o + 2 tan 36° tan 9o Luego de efectuar y simplificar, obtendremos: W = 1 - 2 tan 36° tan 9o + tan236° tan2 9o + tan236° tan29° + 2 tan 36° tan9° W = 1 Identidades Trigonométricas de Arcos Compuestas
17 .- Observamos que: 2x = (x + y + z) + (x - y-z) Luego: tan 2x = tan[(x + y + z) + (x - y - z)] Aplicando la identidad de la tangente del arco compuesto, obtenemos: tan(x + y + z)+tan(x-y-z) tan2x- j_fari(x+y + zj.tan(x-y-z) =* Finalmente: tan 2x = ~^¡- —b a + b , i . o a—b 2a tan2x =-----= “HE -| a + u ¿.t) a—b tan 2x = - 2 b 18 .- Hallemos la relación angular: Aplicando la tangente en ambos lados es la igualdad tenemos: tanF-^+al Lzo j tan tan^-taníyy-a) 4 \ 14 / 1 + tan^.tanÍ7“-o] 4 \14 / Luego de efectuar, obtenemos: tan 5n -------ct 28 1-1 ___2 1+1 2 1 2 _ 1 3 3 2 19 .-De la condición: sen(a - 0) = 3. cos[-(a - P)] => tan(a-P) = 3 ---(I) También: cot(a + P) = — => tan(a + P) = 2 ... (2) La relación angular es: 2a = (a - P) + (a - P) Tomando tangente a ambos miembros: tañía+P) + tan(a-P) tan(2a) = tan[(a + P) + (a - P)J tan(2a) = iZtan(a+P).tan(a_p) Reemplazando (1) y (2) tenemos: 2 + 3 ten<2a)= Th2)P> tan(2a) = -1 R9 392 Problemas de Trigonometría y cómo resolverlos RACSO IDITOIH
20 .- Recordando la Ecuación cuadrática: ax + bx + c = 0 De soluciones (raíces) x^ y x2 b c Se cumple: x¡ + x2 = - ~ ; x¡. x2 = — En el problema: k2x2 + x - k7x - 1 = 2k => k2x2 + (1 - k2)x + (-1 - 2k) = 0 De la condición: x^ = tan p, x2 = tan q , tenemos: -(1-Ar2) k2-l *1 + = tanp + tang = —p— = ~p— _ i _ yk x1.x2 = tan p . tan q = ——- Luego de efectuar el ángulo compuesto y reemplazando lo anterior, obtenemos: tanp + tana tan(p + a) = ---------- ’ 1-tanp.tanq *21 k2 l+2k k2 Finalmente: tan(p + q) = 2** => tan(p + q) = tan(p + q) = K r Z.K T 1 (K T 1J rv T 1 21.- Escribiendo apropiadamente el desarrollo de la tangente de un arco compuesto obtenemos: tan24°+tan21° = l-tan24o.tan21' 1 - tan 24° . tan 21° = tan 24° + tan 21' Luego de efectuar obtenemos: 1 = tan 24° + tan 21° + tan 24°. tan 21° => N = 1 En forma análoga obtendremos: tan(63°-3°) = TT tan 63o-tan 3o 1 + tan 63°. tan 3o J3 + 73 tan63°. tan 3o = tan 63° - tan 3o => 73 = tan 63° - tan 3o - 73 .tan 63°.tan 3o => M = 73 Luego: N.M2=1(73)2 .-. N.M2 = 3 Identidades Trigonométricas de Arcos Compuestos R9 393
IDENTIDADES ESPECIALES 22.- Efectuando y ordenando apropiadamente, obtenemos: W = a sen x - a cos x + fe sen x + fe cos x W = (a + fe)sen x + (fe - ojeos x Construyendo un zá cuyos lados sean: (a + fe) y (fe - a) Así: b-a tan<¡)= --~ a + b La hipotenusa se calcula por Pitágoras, multiplicando y dividiendo la expresión W por ^2(a2 +fe2), tenemos: W= ^2(fl2+fe2) a-~-—.senx + —^=.cosx :«2+b2) W = -^2(a2 + b2) [cos <J>. sen x + sen <¡). cos x] Como se puéde observar la expresión anterior es el desarrollo del seno de un ángulo compuesto, luego: W = Jifa2 + b2). sen( <¡) + r) mínimo valor 1 Wmín Finalmente el mínimo valor de W será: 2 2 23.- Se sabe que: sen(i + y) . sen(x - y) = sen x - sen y Aplicando dicha identidad en el problema, tenemos: a+b a-b sen---•+— 2 2 .sen W = ----------L— cosacosfe+serwsenfe - (cosacosfe — senasenfe) seno.senb Finalmente: W = ---~ 2sena.senfe 24.- Utilizando las siguientes identidades: sen2o - sen2fe = sen(ot + 0).sen(a - 0) cos2fl - sen2fe = cos(ot + 0). cos(ot - 0) Aplicando dichas identidades en el problema, tenemos: 3941 Problemas de Trigonometría y cómo resolverlos 0UACSO L'JITOIBI
sen(38o+8o).sen(38o-8o) M - cos(38°+8<>).cos(38° 8”) M = tan 46°.tan 30°.tan 44' Finalmente por ángulos complementarios, obtenemos: M = tan 46° . cot 46° - tan 30° M= 3 25.- Escribiendo apropiadamente en el numerador y aplicando una identidad especial en el denominador, tendremos: M = 1 J3 —.senl0°+—- cosl0° 2 2 + 3cos70° sen(55°+18o).sen(55o-18°) A continuación notamos que en el numerador aparece el desarrollo del coseno de un arco compuesto, también cambiamos: cos 70° por sen 20°, por ángulos complementarios: 4(sen300.senl00+cos300.cosl0c>) + 3cos70° sen73°.sen37o 4cos(30o-10c)+3sen20° sen73°.sen33° En forma similar escribiendo apropiadamente formamos el desarrollo del seno de un arco compuesto: 5(|cos20°+|sen20o) 5(sen53°.cos20o+cos53°.sen20c) sen73ü.sen37° => sen73"sen37‘> Finalmente, luego de simplificar y reemplazar los valores numéricos notables tendremos: 5.sen(53°+20°) 5 25 sen73°.sen37° = 3/5 M = T 26.- A continuación utilizaremos las siguientes propiedades: sen(« + b) sen(a + b) tan a + tan b =--------r a cot a + cot b = ------------- cosí?, coso senn.senb En el problema, luego de expresar adecuadamente el denominador, tendremos: sen(í? + b) sen(fl + b) w _ cos n. cos b seru.senb - sen(í? + b) sen(« + b) cos a. cos b serv/.senb Efectuando el numerador y factorizando: -sen (a + b) en el denominador obtenemos: Identidades Trigonométricas de Arcos Compite tos
W = sen2(a + b) sena cos asenbcosb —sen(a + b) 1 1 senasenb cos a cos b Luego de efectuar y simplificar, obtenemos: sen(a + b) yy _ sena.cosa.senb.cosb cos(a + b) sena cos asenb cos b W = - tanta + b) 2 2 27 - Se sabe que: cos(a + b). cosía - b) = cos a - sen b 2 2 2 2 De la condición: tan x . tan y = cos a - sen b => tan x . tan y = (1 - sen a) - (sen b) => sen a + sen b = 1 - tan .r . tan y... (1) De las condiciones: tan x = cos(a + b) = cos a cos b - sen a sen b ... (2) tan y = cosía - b) = cos a cos b + sen a sen b ... (3) Sumando (2) y (3), tenemos: tan x + tan y = 2 cos a . cos b ... (4) Reemplazando (1) y (4) en W tenemos: W = ----------:------ 1-tanx. tany Finalmente: W = cot(x + y). tan(x + y) .-. W = 1 28.- Se sabe que: tan a + tan b = -------~ , luego: cosa.cosb De(l): sen(x + y) - m sen(x + y) cosx. cosy => 1 COS X . COS V = — . m --(4) De (2): sen(i/ + z) - n sen (y + z) cosy.cosz => 1 cos y. cos z = — . n ..(5) De (3): ----------- = v sen(x + z) => cos x. cos z = — ... (6) cosx. cos z r p Multiplicando las expresiones (4), (5) y (6), obtenemos: (cos x. cos y cos z)2 = cos x. cos y. cos z = í»wnp)'1/2 Problemas de Trigonometría y cómo resolverlos .4-ACSO WlDITOlll
29.- Dato: tan 14° = a[tan 52° - tan 38o] Expresando: 14° como «52° - 38o» Se tiene: tan (52° - 38°) = a (tan 52° - tan 38°) Efectuando r tan52°-tan38° 1 _ .. , „o™ L1 + tan52°.tan38°J ~ a (tan 52 ' tan 38 > Simplificando: 1 l + tan52°.cot52° “ a Pues: 38° + 52° = 90° De los cual: 1 a= 2 30.- En general: sen(x + y + z) = sen x cos y cos z + sen y cos x cos z + sen z cos y cos x - sen x sen y sen z ¡Dividiendo a todo entre: cos _v . cos y . cos z sen(x + y+ z) Se obtiene: cosrcosycosz ~ tan x + *-an V + *-an z' ^an x ^an V ^an z En el problema: x = 19° ; y = 38° ; z = 33° sen(19°+ 38°+ 33°) Efectuando: cosi9° eos 38° cos 33° = tan + tan + tan 33° ~ tan ^"-tan 38°.tan 33° Como: sen 90° =1 a CqSX = sec x Se obtiene: tan 19° + tan 38° + tan 33° - tan 19° . tan 38°. tan 33° = cos go COggg»cos 330 VV = sec 19° . sec 38° . sec 33° 31.- Según el enunciado, los datos son: x2 + y2 = z2 ...(i) x sen 6 + y cosO = z ... (ii) Apartir de los cuales, por propiedad de identidades se obtienen: x y sen 6 = - a cos 6 = — Se pide: M = V2 sen (o + j Identidades Trigonométricas de Arcos Compuestos , ’ 397|
Que luego de desarrollar, se transforma en: M = sen 6 + cos 6 x+y M =---- z 32.- Recordemos: sen(c±b) tan a ± tan b =-------~ cosa, cos b Luego en la condición: tan x - tan y + tan y + tan z = 2 sen(x + z) Finalmente al simplificar, obtenemos: seníx + z) --------- = 2 sen (x + z) cosx.cosz 1 cos x . cos z = 2 SITUACIONES GRÁFICAS g 33.- De la figura: tan(a + 0) = — 3 tan ct = — 12 tan (ct + 0 + ct) = — tan(a + 0) + tana 1 - tan(a + 0). tana 12 Efectuando, tendremos: 8.3 x + x _ 12 24 x 1 llr _ 12 x2-24 * 11? = 12? - 288 Luego: ? = 288 .’. 34.- En la figura a + 6 = 0 => 6=0-0 Luego: tan 6 = tan (0 - a) tan0- tana tan 6 = ------------ 1 + tan 0. tan a l-1 tan 6 = -1 1+- 3 2 _ 2 ~ 4 3 Finalmente: „ 1 tan 0=2 3981 Problemas de Trigonometría y cómo resolverlos jA RACSO WlDIIOlSI
35.- De la figura: tan a = — a tan(a + 0) = — B (-2)2 - 4(tan 6)(3 tan 6) > 0 Simplificando, obtenemos: 4 - 12 tan 6 >0 => tan 6 < ~ => -<E------------------L- 3 -1/V3 1/-J3 Luego 6 es máximo si: tan 6 = = tan 30° 0 = 30° 37.-Sea UR = 6a, ni Z PTS = a, wi Z QTLJ = P En el LTPS: tana = | ... (1) PT 5a 5 ' 7a EnelLTQU:tanP=^J | ... (2) La relación angular de la figura es: S Identidades Trigonométricas de Arcos Compuestos
a + 6 + P = 7t => 6 = 7t-(a + P) Tomando tangente en ambos lados de la igualdad tenemos: tan7t- tan(a+P) tan 0 = tan [7t - (a + P)] = ---—-----— = -tan (a + P); pues: tan ít = 0 1 ' l + tan7t.tan(a + P) ' H r Por lo tanto: tan 6 = - tan a + tan P 1-tanatanP --.(3) Reemplazando (1) y (2) en (3) tenemos: tan 6 = - r z+á i 5 5 1-Z.l L 5 5j 8 5 _ 40 18 18 25 „ 20 tan 6 = -y 38.-Sea: OE - 2r; OF = r; OB = 2r; BC = 4r , mZFDC = P Prolongamos el segmento DC y bajamos la perpendicular FG. La relación angular es: a + P = 45°; de lo cual: a = 45°-P RACSO WPSDITOBBB R9 400 Problemas de Trigonometría y cómo resolverlos
o. En el LO2B2 => 3 \ B O3 Por Pitágoras: O3B = V132 ~52 = 12 12 5 sen P = — ; cos P = — La relación angular es: 6 = u + P Luego: Finalmente: sen 6 = sen( a + P) sen 6 = sen a sen P + cos cesen p 3 5 4 12 SOTe= 5 ñ+ 5 ' B n 63 sen6= 65 40.- Sea P = m Z EAB y 0 = tn Z ABE tn Z AEB = tn Z CED - a Por ser ángulos opuestos por el vértice. En A ABE se cumple: a + P + 6 = 7t a = 7t - (6 - P) tan a = tanpr - (6 + P)] Desarrollando: tan(7t - x); se obtiene: -tan x; ya que: tan 7t = 0 . 4 m t -[tanO+tanP] tan a - -tan (6 + P) => tan a =-------------— ... (1) 1-tanO.tanp Sea DE _L BC => en el LDFC por Pitágoras tenemos: FC = VdC2-DF2 = 7(^13 )2-22 FC = 3 FB = AD = 3 k AD 3 En el LBAD => tan 6 = = ~ AB 2 BC 6 En el L ABC => tan P = = — = 3 AB 2 reemplazando (2) en (1): tan a = - 9 2^7 y — y COt Ct — ~2 Identidades Trigonométricas de Arcos Compuestos
41.- Sea EF = 2a => BE = a BF = FC = 3a => AB = 6a Sea: m Z BAE = a v m Z BAF = P La relación angular es: 6 = P — a Luego: tan 6 = tan(P - a) tanP-tana tan 6 = ~„___________(1) 1 + tanptana 1 En el LBAE => tan a = ~ 6 1 En el tX BAF => tan P = ~ k-- (2) Reemplazando (2) en (1) tenemos: 1_1 tan 6 = 2 A 1+-.- 2 6 4 _ 12 _ A - 13 - 13 12 La hipotenusa se halla por Pitágoras: 42.- Sea AB = x, tn Z. BAC = ni Z DAE = 0, tn Z CAD = a Planteamos las siguientes relaciones angulares. En el L ABC => tan 6 = ~ 3 En el L ABD => tan (6 + a) = — En el LABE => tan(26 + a) = También se debe cumplir: 6 = (26 + a) - (6 + a) tan 6 = tan[(26 + a)-(6 + a)] tan(2e+a)-tan(e+a) tan “ 1 + tan(26 + a). tan(6 + a) " ( ’ Problemas de Trigonometría y cómo resolverlos RACSO JDIYO1II
Reemplazando (1) en (2): — x2 + 18 = 3X2 => x2 = 9 43.- Sea: AB = x; tn Z CAB = a En el L ABC => tan a = x 3 En el L ABD => tan (a + P) = — En el LABE => tan P = 6 3 3 x x 1 X 1 3* ’ 1+É 3 X “ X2 + 18 x “ x2 + 18 x* ¿ = > x = ± 3 AB = x = 3 m La relación angular es: P = (a + P) - a tan p = tan[(a + P) - a] => tan P = tan(a-P)-tana 1 + tan(a + p). tana 3__1 X Y Y Reemplazando (1) en (2): — =--rnr 6 1 + -.- x x x2 + 3 = 12 x = AB = 3 La hipotenusa se halla por Pitágoras: IDENTIDADES TRIGONOMÉTRICAS DE LA SUMA DE 3 ARCOS 44.- Se cumple que: cos(n-b) --------r = 1 + tan a . tan b cosa.cosb Aplicando dicho teorema en el problema, tendremos: M = 1 + tan x.tan y + 1 + tan y. tan z + 1 + tan x.tan z M = 3 + (tan x . tan y + tan y . tan z + tan x . tan z) Identidades Trigonométricas de Arcos Compuestos R9 403
Como: x + y + z = 90° => tan x . tan y + tan y . tan z + tan x . tan z = 1 Finalmente: M = 4 45.- Del dato: A + B = 90° - C => tan(A + B) = tan (90° - C) ... (1) Pero: tan (90° - C) = cot C (por arcos complementarios) En (1): desarrollando al lado izquierdo tenemos: tanA + tanB 1 ------------= cot C =----— => tan A tan C = tan B tan C = 1 - tan A tan B 1 —tan A tanB tanC => tan Atan B + tan B tan C + tan A tan C = 1 W = 1 46.- Del dato: A + B = 180° - C tan(A + B) = tan(180° - C) ... (1) Pero: tan(180° - C) = - tan C (Por reducción al IC) tanA + tanB En (1): desarrollando el lado izquierdo tenemos: ~~~ = - tan C 1 1-tanA. tanB tan A + tan B = - tan C + tan A tan B tan C tan A + tan B + tan C = tan A tan B tan C => W = tan Atan B tan C - tan A tan B tan C W = 0 47.- Como: cot x - tan y = tn => 1 - tan x.tan y = tn tan x ... (1) cot y - tan z = n => 1 - tan y . tan z = n tan y ... (2) cot z - tan x = p => 1 - tan x . tan z = p tan z ... (3) Sumando las igualdades: (1), (2) y (3) obtenemos: 3 - (tan x. tan y + tan y. tan z + tan x. tan z) = tn . tan x-n . tan y + p tan z... (*) 71 Como: x + y + z= — => tan x. tan y + tan x . tan z + tan y . tan z = 1 Luego en (*): 3 - (1) = tn . tan x + n tan y + p tan z .-. tn tan x + n tan y + p tan z = 2 Problemas de Trigonometría y cómo resolverlos PACSO tOITOlll %
48 .- Como: sen x + cos y . cos z = 0 => = -1 , . (*) Pero: x + y + z = 180° => sen x = sen(y + z) sen(y + z) En (*): ----------= - 1 => tan y + tan z = - 1 ... (1) cosy.cosz J También si: x + y + z = 180° => tan x + tan y + tan z = tan x . tan y . tan z Por (1): tan x - 1 = tan x . tan y . tan z .-. M = tan x - 1 49 .- Agrupando convenientemente los ángulos, tenemos: senj(x+z) + y)] cos[(x+z)-y)] sen(x + z)cosy + cos(x+z).seny cos(x+z).cos y + sen(x+z)seny Dividiendo el numerador y el denominador entre cos y . cos(x + z) tenemos: sen(x + z)cosy + cos(x + z).seny w=__________cosy.cos(x + z)_______ cos(x+z). cosy + sen(x+z).seny cosy.cos(x+z) „ , x tanx+tanz Pero: tan(x + z) =------------ 1-tanx.tanz w_ tan(x+z)+tany 1 + tan(x + z).tany 2 + 3 1-2(3) = -l ...(2) (2)en(l): ______________3 l+(-l)(4) " -3 W = -l 50 .- Sumando miembro a miembro tendremos: tan A + tan B = k tan A + tan C = l tan B + tan C = m . 2(tan A +tanB + tanC) = k +1 + m => tan A.tan B.tan C = __(1) sen C sen(A+ B) De la condición: tan A + tan B = k => cosAcosB ~ Luego: sen C = k cos A cos B Identidades Trigonométricas de Arcos Compuestos
Análogamente: sen B = l cos A cos C sen A = m cos B cos C Ahora multiplicamos las expresiones anteriores, obteniendo: sen A.sen B.sen C = klm cos2A.cos2B.cos2C tan A.tan B.tan C = klm cos A cos B cos C ... (2) De (1) y (2) obtenemos: klm cos A.cos B.cos C = k+l + m 2kltn sec A.sec B.sec C = 51 .- Reduciendo aproximadamente las expresiones indicadas tendremos: sen(B + C) = sen (ít - A) = sen A cos(B + C) = cos (ti - A) = -cos A cos C = cos[7t - (A + B)J = -cos (A + B) cos(B - A) + cos(A + B) = cos B cos A +- sen B sen A + cos A cos B - sen A sen B = 2 cos A cos B Luego en la expresión W, obtenemos: 2 * 22^ sen A-cos B-cos C -cosA.cosB.cosC 2ti 2 * . 2x—> yy _ cos B-sen A+cos C cosAcosBcosC cos(B+ A).cos(B- A)+[- cos(A+ B)]2 => w =------------------------------------- cosAcosBcosC cos( A+ B)[cos(B- A)+cos(A+ B) J W = cosAcosBcosC Finalmente: _ - cosC .2 cosB. cosA - cosA.cosB.cosC W = -2 R9 406 Problemas de Trigonometría y cómo resolverlos RACSO JDITOUI
Cff.flo Reducción al er Cuadrante 01.- Del enunciado del problema, podemos reconocer que: csc (90° - A) - x. cos A . cot (90° - A) = sen (90° - A) V----v-----Z X----v-----Z V-----v-----z sec A tan A cos A Todas las equivalencias indicadas se deducen de la aplicación de reducción al IC ex- puesta en el caso 1 (item 10.1) sobre ángulos complementarios. Ahora, escribimos el problema como sigue: sec A - x. cos A . tan A = cos A => sec A - cos A = x . cos A . tan A Por identidades trigonométricas, deducimos: 1 . . senA 1- eos2 A . sen2A r- - cos A = x^eós A. . => —— = x. sen A => -----------------t- = x sen A cosA ^-cosA cosA cosA Luego de simplificar: sen A, se obtiene. x = tan A 02.- Se pide: E = cos Io + cos 2o + cos 3o + ... + cos 179° + cos 180° ... (i) Que también se puede expresar como sigue: E = cos 179° + cos 178° + cos 177° + ... + cos 1° + cos 180° -.. (ii) Sumando miembro a miembro, y aplicando: cos x + cos y = 0; cada vez que: x + y = 180°. Se obtiene: 2E = 0+ 0 + 0 + ... + 0+ 2- cos 180° 2E = 2 - cos 180° = 2(-l) .-. E = -1 7ye 03.- Por ángulos suplementarios, tendremos: sen = sen ^2 Y por ángulos complementarios: 5rt ít sen = cos 12 7n n sen 12 — cos 77t 571 71 cos = *cos 12 = -sen 12 Reducción al Prime Cuadrante R10 407
Y por ángulos complementarios: cos = sen => cos 12 ~sen 12 Reemplazando en la expresión inicial, obtendremos: 7t 7t cos1Q scnT9 ----77 + = 1 -1 = o cos^ —sen— 04.- a) Eliminamos el número entero de vueltas, para ello se descompone el ángulo a reducir en múltiplos de 2 7t más un cierto ángulo. 52rt 25rt (1^ 4ti\ Lz 7t\ sen -j-.eos = senllOT+-^-Icos L&n-t-—I 52ti 25tt 4tt tt ,. , . ,. sen —3 - .eos = sen -y . cos = (-) (+) = (-) IIIC IC b) Eliminamos el número entero de vueltas 32rt 22ti sen —5— .cot —g- = sen 32rt L 227t 27t . 4tt sen —5— .cot -g— = sen . cot = (+) (+) = (+) IC IIIC . I 205ti \ 737t 2057t 73tt c)senl----g— l.cot^g- =-sen—3— .cot -Jq~ Eliminamos el número entero de vueltas sen(-^y^).cot^ =-sen .cot sen(-^|^) .cot = -sen j . cot = (-) (+) (+) = (-) .—v—< i—„—í IC IIIC Finalmente, los signos serán: (-); (+); (-) 05.- Trabajando por separado con cada expresión, obtendremos: R = cos (72t5° + 90°) + cot (360° + 65°) (eliminando el número entero de vueltas) R10 408 Problemas de Trigonometría y cómo resolverlos RACSO DITOtBI
R = + cot 65° => R = cot 65° 0 S = sen Q6(í° + 90°). tan ^2d° + 65°) => S = sen 90° . tan 65° 1 R.S = 1 Finalmente: R.S. = cot 65°.tan 65° 06.- Descomponiendo cada ángulo convenientemente: (*) 11^ + x = 2ít + 3^ + x => sen (11^+x) = sen ^3^+xj (*) 33^ + y = 87t + j +y => sen ^33^ + y) = sen (f+y) (*) 55^ + x=13n + 3^ + x => cos ^55^+xj =-cos ^3^+xj O 77f +y = 19jl+| +y => cos (77^ +y) =-cos (^ +y) Reemplazando en lo que se pide: A - B, se tiene: A-B=sen (s^+xj.seng + y) - [-cos(^+xjj.[-cosf^+y Factorizando (-) , y ordenando: A - B = -rcos^ + xj-cos^+yj-sen^ + xjsen^+y A - B = - cosÍ3^ + x+^+yj = -[eos (ít + x +y)] = -[-cos(x + y)] A - B = cos (x + y) 07.- Utilizando cada uno de los casos de reducción al primer cuadrante, tendremos: sen (ít + x) = - sen x cos ("2+x) = - sen x cos (x - ít) = - cos x sen +x) = - eos x csc (n + x) = - csc x sec (^+xj = +csc x tan = * cot x « c°t = + tan x Reducción al Primer Cuadrante R10 409
2 2 (-senx)(-senx)+(-cosx)(-cosx) sen x + cos x Luego: W = —--------—---r—~; => *v =---------g Z (-cscx)(cscx)-(-cotx)(tanx) -csc x+1 final mente. W =------5— VV = - tan2x -cot x 08.- Reduciendo cada uno de los factores al primer cuadrante, tendremos: Se observa que: tan -ct) = + cot a sec (3ti - a) = - sec a sen ~ ' cos a cos í" + a) = + eos a (cota)(-seca)(-cosa) cosa 1 W =---------------------- => w = cot a. sec a =-----.------- W = csc a cosa sena cosa u9 - Reduciendo cada uno de los factores al primer cuadrante, tendremos: (.) sen (ti - x) = sen x Luego: (.) cot (r-2£)=cot[-^-x) cot =-(+tanx) (.) cos (r - 36ti) = cos x (.) ^lafí^x= -cot x jfserTxX^tarííjfcosx) XtarTxj(-cot x)(sen xj2* => cot =-cotp?p-xj cot =-tanx (.) tan (13ti + x) = tan x (.) cos (^y^+xj =+senx cosx W = --------— cotx.senx ... cosx w =------- cosx W = 1 Nota: Recuerda que V k e Z R.T.(2/c7t ± a) = R.T.(± a) R. L. [(2k+1) J +a] = ± CO-R-T.(a) R.T.(J:n ± a) = ± R.T.(a) Problemas de Trigonometría y cómo resolverlos -Cmracso WIDITOltl
10.- Utilizando los casos de reducción, tenemos: cot(19957i - 6) = - cot 6 cos(239-^ + ej = cos(1187i+^+oj= + sen 0 sec(-8047i + 6) = + sec 6 sen(-161^ + o) = -sen^SOn + ^-o) = -cos 6 Además se debe tener en cuenta que: 239 = 60(4) -1= 4-1 -161 = -40(4) -1 = 4 - 1 (-cot6)(sen6) Luego. W - W = cot 6 . sen 6 cosO senO . sen 6 W = co* 0 11.- Reduciendo al primer cuadrante, como sigue, tendr mos: (*) tanp7^ + aj =-cota (*) cot(175 ti - a) =-cota (*) sen(809 ti + a) = -sen a (*) cot(72 ti - a) = -cot a (*) sen^91^-aj =-cosa (*) sec^55^ + aj = csc a (-cota).(-cota).(-sena) 1 Luego: ----------------—-----— = - — 1 (-cota).(-cosa).(csca) 9 sena-— -* => x2 = /2-^ =>x2 = 80 => x = -4j5 Finalmente: M = 2-»/5 . .-. M = - 40 + 9 = - 31 12.- Efectuando en la expresión la reducción al primer cuadrante, tendremos: l-sen(x-277t) 1-cos(-10ti+x) l-cos(-x+23^j l-sen(-x + 13^j (*) sen(x - 27ti) = -sen x (*) cos(-10n + x) = +cos x (•) cos(-x+23^j =-sen x (*) sen(-x+13^j = +cos x Reducción al Primer Cuadrante R10 411
1 - (-senx) 1 - (cosx) Luego: W = -—-------r - -—-----7 0 l-(-senx) l-(cosx) W = 0 W = 1 - 1 13.- Para el calculo de «M», analizamos para «n» par y «n» impar, así: Si "n" par: (-l)n = 1 M = sen(n7t-^j .esc => M= (-sen^j (+csc^j =-l Si "n" Impar: (-l)n = -1: M = sen^rat+^j. csc^nn-^j => M = (-sen^j. esc— = - 1 Finalmente: M = -1 14.- Como: 71 x + y= 2 tan 9 = Pero: sen^ + y).tan(7i+y) cos^+xj.tan^+x) (cosy).(tany) tan U - (_senx).(-cotx) seny cosy---— cosy cosx senx----- senx tan 0 = sen y cosx x + i/ = 71 2 => sen y = cos x => tan 6 = 1 „ 371 /-,. e=-TG r'2) 15.- Reduciendo cada uno de los factores de la expresión «M» al primer cuadrante, tendremos: (*) sen(180° - a) = sen a (*) tan(1260° + a) = tan(180° + a) = tan a (*) sen(450° + a) = cos a (sena)(sena)(tana) Luego: M = -------------— 0 (-cosa)(cosa).tana (*) cos(a - 90°) = cos(90° - a) = sen a (*) cos(540° - a) = cos(180° - a) = -cos a (*) tan(360° + a) = tan a sen2a _, 2 M =------2— M = "tan a -cos a R10 412 Problemas de Trigonometría y cómo resolverlos RACSO piDITOftia
16 .- sen (55y - 6 j = - cos 6 cos (?7y + ©) = - sen 6 1 En la condición tenemos: k(- cos 6)(- sen 6) = — 1 => sen 6 . cos 6 = — ... (1) Se pide: W = sen2© + 2 sen 6 cos 6 + eos2©.. .(2) ( 1 1 Jt+1 (1) en (2): W=l+2 — I = 1+ ~ = -j~ l Z/í I JC 17 .- Reduciendo al primer cuadrante, tendremos: (lcn7ti /n 5ti\ 571 71 5717171 159—) =cosJ117i+—) = -cos— = -seny ;pues: 14 + 7 = 2 sec (242^ j = sec ^81 Tt-y) = -sec y sen(125y) = sen(187t-y) =-seny Reemplazando los resultados anteriores en W, obtenemos: W = -seny + (-secy) - (-seny) => W = -secy .-. W = -2 18 .- Aplicando el caso de reducción para ángulos mayores de una vuelta, tendremos 3000° = 8(360°) + 120° => cos(3000°) = cos 120° = - cos 60° 2000° = 5(360°) + 200° => cos(2000°) = cos 200° = - cos 20° Reemplazando: cos 300° = + cos 60° (-cos60")-(-cos20°) 1 En la condición tenemos: -----------------—— = k => — cos6CP-(-cos20 ) k cos60"+cos20" -cos60°+cos20° ’ ’ ( J Se pide: (~cos60°) + (~cos20°) cos60°+(- cos 20°) cos60"+cos20" W = -cos60°+cos20° ''(2) Reemplazando (1) en (2), tenemos: W = y Reducción al Primer Cuadrante
19.- Sean los ángulos: A = a - r; B = a; C = a + r (ángulos en progresión aritmética) Como: A + B + C = 180° => 3a = 180° => a = 60° Escribiendo como sigue «M», tendremos: sen(180°+2B + C) cos(180°+A + 2C) M = ---------------- +----------------- sen(B - C) cos(B C) -sen(2B + C) -cos(A + 2C) sen(B-C) + cos(B-C) Reemplazando sus respectivos equivalentes en términos de a y r, tendremos: -sen(3a + r) -cos(3a + r) M = - ~ . sen(-r) cos(-r) -sen(180°+r) -cos(180°+r) => M = ---------------- +----------- —senr cosr Luego de simplificar, obtenemos: —senr cosr 20.- Reduciendo cada uno de los factores y reemplazando su respectivo valor numérico tendremos: J3 sen 420° = sen(360° + 60°) = sen 60° = 1 cos 240° = cos(180° + 60°) = -cos 60° = - - tan 405° = tan(360° + 45°) = tan 45° = 1 1 sen 210° = sen(180° + 30°) = -sen 30° = - ~ cos 225° = cos(180° + 45°) = -cos 45° = - tan 570° = tan(540° + 30°) = tan 30° = Luego: M= 2 RIO 414 Problemas de Trigonometría y cómo resolverlos RACSO *>HDITCkBB
21.- Si: 2 540° = 7(360°) + 20" luego: sen(2 540°) = sen 20° Del mismo modo: 1910° = 5(360°) + 110° => 2680° = 7(360°) + 160" => 2630° =7(360°)+110° => Reemplazando en W: cos(1910°) = cos(110°) = -cos 70° cos(2680°) = cos(160°) = - cos 20° sen(2630") = sen(110°) = + sen 70° sen20°+2.[-cos70°]3 -cos20°+2[sen70"]3 sen20°— 2sen320" -cos20°+ 2cos320° Ordenando y factorizando: sen20"(l-2sen220°) cos20°(-l + 2cos220") sen20"(l-2sen220") cos20"(l- 2 sen2 20°) Luego de simplificar, obtenemos: VV = tan 20° CASOS PARTICULARES DE REDUCCIÓN AL PRIMER CUADRANTE 22.- Trabajando como en el problema anterior, tendremos : cps-TO0" +'£bs-4Q^ + ajs-50" + cos 90° ... + cos 130° + cos 150" + cos 170° Efectuando las simplificaciones indicadas obtendremos : cos 10° + cos 30" + cos 50" + ... + cos 170" = 0 23.- Si : A + B + C = 180° , entonces: T -— pues: cos 90° = 0 tan A = -tan (B + C) tanB+tanC Desarrollando el segundo miembro, obtendremos: tan A = tanC de donde: Luego tan A - tan A . tan B . tan C = -tan B - tan C tan A + tan B + tan C = tan A . tan B . tan C 1 1 tanA+tanB+tanC ~ tanA.tanB.tanC ——7 —Z 77 = cot A. cot B. cot C tan A+ tanB+ tanC Reducción al Primer Cuadrante
24.- De la ecuación inicial, transponiendo términos, tendremos: /67i+3fl-2b\ tan (—g— J =-cot ( 4 J - TT . p«-3b\ /3n 3fl-2b\ Usando la tranformacion II: tan I—g—I = -cot l-2~+—4—I IV Q /2fl-3b\ J . I3a-2b\\ tan(“8^) =itan(d )) , /2fl-3íA . /3«-2b\ tan (—g— J = tan (— o . . . . - . 2a-3b _ 3o-2b De donde, deducimos. sfá'' — 2fl-3b = 6fl-4b => b = 4a 25.- Como: A + B + C = 180° => sen(A + B) = sen C => cot(A + B) = cot C También: tan(A + B + C + C) = tan (180° + C) = tan C senC Luego reemplazando en M: M = + Ia11 C - (-cot C) => M = 1 - jan C. cot C 1 M = 0 26.-Como: x + y = ti => sen x = sen y L y = ZE 2 + 2 ” 2 2x + 2y = 2ti => tanl 2] = cot => sen 2x = -sen 2y Reemplazando en M obtenemos: y sen y 2 cot— 3(-sen 2y) sen y + cot^ + sen 2y M = 0 M= 1 +2-3 Problemas de Trigonometría y cómo resolverlos RACSO DITOII1
T!7.- Recordemos, sé x + y = ti cos x = - cos y Entonces: 5ti 3ti COS— =-COS — 7ti ti eos— =-cos~ Reemplazando en "M" obtenemos: 71 3ti 371 71 M = COS“ + COS —-COS —-cos~ o o o o M = 0 {sen x = sen y cosx+cosy = 0 28.- Como: x + y = 180° => sen x = sen y => tan x = -tan y Como: y + z = 270° => cos y = - sen z ;=> tan z = cot y Luego reemplazando en "M" obtenemos: ( seny-senzA M = -- + (-2 tan y) . (2cot y) => M = 1 - 4 . tan y . cot y I seny—senz I j Pero: tan y. cot y = 1 => M = 1 - 4(1) .-. M = -3 29.- Si: a + 0 = 270° => tan a = cot 0 => tan(230° + x) = cot(40° - x) Si: a + 0 = 90° => tan a = cot 0 => tan(50° + x) = cot(40° - x) _ , , cot(40°-x) + cot(40°-x) En «M», tendremos: M =----------•---------- .-. M = 2 cot(40 -x) 30.- Como: a + 0 = 180° => esc a = esc 0 2m+l m + 2 ? ? Luego: -------7 =-----7 => 2m -m-l=2m + 3m - 2 0 2m -1 tn -1 A continuación al simplificar obtenemos: 4m - 1 = 0 => m= 31.- Sea: N = sen 120° + sen 140° + sen 160° + ... + sen 260° D = cos 20° + cos 30° + cos 40° + ... + cos 170° {sena = sen0 no n cosa+cos0=O v cosa=-cos0 Reducción al Primer Cuadrante
Ísena = -senf v sena+senf = O cosa = cosP Entonces: sen 120°+ sen 140°+ sen 160°+ sen 180°+ sen200°+ sen220°+ sen240° + sen 260° o => N = sen (270° - 10°) = -cos 10" D = cos20°+cos30°+cos40°+...+cos 160° + cos 170° o D = cos 170° D = cos(180° - 10°) = -cos 10° c- i •. N -coslO , Finalmente: M = M = 1 D -coslO 32.- Como: sen a = cos(f + 0)=>a + f + 6 = (4k +1)- , Vte Z => 2a + 2P + 26 = 4kn + ti tan[(4k + l)| + e] _cot0 Finalmente: M= coLt(4fal+7I + e) J = ” M='1 33.- Como: A + B+ C = 180° => 2A + 2B + 2C = 360° => sec(360° - A) = csc(360° + B) => sec A = csc B A+ B = 90° Luego: C = 90° .-. El triángulo ABC eS un triángulo rectángulo 34.- Como A + B + C = 180° => cos(A + B) = - cos C cos(B + C) = - cos A cos(A + C) = -cos B Luego: —cosC -cosA -cosB M =----— +-----— +----— cosC cosA cosB M = -3 PROBLEMAS GRÁFICOS DE REDUCCION AL PRIMER CUADRANTE 35.- L BMO: tan(7i - 6) = => BM = R tan(n - 0) Utilizando la relación al primer cuadrante: BM = -R tan 6 Problemas de Trigonometría y cómo resolverlos RACSO WlDITOlll
36.- En el gráfico, introducimos el ángulo auxiliar «a», notar que: a + 6 = 180° En el LBAH; calculamos BH, así BH= 7?2-22 = -J45 =3-j5 Por propiedad, si: a + 6 = 180° => tan 6 = -tan a tan 6 = 37.- De la figura: a + (- 6) = ti Luego: Pero: a ti 0 a = 7t + 0 y 7=2+2 . 2tai/—+— sen(7i + 0) b 2 M = ----~ ------^“5 ’L senG 0 cot— 2 sen(7i + 0) = - sen 0 f ti 0A 0 tan ir + iT =-cot - l2 2J 2 -sen© M =------ senG -2cot— _____2 0 cot— 2 Simplificando: M = - 1 + 2 M = 1 38.- De la figura los ángulos a y 0 + ti son coterminales. tan(a) = tan (0 + ti) .-. tan a = tan 0 También a es un ángulo en posición normal - =tana 7 = tan 0 a a tan 0 = b W = b-2b W = -b Reducción al Primer Cuadrante
/ 2 2 39.- Determinamos la longitud del radio vector OP = ^12 +(-5) 1 Luego: OP = 13 De la figura los ángulos 7i + a y 0 son coterminales. sen(n + a) = sen 0 -sen a = sen 0 ... (1) También 6 es un ángulo en posición normal sen 6 = HP -5 OP " 13 En(l): sena = - sen 0 =-^23 Finalmente: 13 sen a = 5 W = 5 ARACSO VpiDITORBa R10 420 Problemas de Trigonometría y cómo resolverlos
f AD M Identidades Trigonométricas del Arco Doble 01.- Escribiendo apropiadamente la expresión W, tendremos: W = 3 + cos 4x - 2(2 sen2x)2 => W = 3 + cos 4x - 2(1 - cos 2v)2 A continuación aplicamos la fórmula para «degradar» cosenos, así: W = 3 + cos 4x - 2 + 4 cos 2x - 2 eos 2x W = 1 + cos 4x + 4 cos 2x - (1 + cos 4a) Efectuando: W = 4 cos 2x 02.- Recordemos: cos4x - sen4x = (cos2x - sen2x)(cos2x + sen2x) = cos 2x 2 Luego: M = 2 cos 2x - 1 = cos 2(2x) /. M = cos 4x 03.- Descomponiendo la suma de cubos como el producto de dos factores, tendremos: (senx+cosx)(sen2x + cos2x-senx.cosx) 1 M= —------------------------ + o (2 sen x.cos x) senx + cosx - A continuación luego de simplificar lo indicado, obtenemos M = 1 - sen x. cos x + sen x . cos x M = 1 2 2 04.- Recordar: 1 + cos 2a = 2 cos a ‘ => 1 - cos 2a = 2 sen a 7 7 2 2cos a eos2 a eos a 1 2 => W =----------5-- =---------------y => W = ~ 2 2— VV = y. csc a 2sen 2a (2sena.cosa)z 4.sen a.cos a « 05.- Recordemos: cos4x - sen4x = cos 2x => M = cos6x - cos2x.sen4x + sen2x.cos4x - sen6x Factorizando como sigue: M = cos2x(cos4x - sen4x) + sen2x-(cos4x - sen4v) 4 4 -» > M = (eos x - sen x). (cos~x + sen~x) .-. M = cos 2x cos 2 r 1 Identidades Trigonométricas del Arco Doble
06.- Utilizando las identidades pitagóricas en los denominadores, tendremos: ,, cosx senx M — 2 2 secx.sec x cscx.csc x A continuación escribiendo en términos de senos y cosenos obtenemos: ,, cosx senx 4 4 M = —í— - —— = cos x - sen x 3 3~" cos x sen x 2 2 2 2 Finalmente: M = (eos x + sen x) (eos Xysen x) 1 cos2x M = cos 2x 1 + tan x 07.- Recordando: sec 2x = ------,— 1 tan x Luego en el problema, tendremos: VV = sec^^-2a j W = sec ^-4a VV = esc 4a 08.- Escribiendo apropiadamente la expresión tendremos: M = tan2x 2tanx 1 + tan2 x 2 - sen 2x . sec x Luego de factorizar «-sen 2x», obtendremos: M = -sen 2x(l) 2 2 M = tan x . sen 2x - sen 2x . sec x 2 2 M = -sen 2x(sec x - tan x) M = -sen 2x 09.- Escribiendo la expresión en términos de la «tan x», tendremos: ,, tanx coty B, tanx cotx tan4 x (1 + tan2 x)2 f 1 + tan2 x (1 + tan2 x)2 (1 + tan x/" [ tan2 x J „ . . tanx(l + tan2x) tanx Factonzando como sieue obtenemos: M =-------------5—5— => M = ------------5— (1 + tan2 x)2 1 + tan2 x sen 2x = Pero: 2tanx 9 1 + tan x 1 M = —. sen 2.x Problemas de Trigonometría y cómo resolverlos ARACSO W^BDITCKBa
10 .- Se pide: W = cot 7o - 2 cot 14° Por propiedad de arco doble: cot x - tan x = 2 cot 2x En el problema: cot 7° - tan 7° = 2 cot 14° Agrupando convenientemente: cot 7° - 2 cot 14° = tan 7° W = tan 7° 11 .- Escribiendo como sigue la expresión: W = cos 2x - (sen x + cos x).(senx-cosx)2 => W = cos 2x - (sen i f cos x).| senx-cosx | <o Pero del dato: sen x < cos x => sen x - cos x < 0 Luego en W, tendremos: W = cos 2x - (sen x + cos x).(cos x - sen x) 2 2. Finalmente: W = cos 2x - (cos x - sen x) => W = cos 2x - cos 2x .’. W = 0 12 .- Escribiendo en términos de senos y cosenos, tendremos: M = sen0/2 COS0/2 „ 2 0 T-é------e-2s€n I 2sen—.eos— 2 2 2e COS - r a 20 l-4sen —.eos — ________2 2 2cos2 — 2 20 . cos — 2 M = Finalmente, utilizando las fórmulas del doble y las identidades trigonométricas funda- mentales, obtendremos: M = 1 0 0? 1 - 2sen—.eos— ( 2 2 J 2 *. cos2e M = 13.- Escribiendo en términos de senos y cosenos y fectuando apropiadamente, tendremos: , . senx W = 1 + cos x + sen x + --- cosx senx(cosx + l) W = 1 + cos x +------------- cosx ... ,, . f, senx'i W = (1 + cos x) 1 +----- cosx J 1A, /i \ (cosx + senx) W = (1 + cos x)------------ cosx Expresando los factores lo más abreviadamente posible, tendremos: Identidades Trigonométricas del Arco Doble
7 X W = 2cosz— . '2 sen . secx W = 2 -J2 eos2 j .sen (x + j .sec x PROBLEMAS CONDICIONALES 14.- Resolviendo en el lado izquierdo de la igualdad, tenemos: = (2 sen2a)2 + 4 (2 cos2a)2 A continuación, utilizamos las fórmulas de «degradación» del doble, obteniendo: 7 7 1 2 = (1- cos 2a)2 +4(1 + cos 2a) 7 7 „ 7 z, 1 1 _ 1 2„ = — - 77 cos 2a + -¿ cos za + -7 + 77 cos 2a + -y cos 2a 4 2 4 4 2 4 = 2-3 cos 2a + 2 cos^a = 2-3 cos 2a + (1 + cos 4a) = 3-3 cos 2a + cos 4a Comparando con el lado derecho: tn =3 ; n = - 3 ; p = 1 sec 2a = Análogamente: l + tan2a l-tan2a 1 + sec 2x = tan2x tanx tan2x tan4x (1 + sec 2x). (1 + sec 4x).(l + sec 8x) =-- . -—— tanx tan2x tan8x - -----= tan 8x . cot x tan4x Comparando tenemos: A = 1, B = 8, C = 1 A+C Problemas de Trigonometría y cómo resolverlos RACSO BDITO1II
16.- Desarrollando en el lado izquierdo de la igualdad: a í 2tanx ) A ------2~ ^1+tan x J f 2 A 1 —tan x 1 + tan2 x => 2A . tan x + B - B tan2* + C + C tan2* = 0 (C-B) tan2* + 2A tan* + B+C = 0 p 9 r p = C-B r=B + C 1 x 18.- De la condición: cos 2x = 1 - 8 cos — 2 . 2* 1-cos - sen2 => cos 2x = 1 - 2^2sen^.cos^j Finalmente: 2 => cos 2x = 1 - 2 sen * => cos 2* = - cos 2* cos 2* = 0 + C = 0 2A q B+C “ r M = 3 19.- Del dato, elevamos al cuadrado: (4 sen a - 3j2 cos a)2 = 52 16 sen2a - 2(4 sen a) (3-J2 cos a) + (3-J2 cos a)2 = 52 8(2sen2a) - 12^2 (2 sen a eos a) + 9(2cos2a) =25 --.(1) l-cos2a sen 2a l+cos2a 8(1 - cos 2a) - 12 42 . sen 2a + 9(1 + cos 2a) = 25 8-8 cos 2a - 12 J2 sen 2a + 9 + 9 cos 2a = 25 cos 2a - 12 J2 sen 2a = 8 Finalmente: W = 8 Identidades Trigonométricas del Arco Doble
20.- De la condición: cos x - sen x = —--— (zcosx)m 2 * 2 cos x - 2 sen x cos x = — tn > 1 + eos 2x - sen 2x = — => cos 2x - sen 2x ----------- tn tn 1. í * m Elevando al cuadrado: (cos 2x - sen 2x) = -------- l ™ 2 2 I 1 — tn = > cos 2x + sen 2x - 2sen 2x.cos 2x = ----------- I tn , -2 íl-mY 2m-l 1 - ---- = sen 4x => sen 4x - —y~ tn j m /i m2 csc4x= =----- 2m- 1 21.- Escribimos convenientemente, para hacer aparecer la fórmula del seno del doble: 3 > 3 , W = 1 - — (2 sen x cos x) => W = 1- — . sen 2x Finalmente reemplazamos el seno del doble, obtendremos: W = l- 3 W= 4 22.- Como: cosx tn senx n tan x = — tn 3 í 1 V 3(1) 4 -1‘ 4 l3J n Reemplazando el valor de la tan x, en la expresión «E»z obtendremos: E = 777 •j l-tan x l+tan2x 2 tan* 1 + tan2 x E = tn Luego de simplificar y agrupar convenientemente tendremos: 3 2 2 m - tnn + 2tnn E = 2 2 tn +n 3 2 tn +mn E= j 2~ m + n 7 2 m(m + n ) Finalmente: E = ----j--5— E = m m +n Problemas de Trigonometría y cómo resolverlos % RACSO DITOBBB
23- Utilizando el triángulo del doble en el problema, asi: 2 tana 1 + 2 En el dato: k=----L±_tan_a 1 1 - tan a 1 + tan2 a 1 + tan2 a + 2 tana 14- tan a______ 2 7 l+tan a-l + tan a 1 + tan2 a 7 (1 + tana) K — -7 2 tan a 2 tan2 a _ 1 (1 + tana)2 k W = 2tan a 1-tan a 24.- Como: cos x . cos y = sen a .. . (1) sen x . sen y = cos a ... (2) Sumando las proposiciones (1) y (2): cos x . cos y + sen x sen y = sen a + cos a Luego obtenemos: cos(x - y) = sen a + cos a ... (3) Elevando al cuadrado la expresión (3): cos2(x - y) = (sen a + cos «)2 A continuación utilizamos la identidad trigonométrica fundamental obteniendo: 1 - sen2(x - y) = pen2« + cos2« + £seno cosa 1 sen 2n 7 1 - sen (x - y) = 1 + sen 2/7 2 sen (x - y) = - sen 2a 25 .- Hallemos la relación angular, así: 2x-15— 17 Aplicando cosenos en ambos miembros de la igualdad: cos 2x-15 A 17 (1) Reemplazando la condición en la expresión (1), obtendremos: Identidades Trigonométricas del Arco Doble
26 .- Hallemos la relación angular: ín x A f 2x A n ^4 5 J 5 J 2 2x^1 (71 xA 71 T J=2[4 + 5 j*2 Aplicando senos en ambos lados de la igualdad: 27 .- Se sabe que: tan a + cot b =------- cosa.senb y Del dato: tan 2x + cot x = 8 cos x Efectuando como sigue en el problema obtenemos: cos(2x-x) 2 cosx 2 ------ =8 cosx => -- = 8 cos x => 1 = 4(2 senx. cosx) .eos 2x cos2x.senx----------------------------cos2x.senx---------------------- sen2x Efectuando como sigue: 1 = 2 (2 sen 2x. eos 2x) sen4x => 2 sen 4x = 1 M = 1-1=O 28 .- De la C.T. se observa que: tan~xj = cot x => tan(37i + x) = tan x En el dato: cot x - tan x = 2k cosx senx De donde: senx cosx cos2x sen2x 2 2 eos x-sen x sen xcosx 1 tan 2x = — k •y y i 1 + tan « ~ I-tan a Luego: sec 2a = -----5— a eos 2a =----------?— 1 —tan a 1 + tan a Problemas de Trigonometría y cómo resolverlos ¿Jkl ACSO niDlIClEI
En el problema: 1 + tan2 2a 1 — tan2 2a 1 - tan2 2a 1 + tan2 2a 1 k2 l4 .2 ____ k2—1 k2-l ’ k2+l Efectuando y simplificando obtenemos: k4 +2k2 + l-k4 + 2k2-1 k4-l w= 4fc2 k4-l 29.- De: _ 2 tanx sen 2x = ------5— 1 + tan x Al despejar obtenemos: 2 1 + tan x tanx 2 sen2x tanx sen2x sec2x Luego en M, tendremos: sec2 a M= — sec2p 1 2 2., —-— = — (sec a - sec b) Pero por dato: 2 2 sec a - sec b = 2 W = 1 1 l! w = 2 Finalmente: M = 1 2 =^> 1 - tan x = 3 tan x 30.- Del dato: 2 - (1 + tan2x) = 3 tan x A continuación: . t 2 tanx 2 tanx tan 2x — , — _. 1 —tanx 3tanx . 4 Luego: , 2 1 - tan2 2x . 9. tan2x- o => cot4x- _ => cot4x- 3 ztanzx 21Z] l3j 5 Finalmente: cot 4x = => cot 4x = 12 .-. cot 4x = 3 31.- Efectuando como sigue obtenemos: 1 - -72 sen x-(l+-j2 senx) 2-72 cos r (l+5^senxXl-5^senx) + sen2x cos2jc -2-72 senx.sen2x +2-72 cosx. cos2x cos2x.sen2x Identidades Trigonométricas del Arco Doble
W = 2-j2 (cos 2x. cosx -sen 2x.senx) 2sen2x.cos2x W = 4j2. cos3x sen4x 71 71 Como: 3x + 4x = —; pues: 7x = — Por ángulos complementarios: cos — = sen — 32 .- Efectuando como sigue: M = 49 c0s22°.cos8o-sen8°.sen22o A sen80.cos80 I pcos(220+8°) Y 2sen8°.cos8° J = 49 \2 senl6° Luego de reemplazar los valores, obtenemos: M = 49 . . 252 = 3. (625) M = 1 875 33 .- Por teoría de ángulos suplementarios tendremos: 771 ( 571A 571 1171 ( 7t A 71 tan—= tan 7i----= -tan — => tan —— = tan 7i-= -tan —- 12 l 12 J 12 12 l 12 J 12 Por lo tanto: W = tan— - - tan— + tan — - - tan— 12 12 J 12 [ 12 Reemplazando los valores notables, tendremos: W = 2[2- 43 + 2 + 43 ] W = 8 => W = 2Ítan—+ tan—1 12 12 J 34 .- Se sabe que: 8 sen4a = 3-4 cos 2a + cos 4a Luego, multiplicando por 8 en la expresión dada tendremos: oiA7 o 4 71 , o 4 37t . 371 . 71 8 W = 8 sen + 8 sen + 4 cos g + 4 cos g- Y reemplazando la identidad inicial, obtenemos: O A 71 71 „ . 371 371 . 371 , 71 8W = 3-4 cos 77 + cos 7+3-4 cos-77 + cos ~¡r + 4 cos -77 + 4 cos 8 4 8 4 8 8 8W = 6+-cos^ +cos3/ 8W = 6+ W= 4 4 4 2 2 4 Problemas de Trigonometría y cómo resolverlos ^RACSt
35.- Como: 771 71 COS — = -COS — 8 8 571 371 COS— = -COS — 8 8 l-cos- 71 1 + eos — 8 , 371 1 + eos— 8 i 371 l-cos- 2 1 - eos2 — 8 2 371 l-cos - 2 71 = sen — 8 2 371 -sen g M = M = „ 371 71 371 71 71 Pero: sen g = cos g- pues: “g“ + g’ = 2* Buscando la fórmula del seno del doble, tendremos: 2 2 71 2 TE 1 M = sen — . cos — = — 8 8 4 71 Tt 2 sen— . eos — 1 2 71 1 M = — . sen — = — -j= 4 4 4 V2 \2 1 8 36.- Se sabe que: 1 + cos 40° = 2 cos220° ^2 eos2 20° Luego: M =---------^=--- . sec 20° . sec 20° M = 1 7 37.- Escribiendo como sigue el problema, tendremos: M = M = sen 5o. cos 5o - (1 - sen2 40°) 2 2M= 2 sen 5o. cos 5° - 2 cos 40' l+cos80° sen 10° Utilizando las fórmulas del doble, obtenemos: 2M = sen 10° - (1 +cos 80°) ' sen 10° 2M = sen 10° - 1- sen 10' M = - K31>—1 VARIACIÓN DE EXPRESIONES 38.- Factorizando tenemos: W = sen x . cos x(cos4x - sen4x) 2 2 2 2 W = sen x . cos x (cos x —sen x) (cos x+sen x) => cos2x 1 W = —. 2senxcosx . eos 2x sen2x 1 W = —. 2sen2x.cos2x 4 -------------' 1 W = — sen 4x 4 Identidades Trigonométricas del Arco Doble R11 431
. Comox g R => - 1 < sen 4x < 1 => -4 < — sen4x < 4 4 4_____ 4 W Finalmente el máximo valor será: ^máx = "4 39.- Debemos plantear la siguiente restricción: 1 - cos 2x + 0 => cos 2x * 1 => 2x * 2kn x + kn => De la C.T tenemos: También recordar: 2. 2 2 2 2sen 2x (2sen*cosx) 4sen xcos x 2 j = 2 2sen x sen x sen x y De la condición (1): 0 < cos x < 1 2 0 < 4cos x < 4 W = 4 cos2r W G [0 ;4) 40.- Debemos plantear las siguientes restricciones: i) tan 6 3 si6 (2k +1)-^,/cg Z ii) tan 20 3 si 26 * (2k +1) => Luego: Resolviendo: W = tan 6 - tan 26[1- tan20] RH 432 Problemas de Trigonometría y cómo resolverlos ‘’Ü RACSO ^^DlTOBBS
=> W = tan 6 - ---- 2 . [1 - tan20] => 1- tan2© De la C.T. tenemos: -1 < tan 6 < 1 W g (-1; 1) VV = - tan 6 IDENTIDADES AUXILIARES *1 ^<51 6 J a 41.- Recordemos: sen x + cos x = — + — cos 4x a sen x + cos x = — + — cos 4x 4 4 o o Reemplazando las identidades anteriores en el problema: 3 1 —+ —COS' —H” —+ —COS' 8 8 Luego de efectuar las operaciones anteriores, obtenemos: Pero: 1 /ti —eos —+ 2a 4 12 3 , —cos(n-a) 2 (-sen2a) 3 (-cosa) 2 2sena.cosa M= ------------- 3 cosa sen a Finalmente: 1 esc a = 4 => sen a = — 4 m-3 42.- Recordar: tan a + cot a = sec a. esc a = 2 esc 2a En el dato: 2n 2n tan — + cot — = k 9 9 (tan§ = cot^ 2csc — = k 9 4n M = 4 3'4 ' 2 Sen 9 “ k 4 3 Finalmente: 1 4 VV = -.-y 4 k2 1 k2 Identidades Trigonométricas del Arco Doble
2 2 43.- Como: tan x + cot x = m Buscando el desarrollo de un trinomio cuadrado perfecto sumamos (2) a ambos miem- bros de la igualdad: (tan x + cot x)2 = m + 2 (2 csc 2x)2 = m + 2 2 csc 2x = -Jm + 2 -Jm + 2 . sen 2x = 2 4 4 4 4 * 44.- Recordemos: cos x - sen x - cos 2x a cos x + sen x = — + — cos 4x 4 4 coS'»;—sen6 * 8x (eos4 x + sen4x).(cos4 x—sen4x) Como: ------------------------------------------- cos2x cos2x cos2x.|v+4-cos4r | \4 4 / 3 1 3 1 . . „ = cos2x =4 + 4cos4x => 4 + 4 cos 4x = A + B cos 4x Luego de comparar las proposiciones anteriores, obtenemos- 3 1 A= - ; B = — A + B= 1 4 4 45.- Recordemos que: cos4x + sen4x = y + -r cos 4x i 4 4 , . j. (eos4 x-sen4 x) En la condición: ----¿¿------------t-----¿— = m (cos x+sen x)(cos x-sen x) Efectuando y simplificando tendremos: 4 4 1 3 1 4 cos x + sen x = — . cos 4x =--- m 4 4 4m 4 4 Luego: 3 - — = cos 4x => 3 - cos 4x = — ° m m 46.- Recordemos: 6 6 3 3 . sen x + cos x = g + g cos Reemplazando en la identidad: = A + B cos 4x => 5+3 cos 4x = A + B cos 4x Al comparar tendremos: A = 5 aB = 3 A-B= 2 Problemas de Trigonometría y cómo resolverlos RACSO innoiri
SITUACIONES GRÁFICAS 47.- En k ADE: ED = AD = V2 En L ADB: BD= V2 cotx ..(1) En L EDC: tan a = ED DC A continuación calculamos «tan 2o», así: tan 2a = 2tana 1 — tan2 a 2# V3 hj2 J3 En kBDC: BD = V3 . tan 2a = = 6^2... (2) De(l) = (2): cotx = 6-^2 cot x = 6 b „ í a'l o b 48.- En el LEFD: tanl — I = — = — — (1) 2 tana= Identidades Trigonométricas del Arco Doble
49.- Sean los ángulos agudos a y 0. En el L ABC tenemos: sen20° 2senl0°cosl0° tan a + cos20° 2cos210° tan a = tan 10° => a = 10° Además: a + 0 = 90° => 0 = 80° 50.- De la figura tenemos que el cuadrilátero ABCD es inscriptible, BD es diámetro de lina circunferencia. tn Z BDC = tn Z BAC - 2a Por ser ángulos inscritos. En k BAD: BD = 3. esc a ... (1) EnLBCD: BD = 7 sec 2a ...(2) 3 7 7 De (1) y (2): =» 3[1 - 2 sen a] = 7 sen a ' ’’ v ’ sena cos2a 1 ' 2 6 sen a + 7sen a - 3 = 0 (2 sen a + 3)(3 sen a - 1) = 0 3 sen a = - — (absurdo) ó 1_ 3 sen a = Luego: esc a = 3 En (1): BD = 3(3) Finalmente: BD = 9 cm RACSO DITOlll R11 436 Problemas de Trigonometría y cómo resolverlos
RELACIONES FUNDAMENTALES 01.- Utilizando la fórmula del «cos» del ángulo mitad obtenemos: eos — 2 1 + cosx 2 X 4 cos y = 3 02.- Escribiendo «M» en términos del cos x, obtenemos. 1 cosx 1 cosx cosx cosx cosx COSX 1 -COSX 11+cosx IVl — 1 1 1 cosx 1 1 COSX -* Vl+cosx + Vi-COSX 1 cosx cosx 1 cosx cosx A continuación observamos que cada una de estas expresiones son tan^ y cot^. Además: 2 e IC, por lo tanto: M = tan ~ + cot ~ = 2 csc x M = 2 csc 1 03.- Escribiendo apropiadamente la expresión, tendremos: x O 2x cot—.cosx 2cos —.tanx ., 2 2 M = x ------------- secx.cot— 2 secx.cot— 2 cosx M= —y- „ 2 x senx 2cos —.----- 2 cosx cosx cos— 2 cosx „__ x sen— 2 A continuación, utilizando arco doble. 2 Y "V M = cos x + 2cos—sen^. sen x tendremos: M = cos2x + sen x . sen x 2 2 M = cos x + sen x M = 1 Identidades Trigonométricas del Arco Mitad R12 437
04.- En la condición: esc 2x - cot 2x = -g tan x Luego: 1 tan x = g ... (1) A continuación: W = esc 4x + cot 4x = cot 2x (fórmula racionalizada) Luego: 1 1 — tan2 x W =----— = —-------- tan2x 2 tanx 1- Reemplazando (1) en W obtendremos: W = — 2 1-1 ___9 2 3 4 w=- 05.- En LABC por el teorema de Pitágoras, tendremos: AB= ^(c2 + b2)2 -(2ob)2 AB = 7 o4 + 2o2b2 + b4 - 4o2b2 AB b2)2 = |«2-b2| tf-tí1 B Como a > b .2 1.2 c2-b2>0 AB = c2 - b2 Entonces: . x . a2+b2 tan 77 = esc x - cot x = —„ , 2 2«b a2-b2 2ab x 2b2 b tan^ = é—r = — z 2ab a . x b tan re = — 2 a 06 - Por identidades trigon métricas, sabemos que: 2 (*) (1 + sen x + cos x) = 2(1 + sen x)(l + eos x) 2 (*) (1 - sen x + cos x) = 2(1 - sen x)(l + eos x) 1 + cosx x 1 cosx % (*) cot ~ = CSC X + cot X = =J> cot — = CSC X + cot X = 2 senx 2 senx Luego trabajando con la condición reemplazando las identidades anteriores, y factorizando tendremos: 2(1 + cos x)[l + sen x + 1 - sen x] = W . 1 + cosx senx W(l+cosx) Finalmente: 4(1 + cos x) = --------- senx W = 4 sen x R12 4381 Problemas de Trigonometría y cómo resolverlos RACSO
07.- Según dato: 71 < x % 6 IIC Además: tan— 1 - cosx 1 + cosx 3 3 Pero: cotx-^ => cosx = -g (2) Además: Finalmente: Reemplazando (2) en (1): cos 2x = 2 cos2x -1=2 cos 2x = - 7 25 í 7 1 43 W=|--J.(.2)=- 43 w=- 08.- Se pide calcular: W = 2 sen | . cos | . tan x - 2 sen2 + 1 arco doble Por degradación: sen2^ = l~cosx W = sen x . tan x - 2 p—2°S* arco mitad A continuación escribimos «W» en términos de senos y cosenos: W = sen x. senx cosx 1-cosx 2 + 1 W = sen2x cosx -1 + cos X + 1 2 2-1 sen x + cos x 1 W =-------- =---= sec x cosx cosx 2J3 W = sec 30°= 3 3 Identidades Trigonométricas del Arco Mitad
FÓRMULAS RACIONALIZADAS 09.- Se sabe que: cot y = csc a + cot a tan = csc a - cot a luego, aplicando dichas propiedades en el problema, tendremos: W = csc 2x + 2[csc 2x + cot 2x] - 3[csc 2x - cot 2x] A continuación efectuando y simplificando obtenemos: W = csc 2x + 2 csc 2x + 2cot 2x - 3 csc 2x + 3 cot 2x Finalmente: W = 5 cot 2x 10.- Utilizando las fórmulas racionalizadas tenemos: (*) tan 20° = csc 40° - cot 40° (*) cot 20° = csc 40° + cot 40° T . , _ csc400-cot400+cot40° _ esc40° Luego: M - csc40o+cot40o-cot40° - csc40° f a'i 1 + cosa 11.- Recordar: cot — = csc a + cot a = ------; además: ^2) sena 2cos2a = 1 + cos 2a M = 1 Luego: W = 1 + cosx senx - (1 + cos x). cosx senx 2 ,,, 1 + cosx-cosx-cos X w =------------------ sen'x senx senx W = sen x 12.- M = tan^ + (1 - cos x). cot x => M = csc x - cot x + cot x - cos x. cot x A continuación luego de cancelar: cot x y -cot x, escribimos «M» en términos de sen x y cos x: 2 2 1 cos x l-cos X M =----- ------ =--------- senx senx senx , sen2x M =------ senx = sen x M = sen x 13.- Por arco mitad: Arco doble- fx^ 1-cosx tan — = csc x - cot x = ----- 2 j senx 2 sen 2x = 2 sen x cos x a cos 2r = 2 cos x - 1 Utilizando las propiedades anteriores tendremos: •JiRACSO *7 *D1TD>1B R12 440 Problemas de Trigonometría y cómo resolverlos
(xA ~ I. tan 2x. f 2 2 eos x-(l-cos x) cosx(l-cosx) w= senx 2 sen2x 2 cos x-1 cos2x ' cosx(l-cosx) , 2senxcosx W = ----------=2 senxcosx W = 2 14.- Utilizando las fórmulas racionalizadas de arco mitad, tendremos: 1, cscx —(cscx - cotx) ------------------ =* —(cscx - cotx) + cotx A continuación simplificamos y obtenemos: 1, —(cscx + cotx) M = -------------= 1 M = 1 —(cscx + cotx) cscx —-CSCX+—.cotx 2! 2 —.cscx--.cotx + cotx 15.- Escribimos apropiadamente la tangente de un ángulo mitad, obteniéndose que: 71 X tan[--- = tan lín 2l 2 = CSC (71 1 ---x = sec x - tan x = 2 J 1-senx cosx n 2 Finalmente al reemplazar dicho resultado en W, obtenemos: 1-senx (1+senx) cosx cosx 1 — sen2x 2». cos x 2 COS X => W=-------ñ-= 1 COS X W = 1 16.- Si: esc x + cot x = cot luego: esc x = cot^ - cot x cscx = cot^-cotx En «M», tendremos: csc~ csc^ 4 CSCg = cot*-cot* = cot^—cot 4 8 4 = cot16-cot8 (+) CSCÍ6 =CCt32'CCt16 Finalmente al simplificar obtenemos: M = cot~— - cotx 32 Identidades Trigonométricas del Arco Mitad
17 .- Sabemos que: tan x + cot x = 2 csc 2x, a continuación reemplazamos dicha propiedad en el problema obteniendo: M = cos22x - tanx + cot x ,2 2 2 M = cos 2x - sen 2x M = eos 4x 2 18 .- Utilizando las fórmulas racionalizadas como sigue: W = csc Zx + cot 4x + csc 4x del arco mitad W = csc Zx + cot 2x del arco mitad W = cot x 19 .- Utilizando las fórmulas racionalizadas en el numerador y denominador se obtiene: W _ (cscx + cotx) - (cscx - cotx) csc2x + cot2x arco de mitad 2 cotx W = ---— = 2 cotx W = 2 20.- Del arco mitad se sabe que: a tan — = csc a - cot a Luego: tan7°30' = csc 15° - cot 15* Con ayuda de las R.T. de 15°, tendremos: tan 7°30' = (Vó + ^2 )- (2 + 73 ) tan7° 30' = V6 - 2 -2 21.- Sabemos que: [al tanl — I = csc a - cot a, luego: (18o>l tan(9°) = tanl I = csc 18° - cot 18° => tan(9°) = 4__ *5-1 J10 + 2V5 J5-1 Racionalizando tenemos: . 4 (TS’+l) Ó0 + 2V5(VV5 + l)2 an ‘ (75-1) ’ (^5 +1) (75-l)(75 + l) Finalmente: tan(9°) - 75 +1 - 75+275 R12 442 Problemas de Trigonometría y cómo resolverlos RACSO
PROBLEMAS CONDICIONALES 22.- Se sabe que: cot x + tan x = 2 csc 2x (1) x tan — = csc x - cot x (2) Utilizando la primera propiedad en el segundo miembro de la condición obteniendo: x -J x + tan — =tant + cot- es " 4 A continuación uti izamos la propiedad (2) en el primer miembro. x x x x x csc ~ - cot— = cot~ ; csc— =2cot ~ 4 4 4 4 4 A continuación escribiendo en términos de senos y cosenos: 1 4 x „ —x=—T' sen7*° sen— sen— 4 4 x 1 x => cos ~ — — => sec ~ = 2 4 2 4 1 Luego: M= — + 2 = 2,5 M = 2,5 23 .- Utilizando las fórmulas adecuadas del ángulo mitad tendremos: csc 20°+cot 20o- (csc 20°-cot 20°) W=---------------COÍ205------------- A continuación simplificando obtendremos: W = W — 2 x 24 .- Recordemos: cot ~ = csc x + cot x Escribiendo como sigue la expresión «E» se tendrá que E = csc(90° - 2x) + cot(90° - 2x) => E = cot 90°-2x 2 = cot(45° - x) E = 1 =1 tan(45 -x) m m Identidades Trigonométricas del Arco Mitad R12
X X 25 .- Como: esc x - cot x = sen 0 => tan ~ = sen() a cot~ =csc0 Utilizando las identidades trigonométricas apropiadas y las igualdades anteriores, obtenemos: 2 X 2 _____i____ 1 + tan —+ cos 0 , 2n 2„ M=--------M=l±sen@±cos 0 =2 . M = 2 CSC0-CSC0 + 1 1 26 .- Como: cot x - tan x = 2 cot 2x, en el problema se tendrá: M = (tan 10° + cot 10° - tan 10°)(csc 20° - cot 20°) Finalmente: M = cot 10°. tan 10° =1 .-. M = 1 27 .- Como: sen x + m cos x = m => sen x - tn(l - cos x) 1 1 cosx 1 x 1 => — =------- ------- => esc x - cot x = — => tan ~ = — m senx senx tn 2 m Finalmente: cot y = tn 28 .- Completamos cuadrados como sigue, tendremos: x2 - (2 esc a)x + csc2x + 1 - csc2a = 0 2 2 2 2 (x - csc a) = esc a - 1 => (x - esc a) = cot a .-. x - csc a = cot a o x - csc a = - cot a => x = csc a + cot a = cot 2 o r = csc a - cot a = tan 29 .- El lado izquierdo de la igualdad, expresando en términos senos y cosenos. cosóx cos(8x-2x) cos8xcos2x+sen8x.sen2x sen8x.cos2x sen8x.cos2x sen8x.cos2x = cot 8x + tan 2x = cot 8x + csc 4x - cot 4x del arco mitad del arco mitad = cot 8x + csc 4x - (csc 8x + cot 8x) = csc 4x - csc 8x RACSO P ID1TO1II R12 444 Problemas de Trigonometría y cómo resolverlos
Comparando: A=4,B = 8 Finalmente: B — =2 30 .- Escribiendo la expresión pedida como sigue: 2 2 l 2 J (n-a\ . —4~I tan Y ahora, usando la siguiente fórmula, tendremos : na 2 2 2 / n Ct x . / 7t ct\ = csc( — " 2 )’cotí 2 ' 2> tan Y reduciendo al 1C nos queda: = sec - tan tan = sec - tan 31 .- Dándole una forma más apropiada a la expresión "p", tendremos: I 1 i i 7 P = 2 I--=—+------------ => p=-------(sen0>0) tisen2 g senO senO r senO ' ' => p = 2 csc 0 6 1 0 Asimismo, del dato deducimos que : k = cot — v v = tan 77 2_ K z A continuación, sumamos las siguientes propiedades y obtendremos: i 6 i 6 o tan + cot 2 = 2 csc 0 0 0 Finalmente tendremos : p = 2 csc 0 = tan + cot .-. P=l+k => p = k + k1 Identidades Trigonométricas del Arco Mitad
SITUACIONES GRÁFICAS 32 .- Sea CD = a => AB = 2a En LDCB: CB=fltan6 CA En L ACD: cot 6 = fl(tan0+2) cot0= —---------- a Efectuando: cot0-tan0 =2 Por arco mitad: 2 cot 20 = 2 => 33 .- Dado que x es agudo y 4x también lo es, podemos usar un método gráfico que a continuación presentamos: 6 3 Se sabe que: sen 4 x = 0,6 = => sen 4 x = Utilizando el "Método Griego" prolongamos el cateto adyacente al ángulo "4x" una longitud igual al valor de la hipotenusa. A continuación unimos su extremo con el extremo de la hipotenusa, logrando un triángulo isósceles tal como ACD => Z ADC = 2 x Luego: tan 2x = (4 + 5) => tan2x= 3 Haciendo el mismo procedimiento, y en base a este último resultado construimos el triángulo adjunto, de donde: tan x = 1 VÍO+3 VÍO-3 ’ V10 + 3 tanx= -TÍO +3 Problemas de Trigonometría y cómo resolverlos RACSO JIDITOIII
01.- Utilizando las relaciones fundamentales del seno y coseno del triple, obtenemos: 3 3 3 3 eos x-(4cos x-3cosx) sen x + 3senx-4sen x W =-------------------------1---------------------- cosx senx A continuación al simplificar obtenemos: 3cosx(l-cos2x) 3senx(l-sen2x) cosx senx W = 3[sen2x + cos2x] .-. W = 3 02.- Recordar que: cos(a + 0). cos(a - 0) = cos2a - sen20 2 2 Luego: W = 4 cos x [ cos 60° - sen x] => W = 4 cos x Escribiendo como sigue, tendremos: W = 4 cos x [^-(1 ~cos2xj => W = 4 cos x^cos2x-^ W = 4 eos' x - 3 cos x .'. W = cos 3x (i)'-sen» 03.- Utilizando las fórmulas fundamentales del triple, tenemos: sen3x ----- 3 3 <-pnr 3cosx + (4cos x-3cosx) 4sen3x 4cos x W = „ sen +-----------------------=> W =--------------------ñ— +-------5- —-sen2x 3senx-(3senx-4sen x) senx(3-4sen x) 4sen x 4 4sen3x 3 Finalmente: W =-------~— + cot x sen3x W = 4 + cot3x Identidades Trigonométricas del Arco Triple R13 447
04.- Escribiendo en términos de senos y cosenos : M = tan x. senx (l-4cos2x) " 9 — cosx’ (3-4cos x) senx( 1 - 4( 1 - sen2x)) — -(4cos x-3cosx) Enseguida damos forma al numerador y denominador formando las fórmulas básicas del seno y coseno del triple: . . senx(4sen2x-3) ~(3senx-4sen3x) _ sen3x -cos3x -cos3x cos3x M = tan 3x 05.- Escribiendo como sigue, tendremos: cos3x cos2x 8 tanx cos3x cos2x tan2x A continuación utilizamos las fórmulas del doble y el triple: 4cos3x-3cosx 2cos2x-1 8tanx Vy — -------------- -f- ---ñ---- — -------- eos' x cos x 2 tanx 1 — tan2x 2 2 2 Finalmente obtenemos: W = 4 - 3 sec x + 2 - sec x - 4(1 - tan r) 7 7 W = 2-4(sec x—tan x) W = -2 i 06. Escribiendo en términos de senos y cosenos: 1 cos2x _____cosx cosx senx „ sen3x -----+ 2senx+ cosx-cosx cosx + l+cos2x w =---------------------- senx + 2senx cos x + sen3x A continuación efectuando operaciones apropiadas en el denominador y factorizando, tendremos: __________cosx + l+cos2x_______ ” sen x+2 sen x cos x+sen x( 2 cos 2x +1) (cosx +1 + cos2x) 2senx(l + cosx + cos2x) W = Finalmente: 1 VV = — csc x Problemas de Trigonometría y cómo resolverlos jÍ RACSO Uf* dDITOlli
5 4 V5 07.- Como: sen x + cos x =- 2 (sen x + cos x)2 2 2 „ 3 sen x+cos x + 2senxcosx = — '-----'------' '------' 4 1 sen2x 1 sen 2x = — 4 3 Luego: sen 6x = sen 3(Zx) = 3 sen Zx - 4 sen' Zx Finalmente: 16 sen 6x = 11 M = 11 08.- Del dato: 2sen2x _ 2cos2x - 4 => tan2x = 4 => tan x = 2, x g IC Luego: tan 3x = 3 3tanx-tan x _ l-3tan2x 3(2)-(2)3 tan 3x — 9 l-3(2)2 => tan 3x = -2 -11 2 tan 3x = -jy 09.- Utilizando la fórmula del coseno doble, tendremos: i J W = 4(1-2 sen29°)- — => W = 4 . cos 18°---------- cos 18° A continuación efectuamos para generar el desarrollo del coseno del triple, así: 4cos218 -3 cosl8 W =-----------.------- cos 18 cos18° 4 eos318°-3cosl8° cos3(18°) eos218° “ eos218° cos54° sen36° 2senl8°.cosl8° Finalmente: W = lf., = W = 2 tan 18° 10.- Trabajando con la ecuación del problema: 2(3 tan x - tan3x) = 1-3 tan2x => 3 tanx-tan3 r •y 1—3tan x Por arco triple: 1 tan 3x = ~ 1 2 Identidades Trigonométricas del Arco Triple
Luego: 3tan3x W = tan 6x = tan 2(3x) = ------ 1-tan 3x Finalmente: W=- 4 fíELAClONES AUXILIARES 11.- Utilizando la siguiente identidad, tendremos: tan a + cot a = sec a. csc a - 2 csc 2a Luego: W = tan 9o + tan 27° + cot 27° + cot 9o En el problema: W = tan9°+cot9° + tan27°+cot27° => W = 2 csc 18° + 2 csc 54° '-----V " v------v ' A continuación recordamos los siguientes triángulos notables: Finalmente utilizando los triángulos notables en el problema obtendremos: W = 4 12.- Si recordamos que: tan a + tan P = sen(K+P) cosa-cosP „ sen(a + P) cot a + cot P =-------- sena.senP En el problema: sen(72°+36°) cos72°.cos36° _ sen72°.sen36° sen(72°+36°) cos72°.cos36° sen72°.sen36° Problemas de Trigonometría y cómo resolverlos RACSO DITOKBM
Luego de los triángulos mostrados: W = tan 72° . tan 36° J10 + 2J5 Jl0-2j5 V5-1 V5 + 1 Finalmente: 7100- 20 4^5 w =--------=----- 4 4 W= V5 13 .- Aplicando reducción al I cuadrante tendremos: cos 380° = cos(360° + 20°) = +cos 20° => cos 140° = cos(180° - 40°) = -cos 40° cos 260° = cos(180° + 80°) = -cos 80° => W = (cos 20°)(-cos 40°)(-cos 80°) Pero se sabe que: cos 3a = 4 cos a. cos(60° + a).cos(60° - a) Luego en el problema, tendremos: W = cos 20°.cos(60° + 20°).cos(60° - 20°) Finalmente: 1 11 W = - cos 3. (20°) = ~ cos 60° = - 4 4 8 2 14 .- Escribiendo como sigue el problema: M = 3(sec 10° . sec 50° . sec 70°) A continuación le damos una forma conveniente a la siguiente propiedad: M = 3[ sec 10°. sec(60° - 10°). sec(60° + 10o)]2 M= 3.16. . sec 10°. sec (60°-10°). sec(60°+10) Pero: 1 ~ sec x . sec(60° - x). sec (60° + x) = sec 3x Finalmente: M = 3.16 . sec230° =3.16. 15 .- Sabemos que: 4 sen x. sen (60° - x). sen (60° + x) = sen 3x a cos4x - sen4x = cos 2x Aplicando dichas propiedades al problema tendremos: 4sen22°sen(60o-22o).sen(60o+22°) sen3(22°) sen66° 4M = o/i4M - — 0,11 cos 2(12 ) cos24 cos 24 w= r . 4 2 f .-. M = 64 Identidades Trigonométricas del Arco Triple R13 451
Como: sen 66° = cos 24° , de donde: 4M = 1 M = — 4 16 .- Escribiendo como sigue la expresión, tendremos: (2cos6x+ l)sen3x (2 cos 6x -1) cos 3x sen3(3x) W ~ cos3(3x) sen9x ~ cos9x W = tan 9x . x „ 3x 4 senx. sen3cos-~- 17 .- Escribiendo como sigue: W =- sen~- sen2-- Luego de efectuar y agrupar conveniente tendremos: 7 X X X Y I 2 Y 4.2sen 7rcos-^ + 3cos-y(2cosx-l) coswl8sen —+6cosx-3 W =------z----2—5—2----------- => W =--------—------~--------- cos£(l + 2cosx) Finalmente: W = ----------------- sen^(2cosx + l) sen^=- W = cot y sen3x cos3x 18 .- Aplicando las propiedades: - = 2 cos 2x + 1 a --------- = 2 cos 2x - 1 r r r senx COSX En el problema tendremos: M + 2 cos 2r + 1 = 2 cos 2x - 1 Luego de simplificar, obtendremos: M = -2 19 .- Escribiendo en términos de senos y cosenos: senlO° 2sen210° cos70° W = ----— ( 3 cos 10° - 2 sen 10°.cos 70°) => W = 3 sen 10° - —-------- - coslO ' coslO Pero, por arco complementario: cos 70° = sen 20° 2sen210° En la expresión tenemos: W = 3 sen 10° - ----. 2 sen 10° cos 10° Ordenando y simplificando: W = 3 sen 10° - 4 sen310° 1 1 Entonces por arco triple tenemos: W = sen 3(10°) = ~ .-. W = ~ R13 452 I Problemas de Trigonometría y cómo resolverlos 1RACSO dITOIEI
PROBLEMAS CONDICIONALES 20.-Del lado derecho de la igualdad tenemos: 3 cosx 3 senx (3sen x - 4sen x).--- + (4cos x -3cosx).--- senx cosx 2 2 = 3cos x - 4sen x cos x + 4cos x sen x - 3sen x => 3(cos x - sen x) + 4 sen x cos x(cos x - sen x) => (cos x - sen x)[3 + 2(2 sen x cos x)] Multiplicando el numerador y denominador por. cos x + sen x (cosx-senx)( cosx + senx)[3 + 2.sen2x] (eos2 x-sen2x)[3 + 2sen2x] cosx + senx senx + cos x cos2x[3 + 2sen2x] 2sen2xcos2x + 3cos2x sen4x + 3cos2x senx + cosx senx + cosx senx + cosx Luego, al comparar tendremos: A = 1 ; B = 3 .’. A + B = 4 21.- De la condición: sen3x senx 1 sec zá - senx senx => _ — 2 COS ¿X + 1 - 1 cos2x => 1=2 cos22x 0 = 2cos22x -1 Pero: 2cos 2x-I =0 cos4x cos 4x = 0 22.- Utilizando una fórmula especial del triple, tendremos: sen3x 1 Por arco triple: ------ = 2 cos 2x + 1 = ~ r senx 3 1 Luego: cos 2x = - — 2 í if 7 Finalmente por arco doble: W = cos 4x = 2 cos 2x -1 = 2-----1 W = -~ 23.-Como: «cscx = 3 - 4 sen2x => a = 3 sen x - 4 sen3x => a = sen 3x ... (1) 2 3 Como: b sec x = 4 cos x - 3 => b - 4 cos x - 3 cos x => b = cos 3x ... (2) 2 2 2 2 Sumando los cuadrados de las relaciones (1) y (2), obtendremos: a + b = sen 3x + cos 3x Finalmente: 2 ,2 , a + b = 1 Identidades Trigonométricas del Arco Triple
24.- En la condición del problema, desarrollamos las fórmulas del seno y coseno del triple de un ángulo, así: sen3x + sen x 1 3 3 3senx—4sen x+sen x 1 q — cos3x —eos x 2 4cos3 x - 3cosx - eos3 x “ 2 2 3senx(l-sen x) 1 senx eos2 x 1 2 -3cosx(l-cos x) 2 cosx ’ sen2x 2 1 Al sin'i lificar obtenemos: cotx = tan x = -2 2 25.- Hallemos la relación angular: 3x = 180° - 3(60° - x) => sen 3x = sen[180° - 3(60° - x)] Luego: sen 3x = sen 3[60° - x] => sen 3x = 3 sen(60° - x) - 4 sen3(60°- x) Reemplazando el dato: sen3x = 3^~= 1 - ~ ~ W = -cos 6x = -cos2(3x) = -[1 - 2 sen23x] Al reemplazar el valor de «sen 3x», tendremos: W = -1+2(i)2 ""S 26.- Si se sabe que: 2 1 - cos 2x = 2 sen x Luego: „ 2 9a 2sen — _____2_ „ 2 3a 2sen — 2 = (x3 - 3x + l)2 => 3a sen-^-.(2cos3a +1) 3a sen— 2 = (X3-3x + 1)2 A continuación comparando podemos lograr la siguiente igualdad que al darle forma quedará: 2 cos 3a = x3 - 3x => 2(4cos3a - 3 cos a) = x3 - 3x (2 cos a)3 - 3(2 cos a) = X3 - 3x x = 2 cos a 27.- Escribiendo en términos de senos y cosenos, tendremos: sen3x cos3x 2 + 2 1 cos x 1 sen x cosx cosx senx senx Problemas de Trigonometría y cómo resolverlos RACSO DITOkM
sen 3x. cosx , cos3x.senx _ 2 * 2 — sen x eos x sen3x.cosx cos3x.senx ----------------------- = ni cosx. cosx senx.senx A continuación utilizamos las fórmulas especiales del triple: (2 cos 2x + l)cot x + (2 cos 2x - 1) tan x = m Al agrupar convenientemente encontramos algunas fórmulas especiales del doble, así: 2 cos 2x . (cot x + tan x) + (cot x - tan x) = m 2csc2x 2cot2v Luego de efectuar tendremos: 4 cos 2x. —+ 2 cot 2x - m sen2x 4 cot 2x + 2 cot 2r = m Finalmente: 6 cot 2x = m ni cot2x = — b 28.- Utilizando las fórmulas básicas del seno y coseno del triple y operando apropiada- mente, tendremos: 3 3 3 3 W = (4 cos x - 3 cos x) - ( 3 sen x - 4 sen x) => W = 4(cos’ x + sen x) - 3(cos x + sen x) 3 3 2 2 Recordemos el producto notable: a' +b = (a + b)(a -ab + b ) Luego aplicando en W: 2 2 W = 4(cos x + sen x)(cos x - sen x cos x + sen x) - 3(cos x +- sen x) i W = (cos x + sen x)[4(l - sen x cos x) - 3] => W = (cos x + sen r)[l - 4 sen x cos x]... (1) Del dato, elevando al cuadrado tenemos: 2 2 2 2 2 (sen x + cos x) = k => sen x + 2 sen x cos x + cos x = k 2 2 2 sen x cos x = k - 1 => 4 sen x cos x = 2(k - 1) ... (2) A continuación (2) en (1): W = /c[l - 2(k2 - 1)] .’. W = 3fc-2fc3 29.- Escribiendo apropiadamente la condición tendremos: I— 2 - V5 .tan <J> = 1 - tan <[>=> 2 tan ó 1 - tan2 Ó ’ 2 =, J5 5 Identidades Trigonométricas del Arco Triple
Luego utilizamos la fórmula de la tangente del triple: 3 tan 20 - tan3 20 tan 60 = tan 3(2<j>) = ~~ 2ZT í— 3 tan 20 Finalmente, si reemplazamos el valor de tan 20, obtendremos: 6^5 8^5 -----+----- •i 25 tan 60=--------t-- 1-3- 5 7 5 tan 60 = 22^5 35 30.- Trabajando con la condición del problema: tan 3x = k . tan x tan3x tanx 2cos2x + l 2cos2x-l => 2 cos 2x + 1 = k. 2 cos 2x - k Luego: Se pide: 1 + Jt 2 cos 2x = ——- - - - (1) W = sen3x senx = 2 cos 2x + 1 ---(2) Reemplazando (1) en (2), obtendremos: W = ~—- +1 .-. W = 31.- Haciendo: x = 3a, reemplazamos en el dato: tan 3a = tan a(2 + J3 ) Luego: Como: tan 3a „ ------ =2+ v3 tana 2cos2a+l 2cos2a-l = 2+ J3 a c a+b c+d = — <=> —~ = —~ b d a-b c-d 4cos2a Aplicando dicha propiedad al problema: ---- 3 + J3 1 + V3 2cos2a=''3T7^' De donde: cos 2a = — 2 => 2a = 30° => a =15° x = 3a = 45° Finalmente: W = sen(75°) = W = V6+V2 4 Problemas de Trigonometría y cómo resolverlos 'i RACSO dITOUI
32.- Escribiendo como sigue, tenemos: 9 3 sen 2x = 2(1 - cos 2_r) => 3(2 sen x cos x) = 2(2 sen x) 71 3 cos x = 2 sen x ; como 0 < x < — 3 tanx= — A continuación la fórmula del triple: sen x * 0 tan 3x = 1-3|- 12 9 27 _ 2 8 = 1-^ 4 9 c Finalmente: tan 3x = —23 r o 9 tan3x = -4g 4 33.- Hallemos la relación angular: 3x = 3(x + 15°) - 45° Luego: tan 3x = tan[3(x + 15°) - 45"] tan 3x _ tan3(x + 15°)-tan45° anX 1 + tan3(.r+15").tan45' (1) 3tan(x + 15")-tan3(x+15°) Pero: tan 3(x+15 ) = 1 —3tan2(x+15") - - (2) Reemplazando el dato: 2 tan(x + 15") = ~ 2 3 En (2) tenemos: tan 3(.r + 15") = 1-3| 2? 3 ) x2 2-A __27 1-1 3 tan 3(x + 15") = - 46 Reemplazando (3) en (1), tenemos: -^-1 9 1+Í--V) 9 r Finalmente: tan 3x = 55 _____9_ “ 37 9 . 1 55 tan 3.r = gy Identidades Trigonométricas del Arco Triple R13 457
34.- Escribiendo el problema de la siguiente manera: ^1 - 6senl0° Jl-6senl0' W = ------------ = 51 cos 80° eos3 80' pero: cos 80° = sen 10' J4(l-6senl0°) V 4sen310° pero: 3 4 sen x = 3 sen x - sen 3x W = Luego: W = J - 4(6senlO°-I) V 3senl0°-sen30° Finalmente: ~4(6sen 10°-l) W~ J 6senl0°—1 1 2 W = ^8 =-2 W = -2 35.- Se sabe: sen3a ----- = 2 cos 2a + 1 sena cos3a -----= 2 cos 2a - 1 cosa Por arcos suplementarios: cos 160° = -cos 20° Por arcos complementarios: sen 20° = cos 70' Aplicando estas propiedades a nuestro problema, tendremos: W = 2(-cos 20°)(2 cos 70° - 1)(2 cos 70° + 1) En las propiedades utilizadas en el problema observamos que: 2a = 70° => a = 35' cos3(35°) sen3(35°) W = - 2 cos20°. ----2. --------*---' cos35 sen35° Luego «W» tendrá la siguiente forma: W = —2cos20°.cos 105°.senl05o ------------------------ 2 2sen35°.cos35° 2cos20° Simplificando: W = -------sen 210' r sen70 pero sen 210° = -sen 30° Finalmente: 1 sen 210° = W = 1 36.- Utilizando ángulos complementarios en el numerador y la propiedad sen2x - sen2y = sen(x + y) sen(x -y), en el denominador obtenemos: (3sen25°-4sen325°)sen50o M= sen(70°+20°).sen(70°—20°) R13 458 Problemas de Trigonometría y cómo resolverlos RACSO
Al efectuar en el numerador y denominador: De donde: M = sen 75° = 4 sen3(25°) K4 = ------------- * sen90°.sen50° " M=—4— 37.- Escribiendo «M» en términos de senos y cosenos: 4 eos218' 3 *, cos 18° cos 18° M~ señÍ8" cos 18° M = 2(4cqs318o-3cos18°) 2senl8°.cosl8° Por identidades del doble y triple tenemos: M = 2 cos 3(18°) sen2(18°) 2cos54° M sen36° Pero: sen 36° = cos 54° , luego: M = 2 sen 36° sen 36° M = 2 38.- Como: cos 3(20°) = 4 cos320° - 3 cos 20° Efectuando como sigue: cosóCT + 3 cos 20° = 4 cos320° ... (1) 1/2 1 + 6 cos 20° = 8 cos320° ^8cos320° Reemplazando en «M», tendremos: M = —----- Z COS _ 2cos20° - 2cos20° M = 1 39.-De(l): tan3x tanx a+b a-b 2cos2x + l Por propiedad del arco triple tendremos: ~—7 ZCOSZX 1 a+b ~ a-b a cos2a=if 2 De (2), multiplicando por 2: a 2sen x = 2b cos 3(2x) 3 2 Del arco doble y arco triple tenemos: c(l - cos 2x) = 2b[4 cos 2x - 3cos 2x] . .. (4) Despejando cos 2x: ... (3) Reemplazando (3) en (4): a a Í2b-a\ _ ó* l 2b I - -3a Identidades Trigonométricas del Arco Triple R13 459
Efectuando y simplificando tendremos: 8b2 -2a2 = ab 2b2 - ab = 2a2 - 6b2 SITUACIONES GRÁFICAS 40.- Haciendo: DE - a y EC = b, luego: EnelLBEC: BE = b.tan53° ...(1) EnelkAEB: AE = BE.tan67° ...(2) (1) en (2): AE = b tan 53°.tan 67°... (3) En LAED ED = AE.tan 7° ... (4) (3) en (4): a = b tan 53°. tan 67°. tan 7o Despejando tendremos: Finalmente: t = tan 3(7°) tan 7°.tan(60° + 7°).tan(60° - 7o) a - = tan 21° b 41.- Sea: AD = a En LADE: ED = a tan 27° En LEDC: DC = a tan 27°.tan(57° + x) En LBDC: m Z BCD = 33° BD = a tan 27°.tan(57° + x).tan 33° , En k ADB: BD = a tan 81° Se cumple: a tan 81° = a tan 27°.tan(57° + x).tan 33° A continuación api camos la siguiente propiedad: tan 30 = tan 0 . tan(60° + 0). tan(60° - 0) Si: 0 = 27° => 57° + x = 60° + 0 => 57° + x = 60° + 27° r = 30° R13 460 Problemas de Trigonometría y cómo resolverlos RACSO tCITOtli
42.- Sea: tn Z BAC = tn Z CAD = m Z DAB = a Sea: AB = x, DE = y 3 En L ABC: tan a = — ... (1) x 1 En LABD: tan 2a = — x 2tana 7 F^“7 -(2) En LABE: Reemplazando (1) en (2): Resolviendo la ecuación: tan 3a = Reemplazando (1) en (3): Pero tan a 7_ x 3 tana-tan3 a V + 7 l-3tan2a ~ 3^7 '' 3______1_ 3j7 " J7 Resolviendo la ecuación: y = 8 u c. - . . a4EC (AB)(BE) (3V7)(15) Finalmente: area A ABE =------------- =----------- .-. área A ABE = 59,53 u2 Identidades Trigonométricas del Arco Triple
TRANSFORMACIONES A PRODUCTO 01.- Escribiendo como sigue: Transformando a producto: cos15* + cos 5* + lOcoslO* senl5* + sen5x + lOsenlO* 2cosl0x.cos5x + lOcoslO* 2senl0*.sen5x + lOsenlO* 2cosl0x(cos5x + 5) Finalmente: M = —----——~--------~ 2senl0x(cos5x + 5) M = cot 10 x —2sen45°.sen* 02.- Transformando a producto: M = ~----ttt----- r 2cosl20 .sen* Luego de simplificar y reemplazar los valores numéricos correspondientes tendremos: -sen45° cos120° ¿2 -2- M = JÍ 2 03.-Escribiendo como sigue tendremos: ^+V3.sen2* M =--------------------------- (cos 2*+cos 60°). tan(x+30°) A continuación trabajando en términos de senos y cosenos: V3(j3/2 + sen2x) 2cos(x + 30°).cos(x - 30°).Sen;X + ^o-; ' ' ' ' cos(x + 30 ) [-Í ^3[“2“+Sen2x ) V3(2sen(30o+%)cos(30°-x)) M= 2cos(x-30")sen(x+30") M= 2cos(x-30")sen(30<1+x) Finalmente simplificamos: .'. M = -J3 RACSO ICITOKEi R14 462 Problemas de Trigonometría y cómo resolverlos %
2 3 04.- Se sabe que: 2 cos a = 1 + cos 2a ; 4 cos a = 3 cos a + cos 3a í a+P'l ( a-P'l cos a + cos P = 2 cosí —— Icosl —~— I Utilizando dichas propiedades en el problema: W = cosl0x + cos8x + 3(2cos 2x) + 3 cos2x - 3 - 2 cos x(4co< 3x) a producto degradar degradar (doble) (triple) W = 2 cos 9x cos x + 3(1 + cos 4x)+ X'cqs2x - 3 - 2 cos x(3 cos 3x + cos 9x) W = ¿cos-9x'Cbs’"x + X + 3 cos 4x + 3 cos 2^ - X - 6 cos x cos 3x - _2£OS-9xt®s-x Agrupando convenientemente, tendremos: \ W = 3(cos 4x + cos 2x) - f> cos x . cos 3x => VV A 3(2 cos 3x cos x) - 6 cos x . cos 3x Finalmente: W = 3(2 cos 3x cos x) - 6 cos x cos 3x \\ = 0 05.- Transformando a producto tanto el numerador como el denominador obtenemos. Z2sen(x + y). cos(x—y) 2 2cos (x-y) + 2cos(x + y).cos(x-y) A continuación simplificamos y luego de factorizar tendremos: M= 2cos(x-y).sen(x + y) cos(x - y)[cos(x - y)+cos(x+y)] Finalmente: 2sen(x + y) M = ---------- 2cosx.cosy tan x + tan y M = tan x + tan y 06.- Escribiendo como sigue: (—4cos2 x)(2senx cos2x) \cos2xsenx ' 2 2 (2sen x + sen2x-l) Simplificando logramos: 2 cosx-4cos xcos2xsenx ______£0g2x .sefíx____________________ (sen2x - cos2x)2 W = W = VV = 2 cos x Transformaciones de Sumas o Diferencias a Productos R14 463
07.- Agrupando como sigue: M = sen x + sen 7x + 2(sen 3x + sen 5x) => M = 2 sen 4x cos 3x + 4 sen 4x. cos x A continuación factorizamos: => M = 2 sen 4x(cos 3x + 2 cos x) => M = 2 sen 4x(cos 3x + cos x + cos x) => M = 2 sen 4x(2 cos 2x . cos x + cos x) => M = 2 sen 4x . cos x(2 cos 2x + 1) sen3x Pero: 2 cos 2x + 1 = ----- senx Aplicando la propiedad a M, tendremos: sen3x M = 2 sen 4x . cos x . --- M = 2 sen 4x cot x. sen 3x senx 08.- Transformando a producto tanto el numerador como el denominador: cos 12°+sen30°+sen6° cos 24°+cos 48° Al simplificar factores comunes, tendremos: cosl20+2sen(180)cos(12°) 2cos(36°)cos(12°) l+2senl8° 2sen54° w_ 2cos72°+l - 2cos36° Utilizando ángulo triple en el numerador y doble en el denominador tendremos: /2cos72°+l\ sen36° 1 2cos36° / sen36° senl08‘ W= "senTT W = 1 2 2 09.- Sabemos que: 2 sen a = 1 - cos 2a a 2 cos a = 1 + cos 2a 2 sen a cos P = sen(a + P) + sen(a - P) Utilizando dichas propiedades en el problema, tendremos: 2cos210°+2sen220°-2sen20°cosl0° W = ------------------------------- 2 _ 1 + cos 20° +1 - cos40° -2 sen 20°+cos 10° w - 2 2 w _ 2 + 2sen30°senl0°-2.2senl0°cos 10° Problemas de Trigonometría y cómo resolverlos ~JtRAf?SO
7 2 + senl0°(l —4cos 10°) W=---------------------------- Efectuando y simplificando, obtendremos: 2 2 + senl0°(4sen 10°-3) W = -----------ñ---------- Finalmente: 2-sen 30° ,,, 2+(4sen310°-3senl0°) => W =------1------------ 2-— 2 3 w=^ w=| 10 .- Utilizando la propiedad: tan x + cot x = 2 csc 2x en el denominador y a continuación expresando «W» en términos de senos y cosenos, se tendrá: 2cos40o—1 1 W = 2 csc 70° = Í cos 20°(2 cos 40° ’ !) Al efectuar en el numerador obtenemos: 1 1 W = - . cos 60° /. VV = - 2 4 11 .- Transformando a producto en el numerador obtenemos: sen4x + senóx 2sen5x.cosx sen2x ~ 2senx.cosx Al simplificar y comparar logramos: senx = senx .’. m = 5 12 .-Transformando_a^roducto como sigue: M = cos 20° + 2 cos 120°. cos 20° => — M'=Tós 20"+ 2(-l/2).cos 20° => M = cos 20° - cos 20° .-. M = 0 En general se cumple: cos x + cos (120° - x) + cos(120° + x) = 0 13 .- Al elegir convenientemente los extremos, transformamos a producto tanto el nume- rador como el denominador: sen5x+sen6x+sen7x _ 2sen6x.cosx+sen6x senx+sen 2x+sen 3x ~ 2sen2x.cosx+sen2x a .- ., , senóx (2cosx + l) A continuación factonzamos: c „ -7=-------4 sen2x (2cosx + l) . senóx sen2x(2cos4x+l) Luego: ----------------------- = 2 cos 4x + 1 & sen2x sen2x Al simplificar obtenemos: M cos 4x + 1 = 2 cos 4x + 1 M = 2 Transformaciones de Sumas o Diferencias a Productos Ü-J465
14.-Transformamos a producto como sigue: sena + sen9a + sen5a M =---------------— cosa+cos 9a + cos 5a 2sen5a.cos 4a+sen5a M = —----------------— 2 cos 5a. cos 4a + cos 5a A continuación factorizamos y simplificamos obteniendo: sen5a(2cos4a +1) cos 5a(2 cos 4a +1) Luego: M = tan 5a Finalmente: M = tan 45° Pero: M = 1 a = 10g = 9° PROBLEMAS CONDICIONALES 2sen8a.cosl5a 15.-Transformando a producto, tenemos W = —-------~— r 2sen8a.cos6a Utilizando la propiedad anterior en el problema: cos« + senb + serw + cosb cos a+senb - sen/?- cos b k+1 k-l (cosa + cos b) + (sen« + sen b) (cosa—cosb) - (sena- senb) Jt + 1 k-1 Transformando a producto tenemos: _ t+i -2sen(^)[s«?(^)+cos(^)] ^PACSO P IDITO1II R14 466 Problemas de Trigonometría y cómo resolverlos
Simplificando, obtenemos: .(a-b\ Jt + 1 Ja-b\ \k + l . (a~b\ 1-k -cot(^-) = i^T => cotrrJ=Yjt -• tan(-2-) = TÑt 17.- Simplificando a producto los términos de «VV»: VV = gen4A + sen4E> + sen 4C a producto W = 2sen(2A + 2B)cos(2A - 2B) + ,sen4C doble Pero: 2A + 2B = 2n-2C => sen(2A + 2B) = - sen 2C Utilizando las propiedades anteriores tenemos: W = 2(- sen 2C). cos (2A - 2B) + 2sen 2C cos 2C Efectuando y agrupando convenientemente obtenemos: W = 2 sen 2C[cos 2C - cos(2A - 2B)] Pero: 2C = 2n - (2A + 2B) => cos 2C = cos(2A + 2B) A continuación factorizamos la parte indicada: W = 2 sen 2C[cos(2A + 2B)-cos(2A-2B)] W = 2sen 2C[- 2 sen (2B) sen (2A)] Finalmente: VV = - 4 sen 2A . sen 2B . sen 2C 18.- Transformando a producto: 2sen4x.cosx M = —------------ = tan 4y 2cos4x.cosx 2 2 Pero: tan 2x = 2 tan* _ 3 i l-tan2x 1-1 | 4 6 6 Finalmente: tan 4x = 2tan2x . _ —4^ _ _4_ . tan M 1-tan 2x 2_ 7 16 16 Transformaciones de Sumas o Diferencias a Productos
19.-Del primer miembro de la igualdad tenemos: Utilizando la propiedad: sen 3a = sen a [2 cos 2a + 1] en el numerador: sen—(2cos5x +1) -----—Zr. >,-- = 2cos5x + 1 sei Comparando, tendremos: P = 2; Q = 1 Finalmente: P+ (J = 3 20.- Transformando a producto ambas condiciones. cos x + cos y = b sen x + sen y = a Al dividir las condiciones (1) entre (2) obtenemos: ...(2) a b Djuego: sen(x + y) = sen (x + y} = Zl.- Como: = -p, usando proporciones: sen5x + sen3x sen5x-sen3x m + 1 m-1 lüransformamos a producto el numerador y el denominador del primer miembro: 2sen4x.cosx m + 1 2 cos 4x.senx m -1 iFinal mente: m + 1 tan 4x . cot x = -- m-1 tan4x m +1 tanx mi Problemas de Trigonometría y como resolverlos AMracso DITO...
22.- Del segundo miembro de la igualdad tenemos: ¡eos 8x + eos 4x + 2(1 - 2 sen 2x) a producto arco doble = 2 cos 6x cos 2x + 2 . cos 2x = 2 cos 2x . (1 + cos 6x) Luego de factorizar y utilizar las fórmulas apropiadas tenemos: 2 2 = 2 cos 2x . 2 eos 3x = 4 cos(2x). cos (3x) Comparando con la expresión inicial, obtenemos: => P = 4 ; Q = 2 ;R = 2;S = 3 /. (P + Q) - (R + S) = 1 23.- Transformando a producto el numerador y el denominador tenemos: -2sen(—13°)-Sen(60°) ^3 f 43/2^ ^3 2sen(30°).cos(13°) “ 3 k =* tan 13 1 /2 J “ 3 k k Luego: tan 13° = ~ ... (1) Hallemos la relación angular: 32° + 13° = 45° => 32° = 45° - 13° 1-tan 13° De donde: tan 32° = tan(45° - 13°) - > tan 32° = 1 + tanl3o - - (2) l-k 1 3 3-k Al reemplazar (1) en (2), tendremos: W = tan 32° =-----.-. W = 1+3 24 .- Transformando a producto: (A + B'i (A-B sen A + sen B - sen C = 2 senl —-— I .cosí —“— - sen C ... (1) Como: A + B + C = 7t=>sen C = sen(A + B) = 2 sen(^*8)cos(^+8 j En (1): sen A + sen B - sen C = 2 sen (A+B\ (A+B\ -2sen( 2 ;cos \ 2 / /a+r\ A B ABC = 2 senl ^ l . 2 seny. sen y =4 sen y .seny. cosy c COSy Transformaciones de Sumas o Diferencias a Productos R14 469
ABC ABC Luego: 4 sen—. sen— . cosy = M seny. seny . cosy .-. M = 4 25 .-Colocando la expresión en términos de senos y cosenos y además transformamos a producto donde corresponde. Por condición del problema se cumple: B + C 2 sen í n A — sen I 2 2 A = cosy eos fn A — I 2 2 A = seny Reemplazando las equivalencias anteriores en «W» obtenemos: A continuación factorizamos y simplificamos, obteniendo: Finalmente: 26 .- Transformando a producto la suma de senos y de cosenos tenemos: = m... (1) = n ... (2) R14 470 Problemas de Trigonometría y cómo resolverlos RACSO
A continuación dividimos las condiciones (1) y (2): tan tn ti - - - (3) 9 l-tan a Asimismo recordamos que: cos 2a =-----?— 1+tan a Utilizando dicha propiedad para un arco tal como: (x + 4) cos(x + y) = Finalmente al efectuar obtenemos: 1 — tan1 2! l+tan2| x+y 2 x+y 2 1- n j 2 2 W = cos(x + y) = ti + m 2 2 W= ”2 m2 n +m 27.- Descomponiendo el numerador como sigue para luego transformar a producto, tenemos: tan 6 = cos 20°— cos40°+ cos 20° sen40° 2sen300.senl0°+cos200 sen 40° A continuación transformamos a producto el numerador tan 6 = senl0°+sen70o sen 40° tan 6 = 2 sen 40° .eos 30° sen 40° Finalmente: tan 6 = 2 cos 30° = 2 V3 -J3 sen0 = - ,6g IIIC 1 + 28 .-Trabajando la condición en términos de cosenos, tendremos: 1 1 --------- + --------- = 2 sec x cos(x+a)-cos(x - a) cos(x + fl) + cos(x-fl) _ 2 cos(x + fl).cos(x-fl) cosx Transformando a producto el numerador: 2cosx,cos« _ 2 cos2x-sen2fl cosx 2 2 2 2 cos x . cos a = 2 cos x - 2 sen a 2 2 => 2 sen a = (1 - cos a). 2 cos x 2 x A continuación efectuamos y despejamos el «cos x», así: 2 2 sen a => cos x =---------- 1-COSfl 2fl 2 a 4sen —.eos — 2 2 o 2 fl 2sen — 2 2 2 eos x = 2 cos a 2 Transformaciones de Sumas o Diferencias a Productos R14 471
29 .- Six = Oyx = <¡) son las raíces de la ecuación, entonces se debe cumplir: a sen 6 + b cos 6 = c... (1) a sen <¡) + b cos <¡> = c. . (2) A continuación igualamos las condiciones (1) y (2): a sen 6 + b cos 6 = a sen $ + b cos $ => fl(sen 6 - sen <¡>) = - b(cos 6 - cos <¡)) o fo+íí a 2senl Icosl I = -b q (6—ó') i -2sen Isenl 6+0 2 fe+<> => tanl 2 a b Luego: VV = tan| 6+<¡)^ a 2 ~b 30 .- Del primer miembro de la igualdad tenemos: sen 5x+sen x+sen 3x = 2 sen 3x cos 2x + sen 3x , sen3j Efectuando como sigue: = sen 3x(2 cos 2x + 1) = sen 3x .---- senx Desarrollando el seno del triple, así: _ sen23x _ (3senx-4sen3x)2 _ 9senzx-24sen4x + 16sen6x senx senx senx A continuación comparando la condición inicial obtenemos: 5 3 = ¿16^ sen x —24;sen x+^ sen x M=16 N=-24 Finalmente: VV = 9 - (-24) - 2(16) = 1 .-. W = 1 31 .- Transformando a producto la expresión VV, tenemos: VV = 2 sen(x - 15°). cos(35°) Para que VV sea máximo => sen(x - 15°) debe ser máximo. sen (x - 15°) = 1 => x - 15° = 90° .’. x = 105° ; x G [0 ; 360o] EXPRESIONES EQUIVALENTES 32 .- Colocando la expresión en términos de cosenos: W 2(2senxcosx) +1 2sen2x +1 cosx cosx A continuación factorizamos el 2, logrando una suma de senos que ha de transformarse a producto. Problemas de Trigonometría y cómo resolverlos RACSO
zFsenZx+il W =------------- = 2 secx. sen2x + sen^ LV3A | O í 2x4-71/6^ Í2x-n/6 W = 2 sec x . 2 sen eos z Finalmente tendremos: 33-- Escribiendo «W» como sigue: W = 2 sen 40° + 2' sen 40° - J3 Pero: J3 = 2 sen 60° A continuación agrupamos convenientemente, tal que: W = 2 sen 40° - 2(sen 60° - sen 40°) => W = 2 sen 40° -2(2 sen 10° cos 50°) Pero: cos 50° = sen 40°; sen 10° = cos 80° (por arcos complementarios). Escribimos la expresión como una propiedad especial del triple, así: W = 2 sen 40 [1 - 2 sen 10o] => W = -2 sen 40°[2 cos 80° - 1] ...(*) cos 3a 2 cos 2a - 1 = cosa (es la propiedad a utilizar) cos 3(40°) Finalmente utilizamos esta propiedad en (*): W = -2sen 40°. ---- rr ' cos40 — 2sen40°f-—| l 2 De donde: W =---------—rr---* W = tan 40° cos40 34.- Escribiendo como sigue la expresión, tendremos: MI. MI — = ~ + 2 cos 20° => — = ~ + cos 20° + cos 20° A continuación transformamos a producto los dos primeros términos, así: M — = cos 60°+eos 20° 4- cos 20° M M — = 2 cos 40°.cos 20° 4- cos 20° => — = cos 20°(2 cos 40° + 1) Transformaciones de Sumas o Diferencias a Productos R14 473
Pero: sen60° v3 2 cos 40° + 1 =--—— = -----—- sen20 2sen20 Luego: = cos 20°. J3 2sen20° M = J3 -cot20° M 2 35. Escribiendo como sigue la expresión, tendremos: W = + l) = 2j3 Pero: = sen 60° y = sen 30°, Luego: W = 2 J3 (sen 60°+sen 30°) a producto Transformando a producto tendremos: W = 2j3.2 sen 45' cos 15° r- J2 1— W = 4 J3 . y- -cos!5“ Vi = 2 sen 75° 36.- Transformamos a producto como sigue: Finalmente: W = 4 cos x. cos W= 1+cos2x +cosx => W = 2 cos2x + cos x .-. W = 2 cosx (cosx + i A1 cambiar 1/2 por n/3 podemos transformar a producto: / ni (x+n/3^ (x-n/3 W = 2 cos x Icosx + cos—j => W = 2 eos x . 2 cos| |cos | a producto X 71A 2 + 6j cos x rtA 2~6 J d doble 37.- Recordando: 9 2cos a = 1 + cos 2a la mitad W = 1+ cos 14° + sen 14° Aplicando dicha propiedad en «W», tendremos: W = 2 eos2?0 + 2 sen 7° cos 7° RACSO IDIIDlll R14 474 Problemas de Trigonometría y cómo resolverlos
A continuación factorizamos 2 cos 7° para luego transformar a producto: W = 2 cos 7°(cos 7o + sen 7o) W = 2 eos 7o (sen 83° + sen 7o) a producto W = 2 cos 7o . 2 sen(45°)cos (38°) 38.- Igualando a una constante «k» tendremos: sen5x sen3x senx , —“b~ = c Luego: sen 5x = ak => sen 3x = bk => sen x =ck Sumando: De donde: gen 5x+senx + sen 3x = k(a + b + c) a producto 2 sen 3x cos 2x + sen 3x = k(a + b + c) Factorizando sen 3x: sen 3x [2 cos 2x + 1] = k(a + b + c) A continuación utilizando la proporcionalidad inicial tendremos: sen 3x sen3x.------ senx bk bk. — =k(a + b+ c) Finalmente simplificamos obteniendo: b‘ b(b - c) = da + d 71 39.- De (1): sen (x + —) - sen x = m o (n I cos (x + — ) = ... (3) 71 12 De (2): 71 cos (x + —) + cos x = n o 2 cos 71 íx+ 12) cos n 12 = n ... (4) Al dividir las proposiciones (3) y (4) obtenemos: Q 71 Z 71 X 2sen—.cos(x+—) _ 12 12 = « 2cos—cos(x+—) n 12 12 n tan- m n Pero: 71 12 Finalmente: m = (2 - -¿3 )n Transformaciones de Sumas o Diferencias a Productos R14 475
SITUACIONES GRÁFICAS 40.- Sea AB = CD = i En el LAEB: BE = i sen ct => AE = i cos a En el LCED: ED = i sen 6 => CE = t cos 6 Pero AC=AE-CE También BD = ED - BE Al dividir las proposiciones (1) y (2) D 4 => 3 = fcosa-tcos6 ...(1) => 4 = i sen 6 -i sen ct... (2) B senB—sena 4 tenemos: ----------~ ~ cosa—cosB 3 A continuación transformamos a producto tanto el numerador como el denominador, obteniendo: 41.- Sea CB = a DC = 2a tn Z DAC = x - y En el k ABC: AC = a . csc y ... (1) En el k ADC: AC = 2a . csc(x -y) ... (2) Al igualar las proposiciones (1) y (2), tendremos: sen(x—y) a csc y = 2a csc(x - y) =$ ------ = 2 seny . sen(x-y) + seny 2 + 1 Por proporciones: -------------— = —--- sen(x-y)-seny 2 — 1 Transformando a producto, obtenemos: Problemas de Trigonometría y cómo resolverlos RACSO IDITOIII
í X | X 2sen! — Lcosí--v) , , . cot(^-y) =3 =* tan J = 3 . tan (|-y 1 Al simplificar: /. W = — 42.- De la figura deducimos que: k2 2 S = -2 sen x cos x. 2 sen 2jc S = k2.2 sen x.cosx .senz2r Finalmente: S = k2.sen32x Transformaciones de Sumas o Diferencias a Productos
CAP. 15 transformaciones de Producto a o Diferencias SIMPLIFICACIONES O 01.- Se sabe que: 4 cos ct = 3 cos a + cos 3a ; 2 cos a cos P = cos(a + P) + cos(a - P) Multiplicamos por 2 el numerador y el denominador para aplicar la primera propiedad y a continuación factorizar en el denominador convenientemente para aplicar la fórmu- la básica del coseno triple: r 4cos x-2cos2x.cosx + 2cosx W = --------------g----------- 2.2cosx(4cos x-3cosx) 3 cosx + cos3x -[ cos3x+ cosx] + 2cosx 4cosx.cos3x Luego de simplificar obtenemos: 4cosx 1 W = -------------- =------ 4 cosx. cos 3x cos3x W = sec 3x 02.- Al efectuar y ordenar apropiadamente tenemos: 3 2 1 2 W= — (2 sen 3x) -~ (2 sen 9x sen 3x) + (2sen3xcos3x) degradar transformar dobles Luego de efectuar y ordenar apropiadamente, obtenemos: 3 1 2 W = — (1 - cos 6x) - — (cos 6x - cos 12x) + (sen 6x) 3 3 1 1 1 2 W = — - — cos 6x - — cos 6x + — cos 12x + — (2 sen 6x) 3 11 W = ~ - 2 cos 6x + ~ cos 12 x + — (1 - cos 12x) Al simplificar tenemos: 3 1 11 W = — - 2 cos 6x + — cos 12 x + — -— cos 12x => W = 2-2 cos 6x = 2(1 - cos 6x) W = 2(2 sen23x) .-. W = 4 sen23x R15 478 Problemas de Trigonometría y cómo resolverlos alRACSO
03.- Al efectuar y ordenar apropiadamente, tenemos: W = 2 cos 5x cos x + 5(2 cos 3x cos x) + 10(2 cos2x) Luego de aplicar las transformaciones indicadas, obtenemos: W = cos 6 y + cos 4x + 5(cos 4x + cos 2x) + 10(1 + cos 2.y) W = cos 6x + cos 4x + 5 cos 4x + 5 cos 2v + 10 + 10 cos 2v W = cos 6y + 6 cos 4x + 15 cos 2x + 10 A continuación expresamos «W» en función del cos 2x, así: W = 4 cos32y - 3 cos 2x + 6[2 cos22x - 1] + 15 cos 2x + 10 W = 4 cos32x + 12 cos22x + 12 cos 2x + 4 Al factorizar 4 notamos que la expresión entre paréntesis es el desarrollo de un binomio al cubo. W = 4(cos32y + 3 cos22x + 3 cos 2x + 1) a 7 "a Finalmente: W = 4(cos 2x + 1) = 4(2 cos x) VV = 32 cos6x 04.- Escribiendo como sigue, tendremos- + cos 6x A continuación transformamos a una diferencia de cosenos, así: cos 4x - ctíS’Gx + ctíS’Sx = cos 4y M = cos 4x 05.- Multiplicando la expresión «M»’ por sen x, tendremos: sen x.M = 2 cos óx.sen x + 2 cos 4x.sen x + 2 cos 2x.sen x - sen 7x Luego de efectuar las transformaciones anteriores obtenemos: Transformaciones de Producto a Sumas o Diferencias
M.sen x = sen 7x - sen 5x + sen 5v - sen 3x + sen 3x - sen x - sen 7x Finalmente: M(sen x) = -sen x .-. M = -1 06.- Al efectuar la expresión tenemos: 1 sen7x W = —-------- - cos 2x - (cos 6x + cos 4x) 2 senx ’ tsen7x-2senx.cos2x—2senx.cos6x-2senxcos4x W = -----------------ñ-------------------- 2 senx Al transformar de producto a suma o diferencia cada uno de los casos anteriores, obtenemos: _ sen7x-[sen3x+sen(-x)]-[sen7x+sen(-5x)]-[sen5x+sen-3x] 2senx En seguida simplificamos, de donde obtenemos: .., sen7x - sen3x + senx - sen7 r+sen 5x - sen5x + sen3x W = --------------------------------------- 2 senx senx 1 Finalmente: W = —-------- = — 2senx 2 07.- Al efectuar notamos que el m.c.m. de M es: 2 cos 72' cosl2°-2cos24°. cos 72' M= 2 cos 72° A continuación efectuamos la transformación presente: -sen 6' cos 12o- cos 96o- cos 48' M= 2 cos 72° 2scn30°.sen 180-cos96' 2 cos 72° M = Luego de simplificar en el numerador obtenemos: sen18c’+sen6' M= —-----“ 2cos72 En seguida transformamos a producto y deducimos que o = 36°: 2senl2c’.cos6' 2senl8° Finalmente al comparar las expresiones trigonométricas: sen 12°. cos 6o sen 2o cos a sen!8° sen 3 o. a = 6o M = Problemas de Trigonometría y cómo resolverlos jl a< so Meditokii I
PROBLEMAS CONDICIONALES 08.- Expresando W en términos de senos y cosenos, tenemos: Transformando a una diferencia o a una suma de cosenos tenemos: cos(B)-cos(A) cos(A) + cos(B) Reemplazando el dato: ,,, cosB-cosBcosC W =-------------- cos B cos C +cosB Luego de factorizar apropiadamente, obtenemos: cosB(l-cosC) cosB(l +cosC) 9 2ÍC 2sen — l 2 2cos — W = tan2 (£) 09.- Expresando en términos de senos y cosenos, tenemos: scn(x + y) seny " cos(x + y) COS x. cos (x + y) sen(x + y). cos x - sen y W - cos.x.cos(x+y) A continuación efectuando la transformación en el numerador obtenemos: 2scn(x + y).cosx — 2seny 2cosx.cos(x + y) scn(2x + y) + sen(y) - 2seny 2cosx.cos(x + y) W = En seguida transformamos a producto el numerador: sen(2 y + y) — sen y 2cos_y.cos(.y +y) 2scnx.cos(x + y) W = v,--------7----r = tan y 2cosx.cos(x+y) Al reemplazar el valor de x al problema tenemos: W = tan x = tan = tan 21° = tan(37° - 16°) tan 37o-tan 16° Finalmente: W = ~------zzz:---— 1 +tan 37°. tan 16' 3 7 44 4 24 96 44 w = — 117 i+U 4 24 ~ 117 96 Transformaciones de Producto a Sumas o Diferencias R1S 481
10.- Agrupando en forma conveniente tenemos: W = eos 6x + eos 2x + cos 4x => W = 2 cos 4x cos 2x + cos 4x a producto W = cos 4x 12 cos 2x +11 => W = cos 4x. scn l---------i senx del arco triple A continuación transformamos ei numerador a producto obteniendo: 2sen3x.cos4x sen7x + sen(-x) W = —------------ => W =----------------------. (’) 2senx 2senx Por dato: sen 7x - 2 sen x, al reemplazarlo en (*), obtenemos: 2senx - senx senx 1 W =-------------- =------ W = — 2senx 2senx 2 sen(x + y) 11.- Sabemos que: tan x + tan y = --------- cosx. cos y Como: tan(a + 6) + tan (a - 6) = 2 sen2a Entonces: ------—----------— =2 => sen 2a = 2cos (a + 6).cos (a - 6) cos(a + 6).cos(a-6) => sen 2a = cos 2a + cos 26 => sen 2a - cos 2a = cos 26 2 2 2 Elevando al cuadrado: sen 2a + cos 2a - 2 sen 2a. cos 2a = cos 26 Aplicando convenientemente propiedades 1 - sen 4a = 1 - sen 26 => sen 4a - sen 26 = 6 .". M = 6 12.- Del dato: 2 sen B sen C = 2 eos2 j Degradando tenemos: 2 sen B sen C = 1 + cos A Efectuando la transformación de producto a diferencia: cos(B - C) - cos(B + C) = 1 + cos A .. (1) Pero: cos(B + C) = -cos A Luego en (1) obtenemos: cos (B - C) + cos A = 1 + cos A Finalmente en un triángulo se cumple que: cos (B-C) = l => B-C = 6 Por lo tanto el triángulo es isósceles. R15 482 Problemas de Trigonometría y cómo resolverlos XHACSO
13.- Del primer miembro de la igualdad tenemos: 6tt . sen— 4ti 7 = cos —.----7— 7 OTt cos— 7 Tt + 2 seny ÓTt 71 Pero: sen — = seny .. (arcos suplementarios) Efectuando como sigue, tenemos: seny (cos y- + 2cosy ) seny (cosy + cosy + cosyj COSy COS-y^ seny ^2cosy .cosy+cos y] Transformando ñ producto como sigue: W =--------------------------- COSy _ 6rt Tt 5rt 2rt . , , . x Pero: cos“y =-cosy ; cos y = -cosy (arcos suplementarios) Utilizando las igualdades anteriores, obtenemos: Tt W = sen ~ Tt 2rt 3rt = sen— 2 cos— + 1 = sen— 7 7 7 = sen(W - 1) y Finalmente: W -1 = 3 W - 4 W 14 .- Efectuando como sigue, tenemos: W = sen 3v. cos 3x - 2 sen x cos 3x + 2 sen 3x. cos x - 4 sen x cos v A continuación transformamos los productos indicados: 1 W = — (2 sen 3x.cos 3x) - [sen 4x + sen(-2x)] + [sen 4x + sen 2x¡ - 2(2 sen x cos x) 1 Al simplificar obtenemos: W = y sen 6x - sen 4x + sen 2x + sen 4x + sen 2x - 2 . sen 2r 1 W = y sen 6x Transformaciones de Producto a Sumas o Diferencias
Por ángulo doble se sabe que: sen 2a = 2 tana l + tan2a 1 2tan3x 2 1 + tan2 3x Finalmente: 2 2 W = ---y = T 1 + 22 5 2 w=- a c a + b a+d 15 .- Si: ~ ~ se cumple que: -r = --- b d r n a-b c-d Aplicando dicha propiedad al problema obtenemos: tanx + tana _ 1+cos x + l + sen x tanx-tana l+cos2x—(l+sen2x) sen(x+y) Como: tan x + tan y = COSXCOSy en el problema: sen(x+a) cosx. cosa 3 sen(x-a) cos x.cos a 2 2 cos x-sen x sen(x + a) 3 sen(x-a) cos2x A continuación efectuamos como sigue: sen (x + a) . cos 2x = 3. sen(x - a) 2 sen(x + a). cos 2x = 6 sen(x - a) => sen(3x + a) + sen(a - x) = 6 sen(x - a) sen(3x + a) - sen(x - a) = 6 sen(x - a) => sen(3x + a) = 7 sen(x - a) Finalmente: sen (3x + a). csc (x - a) = 7 16 .- Multiplicando por (2) ambos miembros: 7 7 7 2W = 2 sen x + 2 sen y + 2 sen z => 2W = 1 - cos 2x + 1 - cos 2y + 1 - cos 2z A continuación transformamos a producto como se indica 2W = 3 - [ cos 2x + eos 2y + cos2z ] a producto doble => 2W = 3 - [2 cos(x + y) . cos(x - y) + 2 cos2z - 1] Pero: cos(x + y) = cos (2n - z) = cos z En seguida agrupamos convenientemente, para transformar a producto: *” DITO»»» R15 484 Problemas de Trigonometría y cómo resolverlos
2W = 4-2 cos z[cos(* - y) + cos z] => 2W = 4 - 2cos z[ cos(* - y) + cos(* + y) ] a producto W = 2 - cos z[2 cos x . cos(-y)] Finalmente al efectuar y reemplazar la condición inicial obtenemos: W = 2 - 2 cos x cos y. cos z .-. VV = 2(1 - m) 17 .- En el dato, transformamos a producto y por arco doble tenemos: Además sabemos que: cos(o - b) = cos a cos b + sen a sen b Aplicando la propiedad anterior al problema, tendremos: x y * y í x y x y cos — eos— + sen—.seny =3 eos—eos— - sen -sen — x y x y Al efectuar obtenemos: 4 seny seny = 2 cos y cos y Finalmente: tan y . tany = y P- Q = 18 .- Del primer miembro de la igualdad tenemos: 32 sen6* = 2(4 sen3*)2 => 32 sen6* = 2(3 sen x - sen 3*)2 Al efectuar el binomio anterior obtenemos: 32 sen x = 2(9 sen x - 6 sen 3*.sen x + sen“3*) Agrupando convenientemente, tendremos: _ A 7 7 32 sen x = 9(2 sen x) - 6(2 sen 3* . sen x) + 2 sen“3* 32 sen6* = 9-9 cos 2* - 6 cos 2* + 6 cos 4* + 1 - cos 6* Al efectuar y simplificar nos queda: 32 sen6* =10-15 cos 2* + 6 cos 4* - cos 6* Identificando, tenemos: M = 10; N = -15 ;P = 6;Q = -1 .'. M + N + P + Q = 0 Transformaciones de Producto a Sumas o Diferencias R15 485
19 .- Del segundo miembro de la igualdad tenemos: sen x (8 sen4x) = sen x(3 - 4 cos 2x + cos 4x) 1 3scnx-2 (2senxcos2x) + — (2senxcos4x) sen 3x-senx sen5x-sen3x , 5 1 Efectuando tendremos: 5 sen x - — sen 3x + — sen 5x 5 1 Luego identificando se tiene: M = 5;N = -~ ;P= — .-. M + N + P = 3 20 .- Expresando «W» en términos de cosenos, obtenemos: 2 cos 10° _ 2cosl0°—J3cos20° cos20° ’ ~ cos20° Pero: 73 = 2 sen 60° 2 cos 10°-2sen60°cos 20° cos 20° Al efectuar la transformación anterior, tendremos: _ 2cosl0°-[sen800+sen400] W cos20° Pero cos 10° = sen 80° (arco complementario) Finalmente transformamos a producto el numerador: sen80°-sen40° cos 20° 2sen20°cos60° cos20° W = tan 20° 21 .- Del segundo miembro de la igualdad tenemos: 1 = ~ [ 2 cos 16° cos 13° - 2 cos 59° cos 20o] 1 = ~ [cos 29° + cos 3o - cos 79° - cos 39o] 1 = - ~ [cos 79° - cos 29° + cos 39° - cos 3o] a producto a producto Efectuando como se indicó, obtenemos: Problemas de Trigonometría y cómo resolverlos Aaracso iDITOlII
- - 11~2 sen 25° sen 54° + (-2 sen 18° sen 21°)] = sen54° . sen 25° + geni 8° - sen 21° P -Q => P = sen 54° Q = - sen 18° Finalmente al comparar obtenemos: P + Q = sen 54° - sen 18° = '75 + 1^1 Íj5-f 4 ' 4 k 7 k 2 1 P + Q= - 22 .- Escribiendo apropiadamente para realizar la transformación presente, obteniendo: W = 2(2 cos 3x . cos x) + 1 => W = 2[cos 4x + cos 2x] + 1 A continuación agrupamos convenientemente logrando: sen3x 2senx. cos 4x + sen3x W = 2cos4x+ 2cos2x + l W = 2cos4x + ~ => W = —— v fecTL-k 3C1U triple Luego de efectuar la transformación anterior obteniendo: sen5x + sen(-3x) + sen3x W =-------------------------- W = sen 5x.csc x senx PROBLEMAS DE CÁLCULO NUMÉRICO 23 .- Se sabe que: -Jl = 2 sen 45° Aplicando dicha equivalencia en el problema obteniendo: cos 19° 2sen45°.sen26° senl9° senl9° A continuación al efectuar la transformación anterior tendremos: cos 19°-(cos 19o-cos 71°) senl9° Finalmente: cos 71 W =------- = 1 senl9° W = 1 Transformaciones de Producto a Sumas o Diferencias
24 .- Al efectuar la transformación inicial, tendremos: _ 2sen40°(sen50o-sen50o+sen30°) - 2sen30°.cos50° Luego de simplificar, obtenemos: 2sen40°.sen30° sen40° M = 2sen30°cos50° =* M = cos 50° Pero: sen 40° = cos 50° M = 1 25 .- Escribiendo como sigue la expresión, tendremos: M= ^^-(2senl5x.sen6x+cos21x) A continuación efectuamos la transformación anterior obteniendo: M = ^Í(cos9x-cos21x + cos21x) => M = ^.cos9x Pero: 7t 27 TI 9x= ~ 3 Finalmente reemplazando el valor numérico del cos 9x en la expresión, logramos: 26 .- Escribiendo la expresión en términos de senos y cosenos obtenemos: l-2(2sen70°cos80°) 2cos80° 1/2 l-2senl50° + 2senl0° 2 cos 80° l-2(senl50°-senl0o) 2cos80° 2senl0° 2cos80° Pero: sen 10° = cos 80° M=1 27 .- Reduciendo al primer cuadrante tenemos: sen 610° = sen(360° + 250°) = sen 250° sen 610° sen 610° = sen (180° + 70°) = -sen 70° RACSO BDITDBBI R15 488 Problemas de Trigonometría y cómo resolverlos
sen 130° = sen(180° - 50°) = sen 50° sen 430° = sen(360° + 70°) = sen 70° cos 280° = cos(360° - 80°) = cos 80° Al reemplazar las reducciones anteriores tendremos: 2W = 2 sen 50°.sen 10° - 2 sen 70°.sen 50° - 2 sen 70° cos 80° 2 = cos 40° - cos 60° - [cos 20° - cos 120°[ - [sen 150° + sen(-10°)[ A continuación simplificando términos, obtenemos: 2W = cos 40° - cos 60° - cos 20° + cos 120° - sen 30° + sen 10° 2W = pos 40°-eos 20° + (-|-|-|) + sen 10° a producto Luego de simplificar obtenemos: 2W = -2 sen 10°.sen 30° + 2) + sen Finalmente: 2W = -sen 10° + (“2) + sen 28.- Efectuando como sigue: M = 3 cos 4x + 2(cos 2x - cos 4x) => M = 3 cos 4x + 2( 2 sen 3x . sen x) M = cos 4x + 2 cos 2x A continuación expresamos «M» en términos del cos 2x , así: 7 M = 2 eos 2x -1+2 cos 2x 1,3 M = 2(cos 2x + “ ) - ~ Utilizando la siguiente igualdad: 2(cos2x+|)2> 0 1,3 3 => 2(cos2x + -)2- - >- - M>-~ M = mínimo _3 2 3 29.- Se sabe que: J3 = 2 sen 60° Luego en el problema: 2sen60°.cos 20°-sen40' tanx = - ~ 1 + senlO Transformando a producto tanto numerador como denominador: Transformaciones de Producto a Sumas o Diferencias
sen80<’+sen40<>-sen 40° 2sen40°cos40° sen 40° tan X = 1 +cos 80° tan x = 2co£.240o = cos40o Simplificando obtenemos: tan x = tan 40° .-. Un valor de t será 40° 30.- Escribiendo como sigue, tendremos: (1—2sen2B) (1 — 2sen2 A) _ 2sen2 A~2sen2 B 2 ~ 2 2 Luego, por ángulo doble recordamos que: cos 2 x = 1 - 2 sen x así: (1—2sen2B)—(1—2sen2 A) 2 sen2 A—2 sen2 B 2 = 2 2 2 sen (A + B) sen (A - B) = sen A - sen B 31.- Multiplicando numerador y denominador por 2 para luego api ¡car las propiedades de transformación de productos a sumas, obtenemos: 2 sen 2a.sena + 2 sen 4a.sena+2 sen 7a.sen 2a = 2sena.cos2a+2sen2a.cos5a + 2sena.cos8a coso-cos 3a + cos 3a - cos 5a + cos 5a -cos 9a ~ sen 3a —sen o + sen 7 a —sen 3a+sen 9a —sen 7a Luego de simplificar la expresión anterior tendremos: cos a-cos 9a _ 2 sen 5a.sen4o_ — sen9a-sena 2 Sen-ig-cos 5a ~ tan 5a- 32.- Efectuando como sigue, tendremos: sen 7a-2 cos2a.sena-2cos4a.sena-2cos6a.sena sena A continuación utilizamos convenientemente las formulas de transformación de pro- ductos a suma o diferencia, obteniéndose: sen 7a - (sen 3a - sena) - (sen 5a - sen 3a) - (sen 7a - sen 5a) sena sen 7a-sen 3a+sena-sen 5a+sen 3a-sen 7a+sen 5a sena sena Finalmente al simplificar, deducimos: senfl = 1 RACSO R15 490 Problemas de Trigonometría y cómo resolverlos
33.- Ordenando como sigue, tendremos: cos 6° . cos 66° . cos 42° . cos 78° A continuación multiplicamos y dividimos por 2 para poder utilizar la fórmula siguiente: 2cos6°.cos66°.2cos42°.cos78° (cos72°+cos60°) (cosl20°+cos36°) 2.2 = 4 Finalmente, al reemplazar los valores numéricos respectivos, obtendremos: 4 16 _ 1 4 “ 16 34.- Agrupando como sigue, obtendremos: sen 6o (sen 54° . sen 60°) = sen 6o . 2sen54°.sen66° 2 Se ha multiplicado y dividido por 2 para poder transformar el numerador a una diferen- cia de cosenos, donde luego de multiplicar nos queda: sen60(cosl2°-cosl20°) _ sen6°.cosl20-sen60.cosl20° 2 - 2 Una vez más multiplicamos y dividimos por 2 en este caso para realizar una transfor- mación de producto a suma de senos: 2sen6°.cosl2°-2sen6°.cosl20° _ 9enl8°-sen6°-2sen6°(-|) 4 4 sen 6o. sen 54°. sen 66° senl8o-_5ert'6°+§errt>6 sen 18° 4 = 4 35.- Multiplicando y dividiendo por 2, obtendremos: 2 sen40°.sen80°+ 2 sen 80°.sen 160°+ 2 sen 160°.sen320° 2 A continuación aplicamos en cada sumando la fórmula de transformación del producto a una diferencia de cosenos, obteniéndose: cos40°-cosl20°+ cos80°- cos240°+ cosl60°- cos480° = 2 En seguida transformando a producto como sigue, deducimos: 2 cos600.cos20°-(-i)-(-l)-cos20c-(-|) ¿obSO5 +1 - cpsSO* 3 2 ' = 2 = 4 Transformaciones de Producto a Sumas o Diferencias
36.- Efectuando como sigue, se obtendrá: senx.sen4x-sen2x.sen3x sen2x.sen4x A continuación aplicando la misma estrategia de los dos últimos problemas, tendremos: 2 sen x.sen4x - 2 sen 2 x.sen3x cos3x - cos5x - (cosx -cos5x) 2sen2x.sen4x 2sen2x.sen4x Luego de simplificar, se tendrá: cos3x-cosx 2 sen2x.sen4x Factorizando el numerador y transformando la diferencia de cosenos a un producto de senos, se obtiene: -2 sen2x.senx 2 sen2x.sen4x senx sen4x 37.- Utilizando la siguiente fórmula especial de transformación: sen a + cos a = -J2 sen (a + 45°) Ahora la expresión dada tendrá la siguiente forma: cos b. y/2 sen (a + 45°) = -J2 cos b. sen (b + 60°) 15°+b Multiplicamos y dividimos por 2 para utilizar la fórmula de transformación de produc- to a suma de senos: -72.2 sen(b +60°) cosb ^2 , --------^2----'----- = 2 5611 + + 561160I Finalmente desarrollamos el ángulo compuesto tal como sigue: (sen 2 b. cos 60° + cos 2 b. sen 60° + sen 60°) ^sen 2b i + cos 2b.+^- j cos b (sen a + cos «) = (sen 2 b + -J3 cos 2 b + -J3 ) RACSO IDITO1II R15 492 Problemas de Trigonometría y cómo resolverlos
38.- Factorizando: 2 sen 6: Por transformación a suma: Según dato: Reemplazando en (i): pues: N = 2sen 6(2sen 66 • sen 46 + 1) N = 2sen 6[cos 26 - cos!6 -6 + 1]... (i) 6=t=6° N = 2sen 6°^cosl2°-i+l cos 60° = Efectuando: N = sen 6°[2cos 12° + 1] Reconociendo: sen x(2cos 2x + 1) = sen 3x Se tendrá: N = sen 3(6°) = sen!8° 39.- Efectuando como sigue tendremos: A continuación multiplicamos y dividimos por 2 así: Vsenx.sen3x +sen2 x I 1 12 sen x.sen 3*+2 sen x 2 Al transformar de producto a suma o diferencia tendremos: /cos2x - cos4x +1 - cos2x Íl-cos4x V 2 ” V 2 9 Pero: 1 - cos 4x = 2 sen 2x Finalmente: cos4x _ ^\ser\ 2x _ |sen 2x| 46.- Utilizando la transformación a producto de una suma o diferencia, tendremos: ló^sen^psen^j = 16(cos 6 - cos 26) cos6-cos2G A continuación, calculamos el valor correspondiente del «cos 26», así: cos 26 = 2 cos26 - 1 = 2 (2-)2 - 1 => cos 26 - 1 = | - 1 => cos 26 = | 8 Finalmente: 32 sen^p .sen^ = 16 (4-3) => ~ 2(5) = 10 32 sen4p .sen^ -= 16 Transformaciones de Producto a Sumas o Diferencias
W.46 Sucesiones y Series SUMATORIAS 01.- De la expresión dada se pueden identificar los siguientes elementos: P = Primer ángulo = x U = último ángulo = 35x r = razón de la progresión aritmética = 2x r 2 n-#de términos de la P.A.------ + 1 35x—x n = —~-- 2x + 1 = 18 r Aplicando la fórmula de la suma de senos para arcos en P. A. tendremos: ... senl8x.senl8x 2_. VV =--------------- .. W = «.vn ISt . cs< i 02.- Identificando los elementos básicos de la serie dada, se tiene que: P = Primer ángulo = x U = último ángulo = 45x r r = razón de la progresión aritmética = 2x => ~ = x U-P 45x-x n = # de términos de la P.A.-- +1 => n = —r--------+ 1 = 23 r 2x Aplicando la propiedad de suma de cosenos para arcos en P.A. tendremos: R16 494 Problemas de Trigonometría y cómo resolverlos RACSO DITOBU
2sen23x.cos23x 2senx sen46x 2 senx 1 W = ~. sen 46x . csc x 03.- Procediendo como en los ejercicios anteriores, podemos identificar que: P = Primer ángulo = 2o U = último ángulo = 540° r r = razón de la progresión aritmética = 2° => U-P n = # de términos de la P.A.-- + 1 r 540°-2' n~ 2° +1 = 270° Aplicando la fórmula de suma de senos para arcos en P.A. tendremos: Pero: W~ . sen sen270° senl . sen 271° sen 270° = - 1 (-l)(-cosl°) w= senl° sen 271° = sen(270° + 1) = - cos 1° W= cotí" P + U 2 04.- En la expresión dada identificamos que: Tt P = Primer ángulo = ~ Tt U = último ángulo = 33 " 4Tt r = razón de la progresión aritmética = ~ r 2n 2 = 15 Sucesiones y Series Trigonométricas
17ti También: sen-j^- = sen 2n 15 2ti = -Sen15 3n -sen— 15 2ti sen— 15 2ti -sen— 15 W = sen — 5 W = sen 36' w= J10-2V5 4 05.- Analizando los términos de la expresión dada se puede reconocer que: Asimismo: También: 12ti Sen^3~ 117t IOtt SCn 13 = sen = sen = sen 7t 13 2n n---- 13 3n n----- 13 71 = 561113 2n sen 13 3ti = Sen!3 Y así sucesivamente, por arcos suplementarios, se puede establecer que: n 2ti W—13 sen— + sen— + sen— vv 13 13 13 3n 6ti +... +sen— 13 13 primer T ángulo P último ? ángulo U De donde reconocemos que: 7t r = razón de la progresión aritmética = — 2 26 W = 13 . ( nr sen — . sen P + U 2 W = 13. 6ti sen— _ 26 7n ----sen — 71 26 sen— D 26 7tt Pero: sen—— = sen 26 n 6n 2 26 6tt = cos — R16 14961 Problemas de Trigonometría y cómo resolverlos ¿4 RACSO wp BDITOKBB
13 w=y. „ 6n 6n 2sen—cos— 26 26 71 sen— 26 12tt sen----- 13 26 2 ' ti sen— 26 7t COS-- 26 71 sen— 26 1271 í ti_____71 Pero: sen-r— = sen Q I = cos ~ 26 ) 26 13 w= 2 13 7t W = — cot— 2 26 06.- Multiplicando por 2 a ambos miembros de la expresión dada, se tendrá: q 7t q 3tt o 5tt q 7t 2M = 2cos — +2cosz— +2cosz— + .. . + 2cos2(2n - 1) n n n n «Degradando»: 2tt 6tt IOtt ti 2M = n + cos— + cos— +cos----------- + ... + cos2(2n -1) — n n n n Usando la serie del coseno: 2M = n + 47tn sen—— 2n —+2(2n-l)— n v ’ n 4ti sen— 2n 2M = n + 2M = n 07.- Aplicando la misma estrategia del problema anterior, multiplicamos por 2 con lo cual cada término adquiere una forma más adecuada para efectuar transformaciones: 2tt 4tt 2tt 6tt 4tt 6tt 2M = 2cos—.eos — + 2cos~. eos — + 2cos“ . eos — Transformando los productos a suma, tendremos: 2tt 6tt 4tt 8tt 2ti IOtt 2M = cos~ + eos— + eos— + eos— + eos— + cos~— 8tt 6tt IOtt 4tt De donde podemos reconocer que: eos— = eos— , eos-~ =005“^“ Sucesiones y Series Trigonométricas
2n 4ti óti 2M = 2(cos — + eos— + eos—) ~ZÍ 2 M = - 2 08.- La expresión será fácil de transformar si multiplicamos por 2 a ambos miembros: j Tt j 2?t j 3ti 2M = 2 sen — + 2 sen — + 2 sen — 7 7 7 Haciendo una transformación para reducir los grados, tendremos: 2rt 4ti 6n 2M = 1- cos — + 1 - cos — + 1 - cos — 7 7 7 I 2tt 4tt 6ti 2M = 3 - eos— + eos— + eos— I 7 7 7 \ 7 T-> ' 2ít 4tt 6tt 1 . De aquí reconocemos que: eos— + eos— + eos— = - — (Propiedad) => 2M = 3 + 1 M = 2 4 09.- Aplicando la identidad: 8 sen4 x = 3 - 4 cos 2x + eos 4x ; tendremos: . 2tt 4it 4rt 8n _ „ 6n 12n 8W = 3-4 005-7-4- cosy 4- 3-4 cos 4- cosy 4-3-4 cosy 4- cos -y 8W = 9 - 4 (eos 4? 4-eos-“4-cos 4- COS^4-COS-^4-COS^ \ 7 7 7 / 7 7 7 * v-------' v *-------z 8W = 9-3Í-|) => 8W = 94-J .’. W = \ z / z lo 10 .- Transformaremos cada término de la expresión dada para aplicar la serie telescópi- ca. Empecemos por el último término, que se constituye en el término general de la serie: . n ____________1______ _________sena_________ sec na. sec(n 4- ija Cosna.cos(n 4- l)a senacosnacos(n 4- l)a , ... 1................... , tan(n4-l)a tanna => sec na.sec(n 4- l)a =-— {tan(n 4- l)a - tan nal =--------- ----~ ' ’ sena ' ' sena sena , tan(n + l)a tanna => sec na.secfn 4- l)a ------------------ sena sena Problemas de Trigonometría y cómo resolverlos -•Mracso Mbditciki
Finalmente dando valores a n, en la expresión obtenida, la serie tendrá la forma: N = tan2a tana sena sena tan3a sena tan2ct sena tan(n + l)a tanna sena sena Donde luego de s mplificar, se obtiene: tan(n + l)a tanna sena sena N = 11 .- Nuestra estrategia consistirá en transformar las expresiones dadas empleando la siguiente identidad trigonométrica: cot a - tan a = 2 cot 2a => tan a = cot a - 2 cot 2a Aplicando dicha identidad a la serie, tendremos tan y = coty -2cotx 1 . x 1 . _2L . x 2 tan = 2 cot 22 - cot 1 , x 1 , x 1 x —5-tan—v = —5-cot—v - 7?cot —5- 2Z 23 2Z 23 2 2Z 1 . x 1 . x 1 . x " i- tan— = „ . cot— - _ cot— 2^^ 2 2 1 2 2 2 2 2 Finalmente sumamos miembro a miembro: 1 x R= ^rcot^r -2cotx 4n 2ji 6n 12 - Se tiene que: VV = sen— + sen— - sen— a producto arco doble Luego de las transformaciones se obtiene: 3n ji 3n 3n VV = 2 sen— . cos ~ - 2 sen —eos — 7 7 7 7 3n 3n n W = -2 sen—[eos— -cos~ 1 7 1 7 7 J a producto 3n n 2ti VV = - 2 sen— [- 2 sen y. sen + — ] 7t 2ti 3ti VV = 4 seny . sen — . sen — Dado que la expresión indicada es una productoria resulta conveniente identificar que: 2n + 1 = 7 n = 3 Sucesiones y Series Trigonométricas
Aplicando la fórmula correspondiente, se tendrá que: W = 4. V2h + 1 2n V7 w = 4.^r 23 V7 W= --- 2 13 .- Nuestra estrategia consistirá en transformar la expresión en términos de coseno, luego efectuar las operaciones indicadas con lo cual surgirán los productos de cosenos, en donde será posible aplicar las fórmulas de productorias. Veamos: 2n + 4n + 6n cos— cos— cos— 7 7 7 M = 2n 4n 2n 6rt 4n 6n cos—.eos— + cos— .eos—+cos—.eos— 7 7 7 7 7 7 2n 4n 6n cos—.eos-—.eos— 7 7 7 Pero: 4n 3n eos— =-cos — 7 7 6ti n cosy- =-cos~ Luego el denominador de la expresión (*) se transforma en: 2n 4n 6n n 2n 3n COS“ . eos—. eos — = cos~ . cos~ .eos— 7 7 7 7 7 7 2n 4n 6n 1 Por la productoria conocida: eos — . cos~^~ . eos— = ~ Por el prob. anterior, el numerador de (*) queda así: 2n 4n 2n 6n 4n 6n 1 COS — .COS“ + COS'T”-COS— + — .COS— =-~ 7 7 7 7 7 7 2 i 2 Luego: M = —M = - 4 8 14 .- Buscando usar el método telescópico, tendremos: . 1 “n2 . Desarrollando el numerador de la expresión anterior y simplificando, encontramos que. 0 csc 6 = cot - cot 6 csc 26 = cot 6 - cot 26 R1S 500 Problemas de Trigonometría y cómo resolverlos RACSO IDITOlll
csc 46 = cot 26 - cot 46 Y generalizando: csc 2" 1 6 = cot 2" ’2 6 - cot 2n T6 Finalmente: S = cot 21 6 - cpíl9 + cp^6 - cob^e +'\+ cpFS^’ 26 - cot 2"" 16 S = cot 2’1 6 - cot 2” "1 6 15 .- Reemplazando x = y en lo pedido; se tendrá: • tl • 2tt . 4tl í°\ R = tan y + tan + tan — .. (i) ti 2tl 4tl En el cual se puede notar que: y + -y + - n Por lo cual se puede aplicar: tan y + tan -y + tan -y = tan y - tan -y - tan -y En (i): R = tan y - tan -y • tan -y .. (n) 4ti 3ti 4ti 3ti Además, por reducción al primer cuadrante: tan -y = -tan -y; pues: -y + -y = ti Que al reemplazar en (ii): se obtiene: R = - (tan ? ' - (“*) La expresión entre paréntesis corresponde a un valor obtenido según: n . 2n . 3n . nn n—~ tan —,7 ‘ tan ----7 tan 7;—,7 .. tan 7;-7 = V 2h +1 2n +1 2n +1 2n + 1 2n +1 Así cuando: n = 3 . tt . 2ti , 3ti trj tany - tan-y tan-y = -v7 Finalmente, en (iii): R = -(V?) 16 .- Multiplicando y dividiendo por "2 sen x" a cada término de la serie, tendremos: 2sen(2n -l)x. senx cos(2n - 2)x - cos2nx 2 senx _ 2senx Luego, dando valores a n, la serie tendrá la forma: ccptf' cos^x co>-^x cos4x 2'senx "¿senx ^>2senx " 2senx cos(2jj^2jx ^<ísenx cos 2nx 2 senx 1 cos 2nx 2 sen2 nx sen2wx Fmalmente, quedará: 2 senx ’ 2 senx = 2 senx " $= senx Sucesiones y Series Trigonométricas
PROPIEDAD.- En general: Si se tiene una suma de senos cuyos ángulos se encuentran en progresión aritmética, se deduce la siguiente propiedad: Sj = sen a + sen (a + r) + sen (a + 2 r) + ... + sen { a + (n- 1) r } nr ser*2 ía+(n-l)rl Sj=------— . sen í--=----> sen' l z J Donde: n ... número de términos de la serie r ..... razón de la progresión nr sen-2~ (P+LI^I Más fácil aún: S, = ------. sen —ñ— 1 r 12 seny V J P ... primer ángulo Ü .. último ángulo En forma similar podemos deducir, la suma de cosenos cuyos ángulos se encuentran en progresión aritmética; así: S2 = cos a + cos (a + r) + cos (a + 2 r) + ... + cos {a + (n-1) r} «r , sen 2 í p+u S2 =-----— . eos 2 sen^- l 17 .- Observando detenidamente los términos de la serie , reconocemos que: n =? r = 20° P = 10° Ú = 170° 2n - 1 = 17 n =9 Luego, aplicando directamente la última relación de la propiedad anterior, la sumatoria tendrá un valor que viene dado así: R16 502 Problemas de Trigonometría y cómo resolverlos RACSO DITOIll
18 .- Inicialmente aplicamos la siguiente propiedad: 2 sen x = 1 - cos 2 x , obteniendo 2 E = 1 - cos 2o + 1 - cos 4o + 1 - cos 6o + ... + 1 - cos 180° 2 E = 90 - (cos 2o + cos 4o + cos 6o + ... + cos 180°) Luego, para utilizar la fórmula (12.9) reconocemos que: n = 90 a P = 2° a r = 2o a Ú = 180° ^2°+180°j 2 E = 90 ---qo— • cos seña- sen 90° (-senl°) 2E = 9°~lenr cos(90°+n =. 2E = 90-~nF~ Finalmente: 2E = 90+l E = 45,5 2 19.- Aplicando la propiedad: 2 cos x = 1 + cos 2 x, tendremos: 2 E = 1 + cos Io + 1 + cos 2o + ... + 1 + cos 180° 2 E = 180 + cos Io + cos 2o + ... + cos 180° Finalmente: 180(1°) sen—s— . cos 1° +180° 2 2 E = 180 - 1 n 179 E="2“ E =89,5 20.- Para utilizar la propiedad del Prob. 16, debemos reconocer que : _ 2n a 2n T~T 6n n = 3 , r= y , P = y , U = y í2nA i sen 3 y L— s=----\—L— 2n i sen-y .i . cos <2ti 671a ~T+~7 2 Sucesiones y Series Trigonométricas R16 503
3n 4n sen cos-y- S =----------— & 2n sen-y 3n 4n 2 sen-y cos-y \ 2n 2 sen -y Utilizando la propiedad en el numerador, tendremos: 7 71 jt sen——seny S=-----------— 2it 2 sen-y Sen7 _ 1 2n — ”2 2 sen-y 2it 4it 6n 1 cos-y + cos y + cos~y= ~2 PROPIEDAD.- 2ít 4ít 2/itt j En general, se cumple: cos 2n+l + cos 2m+1 + — + cos 2n+í = ~ 2 Siendo: n = {1,2, V n e Z...} ít 3ít (2ft-l)7t j Análogamente, se deduce: cos 2h+1 + cos 2m+Í + • • •+ cos 2h+1 = 2 21.- Usando la propiedad del problema anterior, tendremos: 2 2 cos a = 1 + cos 2 a => 2 1 + cos2a cos a —-----5--- 1 + cosy£ 1 + cos4? 1 + cos^ _________________/ __________/ 2 + 2 + 2 3 + cos4^ + cos4^ + cos^ 2 Luego, utilizando la formula del problema anterior deducimos: 2 “ 2 “ 4 2 n 2 2n 2 3n cos y + cos -y + cos y 22.- Utilizando la identidad siguiente, tendremos: sen(Un- Un_i) cosUn-cosU„-i = tan Un - tan Un-1 5 4 tan U2Q - tj 19 Problemas de Trigonometría y cómo resolverlos -'^Aracso VpBDITOftBB
sen(Ui-Uo) Finalmente, obtendremos: cosUrcosU() sen(U20-Ui9) .......... ---H------it— = tan U,n - tan Un COSÜ20-COSIJ19 20 0 „„ 2n 4n 6n 8n 23.- cos-^- + cos^ + cos~tj~ + cos'g- Multiplicando y dividiendo por 2 sen'g , obteniendo: 2sen^cos^ + 2sen^cos4? + 2sen^cos^ + 2sen^cos^ ____y____y_______y____y_______y___y_____ y_____y 2sen| A continuación utilizamos la transformación de producto a diferencia, obteniendo: 2sen| Finalmente nos queda: 6n 2n 4ti 6n 8n 1 COS-g- + COS-g- + COS“g“ + COS g = ~2 24.-Se sabe que: sen2x = l-cos2x 2 cos2x = 1 + cos2x 2 3 sen x = 3senx-sen3x 4 3 COS X - 3cosx+cos3x 4 4 sen x = 3-4cos2x + cos4x 8 4 COS X = 3+4 cos2x+cos4x 8 5 sen x = lOsen x - 5sen 3x+sen 5x 16 cos5x = lOcosx + 5cos3x+ cos5x 16 Al desarrollarse la sumatoria inicial , se obtiene tres sumas parciales así: Xj = cos x + cos (120° - x) + cos (120° + x) E2 = cos2x + eos2 (120° - x) + eos2 (120° + x) = cos3x + eos3 (120° - x) + eos3 (120° + x) Sucesiones y Series Trigonométricas R16 505
Calculando una por una, obtendremos: Ej = cos x + cos(120°-x) + cos(120°+x) 2cos 120° . cos x Ej = cos x + 2 eos x = eos x - eos x Ej = 0 Ahora: E2 = cos* 2x + eos2 (120° - x) + eos2 (120° + x) „ , 2 1 + cos 2a Recordemos que: cos a =---2---- 1 + cos 2x +1 + cos(240° -2x) +1 + cos(240° +2x) Luego, tendremos: E2=----------------------2----------------— Agrupando convenientemente, lograremos: 3 +cos 2x + cos (240°—2x) + cos(240°+2x) Ej= 2 „ „ „ n,no n 3 + COS 2X+2| ¿UlcOS 2X v 3+cos 2x+2 cos 240 cos 2x \ 2 / i2- 2 2 v _ 3 + cos 2x—cos 2x _ _ 3 X2- 2 2 Trabajando con la sumatoria «E3», tendremos: E3 = cos3x + eos3 (120° - x) + eos3 (120° + x) _ , 3 3+cos a + cos 3a Para esta sumatoria, recordaremos que: cos a = -------4------- Luego, se obtendrá: 3cosx + cos3x + 3cos(120°—x)+cos(360°-3x)+ 3cos(120°+x)+cos(360°+3x) £3= 4 Ordenando apropiadamente, obtendremos: 3cosx + cos(120°-x) + cos(120°+x) + 3cos 3x £3= ------- 3 3 Luego: E3= cos3x E3= cos3x Problemas de Trigonometría y cómo resolverlos RACSO 1DITC11I
Finalmente: n E cos" (x) + cos" (120° - x) + cos" (120 + x) = + X2 + E3 n E n=l 3 3 _ 3(2 + cos3x) -0^2 + 4 cos 3x =-------- 4 £ 3(2 + cos3x) n=i - 4 25.- Se sabe que: » 2ti 4n Ó7t (n-l)n 1 * cos — + cos — + cos — +........+ cos —-— = - 2 » n 3n 5n (n-2)n 1 * cos — + cos — + cos — +.......+ cos —~— = 2 Siendo: «n» un número impar. Luego de reducir al primer cuadrante, como se indica obtendremos: cos JT + 2 cos + 3 cos yy +........8 cos-jp + 9 cosA^p + 10 cos-^p 6ti 4ti 2ti cos Ti cos iT cosiT Sumando los extremos, tendremos: 2n 4rt 6rt ,, 10ti Ileos tt + licos TT + licos 3T +........+ 11 cos Ty Finalmente, factorizando «11», lograremos: .,. 2n, 4ti . „ 10nx ll(cos^j- + cos jj+ ... +cos^pj-) = -1 /2(propiedad anterior) -11/2 26.- Se sabe que: Kr £ , , -.x i sen~2~ J2a+(K—l)rl Z sen |a + (n -1) r} =-— . sen (----s----r »=i sen| 1 z J Kr £ . , . SCn^ Í2a+(K-I)rl E cos {a + (n -1) r} =-=- . cos <---s----> n=i sen I 1 J Sucesiones y Series Trigonométricas
Graficando el enunciado del problema, tendremos: Como nos piden calcular la distancia a cada uno de los extremos del diámetro calcularemos primero con respecto al extremo «A», asi: £ distancia - AB + AC + AD +.............+ AQ + AP + AN "A" t n ti o 2n „ (n-1 Luego: £ distancia = 2a sen + 2a sen 2ñ +.................+ sen —2n £ distancia = 2a |sen^ + sen^ +..........+sen^n^J^ senos enprogresión aritmética Finalmente aplicamos la primera fórmula sobre sumatoria de senos y cosenos en progresión aritmética, así: Donde: £ distancia = "A" 2a sen^1*^ 2 ’2n 5—~ - sen sen2'2ñ K = n -1 ....#de términos r =n/2n......razón ct = n/2n....primer ángulo Efectuando, tendremos: £ distancia = 2a . "A" Luego: n (n-l)7t 2n 2n 2 (n-l)n sen . 4n 71 sen-¡¡— 4n J2 2 distancia = a y¡2 "A" 71 71 71 71 sen v cos .— cos . sen „ 4 4n 4 4n sen-¡¡— 4n Problemas de Trigonometría y como resolverlos Tíracso
Luego de efectuar, obtenemos: cos x + cos x = cos x - cos x = 0 E distancia n >Í2 Lnl._JL_il "A" = a V2 . ¡cot4jj Análogamente: = « {co* 4h “T} Finalmente: - 2« (cot£-1| ¿jambos extremos l 4n J 27 .- Transformando a producto como sigue, tendremos: cos x + cos (120°-x) + cos(120°+x) 2cos 120°. cosx 2 2 cos x + cos (120° - x) + cos (12b® + x) = 0 28 .- Efectuando como sigue tendremos: 2cos.cos(120°—x)+ 2cosx.cos(120°+x)+ 2cos(120°—x)cos(120°+x E=-----------------------------2----------------------------- cos(120°—2x) + cos 120°+cos 120°+ cos(120°+2x) + cos2x + cos240° E=-----------------------------2----------------------------- Agrupando los términos señalados y utilizando la propiedad del problema anterior, obtendremos: cos2x + cos(120°-2x) + cos(120°+2x) + 3cosl20° 0 + 3(-1 /2) E=----------------------2--------------------- =------2---- E = -3/4 PRODUCTORIAS 29 .- Nuestra estrategia consistirá en transformar cada término de la productoria en factores que tengan términos comunes en el numerador y denominador. Para ello es pertinente recordar la identidad del ángulo doble: tan 2a = 2 tana 1 - tan2a 1 - tan2 2tana a ~ tan2a Aplicando esta identidad en cada uno de los factores de la productoria, se obtiene: 2tanx 2tan2x 2tan4x 2tan2n l x tan2x ’ tan4x ‘ tan8x tan2nx Luego de simplificar, obtenemos: 2ntanx tan2nx Sucesiones y Series Trigonométricas R16 509
30 .- Debemos reconocer que la sucesión de factores corresponden al cociente de los términos que componen las productorias dadas en la parte teórica. Procedamos así: De donde se obtiene: -Jln + 1 2n 1 2n = V2n + 1 Y de acuerdo con esta nueva productoria, de la relación dada podemos identificar los siguientes términos: a) 2n + 1 = 9 n = 4 factores b) Luego la productoria tiene un valor que se obtiene aplicando la relación obtenida: 1 .----n 1 => W = - J2n + Í W= - J9 W = 1 31 .-En este caso nuestra estrategia consistirá en transformar cada término reduciendo al primer cuadrante. Veamos: 4n 3n 3n 5rt 2n 2tt sen— = sen(7i- — ) = sen — A sen— = sen(7t- —) = sen — ti 2ti 371 => W = sen — . sen — . sen — De acuerdo con la fórmula de la productoria de senos, reconocemos que: 2n + 1 = 7 n = 3 factores A continuación aplicamos la fórmula de productoria y encontramos que: V2n + 1 J? W = ----- = —y 32 .- Nuestra estrategia consistirá en transformar cada factor en una expresión mónica, para lo cual será útil aplicar la identidad del ángulo mitad: 2X 1 + cos x = 2cos ~ o j Tt j 2?t y 371 Tt 27t 371 n => M = (Zcos y )(cos — )(cos —) => M = 8(cosy. cos~ . eos—) R16 510 Problemas de Trigonometría y cómo resolverlos ^^RACSO
ti 2ti 371 1 Pero, podemos identificar que cos —. cos — . cos — = y 1 M = 8. -Z2 o 1 M=i 33 .- Este problema tiene relación con el problema anterior, por lo cual será conveniente transformar uno de sus factores: 5ti 2ti eos— =-cos — De este modo al sustituir en la expresión original, se tendrá que: ti 2ti 3tc M = - cosy . eos — , eos — Y sabiendo que: Concluimos que: ti 2ti 3n 1 cosy. eos— . eos— = y 1 M = -y 34.- Utilizando la identidad: (2cos x - l)(2cos x + 1) = 2cos 2 x + 1 Despejando, se tiene: Entonces tendremos: 2cos x- 1 = 2cos2x +1 2cosx +1 2cos2x +1 2cos4x +1 2cos8x +1 2cosx + 1 x 2cos2x +1 x 2cos4x +1 „ 2cos8x+l " 2cosx+l 35.- Notamos que el término central es: sen yy = sen y = Luego los otros factores los agrupamos de 2 en 2, así:. 2 (2sené sen^) = 2 (cos^-cosf) Análogamente se efectúa con los otras parejas, obteniéndose: Jl 1 3tc 1 2ti 1 ti y x ~2 cos ~T~ x cos ~7~ x ~2 cos ~7 Sucesiones y Series Trigonométricas
24 n 2n 3n cos — x eos — x eos L ✓ ✓ / . 23 Finalmente: 71 3tc scri 2g «seo 2g 28 1371 sen 28 - V2 27 36 .- Recordemos que: „ „ sen2x 2 senr cosx = sen2x —> cosx = --- 2senr sen— senx 2 Sustituyendo en la expresión dada: ------.------- 2sen4 2sen4 2 4 sen — ____4 2sen| sen^-z- 2^-1 2sen-^- 2n Finalmente: cos 2 eos.eos— = senx 2n.sen-3L 2n 37 .- Teniendo en cuenta la siguiente productoria: ti _ 2ti _ 3ít 4ti cos2n+l cos2n+l cos2n+l cos2n+l - nn cos2n+l 2n En el problema reconocemos que n = 3 ; es decir: p- 23 38 .- Aplicando la siguiente productoria: ti 2ti 3tc nrt -J2n+1 sen 2ñ+T ’ sen 2ñ+T ’ sen 2ñ+T.sen 2ñ+T = Luego para di problema: n = 9 k 39 .- * La notación H=1 se desarrolla así: K n an = al - a2 • a3 n=l 1k-i-flK Además se sabe que: p = i 1 29 R16 512 I Problemas de Trigonometría y cómo resolverlos RACSO IDITO1II
» A m n 2n kn -J2k + 1 11 sen2A; + 1 -sen 2k T .sen 2jt + 1 ...sen 2k j - k n=l * A me n 2n kn 1 Il eos 2Jt + 1 -cos 2Jt + 1 .eos 2Jt+1 .cos 2fc + 1 - 2* «=1 f, » n t hti . ti . 2ti . kn in~,—7 II tan 2k+f = tan yj^yy . tan ..........tan yj^yy = V2fc + 1 n=l Desarrollando la expresión pedida obtendremos: (1 + cosy )(1 + cos^j^l + cos'^j (1 +cos^)(l +cos^)(l + cos^) „ ti 6n 3n 4n 5n 2rrt . . Pero: cos y = -cos y , cos y = -cos y , cos y = -cos y (Arcos suplementarios) (1-COSyVl -COS^-V1-COS^-1 T A_______• J\____• J\_____• / . 2 3n t 2 2ti 271 Luego. ~¡~ 4ñ\Í 2n\ ~ ^an 7 •tan 7 ^an 7 ll+COSyjíl+COS-yjll+COSyj ' Yaque. Ticos 2cc = tan2“ (Arco mitad) Finalmente: tan2 y . tan2 y . tan2 y = I tan2 y = ^2(3) +12 = -J?2 = 7 n=l 40.- Sabemos que: sen 2a = 2 sen a cos a , sen 2a Luego: cosa= 2^S Aplicando dicha propiedad en el problema, tendremos: sen^ seny sen^ sen^ sen~^ 2 seny 2senyp 2 sen 4^ 2 seny 2 seny A continuación utilizando reducción al primer cuadrante obtenemos: sen^ seny^ seny j n 32sen4p ' sen^ = 64sen £cos= 7 c. . ti 27t37i 4n 5n 1 n Finalmente: cosy . cosy . cosy . cos-y . cos-y = sen y 1 Tt 64 sen 7 Sucesiones y Series Trigonométricas
Funciones Trigonométricas DOMINIO DELAS F.T. DIRECTAS 01.- Para que/esté definida en IR, se debe cumplir: i) Ya que en/interviene tan x decimos que: x * (2k + 1)— ,kel ii) De la función valor absoluto deducimos que: |senx| * 0 => x *kn, V keZ kn De (i) a (ii): x * ~ , \/ k -G Z kn Dom/=R- VkeZ 02.- Para que la función/esté definida en R se debe cumplir que: 2 1 2 1 1 sen x - ~ > 0 => sen x > ~ => | sen x | > ~ Grafiquemos: y = | sen x | => y = ~ n 5n De esto se deduce que: ~~ < x < — O o Pues en este intervalo: | sen x | > En general: 71 5rt — + kn<x< — +kn 6 6 .-. Dom/= 1, V ke Z Ib o j ¿MRACSO W^BDITOKEB R17 514 Problemas de Trigonometría y cómo resolverlos
03.- Para que la función/esté definida en R, se debe cumplir que: cos x - cos 3x * 0 cos3x n Es decir: cos 3x * cos x => ------* 1 => cos x * 0 ==> x * (2k + 1) . (1) cosx 2 cos3x Pero: ---- = 2 cos 2x - 1 => 2 cos 2x - 1 * 1 => cos 2x 1 cosx => 2x * 2/ot => x * kn... (2) kn kn De(l)y(2): x# — ,\/keZ Dom/=R- —, keh 04.- Sean las funciones:/(x) = 3 tan^3x+^j a /2(x) = cos 2x => Dom/= Dom/j n Dom/2 ... (1) Luego nos proponemos determinar cada dominio por separado. Veamos: a) fi(x) = 3 tan(3x+^j => 3x + 1 (2k + l)y 71 71 71 x*(2Jt + l)-- — , V Jte Z => x*(3Jt+l)~, VJteZ o lo y 71 .-. Dom/j = R - (3fc + 1)~ ,V ke Z ...(2) b) ./i(x) = cos 2x => Dom/2 = R ... (3) c) Reemplazando (2) y (3) en (1): Domf= ^R-(3fc + l)^j n R Dom/= R - (3k + 1) ~, V Ac g Z 05.- Nuestra estrategia consistirá en analizar la F.T. que interviene y a continuación haremos lo propio con los radicandos de la expresión. Veamos: a) Dado que la función depende de la tangente, tan x, podemos afirmar que: x*(2k+T) — ,ke Z b) Además, para que la función/esté definida en R, se debe cumplir que los radicandos deben ser positivos: (tan x-1>0)a(V3 - tan x > 0) Funciones Trigonométricas
Al graficar este resultado, se observa que: 71 71 5ti 4ti — <x< — v — < x <— 4 ~ 3 4 ~ 3 Generalizando este resultado se tiene: 71 4 71 3 Dom/= ^ + kn;^+kn , V ke Z 06.-Puesto que la función depende de tan 5x, podemos afirmar que el dominio de «/> debe ser tal que: 5x*(2Jt+l)^ ,Jte Z => x*(2k+l)^ Tt Luego, se concluye que: Dom/= R - (2k +1) —, V Jce Z 07.- Recordemos la función raíz cuadrada: Por lo tanto para que la función /se encuentre definida se debe cumplir que:
El dominio de f se obtiene al intersectar los 3 dominios: (i n ii n iii) 08.- Nuestra estrategia consistirá en analizar cada F.T. por separado y a continuación haremos el análisis de los radicandos para finalmente intersectar los dominios encon- trados. Veamos: 71 71 37t i) La tan x 3 si x * (2ni + 1) —, m G Z => x * —; — ii) Lacotx3six*mt,nGZ => x # 0; ti iii) La función/existe si: 2(tan x + cotx) - 4 > 0 1 Ahora, de los radicandos se tiene que: tan x + cot x > 2 => tan x +- > 2 tanx Esta relación solo se cumplirá si: x G IC ó IIIC Teniendo en cuenta i), ii), iii) y este último resultado, el dominio de/será: 09.- Analizando cada F.T. por separado, se tiene: i) La sec x 3 si x * (2n + 1) — , n g Z ii) La csc x 3 si x mu, m g Z iii) La fundón/3 si el denominador es diferente de cero. | sec x | - | csc x | * 0 => 1 cosx 1 senx I senxI , . . J1 * 1 => | tan x | * 1 => tan x * ± 1 |cosx| ti 371 5rt 7ti 4 ' T ; T ; T 71 => .-. x * (2<j + 1) ~; q G Z Por lo tanto de i) ii) iii), tendremos: lat Dominio Df = R - —, V k G Z 1 4 Funciones Trigonométricas
10.- Para que la fundón secante exista o se encuentre definida, se debe cumplir que: 71 71 3x + ~ *(2k + 1) — O 2 kn ti — + — 3 9 „ 71 71 => 3x * kit +--— 2 6 71 => x*-(3fc + l) Df=R-{|(2A+l);fcc-z} RANGO DE LAS F.T 11.- Analizando el denominador de la fracdón, se deduce que: sen x * 1 Con el propósito de construir la función, empezamos por: - 1 < sen x < 1 => -2 < sen x - 1 < 0 Invirtiendo: ______ 2 senx-1 ~ '2 1 Multiplicando por 3: 3 3 senx-1 ~ 2 3 “</«<- - / 3 Ran/= 12.- Nuestra estrategia será construir la función dada a partir del dominio establecido. 71 6 De (*) construimos/ veamos: Multiplicando por 2: 1 < 2 sen x < 2 Sumando 5: Dividiendo por 3: 6 < 2 sen x + 5 < 7 2senx + 5 7 13.- Analizando el denominador de «f», reconocemos que:sen x * 1 (1) Para que/esté definida en R, se debe cumplir que: | sen x | - 1 > 0 | sen x | >1_(2) R17 518 Problemas de Trigonometría y cómo resolverlos RACSO DITOKII
De (2) podemos deducir que: *) | sen x | > 1 ; es absurdo *) | sen x | = 1 => sen x = 1; no puede ser por (1) sen x = -1 (✓) sen x = - 1 VI-1 0 Luego: fix) = _ = — = 0 Ran/={0J 14.- De la definición de valor absoluto se puede establecer que: 2 2 sen x = | sen x | Entonces al reemplazar en la función dada, ésta queda así: f{x) = | sen x |2 + 31 sen x | Completando cuadrados: fty = (|senx|+^j - ~ ...(1) Para determinar el rango de f, reconstruimos para dar la forma de (1). Veamos: Por teoría sabemos que: 0 < | sen x | < 1 3 3 3 5 Sumando ~ a cada miembro: ~ < | sen x | + ~ < — 9 / o\2 25 Elevando al cuadrado: ~ < 11 senx | I < — Restando 0< (|senx|+^j <4 => 0 < f < 4 ' 7 Ran/=[0;4] 15.- Efectuando la división indicada, se tiene: ,, % cosx + 1 + 1 , 1 f(x) =----------= 1 +---------- cosx + 1 cosx + 1 De donde deducimos que: cos x * - 1 A continuación reconstruimos la función, de: -1 < cos x < 1 => 0 < cos x + 1 < 2 1 1 Invirtiendo: — <---------- < + ~ 2 cosx+1 Funciones Trigonométricas R17 519
Sumando 1: 3 1 — < 1 + --------- < + oo 2 cosx +1 3 3 — </(x) < + “ Ran/= , — ; +— 16.- De la fracción dada, se puede deducir que: | sen x | * 1 2 2 2 Recordemos la identidad pitagórica: cos x = 1 - sen x = 1 - | sen x | Reemplazando en la función dada, se tendrá -(|senx|2-l) 7 |senx|-l /(x) = -(|senx1+1X1 senx|-l) | senx | -1 Cancelando: | sen x | - 1 * 0 => | sen x | *!...(•) Luego se obtiene: f(x) = - (| sen x | + 1) Por teoría se sabe que: 0 < | sen x | < 1 Y considerando (*), deducimos que: 0 < | sen x | <1__(**) A partir de esta relación reconstruimos f, veamos: sumando 1: 1 < | sen x | + 1 < 2 => -2 </(x) < -1 Ranf = <-2;-l] 17.- En primer lugar transformamos convenientemente la expresión dada y luego re- construimos la función para darle la forma obtenida. Veamos: Si: /(x) = cos x + 4 cos x + 7 Completando cuadrados: /(x) = (cos x + 2)2 + 3... (*) Por teoría se sabe que: -1 < cos x < 1 Construyendo f. 1 < cos x + 2 < 3 Elevando al cuadrado: 1 < (cos x + 2) <9 Sumando 3: 4 < (cos x + 2)2 + 3 < 12 Yde(*): 4</(x)<12 .-. Ran/=[4;12] 2 2 18.- Recordemos la identidad: cos 2x = cos x - sen x cos2x-sen2x (cosx + sen x)(cgSJ>-sefíxy Reemplazando en /: ftx) =----------- =---------------------- R17 520 Problemas de Trigonometría y cómo resolverlos -íí RACSO
Esta simplificación es posible si: sen x * cos x => x * — + kn Luego, nos queda: fix) = cos x + sen x => fix) = Jz sen (x + j Como: x*~ + kn => x + — * — + kn=> sen(x + ^j *sen^ + kjtj => sen(x + ^j *±1...(*) Por teoría se sabe que: - 1 < sen ^x + ~ j <1 Pero de (*): -1 < sen ^x + ~ j < 1 r V x e R - | ^ + kn, k g Z Multiplicando por 4z: -4z < 4z sen^x + ~j <Jz => -Jz <fíx)< 4z Kan/= '-J2; V2) 19.-Transformamos la expresión de f completando cuadrados: fix) = 4 cos2(nx) - 4 cos (nx) => /(x) = [2 cos(itx) - l]2 - 1... (*) 4 Del dato: — < x < 3 => 4n — <7IX<37I => -1 < cos(nx) < 1 De esta relación reconstruimos f, así: Multiplicando por 2: -2 < 2 cos (nx) < 2 Restando 1: - 3 < 2 cos(itx) -1 < 1 Elevando al cuadrado: 0 < [2 cos (nx) - 1] <9 Restando 1: - 1 < [2 cos(itx) - l]2 < 8 De(*): -l</<8 Ran/=[-l’8) 20.- Nuestra estrategia consistirá en transformar la expresión hasta reducirla. Luego analizar la variación de la expresión obtenida. Veamos: 2 2 2 2 Factorizando: f(x) = sen x. cos x (sen x + eos x) 1 2 2 2 Multiplicando por 4: 4/(x) = 4 sen x. cos x = (2 sen x.cos x) sen2x Funciones Trigonométricas
± 2 Despejando se obtiene que: f(x) = — . sen 2x ...(*) 2 Sabemos que para el caso del seno se verifica que: 0 < sen 2x < 1, Vi e R 1 Multiplicando por ~: 1 2 1 0 < ~ sen 2x < — 4 4 0</(x)<| Y de (*): Ran/= 0 2 4 21 .- Nuestra estrategia consistirá en transformar la expresión dada y analizar lo obtenido: Transformamos a producto el numerador de f. senx + sen3x + sen2x 2sen2x. cos x + sen2x sertíx(2 cos x +1) *x)= serílx =* -----J^Tx------ kn Analizando el denominador: sen 2x * 0 => 2x*kn => i / y , i e Z Simplificando "sen 2x", nos queda: fix) = 2 cos x + 1 kn Esto significa que: x * ~ => cos x * (-1; 1; 0} Luego: - 1 < cos x < 0 u 0 < cos x < 1 Construyendo f con cada desigualdad, obtendremos el rango. Veamos: Multiplicando por 2: -2 < 2 cos x < 0 v 0 < 2 cos x < 2 Sumando 1: -1 < 2 cos x + 1 < 1 v 1< 2cos x + 1 < 3 Ran/=(-l;l) u (1; 3 ) 22 .- Nuestra estrategia consistirá en transformar la expresión dada hasta obtener una en términos de una sola F.T.. Veamos: fto = 7 1 + tan x + 2tanx o 1 + tan x Separando en 2 fracciones: 2tanx 2tanx fíx) =~ ~ 2~ +3 = 4+ ~ ~ 1 JL+tan^x 1 + tan x 1 + tan x De lo cual podemos concluir que: /(x) = 4 + sen 2x Problemas de Trigonometría y cómo resolverlos RACSO IDITO1II
Del dato: n < x < — 2ti < 2x < 3ti => 0 < sen 2* < 1 ... (*) Partiendo de (*) construyamos f. 4 < 4 + sen 2x < 5 => 4 <f < 5 Ran/={4:5] 23 .- Nuestra estrategia consistirá en graficar cada termino por separado y luego anali- zar la variación de/en el intervalo dado. Veamos: a) Graficamos: yr = | tan x | 71 71 71 Como: |x|<— => - ~<x< — En este intervalo se verifica que: 0< |tan*| < J3 ... (1) b) Graficamos: y2 = | sen x | 71 71 En el intervalo: - — < x < — 73 Se verifica que: 0 < | sen x | < ... (2) Podemos sumar (1) + (2): 3V3 0 < | sen x | + | tan x | < =* 0</<^ Ran/= [0; 24 .- Como en los ejercicios anteriores, nuestra estrategia consistirá en transformar la expresión dada. Luego de la F.T. que aparece en la expresión obtenida, reconstruimos la función a partir del dominio de la F.T. referida. Veamos: 7 7 Completando cuadrados: /(*) = (tan x + 2) + 3 ... (*) Como: tan2* > 0 construyamos / tan4* + 2 > 2 Elevando al cuadrado: (tan * + 2) >4 Sumando 3: (tan2* + 2)2 + 3 > 7 => /(x)>7 Ran / = [7; + <») Funciones Trigonométricas
25 .- Teniendo en cuenta que x g IIIC; analicemos cada término de «f» en el intervalo dado. Veamos: a) Si: -g- < x < => sen-g- > sen x > sen 4? 1 -J*í r~ => -2>senx>-:5^ => - 1 > 2senx >--¿2 ... (1) . .7n 5n . 7ji , , 5n b)Si: -g- <x< => tan“g“ ctanxctan-^- => < tanx < 1 => - >/3 >-3 tanx>-3 ...(2) c) Sumando (1) y (2) m. a m.: ->/3 - 1 > 2 senx - 3 tan x > - 3--J2 => --J3-1 >/(x) > -3-^2 => -3-V2 </(x) < ->/3-l R/<x) = <-^ -3;- V3 -1> 26 .- Transformamos la expresión desarrollando la potencia indicada: /(x) = tan2x - 2^afí x. póf x + cot2x f(x) = tan2x+ —-2 , tan x > 2 1 Recordando que: «+ — >2, sícgR => fix) >0 .-. Rf = [ 0; + “ ) 27 .- Reconociendo que x g IIIC; procedemos tal como se hizo en el prob. 28. Veamos: 5ít 4n tan— < tan x < tan — => 1 < tan x < ,/3 ... (1) 5n 4ti COS— <COSX < COS —=> 1 2 -(2) Sumando: (1) + (2) , tenemos: V2 1 1----< tan x + cos x < -J3 - — 2 2 2 2-V2 a x 2,/3-l Rí(v)~ '2-5/2 25/3-11 2'2, R17 524 Problemas de Trigonometría y cómo resolverlos RACSO PlOITOllI
28 .- Para determinar el rango, redefinimos la función f usando identidades del arco simple y arco doble. Veamos: fix) = tan x + cot x f(x) = sec x . csc x 2 => /(*) = ñ------- =* fa) = 2 esc 2x... (1) J 2senx.cosx Del dato:— < x < < 2x < ..(2) a Analizando la gráf ca de la cosecante reconoce- mos que ésta, en el intervalo dado por (2), varía así: l<csca<2 => 2< 2csc2x <4 -• R«x)=[2;4] 29.- Para que la función f este definida en R, la tangente debe estar bien definida en R para lo cual se debe cumplir que: x/(2í+l)-,le Z => DomJ = R-(2k + 1)~, V le Z senx f(x) = cos x.-- = sen x J cosx senx *) Como: tan x = -- , cos x -* 0 cosx Como.xe Z-(2k+1) — =>-l<senx<l => Ran/=(-l;l) Luego al intersectar estos intervalos, se tendrá que: Df n Rf = (-1; 1> 30.- La única restricción proviene de la cosecante, donde: cscx3=>x*kn,kG Z analizando por cuadrantes se tiene: 1)Síxg ICóIIC a) | sen x | = sen x; pues sen x > 0 b) |csc x| = csc x . pues csc x > 0 .'. /(x) = genx csc x + cscx senx = 2 i i 2)Síxg IIICóIVC c) | sen x | = -sen x, pues sen x < 0 d) |cscx| =-csc x, pues cscx <0 fíx)= (-senx).cscx + (-cscx).senx =-2 -i -i Funciones Trigonométricas
71 71 371 3) Si x = (2n + 1)— = - , —,... Evaluando f en dichos puntos se tiene que: 71 I 71 CSC ~ + CSC — 71 sen — =2 • 3n CSC— + I 3ti| R* 2I 371 sen— = -2 De (1), (2) y (3) concluimos que: Rf = 1-2; 2} 31.- Analizando por separado cada sumando para las posibles restricciones. i) El sen x, no tiene restricciones pues x g R ii) La cot x y la csc x existen si: x rat; n g Z iii) También se debe cumplir: sen x - 1 > 0 Esta desigualdad implica que: sen x = 1 sen x > 1 absurdo 71 5ti r= 2; ~2 ; • Es decir x solo puede tomar valores dados por: x = (4fc+ 1)“ ,1g Z Df= 1(4fc + l)^LfcG Z iv) Para determinar el rango evaluemos en cualquier punto de su dominio, pues f es 71 periódico en ti, por ejemplo en x = —. 71 71 I 71 71 Veamos: /(x) = sen — +cot— + Jsen— -1 +csc~ ftx) = 1 + 0 + 0 + 1 = 2 Rango Rf = {2} 32.- i) Para que la función f exista, se debe cumplir que el radicando sea positivo o igual a cero. .2 R17 [5261 Problemas de Trigonometría y cómo resolverlos
De esta desigualdad se desprenden dos soluciones posibles: x<-lux>l Df = (-«>;-1) n [1;+ “) ...(1) 71 ii) De la excecante se sabe que- ex - sec(x) <- 2 ex - sec(x) >0, Vx * (2J: + 1) — ,ke Z => | ex - sec(x) | > 2 | ex - sec(x) | > 0 => | ex - sec(x) | > 0 Rg e [0; + ~) ... (2) De (1) y (2) se concluye que: Df n Rg = [1; +«») VALORES MÁXIMOS Y MÍNIMOS DE LAS F.T. DIRECTAS 33.- Por teoría sabemos que. -1 < sen x < 1 ... (*) => - 3 < sen x - 2 < - 1 => | sen x - 21 = - (sen x - 2) => | sen x - 21 = - sen x + 2 senx — 2 1 Reemplazando en fi f(x) = 3^ = 3_senr - 1 • - (*) Partiendo de (*) construimos/ Multiplicamos por (-1): - 1 < - sen x < 1 Sumamos 3 a cada miembro: 2 < 3 - sen x < 4 1 1 1 Invirtiendo: ~ ~ < ~ 4 3 - senx 2 3 1 1 Restando 1: ---------- 1 < _ 4 3 - senx 2 3 1 1 Y de (**) establecemos que: - - </(x) < - - /míx = - ~ 34.- Recordemos la identidad de transformación de producto a suma: 2 sen A. cos B = sen(A + B) + sen(A - B), A > B Aplicándolo en la expresión dada, se tendrá: 2 sen 7x . cos 3x = sen lOx + sen 4x Reemplazando en//x) = sen lOx + >efí 4x - >efí 4x - 5 => /(x) = sen lOx - 5... (*) Por teoría se sabe que: -1 < sen lOx < 1, V x e R Restando 5: - 6 < sen 10 x - 5 < - 4 Funciones Trigonométricas
Y de (*) concluimos que: - 6 </(*) < - 4 fmáx+ 2fmB = -4 + 2(-6) j/rrin ^|/max| -- /máx + 2/.nin=16 35 .- Aplicando el producto notable de suma por diferencia de un binomio, se tendrá: /(r) = 10 - sen2* ...(*) 2 Teniendo en cuenta que- 0 < sen x< 1, V re R Construimos la función / para lo cual procedemos así: 2 Multiplicando por (- 1): -1 < -sen x < 0 Sumando "10": 9 < 10 - sen2* < 10 De (*) se deduce que: 9 </(*) <10 .-. /míx = 10 4 4 3 1 36 .- Recordemos la identidad: sen x + cos x = — + ~. cos 4* 4 4 3 1 Aplicando esta identidad en f. f(x) = ~ + —. cos 4*... (*) Sabiendo que: -1 < cos 4* < 1, V x e R Construyamos la función f, para lo cual procedemos así: 111 1 Multiplicamos por ~~ ~ cos 4* < ~ 3 13 1 Sumando ~ ~ +• ~ cos 4* < 1 4 2 4 4 1 Y de (*) se establece que: ~ </*) < 1 1 3 Luego: tn = — a M = 1 m + M= 37 .- Redefinimos la función f, completando cuadrados fix) = 35- (tan2* - 4 tan * - 4) + 4 => /(*) = 39 - (tan * - 2)2 Para obtener el valor máximo de la función/ se debe cumplir: (tan * - 2)2 debe ser mínimo, es decir cero. Luego: tan *-2 = 0 => tan * = 2 => * — 63°30' Problemas de Trigonometría y cómo resolverlos •jA RACSO ¡P IDITOMBB
38 .-Nuestra estrategia consistirá en transformar la expresión en una ecuación cuadrática 2 Efectuando: y = 8 + 2 tan x - tan x y => tan x - 2 tan x + (y - 8) = 0 (Ec. cuadrática) 2 Para que "tan x " esté definida se debe cumplir que: A > 0 (b - 4ac > 0) Es decir: b = - 2, a = 1, c =y - 8 =» 4-4(y-8)>0 =» l>y-8 =» y<9 Pero: y =f(x) => f(x)<9 fmíx = ^ J 39 .- Transformamos la expresión completando cuadrados: f(x) = (tan a + 1) - 1 71 De acuerdo con la teoría podemos afirmar que: -<» < tan x + 1 < + x * (2J: + 1) — 2 Elevando al cuadrado: 0 < (tan x + 1) < + «> 9 Restando 1: -1 <(tanx + 1) - 1 < + «> => -1</(x)< + ~ fmín = -l 40 .- Analicemos f(x) con las gráficas de las funciones tan x y cot x, y en el intervalo de su definición: 41 .- Del dato x g IC y x = 0 sen x es creciente a tan x es creciente Ahora analizamos cada función en el intervalo dado: 71 Si 0 < x < — 7* sen(0) < sen x < sen — => 0 < sen x < 1 ... (1) \ 71 tan(0) < tan x < tan — => 0 < tan x < + °°... (2) Funciones Trigonométricas R17 529
Entonces (1) + (2), tenemos: 0 < sen x + tan x < + ~ 0 < f(x) < + ~ 42 .- Analicemos con las gráficas de las funciones sec x y csc x. Del gráfico se observa que éstos se intersectan en: 5rt x= T Además: | sec x | < | csc x | ...(*) Comore IIIC => | sec x | = -sec x a | csc x | = -csc x En (*): - sec x < - csc x => sec x - csc x f(x)>0 PERÍODO DE F. T DIRECTAS 43 .- Usando la regla práctica de cálculo del período mínimo, tendremos: 2n 2ti 2n T x = ~í- = T/ = -¡ = => T/ = -i — 8tt (senA) 1 sen| 1 sení 1 2 3 4 => Tf = m.c.m (4n; 6n; 8tt) = 24ti Tmfn = 24n 44 .- Usando la regla práctica de cálculo del período mínimo, tendremos: T =— =— =* tÍcos—1 = — T(sen3x) 3 A T(cosÉÜ) 6 T(C°S 5 / 3 2n 5n A T'T J Tf = m.c.m m.c.m(27i;5n) lOrt m.c.d.(3;3) - 3 10n 45 .- Usando la regla práctica para el calculo del periodo mínimo, tendremos para cada función: a)Tf = 2n a 2n b)T8=T 2n ti ~a = 7 2ti ti T = 6 a = 14 fe = 12 a + b = 26 R17 530 Problemas de Trigonometría y cómo resolverlos 'áLRACSO 'Mbditokes
46.- Sabemos que/(x) es periódica si 3 T * 0, tal que: f(x + T) = f(x), Vx G Dom f, x + T G Dom/ Como: fix) = cos(ti cos x) => f(x + T) = cos(ti cos (x + T)) => f(x + T) = cos(ti cos x . cos T - ti sen x . sen T) Pero por definición se debe cumplir que: f(x + T) = f(x) Entonces se debe cumplir que: ti cos x. cos T - ti sen x . sen T = ti cos x => (cos T = 1 v cos T = - 1) a (sen T = 0) => T = kn,ke Z T - ’rmti = n 47.- Analicemos cada caso por separado. Veamos: a) La función/es de la forma: f(x) = tann(Bx) 71 => Si "n" es par ó impar => el período T = — 2 n 9ti Reemplazando valores: n = 15 a B = — => Tf= -^ => Tf= — 9 b) La función g es de la forma: g(x) = secn(Bji) 71 => Si "n" es par, el período es: T = — 71 Reemplazando datos, tendremos: n = 10 a B = 4 => Tg = — c) La función h es de la forma: h(x) = cscn(Bx) 2ti => Si "n” es impar, Th - — Reemplazando datos se tendrá: 3 2ti 14jt Sím = 3 a B=-,Th=y Th=1F 7 Funciones Trigonométricas
48.- Agrupamos convenientemente en factores las funciones y, cofunciones y luego de aplicar las identidades trigonométricas, tendremos: f(x) = (sen x.csc x) (eos x . sec x) (tanx. cotx) i i i 1 1 1 71 kn SÍ X * H7I si x * (2m + 1)~ six* — kn /(x) = 1; si: x * — Graficando la función /redefinida, tendremos: Finalmente del gráfico se observa que: V I I 71 3x -Tt T1 -- - ------------------- 1 f (mínimo)— 2 2 49.- Debemos redefinir la función f por medio de las identidades trigonométricas. 2 2 1 f(x) = sec x . csc x + sec x . csc x + — Y reconociendo que esta expresión corresponde a un binomio cuadrado perfecto, tendremos: f(x) = (secx.cscx + jj --.(*) , 2 2 Donde identificamos que: sec x. csc x = 7----------=--------- => sec x. csc x = 2 csc 2x 2senx.cosx sen2x Luego en (*) tendremos que la función queda así: /M = (2 csc 2x + |) Aplicando la regla práctica para calcular el período de la cosecante, tendremos: 0» tnín — 2 <TAnin-B GRÁFICAS DE LAS F.T. DIRECTAS 50.- Podemos transformar la función dada, factorizando así: f(x) = -Jsen2 x(l - sen2 x) Usando identidades tenemos: /(x) = vsen2x.cos2 x 1 j 2 Multiplicando y dividiendo por 4: f(x) = — (2senx.cosx) V 4 R17 532 Problemas de Trigonometría y cómo resolverlos 4^ RACSO
Pero: 2 sen x . cos x = sen 2x, se tendrá: I 1 7 .sen 2x Luego nos queda: 1 f(x) = ~ |sen2x| 1 De aquí se observa que la amplitud de/es: A = y El periodo de fes: Luego el gráfico será: 51.- Como: Por dato: Haciendo (1) = (2): P g (y = sen x) => 2a + b = sen xt ... (1) Q g (y = cos x) => a - b = cos x2 ... (2) 71 X1 + X2 = =* sen = COS *2 2a + b =a-b => a = -2b ...(*) AL , a+í} -2b + b -b 1 Ahora reemplazamos () en M: M = ~ "¿fe—b = —3b M = y 52 .- Para que la función/esté definida en R la "tan x" debe estar adecuadamente defini- da, es decir: x*(2k+l)y,itG Z ...(1) Asimismo, los denominadores de /deben ser no nulos, es decir: 71 cos x * 0 => x * (2k + 1) ~ , k g Z ... (2) De (1) y (2) verificamos que ni tan x ni cos x se deben valuar en múltiplos impares de n/2. A continuación, usando identidades, reducimos /(x): /(x) = ^-^x ' fióSX 2senx.cdsx => /(x) = | sen x | -21 sen x | 71 /(x) = -1 sen x | , x * (2k + 1)— Y elaborando la gráfica correspondiente se obtiene: Funciones Trigonométricas R17 533
53 .- Analizando el denominador de la fracción se puede establecer que: sen x * 0 => x * kn, V k g Z El numerador transformado a un producto, nos permite obtener: 2>efíxcos2x pefíx De este modo el período de: 2n f(x) = 2 cos 2x, es: T = — => T= n Asimismo identificamos la amplitud de la función: A = 2 De este modo el gráfico será así: 54 .- Cada vez que la gráfica de /intersecta al eje x:f(x) = 0, es decir: Jx - 41 cos x | = 0 => Jx = 41 cos x | Entonces basta con encontrar el número de intersecciones de las gráficas: yx = Jx ^y-¿- 41 cos x |, en el intervalo: x G ( 0 ; 3ti) Elaboramos las gráficas para yx e y2, observándose que la amplitud de y2 es 4 y su período es n. Si además se verifica que V3n < 4; se concluye que: # de intersecciones : 6 R17 534 Problemas de Trigonometría y cómo resolverlos RACSO
55 .- Si la gráfica de la función f intersecta al eje x, entonces: f(x) = 0 => 21 cos x | - | sec x | = 0 2|cosx| = |secx| .. .(1) Sea: m(x) = 21 cos x | y n(x) = | sec x |, cu- yas gráficas se muestran: Se puede observar que los puntos P, Q, R, S tienen igual ordenada, lo cual resuelve la ecuación (1) Son 4 punto* x 56 .- Analizamos la función en dos intervalos. [0; n] y (n; 2n}. Veamos: a) Si: x g [0; n] => | sen x | = sen x => f(x) = 2 sen x Como: 0 < sen x < 1 , V x g [0; it] b) Si: x g ( n; 2ti) => | sen x | = -sen x => f(x) = - sen x + sen x => /(*) = 0 Graficamos en [0 ; 2n] 57 .- El gráfico c rresponde a un sinusoide de la forma: y = A sen Bx + C, y cuyas carac- terísticas identificaremos a continuación: 1 a) La amplitud: A = ~ (6 - (-2)) = 4 2ti 1 b) El período: T = 6n => ~ = 6n > B = ~ d 3 c) Asimismo se observa que la función se ha desplazado 2 u hacia arriba. Luego: C = 2 x y = 4 sen ~ + 2 3 3 58 .- Del gráfico se puede tabular la función: y = A + B cos x Funciones Trigonométri as
a) 1 = A + B A = 1... (1) b) 3 = A + B cos ti => 3 = A - B ... (2) De (1) en (2): 3 = 1 - B => B = -2 2A-B = 4 59.- De la función: y = 3 sen~ podemos reconocer que la Asimismo, se puede determinar el período (mínimo): — 2ti T = -j- => T = 4ti 2 Luego trasladando estos valores al gráfi- co dado, se tiene que el área de la región triangular sombreada tiene por base: 2ti y por altura: 6- Luego: 1 7 S= j(2ñ)(6)n2 S = 6 n u 60.- Tratándose de un trapecio defini- do por su base, que es el período de la función dada, y por su altura que es 2-J3 , lo que hacemos es determinar el período de la función dada, para lo cual utilizamos la fórmula básica: Si: fM = tan 4x => => Área = base x altura T = ti/4 61.- X y i n 3 amplitud es: A = 3 MISCELÁNEA f(x) = sen x . cos x Multiplicando por 2: 2/(x) = 2 sen x . cos x 1 Pero: 2 sen x . cos x = sen 2x => f(x) = ~. sen 2x____(*) R17 536 Problemas de Trigonometría y cómo resolverlos RACSO BDITOKR1
a) Evaluamos la función en x = 0 a x = ti/4: 1 7t 1 ti 1 /(O) = - sen 0 = 0 a ) = - sen - = ~ Como: 0<x<~ a f(0) </(^) => /es creciente en ^0; I es (V) n 1 1 „ ,, x 1 b) Como: 0 < x < — => 0 < sen 2x < 1 => 0<~sen2x<~ => 0 </(*)< _ 4 2 2 2 / 1 \ => Ran/= ( 0 ; ~ j II es (F) 2ti c) De (*) deducimos el período de la función, por teoría: T = — = ti /. III es (F) 62 .- Nuestra estrategia consistirá en analizar la función a partir de los términos que determinan su dominio y rango utilizando para ello los cuadrantes comunes Veamos: a) Delimitación del dominio.- 71 i) La tan x 3 si x * (2tn + 1) ~; tn 6 Z ii) La cot x 3 si x * «ti; n e Z kn í. Til De (i) y (ii) x * ~ ;ke£=> Dominio Df = R ' p 2J G b) Identificación del rango. Si x g IC ó IIIC => | cot x | = cot x f(x) - tan x (cot x) => f(x) = 1 Si x g IIC ó IV C => | cot x | = - cot x fix) = tan x(- cot x) => /(x) = - 1 Rango Df = (-1; 1} Ahora elaboramos el gráfico según los resultados obtenidos: Del gráfico se observa que el período mínimo es ti . Funciones Trigonométricas
63 .- Analizando los puntos de discontinuidad para la función secante y cosecante, tenemos: I) La secante no se encuentra definida si: ti n ti ti kn ti 2x+— = (2k+1)—;kG Z => 2x = kn.+ ~ => x~ + ~Z 6 z z 6 6 6 II) La cosecante no se encuentra definida si: ti mu ti 2x + — = mu ; pig Z => x = - — 3 2 6 Por lo tanto de (I) y (II) se concluye que: nn n „ x=~2 ± -¡^ ;neZ 64 .- Se presenta una sola restricción, la cscx 3 si x * kn,ke Z .-. Df = R - (kn ; k g Z Del dominio, tenemos que: sen x . csc x = 1 => f(x) = cos( | x |) Además podemos reconocer que: f(-x) = cos | - x | = cos | x | = f(x) La función fes par. Ahora analizamos la función por intervalos. Veamos: i) Si x > 0 =>f(x) = cos (| x |) = cos (x) ii) Si x < 0 =>f(x) = cos( | x |) = cos (- x) = cos x Graficando la función f por cuadrantes y en base a la gráfica de la función coseno se tiene: 65 .- Analizando las restricciones en el numerador y denominador tenemos: i) En el numerador y denominador, la cot x 3 si: x * >m; n g Z RACSO DITOM1 R17 538 Problemas de Trigonometría y cómo resolverlos
ii) En el denominador, la función/3 si cot x 0 => x (2m + 1)~ ,me Z Jc7t De (i) y (ii): x * — ; k e Z => D( = R-|^},iteZ iii) Redefiniendo la función /y recordando que: a, si a>0 a, si a<0 tendremos: a) Si x g IC ó IIIC => cot x > 0 => | cot x = cot x cosx.cotx => fíx) = -----7---- = COS X J ’ cotx b) Si x g IIC ó IVC => cot x < 0 => cosx.cotx I cot X I = - cot X => fíx) = — -—— = - COS X 1 J (-cotx) Graficando la función / por cuadrantes y en base a la gráfica de la función coseno se tiene: /x) “ cos x Rf = < -1; 0) u ( 0; 1> Del gráfico se observa que: 66.- Para que la función / sea no negativa se debe cumplir que: tan2x - 3 > 0 => tan2x > 3 => | tan x | > V3 A continuación elaboramos la gráfica de: y= |tanx| ey= Jí en ^“2'2/ Podemos reconocer, que solo la región sombreada corresponde a la relación: | tan x | > V3 Lo cual ocurre solo en los intervalos: / 71 -71 |tanx| >3,en\ — 2'2 Funciones Trigonométricas
71 2 71 71 3 A 3 71 2 / 71 71 71 71 Dom/= ;jJ u 67.- Para que la función/no esté definida, se debe cumplir: | sen x | - | cos x | < 0 => | sen x | < | cos x |, [71 7jt¡ , , "^"Jz la relación: | sen x | < | cos x | Se verifica en el intervalo: /no es definida en x G ti 7n Í'T VxG 68.- De la condición del problema se debe cumplir que: f(x) >0 => cot x - tan x > 0 .-. cot X > tan x La desigualdad la analizamos con la gráfica de la función tangente y cotangente. Reconocemos que el punto de intersección está 71 en: x= ~ 4 Del gráfico de observa que: x G 69.- Según la condición del problema tenemos que: /(x)>0 => |csc x| - |sec x| > 0 .-. |cscx| > |secx| Por las identidades conocidas se tendrá que: 1 > 1 1 > 1 senx _ cosx |senx| - |cosx| De esta relación deducimos que: sen x * 0 => x * 0; ti |senx| Luego: -1 => | tan x | < 1 > - 1 < tan x < 1 I A I cos 0 => 71 ** 2 La desigualdad la analizamos con la gráfica de la función tangente. Problemas de Trigonometría y cómo resolverlos RACSO ÍDIIO»»i
71 i) tan x = 1 => x = ~ ' 4 3ti ii) tanx = -1 => x = — 4 De la gráfica se observa que: xe ¿°;t] u \ 4J L 4 / 70.- Según condición del problema se tiene que: /(x) > 0 => 3 tan x - 2 sen 2x > 0 3 tan x > 2 sen 2x La solución de esta desigualdad la resolveremos primero en forma analítica resolviendo una ecuación trigonométrica elemental para hallar los puntos de intersección de las gráficas de: h(x) = 3 tan x a m(x) = 2 sen 2x Para luego determinar con las gráficas de dichas funciones los intervalos posibles de solución para la desigualdad: h(x) > m(x) I) 3tan x = 2 sen 2x => =2.2 >efí x cos x cosx Al simplificar sen x, debemos reconocer que ello encierra una solución: sen x = 0 a 2 — = 2 cos x De donde deducimos que: 3 => x = 0 a — =2 cos 2x 3 — = 1 + cos 2x 1 71 5tü => cos2x= 2 => 2x=3'-^ ti 5tt II) Ahora graficamos las funciones h(x) y m(x), y tendremos: Como: h(x) > m(x), entonces se debe cum- plir que: Funciones Trigonométricas
Inversas DOMINIO DE LAS F.T. INVERSAS 01.- Como: 0 < V3x-1 <1 => 0<3x-l <1 Sumando 1 a cada miembro: 1 < 3x < 2 Dividiendo entre 3: [1 ; f] 02.- Para que/esté definida, se debe cumplir: arc sen x - arc cos x > 0 => arc sen x > arc cos x Del gráfico se observa que: arc sen x > are cos x En el intervalo: x G Nota: En el punto de corte se verifica que: arc sen x = arc cos x = tt/4 a x = -J2 /2 03.- Analizaremos la función a partir de sus componentes. Hacemos /i = 2 arc sen A arc cos (^3^^) Dom/= Dom/j n Dom/2 Determinaremos los dominios de cada función componente: a) fi = 2arc sen 1 Multiplicando por 2: -2 < x - 1 < 2 Sumando 1: -l<x<3 => Dom/1 = [-1;3] R13 542 Problemas de Trigonometría y cómo resolverlos 3Í< RACSO IDITOÍll
... , „ Í2x+1\ 2x+l .. b) /2 = 3 arc eos, I—3— I => -1 < —3— < 1 Multiplicando por 3: -3 < 2x + 1 < 3 Restando 1: -4< 2x < 2 Dividiendo entre 2: -2 < x < 1 Dom/2 = [-2 ; 1] Dom/= |-1; 3] [-2; 1] .-. Df = [-1,-1] 04.- Lo primero que reconocemos es que la función/no está definida si: 3^ -arc cos x = 0 71 => are cos x = 3 71 x = cos 3 1 x- 2 05.- La función/la estudiaremos a partir de sus componentes por lo cual hacemos: /j(x) = 3 arc sen I —-— I a f2(x) = arc tan (x - 2) => Dom/= Dom/ n Domf2 Determinaremos los dominios de cada función componente. a) /1W = 3 arc sen 1) Multiplicando por 4: Sumando 1 a cada miembro: 5 Dividiendo entre 3: -1 < x < 3 -4<3x-l <4 -3<3x<5 Dom/ = -1; 5 3 b) /2(x) = arc tan(x-2),“><x-2< + «> Domf2 = IR Luego: Dom/= j n IR 06.- La función arco dada depende directamente del arc sec, por lo cual estará definida si se cumple: 4x-3g <-oo;-l] [l;+°°> => 4x-3<-l vj 4x-3>l 4xs <2 vj 4x > 4 x< 2 v x>l Dominio (““> 2 u 11; + 00) Funciones Trigonométricas Inversas
07.- En la aplicación de la definición del arc sec, se puede establecer que: arc sec (x+^j 6 / 1 \ 1 1 Siempre que: lx+^ I g <-«>; -1] u [1; +«>) => -o° < x + — <-l vi < x + - + ~ 1 31 / 3 1 [1 Restando “: °°< x<-~ v ~ <x < +°° .'. Dom/= (-«>;-£ u 2; + °° 08.- Para que el arco secante dado en el problema, se encuentre adecuadamente definido se debe cumplir que: 8/ - 1 > 1 => 1_ 2 09.- Para que el arco cosecante se encuentre definido, se debe cumplir que: 2x2-7<-1 x2<3 (x + V3)(x-V2)<0 -V3 <x< -J3 Df=(-oo;-2] CJ 2x2 - 7 > 1 x2>4 U (x + 2)(x-2)>0 u (x < -2 vj x > 2) U [-V3;V3]u [2; +°°) 10 .- Ana izaremos la función dada a partir del estudio de los dominios de: a) El arco secante se encuentra definido si: 8x + 3 < - 1 u 8x + 3 > 1 1 1 x<-~ cj x>--...(l) b) El arco cosecante se encuentra definido si: 5x - 2 < - 1 U 5x - 2 > 1 1 3 x<- V ...(2) A ACSU EDITOIII R18 544 Problemas de Trigonometría y cómo resolverlos
Graficamos los resultados obtenidos en (1) y (2) y al intersectarlos tendremos: V---------- T "í 1 1 2 4 5 5 .-. El dominio está indicado por las regiones sombreadas: 11 .- Analizamos cada sumando por separado y tenemos: (1) El are csc x 3 si x G (-“; - 1] cj [1; +•») (2) El are sen x 3 si x G [-1; 1] (3) El are tan x, no representa restricción, x G R Al intersectar estas tres soluciones encontramos solo dos elementos comunes a ellos: -1 a 1. Luego: Df = {-1; 1} RANGO DE LAS F. T. INVERSAS 12 .- El rango de la función depende de su dominio Sea: f¡(x) = 2 are sen (2x - 1) , f2(x) = 3 are sen (3x + 1) => Dom/(x) = Dom/^x) n Dom/2(x) a) En /j el dominio del are sen es: -1 < 2x - 1 < 1 Sumando 1: 0 < 2x < 2 Dividiendo entre 2: 0 < x < 1 .-. Dom/] = [0; 1] b) En f2 el dominio del are sen permite establecer que: -1 < 3x + 1 < 1 Restando 1: - 2 < 3x < 0 2 Dividiendo entre 3: - ~ < x < 0 .'. Dom/2 = [-2/3; 0] Para determinar el dominio de <</>> intersectamos las soluciones: Funciones Trigonométricas Inversas
> Dom/= [O; 1] o 2 -3;° => Df={0} Y para determinar el rango de «/» evaluamos en x = 0, obteniendo: /(Oj = 2 are sen (-1) + 3 are sen (1) Ran/. {$} 2 2 2 13 .-Como: |«| =g f(x) = | are sen x | + 4|orc senx| - 2 7i 2 Completando cuadrados: f(x) = (| are sen x | + 2) - 2 ti - 4 ... (*) 71 Según la teoría podemos establecer que: 0 < | are sen x | < y Sumando "2": Elevando al cuadrado: Sumando 2n - 4": 2 < | are sen x | + 2 < 2 +2 4<(|orcsenx| + 2)2< (^+2) -2n< (|crcsenx| + 2)2-2p-4 < Y de (*) podemos concluir que: -2tT</(x)< g2 Ran/= -2n; n2 4 14 .- Dado que «f» depende de are sen x a are cos x, podemos afirmar que: x e [-1; 1] De acuerdo con la identidad aditiva de las F.T.I.,/(x) se reduce a: f (x) = are sen x + n ...(*) 71 71 Como: <arc sen x< 2 » V xg [-1 ;1] „ . ti _ 3rt Sumando ti: < ti + are sen x < y De (*) se tendrá: -/(x) - Ran/ = 15.- Como: 0< Jx <1 0 < are cos Multiplicando por 2: 0< 2 are cos -Jx < ti Problemas de Trigonometría y cómo resolverlos i* RACSO IDITOtlI
Sumando — cada miembro: it ti 5rt — < — +2 arc cos -Jx < — 4 4 4 71 5rt 4 ~f~ T Ran/= 7t_57t 4' 4 16.- Analizando la variable del arc tan tendremos que ésta se puede transformar en: x2 + 2x + 2 = (x + l)2 + 1 Por definición del cuadrado de un número real, tendremos: 0< (x + l)2 < + ~ , V re R Sumando «1» a cada miembro de la desigualdad: •y 71 1 < (x + 1) + 1 < + «> => — < arc tan(jr + 2x + 2) < — 7L 2 Multiplicado por 2: — < 2 .are tan(x + 2x + 2) < ti 71 =» ~<f(x)<n Ran/= 17.- Según condición del problema tendremos que: 0 < x < 1 => 0< arc tan x < ~ Multiplicando por 2: 71 0 < 2 arc tan x < ~ Sumando rt/4: Tt Tt 371 — < — + 2 arc tan x < — 4 4 4 Tt 31t -</(*)< Y .-. Ran/ = /ti 3ti \4 ; T 18 .- Debemos recordar que: sen4x + cos4x = ~ + — cos 4x, x 6 R Evaluando esta expresión, encontramos su máximo: 1 y su mínimo: 1/2; luego: — < sen4x + cos4x < 1 Funciones Trigonométricas Inversas
Como la función arco cotangente es decreciente en el intervalo [1/2; 1], se tiene: arc cotí -i j > arc cot(sen4x + cos4x) > arc cot(l) Como: cot (12772) = 1/2 => = arc cot 127no n F 7t. 127?r~| =* 360 • ^-1.4 ' 360 ] 19 .- Efectuamos la división indicada de la variable del arc cot: x2 l+x2-l 1 1 + x2 1 + x2 1 1 + x2 Por definición del cuadrado de un número real: x2 V re IR Sumando «1» a cada miembro: 1 Tomando la inversa: 1 +r Multiplicando por «-!»: 1 1 + r Sumando «1» a cada miembro: 1 1 +x‘ Como la función arco cotangente es decreciente en el intervalo [0; 1) se tiene: arc cot (1) < arc cot 11-----I < arc cot (qj V 1 + x ) 71 71 /TI 71 => 4</(x)<- Rí=\4'‘ 20 .- Analizando cada sumando se tiene: i) El arc cos x está definida si x g [1; 1] ii) El arc cot x está definida si x g R Problemas de Trigonometría y cómo resolverlos .'‘•4 RACSO UlDITOUI
=> No hay restricción para la variable de la función «/», por lo tanto de (i) y (ii) al intersectar tendremos que el dominio de «f» es: Df = [-1; 1] Para determinar el rango, debemos tener en cuenta que el arc cos x y arc cot x son funcio- nes decrecientes en el intervalo I-i; 11- Veamos: l)Si:-l<x<l arc cos (-1) > arc cos x > arc cos (1) ti >arc cos x > 0 ... (1) 2) Si: - 1 < x < 1 arc cot(-l) > arc cot x > arc cot (1) ti-arc cot (1)> arc cotx> — => 3tt ti — > arc cot x > — ... (2) 4 4 De (1) + (2) tendremos: 7n ti — > arc cos x + arc cot x > — 4 ' 4 f(x) Rf= 7t_?7t 4' 4 21.- Para las funciones arco tangente y arco cotangente (are tan x, arc cot x) no hay restricciones lado que x G IR. Asimismo debemos recordar la identidad: n n arc tan x + arc cot x = => arc cot x = - arc tan x Sustituyendo en la función dada: f(x) = arc tan x 2 - are tan x Efectuando la multiplicación indicada: 2 Tt 7t^ 7t^ fíx) = - (are tan x) - % arc tan x + — - — L 16 16 _ Completando cuadrados: (71 are tan x - ~ I2 *2 + — I 16 Como: x> 1 arc tan x > arc tan 1 Dado que EL arco tangente es creciente en (1 ; <=>): Tt ( Tt V í Tt V arc tan x-~ > 0 => tare tanx. -7 >0 => -1 arc tanx- — <0 4 I 4 1 l 4 1 *TT A 2 — 2 2 i n' 71 I 71 71 „ _ / are tan x - y I + — < — R/= (-<»; 4 I 16 16 J \ 16 f(x) Funciones Trigonométricas Inversas R18 549
22.- Debemos recordar la identidad: sen x + cos x = V2 sen x + — , x G IR Si evaluamos el máximo y mínimo de esta relación encontraremos que: - -Jl < sen x + cos x < -Jl .. (1) A su vez, la función arco secante se encontrará definida si verifica que: » sen x + cos x < -1 u sen x + cos x > 1 ... (2) De (1) y (2) al intersectar los intervalos tendremos: - -J2 < sen x + cos x < - 1 vj 1 < sen x + cos x < -J2 Tener en cuenta que en [- Jl ; - 1] el arco secante es creciente, por otro lado en [1; J2 ] es creciente. Luego: are sec(- -Jz ) < are sec (sen x + cos x) < are sec (- 1) u are sec(l) < are sec (sen x + cos x) < are sec (-Jz ) 3n Tt y n o </(x) < - Tt 23 .- Tener en cuenta que are sec x se encuentra definida si: x G (-°°; -1 ] [1; +°°) Sea 6 = are sec x, también se debe cumplir: Tt 0 = are sec x g 0; — ;7t ...(1) Asimismo csc 6 se encuentra definida si: 0 * kn; keZ => 0*0;Tt; 2it En (1) tenemos: 6g (0; - la cosecante decrece la cosecante crece => csc 0 g (1; +“) u (1; +°°) Rf = (1 ; +°°) •jiRACSO WBDITOIEI 5501 Problemas de Trigonometría y cómo resolverlos
DOMINIO Y RANGO DE LAS F.T. INVERSAS 24 .- Recordemos que: - — < arc sen x < ~ <=> re [-1; 1] 2x-3 a) Analizando la variable del arc sen dado, debe cumplirse que: -1 < —z— < 1 Multiplicando po r 5: -5<2x-3<5 Sumando 3: Dividiendo entre 2: -2 < 2x < 8 -l<x<4 Dom/= [-1; 4] b) Reconocemos que: 71 ( < arc sen 1 ~3^ 7t 5 J" 2 2 Multiplicando por —: 71 — arcsen <2x-3'\ 7t k 5 J"3 71 Sumando “: 6 => 0< 7t 2 . arcsen D á 2 (2x-3 A 7t “ .flrc sen R + T 0 □ J o V K ¡ <N VI ti | ^0 + cñ l m ü|<N <N VI f(x) n 25 .- a) Para determinar el dominio analizamos la variable del arc sen: -1 < 3x - 2 < 1 Sumando 2: 1 < 3x < 3 1 F 1 Dividiendo entre 3: ~ <x<l Dom/= — ;1 0 L 0 7t 7t b) Para determinar el rango analizamos el arc sen dado: -~ < arc sen (3x - 2) < ~ 2 7t Multiplicado por —: - ~ 7t 7t Sumando 4 12 2 7t = — arc sen (3x - 2) < — 2 7t 7n < ~ arc sen (3x - 2) + — < — 7t 71 7 71 12 '• Ran/= 12 Funciones Trigonométricas Inversas
26.- Tal como hicimos en los casos anteriores haremos los cálculos así: 2 + x a) Analizamos la variable del are coseno, el cual verifica: -1 < —-— < 1 Multiplicando por 4: -4 < 2 + x < 4 Restando 2: -6 < x < 2 Dom f = [-6; 2] ( 2 + x^ b) Construimos la función a partir del are cos, el cual verifica: 0 < are cos I I < n 1 Multiplicando por —: 3rt Sumando —: O 3n 8 1 ¿+x 71 0 < , Jirc eos , < ~ 4 V 4 ) 4 3n 3n 1 f 2 + x^ 5rt T-T 4 arc cos[ 4 J-T 5ti i 3it 5ti <f(x)<— Ran/= | — 27.- Tal como hicimos en el problema anterior, analizamos por partes. Veamos: a) Recordemos que la variable del arc cot debe estar comprendido en el intervalo: -oo < x- 1 < >*> => -oo < x < + “ .'. Dom/=K b) Reconstruimos la función a partir de: 0 < arc cot(x - 1) < n Tt Tt Tt 5n Restando ~: -~ < arc cot(x- 1) - — < — 6 6 6 6 I Tt I 5n Tomando valor absoluto: 0 < Iarc cot(x - 1) - I <~ 5rt Ran/= 0; ~ L 6 28.- Es importante recordar que: arc sen(-x) = -arc sen (x) a arc csc (-x) = -arc csc (x) arc cos (-x) = Tt - arc cos (x) a arc sec (-x) = Tt - arc sec(x) Sustituyendo convenientemente en la función dada, está queda así: f(x) = -arc sen(x) + Tt - arc cos(x) - arc csc (x) + Tt - arc sec (x) R18 552 Problemas de Trigonometría y cómo resolverlos <á<RACSO
f (x) = -[are sen x + arc cos x] - [arc sec x + arc csc x] + 2tt ... (*) Si: x G [-1; 1] y si: x G (-«>; -1] U [1; + ~) a) De (*) podemos afirmar que- al) En el primer sumando: xg [-1; 1] a2) En el segundo sumando: x G (-»; -1] vj [1; +•») Luego el intervalo común se obtiene intersectando los encontrados: Df = {-1; 1} b) Si en (*) aplicamos las identidades correspondientes encontraremos que: /(x) = -^ - +271 => /(x) = 7t = 29.- Analizando cada sumando se tiene: i) El arc sen x está definido si: x G [-1; i] ii) El arc sec x está definido si: x G (-“>; -1] vj [1; +~) Por lo tanto de (i) y (ii) al intersectar obtenemos el dominio de la función: Df = {-1; 1} Para determinar el rango evaluamos para cada "x", del dominio, su respectivo valor f(x) 1) /(-l) = are sen (-1) + arc sec(-l) + 1 => f(-T) = - arc sen(l) + tt- arc sec(l) + 1 o /(-l) = - ~ + Tt + 1 =>f(-l)=^+l 2) f (1) = are sen (1) + arc sec (1) + 1 => /(l) = ~ + 1 o Como los valores obtenidos son idénticos, concluimos que: {Tt 2 +1J Funciones Trigonométricas Inversas
30.- La fundón induye el arco cosecante, la cual está definida si se cumple que: 3xg 1 '”3 i 3 2 3 2 .3 Conociendo el dominio se determina el rango, de tal manera que arc csc (3x) G 1 — arc csc (3x) G 1 ~ arc csc (3x) + 7t G f(x) Rf = 7t 2 TI 8 7n 8 ;7t TI 2 TI 8 9tt n; 8 Rango = Rf Jt, 8 Nota: Los despejes en (*) y (**) han sido directos, ¡verifícalos! PROPIEDADES DE LAS F.T. INVERSAS 31.- Hagamos una evaluación de cada término: a) a = are sen 1 7t sena= — => a= — 2 o b) P = arc tan (1) 7t 4 . . . (**) 8 c) 6 = arc cos (-1) => cos 0 = - 1 => 0 - n Reemplazando en la expresión dada tendremos: 71 71 — 4-- 6 4 M =------- 7t 5tt 7t 5 M=- RACSO Wediioiei R18 554 Problemas de Trigonometría y cómo resolverlos
32 .- Analicemos del siguiente modo: a) a = arc tan(--J3 ) > tan a = - -J3 > a = - b) P = arc sen 1 => sen P = 1 => P = Luego: M = cos (a + P) => M = cos M = cos 5/3 M = 2 33 .- Apliquemos la identidad aditiva especial con x = y, obteniéndose: 2 arc tan x = arc tan 2x JL Ahora evaluamos para x = 1/2: A continuación reemplazamos en la expresión dada para «M» í 4 M = tanl Tt + arc tan ~ = tan 4 arc tan — Por propiedad: tan (are tan x) = x , si: x G IR .-. 34.- Nuestra estrategia consistirá en aplicar el criterio de reducción al primer cuadrante: a) sen 5 = sen (5 - 2it) ... ver Fig (1) => arc sen(sen 5) = arc sen(sen (5 - 2it)) = 5 - 2n G b) cos 6 = cos (2it - 6) ... ver Fig (2) Tt Tt => arc cos (cos 6) = arc cos(cos(2Tt - 6)) = 2it - 6 e [0; Tt] M = 5 - 2it + 2it - 6 M = -1 4 Funciones Trigonométricas Inversas R18 555
36.- Elaboramos un análisis del arc tan para lo cual hacemos: 5 a = arc tan 12 A continuación evaluamos la expresión dada: tana= ~ Usando identidades de la cotangente del ángulo mitad nos queda: M = csc a + cot a En atención al Ex elaborando la expresión queda así: 13 12 25 M= — + — => M=— M = 5 3 3 3 37.- Sea: 6 = arc cos cos 6 = => M = 3 cos 26 Pero: cos 26 = 2 cos26 - 1 => cos 26 = 2 j - 1 => cos 26 = 2Í‘|j - 1 = ~ => M = 3 (|) M = 1 Problemas de Trigonometría y cómo resolverlos <*RACSO W1DITO1I1
38.- Sea: a = arc sen cot ~ arc tan [cos 2a] Recordamos aquí que: cos 2a = 1 - 2 sen2a = 1-2 M = cot 2 are tan En este punto conviene hacer: 6 = arc tan 15 Reemplazando 6 en (1): 0 M = cot 2 M = Aquí conviene recordar que: cot = csc 6 + cot 6 Del triángulo rectángulo: 0 17 15 32 cot 2 - g + g - g M = 4 39.- Nuestra estrategia consistirá en expresar todos los términos de modo que se puede identificar expresiones del tipo: F.T.(F.T.I. x) = x. 2 M = 1 - cos arc eos M = 2 - cos arc eos 2 + 1 - sen arc sen sen arc sen Recordando que: cos (are cos x) = x sen (arc sen x) = x « xg [-1; 1] Entonces: M = 2- 7 1 = 2' 8 " 8 M = 1 Funciones Trigonométricas Inversas
40.- Analizamos la expresión en base a los términos de la fracción. Veamos del numerador: a = arc eos V2 cosa = —¡= V3 A partir de esta razón construimos un triángulo rectángulo: tan a = —j= V2 1 Es conveniente aquí, calcular el valor de tan 2a, para lo cual recordemos la identidad del ángulo doble: 2 „ 2 tana tan 2a — , 1-tan a Jl 4 > tan 2a - n => tan 2a - /— 1-1 2 > tan 2a = 2 -J2 => 2a = arc tan(2 -J2 ) a 1 Reemplazando: M=— .-. M=~ 2a 2 41.- Procediendo como en el problema anterior, hacemos: 6 = arc tan( J5 ) => tan 6 = -J5 Construyendo un triángulo rectángulo: M = sen[2 arc tan (cos 26) ] Recordando la identidad del ángulo doble se tendrá que: cos 26 = cos26 - sen26 => cos 26 = 15 2 cos26=7-7 => cos26 = -< O O o Reemplazando en la expresión para «M», tendremos: = sen -2 arc tan j Problemas de Trigonometría y cómo resolverlos 4» RACSO
a = arc tan ’g => tan <* = 3 Construyendo un triángulo rectángulo M = sen(2a) = -2 sen cccos a A continuación reemplazamos valores según el Ex construido Ahora sea: 12 13 42.- Nuestra estrategia consistirá en analizar cada sumando. Veamos: a) Sea: 12 12 a = arc tan~ => tana= ~ Construyendo un triángulo rectángulo: Ver Fig. (1) „ 8 r, 8 b) También sea: p = arc cot ~ => cot p = ~ Construyendo un triángulo rectángulo Ver Fig. (2) Fig- (1) Fig- (2) M = - Utilizando estos triángulos se puede reemplazar en «M»: Luego: M = 13 sen a + 17 cos P => M = 13^-j^j + 17^-^j .-. M = 20 43.- A partir de la identidad aditiva, se verifica que: 1 1 Tt arc sen ~ + arc cos ~ ~ Reemplazando en la expresión dada se tendrá: Por ángulos complementarios: 1 Aquí conviene hacer: 6 = arc tan ~ fu 1 M = cosí ~ - arc tan ~ M = sen(orctan-|) 1 tan 0 = ~ Funciones Trigonométricas Inversas
Construyendo un triángulo rectángulo: Racionalizando: M = sen 6 = 44.- Sean: 1 1 a = arc cos ~ => cos a = ~ . (1) 1 1 P = arc sen— => sen0= — ---(2) De (1) y (2) podemos deducir que: a + 0 = — ... (cos a = sen 0) 4 4 _ 4 4 Luego: M = cos a - sen p => M = cos a - cos a .-. M = 0 45.- Evaluemos cada término de la expresión: (Va) J3 n —— => sena= —— => a= ~ 7 P = arc tan 1 => tan 0 = 1 => P = ~ 1 Tt 6 = arc sen k2 J => sen 6 - — => 6 = ~ 2 O Jl Tt 8 = arc cos 2 => cos8=— => 8=- < j z. ** Reemplazando estos valores en la expresión dada para «M», tendremos: if TtA Tt —I — H— 4(3) 4 líir'! ñ —I — H----- 2l 6 I 4 4ti 4it 12 M = M = 1 RACSO ^.orro.n R18 560 Problemas de Trigonometría y cómo resolverlos
46.- Como en el ejercicio anterior, conviene analizar cada término. Veamos: V2 a = arc sen-— 2 1 6 = arc cos ~ Luego: M = . 2Í 71 1 4cos — l3 ) ' 2—— M= V 47.- Recordar, arc tan(-x) = - arc tan(x), x G R Asimismo al racionalizar se tiene: = 3 + 2-j2 W=arc tan(3 + 2^2) + arc tan M Reconocemos que la expresión se puede transformar por medio de la identidad aditiva especial, por lo que aplicamos la condición: Si: M.N <0 => k = 0 Luego (*) queda así: W = arc tan W = arc tan 3+^ 2 3+—^— 2 = arc tan (1) 71 Funciones Trigonométricas Inversas
48 .- Nuestra estrategia consistirá en transformar la expresión dada de modo que las funciones arc sen y arc cos quedan vinculadas por una adición lo cual se logrará apli- cando las propiedades de proporciones: arc sen x a Usando proporciones: ------------------ =----r arc sen x + arc cos x a+b n Como. arc sen x + arc cos x = ~ , Vx e [-1; 1] arc sen x a => ----—--- = ~ ~ .-. arc sen x = -=7-—rr n a+b 2la • b) 2 49 .- Transformamos la expresión dada factorizando en el denominador: ,, tan(flrcsenx+crccosx+crcsenx) tan(2(«rc sen x+arc cosx)+arc cosx) tan^+arc senx M tan(7t+arc cosx) 71/2 Usando el criterio de reducción al primer cuadrante en cada término de la fracción, tendremos: cot(«rc sen x) tan(«rc cos x) Pero: tan( arc cos x) = cot (arc sen x) ya que: arc sen x + arc cos x = — M = 1 50 .- Haciendo uso de la definición de F.T.L , transformamos la expresión dada: a = arc tan x => tan a = x 0 = arc tan y => tan 0 = y 6 = arc tan z => tan 6 = z Si : a + 0 + O = 7t => tan a + tan 0 + tan 0 = tan cutan 0.tan 0 Aplicando la identidad condicional: x + y + z = xyz Luego: xyz+2xyz M.= --------- xyz M=^ xyz M = 3 51.- Nuestra estrategia consistirá en expresar la relación dada en base a una nueva variable N. A continuación procedemos a reducir la variable N en base a las identidades trigonométricas. Veamos. Sea: 1 - sen2x - cos2x 1 -sen2x + cos2x RACSO IDITOIia R18 562 Problemas de Trigonometría y cómo resolverlos
2 Usando: 1 - cos 2x = 2 sen x 2 1 + cos 2x = 2 cos x a sen 2x = 2 sen x . cos x Sustituyendo en (*) se tendrá: N = 2sen2 x-2s<?nx.cosx N = ^senx^n^psx) 2cos2 x-2senx.cosx -2cosx^errx-<Qgx) Simplificando: N = -tan x , sen x cos x Luego: M = arc tan(-tan x) = -arc tan (tan x) / 71 7t\ Pero: arc tan(tan x) = x <=> xg M = -x i 52.- Analizamos la expresión dada a partir del radicando del segundo radical, el cual estará correctamente definido, si se verifica que: 71 arcsenx - — >0 => flrc senx > — (no puede ser) 71 71 v => are sen x=—=$ x = sen— =1 71 Ore senx = —(si puede ser) Sustituimos estos resultados en la expresión original, obteniendo: M= ,|7t2 -27t«rccosl + .f——— V V2 2 ' 6 ‘ ' o M= Vrt2 =7t 53 .- En base a la resolución del problema anterior podemos afirmar que: 71 arc sen x = — a x = 1 Al reemplazar estos valores en la expresión dada para «M», tendremos: M = n2.27t.«r<cosl + ~ ________________<2 2 «rctanl + arcsenO 77C2-27t(0)+0 X v+o x M = 4 Funciones Trigonométricas Inversas
54 .- Analizamos la expresión dada a partir de sus términos. Así con el primer término hacemos: 1 a = «re cos ~ cos a = — 4>5 Asimismo, de este triángulo rectángulo reconocemos que: 4V3 sena= -y- a = arc sen Sustituyendo esta expresión en la relación dada «6», esta queda así: 6 = arc sen + arc cos A- * O- 2 55 .- Dado que: 2-3 < 1» aplicaremos la identidad aditiva especial: Así tendremos que: arc tan a + arc tan b = arc tan a+b 1-ab ab < 1 1 arc tan y + arc tan y = arc tan 2 3 1-11 2’3j 1 1 => arc tan y + arc tan y = arc tan(l) Reemplazando en la expresión original tendremos: 1 3 Luego: x = arc tan 1 + arc tan 1 => x = 2 are tan 1, 71/4 71 2 n sen x = seny sen x = 1 56 .- Esta serie la resolveremos a partir del análisis de su término general, el cual nos permitirá poner en evidencia la relación que guardan entre si cada uno de los términos de la serie. Veamos: arc tan = arc tan (n+l)-n l+(n+l)n = arc tan (n + 1) - arc tan n 1 ,2 . . Ahora evaluamos está expresión del siguiente modo: Si: n = 1 1 3 1 _____ Si: n = 2 => «retan y = art 'ARACSO P BDITO1BI R18 5641 Problemas de Trigonometría y cómo resolverlos
arc tan Si: n = 3 1 2 n +n+l = arc tan(n + 1) - arc tan n Al sumar miembro a miembro todos estos términos, sé obtiene: M = arc tan (n + 1) - arc tan 1 Aplicando la propiedad aditiva especial, nos queda: M = arc tan n+1-1 l + (n + l).l í n M = arc tan --------~ l n + 2 57.- Agrupando convenientemente los términos de la variable angular de la cot en cada término de la fracción, podemos factorizar asé cot(2(«rcsenx + arc cos x) + arc cos x) cot( 4(«rcsenx+arc cos x) - arc cos x) Recordando que: arc sen x + arc cos x = — , si x 6 [-1; 1] cot(2| — + arc cosx) W l 2 J________________ w cot(n+arccosx) - cot(4^j-«rccosx) «*(27!-«recosx) Haciendo un cambio de variable: 6 = arc cos x cinc cot(7t + e) cot(2rt-e) Tívc^ cote -cote w = -i W = 58.- Resulta conveniente expresar la variable de la sec en términos del arc tan, para lo cual es necesario recordar que: arc cot(x) = arc tan — I, si x G (0; +°°) => arc tan (-x) = -arc tan(x) ,x e R Aplicando estas propiedades en la expresión original, se tendrá: Funciones Trigonométricas Inversas R18 565
2 1 arc tan — - arc tan"^ VV = sec2 VV = sec 2 i arc tan~ + arc tan| _______________ 1_ 3 2 En esta suma de arc tan observamos que- MN = < 1 => k = 0 Luego aplicando la identidad aditiva especial, se tendrá: VV = sec2 are tan 3 l 1-^f-l 3l 3 VV = sec2 are tai 3 11 2 1 3 Aquí, es conveniente transformar esta expresión en términos de la tangente, para lo cual es necesario recordar la identidad: 1 + tan2e = sec2O VV = 1 + tan2 3 arc tan~ 3 11 12 9 VV= 1 + — 121 130 M = —~ 121 59 .- Nuestra estrategia para la reducción de términos lo logramos haciendo un cambio de variable, el cual consistirá en sustituir «x» por una F.T. En este caso conviene hacer: x = sen a 2 2 2 2 VV = arc cos (1-8 sen a(l- sen a)) => VV = arc cos (1-8 sen a cos a) 2 2 VV = arc cos(l - 2(2 sen a cos a) ) => VV = arc cos(l - 2. sen 2a) VV=4a Pero: 0<4a<it sen a = x Tt 4 a = arc sen x V2 V2 Como a g IC, el sen 6 es creciente, entonces: sen(O) < sen a < => 0 < x < ... (2) V2 De (1) y (2) al intersectar los resultados obtenidos para «x», se obtiene: 0 < x < -y W = 4 arc sen x, si x G RACSO Pbditobbi R18 j566| Problemas de Trigonometría y cómo resolverlos
60 .- Nuestro proceso de transformación consistirá en aplicar lo que hicimos en el ejerci- cio anterior. Empecemos haciendo: 0 = arc cos x => cos 0 = x => W = 20 => cos W = cos 20 2 2 Recordando la identidad del arco doble: cos W = 2 cos 0-1 => cos W = 2x -1 2 Despejando: W= arc cos(2x -1) Tt Pero por definición de cos 20 se debe cumplir que: 0 < 20 < n => 0 < 0 < ~ Como 0 g IC, la función coseno es decreciente, por lo cual se verifica que: cos(O)>cos(0) > cos^^ => 1>cos0>O => 0 < x < 1 W = arc cos(2x2 - 1) , si x G [0; 1] 61 .- Como se hizo en los ejercicios anteriores, haremos un cambio de variable para luego emplear los recuerdos de las identidades trigonométricas del ángulo triple. Expresemos así 0 = arc tan x => tan 0 = x => W = 30 => tan W = tan 30 Recordando la identidad del arco triple: tan W - 3 tan 0-tan3 0 l-3tan20 tan W - 3x—x3 l-3x2 De donde se tiene que: W = are tan 3x-x3 l-3x2 Pero por definición de tan 30 se debe cumplir que: Tt 2 Tt 2 Tt í TtA f Tt => tan —— < tan 0 < tan ~ bj ^6 6 6 W = arc tan ( 3x-x3) I1"3*2 J 62 .- Aplicando la identidad correspondiente a la cot del ángulo doble se tendrá que: cot 2 - tan 2 = 2cot 4 2 cot2-tan2 Xcot4 X Funciones Trigonométricas Inversas
Ahora, hacemos la sustitución de este resultado en la expresión original, obteniéndose que: a = arc cot(cot 4) Como 4 G IIIC, se tendrá que: a = arc cot [cot(7t + (4 - 7t))] => a = arc cot [cot (4 -ti)] .-. a = 4 - n 63 .- Nuestro problema consistirá en aplicar adecuadamente las siguientes propiedades: arc csc (csc x) = x, si x g arc sec (sec x) = x, si x g 7t \ / 71 ; 0) o ( 0; — 71 \ / 71 Asimismo reconocemos que 8 g IIC, por lo cual, al reducir al primer cuadrante, se tendrá: csc 8 = csc[3tc - (3ti - 8)] => csc 8 = csc (3tt - 8) 6 = are csc[ csc (3ti - 8)] - arc sec (sec 2) => 0 = 3tc - 8 - 2 6 = 3ti - 10 64 .- Procediendo como se hizo en problema anterior, se tendrá: arc cot (cot x) = x , si x G (0; 7t) arc sec (sec x) = x, si x g ti 2;It Desde que: 4 G IIIC, tendremos que: cot 4 = cot[7t + (4 - te)] => cot 4 = cot (4 - ti) ... (1) Al analizar el otro término, debemos reconocer que: 6 G IV C, por lo cual: sec 6 = sec[2jt - (2ti - 6)] => sec 6 = sec (2ti - 6) . .. (2) Sustituyendo (1) y (2) en la expresión original se tendrá: P = arc cot[cot (4 - ti)] + arc sec[sec(27t- 6)] => P = 4 - ti + 2ti - 6 .-. P = 7t - 2 65.- Nuestra estrategia consistirá en aplicar adecuadamente las siguientes propiedades: arc cot (-x) = n - arc cot(x) arc sec (-x) = n - arc sec (x) , XG R , x G <-oo ; -1] u [1; +°o) Problemas de Trigonometría y cómo resolverlos RACSO DITOKBI
arc csc (-x) = Tt - arc sen — I , x G ; -1] o [1; +“) k* ) Luego, en la expresión original se tendrá: 1 r- <1A W = ti - are cot(l) + — [rt - arc sec(V2)] + 3. arc senl — I Recordando que: arc cot (1) = ti/4 ; arc sec (V2 ) = Tt/4 a arc sen (1 /2) = Tt/6, se tendrá: 3 w=- 71 71----- 4 Tt 2 3 w=-. 3ti Tt T + 2 9ti Tt T + ~2 13n w= — 66.- Para este ejercicio resulta conveniente recordar las siguientes propiedades: arc tan (-x) = - arc tan(x), x g R arc sen (-x) = - arc sen (x), x g [-1; 1] arc sec (-x) = rt - arc sec(x), x g ; -1] u [1; +°°) Aplicando estas propiedades tendremos: - ( 7} ( 5 } r co = 5 cos I -arc tan~ I + c°t I ~ arc sen I + v6 . sen(Tt - arc sec 5) Para continuar nuestro proceso de reducción vemos que es necesario expresar los arcos negativos en positivos, luego aplicando las propiedades correspondientes nos queda: I ' I I 3 I r~ co = 5 cos I arc tan — I - cot I arc sen — I + V6. sen(nrc sec 5) a ' p e Cada uno de los arcos indicados nos permiten establecer los siguientes esquemas: Haciendo los cambios de variable, nos queda: co = 5 cos a - cot 0 + v6 sen 6 Sustituyendo los valores correspondientes, según los triángulos mostrados, nos queda: co = 12 f- 2^6 — + V6 . --- 5 5 24 12 12 24 5 ’ 5 + 5 " “= 5 Funciones Trigonométricas Inversas
67.- Nos proponemos hacer los siguientes cambios de variables: a - arc tan 5 > tan a = 5 P = arc cot 5 > cot P = 5 71 371 => tan ct - cot P > ct + P = — => 3a = — - 3P De lo cual podemos establecemos que: ( 3n tan 3a = tan I — - 3P > tan 3a = cot 3P .. (1) Al reemplazar en 6 los cambios de variable propuesto se tiene: 6 = tan 3a - cot 3P ... (2) Reemplazar (1) en (2): 6 = cot 3P - cot 3P = 0 6 = 0 68.- Nuestra estrategia consistirá en evaluar la secante y sus potencias, por separado, obteniéndose: 71 sec ~ = sec 15° = 76-72 sec2~ = (7ó - 72 )2 = 8 - 2-712 = 8-473 sec4~~ = ^sec2~^ = (8 - 473 )2 = 112 - 6473 Reemplazando en la expresión dada, tendremos: a = arc secpll2 -6473 -16(8 -473)+20 ) -> a = arc sec ^J112 -£47^-128 + £4'73 + 20 71 a = arc sec(2) = — 71 71 \ / 71 \ Nota: ~ 6 rango del arco secante = 0; ~ ) cj 69.- Agrupamos los dos primeros términos de la expresión dada para aplicar en ellos la identidad aditiva especial de arc tan: R18 570 Problemas de Trigonometría y cómo resolverlos RACSO IDITOKll
1 7 W = arc cotí — + arc tan + arc tan 13 (*) En donde reconocemos que: M=3,N=- MN = — <1 Sustituyendo en la relación: arc tan M + arc . XT _ ÍM + N tanN~ (1-MN W = arc tan 1--.- 3 7 + arc tan 13 W = arc tan + arc tan 1 13 3 7 2 En esta expresión volvemos a aplicar la misma identidad, porque: MN < 1, luego se tendrá: W = arc tan 2___13 1 1 W = arc tan 3 5 2 13 70.- Nos proponemos un cambio de variable para luego construir un triángulo y extraer de el los datos que nos permitan hacer transformaciones pertinentes. Veamos: a) a = arc sec 17 Vi 7 sec a = —— 4 b) P =arc tanl 8 tanP=Ii 17 15 17 ' 4 4 a Reemplazando estos cambios de variable en la expresión original se tendrá: M = "p Pero: 2 eos 2a = 2 cos a - 1 > cos 2a = 2 x2 4 1 17 15 17 Y de (*) se establece que: cos 2a = cos P 2a=P a Finalmente reemplazamos (**) en la expresión M: M = 1 M=- Funciones Trigonométricas Inversas R18 571
SITUACIONES GRAFICAS 71 .- Recordemos: 0 < arc cos x <it <=> -1 < x < 1 Por definición de arcos, sabemos que esta tiene su dominio en [-1; 1]; y su rango [0; ti] luego en nuestro problema se deberá verificar que: a) -1 < — < 1 => multiplicando por 2: - 2 < x b) 0<flrccos 2 ~n x Multiplicando por 4: 0 < 4 arc cos — < 4ti Tt n x ti 15ti Restando ~: - — < 4 arc cos ~~ < —— 4 4 2 4 4 Graficando: 72 .- a) Sabemos que el gráfico de: y = arc tan x es como la Fig. (1). Tt b) El gráfico de: y = arc tan x + — es como la Fig. (2). 73.- Empezaremos nuestra resolución analizando el dominio de la variable del arc sen. Veamos: f (x) = - 3 arc sen Multiplicado por 3: -3 < x + 1 < 3 Restando 1: -4 < x < 2 => Dom /= [-4; 2] R18 572 Problemas de Trigonometría y cómo resolverlos RACSO IDITOtll
Asimismo, el rango de la función arc sen está en el intervalo [Tt. Tt] 2'2j Luego: Tt < are sen Multiplicando por (-3): 3n < -3 arc sen 3ti ~ 2 Ran /= 3ti 3ti T'T En el gráfico dado, se puede reconocer que éste se ubica entre los límites del dominio y del rango. Asimismo, se visualiza una función simétrica respecto del eje «x» y del eje «y», luego: 74 .- Lo conveniente es despejar la inecuación dada, obteniéndose: arc tan (x) < arc cot(x) ...(*) Resolviendo el problema con las gráficas de dichas funciones inversas tenemos: Funciones Trigonométricas Inversas R18 573
Observamos que la desigualdad dada se verifica desde el punto de intersección de las gráficas hacia la izquierda. Luego calculamos el punto de intersección entre dichas gráficas: arc tan x = arc cot x - 0 => x = tan 0 = cot 0 71 7t 0 = ~ > x = tan , = 1 4 4 Por lo tanto para que se cumpla la condición (*), del gráfico se observa que: x G (-»; 1) 75 .- Despejando se tiene: arc sec (x) > arc csc(x) ...(*) Resolviendo el problema con las gráficas de dichas funciones inversas tenemos: Calculando el punto de intersección entre dichas gráficas: 71 71 r- arc sec x = arc csc x = 0 > x = sec 0 = csc 0 > 0 = — => x = sec . = -J2 4 4 Por lo tanto para que se cumpla la condición (*), del gráfico se observa que: x G (-« ; -1] vi [72 ; +“) 76 .- Se debe tener en cuenta que: 2x g Dominio arco secante 1 1 -> 2x<-lv2x>l x<--vx<- ...(1) A su vez el rango de la función arco secante debe encontrarse, según los datos en el intervalo [tt/4 ; ti/2]: 71 71 — < are sec (2x) < - > es creciente > 72 < 2x < + o° -> V2 < x < + oo... (2) Asimismo se debe verificar, por definición, que el rango del arc sec es: Problemas de Trigonometría y cómo resolverlos RACSO V^BOITOKBI
71 — < are sec(2x) < n - - - (3) 2 De (2) y (3) concluimos que: J2 2 - - - (4) . 2 Finalmente de al intersectar (1) y (4) se tiene que: 2 V2 . 2 77.- Por definición, el arc sen tiene dominio tal que: -1-C B 1-C B De acuerdo con el gráfico se puede establecer que: -1-C 3 a) .—§—=-4 => 3B-4C = 4...(1) 1-C 1 b) — - ~ > B + 2C = 2 ...(2) t> Z 8 1 De(l)A(2): B=~ a C=~ J 3 Asimismo, el rango del are sen está comprendido en el siguiente intervalo: 71 71 -— < are sen (Bx + C) < ~ Al multiplicar por «A» y sumar por «D», se tendrá que: Att Ati => -~y + D < A. arc sen(Bx + C) + D < + D Comparando los valores extremos de esta desigualdad con los límites del gráfico en el eje «y», se establece que: Ati 3ti c) Límite inferior: + D = . (3) Alt Tt d) Límite superior: + D = — .. .(4) Z o 7 5n De(3)y(4): A= - a D = -~ Funciones Trigonométricas Inversas R18 575
78.- Analizando el dominio de la función, según la definición del arc cos, se puede establecer que: -1-C 1-C -1<Bx + C<1 —-— <x< — ~ D D Y evaluando los límites de la función en la gráfica, con los extremos de ésta desigual- dad, se puede concluir que: 1-C —— =3 => 1 - C = 3B ... (1) D -1-C —g—=-l => -1 - C = - B ... (2) De(l)y(2): B=|,C = -| Asimismo, por definición, el rango de la función arc cos está en el intervalo: 0 < arc cos (Bx + C) < n => D < A arc cos (Bx + C) < An + D Y de acuerdo con el gráfico, estos extremos deben verificar las siguientes relaciones: 5n D = -T ..(3) Ati + D=~ ..(4) 3 3 571 De(3)en (4): A = - .-. y = -. árceos I 2 2 I " T 79.- Si la gráfica de / intersecta al eje x, entonces: y =f(x) = 0 TT 71 Es decir: 2 arc sen(2x - 1) + — = 0 => arc sen(2x - 1) = - — 3 o 1 1 => 2x-l=-~ x=- El punto será: (4'®) íAracso P«DITOtB« R18 576 Problemas de Trigonometría y cómo resolverlos
MISCELÁNEA 80.- Nuestra estrategia consistirá en reconstruir la variable del arc tan, para lo cual partiremos del siguiente supuesto válido para el cuadrado de todo real. Veamos: Como: (x-1)2>0 , V re R -> x2 + 1 - 2x > 0 -> x2 + 1 > 2x ‘ 2x -> ---<1 , V xg R X +1 Como la función y = arc tan x es creciente VxgR , podemos establecer que: ( 2x 'l n 71 > arc tan —*--- < arc tan > f(x)<— f- = — I x+1 J J 4 •,max 4 81.- Nuestra resolución la iniciamos planteando las siguientes restricciones: 71 1) tan x 3 sí x * (2m +1) — ,ffle Z 2) cotx3 síx*mt ,ne Z Puesto que ambas funciones forman parte de una misma expresión es conveniente ela- borar un dominio común de ambas, con la cual se obtiene: kn x* — , leZ 1 Asimismo conviene recordar que: a + — > 2, sí a G R Pues bien de la expresión dada podemos reconocer que: 2 2 2 1 tan x + cot x - 1 = tan x +--~--1 > tan x >2 2 2 2 tan x * 0 => tan x > 0 .-. tan x + cot x - 1 > 1 Para esta condición podemos afirmar que la función arco cotangente es decreciente y por lo tanto: 2 2 are cotftan x + cot x -1) < arc cot(l) Funciones Trigonométricas Inversas
Y como el arco cotangente está definido en (0 ; n), entonces la función dada queda definida en: 0 <fto < j n ^)]míx= 7 82.- Esta ecuación la resolvemos transformando la expresión con la condición de obte- ner otra con F.T.I. complementarias para luego pasar a la F.T. y aplicar las identidades correspondientes. Veamos: arc cos(-J3 x) = — - arc cos x => arc cos (V3x) = arc sen x = 6 sen 0 = x .. .(1) cos 0 = i/3 x ... (2) Haciendo (l)2 + (2)2: sen20 + cos20 = x2 + 3x2 => 4x2 = 1 => x2 = — 1 1 1 De esta ecuación obtenemos: x = — v x = - — (No cumple la igualdad dada) 1 83.- Nuestra estrategia consistirá en transformar la expresión dada en base a la defini- ción de la F.T.I. Si: arc tan {cot [are sen-J2xj} = — =±> cot[arc sen -J2x ] = tan — Recordando que: cot a = tan P <=> a + P = > arc sen -J2x + i— 71 i— 71 i— i/3 > arc sen -J2x = ~ > -J2x = sen " > V2x = 3 3 Elevando al cuadrado a ambos miembros: 2x = — .'. x = ~ 84.- Para aplicar propiedad para la suma de arcos tangentes, agrupamos los dos primeros términos, en los que se verifica que: M.N. = 1 /6 < 1, luego: k = 0. Por ello se plantea que: R18 578 Problemas de Trigonometría y cómo resolverlos RACSO IDITO1EI
arc tan 2 + arc tan 3 = arc tan M N t_________t => arc tan 2 + arc tan = are tan (1) 1 . 1 71. => arc tan 2 + arc tan 3 = 4 Reemplazando en la expresión original, ésta se ve reducida: 71 are tan x + — = arc sen x + arc cos x 4 En donde el segundo miembro equivale a n/2 siempre que x 6 [-1; 1]. Luego la expresión dada queda así: 71 are tan x = — 4 71 x = tan ~ 4 1=1 85.- Resolver la ecuación nos plantea las siguientes restricciones: 1) Del radicando del primer miembro: x—2 x+1 >0 íwwwo--------- -co -i 2 +00 (+) (-) : (+) > x 6 ; -1 ) u [2; +“) 2) Del radicando del segundo miembro: ~ > 0 => x 6 [0; +°°) Puesto que (x) es un variable común a ambas expresiones, procedemos a intersectar ambos intervalos obteniéndose: x 6 [2 ; +«=) a = arc sec - - (2) - - (3) Sabiendo que: 2 2 1 + tan ct = sec a Sustituimos las relaciones (1) y (2) en (3): x+l+x-2 = * x + 1 2 Funciones Trigonométricas Inversas
7 4x - 2 = x+ x x2-3x+ 2 = 0 > (x-2)(x-l) = 6 De lo cual se obtiene: x = 1 (absurdo, según restricción) .-. x = 2 86 .- Este tipo de problema, en el que participan dos variables en una misma ecuación, se resuelven a partir del análisis del recorrido de uno de ellos y desde ésta, se reconstruye la otra. Veamos: Recordamos que si: f(x) = arc cos (x) > Dominio —> x G l-i; i] => Rango -+ y = arc cos (x) g [0; 7t] 0 < arc cos(x) < ti => 6 < ti . csc (y) < ti => 6 < csc (y) < 1 => 6 < csc y < 1 cj csc y = 1 (:) Reconocemos que (*) es absurdo, dado que: csc y < -1 vj csc y > 1 .’. csc y = 1 => y - (4k + 1)— ;ke Z Asimismo: arc cos(x) = ti => x = cos ti .’. x = -1 87 .- Nuestra estrategia consistirá en reducir el valor numérico del primer miembro de la ecuación de este modo: arc cos 5 + 2 arc cot 2 = arc csc x ... (1) a e A partir de esta propuesta construimos los siguientes triángulos: Asimismo, recordamos que: arc csc x = arc sen ~ __(2) Luego reemplazamos (2) en (1): a + 26 = arc sen~ => sen (a + 26 ) = ~ Aplicando la identidad del seno de una suma de arcos, se tendrá: = sen a cos 26 + cos a. sen 26 (3) R18 580 Problemas de Trigonometría y cómo resolverlos RACSO WlDITOtlI
Ahora, calculamos: sen 26 = 2sen 6 cos 6 = 2^^ x 2 4 a/5 5 Asimismo se tendrá: cos 26 = g r, i 1 4 3 3 4 1 25 Reemplazando en (3): -^ = 5 5 + 5 5 => I = 24 25 24 x = 88 .- De la propiedad referida a la F.T.I. y su var able tendremos: (n A ( n A cot x . cot I — - x I . cotí y + x I = cot 3x Por definición arc cot, su dominio es tal que: arc cot[cot 3x] = 3x , si: / n \ 3xg (6; ti) xg(O;~) 89 .- Por la propiedad de las F.T.I. y su variable verifica que: 71 cos(arc tan[cot(«re sec(csc x))]) = sen“ ., b 1 cos(«re tan[cot(«re sec(csc x))]) = ~ n/3 n/3 71 are tan [cot(«re sec(csc x))] = — 71 r- cot(«re sec(csc x)) = tan~ = v3 tt/6 71 are sec (csc x) = — b csc x = sec 71 6 71 3 Sustituyendo este valor en la expresión dada para «W», tendremos: arc sen^cos^+cos^j W = W = arc «res . ( i1371 «retan cot------- । 3 P «resen(6) 0 W = 0 Funciones Trigonométricas Inversas R18 581
90 .- Nuestra estrategia consistirá en analizar cada miembro de la desigualdad y a con- tinuación elaborar los gráficos correspondientes de modo que sea posible identificar el intervalo de valores de «x» en el que la desigualdad se verifica. Veamos: a) Sea: /(x) = arc csc | x | i) Si: x > 0 => /(x) = arc csc (x) ii) Si: x<0=> /(x) = arc csc(-x) = -arc csc(x) b) Sea: g(x) = arc sec | x | i) Si: x > 0 => g(x) = arc sec(x) ii) Si: x < 0 => g(x) = arc sec(-x) = n-arc sec(x) Analicemos el problema con las gráficas de / (x) y g(x). Aprovechando que las gráficas son simétricas respecto del eje «Y». /(x)>g(x) ...(1) Calculamos el punto de intersección entre dichas gráficas. Veamos: arc sec x = arc csc x = 6 => x = sec 6 = csc 6 71 n f- => 6= ~ => sec "T = V2 4 4 Por lo tanto para que se cumpla la condición de desigualdad dada, del gráfico se obser- va que: x G {—Jl ; -1] u[l; -Jí} Aracso rDirotii R18 582 Problemas de Trigonometría y cómo resolverlos
91.- De acuerdo con la teoría se sabe que: y = arc tan x es creciente en [-1; 1] => arc tan (-1) < arc tan x < arc tan 1 Tt ~4 Tt 4 Multiplicando cada miembro por 3: 3ti 3ti - — < 3 arc tan x < — 4 4 71 Restando ~ a cada miembro: 4 ti n -ti < 3 arc tan x - — < ~ y Tt Finíx- 2 71 92.- Recordemos la identidad aditiva: arc cot x = — - arc tan x, V x e R Reemplazando en la inecuación dada, se tendrá- f ti 3n ti 2 arc tan x + — - arc tan x < — => arc tan x < — I 2 I 4 4 A continuación elaboramos la gráfica de esta relación y obtenemos: Funciones Trigonométricas Inversas
ECUACIONES TRIGONOMÉTRICAS 01.- En este problema la única restricción lo tiene la tangente, pues como se sabe: 71 tan x H, si x + (2n + 1)—, n G Z Nuestra estrategia consistirá en transformar la expresión dada en términos del seno para a partir de ella obtener una ecuación de segundo grado. Para ello debemos recordar que la siguiente identidad del arco doble: 2 2sen a = 1 - cos 2a Sustituyendo en la ecuación original, se tendrá: 2 ^2 sen1 2 jj + 3 tan x = 2 => 2(1 - cos x) + 3 tan x = 2 degradando o 3serrt 2 ------ = 2 cos x => 3 sen x - 2cos x cosx Expresando en términos de senos, se tiene: 3 sen x = 2(1 - sen2x) 2 sen x + 3 sen x - 2 = 0 => (2 sen x - l)(sen x + 2) = 0 Igualando a cero cada factor se establece que: a) sen x + 2 = 0 > sen x = -2 , absurdo pues: - 1 < sen x < 1 1 b) 2 sen x - 1 = 0 => sen x = — 1 ti k 71 EL valor principal 6 = arc sen — => 6 = ~ x = kn + (-1) . — , k e Z " z " O o 02.- En esta ecuación tenemos una restricción por parte de la tangente: tanx =>x*(2k + l)— ,keZ Nuestra estrategia consistirá en transformar la expresión dada en términos de la tan- gente, para así formar una ecuación de segundo grado. Esto se obtendrá apelando a la siguiente identidad: R19 584 Problemas de Trigonometría y cómo resolverlos ’ a RACSO itflTOIl
1 — tan2 x cos 2x = -------j— 1 + tan x Reemplazando en la ecuación trigonométrica: 7 1-tan x 7 1 + tan x -(1 - tan x) = 0 , Por diferencia de cuadrados: j(l—tánx)(l + tanx) (1 + tan2 x) ...(*) Al simplificar se debe considerar que: 1 - tan x = 0 => tan x = 1 71 71 => ep = - => x = kn+- 2 2 En (*), nos queda 1+ tan x = 1 + tan x > tan x - tan x = 0 Factorizando, obtenemos la ecuación: tan x (tan x - 1) = 0 i) tanx = 0 => x= kn , k e Z ii) tan x = 1 (ya fue considerado en el paso anterior) x= ufen 03.- Nuestra estrategia consistirá en transformar la expresión dada de modo que sea posible utilizar la condición dada para la tangente. Esto lo lograremos dividiendo en 2 71 ambos lados por cos x, para x * (2n + 1) —, es decir: -eos x * 0 . Asi tendremos: 3cos2x 2senx.cosx sen2x 0 2 ” " 2 “ 2 COSX COSX.COSX eos x eos x 7 => tan x +2 tan x - 3 = 0 7 => 3-2 tan x - tan x = 0 => tan x + 3)(tan x - 1) = 0 Al igualar a cero cada factor, tenemos que: a) tan x = - 3 , que se descarta pues por condición del problema: tan x > 0 b) tan x = + 1 x = kit+ — Z 4 04.- Transformamos la expresión dada para obtener una ecuación en términos del cose- no, utilizando la identidad pitágorica del seno y coseno: 1 2 1 - COS X = — (1 - cos x) Efectuando y transponiendo términos: cos2x +- 3 cos x + 2 = 0 Ecuaciones e Inecuaciones Trigonométricas
Factorizamos y nos queda : (cos x + 2)(cos x + 1) = 0 Analizando cada factor, tendremos: i) cos x + 2 = 0 => cos x = - 2 (No puede ser) ii) cos x + 1 = 0 => cos x - - 1 x = n 05.- En este problema nuestra estrategia consistirá en factorizar los términos del 1er miem- bro para dar lugar a la aplicación de identidades trigonométricas del ángulo doble y del ángulo mitad de modo que todo sea transformable a una sola función seno. Veamos: 2 2^ -senx. cosx ( cos x—sen x) = — \_______________________' 4 cos2x => - 2 sen 2x -cos 2x = 1 sen2(2x) n => sen 4x = -1 => 4x = (4k - 1) ~ n x=(4fc-l)~ O 2 2 => - 2 . 2senxcosx .(eos x - sen x) = 1 sen2x , V JtG Z 06.- Nuestra estrategia consistirá en obtener una ecuación en términos del seno. En vista que este problema incluye funciones en los denominadores, empezaremos analizando las restricciones del caso: i) sen x * 0 => x*hti,hgZ 71 ii) tan x * 0 => x * (2m + 1) — ,me Z cos3x cos2x-cosx Transformando a senos y cosenos: ------------------ ----- = 1 J senx senx => 2 cos 3x - 2cos2x.cosx = 2 sen x => 2 cos 3x - [cos 3x + cos x] = 2 sen x transformnmosa suma decosenos => cos3x-cosx =2 senx => -\^sen x . sen 2x =\^sen x => senx(sen 2x + 1) = 0 transformamos a » producto Igualando a cero cada factor, se tiene: 1) sen x = 0 => se descarta por la restricción (i) Problemas de Trigonometría y cómo resolverlos ¿RACSO WüDITOdl
2) sen 2* + 1 = O => sen 2* = - 1 71 jt 71 Pero: sen ( 4k- 1)~ = - 1 => 2x = (4k - 1) ; k e Z x=(4fc-l)~ 71 Haciendo: k = 0, se obtiene la mayor solución negativa: x = - — 07.- La estrategia que aplicaremos consistirá en transformar la expresión dada en térmi- nos del seno. Para ello apelaremos a las identidades del ángulo doble y triple. Veamos: Despejando tenemos: sen 5* + sen 3* + sen x = 3 - 4 sen x Multiplicando a ambos miembros por 2 sen x: 2 sen 5-*-sen x+2 sen 3*-sen x + 2 sen2* = 2(3senx-4sen3x) (arco doble) (arco triple) transformando a una diferencia de cosenos cbs^x - cos 6x +^cper^x - cbs^x + 1 = 2 . sen 3x => 1-cos 6x = 2 sen 3x Haciendo a = 3x, aplicamos: 2 sen a = 1 - cos 2a => 2 sen23x = 2 sen 3x => sen 3x(sen 3x - 1) = 0 Al igualar a cero cada factor se tendrá que resolver dos ecuaciones por separado y luego unir las soluciones halladas: 7171 1) sen 3x = 0 => 3* = nn => x = — , n g Z 71 2)sen3x-l = 0 => sen 3x = 1 => 3x = (4»i+l)~ => x--(4í!U 1)^;«eZ .-. sfinal = Si u S2 08.- Transformaremos la expresión dada hasta obtener una ecuación en términos del seno, para lo cual empezaremos aplicando a una diferencia de cosenos: 2 sen 3x . sen x - 2 sen 4x . sen 2* = 0 => cos 2x - cos 4x - (cos 2* - cos 6x) = 0 pos 6*-eos 4x = 0 transformando a producto - 2 . sen x sen 5x = 0 i) senx = 0 => x = mu, me % => x= — ,ke Z ii) sen 5x = 0 => 5x = nn Ecuaciones e Inecuaciones Trigonométricas
De los dos conjuntos solución se observa que en el conjunto solución: Sj vj S2 el conjunto Sj está incluida en el conjunto solución S^. kn x-—,keZ 5 09.- Empezaremos identificando las restricciones: senx*0 cosx*0 x + , k e Z (ángulo cuadrantal) Recordemos la identidad: sen3x ------ = 2 cos 21 + 1 senx cos3x ------ = 2 cos 2x - 1 cosx Reemplazando en la ecuación: 4 cos 2x = 4 cos 4x cos 2x = cos 4x Pero: cosA = cosB<=> A±B = 2kn, keZ Entonces: i) 4x + 2x = 2fcn kn — , keZ ii) 4x - 2x = 2kn => x = kn (No puede ser un ángulo cuadrantal) kn 3 ke Z 10.- Identificamos primero las restricciones: sen 2x^0 => 2x*kn cos2x*0 => 2x*(2k + l)-^ 2x*% senóx Recordemos las identidades: -----— = 2 cos 4x + 1 sen2x COSÓX A -------= 2 cos 4x - 1 cos2x Reemplazando en la ecuación: => csc 2x = 2 2 cos 4x + 1 = csc 2x + 2 cos 4x - 1 1 71 => sen 2x = — => 0p = — Usando el conjunto de solución del seno: t 71 2x = kn + (-1) . ~ O kn v ti x=y +('1)-12 'v*ez R19 588 Problemas de Trigonometría y cómo resolverlos jA RACSO iDITOlll
11.- En este problema resulta beneficioso aplicar la identidad del coseno del arco doble para transformar la expresión dada en otra en términos del seno. Veamos: 2 Aplicando: cos 2a = 1 - 2 sen a , con: a = 2x => cos 4x =1-2 sen zx => 1 - sen 2x + 2 sen 2x + 3 = 0 Ordenando la ecuación cuadrática tendremos: sen22x - sen 2x - 2 = 0 Factorizando nos queda: (sen 2x - 2)(sen 2x + 1) = 0 Igualando a cero cada factor: 1) sen 2x - 2 = 0 => sen 2x = 2; esto es absurdo pues: - 1 < sen 2x < 1 2) sen 2x + 1 = 0 => sen 2x = - 1 Este resultado lo analizaremos de este modo: i) Hallando el valor principal: • 71 sen 6p = - 1 => 6p = arc sen (-1) = - arc sen (1) => 6p = - ~ ii) Planteando la solución general (Sg) para todos los arcos que tiene igual seno, tenemos: Sg = *7t + (-i)k.ep k 71 iii) Además se debe cumplir que: 2x = kn + (-1) . (- — ) kn n v x=y-(-Dk ,ke Z 12.- Ordenando la ecuación: 5(sen4x + cos4x) + 6 sen6x . cos2x = 4 . . .(*) Aplicando la identidad: sen4x + cos4x = 1-2 sen2x . cos2x 2 2 2 2 2 2 En (*): 5(1 - 2 sen x . cos x) + 6 sen x . cos x = 4 => 4 sen x . cos x = 1 2 => (2 sen x . cos x) = 1 Ecuaciones e Inecuaciones Trigonométricas
13.- Aplicamos la identidad del coseno del ángulo doble: 2cos a = 1 + cos 2a => 2cos2^-x) - 2 cos2^+x)= 1 => 1 + cos^-2xj -[j + cos^ + 2xj] = 1 => cos^-2xj - cos^ + 2xj = 1 ransformando a producto, la diferencia de cosenos nos queda así: - 2 sen (- 2x). sen j =1 => 2 sen 2x. =1 => sen 1x = Para resolver esta ecuación trigonométrica efectuamos los siguientes pasos: i) Se calcula el valor principal (0p) V2 n => senep=— => 0p = arcsen — 6p= - > ii) Se plantea la solución general para todos los arcos que tienen igual seno. Sg = fat + (-l)k.0p , ke Z iii) Igualamos la variable angular con la solución general (Sg) y despejamos "x": v 71 2x = fat + (-l)k. - kn +(-l)k. n 8 14 .- Observamos que el 1er miembro del ecuación se puede transformar así: Factorizamos: sen 2x+ J3 cos 2x = 2 ^sen2x+-^cos2x Sustituimos por senos y cosenos: sen 2x+ Js cos 2x = 2 ^cos^sen 2x + sen^cos 2xj Aplicamos la identidad del seno de una suma: sen 2x+J3 cos 2x = 2 . sen ^2x+^ Además por arcos complementar os se tiene que: sen 2x+ J3 cos 2x = 2. cos —2x Reemplazando en la ecuación dada, tendremos: Haciendo u cambio de variable: « = sen^2x+^j, tal que: ...(1) 2sen(2x+^j] - 5 = sen(2x+^j Problemas de Trigonometría y cómo resolverlos -ÍJÍ RACSO Wbditoiii
=> 4«2-a-5 = 0=> (4a-5)(a + 1) = O => a=^ ° a = -l => senÍ2x + ^j=-l •----------------------------------------------V-» absurdo 71 Teniendo en cuenta que: sen (4k + 3) = - 1 71 371 La solución inmediata será: 2x + — = 2krt + — .-. x = (12fc + 7), k e Z 15 .- Nuestra estrategia consistirá en transformar la expresión hasta obtener otra en términos de x. Veamos: Como: | sen 2x | = 12 sen x . cos x | = 21 sen x | | cos x | Reemplazamos en la ecuación dada: 21 sen x | | cos x | + 2 = 21 sen x | + 21 cos x | 21 sen x | (| cos x | - 1) - 2( | cos x | - 1) = 0 => (| cos x | - 1)( | sen x | - 1) = 0 Igualando a cero cada factor tendremos: i)|cosx|-l = 0 => |cosx|=l => cosx = ±l => x=«7t,neZ 71 ii) |senx| - 1 =0 => |senx|=l =>senx = ±l => x = (2tn +1) —, tn e Z 71 JcTI De (i) y (ii) se tiene: x = mtn (2m +1) ~ x = — ; keZ 16 .- Transformaremos la ecuación para expresarla en términos del coseno. Veamos: Dividiendo entre 2 tenemos: 73 1 J2 . sen x + ~ . cos x = Expresando en términos de seno y coseno: Í7t\ /7t\ 72 sen ^3 J .sen x + cos ^3 J .eos x = Reconocemos que se trata del coseno de la diferencia de 2 ángulos. cos n ®P= 4 Usando el conjunto solución del coseno: Tt 71 x- - =2fat+ - Tt Tt x =2krt± — + —, Vfce 4 3 Ecuaciones e inecuaciones Trigonométricas R19 591
•17.- Obtenemos el cos 2x a partir de la expresión dada: cos 2x = 2 1 + V2 8-?2 El 2do. miembro está relacionado con el cos 45° /2, veamos cómo: , x |l + cosa COS 2 = V 2 cosi = l 1 + cos— 4 cos Asi deducimos que en (*): 71 cos 2% = cos — o i) Valor principal (0p) : 71 6 = are cos(cos “ ) " o n ®P = ? ii) La solución general (Sg) es: Sg = 2kn ± 6p, k g Z iii) Se debe cumplir que: 71 2x = 2kn ± ~ O 71 x = kn± ~ lu JtG Z n 2 18 .- Procederemos de modo que la ecuación se transforme en términos del coseno. Así: Despejando se tiene: Recordemos: En (*): cos 2x = 1 4 4 . cos x - sen x = 1... (*) cos x - sen x = cos 2x > 2x = 2k7t x — kn , V xg Z 19 .- Transformamos la expresión en términos del seno, para lo cual empleamos la iden- tidad pitágorica en la ecuación dada: 1 2 1 - sen x = ~ . (1 - sen x) 2 Despejando: sen x - 2 sen x + 1 = 0 2 => (sen x - 1) =0 => sen x = 1 x = (4k + l)^ ,VxgZ 20 .- La única restricción es que: x (2k + 1) — , k g Z para que la sec x y la tan x existan. Para poder emplear la condición implícita en la ecuación, empezaremos por recordar que: R19 592 Problemas de T gonometría y cómo resolverlos Stk RACSO PBDITOina
2 2 sec x - tan x = 1 => Del dato: (sec x - tan x =1 => 2 3.J5 => secx-tanx = Q . - 7= => 3 + vo 3—V5 De la condición implícita: De (1) - (2) tenemos: 2 tan x = J5 => Cuyo valor principal es: 6p = arc tan < j 21.- La resolución requiere de las siguientes re; (sec x + tan x)(sec x - tan x) = 1 1 sec x - tan x = „ . j + vJ 3-2V5 sec x - tan x = —-— ... (1) 3 + 75 sec x + tan x = —-— ... (2) 75 tanx= — i) La tan x 3 si: x * (2k + 1) — , k G Z ii) La cot x 3 si: x * nn , ne Z n iii) También 1 - tan x * 0 => tan x * 1 => x * (4m + 1) — ,me Z Resolviendo: 1 + cot x - (- 1)(1 - tan x) => 1 + cot x = - 1 + tan x cotx - tanx =-2 •---* --v-- Aplicando la identidad especial de arco doble, tendremos: 2 cot 2x = - 2 => 2 cot 2x = - 2 => cot 2x = - 1 3n 3n El valor principal 0p = arc cot(-l) = — => 2x = kn. + ~ kn 3n X=T+T 'fcGZ 22.- Transformamos la ecuación para expresarla en términos del coseno: Transponiendo términos: 3 . cos x - sen x = 1 Dividiendo entre 2: Expresando en términos de seno y coseno: cos 73 1 1 . cos x - — . sen x = ~ 1 . sen x = ~ . cos x- sen Ecuaciones e Inecuaciones Trigonométricas R19 593
Reconocemos que se trata de un coseno suma: cos n ®P= 3 Usando el conjunto de solución del coseno: n n x + — = 2kn ± — 6 3 71 71 71 71 - , x = - - g [0 ; 2ti) O z 3ti 13ti — , x = — g [0 ; 2ti) Z O n Luego las únicas soluciones en [0; 2n) son: x = — O 3n 2 5n suma= " 23.- Reemplazando: cos 2x = ( cos x + sen x)(cos x - sen x) en la ecuación, se tendrá: (cos x + sen x)(cos x - sen x) - (cos x + sen x) = 0 Factorizando: (cos x + sen x)(cos x - sen x - 1) = 0 i) cos x + sen x = 0 => 71 ’p"’ 4 5n n x = kn- — 4 p 4 71 4 3n 4 ii) cos x - sen x - 1 = 0 '2 cos = 1 71 ®P= 4 Esto significa que la solución general es: 71 71 — = 2tat ± — 4 4 n n x = 2kn± ~-~ 4 4 Si:k = —1 = >x- , x = 2ti 2 Si:k = 0 = x = 0 , x = 2 Si:k = l = „ 3tt =>x = 2ti , x= — De este esquema se reconoce que el mayor negativo es: n 4 RACSO If BCITD1BI R19 15941 Problemas de Trigonometría y cómo resolverlos
Asimismo el menor pos tivo es: x = — Tt Suma = — 24.-Nuestra estrategia consistirá en transformar la ecuación dada en una multiplicación de términosque dependan de x. Para ello recurrimos a las identidades trigonométricas: Despejando en la ecuación dada: sen 2.x + 2 cos 2x . sen x = 0 Aplicando la identidad del sen 2x enf): 2 sen x . cos x + 2 sen x . cos 2x = 0 3x x Factorizando: 2 sen x (eos x + cos 2x) =0 => 4 senx. cos — . cos ~ =0 2cos^pcosj Analizando cada factor: i) sen x = 0 - > x = 0 3x 3xn 71 ii) COS — = 0 => — = — => X = — X X 71 iii) eos— =0 => — = — =» x = n 71 .’. La menor solución positiva es : x = ~ 25.- Transformamos a una suma y una diferencia de cosenos los sumandos del 1er miembro de la igualdad, obteniéndose: cos 7x + cos x + cos 3x - cos 7x = cos 4x + cos 2x cos 3x-cos 2x = cos4x-cosx v v y V-----~V--—-* Transformando a producto cada miembro, se tiene: x 5x 3x 5x - 2 sen — sen — = - 2 sen — sen — 5x r 3r rl 5x x Factorizando: sen — sen^-sen^ =0 => sen — . 2 sen ~ cos x = 0 tranformamos a producto Igualando a cero cada factor se obtiene: Ecuaciones e Inecuaciones Trigonométricas
5x 5x 2n 4n i) sen— =0=> — = 0;7t,27t => x = 0, —, ~ .. x x ii) sen — = 0 => — = 0; ti; 2ti => x = 0, 2ti, 4ti ... n 3% 5n iii)cosx = 0 => x= — 2ti ti 9ti La suma es: + — = — Las dos menores soluciones 2ti 71 positivas son: — y — 26.- En este problema la única restricción es para: senx * 0 => x * im; it E Z Convertimos los ángulos en radianes a grados sexagesimales: sen3x -------- + 1 = tan 20°. tan 40°. tan 60°. tan 80° ... (1) senx----' ’ Recordamos la identidad especial: tan 3a = tan a. (60° - a). tan(60° + a) Para: a = 20°, se tiene que: tan 60° = tan 20°. tan 40°. tan 80° ... (2) sen3x sen3x , Reemplazando (2) en (1): ~EC1U: + 1 = tan 60°. tan 60° => = (tan 60°) - 1 2 1 Aplicando la identidad del sen 3x: 2 cos 2x + 1 = (~J3) - 1 => cos 2x = 2 Se debe cumplir que: 2x = 60°; 300°; 420°; 660°; 720° => x = 30°; 150°; 210°; 330° ; 390° -... 390° e [0o; 360o] k--------V--------> La suma de soluciones es: 720° = 4ti rad 27.- En este problema transformamos la expresión en otra que se exprese como el pro- ducto indicado de factores, para lo cual recurrimos a las identidades de transformación le si una de cosenos a producto. Veamos: cos x + cos 9x - 42 . cos 5x = 0 2 cos 5x . cos 4x - -J2 cos 5x = 0 F actorizando nos queda: cos 5x(2 cos 4x - -J2 ) = 0 Analizando cada factor: i) cos 5x = 0 5x=(2k+l)~ x = (2k + l)^ Problemas de Trigonometría y cómo resolverlos RACSC BDITOBBB
Si: k = O => x = 10 •" r- 72 n ii) 2 cos 4x - v2 = 0 => cos 4x = => 0p = — 71 kit 71 Usando la formula general para el coseno: 4x = 2kn + ~ => x = ~ ± “ ,ke Z Si: k = 0 => x = 77 ... (2) lo De (1) y (2), las dos menores soluciones positivas son: x=10;x=Í6 •• Suma»-^ 28.- La restricción de la ecuación es debido a que interviene la tangente, luego: x x n tan — 3 <=> — * (2k + 1) — => x * (2k + l)7t x . sen— Usando identidades en la ecuación: 2l2sen—-cos—j =------2. ...(*) cos— 2 x Eliminando sen ~ e igualando a cero para no perder soluciones deducimos que: x x ík = 0=*x=0 ...(1) => sen 7=0 => — =kit => x = 2kn j t . 2 2 [k = l=>x = 27te[0;n] xl 2 x En (*) nos queda: 4 cos ~ =----— => 4 cos ~ = 1 cos— 2 1 2n Degradando: 2(1 + cosx) = 1 <=> cosx = - ~ => x = ~ ... (2) 2ti De (1) y (2) concluimos que las únicas soluciones en [0; ti] son: x = 0, x = — 2n Suma = ~ 29.- La única restricción en la ecuación es por parte de la cotangente: cot x 3 si: x * nn => x * 0; n Expresando en términos de senos y cosenos: Ecuaciones e Inecuaciones Trigonométricas
COS a 2 2 2 2 2 6----5— - 4 cos x = 1 => 6 cos x - 4 sen reos x = sen x sen x => 3 (2 eos2 x) - (3 sen x cos x)2 = — (^sen2 x) => 3(1 + cos 2x) - sen22x = — (1 - cos 2x) dobfe arco doble doble Expresando en términos de "cos 2x" ,11 , 3 + 3 cos 2x - (1 - cos'Vx) = ~ - — cos 2x => 2 cos'zx + 7 cos 2x + 3 = 0 Y factorizando nos queda: (2 cos 2x + l)(cos 2x + 3) = 0 Igualando a cero cada factor: i) cos 2x + 3 = 0 => cos 2x = - 3, esto es absurdo pues: -1 < cos 2x < +1 „ • 1 2it 4it ti 2n ii)2cos2x + 1 = 0 => cos2x = - — => 2x= —; — => x= ~ — i . i - Tt 2rt La suma de soluciones es: — + — = Tt 3 3 30.- Nuestra estrategia consistirá en expresar todo en términos de tan x; veamos: tan---tanx 3 tan x + --------------2 = 0 71 1 + tan—.tanx 4 => 3 tan x + i +1 tanx - 2 - 0 ---(*) Determinemos las restricciones de la ecuación en el intervalo (0;ti) : Tt Tt 3Tt i) tan x 3 # si: x * (2n + 1)— => x*~ ; — 3n 7n ii) 1 + tan x * 0 => tan x * -1 => x * — ; — 4 4 Efectuando las operaciones indicadas en (*), tendremos: -x- 7 3 tan-xi- 3 tan x + 1 - Xasfx - 2 - 'iXaxfx = 0 7 3 tan x = 1 V3 tan x = ±--- 3 c- . V3 Si: tan x = + Tt ~6 c- . Si: tanx = ---- 3 5n T El producto de soluciones es: Tt 6’ 5n T 5n2 36 Problemas de Trigonometría y cómo resolverlos RACSO BDITO11I
31.- Determinemos primero las restricciones de la ecuación en el intervalo dado(0;n): i) Si tan x 3 => x*(2m +1) — n ii) Si tan 3x 3 => 3x * (2/7 + 1)— 2 n ti 5ít ** 6; 2; ~6 Ahora, recordando que: tan a + tan P = sen(a + P) o (del arco compuesto) cosa.cosp ' r sen(3x + x) cos3x.cosx = 4 sen x sen 4x = 2(2 sen x. cos x). cos 3x arco doble => 2 sen 2x . cos 2x = 2. sen 2x . cos 3x => sen 2x[cos3x-cos2x] = 0 transformamos a producto sen 2x -2sen-| -sen-^ Igualando a cero cada factor: 1) sen 2x = 0 => 2x = 0, n, 2n X X 2) sen— =0 -> ~ = 0,n 5x 5x 3) sen~ = 0 => — = 0,n,2n La diferencia de soluciones: r\ 71 . _ => x = 0; 2 'n - - - > x = 0; 2n „ 2n 4n 4n 2n 2n 5 ' 5 = 5 (no son soluciones) . (no son soluciones) .. (si son soluciones) n = 0 32.- La ecuación dada se puede transformar si aplicamos la identidad del cos 2x : sen 5x + c< = sen x + c< sen 5x = sen x sen 5x - sen x = 0 (5x+x\ I5x-x\ sen I—2— I sen I—2— / = 0 => sen 3x. sen 2x = 0 Igualando a cero cada factor, determinamos los valores de x en I 0;^ i) Si: sen 3x = 0 => 3x = nn + (-l)’\aic-serT(0) => nn x~ 3 Haciendo: n = 0 —» x = 0 a n = 1 —» x = n/3 Ecuaciones e Inecuaciones Trigonométricas
ii) Si: sen 2x = O => 2x = nm +~£l)!ítarC'sén (0) => x = Haciendo: m = 0 —> x = 0 a m = l —> x = n/2 Número de soluciones — 3 33.- Nuestra estrategia consistirá en transformar la ecuación dada en otra expresada en términos de cosenos. Para ello recurriremos a la siguiente identidad especial: cos2a - sen2p = cos (a + P). cos(a - P) Aplicándolo en la ecuación dada: cos 6x. cos 4x = cos 4x => cos 6x. cos 4x - cos 4x = 0 Factorizando nos queda: cos 4x . (cos 6x - 1) = 0 Igualando a cero cada factor, determinamos los valores de x en n i) cos4x = 0 => 4x = (2k + l) — => x = (2k + l)~ 1=0 => x=j e(o; f) * = 1 => x=^g(o; 2^ k=2 => ii) cos 6x - 1 = 0 => cos 6x = 1 6x = 2fat kn Luego, las soluciones en ^0; 2 son: k = 0 => x = 0 e^O; 2^ k = l => x = 2 g^O; 0 i-2 x-^(o,=) 7137171 x = — * x = — : x = — 8' 8 3 Número de soluciones = 3 34.- Como en la ecuación intervienen tan x a cot x, entonces la restricción será: x * —, k g Z (ángulo cuadrantal) 1 2 4 De la ecuación: 3 tan x = ---- => tan x = — => tan x = ±—- tanx 3 3 eP = ± TT Usando la formula general de la tangente: x = kn ± TT 6 VkG Z. R19 600 Problemas de Trigonometría y cómo resolverlos RACSO
Dando valores a k, determinamos los valores de x en [0; 4rt]: k = 0 — Tt * *1= 6 A k = l = 5n > x2= 7n *3= 6 k = 2 = llTt > x4= 6 , x5= 13tt 6 A k = 3 — 17ti 6 6 19n *7= ~6~ k = 4 — 23ti > XK= ,x = B 6 25rt [0; 4ti] Número de soluciones = 8 35.- Transformamos la expresión aplicando la identidad: cos 3a = cos a[2 cos 2a - 1] De este modo, si hacemos: a = 4x => cos 12x = cos 4x [2 cos 8x -1] Reemplazando en la ecuación, tendremos: 1 + cos 4x(2 cos 8x - 1] = 2 cos 8x Transponiendo y agrupando términos: cos 4x[2 cos 8x - 1] - [2 eos 8x - 1] = 0 Factorizando nos queda: (2 cos 8x - l)[cos 4x - 1] = 0 Igualando a cero cada factor, determinamos los valores de x en : Tt 1) cos 4x - 1 = 0 => cos 4x - 1 => 4x = 0; 2ti => x = 0; ~ => 2 soluciones 1 ti 5tt 7n 11ti 13tt 2)2cos8x-l=0 => cos8x=— => Sx= —r~ O O J Tt 5ti 7ti Un 13ti X= 24' 24' 24' 24 ' 24 13ti r Puesto que la solución: e 0; , concluimos que: Existen 6 soluciones 36.- Para transformar la ecuación dada, aplicaremos la siguiente identidad auxiliar: 42 sen^x + ^j = sen x + cos x 2 Reemplazando en la ecuación nos queda: 2 sen x + senx-sen3x = 0 tranformamos a producto 2 => 2 sen x - 2 sen x . cos 2x = 0 => 2 sen x.(sen x - cos 2x) = 0 Ecuaciones e Inecuaciones Trigonométricas
Entonces: i) sen x = 0 => x = 0, x = n (no son soluciones) ii) sen x = cos 2x => sen x = 1 - 2 sen x => 2 sen x + sen x - 1 = 0 Factorizando nos queda: (2 sen x - l)(sen x + 1) = 0 Igualando a cero cada factor, determinamos los valores de x en (0;n): iii) sen x = 1/2 => x = n/6; 5n/6 g (0;n) iv) sen x = -1 => x = 3n/2 e (0;n) El número de soluciones es 2 37.- Sabemos que para fines de «degradación», tenemos: 4 sen3 x=3 sen x - sen 3x 4 eos3 x = 3 cos x + eos 3x Multiplicamos por 4 para aplicar estas identidades, así la ecuación se transforma en: 3 3 sen 3x( 4 sen x) + cos 3x(4 cos x) = 4 cos2x sen 3x(3 sen x - sen 3x) + cos3x (3 cos x+cos3x) = 4 cos 2x Efectuando los productos indicados y ordenando, tendremos: 3(pos 3x-cos x + sen 3*-sen x) + cos23x-sen23x = 4 cos 2x arco compuesto arco doble 3 cos 2x + cos 6x = 4 cos 2x => eos 6x-cos 2x =0 => - 2 sen 2x . sen 4x = 0 transformando a producto Igualando a cero cada factor, determinamos los valores de x en [0;n]: nn i) Si sen 2x = 0 => 2x = nn => x = — ,ne Z mn ii) Si sen 4x = 0 => 4x = mit=> x = —— ,tneZ 4 mu Pero: Sj c 82 => x = n n 3n Si hacemos: tn = 0; 1; 2; 3; 4, deducimos que: x = 0; ~~; — i rc 4 2 4 Existen 5 soluciones Problemas de Trigonometría y cómo resolverlos RACSO D1TD1II
38.- La ecuación la planteamos estableciendo las siguientes restricciones: i) La tan x y la sec 3, si x # (2k +1) — ,ke Z Tt Tt ii) La tan 2x 3, si 2x # (2n + 1)— => x * (2n + 1) — ,ne Z Para resolver la ecuación aplicaremos la siguiente identidad: sen(a + P) sen(2x + x) sen3x tan a + tan p =-------— => ----------- =------- cosa.cosp cos2x.cosx cosx Sustituyendo en la ecuación dada: sel sel ___________ ca^x = cos2x. cos 2x = 1 i) sen 3x = 0 3x = - 2?r; -rt; 0; Tt; 2ti 21171 Tt 2ti 3 ; ~3 i) cos 2x = 1 , solo se cumple para: 2x = 0 x = 0 Son 5 raíces 39.- Transformaremos la ecuación aplicando la fórmula para degradar el exponente cuadrático: 2 sen2a =1- cos 2a cos 2x = 1 - cos 2 ^-x cos 2x = 1 - cos cos 2x = 1 - sen 2r sen 2x = 1 - cos 2x sefbx cos x =v2fsen De donde deducimos que: i) sen x = 0 x = 0; Tt; 2?r ii) cos x = sen x => tan x = 1 5rr 4 son 5 raíces 40.- Se plantea la siguiente restricción: i) ex - sec(x) = sec(x) - 1 => para que la sec x 3 => x # (2k + 1) — ,ke Z Ahora procedemos a resolver la ecuación dada: 1 1- cos x + sec x - 1 = cos x - 2 sen x => 0 = - ---- + 2 cos x - 2 sen x cosx Ecuaciones e Inecuaciones Trigonométricas
O = - 1 + 2 cos2x - 2 sen x cos x sen 2x - cos 2.x tan 2x = 1 2x = Tt 5rr 9?r Tt 5Tt gyr x = — • — • — 8' 8 ' X C<O;n) 2 raíces 41.- De la identidad del arco triple tenemos: 3 3 3 sen x - 4 sen x = 8 sen x □ 3 sen x = 12 sen x •• (*) i) Al simplificar se plantea que: sen x = 0 => X = Tt 1 solución 2 ii) Luego de simplificar en (*) nos queda: 1=4 sen x => -i 2 ~ = 2 sen x 1 — = 1 - cos2x 1 ti 5rt 7it llTt cos2x=- => 2x=3;—; — _ Tt 5rt 7rt H71 x~ f>' 3/ 6 4 soluciones Son 5 soluciones 42.- Se plantean las siguientes restricciones: i) La tan x y la cot x 3 ii) La tan 3x y la cot 3x 3 kn si: x # —,keZ ht si: 3x # — => HTt x# —,ne Z 6 Para transformar la expresión dada utilizaremos las siguientes identidades: tan a - tan £ = sen(a-£) cosct.cosP sen(ct-£) cot a - cot B = “ ñ K sena.senp Aplicando ambas relaciones en la ecuación dada, ésta queda así: 4 ’ 4 tan 3x - tan x = cot 3x - cot x sen(3x-x) sen(x-3x) cos3x.cosx sen3x.senx cos 3x. cosx sen3x.senx (*) Analizamos esta expresión en el intervalo dado : i) Al simplificar se plantea: sen 2x = 0 2x = 3rt x= ~2 — No por (i) Problemas de Trigonometría y cómo resolverlos RACSO DITOBBB
ii) Luego de simplificar en (*) nos queda: cos 3x cos x = - sen 3x sen x => cos 3x cos x + sen 3x sen x = 0 => cos(3x - x) = 0 5n 7n => cos 2x = 0 => 2x = — ; — 5n 7n => x —; — Son 2 soluciones 4 4 43.- Se plantean las siguientes restricciones: 7t fot 7t i) tan 2x 3 si: 2x * (2k + 1) — => x # —,keZ 4 2 4 („\ 7t 71 71 —-xl 3 si: ~-x# (2n + l)y => x*nn-~ ,neZ Aplicando las identidades de la tangente de una diferencia de arcos y del arco doble, tendremos: tan--tanx 2tanx _ (1-tanx)2+2tanx _ =* l + tan-.tanx * l~Lin2x l-tan2x 4 •J3 => 1-2 tan x +• tan2x + 2 tan x = 2 - 2 tan2x => 3 tan2x = 1 => tan x = ± Analizando las posibles soluciones, se tiene: J3 n 7n a) Si: tan x = —5- => x = ~ ; — o O ü J3 5ti lln b) Si: tan x = —=- => x = ~ —~ o 00 4 soluciones 44 .- Nuestra estrategia consistirá en transformar la expresión dada en términos de seno y coseno, lo cual se logrará aplicando las definiciones de las identidades auxiliares: vers a = 1 - cos a a cov a = 1 - sen a => (1 + cos x). (1 - sen x) = (1 + sen x) . (1 - cos x) => 1 + cos x - sen x - sen x cos x = 1+ sen x - cos x- sen x cos x 7T 2 => 2 cos x = 2 sen x sen x = cos x 7t Ecuaciones e Inecuaciones Trigonométricas
45 .- Transponiendo términos, la ecuación toma la siguiente forma: 73 - 73 sen2* = 2 sen x - 2 sen3* Factorizando: (1 - sen2*) = 2 sen *(1 - sen2*)... (*) 2 Eliminamos: "1 - sen *" y para no perder soluciones lo igualamos a cero, tal que: 2 sen * = 1 => sen * = ± 1 => 7e(0;n> 3n * = — 6 <0; n) Nos queda de (*): 73 = 2 sen * => Las soluciones en (0; Tt) son: Tt => * = — e (0; Tt) Tt Tt , 2ti ] 2'3' 3 J 2n a * = y e (0; Tt) 46 .- En primer lugar identificamos las restricciones por para de la tangente: Tt Tt i) 5* # (2m + 1) — => ** (2m + 1)~, m e Z Tt Tt ii) 3* # (2n + 1) — => * * (2n + 1) — ,ne Z 2 o A continuación expresamos todo en términos de senos y cosenos sen5*.sen3* cos5*.cos3* => 2 cos 5* . cos 3* = 2 sen 5*. sen 3* transf. a una transf a una suma de cosenos diferencia de cosenos cos 8x + cos 2x = cos 2x - cos 8x => 2 cos 8x = 0 => cos 8x = 0 7t 8*=- Tt 16 La menor solución positiva es: q. 47.- La transformación de la ecuación propuesta se logrará utilizando las medidas de los lados del siguiente triángulo notable: 15° - 75°. Dividiendo entre 2 72 a la ecuación dada: (73 + 1) (73-1) 72 2^2 SenX+ 272 COSX=272 J3 + 1 => sen (*+-)=- Problemas de Trigonometría y cómo resolverlos RACSO
Usando el conjunto de solución para el seno: Tt v Tt v Tt Tt => x + — = kn + (-l)k - => x = kn +(-l)k - - — , k e Z IZ o O IZ Tt 3ti 25it Haciendo: fc = 0: x = ~ k=l: x= ~ k = 2: x= e (0; 2n) Las soluciones son: Tt 3rt 12' 4 48.- Recordamos la identidad: sen 2x + cos 2x =-J2 —^-senx + —=cos2x \J2 j2 Al aplicarla en la ecuación dada, tendremos: Tt Tt -J2 J2 sen(2x + ~ ) = - 1 => sen(Zr + -) = - — Esta ecuación se verifica si: Tt 571 Tt 2x+? = T =” x=2 Tt Tt Tt ii) 2x+ ~ = - ~ => x = -~ 4 4 4 La menor solución positiva es: x = ~ 49.- Efectuando el producto indicado tenemos: 2 3 3 3 2 sen x . cos x +• sen x = sen x - cos x 2 2 Factorizando "cos x": cos x(2 sen x + cos x) = 0 gualando a cero cada factor, analizamos los valores de «x» en el intervalo [O;Tt]: Tt 3it i) cos x — 0 => x = - ; — 2 2 2 2 2 ii) 2sen x + cos x = 0 => 2 sen x + 1 - sen x = 0 => sen x = -1 (absurdo) 3- solución mayor o De este análisis concluimos que: —;——------------ =- = 3 71 solución menor 71 2 Ecuaciones e Inecuaciones Trigonométricas R19 607
SISTEMA DE ECUACIONES 50.- Nuestra estrategia consistirá en relacionar los arcos dados en términos de razones trigonométricas. Lo conveniente en este caso es que las RT elegidas, concuerden con las que figuran en las ecuaciones propuestas. Veamos: 7t Como: x + y=~ => sen y = cos x ... (3) n De (3) en (2) se establece que: sen x = cos x => tan x = 1 => 6p = ~ n Luego el conjunto solución es: x = kn + — , ke Z n n n Reemplazando en (1): y=~-x => y = — -kn - — n y = — -kn , ke Z 3 4 51.- Procediendo con la misma estrategia del problema anterior, se plantea que: Si : x + y = — => sen x = cos y ... (3) 1 71 Reemplazando (3) en (2): 2 cos y = 1 => cos y = ~ => 6p = ~ n Luego el conjunto solución será: y = 2fcn ± — 71 71 71 Reemplazando en (1): x= ~ -y x= 2 3 , Vke Z 52 .- Nuestra estrategia consistirá en construir dos ecuaciones en términos de seno y coseno. Para ello haremos las siguientes transformaciones: Aplicando propiedad de proporciones en (2) se obtiene: tanx tany 2 + V3 1 tanx+tany (2 + 43) +1 tanx —tany (2 + 73)-! sen(x + y) De la identidad: tan x + tan y = sen(x±y) cosx.cosy tanx + tany tan x - tany sen(x-y) Problemas de Trigonometría y cómo resolverlos Si RACSO V|»IDITOBia
tanx + tany sen(x + y) De lo cual concluimos que: ----------= ----------r tanx —tany sen(x—y) sen(x + y) 3 + 73 Y del resultado obtenido en (3), se establece que: , _ . = ~ ... (4) Reemplazando (1) en (4), se deduce que: sen (x + y) = 3 + 73 73 + 1 Tt . sen — ü 73 => sen(x + y)= — Tt 1 1 r Y como: sen ~ ~ => sen (x + y) = ~. V3 o z z De donde se obtiene que: X + y = kn + (l)k. j ...(5) Si ahora hacemos (1) + (5): v n n 2x = fat + (-l)k. - i - O o kn v Tt Tt T +G1)' 6 + ñ V ke Z 53 .- Nuestra estrategia consistirá en transformar las ecuaciones dadas hasta obtener otra en términos de seno. Veamos: De (1) despejamos: 3n x= y +y tanx = tan^-^ + yj Reduciendo al primer cuadrante: tan x = - cot y ... (3) Reemplazando (3) en (2) se establece que: - cot y. cos y = — 2 cosy eos y 3 Recordando la identidad: coty = => -------= — * seny seny 2 2 Sustituimos cos y por la identidad pitagórica, y efectuando nos queda: - 2(1 - sen2y) = 3 sen y => 2 sen2y - 3 sen y - 2 = 0 => (2 sen y + l)(sen y - 2) = 0 i) sen y = 2 (imposible) 1 u) sen y = - — y = kn + (-l)k .f-— 1 keZ Ecuaciones e Inecuaciones Trigonométricas R19 609
54.- Transformamos el sistema de modo que se obtenga una ecuación en términos de «y»: De (1) despejamos convenientemente: x = — + y => sen x = sen + y j Por reducción al IC: sen + J/) = cos y => sen x = cos y... (3) Reemplazando (3) en (2): 2 sen y . cos y = 1 Tt Por arcos dobles: sen 2y = 1 => 2y= (4k + 1) —, ke Z y = (4k + l)^ A x=(4k + 3)^ V ke Z 3 4 4 55.- Tal como se hizo en el ejercicio anterior, transformamos el sistema así: De (1) despejamos convenientemente: x = ~ + y => sen x = sen + y Por reducción al IC se establece que : sen x = cos y ... (3) cosy 1 1 Reemplazando (3) en (2): seny =5 => tan y - => 0p = are tan ~ 11 flA n y = kn + arctan — I a x = kn + arc tanl — I + ; VkeZ 56 .- Transformamos el sistema de modo que se obtenga una ecuación en términos de la tangente. Veamos: De (1) despejamos así: y-x=~ => y=-+x => tany=tan^ + x Por reducción al IC se obtiene: Reemplazando (3) en (2): Recordando la identidad: tan y = - cot x .. (3) tan x - cot x = 2 ...(*) tan x - cot x = - 2 cot 2x Reemplazando en (*) - 2 cot 2x = 2cot 2x = -1 => tan 2x = - 1 Esta ecuación tiene como conjunto solución: 2x = kn - ~ => kn n 2~ 8 VkeZ Reemplazando en (1): kn 3n v=T- 8 ' Vfc6Z R19 610 Problemas de Trigonometría y cómo resolverlos MA RACSO RbDITOKBI
57 .- Utilizando las identidades de la suma y diferencia de arcos, transformamos el siste- ma dado para establecer una relación entre los arcos dados. Veamos: 71 De (1) podemos establecer que: sen (2x - 3y) = sen — J2 Desarrollando el 1er miembro: sen 2x . cos 3y - cos 2x . sen 3y = -O) Reemplazando (2) en (3) se obtiene: sen 2x . cos 3y - sen 2x . cos 3y = •--(4) Haciendo (2) + (4): sen 2x . cos 3y + sen 3y. cos 2x = 2 Reconocemos que el 1er miembro es el desarrollo del seno de una suma de arcos: V3 => sen (2x + 3y) = — 71 ®P= i lí 71 2x + 3y = kn + (-1) ...(5) Haciendo (1) + (5): 71 i, 71 4x = tai+ - +(-l)k-3 kn ti v ti X= T + 16 +<’1) ’ 12 V ke Z 58 .- A partir de las ecuaciones dadas obtendremos una en términos del seno. Veamos: Transformamos a producto la ecuación (2): í x + j/^ í x-y"! 2 senl 2 I - cosí 2~ I = - - - (3) Reemplazando (1) en (3) se establece que: 2 sen cos 71 1 sen 7 = 2 = 1 Teniendo en cuenta que: De esta ecuación se deduce que: x-y 2 = 2kn;V keZ => x-y = 4kn ... (4) Haciendo (1) + (4) se obtiene: 2x = 4/ctt + — x = 2k7t +7 ; V k g Z O Ecuaciones e Inecuaciones Trigonométricas
59 .- Transformaremos el sistema hasta obtener una ecuación en términos de la cotan- gente. Veamos cómo se logra: cosx + cosy 1+2 Aplicamos propiedad de proporciones en (2) se obtiene: —7 Transformando a producto los términos de la fracción del 1er miembro: 2co; 2sei = 3 => cot x+y 2 x-y 2 = 3... (3) Reemplazando (1) en (3): cot x+y 2 71 . COt — o ..(cotj = 73) o . cot = 3 => cot 3 n %=6 +y ti ti — = kn + — => x + y = 2kn + - ...(4) x + y 2 Haciendo (4) - (1): 2y = 2kn y = kn ; V k g Z 60.- Este sistema se puede resolver si transformamos la ecuación (2) a un cosenos, de modo que sea posible obtener una relación adicional para los producto de arcos dados: => 2 eos x+y 2 . cos x-y 2 = 1 ... (3) 71 Reemplazando (1) en (3): 2 cos — . cos x-y 2 = 1 => 2 eos x-y 2 = 1 = 1 De esta ecuación se deduce que: x-y -r- =2kn 2 x - y = 4kn .. (4) Haciendo (1) + (4), obtenemos «x»: 2x = 4krt + 271 3 71 x = 2k7t + — ; V ke Z 2ti Efectuando (1) - (4), obtenemos «y» : 2y = ~ 4kn y = — - 2k7t ; V k g Z 3 3 61 .- Tal como se planteó el problema anterior, transformamos a producto la ecuación (2) con el específico propósito de obtener una nueva relación entre los arcos dados: => 2 cos(x + y). cos (x - y) = 0 ... (3) R19 ¡6121 Problemas de Trigonometría y cómo resolverlos RACSO
2n Reemplazando (1) en (3): 2 cos~ . cos(x - y) = 0 => 2 j . cos(x - y) = 0 => cos(x - y) = 0 => x - y = (2k + 1) ~ . (4) 71 2ti Haciendo (1) + (4) se obtiene «x»: 2x = (2k + 1) — + — 71 71 .-. x=(2k + l)- + - ; VkeZ 62 .- En este problema transformamos el sistema de modo que sea posible obtener una nueva relación en términos del seno. Veamos: De(l) se puede establecer que: sen y — - sen x 2 2 Elevando al cuadrado ambos miembros: sen y - sen x .. (3) Reemplazando (3) en (2) se obtiene: 2 sen x = 0 => sen x = 0 .-. x = kn ; VkeZ 63 .- Este tipo de sistema se resolverá transformando a producto cada ecuación. A partir de esto se obtendrá una nueva relación entre los arcos dados en términos de la tangente. Veamos: rx+y^ I 7ó a) Transformamos a producto la ecuación (1): 2 sen 2 ; . eos l 2 J | = T -(3) x + y 'i 1 72 b) Transformando a producto la ecuación (2): 2 eos 2) . eos l 2 J |=V--(4) í x+ y Dividiendo miembro a miembro (3) (4): tanl J | = 73 => 6P= 71 3 x+y 71 Luego el conjunto de solución será: —= kn. + ~; V ke Z x + y = 2kn + ^r ;VkeZ 64 .- Nuestra estrategia consistirá en transformar el sistema dado de forma que se obten- ga una relación en términos de «x» . Veamos: 2 2 2 2 Usando diferencia de cuadrados en (1): (sen x cos y)(sen x - cos y) = 1 . . (*) Ecuaciones e Inecuaciones Trigonométricas
•y 7 Reemplazando (2) en (*) se obtiene: 1. (sen x - cos y) = 1_(3) Haciendo (2) + (3) : 2 sen2x = 2 => sen2x = 1 => sen x = ± 1 x = (2fc + 1)2 ; V 1 e Z 65 .- Este sistema requiere ser transformado a una nueva ecuación en términos del cose- no. Para ello debemos apelar a un grupo de identidades especiales. Veamos: Transformando a producto la ecuación (1): Transformando a una suma la ecuación (2): „ (x + y 1 [ x-y 1 . 2 sen ---2- . eos ---— = 1... (3) l 2 J l 2 J 3 cos(x + y) + cos(x - y) = ~ ...(*) 2l “I Aplicando en (*) la identidad: cos(x - y) = 2 cos I „ 1-1 . 21 X-y => cos(x + y) + 2 cos -— 7 2cos - - cos(x + y)... (4) De la ecuación (3) despejamos: Reemplazando (5) en (4) obtenemos: 1 O 2f X+y) 2sen ---— l 2 J 5 - - cos(x + y) ... («) Y recordando que: 2 sen2í X + ^ 1=1- cos(x + y) 1 5 Al aplicarlo en (**), se obtiene: i_cos(x+y) = ~ cos(x + y) ... (6) Resolviendo (6), se obtiene: 1 cos(x + y)= - n x + y = 2kn ± — V ke Z 66.- La estrategia consistirá e transformar el sistema dado de modo que se obtenga una nueva relación en la que solo figuren las tangentes de «x» e «y», lo que permitiría iden- tificar el tipo de relación que existe entre dichos arcos. R19 614 Problemas de Trigonometría y cómo resolverlos RACSO JP-DITO...
Aplicando la identidad de la tangente de una suma de arcos en (2), tendremos: tanx + tany 4 1-tanx.tany = 3 - - - (3) 1 4 1 Reemplazando (1) en (3): 1_tanx tany = 3 => tan x . tan y = - ... (4) 1 Resolviendo (1) y (3) encontramos que: tanx = tany =~ => y = arc tanl~l Luego el conjunto solución será: y = kn + arc tan (y I ; V k e Z 67.- En base a la aplicación de identidades recíprocas, debemos obtener los valores de la tan x y tan y, para luego y a partir de éstos, identificar las soluciones generales. Esto lo empezaremos expresando (2) en términos de tangentes: 1 tanx tan y = i/3 tan y = i/3 . tan x ... (3) Reemplazando (3) en (1) se obtiene: tan x + V3 tan x = 1 + 43 n Transponiendo términos: tan x = tan x = 1 => 0p = Luego el conjunto solución para x: x = fai+7; V lee Z 4 Reemplazando tan x = 1, en (1): 1 + tan y = 1 + J3 =* tany=V3 6p=^ Luego el conjunto solución para y: y = nn + , V 11 e Z J 3 68.- Este sistema lo resolveremos transformando el sistema en una ecuación de 2do grado y en términos de la tan x. Veamos: Aplicando la identidad recíproca cot y = -- en (1): 1 O (3) tail A — Z tany 1 De (2) despejamos convenientemente: tan y = 2 - * . (4) 1 n t tanx „ Reemplazando (4) en (3). tan a + i 2 — z. Ldll A -t- o . ^1— Z 2 . tanx-1 tanx 4 Ecuaciones e Inecuaciones Trigonométricas R19 615
2 71 Efectuando y factorizando nos queda: (tan x - 1) = 0 => tan x = 1 => 0p = — El conjunto de solución sera: n x = kn+ — ke Z 4 SOLUCIONES PARTICULARES 71 69.- Como: x + 2y = ~ =x sen 2y = cos x... (3) V7 Reemplazando (3) en (2): sen 2y = . sen y Sustituyendo el 1er miembro por la identidad del sen del arco doble: \ V7\ 2sfeqv.cosy = — -SetJ^y V7 cosi/= — 3 4 sen y * 0 De estas relaciones se concluye que: y = arc cos /7 4 Tt Reemplazando en (1): x + y + y = y => x + y + arc cos J7 4 71 7 n Tt x + y = - are cus 70.- Nuestra resolución la iniciaremos aplicando la identidad de la tangente de una diferencia en la ecuación (2), para de allí obtener la tangente de la suma de arcos: Tt tan---tanx 4 ----- = tan y ,---71------3 1 + tan—.tanx 4 1-tanx T+tañx =tany 1 - tan x = tan y + tan x . tan y tanx + tany 1-tanx. tan y tan(x + y) = 1 x + y = 7 • • (3) Haciendo (1) + (3), se tiene: 5ti 2X= 12 5n 24 Haciendo (5) - (1), se tiene: 71 2y= 12 5 RACSO WIDITO1IÍ R19 ¡616| Problemas de Trigonometría y cómo resolverlos
71.- Nuestra estrategia consistirá en degradar la ecuación cuadrática para luego obtener una ecuación en términos del coseno de una diferencia de arcos. De allí se establecerá una nueva relac jn entre los arcos participantes. Veamos: o o J J Hacemos 2x (2): 2 sen x + 2 sen y = ~ => 1 - cos 2x + 1 - cos 2y = — 1 1 => cos 2x + cos 2y = - => 2 cos(x + y). cos(x - y) = — .... (3) * í ” 1 1 Reemplazando (1) en (3): 2 cos I I cos^x " V) = . cos(x - y) = cos(x-y) = 6-V2 5n x-y = 2kK± — . ...(4) 5ti 71 Haciendo (1) + (4): 2x - 2kn + ~ + ~ 5n n x = kn± — + —; V keZ Para obtener la menor solución positiva hacemos: k = 0 71 4 72.- Procediendo tal como lo hicimos en el ejercicio anterior, tendremos: 2 2 Hacemos 2 x (2): 2 sen x + 2 sen y = 2 => 1 - cos 2x+ 1 - cos 2y = 2 => cos 2x + cos 2y = 0 => 2 cos(x + y). cos(x - y) = 0 . (*) De (1) reemplazamos en (*): 2 cos 6 . cos(x - y) = 0 71 Por condición: cos 0*0, entonces: cos (x - y) = 0 => x - y = — ... (3) 71 0 7t Haciendo (1) + (3) encontramos: 2x = 0 + — .-. x = — + — 73.- Lo que haremos es transformar a producto la ecuación (1) para obtener el coseno de la diferencia de arcos y a partir de allí obtener una nueva relación entre los arcos: 2 sen(x + y). cos (x - y) = -J2 ... (3) Reemplazando (2) en (3): 2. 1 . cos(x - y) = J2 J2 cos(x-y)=—. .(4) 2 Ecuaciones e Inecuaciones Trigonométricas
x-y= 7 -- (5) Como: x, y e \0; a sen(x + y) = 1 => x + y = — ... (6) 3tt Tt Resolviendo (5) y (6), obtenemos: x = “ y = — 74 .- Transformaremos el sistema a fin de obtener ecuaciones en términos de la suma y diferencia de los arcos dados. Veamos: > Hacemos (1) + (2): sen x. cos y + cos x . sen y = 1 Tt => sen(x + y) = 1 => x + y= - 1 Efectuamos (1) - (2): sen x. cos y - cos x. sen y = - — ...(3) 1 sen(x-y) = - - x - y = arc sen Resolviendo (3) y (4): Tt fl y = ~ + arc sen — 4 I 5 75 .- Tal como se hizo en el ejercicio anterior, transformamos el sistema a fin de obtener ecuaciones en términos de la suma y diferencia de arcos. Veamos. Haciendo (2) + (1): cos x. cos y + sen x. sen y = 0 Tt => cos(x-y) = 0 => x-y= ~ (3) Efectuamos (2) - (1): cos x. cos y - sen x. sen y = 1 => cos (x + y) = 1 => x + y = 0... (4) Tt Tt Resolviendo (3) y (4) obtenemos: x = — A y = - 76 .- Transformamos el sistema de modo que se obtenga una ecuación en términos del coseno. Veamos: 2 1 Haciendo (1) - (2), se obtiene: sen y - 2 cos y = - — 2 2 Aplicando en (*) la identidad pitagórica: sen y = 1 - cos y, se obtiene: 9 1 9 => 1 - cos y + 2 cos y = - ~ => 4 cos y + 8 cos y - 5 = 0 R19 618 Problemas de Trigonometría y cómo resolverlos RACSO BDITOUI
Factorizando, se establece: (2 cos y + 5)(2 cos y - 1) = 0 Igualando a cero cada factor: 5 1 cos y = - ~ (No puede ser) v cos y = ~ => y = 60° (H 1 1 . Reemplazando en (2): 2l — l-senx=~ => senx=~ x = 30° 77 .-Transformamos el sistema de modo que sea posible obtener un sistema equivalente en términos de cosenos. Para ello aplicamos las identidades básicas en (1): ----2—— =1 => cos x. cos y = sen x . sen y... (3) cosx.cosy 1 Reemplazando (2) en (3): cos x. cos y = ~ ... (4) 1 Haciendo (4) + (2): cos x. cos y + sen x. sen y = — 1 71 => cos(x-y)=~ => x-y=-...(5) Efectuando (4) - (2): cos x . cos y - sen x . sen y = 0 71 => cos(x + y) = 0 => x + y= - ...(6) Resolviendo (5) y (6): y = — 78 .- El sistema tiene una dependencia de «a» que contribuye en definir signos para el seno y coseno. Por tal razón procederemos a transformar de manera que se obtenga el seno y coseno de la suma y diferencia de los dos arcos. Veamos: Sabemos que: sen a. cos 0 < 1, V (a, 0| e R => sen x. cos 2y < 1... (3) Reemplazando (1) en (3): c2 + 1 < 1 c2<0=> fi2 <0 fi2 =0 (No puede ser) (Si)V De este análisis concluimos que: sen x. cos 2y =1... (I) Asimismo en (2), nos queda: cos x . sen 2y = 0 ... (II) Ecuaciones e Inecuaciones Trigonométricas
Haciendo (I) + (II): sen x. cos 2y + cos x. sen 2y = 1 => sen(x + 2y) = 1 Efectuando (I) - (II): sen x. cos 2y - cos x. sen 2y = 1 => sen (x - 2y) = 1 71 Resolviendo (4) y (5) se obtiene: x = — A y = 0 7t => x + 2y= — ...(4) > x-2y = ~ ... (5) 79 .- Transformaremos el sistema para obtener una ecuación en términos del seno y de ella obtener los valores correspondientes para «y». Transformamos (1): sen x . —-— =1 => sen x = cos y ... (3) cosy 1 Reemplazando (3) en (2): sen y. cos y = — 1 Multiplicando por "2" ambos miembros: 2 sen y. cos y = — Por arco doble: sen 2y = — => 2y = — v 2y = — 2 6 6 ti 5n v y=12 80 .- Transformamos el sistema para expresarlo en términos de cosenos de la suma y diferencia de arcos. Veamos: Haciendo (2) + (1): cos x. cos y + sen x. sen y = '6+V2 4 cos(x-y) = 6+V2 4 7t x-y= — ... (3) Efectuando (2) - (1): cos x. cos y - sen x. sen y = '6-V2 4 cos(x + y) = - 6-V2 4 7ti X + y= 12 "-(4) Resolviendo (3) y (4): x = j e ( 0; A y = e < 0; RACSO PIDITOB.B* R19 620 Problemas de Trigonometría y cómo resolverlos
81.- Multiplicando por (3) la ecuación (2): 18 sen x - 3 tan y = 6 ... (3) Haciendo (1) + (3): Reemplazando en (1): 20 sen x = 10 -Js V3 senx= — 2 + 3 tan y = 4 J3 tan y = V5 n 3 y= «13 82.- Aplicando la propiedad de proporcionalidad en (1): tanx + tany tanx-tany (J3 +1) + (J3-1) (V3 + l)-(j3-l) tanx + tany tanx —tany = J3 .. (3) Pero: tan x ± tan sen(x + y) y _ cosx.cosy => sen(x + y) sen(x-y) " J3 . (4) (2) en (4): sen (x + r~ 71 y) = v3 sen - => 71 sen — = o 1_ 2 => sen(x + y) = — y (x + y)e(^,n) => x + y=- 2n 3~ •(5) 5n Haciendo (2) + (5): 2X 6 x, _ 5 Efectuando (5) - (2): 3n 2y= ~6 y 3 83.- Este sistema se resolverá por eliminación y sustitución de algunas RT. Veamos: Dividimos (1) (2): senxseny se ni/.se 112 2 => sen z = 2 sen x... (4) Reemplazamos (4) en (3): (sen x)(2 sen x) = 2 2 => 2 1 sen x = ~ 4 => sen x = 2 => x = 30° 1 Reemplazando en (3): — 1 . sen z = — > sen z = 1 => z=90° Ecuaciones e Inecuaciones Trigonométricas
1 i i ,'2 -J2 Reemplazando en (1): I 2 I sen V = => sen V = ”2” =* V = 45* Finalmente: x + y + z = 165" INECUACIONES TRIGONOMÉTRICAS 84.- Recurrimos a la identidad del arco doble, para transformar la inecuación y elaborar otra que sea factoiizable. Veamos: 2 sen 3x . cos 3x < 5 sen 3x => sen 3x (2 cos 3x - 5) < 0 ...(*) Recordamos que: Multiplicando por 2: Restando 5: Por lo tanto en (*): -1 < cos 3x < 1, v k g R -2<cos3x<2 -7< 2 cos 3x-5 <-3 U(+) s«K3x COVo <0 sen 3x > 0 De la circunferencia trigonométrica (C.T), se observa que: 0 < 3x < 71 Pero por ángulos coterminales se tiene: O + 2Jc7i<3x<7t + 2krt; V ke Z Despejando: x e 85.- Transformamos a producto la suma de senos, y tendremos: í 7x + 3x í 7x — 3x 2 senl —-— I cosí —-— I < 2 sen 5x => sen 5x . cos 2x < sen 5x Transponiendo términos y factorizando: sen 5x(cos 2x - 1) < 0 ...(*) Recordamos que: -1 <cos2x< 1, V te R => - 2 < eos 2x-1 <0 <6 Por lo tanto en (*) se debe cumplir que: sen 5x > 0 De la circunferencia trigonométrica (C.T), se observa que: 0 < 5x < n Pero también por ángulos coterminales se tiene: 2kn < 5x < ti + 2kn, VkeZ /2kn. ti . 2fc7t\ XG \5~' 5+~57 RACSO IOITO11I R19 622 Problemas de Trigonometría y cómo resolverlos
86.- Transponiendo términos y factorizando tendremos: (2 senx + ^3 )(2 sen x - ) > 0 Analizando por el método de los puntos críticos: i) 2 sen x + v3 = 0 => sen x - —y- 2 sen x - -J2 =0 => Analizando en la C.T se concluye que: /n3n\ /4n5n\ XG \4Z 4 / v \ 3 ' 3 / 87.- Transformamos la expresión en términos de coseno. Veamos: 1 - cos26x 4- 1 - cos 6x > 2 => - cos26x - cos 6x > 0 => cos 6x(cos 6x + 1) < 0, Analizando por separado tenemos: cos 6x . (cos 6x + 1) < 0 cos 6x(cos 6x + 1) = 0 a) De la primera relación: 71 0 <x < ~ => 0 < 6x < 3ti => -1 < cos 6x < 1 => 0 < eos 6x+l ^2 >0 cos 6x < 0 ti 3ti 5ti 7n ti 6xG<y, ^) <-£,—) => *e<12 -(I) Por condición: x g (0; => x G (~ , — ) b) De la segunda relación: eos 6x = 0 u cos 6x = - 1 71 371 571 6x= ~ u 6x = ti, 3tt • 71 71 571 X= 12'4'12 u 71 71 X= 6'2 (II) (III) Ecuaciones e Inecuaciones Trigonotn tricas R19 623
De (I), (II) y (III): 71 Tt 12' 4 5n Tt 12' 12 88.- De álgebra sabemos que: | a | <b <=> -b<a<b Aplicando esta propiedad en el problema: - 1 < sen x + cos x < 1 Dividiendo por 2 a todos los términos: Identificamos el desarrollo del seníx+5 11 V3 1 y < — . sen x + cos x < cosí^) -|<sen(x+f)<|... O Para que se cumpla la condición (3), de la C.T. se observa que: 71 71 71 6<X+ 3 <6 Asimismo los otros intervalos de solución se obtiene sumando kn. vueltas a los valo- res extremos: 71 Tt 71 + kn<x + — < kn+ ~ b o o => re re ^(2k —1)~; (6k-l) VkeZ 89.- Factorizando tenemos: cosx (cos x - 2) <0 ...(*) Analizamos cada factor para que verifiquen la desigualdad. De la condición tenemos: _ -Tt - 2n < x < 0 => -1 < cosx < 1 Sumando -2: - 3 <cos x-2 <-1 ' <6 -2n 0 Reconociendo que este término es negativo, en (*) concluimos que: cos x > 0 Finalmente en la C.T. se observa que: .-¿A RACSO 'JPbditobbb R19 624 Problemas de Trigonometría y cómo resolverlos
90.- Transformamos la expresión en otra que presente un producto indicado de factores. Transponiendo términos: Transformando a producto: Dividiendo por -2: eos 7*-eos 3x <0 a producto - 2 sen 2x. sen 5x < 0 sen 2x . sen 5x > 0 (*) Analizaremos cada factor, empezando por la condición del problema: n 0<x< ~ > 0 <2x <7t => 0 < ^en 2x < 1 >o Si este factor es positivo, en (*) concluimos que: sen 5x > 0 De la C.T. se observa: 5x e ( 0; Tt) o ( 2n; 3ti) ” xe (°; ?)u (¥; ¥) Tt Pero dado que: 0 < x < — 91.- Despejando: cos 2x < cos 2tt 3 Debemos tener en cuenta que: - 1 < coseno < 1 1 cos2x<- — - --(*) Ecuaciones e Inecuaciones Trigonométricas R19 625
2ti 4n — + 2kn<2x< — + 2kn ; VkeZ xg [f+fcK; ^+kn\ 92.- Empezaremos planteando las siguientes restricciones: i) sen x * 0 => x * mu, tneZ 71 ii) cos x * 0 => x * (2n + 1) —, n g Z kn De (i) o (ii) se tiene que: x * V k g Z Aplicando las identidades del arco triple, tendremos: senx(2cos2x+l) cosx(2cos2x-l) senx + cosx < Simplificando: 2 cos 2x + 1 + 2 cos 2x - 1 < 2 kn De la restricción: x * — => 2x*kn =: cos 2x * cos(/c7t) cos 2x * ± 1 Para que se cumpla la condición (*), usamos la circunferencia trigonométricas (C.T.): 7t 5ti — <2x<n u tt<2x< — Pero también por ángulos coterminales se tiene: — + 2Á7t < 2x < 7i + 2Á7t u 7t + 2kn < 2x < — + 2fat Despejando: xg ^ + kn; ^ + kn^ VJ ^ + fcn; ^ + kn^, VkeZ 93.- En este caso es imprescindible recordar la identidad del arco triple (degradación) y del arco compuesto. 3 3 4 sen x = 3 sen x - sen 3x A 4 cos x = 3 cos x + cos 3x cos(3x + x) = cos 3x . cos x - sen 3x . sen x ,, , , „ /'3senx-sen3x’l „ /'3cosx-cos3x V 1 Reemplazando: sen 3x1-------------1 - cos 3x1------------l> — ^RACSV ^*BDlToa.ki R19 626 I Problemas de Trigonometría y cómo resolverlos
Efectuando y ordenando: -3(cos 3x-cos x-sen 3x-sen x) - (sen* 23x+cos23x) >2 cos(3x+x )=cos 4x Sustituyendo por las identidades: Multiplicando por (-1): - 3 cos 4x > 3 3 cos 4x < - 3 => cos 4x < - 1 De lo cual se deduce que: cos4x < -1 absurdo pues —I<cos 4x<l u cos 4x = - 1 cos 4x = - 1 Resolviendo esta ecuación trigonométrica: 4x = (2/c ± 1)ti T ± 4; v fcG z 94 .- Empezamos por las restricciones: Tt 371 4(1 + tan2x) - 8 tan x < 1 71 i) Tanto para la tan x y sec x; 3 , si x * (2n + 1) ~, n 6 Z 2 Aplicando la identidad pitagórica de la sec x : Efectuando y factorizando: (2 tan x - 3)(2 tan x - 1) < 0 1 2 3 2 © r Resolviendo por el método de los puntos críticos: 3 i) 2 tan x - 3 = 0 => tan x = ~ Ecuaciones e Inecuaciones Trigonométricas
1 3 arc tan*2 + kn<x <arc tan2 + kn;Xf ke Z /. x e {are tan~ekn; are tan-^+kn^ 95 .- Sea: a = 3x + — => tan a > 1. Planteando la restricción: tan a 3 si: a * (2n + 1) y, n g Z 71 5n Recordamos que: tan~ = tan— = 1 4 4 PSra que se cumpla la condición (*), de la C.T. se observa que: n 71 4 <a< 2 Así mismo los otros intervalos de solución se obtienen sumando kn vueltas: — + kn < a < — + kn: V k g Z => — 4 2 4 kn<3x < — + kn => 4 xg fe2L + ^E\; vfcez \ 3 12 3 / 96 .- Empezaremos planteando las restricciones: 71 tan x 3 y sec x 3 si x * * (2n +1) —, n g Z Aplicando la identidad pitagórica y del arco doble, tendremos: tan4x + 8tan3x + 2(1 + tan2x) - 8 tan x - 1 < 0 => tan4x + 8tan3x + 2tan2x - 8 tan x + 2 - 1 <0 Agrupando convenientemente: tan4x -1+8 tan3x - 8 tan x + 2 tan2x + 2 < 0 (tan2x - l)(tan2x + 1) + 8 tan x (tan2x - 1) + 2(tan2x + 1) < 0 R19 628 Problemas de Trigonometría y cómo resolverlos RACSO DITOM.BB
Factorizando: (1 + tan2x)[tan2x - 1+ 7 8tanx(tan x —1) ------*---5------ + 2]< 0 1 + tan x Pero: 1 + tan2x * 0, simplificamos : tan2x- 1+ 7 8 tan x( tan x-1) l + tan2x + 2 < 0 7 2 8tanx(l-tan x) Despejando: 1 + tan x <-----—5^-------- 1 ~ < 2 . sen 2x . cos 2x 1 sen 4x > — ... (*) Usando la circunferencia trigonométricas ti 5rt (C.T), se observa que: ~ < 4x < — O o Pero también por ángulos coterminales se tiene: Tt 5n — + 2kn < 4x < — + 2krt; VkeZ 6 6 Despejando: x G 1 2 tanx l-tan2x < 2. 7 7 2 1 + tan x 1+tan x 97 .- Empezamos planteando las restricciones, tanto la cot x y csc x , 3 , si: x*hti, hgZ => x*0;Tu2rt Notamos que el denominador es diferente de cero, pues: csc x < -1 v csc x > 1 Expresando en términos de senos y cosenos cosx COSX +----- -----2 serLY <0 => cos x(sen x + 1) < 0 ...(*) senx Para el análisis de cada factor, empezaremos por la restricción establecida: Si x * nn => sen x * sen(nrt) => sen x * 0 => -l<senx<0 u 0<senx<l => 0<senx + l<l usenx + l<2 Como en ambos casos el factor es positivo, en (*) tendremos que: cos x < 0 ... (**) Ecuaciones e Inecuaciones Trigonométricas R19 629
Considerando el intervalo dado para «x», la relación (**) se determinará usando la circunferencia trigonométrica (C.T), donde se observa que: 71 3ti Pero de la restricción x * ti: 98 .- Observamos que tanto la tan y la cot, en el intervalo dado, no presentan restricción alguna ya que x * 0; ti/2. A continuación transformamos la inecuación aplicando la identidad del arco mitad: tan x = csc 2x - cot 2x. Veamos: cot 2x + csc 2x - cot 2x < 2 csc 2x < 2... (*) Sea: f(x) = csc 2x, determinamos su período: 2ti T=T=" Al elaborar la gráfica de la función f y encontra- mos que: 1 csc 2x = 2 => sen 2x = — 71 5ti De lo cual se deduce que: 2x = ~; — => 6 6 Para que se cumpla la condición (*), del gráfico se observa que: 99 .- Si la función / se encuentra definida, entonces se debe cumplir que el radican- do debe ser positivo, es decir: 2 2 |senx| - “ |x| >0 => |senx|> — |x| 2 Sean: g(x) = | sen x | y h(x) = — | x | funcio- nes cuyas gráficas son las que se mues- tran al lado. De este gráfico se observa que: RACSO DITOHIB R19 630 Problemas de Trigonometría y cómo resolverlos
g(x) > h(x) Solo se verifica en: „e y {ojf) 100 .- Por condición el problema: f(x) > 0 sen 3x + sen x - sen 2x > 0 2 sen 2x cos x - sen 2x > 0 transformando a producto Factorizando la expresión común: sen 2x(2 cos x - 1) > 0 Teniendo en cuenta la regla: ab >0 Analizamos en el intervalo dado: i) Si: sen 2x > 0 => ( 0; n) y: 2 cos x - 1 > 0 => cos x > — De estos resultados: x 6 ii) Si: sen 2x < 0 2X6 <7i;2ti) y 2 cos x - 1 < 0 1. 2 ó De estos resultados podemos deducir que: - (2) De (1) y (2) concluimos que: XG ¿0; u jó Ecuaciones e Inecuaciones Trigonométricas
101.- Procederemos como lo hicimos en el Prob. 10, esto es, graficamos las funciones f y g en el intervalo (0; 2ti), tal como se muestra en la figura adjunta. De este gráfico se observa que: f(x) > g(x) solo se verifica para el intervalo: ti 3n\ 2'T/ - fot} 102.- Dividimos entre 2 y se tiene: 1 V3 — sen 2x + cos 2x < 0 Reconocemos que el 1er miembro es: cos(2x^) <0 Usando la circunferencia trigonométricas (C.T.) reconocemos que para que se cum- pla la condición (*), se debe cumplir que: ti n 3n Por ángulos coterminales, se tiene: 71 71 371 — + 2krt < 2x - — < — + 2kn 2 6 2 VkeZ 2ti 5ti => — + 2kn <2x < — + 2kn ti 5ti — + kn < x < — + kn ... Solución General 3 6 Para obtener los intervalos de solución, daremos valores enteros apropiados para "k" haciendo una evaluación en el intervalo dado: [0; ti]. Si: k = 0 Si: k = 1 71 5ti 3 <x<~6 4ti 1171 => — <x< — *[0;ti] In 5n \3' 6 , Problemas de Trigonometría y cómo resolverlos A RACSO P BDITOIBI
103.- En primer lugar veremos la forma de transformar la inecuación para ponerla en términos del seno de una suma de arcos. Para ello recordamos la identidad del arco doble (degradación): 2senz(“x) = l-cos 2^-xj => 2sen2 = 1- sen 2x 2senZ^-xj = 1 - cos^y-2xj Reemplazando, tenemos: cos 2x - [1 - sen 2x] >0 => sen 2x + cos 2x > 1 1 1 1 1 Multiplicando por sen 2x + cos 2x > r Reconocemos que el 1er miembro es el desa- rrollo del arco compuesto: De la circunferencia trigonométricas, vemos que la condición (*) se verifica si: 3rr T Por ángulos coterminales se tiene: ti ji 3n — + 2kn<2x + — < — + 2kn; V fc e Z 4 4 4 ^ + /c7t]... Solución General 71 => 2kn<2x< — + 2kyi Para obtener los intervalos de solución, daremos valores enteros apropiados para "k" haciendo una evaluación en el intervalo dado: [0; 2tt]. Si: k = 0 => x e M] k = 1 => XG k=2 => xg XG [o; u[ít; o{271} Ecuaciones e Inecuaciones Trigonométricas
104.- Factorizando: ( 2 sen x + V3 )( 2 sen x - 1) < 0 Por el método de los puntos críticos: 2 sen x + V3 = 0 => sen x = - —— De la circunferencia trigonométricas (C.T.) la condición (*) se verificará si: (ti. ti\ /5ti. 4ti\ /5ti. 13tt\ 3' 6/ ° \~6' -3) \~3' T7 Nótese que sólo estamos planteando tres intervalos para resolver el problema en base a la condición inicial que indica x e (0; 2ti). En cualquier otro caso se recomienda plantear el conjunto solución. Para nuestro caso la respuesta final es: *e (0;í) 105.- Aplicando la identidad del arco doble tendremos: 2 2 sen x. cos x + 1 - 2 sen x + sen x - cos x < 1 => 2 sen x(cos x - sen x) - (cos x - sen x) < 0 => (cos x - sen x)( 2 sen x -1) < 0 ... (1) 7t 5rt En la circunferencia trigonométrica (A), analizamos para: ~ < x < — En este intervalo se cumple que: cos x < sen x => cos x - sen x < 0 ... (2) 1 De (2) en (1): por lo tanto se cumple que: 2 sen x - 1 > 0 => sen x > — - - - (3) R19 634 Problemas de Trigonometría y cómo resolverlos RACSO WlDITCIBl
De la circunferencia trigonométrica (B), analizamos la condición (3), la cual se cumple cuando: /ti. 5tt\ \6' 6/' pero de la condición del problema: x e /ti. 5n\ \4' 4/ Por lo tanto al intersectar los dos intervalos se tiene que: 71.571 4' 6 , 106.- Transformamos la inecuación aplicando la identidad del arco doble: Multiplicamos por 2: 2 sen2x + 2 eos2* <2 => v V 17 -v---z 1 - cos 2x + 1 + cos 4x < 2 Reducimos términos: cos 4x - cos 2x < 0 Transformamos a producto: Multiplicado por 1/2 : - 2 sen x sen 3x < 0 sen x . sen 3x < 0 Esta relación la analizamos así: sen x. sen 3x = 0 o i) De la Ira. se plantea: sen x = 0 v => x = 0; ti. . .(1) 71 2ti => x=0;—;ti.. .(2) En (*), como: 0 < x < ti => 0 < sen x < 1 iii) Solo falta analizar: sen 3x < 0 ... (3) Usando la circunferencia trigonométricas (C.T.) Para que se cumpla la condición (4), de la C.T. se observa que: sen x. sen 3x < 0 ...(*) sen 3x = 0 3x = 0; ti; 2ti; 3ti (seno) f y 1 Ecuaciones e Inecuaciones Trigonométricas
n<3x<2n => x g . (4) Finalmente de (1), (2) y (4), la respuesta es: XG 71.271 3' 3 u {0; se} 107.- Reduciendo al I cuadrante: cos\2+^/ =-sen ~2 Además por arco complementario: sen^-xj = cos x Reemplazando tenemos: -senl I - cos x > 0 Transformando por arco doble: -sen — - (l-2sen2-^) >0 9/ X í X Ordenando convenientemente: 2 sen — - sen — - 1 > 0 l2J l2J Factorizando: ^2sen-^ + 1) (sen^'1) > 0 - - - (1) Analizamos estos factores, según condición: x x G ( 2ti; 4ti> => — G ( ti; 2ti) x x a) En este intervalo, se verifica que: -1 < sen— <0 => -2 < sen— - 1 < - 1 Esto significa que este factor es negativo b) Según el análisis anterior deducimos que (1) se cumplirá si: R19 636 Problemas de Trigonometría y cómo resolverlos RACSO 1DITOUI
108.- Despejamos y aplicamos una factorización por diferencias de cuadrados: 4 4 eos x - sen x < 0 (eos2 x - sen2 x) (eos2 x + sen2 x) < 0 cos2x 1 eos 2x < 0...(*) En la circunferencia trigonométrica, he- mos ubicado los siguientes valores: Para que se verifique (*) se observa que: 371 2 <2x<- 71 2 7tt 2 5ti ~2 Solo estamos proponiendo dos intervalos con arcos negativos para resolver el problema en base a la condición inicial que indicas: x e [-2ti; ti]. En caso contrario se recomienda plantear el conjunto solución. Despejando tenemos: -3Z<x<-% ,4 4 se descarta por condición inicial 7tt T 5ti 4 / 7tc. 5ti\ xe V 4' 4/ 109.- Transformando la expresión por medio de la identidad del arco triple y del arco compuesto: (3cosx + cos3x\ Q /3senx+sen3x\ _ 5 ------4------) cos3x-^--------) sen3x < g , , 5 3(cos 3x-cos x-sen 3x-senx) + (cosz3x + sen 3x) < — cos(3x+x) = cos4x =1 3 1 => 3 cos 4x < — => cos 4x < ~ ....(*) En la circunferencia trigonométrica (C.T.) hemos ubicado arcos relacionados con el valor del coseno de (*): 71 5tt 1 cos 3 — cos 3 - 2 Ecuaciones e Inecuaciones Trigonométricas R19 637
Para que se cumpla la condición (*) de la C.T. reconocemos que: ti 5ti 7ti IIti Despejando: ^2 <x< ]2 ° ^2<X<^^ se descarta pues x e M) 110.- Empezamos planteando las restricciones del 1er miembro de la inecuación: i) sen x * 0 => x * mk, m e Z => x * 0; ti 71 71 ii) cosx*0=> r*(2n + l)-,ne Z => x*~ En forma analítica y recordando la identidad del arco triple y arco doble, tendremos: 3senX—4sen x (4cos x —3cosx) cos3x 3 sen x 3 2 sen x 4-^- . cos2x Ordenando: sen2 x + eos2 x 9 7 sen xcos x >24 2 2 1 > 8 sen x cos x 2(2 sen x cos x) < 1 9 2 . sen 2x < 1 3 1 - cos 4x < 1 cos 4x > 0 ... (1) En la circunferencia trigonométricas C.T. ob- servamos que la condición (1) se cumplirá si: 71 ti 3tt 5ti Notar que sólo estamos planteando dos in- tervalos para resolver el problema en base a la condición inicial que indica xe ^0; . R19 638 Problemas de Trigonometría y cómo resolverlos RACSO P*DITOKII
En caso contrario se recomienda plantear el conjunto solución. Despejando tenemos: n 8 71 8 3ti 5tt Finalmente por la condición x e M)- la respuesta es:x G 0; ?1 u fe %\ i o j L ° 111.- Sean/(x) = |sen'|j y g(x) = Ieos x I# funciones cuyas gráficas se muestran para x en <-7t; 2ti). Por lo tanto para que se cumpla: f(x)<g(x)....(l) debemos hallar primero los puntos de intersección entre dichas gráficas re- solviendo la siguiente ecuación trigonométrica: n = | COS X | |sen^|= |cos x| = COS2X 2. I I 2 => 2sen I 2 I = 2 cos x => 1 - cos x = 1 + cos 2x Transformando a producto, nos queda: => cos 2x + cos x = 0 3x x 2 eos— cos ~ = 0 Igualamos a cero cada factor, para evaluar «x» en el intervalo dado: 3x 3x 3ti ti 3ti 5tt 7ti i) cos~=o => -^=--^'-2'T;T'T ti ti 5ti 7it x = -ti;- 3 'n'~3 ;~3 x x 3ti ti n 3n u) eos- =0 => 2='T;'2;2;T Los puntos de intersección que nos interesan son: x = -3ti ; -Tt; Tt; 3tt 71 71 571 x = -7t;- De las gráficas se observa que la condición (*) se verifica si: Ecuaciones e Inecuaciones Trigonométricas R19 639
112.- Despejando se obtiene: sen(27tx) > cos(2tix) Sean: f(x) = sen(27tx) y g(x) = cos(2kx), fun- dones cuyas gráficas se muestran para: x g ^0; ’ es decir x g ( 0; 0,78) 2n Notar que el período: Tf = Tg = — = 1 Hallemos los puntos de intersecdón entre dichas gráficas resolviendo la ecuación trigonométrica: 5n ~4 sen(2?tx) = cos(2tix) - > tan(27tx) = 1 1 5 8' 8 De las gráficas se observa que: sen(2itx) > cos(2tix), es decir:/(x) > g(x), si- xG (l;i) 113.- Nuestra estrategia será recurrir a la siguiente reladón: - 7a2+B2 < A sen x + B cos x < 7a2 +B2 , AyBeR;VxeR Evaluando para A = 1, B= 1, tendremos: -72 < sen x + cos x < -Jl 1/2 - -J3 <senx + cosx- 75 <75 - -J3 Asi reconocemos que este factor es negativo, y recordando la regla: ab<0 <=> n < 0 y fe > 0 ó a > 0 y b < 0 Planteamos que en la ecuadón dada se debe cumplir que: 2 1 cos x - — > 0 3 cos2x~l >0 cos 2x > 0 .. .(*) arco doble Usando la circunferencia trigonométrica C.T.) observamos que para condidón (*) se cumplirá si: ti rt 3ti 5n 7ti 9ti - — <2x< ~ u — <2x< — u — <2x< — Problemas de Trigonometría y cómo resolverlos RACSO ^paDiTOk»
Notar que sólo estamos planteando tres in- tervalos para resolver el problema en base a la condición inicial que indica x e { 0; 2tt). En caso contrario se recomienda plantear el con- junto solución. Despejando se tiene: * Finalmente por la condición x g ( 0; 2tt) la respuesta es: 114.- Resolveremos como se hizo en el problema anterior. Esto es: - 7a2+B2 < A sen x + B cos x < 7a2 +B2 , AyBGlR;VxGlR Evaluando para: A = 1, B = - 1, se tendrá: '2 < sen (3x) - cos (3x) < -J2 '2 +2< sen 3x-cos 3x+2 < -J2 +2 >o Con esto hemos demostrado que el numerador de la ecuación es positivo. Luego recordando cuando existe la regla: b * 0, a b Ahora analizamos el denominador: '2 sen x - 1 * 0 J2 2 7t 7 Además según la regla, se debe cumplir que: 2 sen x - 1 > 0 J2 2 (seno) 371 4 De la circunferencia trigonométricas (C.T.), observamos que la condición (*) se cumplirá si: Ecuaciones e Inecuaciones Trigonométricas R19 641
115.- Por definición se sabe que: -1 < sen(3x) < 1, V x g R Restando V5 a cada miembro: -1 - V5 <sen 3x—J$ < < 1 - -J5 <o Ahora que hemos comprobado que el numerador del 1er miembro de la inecuación es negativo, aplicaremos la misma regla del problema anterior, es decir: cos 2x + 3 cos x + 2 < 0 7 2 cos x -1 + 3 cos x + 2 < 0 2 2 cos x + 3 cos x + 1 < 0 (2 cos x + l)(cos x + 1) < 0 Por el método de los puntos críticos: 1 2 cos x + 1 = 0 -> cos x = - — cos x + 1 = 0 -> cos x = - 1 Elegimos los intervalos donde el cociente se verifique como positivo: 1 -1 < cos x < - — ...(*) De la circunferencia trigonométricas (C.T.), observamos que la condición (*) se cum- plirá si: /2tt. 4tt\ . . xG \ 3 ' 3 / 1 116.- Empezaremos planteando la restricción en el intervalo dado: i) 1 - cos x * 0 - > cos 2x z 1 2x # 2áti x*kn x * 0; 7t; 2n Recordando la identidad del arco doble: 2 1 - cos 2x = 2 sen x Reemplazando en la ecuación dada: senx 2 < 0 2sen x 2 De donde: sen x > 0 a sen x < 0 Y analizando en la circunferencia trigonométrica (C.T), se observa que: Tt < x < 2Tt x G (tt ; 2it> Problemas de Trigonometría y cómo resolverlos >KACSO IDlTOkll
117.- Empezaremos planteando la restricción: Tt Tt tanx 3, six * (2n + l)y ,«£ Z x+ — Recordando la regla: \a\ > b <=> a > b ó a <-b tan x > 1 U tan x < - 1 ...(*) Usando la circunferencia trigonométrica (C.T.) Es necesario recordar: Tt 5rr 3ti 7n tan , - tan~ = 1 a tan" = tan" = -1 4 4 4 4 Para que se cumpla la condición (*) de la C.T.. se observa que: XG (Tt. 3tt\ (ttI 4' 4 / ’ Í2J 118.- Como en el problema anterior, empezaremos planteando la restricción: Tt Tt tanx 3, six* (2n + 1)~ , n g Z => x* — Factorizando el 1er miembro se tiene: (tan x - 75 ) (tan x - 1) < 0 + 1 -J3 -00 ® ! O ! ® +a> I Aplicando el método de los puntos críticos, igualamos a cero cada factor para determi- nar dichos puntos. Luego graficamos en la recta numérica y elegimos los intervalos don- de el producto es negativo, entonces: tan x - 75 = 0 tanx= 75 tan x - 1 = 0 - > tan x = 1 Se observa que: 1 < tan x < 75... (*) De la circunferencia trigonométricas (C.T.), ob- servamos que la condición (*) se cumplirá si: /Tt, 7t\ xe (4' 3/ Ecuaciones e Inecuaciones Trigonométricas
119.- Planteando la restricción: 7t 71 371 tanx3,six*(2n + 1)~ ,he Z => Factorizando tenemos: (tan x - l)2. (tan x - 75 )(tan x + 75 ) < 0 2 Teniendo en cuenta que: (tan x - 1) > 0 tan x * 1 7t 5n Por lo tanto: (tan x - 75 )(tan x + 75 )< 0... (1) Por el método de los puntos críticos, se tendrá que: tanx - 75 =0 tanx— 75 tan x + 75 = 0 —> tan x = - 75 -73 Elegimos los intervalos donde el producto sea negativo: - 75 < tan x < 75 ... (2) De la circunferencia trigonométricas (C.T.), observamos que la condición (2) se cumplirá si: 120.- En primer lugar planteamos la restricción: i) tan 3x 3, si. 3x * (2n + 1) y ->xz(2m + 1)7 ,me Z -> x* 7;^ o o z 71 71 ii) tan x 3, si: x * (2n + 1) ~ , n G Z, — Transformamos el 1er miembro aplicando la siguiente identidad del arco triple: tan3x 2cos2x+l tanx 2cos2x-l Reemplazando en la inecuación: 2cos2x + l 2cos2x-l + 1 >0 4cos2x 2cos2x-l R19 644 Problemas de Trigonometría y cómo resolverlos RACSO WIDITOlll
1 La restricción es: 2 eos 2x - 1 + 0 => cos 2x + ~ 7t 571 3 ' T Tt 571 T Aplicando el método de los puntos críticos cos 2x = 0 1 2 cos 2x - 1 = 0 - > cos 2x = ~ 1 0 2 -O> ® ! G) ! ® +“> I I Elegimos los intervalos donde el cociente es negativo: 1 0 < cos 2x < — ...(*) Usando la circunferencia trigonométrica ob- servamos que la condición (*) se cumplirá si: 71 3 7t 371 2 U ~2 5tt 3 Notar que sólo estamos planteando dos in- tervalos para resolver el problema en base a la condición inicial que indica x G ^0; En caso contrario se recomienda plantear el conjunto solución. Despejando tenemos: X G /n. ti\ \6' 4/ se descarta por condición inicial 121.- Empezamos estableciendo las restricciones: i) tan — 3, si. — * (2m + 1) — => x * (2m +1 )rt, m e Z ii) cot x 3, si: x * rm, n g Z => x * 0; ti Recordando la identidad del arco mitad: x tan— = csc x - cot x csc x - cot x > 1 - cot x => csc x > 1 .. .(*) Graficando la función csc x en (0; 7t), tenemos: Ecuaciones e Inecuaciones Trigonométricas
Para que se cumpla la condición (1) del gráfico se observa que: x G < 0; Tt) - {y} 122.- Establecemos las restricciones del problema: (_ \ Tt 71 71 71 - x I 3. si. ~ - x * nm x* ~, m&'Z. * * "7 i V 4 / 4 4 4 4 71 71 ti 3tt 5it ii) tan 2x 3, si: 2x * (2n + 1) — => x + (2n + 1)~, n G Z Recordando el teorema del complemento : cot^-xj = tan^+x sen ^ + x-2x Reemplazando tenemos: taní^+x)- tan (2x) >0 => ---;----------- > 0 U 1 cos(^ + x]-cos(2x) — < X — — < X < — 4 4 4 4 Finalmente por la condición x G (0; 7t), la respuesta es: XG (<* ?) u R19 646 Problemas de Trigonometría y cómo resolverlos RACSO Pbditobis
123.- Determinamos las restricciones del problema: 71 71 371 i) tanx3, si: x * (2m + 1)~, m e Z => x* —; ~ ii) cot x 3, si: x * «71, n g Z x*0;7t 2tt En forma analítica y recordando la identidad del arco simple y del arco doble: tan x + cot x = sec x . csc x = 2 csc 2x, 2 csc 2x < 1 1 csc 2x < —... (*) Sea:/(x) = csc 2x, determinemos su período: 2n T = — = ti (por regla práctica) Graficamos la función f, y de ella Para que cumpla la condición (*) del gráfi- co se observa que: 124.- Determinamos la restricción del problema: 71 7t 371 i) secx3, si: x + (2n + l)y, n g Z xt — ~ Factorizando tenemos: ( 75 sec x - 2)(sec x - 75 ) < 0 Por el método de los puntos críticos: 75 sec x - 2 = 0 - > sec x = 275 3 2^3 3 &. secx- 75 =()->secx= 75 Elegimos los intervalos donde el producto sea negativo. 275 ---- < sec x < 72 ... (1) 3 Sea/(x) = sec x , y graficamos la función. Ecuaciones e Inecuaciones Trigonométricas
Debemos recordar los siguientes valores: 7t 771,— sec — = sec— = -J2 4 4 n 117t 2^3 sec ~ = sec ~~ =------------ 6 6 3 Para que se cumpla la condición (1) del gráfico se debe cumplir que: 125.- Planteamos las restricciones: 7t i) secx 3, si: x + (2m + 1)~, m g Z 71 2 ii) csc x 3, si: x * im, n g Z Transformando mediante las identidades del arco simple y arco doble, tendremos: 2 2 sec x - csc x = 1 1 2 " 2 cos x sen x —(eos2 x-sen2*) 2 2 sen x.cos x 2 2 sec x - csc x = o (2senx.cosx) -4cos2x 2 sen 2x -8cos2x 2sen 2x -8cos2x -8cos2x l-cos4x vers4x Reemplazando en la ecuación : -8cos2x vers4x 4 vers4x 8cos2x -4 vers4x ~ vers4x kit De las restricciones: x * — 2cos2x +1 vers4x 4x < 2krt —> cos 4x # cos(2Ati) —> cos 4x * 1 2>l-cos 4x >0 ...(1) -1 < cos 4x < 1 - - (2) Esto significa que el denominador de (1) es positivo, luego: 2 cos 2x + 1 < 0 Elaboramos la circunferencia trigonométrica. 2 2 R19 648 Problemas de Trigonometría y cómo resolverlos 'X RACSO W dITOI I
Es importante recordar los siguientes valores: 126.- Planteando la restricción: 71 7t 37t tan x 3, si: x # (2m + l)y, m G Z — 2 2 „ . , . senx sen x + cos x Sustituyendo la, tendremos: sen x . ---- + cos x < 2 ------------- < 2, cosx cosx 2 2 Teniendo en cuenta que: sen x + cos x = 1, y además la condición de que: xg 2ti } ] => cosx>0 1 Despejando: cosx> ~ ... (*) Elaboramos la circunferencia trigonométrica (C.’ Es importante recordar los siguientes valores: ti 5n 1 cos-^cosy = - Para que la condición (*) se verifique, según C.T, se debe cumplir que: xe [f ;2n] Ecuaciones e Inecuaciones Trigonométricas
127.- Establecemos la restricción del problema: 7t i) tan x B, si: x * (2n + 1) —, n e Z 7t 3tt 2' ~2 Transformamos la expresión aplicando la identidad del arco simple y del arco doble (degradación). 1 2 COS X 1 < 1 + 2 cos 2x + cos^x 2 2 => sec x < 4 cos x 2 < 4 cos x => 1 < (2 cos2x)2 => 1 < (1 + cos 2x)2 cos 2x(cos 2x + 2) > 0 .. .(*) n Pero de la restricción: x * (2n + 1) — => 2x + (2n + l)xt => cos 2x * cos[(2n + l)n] cos 2x * 1 Además se sabe que. - 1 < eos 2x < 1 => 1 < eos 2x + % - 3 En (1) tenemos: cos 2x > 0 Usando la circunferencia trigonométrica observa que: 7t 2 7t 2 Pero también por ángulos coterminales: 7t 7t - ~ + 2fai < 2x < — + 2fai Del conjunto solución que se ha determinado, damos valores apropiados para k. Si: k = 0 k=l '3ti . 57t' ' 4 ¡ '7n. 9rt t 4 ' Pero por la condición inicial del problema: x e( 0; 2ir) xe (0;? 3n. 5n V fce Z 4 ' k = 2 4 R19 650 Problemas de Trigonometría y cómo resolverlos ák RACSO jPbditokbb
128.- Nuestra estrategia será recurrir a la siguiente relación: - Ja2 + B2 < A sen x + B cos x < Ja2 + B2 , AyBeR;VrcR Evaluando para A = 1, B= 2, tendremos: - V5 < - sen x + 2 cosx< -J5 => -Jé- -J5 < >/6-sen x + 2 cos x < -Jf> + -J5 >o a De la regla: ~ <0«-»(a>0yb<0)ó(a<0yb>0), con: b * 0 Pues bien, habiendo probado que el denominador de la fracción es positivo, nos queda analizar el numerador, el cual deberá ser así: 2 a 2 r / 7 _* 1 tan x- ~ <0 => tan x < ~ => vtan x < => I tanx I < —f= ... (1) 3 3 vá ' ' _/3 De la regla: | a | <b <=$-b <a <b Aplicándola en (1), tendremos: J3 J3 - — < tan x <------ ... (2) 3 3 Tt 3tt Restricciones: x * —; — para que tan x 3. Elaboramos la circunferencia trigonométrica (C.T.), para lo cual es im- portante recordar los siguientes valores: Tt 7n 73 tan— = tan-— =-------- 6 6 3 5n Un J3 tan~ = tan-— = -- 6 63 Para que se cumpla la condición (2), de la C.T. se observa que: 129.- Planteando la restricción: Tt Tt 3tt secx3,si: x* (2n + 1) —, ti g Z :. Despejamos: sec x < - cos x ...(*) Ecuaciones e Inecuaciones Trigonométricas R19 651
Sean: f(x) = sec x y g(x) = - cos x, funciones cuyas gráficas se muestran en [0; 2ti] Para que se cumpla la condición (*), del gráfico se observa que: XG /n. 3n\ \2' 2 / R19 652 Problemas de Trigonometría y cómo resolverlos ►sí RACSO eDITOIIl
M) M a© Resolución de Triángulos A LEY DE SENOS ,, a senA 01.- Tenemos: M = t - ,, b sen o Sabemos por la ley de senos: = „ , , a senA _ De donde: t = M = () b sen o 02.- Usando la ley de senos tenemos: 2________3 sen3x _ 3 senx - sen3x =* senx “ 2 ‘ ' _ sen 3x _ _ „. Pero: senx7 = 2 eos2x + 1... (1) 3 1 1 (l)en(*): 2cos2x+l= => cos2x = => 2x = árceos^ c. - 1 1 Finalmente: x = 2 'arc cos 4 03.- Por la ley de senos: 8 4 sen 66 _ sen66 - sen26 sen26 - Pero: sen2Ó =2cüs46 + 1 luego: cos 46= Como: cos 60° = De donde: 46 = 60° .-. 0 = 15° Resolución de Triángulos Oblicuángulos
04.- Usando la ley se senos: = sen(180°-26) Pero: sen (180 ° - 26) = sen 26 = 2 sen 6 . cos 6 F 3 4 n' ' sen6 2sene.cos6 Eliminando: sen 6*6 tenemos: cos 6 = Luego como: 6 /l-cos6 tan 2 - ^1 + cos0 Finalmente racionalizando obtenemos: sen(A + B) 05.- Recordemos: cot A + cot B = (1) Por la ley de senos: a = 2R sen A , b = 2R sen B , c = 2R sen C => abe = 8R3 sen A senB senC . .. (2) (1) y (2) en M: M = 8R3 sen A sen B sen C . ~ 8R sen C . sen (A + B) Como: A + B + C = 180° => sen(A + B) = sen C => M = 8R3sen3C = (2Rsen C)3 Por la ley de senos: M = c 06.- Tenemos: M = Usando la ley de senos: M = Efectuando, tendremos: M = Pero: sen2B - sen2C = sen(B + C). sen (B - C) sen(B + C)sen(B-C) sen(B + C) => M =--------------------- =---------- 2senA.sen(B—C) 2senA b sen B - esen C 2a.sen(B-C) (2RsenB)senB- (2RsenC)senC 2(2RsenA).sen(B-C) 2R(sen2B - sen2C) Problemas de Trigonometría y cómo resolverlos jA RACSO Wbditoibi
Como: A + B + C = 180° => sen (B + C) = sen A En(’): 07.- Por uno de los datos: b + c = a-j2 ---(*) Usando la ley de senos en (*): 2Rsen B + 2R sen C = (2R . sen A). -J2 Luego: sen B + sen C = (sen A). V2 Transformando a producto: 2 sen Pero: B - C = 90° => cos B-C 2 •J2 = cos 45 = y- ... a(l) A + B + C = 180° 2 B + C -y— =90° B + C 2 = cosy ... (2) (l)y(2)en(w): A 2. cos — . J2 2 „ A A 2sen—.eos— 2 2 2 Simplificando: A sen — 2 2 A — =30' 2 Finalmente: A = 60° 08.- Por la ley de senos, sabemos: a = 2Rsen A, b = 2R sen B, c = 2R sen C Reemplazando en M: M = R.2senB.cos B + R.2senC.cosC cos (B-C) Pero: 2 sen x . cos x = sen 2x, luego: M = R(sen2B + sen2C) cos (B-C) Transportando a producto: M = 2Rsen(B + C).cos (B - C) cos (B-C) Como: A + B + C = ir => sen (B + C) = sen A M = 2R sen A Por ley de senos, tendremos: M = « Resolución de Triángulos Oblicuángulos R20 655
09.- Sabemos por la ley de senos: sen A = senB 20 30 Usando los datos: sen37° = <^0 Despejando: sen B = ^ . sen 37° 3 3 3 9 Pero: sen 37° = => sen ®= 2^ 5 = jq - - (1) 2 También sabemos: cos 2B = 1 - 2 sen B ... (2) (81 \ 81 íóó) cos2B = '5(j B í> = 30 10.- Nos piden: cos 26 Usando la ley de senos: 5 7 sen36 7 sen6 - sen36 sen6 " 5 ’ ’ 7 Pero: "3 = 2 cos 26 + 1 7 En (*): 2 cos 20 + 1 = 3 2 cos 26 = | Finalmente: 1 cos 26 = 3 11.- Tenemos: a _ b cosA - cosB = Cr ...(*) cosC ' ' Por la ley de senos sabemos: «= 2Rsen A b = 2RsenB c = 2RsenC (1) 2RsenA _ 2RsenB _ 2RsenC (1) en ( ). cosa — cosB — cosC Simplificando tenemos: tan A = tan B = tan C de donde: A = B = C = 60° Finalmente: El triángulo es equilátero 656 Problemas de Trigonometría y cómo resolverlos •jáP.ACSO WbDITOIEI
12.- Por dato: cosA cosB a + b cosC c Usando la ley de senos en (*): cosA cosB cosC 2RsenC 2RsenA + 2RsenB + 2RsenC — (2RsenA)(2RsenB) Simplificando y usando: cotx = cosx senx Nos queda: cotA + cotB + cotC= (1) Pero: A + B + C = 180° => sen C = sen(A + B) y --z——~ = cot A + cot B ... (2) ' '} senA.senB ' ' (2)en(l): cot A + cot B + cot C = cot A + cot B Simplificando: cot C = 0 de donde: C = 90° 13.- Tenemos: _ senA+senB c—a ~ senB+senC + b+c ''' ' ' Por la ley de senos tenemos: a = 2R sen A, b = 2R sen B , c = 2R sen C Reemplazando en (*): _ senA+senB 2RsenC- 2RsenA " senB+senC + 2RsenB+2RsenC Ordenando y simplificando obtenemos: _ senB+senC ~ senB+senC M = 1 14.- M = a sec A + b. sec B + c. sec C Usando la ley de senos: M = 2Rsen A . + 2Rsen B . cc^sb + 2R sen C . co^-. spn x Pero: = tan x => M = 2R(tan A + tan B + tan C) .. . (*) COSX Como: A + B + C = 180° => tanA + tan B + tan C = tan A . tan B . tan C En (*): M = 2R . tan A . tan B . tan C Pero por dato: R . tan A . tan B . tan C = 2 Finalmente: M = 4 Resolución de Triángulos Oblicuángulos
15.- Ley de senos: B AABD: f=S86-W ABDC: Igualando las expresiones (1) y (2): = A/86 39 A_________ A D ° C sen 6 sen 26 Pero: sen 26 = 2 sen 6 . cos 6 sen 30 sen6 , „ Q „„ => ----¿t. = 75--ñ---~ => 2 sen 36 . cos 6 = sen 86 sen86 2sen6.cos6 Transformando a una suma: sen 46 + sen 26 = sen 86 => sen 26 = sen 86 - sen 46 Transformando a producto: sen 26 = 2 sen 26 . cos 66 Al simplificar obtenemos: cos 66 = => 66 = 66° Finalmente: 6 = 16° 16.- Recordemos: (*) a.senC =c.sen A ¿sen A = a.sen B c.senB =¿.senC M = a. sen C - b. sen C + b. sen A - c. sen A + sen B - a. sen B Agrupando: M = («. sen C - c. sen A) + (fc. sen A - a. sen B) + (c. sen B - b . sen C) Por (*): M = 6 LEY DE COSENOS 17 .- Usando la ley de cosenos: x2 = (3 -75 )2 + (2-75 )2 - 2(2-75 )(3 -J5 ) cos 66° 2 Al efectuar obtenemos: x = 35 Finalmente: x= -735 Problemas de Trigonometría y cómo resolverlos RACSO BDITO1BI
18 .- Sea: BC = x, B Por la ley de cosenos, tenemos: cos 30/ x2 = cos236 + cos26 - 2 cos 6 . cos 36 . cos 46 --------------— A cos 6 C x2 = cos236 - cos 6 . cos 36 . cos 46 + cos26 - cos 6 . cos 36 . cos 46 Factorizando apropiadamente, tenemos: x2 = cos 26( cos 36 - cos 6 . cos 46) + cos 6 (cos 6 - cos 36 . cos 46)... (*) Pero: cos 36 = cos (46 - 6) = cos 46.cos 6 + sen 46.sen 6 => cos 36 - cos 6 . cos 46 = sen 46.sen 6 ... (1) cos 6 = cos( 46 - 36) = cos 46.cos 36 + sen 46.sen 36 => cos 6 - cos 36-cos 46 = sen 46.sen 36 ... (2) (1) y (2) en (*): x2 = cos 36(sen 46 . sen 6) + cos 6(sen 46.sen 36) Factorizando convenientemente, tendremos: x2 = sen 46 (eos36.sen6 +eos6.sen36) => x2 = sen246 sen (30+0) Finalmente: x = sen 46 19 .- Como: 3a = 7c => c = - 0) h R 3b - 3c => ...(2) De (1) y (2): a = 7k, b = 8k y c = 3k . . , , b2+c2+«2 Por la ley de cosenos sabemos: cos A =-----’ „ . . , . . . . 64*2+9*2-49*2 24A:2 Reemplazando los valores de a, b y c: cos A =--------ñ--- = , 2. F 2 2(24* ) 2(24* ) Finalmente: cos A = -~ de donde: A = 66° Resolución de Triángulos Oblicuángulos
20.- Tenemos: Por la ley de cosenos sabemos: a2-b2-c2 = |fec. ..(*) a2 -b2 - c2 = 2bc. cos A... (1) (1) en (*): Simplificando obtenemos: -2bc. cos A = ^bc _ 1 cos A = - 2 Luego como: tany 11-cosA V 1 + cosA De donde: tany = finalmente: tany = y/2 21.- Tenemos: 3(a2 - b2 - c2) = 2bc ...(*) Por la ley de cosenos sabemos: n2 - b2 - c2 = - 2fec.cos A - • - (1) (l)en(*): Luego: Por la expresión (2): Finalmente: -6bc. cos A = 2bc => cos A = - ^ .. .(2) 2 A COS y 2 A COS y 1 + cosA 2 2A____ cos 2 - 3 2 1 22.- Como: , , w , . 7ab (a + b + c)(a + b-c) = -y 2 2 7 ab 2 7 Por diferencia de cuadrados: (a + fe) - c = —y => a + b - c = .ab - 2ab Al efectuar obtenemos: c2 + b2 - c2 = |. ab . (1) Por la ley de cosenos, sabemos: fl2 + b2 - c2 = 2a.fe.cos C .. (2) De (1) y (2), deducimos: 2«fe.cos C = ^ ab Finalmente: ... 1 cosC= g Problemas de Trigonometría y cómo resolverlos RACSO WbDITOIES
23.- Tenemos: M = ab. cos C - ac. cos B Por la ley de cosenos tenemos: c2 = a2 + b2 - 2ab. cos C => 2ab. cos C = a2 + b2 - c2... (1) b2 = a2 + c2 - 2ac. cos B => 2ac. cos B = a2 + c2 - b2 — (2) Al restar la expresiones (1) y (2), obtenemos: 2(ab cos C - ac. cos B) = 2b2 - 2c2 2 2 Finalmente: M = b -c 24.- De la figura: m Z B es el mayor Por la ley de cosenos: 2 . „2 1,2 4" c b COS D - -------------- 2a.c 49k2 + 64k2 - 169k2 _ 2 cos B - 2(7k)(8k) ~~2 Finalmente: B = 120° 25.- Utilizando la ley de cosenos tenemos: (2x + 3)2 = (2x - l)2 + (2x + l)2 - 2(2x -l).(lx - 1). cos 120°... (*) Pero: (2x + 3)2 = 4x2 + 12x + 9 .. (1) (2x - l)2 + (2x + l)2 = 2(4x2 + 1) ...(2) (2x - l)(2x + 1) = 4x2 - 1 ... (3) (1), (2) y (3) en (*): 4X2 + 12x + 9 = 8x2 + 2 - 2(4x2 -1) Al efectuar, obtenemos: 8x - 12x -8 = 0 Finalmente: 4(x - 2)(2x + 1) = 0 Entonces: 2x + 1 = 0 v x - 2 = 0 1 o > x="2 v x=2 (imposible) .'. X = 2 Resolución de Triángulos Oblicuángulos
26.- Como: a + b = -J2 . c Elevando al cuadrado: (a + b)2 = (-j2 c)2 => a2 + b2 + 2ab = 2c2 ... (1) 2 2 2 Por la ley de cosenos: a + b - 2ab. cos C = c ... (2) Al restar las expresiones (1) y (2), obtenemos: 2ab + 2ab . cos C = c2 => 2ab(l + cos C) = c2 Finalmente: 1+cosC=éb 27.- Usando la ley de cosenos, tenemos: 49 = 64 + c2 - 2(8)í. cos 60° => C2 - 8c + 15 = 0 Factorizando: (c - 3)(c - 5) = 0 => c = 3 (No cumple con los datos) v c = 5 (si cumple) 2 2 2 t i i . r d +c* Luego por la ley de cosenos: cos C -------------- 82+72—52 2(8)(7) Efectuando las operaciones indicadas, obtenemos: cos C = Construyendo un triángulo rectángulo para el ángulo C. • Luego: 11 C°tC=^/3 5^3 Finalmente: 11 '3 cot C = — 28.- Como: (a2 - b2 - c2)2 = a4 + b4 + c4- 2a b2 - 2a c2 + 2b2c2 ...(*) Por dato: a4 + b4 + c4 = 2a2b2 + 2a2c2 __(1) (1) en (*): (o2 - b2 - c2)2 = 2a2b2 + 2a22 - 2ab2 - 2a2¿ + 2b2c2 (a2-b2-S)2 = 2b2c2 a2-b2-c^=^bc (2) Pero por la ley de cosenos: a2 - b2 - c2 = - 2bc. cos A ______(3) RACSO V^CDITOKIB* R20 662 Problemas de Trigonometría y cómo resolverlos
(3) en (2): -2bc. cos A = -J2 bc, luego: cos A = - J2 2 Finalmente: A = 135° 29.- Sean: x - 1, x , x + 1 los lados de un triángulo Por la ley de senos: x + 1 = X~1 sen26 senG Pero: sen 2G = 2 sen G . cos G x+1 _ 2sen0.cos0 x-1 " senG JC 4- 1 => 2 cos G = ----r - - - (1) x — 1 Por la ley de cosenos: (x - l)2 = (x + l)2 + x2 - 2x(x + l)cos G ... (2) Reemplazando la expresión (1) en la expresión (2), obtenemos: (x-l)2=(x+l)2 + x2-^Í Resolviendo: Luego: Finalmente: x = 5 perímetro = 3x = 15 perímetro = 15 30.- Sabemos que: Luego en el problema: 2 eos2 a - 1 + cos 2a a ^2cos2^j +fe^2cos2iyj =2-^3 1 + cosB 1 + cosA a + a cos B + b + b cos A = 2 -J3 Ordenando, como sigue: a + b flcosB + FcosA = 2 c (Por ley de proyecciones) Finalmente: 2p = a + b + c 2p = 2-j3 Resolución de Triángulos Oblicuángulos
LEY DE TANGENTES 31.- Por la ley de tangentes sabemos: a-b _ a + b ~ tan tanl --(I) Como: tan tany-tan® 1+tany. tan® Reemplazando los datos: tan ¿4 3 *4“’ tan tany + tan® 1-tany. tan® Reemplazando los datos: tan 4 >4 _ 5 “ 3 Luego en (1): 3 a-b = _5 a+b 5 3 Finalmente: M=é 32.- Por la ley de tangentes sabemos: Usando proporciones tendremos: Reemplazando los datos: =---- tanl 8 Como: A + B+C = 180' Por co-razones: tan C = cot y Finalmente: J r- coty =5 = 5 , tan a-b _ ____ a + b ~ . tan . . tan a-b+a+b a + b tan A+B 2 + y =90° ] 6641 Problemas de Trigonometría y cómo resolverlos • RACSO WlDITOlXI
33.- Por la ley de tangentes sabemos: Usando proporciones: (b-c)-(b+c) b+c Como: A + B + C = 180° => y + = 90° Por co-razones trigonométricas: (B+C\ A tanl—) - cot 2 En(*): b+c COty ., -2c M = t— b+c Reemplazando en M, obtenemos: fe = 5c M = -2c _ -2c 5c+c ~ 6c Finalmente: 34.- Por la ley de tangentes sabemos: Como: b cot B = (2c - fe)cot A => fe(cotB + cot A) = 2c. cot A ... (1) Pero: cot B + cot A = sen(A+B) = senC senA.senB senA.senB - - (1) (1) en (I): fe.senC _ 2(2RsenC)cosA senA.senB ~ sen A Simplificando: cos A = => A = 60° => B + C = 120° Luego en (*): . (B+C\ M = tan I 2 I tan 60° = t¡3 Resolución de Triángulos Oblicuángulos
35.- Por la ley de tangentes: Por dato: A + B+ C = 180° a C = 60" => A + B = 120° ... (1) También: (1) y (2) en (*): a = 3b ... (2) 3b-b 3b+b tan 60° Pero: tan 60° = -J3 => tan^^) ~ Luego por arcos dobles: tan(A - B) =--. • l-tan2í^| Por (3): Finalmente: tan(A - B) = tan (A - B) = 4-J3 tan (A - B) = “p- LEY DE PROYECCIONES 36 .- M = . sec C... (*) Por la ley se proyecciones sabemos: a = b cos C + c. cos B => a - c. cos B = b. cos C... (1) (1) en (*): M = (bc°sC). sec C Al simplificar, obtenemos: M = 1 37 .- Por la ley de proyecciones sabemos: a = b. cos C + c. cos B => a - c. cos B = b cos C c = a. cos B + b. cos A => c - a. cos B = b cos A R20 666 Problemas de Trigonometría y cómo resolverlos RACSO
Reemplazando en M: Luego de simplificar obtenemos: fr.cosC.secC cos A M=7¿A M = sec A M = 38 .- Como: A + B + C = 180° cos(A + B) = -cos C cos (A + C) = -cos B Reemplazando en la expresión M: Por la ley de proyecciones: -c.cosB-b.cosC M=---------------- a a = b cos C + c. cos B -(c.cosB + fe.cosC) Finalmente: M = fe.cosC+c.cosB " M = -1 39 .- M = be. cos A + ac. cos B + ab. cos C Agrupando convenientemente: M = c(b cos A+ a cos B) + ab cos C ... (1) ó M = fec.cos A + a (c.cos B + b cos C)... (2) (1) + (2): 2M = a(c . cos B + b. cos C) + c(b cos A + a cos B)+ b(c cos A + a cos C) Pero por la ley de proyecciones: a = b cos C + c. cos B b = a cos C + c. cos A c = a cos B + b . cos A 2 2 2 Luego: 2M = a(a) + c(c) + b(b) => M = a +C = .'. M = 5 40 .- A, B y C ángulos agudos: 2 Recordemos: 1 + cos 2x = 2 cos x Luego en la expresión "M": M = be. Vi + cos 2 A + ac. Vl + cos2B + ab. Vi+cos2C Como: A, B y C son agudos entonces: M = J2 (fec.cos A+ ac. cos B + ab.cos C) Por el problema anterior usando la ley de proyecciones: 2 2 2 fec.cos A + flc.cos B + nb.cos C = a + b + c Finalmente: M = -J2 (az+ b2 + c2) Resolución de Triángulos Oblicuángulos RÍO 667
ÁREA DE UNA REGIÓN TRIANGULAR ABC 41.- Del dato: = 2 = = t Pero: A + B + C = n => f + 2f + 3f = 180°, luego: A = 30°, B = 60°,C = 90° Se tiene un ÍX ACB (recto en C) Del dato: 2n - n = 2k° => n = 2k (CA)(CB) _ -’aabc 2 2 2 (2k)2^3 2 bAABC “ 2 Finalmente : SAABC = 2-73 .fc2 42.- Del dato: senA 5 senB _ senC 7 8 De la ley de senos: st-nB = senC a____L _ ______r. 51 ~ 7t ~ 81 ~k Luego: a = 5k => a + b + c = 2p=> p-a = 5k b = 7k 20k = 2p p - b = 3k c=8k p = 10k p - c = 2k Se sabe que: S = -Jp.(p-fl)(p-fe)(p-c) => 90-J3 = -J10k.5k3k.2k => 90 Je2 = 9 => k-3 > a =15 cm => b = 21 cm > c = 24 cm Se sabe que: S = ^ac. sen B 2S 2(90-^) J3 senB = — = =X^ => B = 60° ó B = 120° ac 15.24 2 = 10-73. k2 ^ARACSO W^EDITCKBS R20 668 Problemas de Trigonometría y cómo resolverlos
2 2 2 43.- Recordar: a = b + c - 2bc cos A.... ley de cosenos “a mayor ángulo se opone el mayor lado" S = bc. sen A. área del AABC sen 120° = sen 60° .. . arco suplementario. sean 2k - 1,2k +1,2k + 3, las longitudes de los lados del AABC. Por ley de cosenos en AABC, tenemos: (2k +3)2 = (2k +1)2 + (2k -l)2 - 2(2Jt 11 )(2A - 1) cos 120° 4k2 + 12k + 9 = 4A2 + 4k + 1 + 4k2 - 4¡t + 1 -2(4k2 - 1)(“^) 12k + 9 = 4k2 + 2 + 4k2 - 1 2 M Luego: 2A?-3k-2 = 0 => (2A:+l)(k-2) = 0 2 44.- Recordar: C kp-oxp-í») sen— = •--------- ab C_P-(P-c) COS2 ab fórmula de los semiángulos S = ^¡ab. sen C ... área del A ABC En el dato, al reemplazar tenemos: . 1 , „ , 2C , 2C 4.2 ab . sen C = ab . sen + ab.cos Resolución de Triángulos Oblicuángulos
2ab sen C = ab (sen2 y + eos2 j Finalmente: sen C = .’. C = 30° ó C = 150° 45.- Recordar: S = ab. sen C ... área del AABC C C sen C = 2 seny. cosy ... arco doble 2 2 sen a - 1 - cos 2a ... arco doble 1 C En el dato al reemplazar tenemos: 4. t, ab. sen C. tany = ab Q Q C C senf . _ 2C 1 2. 2 seny cosy -----y- =1 => 2 sen y = 2 COSy 1 1 Al efectuar, obtenemos: 1 - cos C = 2 => cos = 2 =* = C r- Finalmente: tan = tan 15° = 2 - v3 46.- Recordar: a = 2 R sen A => sen A = yp; .. . ley de senos S=^ ... área de un AABC => 64R3.S3 = arfe 4R ... 2.2 2 a b c W-a be . 2r 2R ' 2R Finalmente: W = 8S3 8R3 64R3.S3 8R3 W = 47.- a + b + c = 2p ... perímetro AABC Recordar: S = = 4R . .. área de un AABC. Luego: W=(£^T£) abe 4SR 2p = 2p 2(pr)R W = P Finalmente: W = 2R.r Problemas de Trigonometría y cómo resolverlos I ftíRACSO WlDlTOlEJ
48.- Del dat >: media geométrica = 4abe = 2 a/51 elevando al cubo: abc = 72S Se sabe que: abe 4R Finalmente: = 728 = 546 42 ’aabo 13^/3 43 43 3 I— 7 SAABC=14^3 Cm ' V- sen 2 A ; en 2B + sen 2C = 4 sen A sen B sen C ... en todo AABC Recordar fl = 2Rsen A b = 2RsenB ley de senos c = 2 R sen C 7 ohc S = 2R senAser B sen C= .. área de un AABC W = 1R‘ 2 sen2 A. cosA sen2B.cosB sen2C.cosC senB senA senC 7 V/ R [2sen A cos A+2 sen B cos B+2 sen C cos C] W = 2R“[2 senA cos A +2senB cos B +2sen C cos C] W = 2R2[sen 2A + sen 2B + sen 2C] VV = 2R2[ 4senA senB senC] VV = 2R2[4sen A sen B sen C] W = 4 [ 2R2senA sen B sen C] = 4S = 4 y (dato: R = 1) Finalmente: V* = 50.- sen(B + C) = sen A __en todo AABC Recordar: b = 2R sen B, c = 2R sen C 2 2 sen(B + C). sen (B - C) = sen B - sen C ... arco compuesto 7 S = 2R sen A sen B sen C ... área de un AABC Resolución de Triángulos Oblicuángulos R20 671
4 R2 (sen2B - sen2C). senBsenC W= 2sen(B-C) 2 R2. sen(B - C). sen(B+C). senBsenC sen(B-C) W = 2R2. sen A sen B sen C W=S 51.- Recordar: sen(B + C) = sen A _ sen(B+C) cot B + cot C =--ñ----7T senB.senC ... en todo AABC ... arco compuesto S = 2R2sen A sen B sen C ... área de un AABC ... ley de senos. 2 . sen(B+ C) W = 2(2RZ sen A sen B sen C).---„---7^ senBsenC a = 2R sen A W = 4R2 . sen A. sen A = (2R sen A)2 \N=a 52.- Recordar: a = 2R sen A ... ley de senos S = 2R2 . sen A sen B sen C ... área de AABC sen 2A + sen 2B + sen 2C ... 4 sen A sen B sen C ... en todo AABC S = R(2 sen A cos A + 2senB cos B + sen C cos C) S = R . (sen 2A + sen 2B + sen 2C) S = R 4 sen A sen B sen C S 2R2 2R2 = 4R R(R - 2)2 = 0 R=0óR=2 53.- sen 2A + sen 2B + sen 2C = 4 sen A . sen B sen C ... en todo AABC Recordar: a = 2R sen A .. ley de senos 7 S = 2R . sen A sen B sen C .. área de AABC Factorizando el circunradio en el lado izquierdo de la igualdad. 2 R(2 sen A cos A + 2 sen B cos B + 2 sen C cos C) = r R2(sen 2A + sen 2B + sen 2C) = 2 => R2. 4 sen A sen B sen C = 2 2 R2 sen A sen B sen C) = 2 c- i 2 S = 1 g S Problemas de Trigonometría y cómo resolverlos * A RACSO WlDITOlBS
54.- En todo AABC: A + B + C = ti => y + Luego, se cumple que: A B B C A C , tan "2 - tan + tan 7^ . tan^- + tan tan-^ = 1 .... propiedad suma de tres arcos ABC Se convierte en: W = cot-2 -cot^ . cot , del dato 2p = 4r => p = 2r 2 A BC , A B C Pr Pero: S = r .cot - cot 2 • cot =pr despejando: cot Tycot 7^ cot y = ^2 pr p 2r Al reemplazar en W, tendremos: W = = — = — = 2 Finalmente: W = 2 A 55 .- Se sabe que: S = p .(p - a) tan-^ . - - área del AABC A Para que se cumpla el dato: tan "2=1 Luego: y = 45° A = 90° 56 .- Recordar: a = 2 R sen A ... c = 2R sen C 7 S = 2R sen A. sen B sen C .. área de un AABC (2R sen A), sen B + (2R sen B). sen A = c 4R sen A sen B = c Multiplicando en ambos lados por: Rsen C 7 2(2R sen A sen B sen C) = c. R sen C c 9 i— 2S = c. 2 => c = 4S c = 2 VS 57 .-Recordar: a cos B + b cos A = C ... ley de proyecciones c = 2R sen C ... ley de senos S = 2R2sen A sen B sen C ... área de un DABC. Expresando W en términos de senos y cosenos; se tiene: Resolución de Triángulos Oblicuángulos
a | b ccosB+ bcosA cosA cosB___________cosAcosB_______ 1 1 ” 1 sen2A’sen2B 2senAcosA.2senBcosB W = c . 4 sen A sen B = 2Rsen C . 4 senA senB W = c . 4sen A sen B = 2 Rsen C. 4 sen A sen B (2R2senAsenBsenC'l S W = 4^ r J w = 4 r 2 x 58 .- Recordar: S = 2R . sen A sen B sen C... área de AABC ( n \ ( 2n \ ( \ ( im \ V2n+1 sen \2ñ+"l / • sen\2ñ+T/ senUhJ • • • sen\2n + l/ = 2" S = 2(4)2 . sen y . sen . sen r> 471 i™ 3n . Pero: sen -y = sen I ti—y I = sen “y . . . arco suplementario c no ti 2ti 3ti . c -J2(3)+1 . c A i= 2 S = 32. sen—.sen—. sen— => S = 32. -------5--- .. 5 = 4 -v7 m 7 7 7, 2 serie trigonométrica con n - 3 59 .-Recordar: a = 2R sen A_ley de senos S = 2R2 . sen A sen B sen C... área de AABC sen 2A +- sen 2B + sen 2C ... 4 sen A sen B sen C ... en todo AABC Reemplazando y factorizando, tenemos: 4™'»^ +setóB.SSE +sa?c. JgLl =4 L senA senB senvj 2R2[2 senA . cos A+2 senB cosB+2 sen C cos C] = 4 2R2[sen 2A + sen2B + sen2C] = 4 => 2R2[4 sen A sen B sen C] = 4 2R2 sen A sen B sen C = 1 .’. S = 1 60.- Recordar: cot A + cot B = sen(A+ B) senA.senB .. arco compuesto R20 674 Problemas de Trigonometría y cómo resolverlos •^¿RACSO
S = 2R2. sen A. sen B. sen C_área del AABC Del dato: sen(A+B) senA. senB Pero: sen(A + B) = sen C______en todo AABC sen C = sen A sen B 3 Si: cos C = 5 => 4 => sen C = jj’ S = 2R2. sen A sen B . sen C = 2R2 sen2C sen C Reemplazando los datos: S = 2(10)2. = 200 . S = 128 m 61.- Del dato: cot A _ cotB _ cotC _, 3 _ 5 _ 7 cot A = 3k cot B = 5k cot C = 7k Si: A + B + C = ti Se cumple: cot A cot B + cot B. cot C + cot A. cot C = 1... propiedad suma de 3 arcos (3k)(5fc) + (5k)(7k) + (3k)(7k) = 1 => 71k? = 1 => 2 / ‘ Además área círculo = tiR -- 90ti => R = 3 VIO cm. S = 2R2sen A sen B sen C = 2(90). 445 446 2430 c 213 r- 2 S = V71 cm Resolución de Triángulos Oblicuángulos R20 675
2 A B C 62.- Recordar: S = p . tan y - tan y - tan ____área del A ABC w = tan-y. tany. tany .p2 P2 w=4=s.p'2 P 63.- Recordar: a + b + c - 2p .. perímetro AABC S = r2 A B . cot ~2 cot C . cot ~2 - - - área de un AABC 2p 2pr2 W~ S_~ pr _2 W = 2r 64.- En todo AABC: A + B + C = ti 2 B C n 2 + 2 ~ 2 eo.(A+f+£ A B cot y +cot y C + coty A B = coty • cot ~2 C . coty .... propiedad suma de tres arcos c 2 , A S = r . cot ~2 B + cot C cot y ... áreas del AABC Del dato: B C coty - cot y • coty 2 = k k S = 3k p2 65.- Recordar: S = p(p - fl)tan .. área del AABC Se sabe que: a + b + c = 2p_perímetro del AABC 2a 3a = 2p => 3fl P=T Reemplazando en la fórmula del área AABC . tan C 3«2 a A S= 4.tan2 66.- Sabemos que. ra = p tan y Reemplazando en el dato, tenemos: R20 676 Problemas de Trigonometría y cómo resolverlos 4A RACSO P 1DITOII1
A. B, 2 B. C 2. A. C 4k2 P tan— tan— +p tan—tan— + p tan—-tan— = —=- K Z Z Z Z Z Z y p2ítanJytany + tany tany + tangían y) = \ ¿ ¿_____2 2 2 2 / r 1 propiedad 2 2 2 Luego: p r = 4k pr =2k S = 2k 67, Recudan sen^ - /ESES 2 y be S = pr ... fórmula de los semiángulos ... área de un AABC W=A 1 + 1 + J- - be „„ 2 A ac 2 B ab' 2 C sen — sen — sen — W (p-b)(p-c) + (p-a)(p-c) + (p-a)(p-b) (p-a)+(P~b)+(p-c) W~ (P~a)(P~b)(p—c) =-¿-= A S2 (pr)2 r2 P 3p-(a + b+c) = (p-a)(p-b)(p-c) 68.- S = -Jp.(p-a)(p-b)(p-c)... área de un AABC Recordar: A seny f(p-b)(p-c) be A _ kp-b)(p-c) lan 2 - V F(p-«) fórmula de los semiángulos Resolución de Triángulos Oblicuángulos R20
w = w = (p-b)(p-c)+(p-«)(p-c)+(p-«)(p-fe) P(p-fl) F(p-b) P(P-í) .S2 (fcc)<<pj£(^+(K)2.<Ezíñ^+(fc)2.(£zfñrf ' ' (M («I2 (»<’) (p-b)2(p-c)2+(p-fl)2(p-c)2+(p-g)2(p~b)2 p(p-«)(p-b)(p-c) ,S2 (p-b)2(p-c)2+(p-fl)2(p-c)2 + (p-fl)2(p-b)2 W = 1 69.- Recordar que el punto de intersección de las mediatrices es el circuncentro S que viene a ser el centro de la circunferencia que circunscribe al AABC. Del gráfico: AD = ÓC = 2B m Z MSC = B ÉE = ÉA = 2C mZBS = C SB = SC = R circunradio área SAPSM = (PS)(MS). sen(n- A)... (1) EnelLBPS: En el LSMC: por la ley de senos: PS = R cos C MS = R cos B a = 2R sen A => 2^2 = y/2 2 senA senA 1 1 7 En (1): SAPSM = "2 (R cos C)(R cos B). sen A = R cos C cos B sen A . cosC cos B sen A cosB.cosC senA 70.- Recordar que: S = pr = ra(p - a) = rb(p - a) = rb(p - c)... área del AABC W = 5 5,5 S p'p-a p-b'p-c 2bc S2 W = — (p-b)(p-d+p(p~fl) p(p-c)(p-b)(p-c) 2bc R20 678 Problemas de Trigonometría y cómo resolverlos ARACSO
También: S = -Jp(p—a)(p-b)(p-c) — fórmula de Herón w_ p2 -pb-pc + bc+p2 -pa _ 2p2 -p(a+b+c)+bc 2bc ~ 2bc 2p2-p(2p)+bc _ _bc_ 2bc ~ 2bc 71.- Se sabe que: S = ra(p - a) = rb. (p - b) = rc. (p - c)... área del AABC Reemplazando tenemos: S___Sjl ( S___S_^ _ S S p-b p-clip-c p-a J ~ ¿ ’ p-b ' p-c S. (p-c)-(p-b) (p-b)—(p-c) .S. (p-g)-(p-c)! _ 2S2 (p-c)-(p-c)J - (p—b)(p—c) (b-c). (c-c) = 2(p-c)2 => (b-«).(c-fl) = 2[£±|±£-«]2 => (b-c).(c-c) = (fc+C2 C) 2bc - 2ac - 2ab + 2a2 = b2 +c2 + a2 - 2ab - 2ac + 2bc 2 2 2 a =b + c ... teorema de Pitágoras Es un triángulo rectángulo, recto en A (A = 90°) 72.- Recordar: S = pr = ra(p - fl) = rb(p - b) = rc(p - c) área del s= Vp(p-«)(p-fc)(p-c) AABC Reemplazando tenemos: s___s a P-b P q W= - a p-(p-fl)- • . p-(p-fl). p-(p-b) p.(p-b) q W= - P 2p-(c+b) p-a p-bj p ’ (p-c)(p-b) w______________ w ~ p(p-«)(p-b) p-c _ S-c.(p-c) p-c “ S2 1 . 1 s s b s b S c-(P-c) c 4 w=__= _ W = 2 Resolución de Triángulos Oblicuángulos R20 679
73.- Recordar: S = pr = p(p - fe) = tan = pb(p - fe) ... área del AABC B B r = (p - fe) tan yr^p-tany (p-fe)tan^-ptan^ Reemplazando tenemos: W =------------------ + 1 -fe. tan -(2RsenB).tan(Bj W = —2r— + 1 =-----------2p------- + 1 - - • ley de senos „ B B W =-I zsen-^ cos^ sen^ cos^ ... arco doble W = 1 - 2sen2 (y j ... arco doble W = cos B LÍNEAS NOTABLES 74.- Por medianas: Por ley de cosenos: (1) + (2): Asi mismo tenemos: (3) + (4) + (5) tenemos: 4[(wia)2 4(ma)2 = b2 + c2 + 2bc cos A o2 = fe2'+ c2 - 2bc cos A 4(ma)2 + «2 = 2b2 +- 2c2 ... (3) 4(wib)2 + fe2 = 2a2 +2c2 ... (4) 4(mc)2 + ¿ = 2a2 + 2b2 ... (5) (r»b)2 + (mc)2]=3(«2 + fe2 + c2) ...(1) - - (2) W = 2 2 2 mfí + mb + mc «2 +b2 +c 2 Finalmente: W-l 75.- Recordar: "a mayor ángulo se le opone el mayor lado" Sean los ángulos: a - r, a, a +- r, las medidas de los ángulos interiores del AABC y r la razón de la progresión aritmética. En todo AABC: A + B + C = n R20 680 Problemas de Trigonometría y cómo resolverlos RACSO M•D1TOBES
a + (a - r) + (a + r) = ti 71 a= 3 2 2 2 Por medianas: 4(wia) = b + c + 2bc cos A 4(ma)2 = 262 + 102 + 2(26)(10). cos j i \2 1036 nrn í'tcrx = $ = 259 =* ma = *259 Finalmente: ma = 16,09 cm 76.- Del dato: Por medianas: Pero: 2 sen"2 A sen~^ 4(ma)2 = b2 + c2 + 2b cos A... (1) c—b 2-Jbc cos A = 1 - 2 sen2 j ... arco doble (2) .2" Reemplazando (2) en (1): 4(wia)2 = b2 + c2 + 2bc 1 — 2¡ c — b j 2-Jbc J 4(m)2 = b2 + <? + 2bc - 4bc. a 4ab 4(wa)2 = b2 + c2 + 2bc - c2 + 2bc -b2 = 4bc Finalmente: wi. = Vbc 77.- Del dato: Por medianas: Pero: 4(ma)2 = b2 + c2 + 2bc cos A ... (1) cos A = 1 - 2 sen2 j ... arco doble En (1) tenemos al reemplazar: 4bc = 1? + + 2bc 1-2 sen2 2 A b2+c2-2bc 5611 2 = ---------- 4bc.sen2 = b2 + c2 + 2bc - 4bc Finalmente: 4bc Resolución de Triángulos Oblicuángulos
78.- o2 = b2 + C2 - 2bc cos A_ley de cosenos a = 2R sen A ... ley de senos 2 Recordar: S = 2R . sen A sen B sen C ... área del AABC 4(ma) = b + c + 2bc cos A ... medianas Multiplicando por 4 en ambos lados y agrupando tenemos: 4R . senA.sen A + 8S.cos A = 4 sen A fl2. sen A + 8(2R2 sen A sen B sen C). cos A = 4 sen A 2 2 C a + 16R . • 2R - cos A = 4 => a + 4bc cos A = 4 2 2 2 2 b + c - 2bc cos A + 4fec cos A = 4 => b + c + 2bc cos A = 4 4(ma)2 = 4, tna = 1 79.- Se sabe que: i 2bc A , . Va = k= ^¡^.cos 2" bisectriz interior 2bc A Va =‘l=b^sen2 bisectriz exterior En el dato, al reemplazar tenemos: b+c b-c _ JL 2bc + 2bc ~ 3 Al efectuar obtenemos: Finalmente: 2b 1 2bc ~ 3 c = 3 80.- Se sabe que: i. 2bc A VA = ^=fe^-cosy Reemplazando tenemos: .. 2ac B VB = 9=^-cos 2 C . cos bisectrices interiores del AABC VC-t~ a + b a + b b+c a+c a+b 2bc + 2ac + 2ab ab + ac+ab+bc+ac+bc abe 1 2(ab + bc+ac) W = q ,----- 2 abe «r 1 1 w = — + + a b R20 682 Problemas de Trigonometría y cómo resolverlos RACSO iDITClH
81.- Bisectriz interior: 2ab C VC=^b cos 2 •W bisectriz exterior: 2ab C Vc=^K sen 2 &> (1W2), tenemos: Vc a-b . C V’c “ a + b cot 2 Por ley de tangentes: a^b=^ a + b tanl Pero: A+B+C=n A+B _ n C 2 “ 2 ’ 2 tan C = cot^ Pero: 82.- cos(B Vc tan| V'C tanl C . cot ~2 Vc = (j2 +l).tan tan A = 60' = csc 45° - cot 45° = v2 - 1 B + C = 120° + C) = cos 120° = n Del dato: —— = -. m VA (VA)2 = mn Vc = 1 cm En AABDC (ley de senos): Va m senB sen 30° => VA = 2 m . sen B ... (1) En AADC (ley de senos): Va = n senC sen 30° => VA = 2n . sen C ... (2) (1). (2): (VA)2 = 4mn sen B sen C = 2 sen B sen C => transformando a una diferencia de cosenos = cos(B - C) - cos(B + C) Resolución de Triángulos Oblicuángulos R20 683
cos(B-C) = | | =0 => B-C = 90° ...(3) Pero: B + C = 120° ... (4) De 3 y 4: B = 105° y C = 15°, finalmente: B = 105° y C = 15° 83.- S = pr... área del AABC A Recordar: cot-^- = csc A + cot A_arco mitad a = 2 R sen A... ley de senos A r = (p - a) tan~2 - - - inradio Al reemplazar las expresiones anteriores en W, obtenemos: W = 2Rr sen A + r2(csc A + cot A) => W = r(2Rsen A) +- r2 . cot W = r a+ r. cot A A W = r a + (p - a) tan - cot W = r[« + p - «] = pr W=S 84.- Recordar: S = p(p-«)tan y =ra.(p-«) s = Vp(p-«)(p-*Xp-c) s- 4R - Pr área del AABC abcr„______ = 4RS re(p-a)(p-b)(p-c) s2 P w=«p = 4£p=á?=4(í) K=w 85.- Recordar: sen(A + B) = sen C__en todo AABC S = 2R2 sen A sen B sen C... área del AABC S = pr = (p - «). ra ... área del AABC c = 2R sen C... ley de senos. Z^>1684l Problemas de Trigonometría y cómo resolverlos RACSO BDlTDtlI
W = (pr). cot A +pra. cot B - ara. cot B W = (pr). cot A + ra(p - a). cot B => W = S[cot A + cot B] ,4r „ sen(A+B) „ senC W = S.----r---77 = 21< .senAsenBsen C.-r— senAsenB senAsenB W = 2R2. sen2C = y. 4R2 sen2C Al utilizar la ley de senos, obtenemos: W = 86.- Se sabe que: ra = p . tany - - - radio exinscrito A A Reemplazando en el dato tenemos: p - tany . cot y = 2a => p = 2a ... (1) También recordar que: r = (p - «). tany ... (2) (1) en (2): r = (2fl - o)tan y => Y — = tany — arco mitad r ~ = csc A - cot A a r csc A - „ = cot A a 87.-Recordar que: r = 4Rseny seny seny ...inradio w B A C . . rb = 4R sen y . cosy cosy ... radio exinscnto r, „ i . . xt> B B C _f A B C Reemplazando tenemos: 4R seny .cosy .cosy = 3i 4Rseny seny sen y A C 1 = 3tany . tany ... fórmula de los semiángulos |(P-fe)(P~c) l(p-a)(p-b) P(P-a) V P(P-f) 3b - 2p => 3b = a + b + c ( P~b\ ”T¡d 2b = a + c = 2b p = 3p - 3b 88.- Recordar que: . A . c tan y + tany = COSy-.COSy .. .arco compuesto Resolución de Triángulos Oblicuángulos
/A+C\ /ti B\ B senl -¿"I = senl —-?! =cos y B A C => rb = 4Rsen y - eos cos B „ , , B A C C°S2 Reemplazando tenemos: W = 4Rsen . cosy cos cos—.eos— 2 2 । B B i W = 2R| 2seny cos I = 2Rsen B .. .ley se senos W = b A B B 89.- Recordar: ra = r. tan ;rb = r. tan^, rc = r. tany Reemplazando tenemos: cos y. cos W = p. tany.p^ tany + tan—j. csc A p. tany.p( tany + tan y) COSy seny COSy sen(^-y) seny COSy COSy sen^y-yj 1 2 sen y cos y i senA senA ' 2senycosy senA ’ W = csc B CUADRILÁ TEROS 90.- Recordemos que en todo cuadrilátero se cumple: SI. S3 = í>2. S4... (1) Por dato: Sx = 1 m1 2, = 2m2, S3 = 4 n? Nos piden "S4" En (1): 4 = 2S4 1 j Finalmente: S4 = 2m Problemas de Trigonometría y cómo resolverlos RACSO wp BDITOKBS
91.- Los AABD y ZACD son notables BD = 24 y AC = 20 Como es inscriptible se cumple el teorema de Ptolomeo: (20)(24) = (7)(15) + (25)x 480 =105 + 25x 25x = 375 Finalmente: x= 15 92.- En un cuadrilátero inscriptible: Sabemos que:S = -J(p-«)(p-fe)(p-c)(p-d) ... (1) Donde: 2p = 3 + 5 + 6 + 8 = 22=> p = 11 En (1): S = SV8.6.5.3 = 12 V5 - - - f) Se cumple el teorema de Ptolomeo: AC.BD = 3.6 + 45. 8 AC . BD = 58 _ ... _ AC.BD También: S =------ sen a C S = 29 sen a ... (**) (*) = (**): 29 sen a = 13 ^5 93.- Por ser cuadrilátero circunscriptible se cumple teorema de Pithot: 12 + 52 = x + 25 x = 39 También se cumple: S = -Jabcd .sen Pero por dato: S = 650 iz2 => V12.25.52.39 .sen Efectuando: sen Pero: cos(A + C) = 1 - 2 sen2 = 650 Finalmente: cos(A + C)=Z Resolución de Triángulos Oblicuángulos R20 687
94.- No pide: " cos 6" Usando ley de cosenos es: AABC: x2 = l2 + 22 + 2(1)(2) cos 6 ... (1) AACD: X2 = 32 + 42 -2(3)(4). cos(Tt-O).. .(2) -cosG (1) = (2): 5 - 4 cos e = 25 + 24 cos 6 5 => 28cos e = -20 cos6 = -y c 95.- Como es un cuadrilátero bicéntrico: S= -Jsent). tan6.cos6.cot0 = -JsenO.cosOzí2 También se cumple el teorema Pithot: sen 6 + tan 6 = cos 6 + cot 6 Rápidamente: 6= Luego: S = Jsen^.cos^ = V 4 4 c V2 2 S= -y U 96.- Usando la ley de cosenos: A BCD: x2 = «2 +- d2 - 2nd . cos 0 ... (1) ABCD: x2 = b2 + c2 - 2bc. cos(ti - 0) ... (2) (1) = (2): o2 + d2 = 2ad cos 6 = b2 + c2 + 2bc cos 6 despejando: cos 6 = l-cos 6 l + cos6 - - (4) A ¡(b+c)2 -(a-d)2 (3) en (4): tan y = ^(fl + d)2 _(fc+c)2 A tan = !(b+c+a-d)(b+c -a+d) (a+d+b-c)(a + d+c+b) A /(2p—2d)(2p—2a) 2 - y (2p—2c)(2p—2b) Simplificando: A | (p - a)(p - d) ~2 = y(p-b)(2p~c) Problemas de Trigonometría y cómo resolverlos ARACSO WeDITO«M
97.- Como el cuadrilátero ABCD es inscriptible se cumple: C = 7t - A => sen C = sen A S = Sj + S2 „ be . ad . S = ~2 .sen A + .sen A («íf+fec) S =----2---- sen A 2S = {ad + be) sen A Como se cumple por dato: a + c = b + d Entonces el cuadrilátero es circunscriptible, luego el cuadrilátero es bicentrico S= -Jabcd Finalmente: .._ S _ 1 M- 2S “ 2 98.- Por ser circunscriptible se cumple: teorema de Pithot: a+c=b+d => a-b = d-c Elevando al cuadrado: a2 + b2 - 2ab = c2 + d2 - 2cd... (1) Usando la ley de cosenos: AABC: AC2 = «2 + b2 - 2ab. cos B ... (a) A ADC: AC2 = c2 + d2 - 2cd. cos D ... (P) (a) = (P): «2 + b2 - 2ab cos B = c2 + d2 - 2cd. cos D... (2) (2) - (1): 2flfe(l - cos B) = 2cd (1 - cos D).. . (*) Pero: 2 % 1 - cos x = 2 sen Enf): . , 2B , 2D 4z?b.sen = 4ca.sen Extraendo la raíz cuadrada: Con lo que queda probada la demostración 'ab sen 2 - •ded. sen 2 Resolución de Triángulos Oblicuángulos
99.- Sabemos que el área de un caudrilátero circunscriptible es: S = m abcd - sen (1) Del problema anterior: ’ab . sen y = -dcd . sen (2)en(l): ab sen esc = -Jabcd ... (2) c , B D ÍB+D S = ab sen -esc -sen I—— Con lo que queda probada la demostración. 100.- Sean AD = BC = tn, AB = DC = n En ADAB por la ley de cosenos: (2b)2 = m2 + n2 - 2m. n cos a ... (1) En AABC por ley de cosenos: (2a)2 = m2 + 11- 2mn . cos (n - a)... (2) De(2)-(1): 2. 2 4(o - b ) = 4 mn cos a El área del paralelogramo: S = 2(SADAB) „2_i,2 De (3): S =-------.sen a ' ’ coso. S = 2-^mn sen a = mn sen a S = (a2 - b2). tan a Problemas de Trigonometría y cómo resolverlos RACSO IDITOÍ1I
Números Comntejos FORMA POLAR Y EXPONENCIAL 01.-Sabemos: sen^ =cos^ = => Z = ícos“ + isen—) 4 4 2 \ 4 4/ Por la fórmula de Moivre, tendremos: Z = cos 40? + i sen 40. ? => Z = cos lOn + i sen lOn 4 4 Pero: cos lOn = 1 y sen lOn = 0 .-. Z = 1 02.- Tenemos que: Z = 1 + i-j3 Multiplicando y dividiendo por 2 el segundo miembro obtenemos: z-2[2+'-rj 71 1 7t ! tr n Pero: eos— = — , sen— = — => Z = 2 lcos^+ísen^ Elevando a la sexta y usando la fórmula de Moivre: Z6 = 26^cos^6.—j+isen^6.^jj => Z6 = 26(cos 2ti + i sen 2rt) Pero: cos 2n = 1 => sen 2n = 0 .‘. Z6 = 64 m „ j 7t -Jh+^lZ ti •Jb-Jz 03.- Recordemos: cos^tt =----, sen^r = , - 12 4 12 4 Por la fórmula de Moivre: Zol *“ " •‘'I = 21 eos 12 +isen Z - 2 cos 2 + i sen I - 64 i Estudio de la Trigonometría con Números CQmplejos R21 691
nz4 1 71 - 71 27=161 cos-j + isen — Z4 = 8 (1 +173) Reemplazando en M: M = [ - 75 .641 + 8 (8) (1 + i J3) ] Efectuando: M = 1 04.-Como: Z = cos 20 + i sen 20 Z = cos 20 - i sen 20 Luego: Z. Z = 1 Z + Z = 2 cos 20 Reemplazando en (1): ^=-1 2cos20 cos 20 = - J3 2 12 05.- Tenemos: Z 1-75* 2 pero: 1 = -1 V3 + i 1-73i Dividiendo entre 2 el numerador y denominador: Pero: J3 2 57t 1 senT =2 V3 , 1 . ' 2 21 1_73 . 2 2 1 Tt 1 cos3 - 2 ti V3 sen 3 - 2 cos^ + isen~ Z=-----------— 71 • 71 eos—-zsen^ í5tt 7 = 171 e 3 í7tt e 6 ™ 7it (Z) = -g- 06.- Tenemos: Zj = 2 + i Z2 = 3 + i Zj . Z2 — 6 + 5i + 2 -2 1 pero: i = - 1 r- 71 cos 5 6 = Z = Z = * +i. * Jl 42 Pero: 71 71 _JL sen -r = cos -r = 4 4 V2 => ZrZ2 = 572 (cos^ + í.sen^j .-. ZrZ2 - 541 .cis — .'jí RACSO V^BDITOREH R21 692 Problemas de Trigonometría y cómo resolverlos
07.- Sea: Z = , multiplicando por (a + bí) el numerador y denominador. (a + bi)(a+bi) a2-b2+2abi - a2-b2 lab . (a-bi)(a+bí) a2+b2 a2+b2 + a^ + b21 Pero también: Z = e,a = cos a + i sen a ... (2) (1) = (2): cosa = ab a2 + b2 ...(3) sen a = lab ...(4) a2 + b2 (4) .(3): tana = 2ab a-b FORMA EXPONENCIAL 08.- Como: Z = eiq = cos q + i sen q => Z3 = cos 30 + i sen 30 => Re(Z3) = cos 30 Z2 - cos 20 + i sen 30 => Re(Z2) = cos 20 Z = cos 0 + i sen 0 => Re(Z) = cos 0 => Re(Z3 + Z2 + Z) = Rr(Z3) + Re(Z2)+ Rr(Z) Reemplazando en "M": M = cos 30+cos 20+cos 0+1 2cos0 Transformando a producto el numerador: (cos 30 + cos 0) + (1 + cos 20) - 2cos0 Simplificando: M = cos 20 + cos 0 Pero: cos 20 = 2 cos20 - 1 => M = 2 cos20.cos 0 + 2cos20 2cos0 7 M = 2 eos 0 - 1 + cos 0 => M = 2(cos0 + |) - | "M" toma su máximo valor cuando: cos 0 = 1 m_ 25 9 16 M-8’8'2 M =2 Estudio de la Trigonometría con Números Complejos R21 693
09.- Tenemos: Z = cos 5o - i sen 5o = é -,27 -i,135‘ Z = e 27 Í.135‘ = e Luego: . . 1 . -¡.135° M= . ¡135“ + e Pero: ¿135“ + e~í l35° = 2 cos 135° = 2 /2 2 2 ... (1) (1) en (*): M= -U(-V2) v'2 M = -1 e 10.- Tenemos: Z = 3tt . sen -¿-+1 eos L___O_______ V2+ÍV2 Por la propiedad de co-razones sabemos: 371 71 sen~g- = eos g 371 71 cos g — sen g ..(1) También: v2 + i v2 = 2 J2 -V2 2 2 71 eos—+/sen 4 - - (2) e (1) a (2) en (*): Z= — 71 - 71 cos 77 +/sen— L O O 7t 7t eos—+ »sen— Usando la fórmula de Euler tenemos: „ _ e 8 8 - 15 2e 4 e 4 2e 4 z=-2 11.- Tenemos: Z = 2 sen 0 + 2 V3 cos 0 - 4 i sen 71 Factorizando: 1 -J3 2 sen 0 + -y- .eos 0 - 1 sen sen- - 6 cos— 6 Pero: „ 7t cos 0.cos + sen „ 7t 0.sen = cos A e' 6 RACSO vp EDtTOBLBl R21 16941 Problemas de Trigonometría y cómo resolverlos
Z = 4 Por la fórmula de Euler: Z = 4 . | Z | = 4 12.- Tenemos: Z = e"2l0(l - e8,e) Multiplicando termino a termino: Z — e216 - ei60 Factorizando: Z = -ei2e(í>140 - e"'40) Pero: e140- e140 = 2 i sen 40 => Z = ei2e .(2 i sen 40) Por la fórmula de Euler: Z = - (cos 20 + i sen 20) 2i sen 40 Evaluando: Z = (2 sen 20 . sen 40) + (- 2 cos 20.sen 40) Re(Z) Im(Z) sen20 2 M= -cos20 ’ -- M =-tan 20 13.- Tenemos: -iG • rx e = cos 6 -1 sen 6 Reemplazando en "W": VV = -C°S^ yy _ *cos0—i -sen0—i u 6 Pero: i2 = -1 => VV = . [sen 0 - i (1 - cos 0)] Pero: sen 0 = 2 sen | . cos , 1 - cos 0 = 2 sen2^ , «*r M Q ® 6-0 2 0 => W = q 2 sen —- cos 77 - i. 2 sen t? vi 2 2 2 Factorizando: 2 sen tenemos: VV = |j.2sen®.^cos^-isen^ ) = ^.sen ®.e’iG/2 Como: -7t<0<O=> <0 => sen |<0a sen > 0 => sen >0 |W| = |.sen f Estudio de la Trigonometría .con Números Complejos
14.- Tenemos: 2 eos2 ® + i sen 0 j = M eos2 .cis 0... (*) D o 0 0 Como: sen 0 = 2 sen . cos En (*): ( 2 eos2 ® + 2i sen — . cos^ J2 = M cos2^ . cis 0 Factorizando el primer miembro: 2 cos f j2 - cos^ - i sen j2 = M eos2 ® .cis 0 . 2 0 , ie/2.2 20 ie , . ¡e .. ¡e . .. . => 4 eos 2 - (e ) = M • eos 2 - e =* 4 . e = M . e 15 .- Tenemos: Z + ^ = 2 sen 0 ...(*) r-y r-j | r-j | ¡6 1 1 ~Í6 Sea: Z = | Z | . e => - e En (*): | Z | eie + .eie = 2 cos 0 Pero: 2 cos 0 = elG +• e,e ।..। ie 1 -ie ie -ie ________ vi i => |Z|.e+j2j.e=e+e =>|Z|=1 ie _ -i4e 1 Z = e => M = e - “He e Pero: ei4e - e’40 = 2¿ sen 40 => M = (2i sen 40)i = 2Í2 . sen 40 /. M = -2 sen 40 16 .- Sean: Z = | Z | . e,e el número complejo buscado Zi = (- 1 +-^i)4 = 24 2n 1 2n V3 Pero: cos-^- = 3 = ~2~ => Zj = 16 (cos2 + isen^ j = 16 . e12n/3 R21 696 Problemas de Trigonometría y cómo resolverlos RACSO
Por dato del problema: Z.Zj = 4 . e' ,ie. 16. ei2,t/3 = 4ei’1 |Z|.eie= |-ein/3 Z= |.ei“'3 4 17.- Ordenando "Z" obtenemos: Z = (sen 3x - sen x)+i(cos x+cos 3x) eos*+cos 3* Usando las fórmulas de transformaciones trigonométricas: Z = (2cos2xsenx)+¿(2cos2x+cosx) 2cos2x.cosx Eliminando "2 cos 2x": sen x+t cosx cosx Pero: sen x = cos a cos x = sen Z = —“— eos” * Usando la fórmula de Euler: .. n Z = sec x. e L Como:xe (0;n/2) sec x 18.- Tenemos: Z = sen 82° + i cos 82°.. Por co -razones trigonométricas tenemos: sen 82° = cos 8o a cos 82o = sen 8° Reemplazando en (*): Z - cos 8o + i sen 8o = e'8 -,15“ , i.8°.15" ¡.120“ 1 -¡.120“ z =(c ) = e A Z!5 e Luego: W = e¡120“ + ei-120’+l Pero: ¡.120” . -¡.120“ o Q e + e =2 cos 120 = 2 -1 W = 0 Estudio de la Trigonometría con Números Complejos R21 697
19.- Usando la formula de Euler en "M" tenemos: (^3ei 3r^3 '-51 M=------ .eiir Usando teoría de exponentes: zcJ60’ M=(3y (15.e'33 ) i. 120° = e M = pos 120° + i sen 120* 1 2 J3 2 1 - V3 M "2 +1 2 20.- Tenemos: eS'*+e4'V 12 Efectuando: Pero, recordemos: e4“-l ix i n _ ir/2 e -1 = 2 i sen e M = 2i.sen2xe'2* -2í.sen4xe4x M = sen2x .¡2x sen4x ’ -i2x Pero: sen 41= 2 sen 2x . cos 21 a e - cos 2x - i sen 2x M = ’ 2sen 2xcos2x ‘ <cos ' 1 sen 2x> M = - ^ + i tan 2x Re(M) = -| 21.- Tenemos: |e®- 11 =2...r) Pero: ¡e . o - 0 ¡e/2 e - 1 = 2 i sen . e En(*): L- 0 ¡e/2 2i. sen 2 e I-2- Sea: o 6 ¡e/2 Z = 2 sen . e Como: e 2 6 n sen 2 > 0 e 2 Problemas de Trigonometría y cómo resolverlos 0ÍACSO LdlTOBBI
Pero: |Z| = |ÍZ| | ¿Z | = 12i.sen 6 .ele/21 => | ÍZ | =2 sen . r} G n 6 -i 6 Tt En (*•): 2 sen % = 2 => sen = 1 =*2 = 2 " fi = ít 22.- Como: sen x = cos^-x a cos x = sen > Z = cos^-xj -1 sen Pero: eix+l = 2cos| .eix/2 > Z = e i(7t/2 'x) Z + 1 = 2 cos(^-|. e"'W Como: Multiplicando por (-1): n 2 < X < Tt 71 * Tt 4 < 2 < 2 7t X Tt 2 < 2 < 4 Sumando <0; e IVC => Z + 1 = 2 cosí’?2 *). í^4 2^ 442'42 \4 2/ Como: ' 2 e IV=>cos^-^j >0 arg(Z + l)= * - 23.- Por el desarrollo de un binonuo al cuadrado tenemos: ¡46 . o i2e , , , ¡26 , , .2 . ¡26 , „ ¡6 , , , ¡6 , x2 e +2e +1 = (e +1) => e +2e + 1 = (e -1) I e +11 Luego: Z=l^e^l .(•) Pero recordemos: ex + 1 = 2 cos. elx/2 => elx - 1 = 2i sen—. elx/2 Reemplazando en (*): 2cos6,e16 2isen—.eie/2 \ ¿ J Simplificando: - 2 a je 7 _ cos o e “ .2 2 0 i -sen 2 cos20 Te sen — 2 (cos 0 + i sen 0) Iwi(Z) = - eos2 6. sen 6 20 sen 2q/o 0 0 cos O.^sen^ cosíj 20 sen - Estudio de la Trigonometría con Números Complejos
Im(Z) = -2 cos20 . e cos 2 ' e sen— Im(Z) = -2 cos26. cot 24.- Sea: 7 _ (1 + cosO+isenO)2 cosB + ¿sen 0 „ O • O -7 (1+€ ) Pero: cos 0 + i sen 0 = e => Z = ----gj- e A, 0 ¡e/2\2 „ |2cos—£ I r, , Í0 n 0 ¡6/2 ~ \ 2 I Pero: 1 + e = 2cos^.e => Z=-------------jg------ 4 20 --------- = 4 eos y e---z Por la fórmula de degradación: Z = 2 (1 + cos 0) = 2 + 2 cos 0 =>2 + 2cos0 = a -b cos 0 => a — 2/\b = -2 2a + b = 2 25.- Sea: Z = 2 + -Js + i => „ n J3 Jt 1 Pero: cos z = -’x-, sen = z => t> 2 2 Por la fórmula de Euler: Z = 2(1 + em/6) 7 1- Z = 2 Ilb3 +2' Z = 2(1 + cos + i. sen ) => Z = (>/6 + ^)ei7t/12 =(VA + Jb ).cis0 Luego: A = 6,B = 2,0=^ /. A0 = A 612 26.- Como: Z = cos 20 + i sen 20 Por la formula de Euler: Z = e12e => M = 1 .i. J2e A2 1 + e I 2O 1_£,í26 I +cot0 Pero sabemos: eix + l eix-l 1 JC 7 cot2 R21 700 Problemas de Trigonometría y cómo resolverlos - J^ACSO PBDITOKEa
Luego: M = - y . cot 0 j + cot20 => M = . cot2© + cot20 Pero: i2 = - 1 => M = - cot2© + cot2© .-. M = 0 27.- Tenemos: _ sen0+tcos0 ~ sen0-ícos0 Multiplicando por "i" el numerador y denominador tenemos: 2 i cos0+«sen0 • —í2cos0—ísen0 Pero: .2 -(eos O í sen 0) 1 ~ " cos0+isen0 Luego multiplicando por "-1" ambos miembros y usando la formula de Euler tenemos: -Z = -¡e e___ eie e-i2e arg(-Z) = - 20 28.- Por co-razones trigonométricas: sen 20 = cosí y - 20 | => cos 20 = sen! y - 20 Por la formula de Euler: Z = e'W2~2e> Luego: Z + 1 = c*"'220» Pero: " = 1 = 2cosy . e™'2 => Z + 1 = 2eos^ J - 0 j. ei(,l/4‘0) Como: 0 < 0 < y => -y < - 0 < 0 => cos|£-0|>O .-. |Z+1| = 2 cosí % -0 4 4 4 I 4 I 11 14 29.- Tenemos: Pero: a+íb ¿7 , - e = V3 + i... (*) /5 - -J3 - 1 V3 +I = 2^—+ l.-j Tt -J3 Tt 1 cos 6 = 2 =* Sen 6 = 2 Estudio de la Trigonometría con Números Complejos R21
=> y¡3 + i = 2 í cos z + i sen z I a eLn2 = 2 l ° 6 I Por la fórmula de Euler: J3 + i = 2 ein/6 = í’L1'2 . ei,l/6 = eLn2 + in/6... (**) (•) = (”): ea+ib = eLn2 + in/6 . « = L«2 30.- Tenemos: _ 1 + cos26 - ¿sen 26 ~ l-(cos26-isen26) _i20 Pero por la formula Euler: cos 26 - i sen 26 = e Z = l + eí2e l-ei2e _ (e i2B + l) (ei2e-l) (*) Recordemos: eix + l rlx l 1 x 1 cot2 En (*): Z = - j . cot (~^F) = f • c°t 6 => Zí = cot 6 Pero: |Z| = |Zí | Como: 6 e IIIC, cot 6 > 6 |Z| = |ZÍ| =cot6 31.- Tenemos: Como: íy2 W = e ...(*) -y 10 Z = r. e ^2 2 i2(J Z = r . e ir2 pl2n Reemplazando en (*): W = e Pero: e126 = cos 26 + í sen 26 => Pero: í2 = -l yy _ (cos2G + isen20) < A t ii2. cos20 + i^.r^.sen 20 W — e W = í,-r2sen26 e¡r2 .cos26 Luego: |W| = e-f2“n2B 32.-Tenemos: W = => Como: Z = >/5 . e,e => -5 ¿20 Reemplazando en (*): W = e ‘ Pero: e126 = cos 26 + i sen 26 => i2z2 -z2 W = e z =ez ...(*) Z2 = 5ei2e t * r -5.cos20 -i. 5 sen 20 W = e . e - - (1) Pero: 1 1 6 = arc tan => tan ® = 2 R21 702 Problemas de Trigonometría y cómo resolverlos RACSO B^EDITOKES
Recordemos: cos 26 =------ñ— =----f 1+tan 0 1+— 4 _5Í3\ Luego en (1): W = e *5' . e“,scn2° |W|=e’3 33.- Tenemos: Por la formula de Euler, en (1): Z = l + ícos^+Zsen^ \ 2 2 l-(cos®-ísen^ j6 (e2 + l) - ¡e -(/ 2 -1) ..(1) (2) Pero: ¿e/2 + 1 = 2 cos £ . ? 6/4 4 eíe/2-l=2/sen(--) eie/4--2i sen(-) eie/4 O £ — c i En (2): Z=------i—y = T cotj-e”'2 => ÍZ = (ct*®) f‘n 4 3tc gy q 3 ti O tt , 0 Como- — < 0 < 2ti => -ó < v < „ => scot / > 0 2 o 4 4 4 |Z| = |¿Z| =cot$ 34.- Tenemos: Recordemos: En(*): Pero: M = csc 6 0 G ( 0; n/2) Estudio de la Trigonometría con Números Complejos
35.- Tenemos: Z = sen 46-¿cos 46 sen46+ícos46 o Por el criterio de reducción al primer cuadrante: sen 46 = cos ( - 26) a cos 36 = sen( y - 46) c°s(^-4e)-¿.sen(^-4e) Reemplazando en (*): Z =------\;----------------c cosí 4®)+ ,sení) /(í'46) Por la formula de Euler, tenemos: Z = —r~ = e',(’I‘80> => z = ei(8e-,l) => org(Z) = 86-7t Pero: arg(Z) = y => 86-7t= J 6 = § 36.- En la expresión propuesta usamos la formula de Euler: e-i(4a+P) e¡(4a+P) c-i2¿ + c¡2a = A. cos(Ba + P) Usando teoría de Exponentes: e'1(2a+^+ e,(2a+^ = A. cos (B a +P)... (*) Recordemos: eIX + e'x = 2 cos x En (*): 2 cos (2a + P) = A. cos (B a + P) => A = 2aB = 2 A + B = 4 _ (l+e’28)4 37.-Sea: Z = 5 - - -C) Recordemos: e,x + 1 = 2cos . eix/2 p 7 (2cos6-<’ie)4 16-cos4O.r,4B 4„ „„ 4„. En (*): Z =-------¡40--- = ¡46 => Z = 16 cos O = 2(8 cos 6) e e Z = 2(3 + 4 cos 26 + cos 46) => Z = 6 + 8 cos 26 + 2 cos 46 Problemas de Trigonometría y cómo resolverlos J^RACSO Pbditobbi
Luego: 6 + 8 cos 20 + 2 cos 40 = A + B cos 20 + C . cos 40 => A = 6, B - 8, C - 2 -> M = M = 1 o i_ 38.- Tenemos: Z = l+¿2e+¿49+¿6e ¿2e + l Factorizando el numerador: Z = i2B+ei4B+ei6B í-12B + 1 Z = (l + ei2B)(l + ei4B (ei2B + l) Simplificando nos queda: Zi = e i40 Pero: elx + 1=2 cos Z = 2 cos 20. <’12e Pero: 71 4 20 e cos 26 > 0 | Z | = 2 cos 26 39.- Tenemos: 7 ?(-3x) + l Recordemos: ix -i n x ix/2 e +1=2 cos 2 e e - 1 = 2í. sen En(*): 2ésen(-3A).e,(“3x) cos\ 2 r - ésen(3*) Pero: sen 3x = 2 . sen 3jc . cos ~2 e i(3x/2) : - cos +1 sen eos Z= — -í.2sen| Simplificar: 7 1^1 Í3jt1 Z = - 2 +i.-.cot(T) Re(Z) = -| Estudio de la Trigonometría con Números Complejos R21 705
REGIONES SOMBREADAS 40 .- Sea: Z = x + iy = re18 Como: Z. Z 1 => (x + iy)(x -iy)<1 Por diferencia de cuadrados: x - i y <1 Pero: i2 = - 1 => x2 + y2 < 1 . . (*) Como: Z = re18 -> arg(Z) = 0 Pero: 0 < org(Z) < ~ => 0 < 0 < | ... (*’j De (*) a (**) obtenemos: De la figura: S = ~ (l)2 .'. S = ií2 41 .- Sea: Z = x + iy = |Z| eif> => |Z| = Jx2 + y2 ^arg(Z) = 6 Como: |Z|e[2;4] => 2< Jx2 + y2 <4 => 22 + y2 <42 ... (*) Como: flrg(Z) = 0 => flrg(Z3) = 30 Pero: flrgfZ3) e [0; 0 <30 < Interceptando (*) a (**) obtenemos: Por la formula del trapecio circular: S = JCM2 RACSO 1D1TO1KI R21 706 Problemas de Trigonometría y cómo resolverlos
42 .- Sea: Z = x + iy = | Z| . r'B Z = x-iy => | Z | = yjx2 + y2 Pero: 1<|Z| <2 -> l2^*2 + j/2<22 ... (*) Como: Z= |Z|.eie => Z3= |Z3|.ei3e => arg(Z?) = 30 => 0<36<3n -> o<e<7t... (**) De (*) a (»»): De la figura: S = | (22- l2) .-. S = w2 43 .- La ecuación de la circunferencia: (x + 2)2 + (i/2 ) = 22 En el plano complejo: | x + 2 + iy | < 2 , pero: z = x + iy |z + 2|<2 a %<arg(Z)<^ R= |zeC/ |Z + 2| <2, %<arg(Z)<3^j 44 .- Sea: Z = x + iy = rel0 Tenemos: (Z + 1)( Z + 1) < 1 => (x + iy + l)(x - iy + 1) < 1 Por diferencia de cuadrados: (x + l)2 - z2!/2 < 1 Pero: i2 = -1 => (x + 1 )2+ y2 < 1 ...(*) Como: Z = rf i0 => Z2 = rV20 => 2? = írV20 Pero: i = cos 7^ + i sen 7^ = en/2 => iZ2 = 2. e120 iZ2 = r2 ein/2+2S => arg(iZ2) = ^ + 26 Pero 2ít < flrgfiZ2) < 3tt => 2tt < ^ + 26 < 3tt => — < 6 < — ... (**) Estudio de la Trigonometría con Números Complejos R21 707
o 7t 7T V r-» X-» z 71 • 71 * Pero: sen — = cos = -77- Z = 8(cos ~ + 1 sen -r ) 4 4 2 v 4 4 ' Usando la formula de Moivre: Z1/3 = 3/r - (cosí n^4 + ¿senf 71 + I I 3 I 14 Si: k = 0: Vz = 2(cos— + i sen—) De la figura: S = S = 2’l/3 w2 46.- Tenemos: | Re(Z) | + | Im(Z) | < 1 Sea: Z = x + iy, donde: x = Re(Z), y = bn(Z) Se presentan 4 casos: | x | + | y | <1 R21 708 Problemas de Trigonometría y cómo resolverlos ARACSO IflDITOlll
i)x + y<l iii) - x + y < 1 ii)x-y<l iv)-x-y<l De (i), (ii), (iii) y (iv): Área = 4A = = 2w2 4(1)(1) 2 47.- Si: Zj = 2Z17:/3 => | Zí | = 2 a argfZJ = - n/3 Zj = 2?’t/6 => |Zj| = 4aflrgíZJ = n/6 Zj = 2e'T/2 => |Zj| = 1 aarg(Z¿ = n/2 Graficamos y usando los ángulos notables para determinar Zt, y ^3- Usando el método de determinantes: Luego: S= |(2+2^-(-5))i/2 S= | (7 + 273)1? * Estudio de la Trigonometría con Números Complejos R21 709
«CO2* Límites y Derivadas Trigonométricas LÍMITES TRIGONOMÉTRICOS sen(2cos6) ti 0 01.- Si: W = ---------, en el límite cuando evaluamos con 6 = ~, tendremos: ~ cosü 2 0 En este caso conviene transformar así: 2.sen(2cos6) 2cos6 Ahora hacemos el cambio de variable: x = 2cos0 Además reconocemos que al evaluar: 0-> o x—>0 Luego tomamos el límite: lím w = 2.sen(2cos6) senx 2cosfi ~ ’ *->0 x lím W = 2 sen(x + /¡)-senx 02.- Si: W = ' ' h senx—senx Evaluamos el límite cuando h = 0: W =------- 0 ~ => forma indeterminada Transformando a producto la diferencia de senos, la expresión W se convierte en: (h\ seri — => lím W-lím -------í—‘ .lím cosíx+^J => lím W = (1).cosx = cosx /i->o /i->o h ii->o \ 2/ fr->o 2 R22 710 Problemas de Trigonometría y cómo resolverlos RACSO JDiTOlll
03.-Si: W = 2 2 sen x-sen a .2 „2 Evaluando en el límite cuando x = a: W =-5-5--- = — => forma indeterminada a -a 0 En base a la identidad del arco compuesto y descomponiendo la diferencia de cuadra- dos en el denominador, la expresión W se convierte en: sen(x - a).sen(x - a) (x + a)(x-a) Es pertinente reconocer que cuando: x-+ a o (x - a) —> 0 lím x—*a lím sen(x+fl) sen(x-o) x + a x-a Aplicando límite para el producto de dos funciones, tendremos: ,, >*7 i' rsen(r+fl)l Fsen(x-fl) hm W = hm -------------- . hm ------------- x—yfí x->a L x+a J x-o->0 L x—a límW = sen(fl + fl) lím W = sen 2a 2a w = x3+l 04.- Si: VV = ~ 32? sen(l-xj (-1)3 + 1 Evaluando en el limite cuando x= -1: W = . ,.2, sen[l-(-l) ] 0 ~ => forma indeterm. Descomponiendo en el numerador la suma de cubos y en el denominador factorizando el signo ( - ) para el argumento del seno, la expresión W se convierte en: (x+l)(x2 - x + l) ~ -sen(x2-l) Multiplicando numerador y denominador por: x - l x2 -1 x—x2—l sen(x2-l)‘ (x-l) Es preciso reconocer que cuando: x -+ - l o x2 —> l o x2 - l —> 0 lím x-»-i VV= lím x-^-l . x-x2 -l (x-l) x2-l sen(x2 -l) Límites y Derivadas Trigonométricos
Aplicando límite para el producto de dos funciones: lím W - hm 1—>-l X—*—1 x-x2-! lím ---------¿ - 2-i ,o |_sen(r -1) (-l)-l-l límW= .(1) -3 lím W = 05.- Sea: ... senvx +4-2 W=-------j--- 3 lím W = ~ 2 sen(2-2) Evaluando en el límite cuando x = 0 tenemos: VV = --- 0 = ~ => forma ndeterm Multiplicando el numerador y denominador por: Jx2 + 4 - 2, y luego por: Vx2 + 4 +2; la expresión W se convierte en: sen(Vx2 + 4-2) Jx2 + 4 -2 -Jx2 +4+2 (A2+4-2) r2 Jx2 + 4+2 sen(Vx2 +4-2) (Vx2+4-2) J' >^/x2+4+2 sen(Vx2 +4-2) 1 (Vx2+4-2) J (Vx2+4+2) -> W = Cuando: x —> 0 o Jx2 +4 -2 —> 0 < > y —> 0 y t > sera/ lím W = lím ----------- x-»o y->o y , (Vx2+4+2) lím W = (1) x-»0 1 (2 + 2) lím W = — x—>0 4 c ... Vi + senx-Vi-senx 06.- Sea: W =------------------- 1-1 o En el limite cuando x = 0 tenemos: VV = —q- = — => forma indeterminada Racionalizando el numerador, la expresión W se convierte en: (1 + senx) - (1 - sen x) x.| Vi + senx + Vi - senx ] Problema de Trigonometría y cómo resolverlos RACSO Wy IDlTOlll
(senx^ i lím VV =2 lím ~~ . lím k , h ~ x-»o x—»o \ x J x->o vl + senx+-Jl-senx 1 lím W = 2.(1) - x—>0 2 07.- Sea: 1—sen— VV =------2- Tl-X lím W=1 >->o 1 71 1SCno 0 En el límite cuando x = it, tenemos: W =-------------— = — 71 — 71 0 => forma indeterminada sen---1 Ordenando convenientemente tendremos: W = 2 x—Tt Cuando: x —> Tt o x - ti —> 0, y si hacemos: x - ti = y => x —>7i o y—> 0 Asimismo, si: y = x - ti => x = ti + y ... (2) Reemplazando (2) en (1), tenemos: W = sew^+2^~1 y y Aplicando reducción al I cuadrante: y l-COSj — 12 lím W= lím ----------— x-»n y-»0 y 1 lím 1-co{l S; w=-2^—¡yT (2 J Aplicando la regla de H’ospithal: /úmW=0 X-»1t Límites y Derivadas Trigonométricos R22 713
08.- Si: sen(x-2) W=^i" sen(2-2) 0 En el límite cuando x - 2, tenemos: VV = Z3 ~ = T => forma indeterminada Z —o U Descomponiendo en el denominador la diferencia de cubos, la expresión VV se convierte en: sen(x-2) (x-2)(x+2x + 4) Luego de ordenar convenientemente evaluamos: lím VV = lím x—>2 x-»2 sen(x-2) x-2 1 9 x + 2x+4 Debemos reconocer que cuando: x —>2 o x-2 —> 0 lím W = ’x%^0 x->2 sen(x-2)l 1 x-2 jj™ x2+2x + 4 1 => hm W = (l). 22 + 2(2) + 4 lím VV = — X .C 12 O9._ Sea: w Jsen3x)(s^ (xx)2 0 En el límite cuando x = 0, tenemos: VV = ~ => forma indeterminada Factorizando en el denominador, la expresión VV se convierte en: (sen3x)(sen5x) (sen3x)(sen5x) W= [x(l-x2)]2 = x2(l-x2)2 Multiplicando el numerador y denominador por : 15 sen3x sen5x 15 VV =----- -------. ----y, 3x 5x (1- x2)2 Luego de ordenar convenientemente evaluamos: lím VV = lím x—>0 x-»0 15 (1-x2)2 R22 714 Problemas de Trigonometría y cómo resolverlos RACSO DITO1B1
Debemos reconocer que cuando: x —> 0 o 3x —> 0 a 5x —> 0 lím x—>0 VV = lím 3x—>0 sen3x 3x . lím 5x—>0 sen5x 5x 15 S a-*2)2 lím W = (1). (1). —~ x—»o (l)2 lím W = 15 x—*0 10.- Sea: VV = sen3x 5x22x En el límite cuando x = 0, tenemos: VV = — => forma indeterminada. En base a la identidad del arco triple y factorizando “x" en el denominador, la expresión VV se convierte en: VV = (senx)(2cos2x +1) x(5x-2) lím VV = lím x—»0 senx'i f2cos2x + l x l'l 5x-2 Aplicando límite al producto de dos funciones, tendremos: lím VV = lím x—>0 x—>0 . lím x—>0 í 2cos2x+lA l 5*-2 J lím W = (1). x—>0 l0-2 J 11.- Sea: VV = -sen3x senx-sen2x 0 En el límite cuando x = 0; tenemos: VV = — => forma indeterminada. En base a la identidad del arco doble y transformando a producto la diferencia de senos, la expresión VV se convierte en: 3x x x —> 0 o — - > 0 a — -> 0 Debemos reconocer que cuando: Límites y Derivadas Trigonométricas
itoHf). 3x • => lím VV = --------2t-v- límsen\2/ x 2 lím VV = - 3 x->0 12.- Sea: x-sen2x x + sen3x lím x—>0 (1)0) (1) W = - 0 En el límite cuando x = 0, tenemos: VV = ~ => forma indeterminada Dividiendo el numerador y denominador entre x; la expresión VV se convierte en: x-sen2x x w —------~— x + sen3x x 1 sen2x 2 VV =-----— sen3x + 3x Debemos reconocer que cuando: x —> 0 o 2x —> 0 a 3x —> 0 2 2(sen2x lím VV = +3|seJ?3A lím VV= x—>0 1- lím 2x >0 \ 2x I 1 + lím 3.(ge¿p*^ 3x—>0 \ 3X I 1-2(1) S W“ 1 + 3(1) 13.-Sea: VV = (6 + 4).sen(it6) 62-16 lím W x-»o 8.sen(47t) 0 En el límite cuando 0 = 4 tenemos: VV = —77—~r~ = ~ 16—16 0 => forma indeterminada. Descomponiendo la diferencia de cuadrados en el denominador, la expresión VV se convierte en: R22 Problemas de Trigonometría y cómo resolverlos Sí RACSO ^BDITCMEB
(6 + 4).sen(7i6) W= (e + 4).(6-4) (1) Debemos reconocer que cuando: Además hacemos: a - 6 - 4 6 = a + 4 ... (2) sen[n(a + 4)] sen(47i + 7ia) Reemplazando (2) en (1), tenemos: W =--------- ------------ a a Pero por ángulos coterminales: sen(47t + na) = sen (na) sen(7ia) lím W = lím ------------ e-»4 a—»o a lím W= lím 6—>4 7»-»0 sen(na) Tía . 71 lím VV = (1) . Ti = n 3sen(7tr)—sen(37tr) 14.- Si: W =------------------------ o o En el límite cuando x = 0 tenemos: 3(0)-0 w=-F- forma indeterminada lím W= lfm 3sen(7tx) - lím sert(37tr) => x—>0 X x—>0 W= sen(Ttx) hm ———-3ti - TUC „ sen(3rcr) „ Zm —qLT—-dTt x^O (1). 371 -(1).3ti lím \N= x—>0 lím W = 0 x—>0 15.- Sea: cos(mx)-cos(nx) VV = -------2------- 1-1 . - - 0 En el límite cuando x = 0, tenemos: VV = —~ = ~ => forma ind terminada En base a la identidad de transformación trigonométrica, la diferencia de cosenos la pasamos a producto y la expresión W se convierte en: Sea: 2se w =---- => lím W = -2. lím x—>0 x—»o sen 2 sen 2 4 y=(m-n).- a z = (m + n).~ Debemos reconocer que cuando: (m - ri). - (m + n).— ->0 Límites y Derivadas Trigonométricas R22 717
1 lím VV = - — lím x->o 2 y-»° seny y senz ,(m-n).lím -----.(m + n) 1 => lím VV = - — . 1. (m - n). 1. (m + n) x—>0 2 lím W = x—>0 2 2 n —tn 2 16.- Sea: l-cos 3% 1 -cos4x 1-1 - - 0 En el límite cuando x = 0, tenemos. VV = ——~ = — => forma indeterminada. En base a la identidad del arco doble (degradación) la expresión VV se convierte en: VV = 9 ^2Í 3* zsen — J 2 , 2sen (2x) Debemos reconocer que cuando: x —> 0 3x 2 2f 3x' sen — VV = lím------2,3 x—»o sen (2x) 3x\2 sen^ Zím x—»o sen2x lím x-»0 f,. 3XV2 Zimsen^r- x—»o 2 lím sen 2x x—»0 Q sen^.- |. lím 2 3x .n 2 2X 2 sen2x 2x—>0 2x Otro Método: a cuando: x —> 0 o 3x —> 0 a 4x —> 0 lím 3x-»0 (3x)¿ l-cos4x (4x)2 .16 —.9 2 —.16 2 9 lint W = — x—>o 16 lím 4x-»0 Problemas de Trigonometría y cómo resolverlos RACSO
2 17.- Sea: W = ----7= 1--Jcosx 0 0 En el límite cuando x = 0 tenemos: W - ——- = — => forma indeterminada Multiplicando el numerador y denominador por: 1 + Jcosx , la expresión W se convier- te en: x2 (14-Jcosx) _ (1+Jcosx) (1 --Jcosx) (1 + Jcosx) 1-cosx x2 lím x—>0 W = lím x—>0 (14-Jcosx) 1—cosx lím x-»0 X^ofl + Vcosx) lím 1 COSX x->0 2 X 2. 1 2 lím W = 4 x-»0 18.- Sea: VV= 2-Jcosx-cosx x2 En el límite cuando x = 0, tenemos W = 2-1-1 0 => forma indeterminada £ 0 También VV se convierte en: 1-cosx 1-Jcosx w= x2 + x2 Evaluando el límite se tiene: lím x-»0 ,, 1-cosx 1-Jcosx W = hm -----5— + hm -----5--- x—>0 X x—>0 X 1 + -Jcosx 1 +-Jcosx lím W = ^- + lím ^~c~sX. lím------2 - - x—>0 2 x—>0 Xa x—»o 1 +-Jcosx 1 2 2 2 + 2 ’2 19.- Sea: VV = l+cos(7tr) x2-2x + l 3 .'. lím VV = ~ X .o 4 En el limite cuando x = 1, tenemos: 14- cosit 0 W = q _ = ~ => forma indeterminada Debemos reconocer que cuando: x —> 1 o x - 1 —> 0 o y = x- l —>0 limites y Derivadas Trigonométricas R22 719
Sí: y = x - 1 o x = y + 1 Reemplazando en la expresión original, se tendrá: l+cos[7t(y + l)J l+cos(7t+7ty) VV = -------2------ y l-cos(Tty) y2 => lím VV = lím 1 c„os(Jiy) -TI2 *->1 x—>0 Ti2.y2 Debemos reconocer que cuando: y —> 0 o ny —> 0 lím VV = ti2- lím x->i «y->o 1 —cos(7ty) 2 1 2 ~ ‘ O (™/) 2 lím W = 20.- Sea: En el límite cuando x = 1, tenemos: VV = 71 eos— n ___2. =2 1-1 0 => forma indeterminada Multiplicando el numerador y denominador por: 1 + Jx, la expresión VV se convierte en: Si ha emos el cambio de variable: y = 1 - x => x — 1-y ... (2) Debemos reconocer que cuando: x> 1 o 1 - x -> 0 o y = 1 - x -> 0 (l + Jl-yXsenj^y^ Reemplazando (2) en (1) tendremos: VV =----------1¿ (l+>/l^y).sen(^y) --- „ =$ lím W = lím ————-—- => lím W = lím (1+ Jl-y )- lím —é-¿ T y->° y x-u y->° v y->° ZE v 2 2‘* 7t Debemos reconocer que cuando: y—>0 o ~ y —> 0 R22 720 Problemas de Trigonometría y cómo resolverlos RACSO ^DITORBB
senfyy => lím W = 2 lím --------'---- X->1 |y_,0 2E V 2 * n n 2=2^ 2 hm W = ti 21.- Sea. W =---------- 1-cos6x o En el límite cuando x = 0, tenemos: W = -—- 0 = ~ => forma indeterminada Transformamos la expresión de manera que: 1 W= — 36 - (I) 1 - cos6x Si hacemos el siguiente cambio de variable: V = 6x (2) Debemos reconocer que cuando: 0 6x Reemplazando (2) en (1), tenemos: y-»0 2 y 1 W= — 36 1-cosy lím 22.- Sea: 2 1 W = lím ~ 36 1 — COS V 1 lím W = —- lím x—»o 36 y—»tí 2 y 1-cosv 1 1 /,m W = 36 T x->0 •’Ó 2 1 lím W= ~ x—»o 18 18 7t-3x W=--------- l-2cosx Tt En el límite cuando x = —, tenemos: W = 71 . 1 -2cos— 3 71- 0 0 forma indeterminada Ordenando convenientemente tendremos: —(Tt—3x) W= —'------- 2cosx-l En base a la identidad del arco triple, la expresión W se convierte en* (7t-3x) W = -------r cos3— 2 (7t-3x).cos— W = ------t-----v- (n 3x^ sen------- 12 2 1 cos— 2 Límites y Derivadas Trigonométricas
Si hacemos el siguiente cambio de variable: Debemos reconocer que cuando: n n-3x y=\ 7t-3x y-* o lím \N = lím 2.sen -eos—.(n-3x) 7t-3x 2 - - (2) n lím VV = -2 cos —. lím Y—>5 r— 3 sei 7t-3x 2 y V3 lím W =-2. —— . hm 2 y—>0 SenV 3 => lím VV = -V3(l) 3 3 cosx-cos2x 23 .- Sea: VV = —------- 1-cosx 1-1 En el límite cuando x = 0, tenemos: _ _ 0 VV = ~~ = ~ => forma indeterminada En el numerador la diferencia de cosenos pasamos a producto y en el denominador por la identidad del arco doble (degradación), la expresión VV se convierte en: —2 sen VV =------ 3x 2sen* Debemos reconocer que cuando: x —> 0 lím VV = x->0 lím x-»0 lím W = senl lím ^->0 a 3x 2 2_____ senh lím____L x n 3x 2 1 ’2 2 lím W = I(1) lím VV = 3 x->0 R22 |722 | Problemas de Trigonometría y cómo resolverlos RACSO WEDITOIBI
c l-cos(l-cosx) 24 .- Sea: VV =------------ .4 . 11 0 En el limite cuando x = 0, tenemos: VV = —-— = — => forma indeterminada Transformaremos la expresión multiplicando el numerador y denominador por: (1 - cos x)2 convirtiéndose en: VV = .2 1 — cos(l — cosx) (i- cosx): (1-COSX)2 .4 Si hacemos el cambio de variable: y = 1 - cos x Reconocemos que cuando: x —> 0 0 a y —> 0 VV = lím l-cos(l—cosx) (1-cosx)2 1 - COSX 1 Z2 1 - cosy => lím VV = lím ,,2 x-»o y->o y lím x2 1-cosxV ji lím 1 — cosx x—>0 2 if lím W = ~ . x-íO ¿ f-- “ 2 ’ I 2 1 lím W = - x-»0 o 0.sen0 25 .-Sea: VV = ----- l-cos 0 En el límite cuando 0 = 0, tenemos: 0 W= 1-1 0 — =» forma indeterminada En base a la identidad del arco simple, la expresión VV se convierte en: ©.sen© 0 VV =-----2Z =------ sen 0 sen0 0 lím VV = lím -------- e->o o->o sen0 26.- Sea: 1 + senx vv= ------- 1 + cosx 3n En el límite cuando evaluamos para x = —, tenemos: 1+sen— l + (-l) 0 VV = -----2_ = = — => forma indeterminada 1+cos3rt ’+l1! 0 Límites y Derivadas Trigonométricas
Multiplicando el numerador y denominador por: 1 - sen x y en base a la identidad del arco simple y arco doble (degradación), la expresión W se convierte en: w _ 1 + senx 1 -senx =* ~ l + cos2x ' 1-senx 2 1—sen x => W = ------9---------- 2 cos x.(l — senx) cos1 2x => W = -------2---------- 2 cos x.(l — senx) 1 1 => lím W = lím ------------r => lím W = -------—r x_^ 2(1-senx) 2(l-sen^) 1-^COSX 27.- Sea: W =------------ x.senx 1-1 0 ' En el límite cuando x = 0, tenemos: W = = ~ => 1 lím W = - ,3n 4 forma indeterminada Aplicando la identidad algebraica de la diferencia de cubos: a3 - b3 = (a - b)(fl2 + ab + b2) => 1 - (^cosx)3 = (1 - ^cosx )(1 + í/cosx + -t/cosx 2) 1-cosx i 1+cosx La expresión W se convierte en: W =---------_ —. . _ ~------- r . 3/ - . 3/ 2. X-Senx 1 + rncY 1 + Veos* + Veos X 1-bCUbA Asimismo al multiplicar y dividir por: 1 + cos X, W se convierte en: 2 sen x W= ------3f---- 3/~~2~------------------ (1 + Vcosx + Veos x).xsenx.(l + cosx) ,, 1A, ,, senx 1 hm W - hm ----- . hm -- —. -------- x-»0 x—>0 X x-M) (1 +^fcosx + Veos2 x)(l + cosx) 1 fimW = l. (1 + 1 + 1)(1 + 1) 1 lím W = x-»o 6 28.- Sea: 1 - cos8x sen8x En el límite cuando x = 0 , tenemos: 0 0 => forma indeterminada En base a la identidad del arco doble (degradación) la expresión W se convierte en: Problemas de Trigonometría y cómo resolverlos ¿Ííracso 'P*D1TOKBS
=> w = 2sen24x 2sen4x.cos4x VV= sen4x cos4x W= tan 4x => lím W = lím tan 4x x>0 i-»O lím W = O x-»O 1 +senx-cosx 29.- Si: VV = ----------- 1-senx-cosx En el límite cuando x = 0, tenemos: 1+0-1 VV =------ 1-0-1 o “ => forma indeterminada Transformaremos la expresión W ordenando y aplicando la identidad del arco doble: senx +1-cosx VV=------------- -senx+1-cosx 2 sen cos +2 sen2 -2 sencos 7^ + 2sen2 2 sen y í cosij+sen VV= ----- 2sen^- cos^ + sen => lím VV = lím x-»0 x—»0 cos—+sen— 2 2 sen---cos— 2 2 lím VV= x->0 o-l lím W - - 1 sen2x 30.-Sea: VV=-----------~ cosx + cos 3x 71 En el límite cuando x = ~, tenemos: VV = 2 cos3- + cos- 2 2 senil 0 ~ => forma indeterminada En el numerador aplicamos la identidad del arco doble y en el denominador la suma de cosenos la transformamos a producto. La expresión VV se convierte en: 2senxcosx W=------------ 2cos2xcosx senx W = —— cos2x 31.- Sea: lím VV = lím ----— x_>* cos2x 71 sen— lím VV=-----2- ,-5 COSTt 2 lím W= -1 *-»o (1 -senx)3 (1 +cos2x)3 Límites y Derivadas Trigonométricas
sen| Tt 0 En el límite cuando x = ~ tenemos: W = —-----------------5- = — 2 (1 + cosTt)- 0 forma indeterminada. En base a la identidad del arco simple y el arco doble (degradación), la expresión W se convierte en: (1-senx)3 (1-senx)3 W = '-------— -------------------- (2cos2x)3 8(1—senx)3(l + senx)3 => lím W = lím 2 _ 8(1 +senx)3 2 lím W= 2 1 8(1+ 1)3 1 lím W = — ' 64 32.-Sea: W = 2 sen2x + 2sen x—2senx 2 cosx-cos X 0 En el límite cuando x = 0, tenemos: W = ~—~ 0 “ => forma indeterminada En base a la identidad del arco doble la expresión W se convierte en: W = 2 2senxcosx+2sen x-2senx cosx(l - cosx) w = 2senx(cosx + senx -1) cos x(l-cosx) w = 2.2sen—eos—(cosx + senx — 1) cosx.2sen2 — 2 w = —2 eos—(1 - cosx - senx) cosx.sen— 2 W = -2 eos—. 2sen2 —-2sen—eos— 2 2 2 2 w = lím W = cosx.sen— 2 4 cos— 2 cosx sen---cos— 2 2 4cos— lím - 4 .----— x-»0 COSX sen---cos— 2 2 4(1) lím W = --t2 (0-1) x—»0 4 lím W = 4 x-»0 x.sen(sen2x) 33.- Sea: W ~ ~r 1 - cos(sen4x) En el límite cuando x = 0, tenemos: 0 0 W = -—“ = — => forma indeterminada 4b RACSO WBDITOIII R22 7261 Problemas de Trigonometría y cómo resolverlos
2 Multiplicando el numerador y denominador por: sen 4x y en base a la identidad del arco doble y propiedades de límites trigonométricos, la expresión W se convierte en: 2 . sen 4x x.sen(sen2x) =-------------- ------------- 1 —cos(sen4x) sen4x.sen4x sen2(4x) W =-------1—-— 1 - cos(sen4x) x.sen(sen2x) 4 2sen2xcos2x.sen4x ' 4 2 sen (4x) sen(sen2x) 4x 1 1 -cos(sen4x) ' sen2x ’ sen4x ' 8cos2x Reconocemos que cuando: 2 sen 4x 4x sen(sen2x) lím W = lím ~ ~ ~:. lím ---------. lím ------- - lím ----~ x-f0 sen4x—>0 l-COS(Sen4x) sen2x->0 Sen2x 4x—»0 Sen4x 4x-»0 8cos2x sen(A + B).cos B—sen(A + Cl.cosC 34.- Sea: W =------------------------------- sen(B-C) En el límite cuando B = C tenemos: sen(A + C). cosC - sen( A + C).cosC sen(C-C) 0 ~ => forma indeterminada Entonces transformamos los productos a sumas en el numerador de la expresión: 2sen(A + B)cosB-2sen(A + C)cosC 2sen(B-C) sen(A + 2B) + senA - [sen(A + 2C)+senA] 2sen(B-C) sen(A + 2B) -sen(A + 2C) W= 2sen(B-C) Transformamos la diferencia a un producto: -25en(B-C)co§(A + B + C) —2Sen(B-C) => lím W= lím cos(A + B + C) B—»C B—»C lím W = cos(A + 2C) B >C =* lím W = (2)(1)(1) 2 8 1 lím W = 7 i-»o 4 1 35.-Sea: W = ^3 -1/2+ cosx 2Z sen 3x En el límite cuando x = 0 tenemos: W = 3 ¿3 0 ------= ~ > forma indeterminada 0 0 Límites y Derivadas Trigonométricas R22 727
También VV se puede expresar como: V3 - V2 + COSX J3 + V2 +cosx VV =-----5~---’ “7=—/ sen 3x V3+v2 + cosx ... 1-cosx 1 y» — 2 - _ Debemos reconocer que cuando: x -> 0 o r w r í1 1-cosX 1 í 3x \2 to w=i„„ -fe) IA7 1 >' 1-cosx 1 f lím 3x Y hm W = - hm -----2— Ittn ' m , . 3x^07—57 x—»o y x—»o x x—>0 v3 + -J2 + cosx V senjx j 1 1 1 ¿3 lím VV = — — _ cr . 1 lím VV = —— x >C 9 2 2V3 , ,0 108 36.- Sea: senx—cosx VV = —------- 1-tanx Tt En el límite cuando x = ~, tenemos: VV - 4 Tt Tt sen--cos— n 4 4 = £ . . Tt 0 1 - tan— 4 => forma indeterminada Expresamos VV en términos de senos y cosenos: senx-cosx cosx.(senx-cosx) (cosx—senx) VV =----------- =----------------- => VV = - cos x---------------- = - cos x 1 senx cosx—senx cosx—senx cosx => lím VV = lím (- cos x) = - cos T lím VV = ------ X^J 4 x->f 2 4 4 4 tanx—senx 37.- Sea: VV =------3----- En el límite cuando x = 0, tenemos: tan(0)-sen(0) 0 VV = ------------- = — =» forma indeterminada. Expresamos VV en términos de seno y coseno, luego agrupamos convenientemente para aplicar las propiedades de límites trigonométricos: senx ---—-senx w= -cosy..------ senx(l-cosx) COSX.X3 Problemas de Trigonometría y cómo resolverlos "jí RACSO V^BDITOSOS
=> lím W = lím x—>0 x—»0 senx 1-cosx 1 X ’ r2 COSX IA7— i' Senx n 1~COSX ,, 1 llfTl »— llfTl - llfTl o * x—>0 x—>0 % x—»(J x x—»0 COSX r n => lím W = (l) y (1) .n l x. I lím VV = jr—>0 2 38.- Sea: 2 l-cos X tan2x En el límite cuando x = 0, tenemos: 1-1 0 W = o = — => forma indeterminada. W = Aplicando la identidad algebraica de la diferencia de cubos, y expresando en términos de senos y cosenos, W se convierte en. (1 - cosx).(l + cosx+eos2 x). eos2 x _P-—<0sxJ7(l + cosx + eos2 x). eos2 x => W =-----------------ñ--------------=> w =----------;---------------------- sen x 41—cúsxXl + cosx) lím VV = x->0 lím x—»0 (1 - cosx+cos x)(cos x) (1+cosx) lím W = x—»0 (1 + 1 + 1)(1)2 (1 + 1) 3 lím W = - x-»0 2 39.- Sea: En el límite cuando x = 0, tenemos: => forma indeterminada También W se puede expresar como: Es necesario reconocer que: x —> 0 o Tt 2 2 Límites y Derivadas Trigonométricas
40.- Sea: W = sec x - tan x 71 71 71 Cuando x = se tiene: W = sec ~ - tan — => forma indeterminada Pasando a senos y cosenos, multiplicamos y dividimos por 1 + sen x, la expresión W se convierte en: W = 1-senx 1 + senx cosx ' 1+senx 2 COS X W =-------------- cosx(l+senx) => lint W = cosx lint ~ r ,n 1+senx 2 71 eos— lint W =---2— 1 +sen— 2 0 lint W = —— 2 2 2 lím W= 0 2 41-Sea: W = *-2tanx l+cos4x 71 En el límite cuando hacemos x = — tenemos: 4 2 71 „ 71 sec--2tan— w=—----------* 1 + COS7I ¿(2) -2 Q l+(-l) " 0 forma indeterminada Transformamos la expresión en base a la identidad del arco simple y del arco doble (degradación), convirtiéndose en: 1 + tan2x-2tanx l + cos4x w= (tanx-1)2 l + cos4x Expresando en términos de senos y cosenos tendremos: x2 senx j cosx I W = l+cos4x (senx-cosx)2 cos2x.(l + cos4x) •y (cosx-senx) W =-----2------T--- cos x.2cos 2x •y (cosx-senx) W=------ó-----------------------2 2cos x.(Cosx -senx) (cosx + senx) lint W = lint , , 2 cos x.(senx + cosx) lint W = n J'-t 1 i2r t2 1.1 2 V2_ 4 1 4 R22 730 Problemas de Trigonometría y cómo resolverlos RACSO
42.- Sea: 3sen2x x.tan4x lím lím VV = — 2 En el límite cuando x = 0, tenemos: W = — => forma indeterminada 3sen2x Tambiéff’W se puede expresar como: W = x.tan4x / I serve 1 3 l x ) => W = —. tan4x 4x Reconocemos que cuando: x —> 0 <=> 4x —> 0 Í-.2 senx ] x ) 3 (1) lim w = ~ tan4x => límW= ~ m x-»0 * 4x->0— - x-»o * 4x 3 /mW = - x->® 4 tan(l + cosx) 43 .- Sea: VV = 77 x 7 cos(tanx) —1 En el límite cuando evaluamos en x = n, tenemos: tan(l + cos7i) o 0 W = ___7 = ~~ = ~ => forma indeterminada cos(tanTi) —1 i — i o o Multiplicando el numerador y denominador por 1 + cos x y tan x: tan(l + cosx) (1 + cosx) tan2 x W ~ " i - cos(tanx)' (1 + cosx) ’ tan2 x Agrupando en convenientemente para luego aplicar las identidades trigonométricas y las propiedades de límites trigonométricos, la expresión W se convierte en: tan(l + cosx) tan2x (1 + cosx). eos2 x (1 + cosx) ’ l-cos(tanx) ' (1+cosxXl-cosx) Si hacemos el siguiente cambio de variable: y = 1 + cos x a z = tan x Reconocemos que cuando: x no 1 + eos x —> 0 o y —> 0 Asimismo cuando: X—>710 tan x —> O o z —> O Límites y Derivadas Trigonométricas R22 731
tany z „ cos x hm W = - hm ------. hm ------ . hm ------- X-Wt y-*0 y z->0 l-cosz x~vn 1-COSX 2 => lím VV = - (1) (2). • COS 71 1—COS7I -2.(—l)2 lím VV = -:— x-tn 1—(—1) lím W = -1 X—Hl tanx+tan2x 44 .- Sea: VV =------— cosx + cos2x n En el límite cuando x = ~, tenemos: VV = 2ti t 2ti tan— + tan— 3 3 71 71 cos—+ cos— 3 3 3+V3 1 1 --1-- 2 2 = ~ => forma indeterminada En base a la identidad del arco compuesto y de transformación trigonométrica a produc- to, la expresión VV se convierte en: sen(2x + x) _ cos 2x. cosx “ 3x x 2 cos—.eos— 2 2 sen3x VV =---------------=----- 3x x 2cos2x.cosx.cos—.eos— 2 2 Y empleando la identidad del arco triple tenemos: „ 3x 3x 2sen—. cos— => W------------------------- ** ~ 3x x 2cos2x.cosx.cos—.eos— 2 2 3x sen— => lím W = lím 2 x-»— x-t— 3x 3 3 cos—.cosx.cos2x lím VV = 71 sen— 2 71 71 271 cos—.eos—.eos— 6 3 3 lím VV = ,71 3 1 J3 1 1 2 ’2‘ 2 8-^3 ^3 2 „ . , (tanx + cotx-cscx) 45 .- Sea: VV = *------------— secx-1 Reconocemos que en el límite, cuando x = O, se tiene: tan x = 0; sec x = 1 cosx-1 1-1 O Y: cot x - csc x =----= ~ = — => se presenta la forma indeterminada senx 0 0 r Problemas de Trigonometría y cómo resolverlos RACSO BDITO1BI
En base a la identidad del arco triple, la expresión se transforma en: 2 (secx cscx - cscx) W = ----------------- secx-1 csc2 x.(secx - W = 2 => W = csc x . (sec x - 1) 1 2 sen x 1 -cosx cosx W = (1 - cosx)______ (1 - cos x)(l + cosx).cosx => W »« (l + cosx).cosx - (1 + 1)(1) 1 lím W = - x—»o 2 tan(«x) 46.-Si. VV- 0 En el límite cuando x — 0, tenemos: VV = ~ => forma indeterminada Transformamos la expresión en términos de seno y coseno: sen(flx) sen(cx) 1 — w —---------------- ---------------- —s w =____________—_______ cos(flx).sec(flx) ’ (l-cos(ox)+x) l-cos(cx) x ox ax Reconocemos que cuando: x —> 0 o (ax) 0 Luego aplicando las propiedades de límites tenemos: 1 1 lím VV = x—>0 lím sen(flx) ax—>0 ax lím 1 COs(flX) Km % ax—>0 „ +x-»0 lím VV = x->0 lím VV = a x-tO ax ax x.tanx.secx 47.-Sea: VV = n+x2)sec3x-l En el límite cuando x = 0 tenemos: 0 VV = — => forma indeterminada Transformamos VV en términos de seno y coseno: senx 1 yy _ cosx cosx 3 1 cos x x.senx.cosx 2 3 1+X -COS X 1 1 a Límites y Derivadas Trigonométricas R22 733
48.- Sea: 1A, _ x senxcosx VV - 3.2 1-cos x+x x senx. cosx 2 W = ------------ -------2----2 (l-cosx)(l + cosx+cos x)+x lint í-^^Icosx _______x-»(A x /________ lím(—^P^líl + cosx+cos2 x)+2 x—>0\ X2 / lint W = x-*0 lím W = x—>0 2 lím W = — x—>o 5 W = ---------- 2arccosx-n 2 En el hmite cuando x = 0, tenemos: 0 0 _ 0 2nrccos(0)-7i 0 12 => forma indeterminada. En base a las propiedades de las funciones trigonométricas inversas y propiedades de los límites de las funciones mencionadas, la expresión W se convierte en: W = ______x______ 2^-are senx] -n W= ---------- — Zarcsenx 1 x 1 lím W = lím - — .------- =-—(1) x-w x-»o 2 aresenx 2 lint W = - x-»0 2 2 49.- Sea: 2 tanx-are senx senx En el límite cuando x = 0, tenemos: 0 w=-^ forma indeterminada También VV se puede expresar así: W = 2 tanx senx /ircsenx senx 2serpr W=<C^X ^Senx are senx x senx x W = aresenx' lím aresenx => lím W = lím 2 secx => lím W = 2 . lím secx x-»0 x—>0 senx x->o x-»o lim senx l x J x—>0 x 734 Problemas de Trigonometría y cómo resolverlos R22 ’jí - ACSO WlDITOlEI
lím W = 2 - x—»0 1 1 lím VV =1 x-*0 50.- Sea: W = (1 - x). Al evaluar para x - 1, tenemos: W = 0. ["] > forma indeterminada. Pero W también se puede expresar como; W = (1 - x). ctg 71 7tX ~2~~2 71 tan -(1-x) tt r tt zx x — tan— (1-x) 2 [2 W = 71 Haciendo: y = — (1 - x) Reconocemos que cuando: 71 o ~ (1 - x) —> 0 o y -» 0 x —> 1 1 => /ímW= — x-»i 2E 2 lím x-»l fd-x) tanfel-x) 2 lím W = — x—>1 71 y 2 lím tan ii = — • (1) y->o tany ti 2 lím W = ~ X>1 Tt DERIVADA DEFUNCIONES TRIGONOMÉTRICAS 51.- Tenemos: y = sen (3X2 + 2x + 1) => ~~ = cos (3X2 + 2x + 1). — (3X2 + 2x + 1) dx ax = («os (3x2 + + 1)). (6x + 2) ax 71 52.- Tenemos: y = sen (e - sen x) + cos , d / xrX — = cos (e - sen x). — (e - sen x) + — (cos^ I dx dx dx \ 9/ «y x r d v d — = cos(e - sen x). —(e )-----(senx) ux I dx dx + 0 Límites y Derivadas Trigonométricas
• dy —— = cos(ex - sen x). (c* - cos x) dx 53.- Tenemos: y = 3 sen4(sen x + 5) + 1 dy d A d — = 3 . — (sen (sen x + 5)) + — (1) dx dx "dx dy 3 ¿ ~ =3.4 sen (sen x + 5) . cos (sen x + 5) — (sen x + 5) + 0 dy 3 *T— = 12 sen (sen x + 5) . cos (sen x + 5) (cos x) dx 54.- Tenemos: y = sen(sen (sen x)) = cos (sen (sen x)) — (sen (sen x)) dy dy - cos (sen (sen x)) . cos (sen x) ~ (sen x) dy — = cos(sen (sen x» . cos (sen x) . cos x dx 55.- Tenemos: y = sen2(3x + 1) - 3 dy d , x d ¿ = 2sen (3x +1). — (sen (3x + 1)) (3) dy d — =2 sen (3x + 1). cos(3x + 1). — (3x + 1) - 0 dy — = 2sen(3x+l). cos(3x+l) .3 Por arco doble: dy dy -jr = 3 sen 2(3x + 1) .-. — = 3 sen (6x + 2) dx dx 56.- Tenemos: y = cos(^ + sen2xj- - =-sen ^ + sen2xk ~ í^ + sen2x) í?,] dx \4 / dx \4 I dx { o I Problemas de Trigonometría y cómo resolverlos RACSO BDITOKEI
dy ln 2 \ d(n\ d 2 x y = -sen l-^+semxl. — — +—(sen x) dx \4 / dx\ 4 I dx -O = -sen^+sen2xj. [ü + 2senx-~(senx) — = -sení— + sen2x| . (senx . cos) dx \4 / '---“---' = sení^+sen2r) . sen 2x dx \4 I 57.- Tenemos: y = 4 cos3(2x + 5) - 7 dy _ d d — =4.3 cos2(2x + 5) . (cos(2x + 5) - — (7) dy j d -^=12 cos2(2x+5) . (- sen (2x+5)). — (2x + 5) - 0 dy 2 = (12 cos2(2x + 5)). (- sen (2x + 5)). 2 dy 2 , -/ = - 24 cos2(2x + 5). sen (2x + 5) dx 58.- Sea: y = tan(sen x) + 1 dy , d d — = sec (sen x). — (sen x) + — (0) dx dx dx uy 2 , = sec (sen x). cos x dx 59.- Tenemos: y = tan (ln (x2)) + csc2^^j dy q n d n d o n — = sec2(Zn(x2)). — (ln (x2)) + ~ (csc2 —) dx ''"dx dx 10 dy j 2 i d o ¿=sec2(Mx2)).^.-(x2) + 0 = sec2 (ln (x2)). ^2 . 2x — .sec2(/n O»2)) dx x Límites y Derivadas Trigonométricas R22 737
dy d y d 60.- Aplicando: — = 3~ (tan (x + sen x)) - — (1) dy 2 2 d 2 — = 3 sec (x + sen x). — (x + sen x) dy n 2 “7“ = (3 sec x(x + sen x))(2x + cos x) dx 61 .- Tenemos: y = (3x4 - 7). cot (x2 + 1) - 5 Usando la derivada de un producto tendremos: = (cot (x2 + D) - ¿ (3/ - 7) + (3x4 - 7). •£ (cot (x2 + 1)) - ¿ (5) = (cot(x2 + 1)). (12x3) + (3x4 - 7). (csc2(X2 + 1)). £ + 1) - 0 = (cotíx2 + 1)). (12x3) + (3x4 - 7). (- cscV + 1)). (2x) Ordenando: = IZx3. cotíx2 +1) - 2x (3x4 - 7). csc2 (x2 + 1) dx 71 62 .- Tenemos: y = sec(2x + tan x) - ~ dy d d f’i'l => ~T = sec(2x + tan x). tan (2x + tan x). — (2x + tan x) - — "Z dx ' dx dx V 8) => = sec(2x + tan x). tan (2x + tan x). (j^(2x)+^(tanx)j - 0 dy 2 , = sec(2x+tan x). tan(2x+tan x). (2 + sec x) dx 63 .- Tenemos: y = csc (sen x - x2) + 1 dy d 2 d =» di = di («císenx-x2»^. ^(!) ¿iy 2 2 2 => — = - csc(sen x - x ). cot(sen x - x ). — (sen x - x ) + 0 R22 7381 Problemas de Trigonometría y cómo resolverlos
dtJ 2 2 — = -csc(sen x- x ). cot(sen x - x ). ( cos x - 2x) ax 64 .- Tenemos que: f(x) = a cos x + b / ttA í rtA 1 Pero: /I 3 I = 2 a cos I — I = ~ r. d d Ahora: /(x) = — (a cos x) + (b) 3 n 1 Per°: = 2 * Sen 6 = 2 Reemplazando (2) en (1): = > a. j + b = 2...(l) => f = a sen x J (X> => a=-3...(2) (1 I -31 2 I +b = 2 7 b‘2 ab = - 21 2 2 65.- Tenemos:/(x) = sen x - —, x G { 0; 7t) Derivando respecto a "x" e igualando a cero: d 1 d f’M = 2senx. ^(senx)-- —(x) = 0 1 => 2 sen x. cos x - — = 0 1 Pero: 2 sen x . cos x = sen 2x => sen 2x = ~ 71 5n 2x = — r\ 2x = — 6 6 {n _5ti1 12'12 J Limites y Derivadas Trigonométricas R22 739
DERIVADA DE F. T. INVERSA 66.- Tenemos: n --------arcsenx y= 21 > n Pero: ~ - arc sen x = arc cos x => u = 2 3 arccosx Usando la derivada de un cociente: j Jlx2 .—(are cosx) - (arc cosx).—Vi—x2 ay__________dx________________dx dx ~ (Vl-x2)2 -l-(flrccosxU i1 . -(^) dy =Wl-x2 dx 1-x2 Simplificando: + x-«rccosx (1 67.- Tenemos: „ x (1 y = arc sen (ín x) + arc sen I — => = 7~ ~ (Znx)+y- íflrcsen(l)) d* Jl-(lnxf dx dx \ \2IJ dy = 1 1+0 ¿y = 1 dx ^-(Inxf ' x + • dx 68.- Tenemos: y — cos (arc sen Jx ) - 1 dy f- d r~ d => ~ = - sen (arc sen Jx). — (arc sen Jx ) - — (1) dx 'dx 'dx RACSO ^p.n.TOB.. R22 7401 Problemas de Trigonometría y cómo resolverlos
— = - sen (are sen -Jx ).-¡=-^-(-Jx)-O dx l-(VJt) dx dy _ 1 1 d — = - sen (arc sen -Jx ).-----. o r~ . — (x) dx ' ' 1-x 2-Jx dx ' Evaluando y usando propiedades de funciones trigonométricas inversas: p/ 1 1 - dx 1-x 2xx dy -1 dx 1-x 69.- Tenemos: y = arccos ( v 1 - x2 ) + ~ dy -1 d i-------- d fn'l =* dx l-fJl-x2)2 dx^ 1 X )+ dx [sj dy -1 -1 d 2 dx ~ l-(Vl-x2) 2Ó-x2 ' dx (1'X) + ° dx ¿ \Jl-x2 dx x.yll-x2 arc tan (3) d )- — (are tan 3) dx 70.- Tenemos: y = arc cos Jx - dy d => — = — (are cos -Jx ' dx dx dy = -1 1 dx Jl-x 2jx dy = dx ljx-x2 71.- Tenemos: y = arc cos (ex) + arc cos Vi—f2x dy -1 d x -1 d i-----57 Límites y Derivadas Trigonométricas R22 741
I dy -1 X , 1 d dx ~ Jl-e2x e + 71-(1-e2x) ' 2Ó-e2x ' dx^~e > dy ~e* -1 \ 7v -ex i \Y ~d* = Jl-e2x +\fXJl-e2x '} = +XJl-e2* (^ dy = ~eK + gx . = Q dx Vl-e2x + Vl c2x " dx 72.- Tenemos: y = 2aresen3x + are sec j => _ _É_ ^2arcsen3x) + — íflrcsec(^)) dx dxK ’ dx \ \5ll Sea: K = 2arcsen3x => ln u = ln (2arcsen3x) = (are sen 3x) ln 2 Derivando ambos miembros respecto a "x" 1 du -1 du , u — . — = ln 2. । = => — =ln2. ¡ U dx Vl-(3*)2 dx VI-9x2 H (ln2)2arcse”3x Pero: U = 2arcsen3x => — (2^sen3x) = --- (w) dx V1-9X2 dy (/M2).2arcsen3x Reemplazando (**) en (*): y 73 .- Tenemos: y = arc tan(2x - 5) - arc csc (4) dy d d => — = ~ (arc tan (2x - 5)) - — (arc csc 4) dxdx dx dy 1 d dy 1 dx l+(2r —5)z dxK ’ dx 4x2-20x + 26 Simplificando: ¿y _ 1 dx 2x2-10x+13 Problemas de Trigonometría y cómo resolverlos Mracso ^DITOK»
74 .- Tenemos: y = arc tan ( V 4x2 — 1 ) dy = 1 dx l+(V4x2-!)2 .^-(^4x2-l) dx dy _ 1 1 dx l+(4x2-1) ' 2y¡4x2-l d 2 -(4?-i) dy _ 1 d* x.iMx2 — 1 75.- Tenemos: y = arc tan (3 sen x) - cos ~ . dy 1 d „ d { n\ => — =-------------. — (3senx)- — Icos^l dx l+(3senx)2 dx dx \ 7 / dy 1 dy 3cosx — =----------5— . 3 cos x - 0 .'. -7- = -— ----5— dx l + 9sen x dx l + 9sen¿x 76 .- Tenemos: y = - tan - arccosx j + tan — Por propiedad de funciones trigonométricas inversas: 3n y = - tan (arc sen x) + tan ~ dy j d d / o_\ => — - - sec (arc sen x). — (arc sen x) + — I tan^ I dx dx dx \ 5 I 2 * — = - sec (arc sen x). . + 0 dx Por identidad trigonométrica: dy _ -1 1 dx eos2 (arcsenx) ' Ji-x? dy = -1 1 dx 1-sen2 (arcsenx) dy -1 dx (l-x2).Vl-x2 Límites y Derivadas Trigonométricas
77 .- Tenemos: y = arc cot (\2x2 — 1 ) - — O - -------------------- dx l+(V2x2-!)2 dx dx ' dy = -1 1 dx X+(2x2 ’ 27x2-1 dy -i 1 v dx =s2^2 '^Ix2-! d 2 ,-(2x2-1)-0 dy 1 dx x.^2x2—1 78 .- Tenemos: f(x) = x . arc sen x Derivando/(x) respecto a “x" usando la derivada de un producto obtenemos: d d x > /'(X) = x • (arc sen *> + <flrc sen x> ~dx + (arc sen x). 1 DERIVACIÓN IMPLICITA 79 .- Procedemos a derivar la expresión dada, y obtenemos: d d d dy — (seny)+ — (y) - — (l) = 0 , —: derivada implícita dy dy (cos y) ~ + — - 0 = 0 3 dx dx dy dy Factorizando ~: (1 + cos y). ~ = 0 dx ' 3 dx Como: y * (2k + l)n, k e Z => 1 + cos y * 0 dy dx = 0 R22 Problemas de Trigonometría y cómo resolverlos RACSO JDITOtlI
J d 80 .- Derivando implícitamente tenemos: 2 sen y . — (sen y) - 2x = 0 2 sen y. cos y . (y') - 2x = 0 , y': derivada implícita Recordando que: 2 sen y . cos y = sen 2y 2x y ~ sen2y 81 .- Tenemos: y . sen x - x cos y + 1 = 0 Derivando implícitamente y usando la derivada de un producto, tendremos: dy d d d d => (senx). — + y. — (senx)-x. — (cosy)- (cosy). ~ (x) + ^(l) = 0 dy dy (senx). — + y. cosx-x(-sen y). — - (cos y) . (1) + 0 = 0 dy Factorizando: — (sen x + x sen y)+ y cos x - cos y — 0 Despejando tenemos: dy cosy-y osx dx ~ senx + xseny 82 .- Derivando implícitamente: d dy d =* V' (scrlx) * (senx). — + — (cosy) = 0 dy dy y. cosx + (senx). — +(-seny). — =0 Factorizando: dy — .(sen x - sen y) + y. cos x = 0 Despejando tenemos: dy _ -yxosx dx ~ senx-seny 83 .- Tenemos: y = cos (xy) + y2 Derivando implícitamente y usando la derivada de un producto: dy d dy — = - sen (xy). ~ (xy) + 2y. ~ dx K dxv s dx Límites y Derivadas Trigonométricas
dy dy d dy — = - sen (xy). (x . — + y . — (x)) + 2y. — dx dx 3 dx 3 dx dy dy dy — = - x. sen (xy). ~ - y- sen (xy) + 2y. — dx 3 dx 3 ' 3 3 dx dy Factorizando ~: dx —. (1 + x sen (xy) - 2y) = - y sen (xy) Despejando: dy ~yjsen(xy) ~dx = l + xsen(xy)-2y d 2 d dy 84.- Derivando implícitamente: — (cos (x )) - — (sen y) + — = 0 , d 7 dy dy - sen (x2) - 3- (x2) - (cos y). — + ~ = 0 dx dx dx > ty dy - sen (x ). (2x) - (cosy). ~ = 0 2 dy Efectuando y factorizando: - 2x. sen (x ) + (1 - cos y) ~ =0 Despejando: dy 2x-sen(x2) dx ~ 1+cosy 85.- Derivando implícitamente: d d d -(tan(x + y))+-(cosy)-^(l) = 0 2 d dy sec (x+y). —(x + y)-(seny). — -0 = 0 2 sec (x + y) . dy] dy ' + di -(seny)-o 2 2 dy dy sec (x +y) + sec (x + y). — - (seny). — = 0 Factorizando: 22 dy sec (x + y) + [sec (x + y) - sen y]. — = 0 RACSO P BOITO**! 746 Problemas de Trigonometría y cómo resolverlos
Despejando: dy eos2 (x + y) dx ~ eos2(x+y)-seny 86.- Derivando implícitamente: d d 7 d =» -(sec(x + y))--(y)+-(l) = 0 d dy sec (x + y) . tan (x + y) . —(x + y)-2y. — =0 sec(x + y). tan(x + y) . 1 + ^| -2y. dy ~ =o dx dy Factorizando: (sec (x + y). tan (x + y) + 2y) — = - sec (x + y) . tan (x + y) dy -sec(x + y).tan(x+y) dx ' sec(x + y).tan(x+y)-2y 87.- Derivando implícitamente: d d , (tan xy) - — (y3) = 0 9 d i dy sec (xy) - — (xy) - 3y2. — = 0 2 dy j dy sec (xy).(x. — + y)-3y2. — =0 ? dy 2 2 dy X. sec (xy). — +y . sec (xy) -3/. — = 0 Factorizando: (x. sec2 (xy) - 3y2). + y . sec2(xy) = 0 Despejando: dy -yse<?(xy) dx xse^(xy)-3y2 88.- Derivando implícitamente: d d -1 1 dy — (arc cos x) - — ( Jy ) = 0 => . — dx ' dx ' Jl-X2 2v dx Despejando obtenemos: dy -2y Límites y Derivadas Trigonométricos
89 .- Derivando implícitamente: d d d — 1 dy dx' dx dx Jl~y2 dx dy i—2 Despejando obtenemos: = - y 1 - y 90 .- Derivando implícitamente: d d -1 d d - (arc sen (xy)) + - (yx) = 0 =* .-(xy)+- ^ = 0 Factorizando: *0 d d dx^X) = 0 REGLA DEL’HOSPITAL x — sen(nx) 91 .- Sea: W =----- x + sen(nx) 0 Al evaluar para x = 0, tenemos: lím W = — => forma indeterminada r x—»o 0 Por lo tanto el límite se determinará aplicando la regla de ¿'Hospital, que consiste en derivar el numerador y denominador hasta levantar la indeterminación, es decir, hasta que al evaluar en el límite la expresión posea una valor determinado. Veamos: d La primera derivada del numerador: — (x - sen (nx)) = 1 - n . cos (nx) La primera derivada del denominador: — (x + sen (nx)) = 1 + n . cos (nx) Evaluando el límite: lím W = lím x-»0 x->0 1—Jtcos(nx) 1 + Jtcos(nx) 1 -ncos(0) i^b W = 1 + ncos(O) lím W = x->0 1-Jt 1 + Jt Problemas de Trigonometría y cómo resolverlos fjA RACSO UioiroiEt
Tt O 92.- Al evaluar para 6 = —, resulta — forma indeterminada aplicamos ¿'Hospital. lím o 71 2 sen(2cos6) 2 cos 6 d[sen(2cos0)] flzSsO] dx => lím sen(2cos6) 2 cos 6 cos(2cos0).f^2sert0T~" lím , TT 2 sen(2cos0) 2cos0 cos ^2cos^j lím sen(2cos0) 2cos0 93.- Sea: W = 5x2 -2x sen3x 2 Al evaluar para x = 0, tenemos: lím^ VV = — => forma indeterminada entonces aplicamos ¿'Hospital La primera derivada del numerador: La primera derivada del denominador: d 2 — (5x2 - 2x) = lOx - 2 — (sen 3x) = 3 . cos 3x El límite será: lím VV = lím x-»0 x-»0 10x-2 _ —2 3cos3x 3cos(0) lím VV = - x—>0 2 3 1—senx-cosx 94.- Sea: W = ------------ 1 +senx-cosx Al evaluar para x = 0, resulta: ~ ; forma indeterminada entonces aplicamos ¿'Hospital. lím VV = lím x-»0 x->0 —(1 - senx - cosx) dx —(1 + senx-cosx) dx lím VV = lím x—tO x—>0 - cos x +senx cosx + senx -1 1 lím VV = - 1 x-»0 sen(l-x2) 95.- Sea: VV =--3---- x+1 Límites y Derivadas Trigonométricos
Al evaluar para x = - 1, tenemos: sen(O) 0 VV = —= q" * forma indeterminada entonces aplicamos las reglas de L Hospital. Determinamos la primera derivada del numerador: d 7 7 ~¡¿ (sen (1 - x)) = cos(l - x ). [2x] La primera derivada del denominador: y-(? + 1) = 3x2 dx El límite será: lím VV = x-»0 9 —2*.cos(l—* ) 3*2 —2cos(l -x2) -2cos(0) ZíJfí 9 - Q/ \ r->-l 3* 3(-1) lím W = x—>0 96.- Sea: W = 3x-2sen4x 6x—7sen5x 2 3 0 Al evaluar para x = 0 , resulta — forma indeterminada entonces aplicamos L' Hospital lím VV = lím x—>0 x->0 —(3x - 2sen4x) dx —(fix - 7sen5x) dx taz t' 3-8cos4x hm W = hm -----—----— x—>0 j x—»o 6-35cos5x lím VV = x—>0 3-8 6-35 lím VV = — x—>o 29 62-16 97.- Sea: VI = ~—77----7^7 (6+4).sen(ii6) Al evaluar para 6 = 4 tenemos: VV = — ; forma indeterminada entonces aplicamos L'Hospital d 2 La primera derivada del numerador: — (6 - 16) = 26 La primera derivada del denominador (para un producto). d r 1 d — [(6 + 4) . sen (716)] = ^(6+4)J . sen (716) + (6 + 4). — sen (7t6) R22 750 Problemas de Trigonometría y cómo resolverlos -^4 RACSO
— [(6 + 4) sen (tiO)] = sen(7t0) + (6 + 4) ti cos (tiO) e->4 o->4 sen(7ie) + 7i(e+4).cos(7te) sen4n + 8n.cos4n 8n lím VV = — e->4 x2 1-^2 98.- Sea: W =--- senx 0 Al evaluar para x = n, resulta ~ forma indeterminada entonces aplicamos ¿/Hospital lím W = lím x—>n —í-¡ dx TI2 d~ , —(senx) dx lím VV = —^-.2x 7t cosx lím W = —%-------- X-Mt COS 71 lím W=-^- X—>7t — 1 lím = — x—m Jl senx-sen2x 99.- Sea: W =----------- sen3x Al evaluar para x = 0, tenemos: 0 VV = — => forma indeterminada entonces aplicamos L'Hospital d La primera derivada del numerador: — (sen x - sen 2x) = cos x - 2 cos 2x La primera derivada del denominador: d — (sen 3x) = 3 cos 3x El límite dará: cosx-2cos2x lím W = lím ----------- x-»o x->o 3cos3x 1-2 lím x->0 3 1 3 .2 100.- Sea: W = senl + 4-2) Límites y Derivadas Trigonométricos
Al evaluar para x = O, tenemos: o o sen(2-2) ~ o ; forma indeterminada entonces aplicamos las reglas de ¿'Hospital. Hallemos la primera derivada del numerador: d 2 -j- (xz) = 2x dx' La primera derivada del denominador: — [sen (v * + 4 -2)] =-» - . 2r dx 2Vxt4 [sen ( Vx2 +4 -2)] = dx x.cos(Vx +4 -2) Vx2+4 101.- Sea: W = El límite será: lím W = lím x-»0 x-»0 > lím W = lím x-»0 x->0 2x lím VV = 4 x-»O Vi + senx-Vi-senx x Al evaluar para x = 0 tenemos: 1-1 0 = 2. lím x->0 Vx2+4 cos(Vx2 + 4 — 2) 2 lím W = 2.--------- x—»o cos(U) ; forma indeterminada entonces aplicamos las reglas de ¿'Hospital. Determinamos la primera derivada del numerador. d ,________ ,________ cosx -cosx ~ (Vi + senx - Vi-senx) = - c---= - - g----= dx 2vl + senx 2vl— senx La primera derivada del denominador: d -T- (X) = 1 dx V El límite será: lím \N = lím x->0 x—>0 COSX 2 1 । 1 Vi + senx Vi-senx W = W = o o RACSO Pbditobbb 752 Problemas de Trigonometría y cómo resolverlos
lím W = x—»0 - -1 1 + 1J lím VV = 1 x—>0 102.- Sea: W = n-3x 1 -2cosx Tt 0 Al evaluar para x = —, resulta ~; forma indeterminada entonces aplicamos L'Hospital d , „ , —(n-3x) dx________ lím W = lím d ... „ . r_>’E — (1-2 cosx) 3 3 dx lím W = lím -3 - 2.(-senx) lím W = 3 -3 „ Tt 2sen— 3 lím W = -3 lím VV= - ^3 y — 103.- Sea; 2 o Al evaluar para x = 1, resulta —; forma indeterminada entonces aplicamos L'Hospital lím W = lím x->0 x-»0 lím W =¡ n x—>0 „ 1-cos8x 104.- Sea: W = sengx , 0 Al evaluar para x = 0, resulta ~ ; forma indeterminada entonces aplicamos L'Hospital. lím W = lím x—>0 x-»0 —(l-cos8x) dx —(sen8x) dx lím W= lím x-»0 x—>0 8sen8x 8(0) cos8x ~ 1 lím VV = 0 x »0 Límites y Derivadas Trigonométricos
X 105.- Sea: W = TLtan^J Al evaluar para x = 0, resulta^; forma indeterminada entonces aplicamos L Hospital. 2 lím VV =—7 x-X) 71 106.- Sea: W = 1 - tanx senx-cosx 7t 0 Al evaluar para x = — ,resulta — forma indeterminada entonces aplicamos L' Hospital. -7-(1 —tanx) /¿Y lím W= lím ------------- —(senx-cosx) dx lím W = -sec2x lím ----------- cosx + senx 4 2 71 -sec — 4 cos—+ sen— 4 4 lím W = -2 lím W= - 72 Tf 107.- Sea: W = cos2x + cosx tan2x+ tanx Al evaluar para x - 0, resulta — forma indeterminada entonces aplicamos L'Hospital lím W = lím v v— -r_ 3 d . . —(cos2x + cosx) dx —(tan2x + tanx) dx —2sen2x—senx lím W = lím ~ 277 277 r n rn 2.sec 2x + sec 2x ~*3 3 R22 lím W = 3 „ 271 71 —2sen-----sen— ________3 3 „ 2„7t 71 2sec 2—+ sec— 3 3 l 2 2 12 7541 Problemas de Trigonometría y cómo resolverlos RACSO ^DltOKBS
tan(a*) 108.- Sea: W = [i+x_cos(flx)][sec(flx)] —J3 lím Vf =—~— r .n 8 3 Al evaluar para x = 0, resulta ~ forma indeterminada entonces aplicamos L'Hospital lím W = lím x—>0 x-»0 d , , „ —(tan(o*)) dx —[1 + x - cos(flx)][sec(flx)] dx lím Vi = lím x-»0 x—>0 _________________________fl.sec2(«*))________________________ (1 +flíen(a*))(sec(«*))+[l + *—cos(fl*)].fl.sec(«*).tan(«*) lím W =--------------- x—>0 (l + 0)(l) + 0 lím W = a x-»0 109.- Sea: arcsen5x atetan* Al evaluar para x = 0, resulta — forma indeterminada entonces aplicamos L'Hospital lím W = lím x—>0 x->0 —(orcsen5*) dx —(ore tan*) dx lím VV = lím x—>0 x->0 1 .5 Jl-(5x)2 1 lím W= 5 »->o 110.- Sea: 2 tan*—ore sen* W =--------------- sen* 1 + *2 Al evaluar para * = 0, resulta 0 0 forma indeterminada entonces aplicamos L'Hospital lím VI = lím x—>0 x—>0 —(2 tan*—aresenx) dx - 2sec *— 1 d , —(sen*) dx lím W = lím x—>0 x—»0 cos* lím VV = x—>0 2-1 1 lím VV = 1 x-»0 Límites y Derivadas Trigonométricas R22 755
111.- Sea: Tt—2arccosx W =------------- Al evaluar para x = O, resulta ~ forma indeterminada entonces aplicamos ¿'Hospital lím W = lím x-»0 x->0 —(ti - 2arccosx) dx d . X dx lím W = lím x-»0 x-»0 112.- Sea: 7 7 sen x-sen(r) W=-------j lím VV = 2 x-v» Al evaluar para x = 0, resulta — => forma indeterminada entonces aplicamos ¿'Hospital: —[sen2x - sen(x2)] lím W = lím —--------j----------- x-»0 x->0 a (y2) dx lím W = lím x->0 x-»0 2senx.cosx —2x.cos(x2) 0 2x ~ 0 Como el resultado es indeterminado, aplicamos ¿'Hospital por segunda vez: y-[2senx.cosx - 2x.cos(x2)] > lím W = lím „ 2cos2x-2[cos(x2)2x2sen(x2)] = hm ---------------------------- dx 2-2 > lím W =------ x->0 2 lím VV— 0 „ ... 1--Jcosx 113.- Sea: W = ----5--- 0 Al resolver para x = 0, resulta — => forma indeterminada entonces aplicamos ¿'Hospital: lím W = lím x-»0 x->0 ——(1 —i/cosx) dx y-(x2) = lím x—»0 1 —i - (~senx) 2-vcosx senx u ---------------= hm -—. ~ 2x x-»o 4xVcosx 0 0 2 Como el resultado es indeterminado, aplicamos ¿'Hospital por segunda vez: Problemas de Trigonometría y cómo resolverlos 3» RACSO
lím W = lím x-»0 x—>0 d —(1-senx) dx_________ 4.—(x.Vcosx) dx lím Ví = lím x-»0 x->0 COSX 4 Vcosx----.X .i 2 V cosx lím W= A—>0 2cosx.%/cosx 2 lím "7777------------7 = 77 x—»o 4(2 cosx+ xsenx) 8 lím VV = x-»0 2 4 r2 — 2x +1 114- Sea: W=1T^(4 o Al evaluar para x = 1, resulta ~; forma indeterminada entonces aplicamos L'Hospital: -^-(x2—2x + l) => lím W = lím ------------------ x->1 —(1 + cos(tix)) 2x-2 0 => lím W = lím ----------;—7 = ~ x-»o x-»i -7t.sen(7tx) 0 Como el resultado es indeterminado, aplicamos L'Hospital por segunda vez: -^(2x-2) lím W = lím ,ax---------- X—>0 X->1 « r z —l~-7Lsen(7tx)] dx 2 lím W = lím -------5--------- x—>0 x-»l -7t -COS(TTX) 2 lím VV= 73" 2 lím W = —5----------- x—>0 —71 .COSTt 115.- Sea: W = cos(wx) - cos(nx) Al evaluar para x = 0, resulta 0 0 forma indeterminada entonces aplicamos L'Hospital lím W = lím x-»0 x—>0 —(cos(mx)- cos(nx)] dx ~(*2) dx lím VI = lím x-H) x->0 -msen(mx) + nsen(nx) 0 2x =0 Como e] resultado es indeterminado, aplicamos L'Hospital por segunda vez: =$lím W =lím x-X) x-M) —[-msen(mx) - nsen(nx)] dx t-(2x) dx 2 2 „ ,, -m cos(mx)+n .sen(nx) hm W = hm ------------------------ x->0 x->0 2 Límites y Derivadas Trigonométricos 1*3757
lím W = x—>0 116.- Sea: W = l-cos4x 1 -cos3* Al evaluar para x = 0, resulta — => forma indeterminada entonces aplicamos L'Hospital. —(1 - cos4x) lím W = lím ----------- x^O x-,0 A(i_cos3x) dx lím W = lím x-»0 x^O 4sen4x 3sen3* 0 0 Como el resultado es indeterminado, aplicamos L'Hospital por segunda vez: lím W = lím x->0 x->0 —(4sen4x) dx —(3sen3*) dx lím W = lím x->0 x-»0 lóeos4* 9cos3* 16 Zím = “ x-»o 9 117.- Sea: W = 2 _____COS*-eos X____ 7 sen2*+2sen *-2sen* Al evaluar para x = 0, resulta ~ => forma indeterminada entonces aplicamos L'Hospital (cos*-cos *) -senx+2cos*.senx 0 > lím W = lím ~j----------,--------= lím ~ ~ . z = ~ -o ^(sen2x+2senx-2senx) »-»o 2cos2x + 4senxcosx-2cos* 0 Como el resultado es indeterminado, aplicamos L'Hospital por segunda vez: -cos x+2[-senx.senx + cosxcosx] x”xo -4sen2x + 4[cos*.cos*-senr.senx] -1 + 2[1] 2 4 S>w = ni] lím W = x->0 118.- Sea: -J3 - V2 + cos* sen 3* 0 Al evaluar para x = 0, resulta ~ forma indeterminada entonces aplicamos L'Hospital. R22 758 Problemas de Trigonometría y cómo resolverlos RACSO pBOITOKBg
lím W — lím x->0 x-»0 —(i/3 --J2 + cosx) dx__________ —(sen* 23x) dx (-senx) 2^2 +cosx hm W = hm —-------„ ---—— x—»o x—»o (2sen3x)(3cos3x) lím W = lím x—>0 x-»0 ______senx_______ o 6sen6x.-j2 + cosx - 0 Como el resultado es indeterminado, aplicamos ¿'Hospital por segunda vez: lím W = lím x—>0 x-»0 cosx 6.cos6x.V2 + cosx + senóx. (-senx) 2^2 +cosx 1 ^oW = 36j3 lím W= — x-íO 108 6 x.sen(sen2x) $ea: W= 1—cos(sen4x) 0 Al evaluar para x = 0, resulta —; forma indeterminada entonces aplicamos ¿'Hospital. lím W lím x—>0 x-»0 —(x sen(sen2x)) dx —(1 -cos(sen4x)) dx lím W = lím x-»0 x->0 sen(sen2x)+x.cos(sen2x).2cos2x + sen(sen4x).4cos4x 0 lím W = — x—»o 0 => forma indeterminada Como el resultado es indeterminado, aplicamos ¿'Hospital por segunda vez: cos(sen 2x) + 2 cos 2x + 2¡cos 2x. cos(sen 2x) + x{-2 sen 2x. cos(sen 2x) - 2 cos 2x.sen(sen 2x)} 4[cos(sen 4x).4cos4x. cos4x - sen(sen 4x). 4sen 4x] 2 + 2(1] 4 lím W = = — x-»o 41*1 16 lím VV= ~ x-»o 4 120 - Sea: W = 1-^cosx x.senx Límites y Derivadas Trigonométricos
0 Al evaluar para x = 0, resulta — => forma indeterminada entonces aplicamos L'Hospital: —(1-^cosx) -—(cosx) 2/3.(-senx) => lím W=lím ^3--------------= lím —3------------------ x-»o x->o _(x.senx) x->0 (l)senx +x.cosx dx Como el resultado es indeterminado, aplicamos L'Hospital por segunda vez: C5/3.(senx)(senx) + (cosx)_2/3.(cosx) cosx + cos(senx) - xsenx => lím W = lím x->0 x->0 -i[+|(cosx) => lím W = 2 3. 2 1 lím VV = ~ x-»0 O 0 = ~ => indeterminada . 3 1-COS x 121.- Sea: W =------2— tan x 0 Al evaluar para x = 0, resulta — forma indeterminada entonces aplicamos L Hospital d .. 3 . dx' COS * (-3cos2 xX-senx) 0 => lím VV = lím —j------------ => lím VV = lím ---------- 2 x = ñ x-»o x->o a (tan2x) Jr^iü x->0 (2tanx)(sec x) 0 dx Como el resultado es indeterminado, aplicamos L'Hospital por segunda vez: 2 3[2cosx(senx).(senx) + cos x.cosx] => lím W = lím ——5--------2---------~--------------- x->o x-*o 2[sec x.sec x+tanx.2secx.secx.tanx 3[0 + l] => lím = or 1 j. ni z-»o 41 + OJ 122.- Sea: W = -&&&• x.tan4x 3 lún = 1 x-tO 2 0 Al evaluar para x = 0, resulta ~ => forma indeterminada entonces aplicamos L'Hospital. —(3sen2x) => lím W = lím ---------- x-»0 x-»0 _±(xtan4x) dx 6.senx.cosx lím W = lím ~ ~ ~ TT~ x-to x-»o tan4x + x.4sec 4x R22 76Q | Problemas de Trigonometría y cómo resolverlos RACSO
3sen2x lím VV = lím ~ ~ TT~ x-»o x—»o tan4x + 4x.sec 4x O lím VV = — x-»o O Como el resultado es indeterminado, aplicamos L'Hospital por segunda vez: ________________3.(2cos2x)________________ 3(2) 4.se<?4x+4{sec?4x+x.2sec4x.sec4x.tan4x.4] - 4 + 4(1) 6 lím VV = ~ x—>0 o 3 lím = T x->o 4 123.- Sea: VV = 2 sec x-2tanx l + cos4x Al evaluar para x = 0, resulta — forma indeterminada entonces aplicamos L'Hospital. d 2 —(sec x—2tanx) 2 2 , , , , „ dx 2[2secx.secx.tanx.(tanx-l)+sec x.sec x] lim VV = hm---j-------------=---------------—--------------------- = x-xj —(l+cos4x) -16.cos4x dx 8 lím W = — n 16 4 1 lím VV = - 2 4 tan(l + cosx) ^2^‘" $ea; ~ cos(tanx)-! Al evaluar para x = n , resulta por primera vez: 0 — forma indeterminada entonces aplicamos L'Hospital 2 sec (1 + cosx).(-senx) 0 lím VV = lím r 2 „ = 77 x-wt x-wt [-sen(tanx)].sec x 0 => lím VV = lím —(tan(l+cosx)) dx —(cos(tanx)-1) dx Como el resultado es indeterminado, aplicamos L'Hospital por segunda vez: lím W = lím X—X—»7t -2sec(l + cosx).sec(l + cosx). tan(l + cosx).sen2 x + sec2 (1 + cosx).cosx cos(tan x).sec2 x.sec 2x + sen(tan x).2 secx. secx. tan x -1 lím VV = — x—KI 1 lím VV = -1 x-»n Límites y Derivadas Trigonométricos
(x-x3)2 125. Sea. W- ^sen3x^sen5xj Al evaluar para x = O, tenemos: las reglas de L'Hospital 0 o ; forma indeterminada entonces aplicamos W = Hallemos la primera derivada del numerador £ (x - x3)2 = 2(x - x3). (1 - 3x2) La primera derivada del denominador (para un producto). [(sen 3x). (sen 5x)] = |j^(sen3x)]. sen 5x + sen 3x [¿(sen5x)] = 3 cos 3x . sen 5x + sen 3x . 5 cos x 2(x-x)(1-3x) 0 El límite será: lím VV = lím -------EZTE----Z--EZ = 77 x-»o x->o 3cos3x.sen5x + 5senx.cos5x 0 Por que aparece nuevamente la forma indeterminada, aplicamos por 2da vez L' Hospital. Derivamos el numerador (para un producto). ¿ 2(x - x3) (1 - 3x2) = 2(x- x3)](l - 3x2) + (x + x3).^(1 - 3x2)] = (1 - 3x2). (1 - 3x2) +(x - x3)(- 6x) = 2(1 - 3x2)2 - 12x(x - x3) Derivamos el denominador (para un producto): d — [3 cos 3x - sen 5x + 5 sen 3x . cos 3x1 dx = 3;(cos3x) j sen 5x + 3cos 3x. (sen 5x) +5 eos 5x + 5 sen 3x. — (cos 5x) = - 9 sen 3x . sen 5x + 15 cos 3x . cos 5x + 15 cos 3x cos 5x - 25 sen 3x sen 5x = 30 cos 3x . cos 5x - 3 sen 3x sen 5x El límite será: r r 2(l-3x2)2-12x.(x-x3) 2(1)2 hm W =hm —----------------——----------— = onz-ixz-ix x->o x-»o 30cos3x.cos5x-34sen3xsen5x ^0(l)(l) 1 lím VV = ~ x->o 15 ARACSO Deditokii R22 762 Problemas de Trigonometría y cómo resolverlos
126.- Sea- W = 7 2cosx-2 + x 3x4 O Al evaluar para x = O, resulta ~; forma indeterminada entonces aplicamos L'Hospital. lím W = lím x->0 x—>0 d „ 2\ —(2 cosx -2 + x) dx_______________ y-(3x4) dx lím W = lím x—>0 x—>0 -2senx + 2x 12X3 0 0 Por que aparece nuevamente la forma indeterminada, aplicamos por 2da vez L' Hospital. —2cosx + 2 0 => lím W = lím 2 = 77 x-»o x-»o 36x 0 Por que aparece nuevamente la forma indeterminada, aplicamos por 3ra vez L' Hospital 2senx 2 senx 1 1 lím VV = lím —— => lím W = — . lím -------------- = — (1) lím - ~ x-»o x-»o 72x x-»o 72 x—»o x 36 x-»o 36 127.- Sea: W = X tan* x-senx 0 Al evaluar para x = 0, resulta “; forma indeterminada entonces aplicamos L'Hospital. lím W = lím x-fO x-»0 —(x-tanx) dx_______ d , —(x—senx) dx 1 — sec2 x 0 => lím W = lím-----------= 77 x—>0 X—>0 1 —COSX 0 Por que aparece nuevamente la forma indeterminada , aplicamos por 2da vez L' Hospital. lím W = lím x—>0 x->ü —(l-sec2x) dx —(1-cosx) dx lím W = lím x—>0 x—>0 -2secx-secx. tanx 0 + senx 0 Por que aparece nuevamente la forma indeterminada , aplicamos por 3ra vez L' Hospital „ w , dx^ 2sec2*-tanx) -2[2secx.secx.tanx.tanx + sec2x.sec2x] lím W =hm ——t--------------= hm — ---------------------------------¿ x^o A(sem) —o cosx dx Límites y Derivadas Trigonométricos
-2[2sec2x.tan2x+sec4x] —2[0 + l] => hm W = hm --------------------------- =--------- x—>0 x->o cosx 1 lím W = - 2 ’ x-»0 128.- Sea: W = tan x-senx 0 Al evaluar para x = 0, resulta —; forma indeterminada entonces aplicamos L'Hospital. —(tanx-senx) dx lím W = lím x-fO x->0 d . 3. > dx sec2 x-cosx => hm W = lím ---------5----- x-xO x-»0 3x 0 0 Por que aparece nuevamente la forma indeterminada , aplicamos por 2da vez L' Hospital. 2secx.secx. tanx + secx lím W = lím ------------------------- x-»0 x-»0 OX , _ , , 2sec2x.tanx + secx 0 => hm W= hm -----------------------= 77 x-»o x—»o 6x U Por que aparece nuevamente la forma indeterminada , aplicamos por 3ra vez L' Hospital. , , 2[2secx.secx.tanx.tanx + sec2x.sec2x] ,, 2[0+l]+l => hm W = hm ------------------------------------------ hm W =-------------------- x—>0 x—>0 X x-»o O 1 lím = x-»0 1—cos( 1-cosx) 129 .- Sea: W =-----4------- 0 Al evaluar para x = 0, resulta —; forma indeterminada entonces aplicamos L'Hospital. lím W = lím X^O x-»0 —[1 -cos(l - cosx)] dx ^-(x4) dx sen(l -cosx)(senx) => lím W = lím ---------------------- x-»o x-»o 4x 0 0 Por que aparece nuevamente la forma indeterminada, aplicamos por 2da vez L' Hospital. lím W = lím x—>0 x-»0 —[sen(l - cosx)(senx)] dx 7“(4x3) dx Problemas de Trigonometría y cómo resolverlos ’at racso U JDITOB.BA
=> lím VV - lím cos(l ~ cosx)[+senx](+senx)+ sen(l - cosx),[cosx] _ £ x-»o x-»o 12x2 O Por que aparece nuevamente la forma indeterminada , aplicamos por 2da vez L' Hospital. , , , , +sen2x.cos(l - cosx)+ cosx.sen(l-cosx) 0 lim VV = hm --------------------2----------------= 77 x-»0 x-»0 12x 0 Por que aparece nuevamente la forma indeterminada , aplicamos por 3ra vez L' Hospital. +2senxcosx.cos(l - cosx) + sen 2x.{-sen(l - cosx)[senx]} + (-senx).sen(l - cosx) + l¡m cosx.cos(l-cosx)[senx] x->o 24% Aplicando la identidad del arco doble y reduciendo términos, tenemos: +sen2x.cos(l -cosx)+sen3x.sen(l - cosx)-senx.sen(l -cosx) + -isen2x.cos(l - cosx) lím -------------------------------------------------------------------------------- x-»o 24x ^sen2x.cos(l - cosx) + sen3 x.sen(l - cosx) - senx.sen(l -cosx) lím -í---------------------------------------------------------- x-»o 24x 0 0 Por que aparece nuevamente la forma indeterminada , aplicamos por 4ta vez L' Hospital. 3 cos 2x. cos( 1 - cos x) 1 sen 2x. sen( 1 - cos x).[sen x] + 3 sen2 x. eos x. sen( 1 - cos x) + sen3 x. cos(l - cosx)[senx] - cosx.sen(l - cosx) - sen x.cos(l - cosx)[sen x] • 24 lím VV = lím x-»0 x-»0 3cos2x 24 1 lím VV = - x-»o 8 130 .- Sea: VV = (1 + cos Tt)(l + cot Tt) Al evaluar para x = Tt, resulta: => lím VV= [1 + (-1)1(1 + °°) = 0.00 x-»0 => es una forma indeterminada 0 00 Nota: Para poder aplicar L'Hospital nos debe quedar una expresión de “ ó — Nuestra estartegia consistirá en transformar la expresión dada en otra sobre la cual se pueda aplicar L'Hospital, para ello recurrimos a las identidades trigonométricas. Límites y Derivadas Trigonométricos
lím W = lím (1 + cos x). X->0 X—>71 . COSX 1+----- senx lím W = Hm X-^0 X—>71 (1 + cosx)(senx + cosx) senx 0 Al evaluar para x = 7t, resulta —; forma indeterminada sobre la que sí podemos aplicar L'Hospital. lím W = lím X->0 X—>71 —-(1 + cosx)(senx + cosx) dx____________________ d / —(senx) dx lím W = lím X—>0 X—>71 (-senx)(senx + cosx) + (1 + cosx)(cosx - senx) cosx 0 -1 lím VV - 0 x—m 131 .- Sea W = 2x 3x e —e sen2x-sen3x 0 Al evaluar para x = 0, resulta —; forma indeterminada entonces aplicamos L'Hospital. lím W = lím x->0 x->0 rf . 2x 3xx —(e -e ) dx —(sen2x—sen3x) dx lím W = lím x->0 x->0 2.e2x-3.e3x _ 2-3 2cos2x-3cos3x 2-3 lím VV = 1 x->0 132- Sea W = 1- V1+ ^ Al evaluar para x = 0, resulta L'Hospital. 0 0 ' forma indeterminada entonces aplicamos R22 Problemas de Trigonometría y cómo resolverlos XjlRACSO
lím W = lím x—>0 x-»0 lím W = lím x-»0 x—>0 d , —(tanx) dx_________ —(1 -Vi + tanx) dx sec2x 2 sec x 2-J1 + tanx lím W = lím -2. Vi + tanx x-»0 x-»0 lím W = -2 i »o Tt 133.- Sea W = cot 2x . cot( - x) Al evaluar para x = 0, resulta °°. 0, forma indeterminada. Como hicimos en el ejercicio 0 oo anterior debemos modificar la expresión para que al evaluar resulte — o — y recién podamos aplicar L'Hospital. lím W = lím x—>0 x->0 tanx 0 tan2x 0 De acuerdo con este resultado, podemos aplicar L'Hospital: lím W = lím lím W = lím x-»0 x-»0 d , —(tanx) ax —(tan2x) dx 2 sec x 2. sec2 2x 1 x->0 2 134.- Sea W = (1 + cos x) 3sec x Al evaluar para x = —, resulta 1“, forma indeterminada. Lo que corresponde hacer es transformar la expresión dada en base a las siguientes reglas exponenciales: x = ln(b)x A (b)x = elnfcl = exinb Límites y Derivadas Trigonométricas
H(3secx)./n(l+cosx) lím W = lint (1 + cos x)3secx = ¿/“z (1) J 1 . o 1/1 . 3/n(l + cosx) o Evaluamso el limite del exponente: hm 3secx./n(l + cosx) = hm -------------- = ~ - „ .n cosx 0 2 2 Es una forma indeterminada que se puede levantar con L'Hospital j „ -senx , [3Zm(1 +cosx)] 3 ------- ax1 ,, 1 + cosx hm -----□---------= hm ------------ J—(COSXl x-x? ~sen* ax 2 1 2 Al reemplazar (2) en (1), tenemos que el límite es: = 31ím -------- = 3... (2) „ ,7t 1 + cosx 2 . lím W = e3 135.- Sea W = (1 + tan2x) c°‘2x Al evaluar para x = 0, resulta 1“, forma indeterminada. Luego procediendo como se hizo en el ejercicio anterior tendremos: lím W = lím (1 + tan2x) c°‘2x = x->() x->0 /«(l+3tan2x) 0 Evaluamos el límite del exponente: lím -----,---- = — x-»0 tan x 0 Es una forma indeterminada que se puede levantar con L'Hospital ^-[/»(1 + 3 tan2 x)] , J 2 -6 tan* “sec2 x lím ----------------- = lím ^tan2x---------------- x-xo f [tan2x/ x-><’ 2tanx.Sec x n-v1 -j^[ÍH(l + 3tan2x)] 3 ;------ = lím ; ' 2 = 3 ... (2) ^[tan2xj x-w l + 3tan x Al reemplazar (2) en (1), tenemos que el límite es: lím W = e3 Problemas de Trigonometría y cómo resolverlos RACSO WpBDITCBEa
APLICACIONES DE LA DERIVADA 136.- Tenemos: = sen2x - x - 1, x G ^0;^ Nuestra estrategia consistirá en evaluar la función a partir del criterio de la Ira y 2da derivada. Los ubicara en un máximo o mínimo y la segunda identificará al máximo. Derivando respecto a "x" e igualando a cero: d J3 d ¿3 f'M = 2 sen x. — (sen x)---— (x) - 0 => 2 sen x. cos x -- - 0 ' M dx' 2 dx 2 V3 xt Tt / ~\ Por la identidad del ángulo doble: sen2x = —- > 2x= ~ x= — g (0;—) 2 3 6 \ 4/ Derivando por 2da vez: = 2 cos 2x > 0, V x g ^0;^ Tt Por el criterio de la segunda derivada, concluimos que. fes mínimo en x = — 137 .- Nuestra estrategia consistirá en aplicar la ecuación de la recta tangente, así: Como: y = sen x + 2 a (%(,; y0) = (0; 2) => f\x)~ cosx => /'(0) = cos 0 = 1 Luego la ecuación de la recta tangente será: y-2 = l(x-0) y=x + 2 138 .- Empleado la misma estrategia del problema anterior, tendremos: Tt 1 y =fW = tan x - - => /'(x) = 1 + x2 => /'(1)=2 A = (!; °) Luego como en el problema anterior, la ecuación de la recta tangente será: 1 y-y0=f^-(*-*o) => y-o=-(x-i) 1 1 v = „ x- „ •’ 2 2 139 .- Para resolver este problema debemos recordar que la ecuación de la recta normal a una curva en el punto (Xq; y0) está dada por: 1 y-j/o= 7— (*-x0) - (*) -HxJ Tenemos: y = = 2 sen x - 3 a (Xq; y0) = (0; -3) Límites y Derivadas Trigonométricas
/'(1) = 2cosx => í'(0) = 2 Luego por (*) la ecuación de la recta tangente será: y-(~3) = ~ y (*-0) y = -~x-3 140 .- Procediendo como en el problema anterior, tendremos: 71 y = arc tan (* + (xo' Vo) = (°; °) (x) (x + l)2 + l (0)~ 2 Luego como en el problema anterior la ecuación de la recta normal será: 1 1 y-yo=-~7,— (*-•*<)) =» y-o = -Y (X-O) y = -2x ' (X|>) — 141 .- Nuestra estrategia consistirá en construir una expresión del área del sector circu- lar en función del radio. A partir de ello aplicaremos el criterio de la Ira derivada. Veamos: Por dato se sabe que: perímetro = 8 => 2R + 6R = 8 8-2R e= —...(i) Además se sabe que: OR2 S= —-..(2) ( 8-2R Reemplazando (1) en (2): S = I 2 Derivando respecto a R: S' =4 - 2R Igualando a cero: 4 - 2R = 0 dy 142 .- Dato: -37 = 9 dt dy Nos piden : — cuando x = 6 dt En el triángulo rectángulo se cumple: y2 = (30 + x)2 - 182 R22 770 Problemas de Trigonometría y cómo resolverlos RACSO DITOKIt
Derivando implícitamente respecto a "i": dy dx 2y. — = 2(30 + x). — , cuando: J dt di x = 6 r- dx Reemplazando los datos: 18 v3 . (9) = (30 + 6) ~ dx , dt ~ 2 m S 143 .- Sea S el área de la región rectangular: Por dato: S = 4 000 m2 b a S a ab = 4 000 4000 -d) Donde "L"es la longitud del material de la cerca: L = 2a + b... (2) 8000 Reemplazando (1) en (2): L = —~— L = (8 000)b 1 + b Derivando respecto a "b" e igualando a cero: dL , = - 8 000 b2 + 1 db 8000 b2 b2 = 8 000 => b = 40 J5 + 1 = 0 Reemplazando en (1): n- 4000 40^5 a = 20 V5 144 .- Sea x la distancia del observador a la estatua, nos piden “x" para que 0 sea máxi- ma. Del diagrama elaborado se puede apreciar que 6 tan(a + 0)= — tana+tanO 6 1-tan a. tan 6 x (1) Asimismo se verifica que: 2 tan a = — (2) 2 — + tanO Reemplazando (2) en (1): 2 1---.tanO 6 Límites y Derivadas Trigonométricas J+1771
4x Despejando tan 6: t an 6 = ^2 + 6 = arc tan 4x A x2+12 J Esta relación expresa al ángulo 6 que buscamos en función de x. Luego derivando res- pecto a “x" e gualando a cero tendremos: (x2 + 12).(4x)-4x(x2 +12) d6 _________(x2+ 12)2________ dx~ (x2 +12)2 + 16x2 (x2+12)2 de 4(x2 + 12)-4x(2x) _ dx= (x2+12)2+16x2 =0 x2 + 12 = 2x2 => X - ± 2J3 , pero: x > 0 0 es máxima si: x — 2j3 u 772 I Problemas de Trigonometría y cómo resolverlos ^RACSO ^DlTOkKS
PROBl€MflS D€ TRIGONOMETRÍA V COMO RESOLVERLOS
Sistemas de J_ Medida Angular ~ 11 I I I i MR / ÁNGULOTRIGONOMÉTRICO A)X-e+p B)o-x-p Q-e-x-p 01.- Del gráfico mostrado, indicar la veracidad o falsedad de las siguientes proposiciones: I. a y P son ángulos negativos D)X + 6 + P E)6-X+P 04.- De la figura calcular: M = ^20x+12y II. a + P es igual al ángulo de una vuelta III. P es mayor que a IV. 0 es negativo A)FFW B)FFFV QFVFV D)VFVF E)WW 02.- De la figura calcular el valor de x. A)P-6-a B)P+6 + a C)P+6-a D)6-a-P E)0 + a-P 03.- Determinar: M = 3y - 2x - 270“ en función de: 6, X y P, a partir de la figura adjunta. A) 9 B)10 C)ll D)12 E)13 05.-En el siguiente gráfico: OM es bisectriz del Z AOB; ON es bisectriz del Z COD. Ade- 40x más: 6y - 10.r= 150; calcular: P= ------- CONVERSIONES A) 71 B)72 C)73 D)74 E)75 07.- El valor de la siguiente expresión es: 11 rad + 33 rad +55 rad + ... +121 rad E~ 3°+ 9°+ 15°+ ... + 33° A) 200 B)205 C)210 D)215 E)220 PP1 774 Problemas de Trigonometría y cómo resolverlos RACSO D1TOK1S
08.- Sabiendo que (3x - y)° equivale a (x + 2y)E, calcular: _ A) 1/4 B)7/4 C)l/7 D)4/7 E)7 09.-Si: 7i/32raJ<>jc°y'z"<> AEBmCs, i > y+z-x calcular: M= ——~ A+B + C A) 1 B)2 C)3 D) 1/2 E)l/3 10 .- Si: M = dab‘c", además a, by c están en progresión aritmética y c" en el sistema radial es 7t/10 rad, i? en el sistema centesimal es 60g, calcular «M». A) 18° 23'28" B) 18° 28'38" C) 18° 33'48" D) 18° 36'54" E) 18° 38'58" 11 .-Si se cumple: calcular <<x» (x+3)’ 5* (4x-18)°Y 15* I A) 40 B)41 C)42 D)43 E)45 12 .- Se tiene un sistema de medida angular «P», un grado «P» (1P), equivale a 1/480 del ángulo de una vuelta. ¿A cuántos grados beta equivale 44/21 rad?. Considere: 71 = 22/7 A) 150? B) 160? C) 170? D)180P E)190P 13 .- Calcular el menor número entero en gra- dos centesimales que tiene el ángulo 6 si: 6 = 1^2™ + 3E4ra + 5E6m +... A)2916 B)3216 C)491E D)522e E)582e 14.- De la figura, calcular: A) 1/100 B)2/100 C) 3/100 D) 4/100 E)177O 15 .- De la igualdad = y', calcular el valor de: 2 y J ' A) 59 B) 1/2 C) 59/29 D)l/29 E) 59/58 16 .- Sabiendo que: 94,46 o PE RU VI , ii ,^ÍP + R + V\ calcular el valor de: 171 ——r;—r | \ E+ U + I / A) 15 B) 17/15 Q17 D)16 E) 14/17 17 .- Si se cumple que: calcular el valor de: z-y + x F= 2x + 2y A) 0,1 B)0,2 C)0,25 D)O,35 E)0,45 18 .-Si un ángulo se expresa como (xx)° y(xx + l)g, calcular (rx) en el sistema radial. A) 40 rad ^5 rad W rad D) y rad E) rad 19.-Reducir: M=—--------—--------— ^rad 4 A) 1 B)2 C) 1/8 D) 1/2 E)l/32 2ti 20.- La suma de dos ángulos es y rad, el mayor excede al menor en 36°. Calcular el su- plemento del mayor en radianes. ., 71 rad n. 71 , 13 71 rad A)_M“ B’Ü™' «^¡Ó- Sistemas de Medida Angular PP1 775
MEDIDAS ANGULARES RELACIONADAS 21.- Reducir: 27iS + 3tiC-40R /2C + S . E=-------------+ A'CZS~35 ; 92/? si se sabe que S, C y R son los números con- vencionales para un mismo ángulo. 26.- Si S y C son los números de grados sexagesimales y centesimales para un mismo ángulo, calcular la medida de dicho ángulo en el sistema radial, si: A)7t/1620 B)ti/810 C)7t/324 A) 12 B) 14 C)16 D)18 E)23 D) 71/162 E)rJ81 22.-Calcular: IC + 5S C + S , /c + 2S t ¡C + 6S ] C-S VC-S+V c-s V C-S 27.- Determinar la medida del ángulo en gra- dos sexagesimales, si se verifica: S + C _ 2R+7I C-S - 2R-7I ’ Siendo S y C los números de grados sexa- gesimales y centesimales de un mismo ángulo. A) 1 B)2 C)3 D)4 E)2-/Í5 23.- Calcular la medida de un ángulo en el sis- tema radial, si se cumple que: 7tC-197 R 7IS-52R Donde S, C y R son los números de grados sexagesimales centesimales y radianes de di- cho ángulo. A)7t/7 B)2k/7 C)tc/15 D)2k/15 E)rJ5 siendo S, C y R las conocidas A) 100° B) 120° C) 10° D)( 1/100)° E)(5/9)° 28.- Siendo S, C y R lo convencional para un ángulo trigonométrico y que cumple: = 95;(C + S)*0 ?S+C+R\(C io 2o 7i r 173(C+S) .2 24 .- Calcular la medida de un ángulo en el sis- tema internacional si se cumple: 718 \3 72O\3 7 7t V _ 1 \S/ + \ cj + \1ür) " 9 siendo S, C y R lo convencional A) 37t/10 rad B)37i/20rad C) 771/15 rad D) 7t/5 rad E) 37t/5 rad 25 .- Calcular la medida de un ángulo en el sis- tema radial sabiendo que: S + C+R/7t=9,4248 A)tT/123 D)tT/129 B)7í7125 E)7f/131 C) 717127 calcule el número de radianes de dicho ángulo A) 71 rad B) 71 radJYTS C) (7t/2) rad D)(37i/2)raJ E) (71/190) rad 29 .- Siendo S, C y R los números convencio- nales para un mismo ángulo tal que verifica: 3S-2C+~~ =1410 calcule la medida de dicho ángulo en el siste- ma centesimal. A) 2000® B)200^ C) 1008 D)4008 EJIOOO8 30 .- Sea dos ángulo a y P, si: Sa + Cp=95 y Ca - Sp = 5, calcule: Sa Problemas de Trigonometría y cómo resolverlos RACSO UlDITOBIl
donde Sa medida sexagesimal del ángulo a. A)50 B)30 C)45 D)40 E)55 31 .- Calcular, en radianes, la medida del mayor de dos ángulos si la suma de la cuarta parte del número de grados sexagesimales de uno de ellos y los tres quintos del número de gra- dos centesimales de otro ángulo es 70. Se sabe también que estos son suplementarios. A) 7t/6 B)57t/6 C) 271/3 D)4ti/3 E)3ti/2 32 .- La suma del número de minutos sexagesimales y centesimales de un mismo ángulo en 6160. Calcular el número de radianes del ángulo. A)7t/4 B)7t/5 C)7t/6 D)27t/3 E)7t/8 33 .- Dado un ángulo tal que el número de gra- dos sexagesimal y centesimal de un mismo án- gulo difieren en 5, calcula el complemento de dicho ángulo en radianes. A) 7t/2 ra d B) 71 ra d C) 7t/4 rad D) ti/3 ra d E) 7t/6 rad 34 .- Un ángulo es tal que el número de grados que representa su suma en los sistemas sexagesimal y centesimal es igual a 29 más su número en grados sexagesimales, dividido en- tre Z Calcular dicho ángulo en el sistema radial. A) Tirad B)7t/5 rad C)7t/10rad D) 71/20 ra d E) 371/10 rad 35 .- El triple del número de grados sexagesi- males menos el doble del número de grados centesimales del mismo ángulo es 70. Calcular la medida de dicho ángulo. A)7t/4rad 6)20® C)75° D) ti/2 rad E)7° 36 .- Determinar las medidas sexagesimales de dos ángulos, si la diferencia de sus medidas centesimales es 100 y la suma de sus medidas radiales es 371. A) 315°; 200° B) 115°; 225° C) 100°; 200° D) 315°; 225° E) 300°; 135° 37 .-Si: x... número de segundos sexagesimales de un ángulo. y... número de minutos centesimales del mis- mo ángulo; x+162y calcular: N = x_162y A)-2/3 B)-4/3 C)-3/5 D)-7/3 E)-3Z2 ÁNGULOS EN FIGURASGEOMÉTRICAS 38 .- Del gráfico adjunto, calcular la diferencia del ángulo mayor y el menor en grados centesimales. A) 200® B)144E C)54e D) 100® EJ150® 39 .- Del gráfico mostrado, calcular la medida del menor ángulo. A) 14° B)24° Q 32° D)26° E)28° 40 .- En el siguiente gráfico, si: m Z A, es igual a la m Z C, calcula «P». A) 10 B)9 Q9/10 D) 1/10 E)3/10 Sistemas de Medida Angular PP1
LONGITUD DE ARCO I Longitud de Arco CAI». 2 01.- En la figura AOB y CBO son sectores cir- culares con centro en «O» y «B» respectiva- mente. Calcular L^ILt. A) 4/7 B)7/4 C)3/7 D)7/3 E)5/7 02.- Se tiene un sector circular de ángulo cen- tral «6» y radio «R». Si la medida del ángulo central se reduce a la cuarta parte del anterior, ¿en cuánto se debe aumentar el radio para que la nueva longitud de arco no varíe? A)R B)2R C)3R D)R/2 E)3R/2 03.- Calcular «6» (en radianes), si en la figura: Lj = 4n ; L3 = 8n. O es centro. A) 2ít rad B) 2 rad C) 7t/4 rad ü)rJ5rad E)rd 12 rad 04.- Los sectores circulares mostrados tienen sus perímetros en progresión aritmética. Cal- cular «6» en términos de q, r2 y r3. O es el centro de los sectores circulares. 05.- Calcular el perímetro de la región sombreada, sí: R = 3, r— 1. Además: A, B, C y D son puntos de tangencia. A)2^/5 B)4,/3 +3 1971 JT C) , + V3 +6 o 71 r~ D) j + 9 + 3V2 E)45/3 +9 06.- De la figura, se tiene el cuadrante AOB, siendo O, A y B centros de los arcos AB, OQ y OP respectivamente; AO = OB = 6 ni. Problemas de Trigonometría y cómo resolverlos RACSO ^PBDIYOIEI
Calcular el perímetro de la región sombreada. A)7Wl B)2twi C)37tw D)47im E)5twi 07.- Del gráfico adjunto calcular el valor de 6 en el sistema sexagesimal en función de l2 y r. A)(ll+l2)/r ÁREA DE UN SECTOR CIRCULAR 08.- Se desea calcular el ángulo central de un sector circular, sabiendo que su longitud de arco es a su radio como 3n es a 7. *\ 71 j , 4ti , A) y rad B) y rad C) y rad D) y rad E) -y rad 2 09.- Si en la figura: r - 8/71, calcular el área de la región sombreada. A)1 B)2 C)4 D)8 E)10 10.- De la figura calcular «6», si se sabe que las regiones S. y S2 tienen igual área, además AO = OB = BC. A)2ti/3 B) 5 ti/6 C) 37t/4 D) 1171/12 E)7ti/8 D AOB C 11.- Del gráfico mostrado, calcular el área de la región sombreada, siendo «O» centro. Ade- más: AD = FD = EC = BC. A) 7 u B)8m2 C)9m2 D)10m2 E)11m2 12 .- El área de un sector circular cuyo ángulo central mide 72° es de 4571 cm . Se sabe que si duplicamos el radio de dicho sector y dismi- nuimos a rad a su ángulo central, el área del nuevo sector disminuirá en un tercio. ¿Cuál es el valor de a? A)7t/4 B)7t/3 C)tV6 D)tt/5 E)2ti/5 13 .- En la figura, se sabe oue el área de la región sombreada es 12/n u. Calcular el área del sector circular COD, siendo O el centro de los sectores circulares AOB y COD. A) 1 m2 B)l,5u2 C)0,25m2 D)O,5m2 E)2m2 Longitud de Arco
14 .- Determine el área de la región sombreada si las áreas de los sectores circulares POM y NOQ son iguales. Además: r — 1. A) 7t/20 B) 7*721 C) ti/22 D) ti/23 E)7i/24 15.- De la figura adjunta, si: r = 2, calcular el área de la región sombreada. 17.- Calcular el área de la región sombreada, si BO’C es un sector circular de radio 2R A)R2(73 +71/6) B)R2(73 -ti/6) C)R2(73 -tt/2) D)R2(73 +7t/2) E)R2(73 +7t) 18.- En la figura «O» es el centro de los sectores circulares AOB y DOE. Si «S» y «2S» son las áreas de las regiones sombreadas, calcule x/y. A)(2ti- 71) B) (271-273) C) (271-3^/3) D) (271-473) E) (2tt-5 73) 19.- De la figura, si A y C son centros. Calcu- lar el perímetro de la región sombreada. A) 12/7 B) 13/2 C) 1/12 D)5ti+2 E)5tt-2 A)7t(T3 +1) + ó73 B)tt(73 + 2) + 6(73 -1) ¿RACSO Ubditokii PP2 780 Problemas de Trigonometría y cómo resolverlos
C)n(-j3 +2) + (-j3+l) DJtiÍa/í + 6) + -j3 E)(a/3 + 1) + tt( +3 20.- En la figura se verifica que: — = A)1 B)2 C)4 D)16 E)8 S 5 Calcular el valor de «/». A) 1,5 B)6,5 C)3,5 D)4,5 E)2,5 21.- Calcular el perímetro de la región sombreada, si A, B y C son centros. A) (1+71/3) B) 2(1+71/6) C) 4(1 +71/4) D) 4(1 +71/6) E) 2(1+71/3) 22 .- Del gráfico calcular S-^Sl. si O A = 3 OC. A)1 B)3 C)6 D)7 E)8 23 .- En el gráfico se cumple que 2OA = AB, ____________ S, siendo OC bisectriz. Calcular . 24 .- Determine el área de la región sombreada si las áreas de los sectores circulares POM y NOQ son iguales. Además: r = 1. 25 .- Del gráfico adjunto: S, y S-, representan áreas donde S2 = 2Sp Si ademas 0 es un án- gulo expresado en radianes, calcular el valor de: 6 + sen 6 cos 6. A) 7i/4 B)7t/6 C)7i/8 D) 7t/12 E)7t/3 26 .- En la figura , O es centro de los arcos AB, CD y EF; siendo M y B puntos medios de los segmentos OE y OD respectivamen- te. Si OA = l y el área de la región sombreada 2S equivale a S, calcular: k — yyyy. Longitud de Arco PP2 781
29.- En la figura mostrada la rueda de radio igual a «r», recorre desde la posición A has- ta la posición B, sin resbalar, dando así 3 vuel- tas. Calcule el radio de la rueda, si además se sabe que: l = 1971 tn ; R = 8 tn. A)2m A) 20 B)21 C)22 D) 18 E)12 27.- Del gráfico mostrado, calcular el períme- tro de la región sombreada, si MN es perpen- dicular a OP. Además O y O1 son centros. 1 Itt i— tt j— A) -^+2^3 -2 D)j-2V3+2 B)^y-2-73+2 E)^ +2-73-2 1171 r- C) + 2V3 + 2 28.- Determine el número de vueltas que da el disco de radio (2 -j2 - 2), cuando rueda desde «A» hasta «B» sobre las superficies circula- res fijas de radios iguales a 2. Superficies fijas A) V2/2 B)3-72/2 C)5-72/4 D) -Jí E)2^2 B)3m C)4wí D)5m E)6m 30 .- Cuando el eje AA’ gira un ángulo de 270° en el sentido dado, en el cono de revolución se genera un área «S». Calcular el valor de «S». A) Jim2 B) 71/2 m2 D)2n/n2 E)7i/4m2 C) 71/3 tn2 31 .- En el sistema mostrado. Calcular la longi- tud descrita por el punto «P», si el engranaje de radio igual a 6 tn, gira un ángulo de 60°. 32 .- En el engranaje mostrado la polea de ra- dio 2 gira 120°, ¿qué ángulo girará la polea de radio 4? ^ARaCSO VpiDlTOBBB PP2 782 Problemas de Trigonometría y cómo resolverlos
A) ti/6 rad B) 71/3 rad C)7t/8 rad D) 7t/4 rad E)7t/10rad 33 .- Si el número de vueltas que gira la rueda de radio «/?» es numéricamente igual a 7L, cal- cular «R» sabiendo que su centro recorre una distancia igual a «44L». Asumir: it=lUl. 34 .- En el sistema mostrado, si hacemos girar la polea de radio 14, los bloques X e Y suben h y H respectivamente. Evaluar: «h/H» A) 147/16 B) 141/8 C) 113/2 D)17/8 E)7/6 35 .- Dos engranajes A y B tienen radios r y R (R > r). Cuando «A» recorre el perímetro de «B» ha barrido 900°, ¿qué ángulo barrerá B para recorrer el perímetro de A?. Ambos en- granajes están en contacto. A) 600° B)372° C)144° D)360° E)288° 36 .- Calcular el número de vueltas que da una rueda que se mueve recorriendo todo el perí- metro de otra de radio doble de la anterior. A) 1 B)2 C)3 D)4 E)5 37 .- En la figura, se tiene una rueda de radio r, inicia su movimiento en el punto M y luego de caer rueda por la superficie cilindrica de radio «a», llegando al punto N y en la forma que se muestra en la figura. Si la rueda en este trayec- to ha dado dos vueltas, calcular ría. MN: diá- metro. A) 4 B) 1/4 C) 1/2 D)2 E)1 38 .- Los radios mayor y menor de la rueda de un avión son entre si como 3 es a 1. Calcular el número de vueltas que da la rueda menor cuan- do la rueda mayor barre un ángulo de 1 840 Tt radianes. A) 3680 B)2760 C)1840 D)1380 E)4600 39 .- Un tronco de cono gira 180° sobre un pla- no horizontal de forma que su eje barre un ángulo a rad. Si los radios de sus bases son r y R(r< R), con generatriz «g», calcular el valor de: M = . R-r A)7t/2 B)7t C)27t D) 371/2 E)1 Longitud de Arco PP2 783
rs * . . I Razones Trigonométricas de Ángulos Agudos RAZONES TRIGONOMÉTRICAS DIRECTAS 01.- En un triángulo rectángulo ABC (C = 90°). cular DG al lado AB (D en sAB). Si sen A = 2/3, calcular BD/AB. A) 2/3 B)4/9 C)8/27 D)3/8 E)9/15 A)1 B)2 C) 1/2 D)0 E) 1/3 02.- En un triángulo rectángulo (C=90°) se cumple: sen A.tan B - sec B.tan A = -2. Calcular: T = 2 tan B.csc A - 3. A)-5 B) 1 C)l/2 D)-2 E)-3 03.- En un triángulo rectángulo ABC (C = 90°), se cumple: (csc A - tan C)cot A/2 = cot(0 - 8o) siendo: 6 es ángulo agudo, calcular E= -Jsen2 6 —cos(6+ 21°) A) 2/5 B)3/5 C)4/5 D) 1 E) 1/2 04.- En un triángulo rectángulo (B = 90°) se cumple: cot C sen A = 3, calcular: M= -Vcsc2 C -3 sec A A)0 B)^2 C)1 D) ^4 E)2 05.- En un triángulo rectángulo ABC (C = 90°) se cumple: (sec B - cot A)(l + sec A) 2 senB-cos A 3 Calcular: H = sec B - sen A A) 1/2 B)3/4 C)l/3 D)2/3 E)3/5 06.- En un triángulo isósceles ABC (AB = BC), desde el baricentro «G» se traza la perpendi- 07.- En un triángulo rectángulo ABC, recto en A, reducir la expresión: «2secB-¿>2cscC «tanB -¿> cot C ~ «cscC-£>secB tanB A) a B) b C) 2a D) 2b E) a/b 08.- Si la hipotenusa de un triángulo rectán- gulo mide 51 m y la tangente de uno de los ángulos agudos del triángulo es 8/15, calcular (en /n) el perímetro del triángulo. A) 80 B)90 C)100 D)110 E)120 09.- Del gráfico mostrado calcular «cot 0», si: AB = BCaBM = MC A) 1,5 B) 1,75 C)2,25 D)2,4 E)2,75 10.- Si: AD = m y BD = w, calcular «tan 0» clRACSO Woitoiii PP3 784 Problemas de Trigonometría y cómo resolverlos
A) m — n m + n B) m + n m — n C) m — n m + n 15.- Del gráfico mostrado AC = 2CD; CD = BC, calcular cot p. D) 'm + n m - n E) m — n mn 11.- Sobre la hipotenusa AB de un triángulo ABC se construye un triángulo equilátero ABD, siendo m Z CDB = 0. Además se sabe que: m Z CAB = 30°; calcular: tan*" 0. A)0,08 B)0,12 C)0,16 D)0,18 E)0,24 A) 1+ 73 B)2+ Jí C)3+ 73 D)4+ Jí E)5+ 73 12.- Si ABCD es un rectángulo; BE = EF, cal- cular: cot(a + 0) A) 3/4 B)4/3 C)3/5 D)5/3 E)1 17.- Del gráfico, calcule tan 0. 13.- Si: tan a + tan p + tan A + tan 0 = 20, calcular: tan A - tan a A)1 B) 1/2 C)l/3 D)3 E)2 A) 25/21 B) 125/121 C)123/121 D) 103/93 E) 100/93 R.T RECÍPROCAS 18.- Siendo x ángulo agudo se cumple que: 14.- De la figura M es un punto medio ade- más: 2 AC = 5 ED,calcular: secx. sen (x + 2 Io) tanfx + 22°) = cos (69° - x) Calcular: cot íx + 7°\ \ 2 ) A) 7ÍTT/5 B) 7102/3 C) 7104/5 D) 7106/9 E) 7112/9 A)2+ 73 D)4+ 73 B)2- Jí E)5+73 C)l - 73 19.- Si se cumple que: cos x csc y + cot 26° 30’ = cot 18o 30’ siendo «x» e «y» agudos, calcular el valor de: Razones Trigonométricas de Angulos Agudos PP3 , 785
f x+ v 1 i *+ y | W = sen I —2^" |cos I 3 |tan •* tan J A) Vó/4 B) -J3/4 C) ^2/4 D) a/6/2 E) V2/2 20.- Siendo «x» , «y» , «z» ángulos agudos que cumplen: sen 2a.csc (a + y) = 1 tan 2x.tan z = 1 C)TT -»P9 E) n 25.- Si: sen(20° + v) sec (30° + y) = 1 tan(30° + x) tan (40° - y) = 1 Además «x» e «y» son agudos, calcular; E — 4 sen x .eos (50° + y) Calcular: M = 3cos(x + 20°) sen(70°—y) + 2tan(y+z)tanx A)2 B)3 C)5 D)6 E)7 21.- Siendo ct, 0 y 0 ángulos agudos: tan (30° + 0) csc(ct+ 18°) = sec (0+ 27°) y además: a + 0=45°. calcular sen(0 + 22°) ax nx 24 n 3 rxx 4 r;x VÍO A)-r B) 25 C) 5 D) 5 E)-¡0~ 22.- Calculan E = cot l°.cot 2°.cot 3o... cot 88°.cot 89° A)0 B)l/2 C)1 D)a/3 E)-l/2 23.- Si: x, y, z son ángulos relacionados así: sen(x + y) - cos (85° - y - z) = 0 tan 2x. tan 3z = 1 Calcular: M = tan (2x +11°)- tan (x + 2o) A) 8/5 B)7/12 C)6/ll D)5/9 E) 1/2 24.- Siendo x un ángulo agudo, se cumple que: sen -CSC = 1 A)2 B) 1 C) 5/2 D) 1/2 E)V3 26.- Si a y 0 son ángulos complementarios tales que: X--J3 „ V6+V5 csc a = —¡=-j= ; sec 0 =---- V6-V5 x + V3 Calcular: A) 5/4 Cf-Jl D)25/7 E) 2 ^3/3 27.- Siendo a y 0 ángulos agudos tales que: cos (a+30).csc(2a+30) = 1 Calcular: 2 cos(2ct + 40) + sen(ct + 20) sec(ct + 20+15°) A)^ B)3-j2 C)^ D)1 E)^ 28.- Si: a, 0,6 son ángulos agudos, tales que: «+0 + 0 = 90° sen(ct + 0) + tan(0 + ct)—cos 0 tañé =3 Calcular: M = sen 6 + csc4 (0/2 + 30°) A) 5/2 B) 17/2 C)9/2 D)21/2 E) 13/2 29.- En la figura calcular tan 0. calcular cot x. Problemas de Trigonometría y cómo resolverlos RACSO JDITOIII
A) 1/2 B)2/3 C)3/4 D)4/5 E) 1/4 30.- Calcular tan a, si CD = 3 AB A) 1/3 B)2/3 C)3/4 D) 1/2 E)2 A) Jó B)7ó/2 C)7ó/3 D)72/2 E)7z 35.-Simplificar: a2 -¿/eos 50°+ n¿>(sen40°-l) a2 + b2 sen 40°+ «¿(eos 50°+1) A) B) — y- C) " a + ab a—b a + b _ a + b D)----- E) ab -1 a— b ' 31.- En un triángulo rectángulo ABC (recto en C), se cumple que: 8ab - c . Según esto deter- minar tan (B - A) 36.- Del gráfico, calcular: M = cot a - 2 cot 0 37.- Del gráfico calcular Vtan x A) 2-^2 B)4 C) VÍ5/4D) 715 E) 32.- Siendo x,y, z ángulos agudos que cumplen: sen 40° = cos x ; tan 70° = cot y ; sec 10° = csc z Calcular el valor de: E= 72sen(x+ y + z) + 6sen(x—y) A) 1 B)0 C)2 D)3 E) 41 A) sen 7t/3 B) sen 7t/6 C) sen 7t/4 D) sen 7i/8 E) sen 7t/12 33.- Calcular x , si es agudo, de la siguiente igualdad: sen (a + sen x)csc(x + cos 7t/12) = 1 A) 7t/12 B) 571/12 O ti/6 D)7i/3 EJti/9 34.- Siendo y agudo, tal que: tan (60° - x) = cot (x + 30°) tan (y + 20°) sen(x + y + 50° ) cos(20°+y) cos(y-x-10°) 38.- De la figura, calcular tan 0 A) 71 B)72+2 C)72- i D) 72 +1 E) 71 -2 Razones Trigonométricas de Angulos Agudos PP3
01.- En la figura mostrada: BD = AD, AC = b y BC = a. Calcular cot 0. 04.- En la figura ABCD es un trapecio rectán- gulo. Asimismo BM = MC y AD = 2BC; calcu- lar tan a. A) 1/2 B) 1/3 C)l/4 D) 1/5 E)l/6 D)2«h E)-^ 02.- En la figura: PB = AQ=L. Se pide expresar PQ en función de «L», si además se sabe que ABCD es un cuadrado. 05.- En un triángulo rectángulo, la hipotenusa mide L y un ángulo agudo mide «0». Calcular la longitud de la bisectriz relativa a la hipotenusa. A)15173 B)7L/3 C)5L/2 D)3L E)L73 LsenB 1 + tanB B) Q 03.- Del gráfico, calcular T = sena. senB senP L-Jl sen 6 l+tan0 ¿V2cot0 1 + cotB ¿V2cos0 l + tan0 ¿V2 tanO 1+ tan0 06.- De la figura BC = 2 y E es punto medio de BD . Calcular AD en función de x, y, z. A) 3 B)l/3 C)2 D) 1/2 E)1 RACSO MeDITOIII PP4 788 Problemas de Trigonometría y cómo resolverlos
A) sen z(cot x + cot y) B) sen z(cot x - cot y) C) sen z(cot y - cot x) D) sen z(tan y - tan x) E) sen z(tan x - tan y) 10.- En la figura se tiene el cuadrante AOB en donde M y C son puntos de tangencia. Se pide determinar el valor de: E = tan 0-1. 07.- En la figura AC = 7, BC = 5 y BM = x. Calcular: H = x2 + cot20. A) J2 B) ^3 C)2^2 D) V2/2 E) ^3/2 A) 10 B)9 C)5 D)3 E)7 11.- En el gráfico mostrado, se sabe que FD=DC; 08.- En la figura, las regiones triangulares ABP, PBQ y QBR son equivalentes. Si sen a = 1/6, calcule PQ en función de «h». calcular tan 0. A) 3/17 B) 17/3 C) 17/6 D)6/17 E)11/17 A)/¡/2 B)2A C)3/i D)4h E)A/3 09.- En la figura mostrada M es punto medio de AB y ABCD es un cuadrado. Si además se sabe que ABC es un cuadrante, calcular cot 0. 12.- Si CE = a y ET — b, calcular k, si: A)1 B)2 C)3 D)l/2 E)l/3 13.- En el gráfico mostrado se sabe que: AB =CD. Calcular el valor de: tan 0 + tan <J>. A) 3 B)3/2 C) 1/2 D)1 E)2 Resolución de Triángulos Rectángulos
14.- En la figura, CM es mediana. Calcular k, Si: A) 1/3 B)1 C)l/4 D)2 E)l/2 A) 1/2 B) ^3 C)1 D) VÍ/3 E)7/24 15.- En el gráfico mostrado ABCD es un rec- tángulo. Además se sabe que: CE = 2ED = 2. Calcular AG en términos de 0. 18.- Interiormente a un triángulo rectángulo ABC (B = 90°), se traza otro triángulo rectán- gulo AMD (M = 90°), donde M está en BC y D en AC. Si además se sabe que: AD = 3DC y m Z MAD = mZ DMC = a; calcular: 2 A COS OC A=-----x— cos 2a A) 2/3 B)l/2 C)4/3 D) 1/3 E)2 19.- A partir de la figura mostrada calcular: A = tan 0 cos a + sen a ABCD es un cuadrado, ADC es un sector cir- cular y T es un punto de tangencia. A) 2 sen 0 tan 0 D) 4 csc 0 sec 0 B) 2 sen 0 cos 0 E) sen 0 cos 0 C) 3 sen 0 cos 0 16.- Calcular* en función de n y 0 siendo BE=x; CD - n y m Z B AC = 0. Además se sabe que AD es bisectriz. A)0 B)1 C)^ D)-j3 E)2 20.- En el gráfico se sabe que: CD = DO. Se pide calcular el valor de: 2 E = cot a-2cosa A) n sen 0 B) n cos 0 C) n tan 0 D) n sen 0 cos 0 E) n sec 0 17.- En la figura T es punto de tangencia. Se pide determinar el valor de tan 0. A) 1/2 B)1 C)2 D)l/3 E)3 RACSO Pbditobbb PP4 790 Problemas de Trigonometría y cómo resolverlos
A) 1/2 B) 1/3 C)1 D) 1/4 E) 1/5 21 .- En la figura mostrada se verifica que: tan 6 + tan 0 = 5/3 Calcular: E = 3 tan 0+1 A)1 B)2 C)3 D)4 E)5 22 .- Se tiene un trapecio cuyos lados no para- lelos forman con la base ángulos 0 y 20. Cal- cular la longitud del lado no paralelo que for- ma al menor ángulo, sabiendo que el otro lado no paralelo mide tn. A) 2m sen 0 B) 2m cos 0 C) m tan 0 D) tn sec 0 E) m (sen 0 + cos 0) 23 .- En la figura mostrada MNPQ es un cua- drado, determine el valor de: E - tan 0 - cot <¡>. Además se sabe que O y B son centros de dos arcos de circunferencia del mismo radio. A)1 B)2/3 C)l/3 D)-2 E)-l 24 .- En un triángulo acutángulo ABC, calcular la altura relativa al lado AC si el área de la región 2 triangular es de 56 tn . Además se verifica que: cot A + cotC =7 A)4m B)3/n C)2m D)6m E)8/n 25 .- En el interior de un triángulo rectángulo ABC (recto en B), se ubica un punto M tal que las áreas de las regiones ABM y AMC son iguales. Si además: tn Z B AM = m¿ BCA=0; calcular: F = cos 2O.csc 0 26 .- En la figura mostrada, se pide calcular la longitud de AB en términos de a y /?, si ade- más se sabe que CD es diámetro. C) V2R Vi-eos2a 27 .- Del gráfico, expresar DE en términos de 0 ym. A) tn sen 0 D) m cot 0 B) tn cos 0 E) m sen 0 cos 0 C) tn tan 0 28.- A partir de la figura mostrada, se pide calcular: tan 0. A) 1,5 B) V2 C) J3 D)2 Resolución de Triángulos Rectángulos
29 Siendo ABCD un cuadrado, calcular «a» para que se verifique la relación: A)sen y B)cosy C)tany D)coty E)sen ycos y 30.- En un triángulo rectángulo la hipotenusa mide «/i» y uno de sus ángulos agudos «<J»>. Se ubica en su interior un rectángulo cuyos lados adyacentes se superponen sobre los catetos del triángulo. Calcular el área de la re- gión rectangular, si la diagonal del rectángulo es perpendicular a la hipotenusa del triángulo. A)/i sen<J>cos<l> D)/i (sen<J> + cos<¡>) B)/i2 sen2 <¡> eos2 <¡> E)/i2(sen2<J>+cos2<¡>) C) A2 sen3 <]> eos3 31.- Dos polígonos regulares de «m» y «n» lados tienen perímetros iguales. Calcular la re- lación de sus áreas. m 7t 7t tn 7t 7t A)—tan—cot— D)—sen—csc — n m n n m n _. n 7t ti n 7t 71 B) — sen— esc— E) — cos — sec — m m n m m n „ m n 7t C) — sec—eos — n tn n 32.- A partir del cubo mostrado, calcular: M = sen 6 sec a sec 0 A)1 B) ^2 C) V? 0)2^2 E)j5 33 .- En un triángulo ABC, la bisectriz interior CD (D en AB), verifica que: J______1_ 1 CD - AC + BC Calcular la medida del ángulo C. Sugerencia, utilizar: sen 2a = 2 sen a cos a. A) 30° B)45° C)60° D)9(F E)12(r 34 .- En un triángulo rectángulo ABC, recto en «C», se toma un punto P de AC , del cual se traza PQ perpendicular a AB (Qen AB). Si el área de la región cuadrangular BCPQ es ocho veces el área de la región triangular AQP, calcular AP, si además se sabe que: AB = 24. A) 6 B)7 Q8 D)9 E)10 35 .- A partir del gráfico adjunto, calcular: PP4 792 Problemas de Trigonometría y cómo resolverlos RACSO vp BDITCKia
36 .- Del gráfico adjunto, calcular «AE» en tér- minos de «a» y «6», siendo ABCD un cua- drado de lado igual a la unidad. E) (I +tanot)tanO+ 1 2 37 .- Del gráfico, el equivalente de 4S tan x en términos de «m» es: A) -J3 tn' B)m2 C)^m2/2 D)m2/2 E)2rn2 A)(^+l)// D) 2(^-1)// B)2(^ + l)// E) (2^/3+1)7/ -1)7/ 39.- En la figura mostrada, calcular «cos 0», si BM es mediana y además se sab^que el área de la región triangular ABC es lu. A) 1/2 B) 1/3 C) 1/4 D) 1/5 E) 1/6 38.- De la figura mostrada, calcular la altura del faro en términos de «H», siendo «M» pun- to medio de aquel. Resolución de Triángulos Rectángulos PP4 793
ÁNGULOS VERTICALES 01.- Desde el pie de un muro de 4 m de altura el ángulo de elevación de la parte superior de un edifico es de 53°. Si desde la parte superior del muro el ángulo de elevación del punto anterior es P, calcular (en m) la altura del edificio. Con- sidere: tan P= 1,2. A) 30 B)38 C)42 D)40 E)20 02.- Desde un punto sobre el suelo se ve lo alto de una casa con un ángulo de elevación 0. Si nos acercamos una distancia igual al tri- ple de su altura, el ángulo de elevación de la parte alta de la casa es el complemento de 0. Entonces el valor de: cot 0 - tan 0 será: A) 1 m B) 2m C) 3 m D) 4 m E) 5 tn 03.- Un turista contempla un monumento so- bre un pedestal con un ángulo de observa- ción de 8°. Si la parte más alta del pedestal lo observa con un ángulo de elevación de 45°, determinar la altura del monumento, si se sabe que la altura del pedestal es de 18 m. A)lm B)8/n C)2m D)3/n E)6m 04.- Una persona sube por una loma inclinada un ángulo de 30° respecto de la horizontal. Desde el pie de la loma observa la parte supe- rior de una antena, ubicada en la cima, con un ángulo de elevación de 45°. Luego de subir tn hacia la antena, el nuevo ángulo de elevación es de 60°. Calcular (en tn) la altura de la antena. A)2 B)4 C)5 D)1 E)7 --j-1--r --1 Situaciones Contextualizadas 7, ! K ' 1_L MP. 5 05.- Una piedra se encuentra a una distancia de 9 m de un edificio de 4 pisos. Desde la parte superior e inferior del segundo piso, se obser- va dicha piedra con ángulos de depresión P y 0 respectivamente. Asimismo, un observador ubicado en la azotea localiza a la piedra con un ángulo de depresión «a»; calcular (en m) la altura del edificio, si se verifica que: cotot+cotP + cotO =21/4 A) 11 ni B) 12/n C) 13m D)14m E)15m 06.- Un avión desciende rectilíneamente, con una inclinación de 53° respecto de la horizon- tal. El piloto obse va delante de él la pista de aterrizaje bajo un ángulo de 23°. Si en ese ins- tante el avión está a 1000 m de altura, calcular la longitud de la pista. A) 880 ni B)980»n C)750ni D) 650 tn E) 800„i 07.- El asta de una bandera está colocada al borde de un precipicio de 40 m de altura que se encuentra a la orilla de un río de 30 m de ancho. Una persona al lado opuesto del río observa el asta bajo un ángulo de 8°30’. Con- siderando: tan 61° 30’ = 1,8; la longitud del asta será: A) 41 ni B) 14m C) lOni D)7m E)15m 08.- Desde dos aviones que vuelan a la misma altura, con la misma velocidad y separados una distancia determinada, se puede observar un punto «P» en el suelo, con ángulos de de- presión «0» y «a» (0 > a) respectivamente. Cuando el avión que estuvo más lejos de «P», PP5 794 Problemas de Trigonometría y cómo resolverlos RACSO DITOlll
se ubica en la misma vertical de aquel, desde «P» se observa al otro avión con un ángulo de elevación «<¡»>. Calcular: M = tan <¡) (cot ct - cot 6) A) 2/3 B)l/2 C)1 D)5/2 E)3/5 09.- Un basquetbolista observa la copa de un árbol con un ángulo de elevación de 37°. Si el deportista dista 8 m del árbol, calcular el valor del ángulo de observación del árbol sabiendo que su estatura es la cuarta parte de la del árbol. A) 42° B)48° C)51° D)58° E)64“ 10 .- Las caras de la pirámide de Kefren en el valle del Nilo, están inclinadas aproximada- mente 53° con la horizontal. A una distancia de 84 m directamente a partir de la base, el ángulo de elevación a la cúspide de la pirámi- de es de 37°. ¿Cuánto vale (en ni) la extensión de la cara, desde la base hasta el vértice de la pirámide? A) 140 B)150 C)160 D)170 E)180 11 .- Desde la base de una colina inclinada un ángulo «a» respecto a la horizontal, una per- sona observa la parte superior de una torre de altura «/i» con ángulo de elevación «2a». ¿Qué distancia debe ascender la persona sobre la colina para que el nuevo ángulo de elevación sea «3a»? A) h sena B) 0,5/icos a C)2/itana D) 2/¡ cot a E) 0,5 h csc a 12 .- La elevación angular de una torre CD desde un lugar A al sur de ella es 30° y desde un lugar B hacia el oeste, la elevación es 18°. Si AB = a, se pide calcular la altura de la torre. 13.- Desde la base de un faro parten 3 perso- nas A, B y C siguiendo las direcciones sur, oeste y el tercero siempre equidistando de los otros dos pero alineado con ellos. Si en un instante dado A y B observan al faro con án- gulos 0 y <¡) respectivamente, ¿cuál es la cotangente del ángulo de elevación con el que observa «C» al faro? A) i-7cot26+cot2<¡> B) -^--Jcot2 0—cot2 <}> C) -Jcot20 + cot2<j> D) i-^cot20 + cot2<¡) O E) -Jcot20-cot2<j> 14.- Un asta de bandera se levanta en el cen- tro de un terreno que tiene forma de paralelo- gramo y que es vista con ángulos de eleva- ción «a» y «0» desde dos de sus vértices con- secutivos. Calcular la relación entre las diago- nales del paralelogramo. tana tan0 B) csc a CSC0 seca sec0 D) cosa COS0 ... sena E)I^0 15.- La elevación angular de una torre desde un lugar «A» al sur de ella es 30° y desde un lugar «B» hacia el oeste de «A» la elevación es 18°. Si AB = 40 -^2-j5 + 2 in, calcular (en tri) la altura de la torre. (J5-1) Considerar: sen 18°=---¿--- ™ a-^3+2-j5 g, a-j3 + -j5 2 2 A) 20 B)40 C)60 D)80 E)10 16.- Un caminante observa un mástil con un ángulo de elevación a. Cuando la distancia que los separa se ha reducido a la tercera par- te, el ángulo de elevación se ha duplicado. ¿Cuánto mide a? Razones Trigonométricas en Situaciones Contextualizadas
A) 60° B)3(T C)45° D)37° E)53° 17 .- Desde un punto situado a 50 m del pie de un edificio se observa con un ángulo de 9o el asta de una bandera ubicada en la parte supe- rior de aquel. Asimismo, la parte inferior del asta se observa con un ángulo de elevación de 36°. Calcular (en m) la longitud del asta (con- siderar cot 54° = 0,72). A) 18 B)15 C)13 D)14 E)12 18 .- Una persona colocada en la misma hori- zontal del pie de una torre, observa la parte superior de ésta con un ángulo de elevación de 37°. ¿Cuántos metros debe caminar hacia la torre para estar a 180 m de ella y divisar su cúspide con un ángulo de elevación igual al complemento del anterior? A) 120 B)140 C)160 D)180 E)200 19 .- Desde lo alto de un edificio se observa un automóvil con un ángulo de depresión de 37°. Si dicho automóvil se desplaza rectilíneamente con una rapidez constante acercándose 28 m al edificio es observado, desde la azotea, con un ángulo de depresión de 53°. Si de esta po- sición tarda en llegar al edificio 6 s, calcular (en m/s) la rapidez del automóvil. A)8 B)12 C)3 D)4 E)6 20 .- Desde la parte superior de un acantilado se divisa un barco en el mar, con un ángulo de depresión «a». Un aeroplano que viaja hori- zontalmente al mismo nivel del acantilado, ob- serva al barco y a la base de aquel con ángu- los de depresión «20» y «0» respectivamente (20 < 90°). Calcular: sen 20 cot a. A) 1 B)2 C)l/2 D)3 E)3/2 21 .- Desde un punto al sur de un árbol se observa la parte superior del mismo con un ángulo de elevación «a» encontrándose di- cho punto a una distancia «a» respecto de la base. Si desde la parte superior se observa, con un ángulo de depresión «0», otro punto ubicado al norte del árbol, ¿a qué distancia se encuentra dicho punto de la base? A) tan a tan 0 B) cot a cot 0 C) tan a cot 0 D) cot a tan 0 E) tan a sen 0 22.- Desde lo alto de un acantilado se obser- va, en dirección sur, una boya con un ángulo de depresión de 45° y en dirección este un bote con un ángulo de depresión de 30°. Si la distancia que separa a la boya y al bote es de 80 m, calcular la altura del acantilado. A) 20 m B) 24 m C) 30 m D) 40 m E) 48 m 23.- Un poste se encuentra equidistante de otros dos iguales pero de menor altura. Un ob- servador, que ubicado en la misma línea y a un extremo de los tres, visualiza con ángulos de elevación a, 0, 0 respectivamente (0< a < 0) sus extremos superiores. Calcular la altura del poste mayor si los más pequeños miden h. A.) h/2 (cot a + cot 0)tan 0 B) h (cot a + cot 0)tan 0 C) h/2 (cot a - cot 0)tan 0 D) h/2 (cot 0 + cot 0)cot a E) h/2 (cot a - cot 0)cot 0 24 .- Un avión y un barco viajan en la misma dirección. En la primera observación efectua- da desde el barco se ubica al avión adelante y con un ángulo de elevación de 53°, marcando dicho punto con una boya. En la segunda ob- servación lo visualizan con un ángulo de ele- vación de 37°. Si la velocidad del avión es 4,5 veces que la del barco, calcular la cotangente del ángulo de depresión con la cual se ve a la boya desde la segunda posición del avión. A) 1 B)2 C)3 D)2,5 E)L5 ÁNGULOS HORIZONTALES 25 .- A las 2 de la tarde, desde un barco se observa hacia el norte dos faros alineados. Luego de navegar hacia el NE, a las 4 de la tarde se observa a los faros en las direcciones OaS y NctO. Nuevamente a las 10 pm desde el barco se observa a los faros en las direccio- nes: O0S y N0O. Si la distancia entre los faros PP5 796 Problemas de Trigonometría y cómo resolverlos 4^ RACSO PBDITOKKB
es de 20 km y el barco navega a v locidad cons- tante, calcular el valor de la rapidez (en km/h) C) 17 26 .- Un barco se desplaza 40 km siguiendo la dirección S60°O con respecto a un puerto. Luego se desplaza 20 km según el rumbo N60°O. Calcular (en km) la distancia del puer- to a su posición final. A) 20-7? B)10-73 C) 10-7? D)15-7? E)25-7? 27 .- Indicar verdadero (V) o falso (F) según corresponda en las siguientes proposiciones: I. N 4 NE es la notación de 11°15’ al norte 4 del nor-este. II E15°N es la notación de 15° al norte del este. m Si el rumbo de «A» respecto de «B» es N37°E, entonces el rumbo de «B» respec- to de «A» es S37°O. IV Si la posición de «P» respecto de «Q» es NE“ S, entonces la posición de «Q» res- pecto de «P» es SO ± S . A)FFVF B)VFVF C)WFF D)VFFF E)FVW 28 .- En un instante determinado el vector po- sición de una araña es (3; 4; 40), desde donde observa tres moscas, en el suelo exactamente en los puntos (0; 0; 0), (3; 0; 0) y (0; 4; 0), bajo ángulos de depresión a, 0,0 respectivamente. Calcular: M = tan a + tan 0 + tan 0 A)84/5 B)73/3 C)94/3 D)91/3 E)88/3 29 .- Un niño se encuentra entre dos árboles, observando un ave en cada copa de los árbo- les. Si el ángulo que forman las visuales es 135°, determinar la distancia en metros entre las aves, si además se sabe que el niño se encuentra, de éstas, a una distancia de 7-72 y 17 metros respectivamente. A) 25 B)24 C)24-72 D) 25-72 E) 24-73 30 .- Desde un punto exterior a la luna, se la observa con un ángulo «20». Encontrar la menor distancia a la que se encuentra dicho punto de la superficie lunar, si el radio de la luna es igual a «r». A)rsec6 B)r(secO-i-l) C)r(csc0-l) D) r tan 0 E) 2r cot 20 31 .- Dos móviles parten de un punto, el prime- ro con dirección N0E y el segundo con rumbo S20E. Cuando el primero recorre 4 km. el se- gundo recorre 4,2 km. Si la distancia que los separa es de 5,8 km, calcular tan 0. A) 1 B)l/2 C) 1/3 D) 1/4 E) 1/5 32 .- Una persona parte de una ciudad A y lle- ga a una ciudad B describiendo una semicircunferencia de longitud «/». Si la ciu- dad B se ubica en la dirección E0N respecto de la ciudad A, calcular la menor distancia de la persona a la ciudad A, cuando ésta parte rectilíneamente de B hacia el Sur. A) Un sen 0 B) U2n sen 0 C) 2//n cos 0 D) U2n cos 0 E) Un cos 0 33 .- Un bote navega rectilíneamente hacia el este a 2-717 millas/h. A las 13:15 horas ob- serva la cúspide de un acantilado en el rumbo NE-^ E con un ángulo de elevación de 30°. A las 13:45 horas observa otra vez la misma cús- pide que ahora se visualiza en el rumbo NO N y con un ángulo de elevación de 60°. Calcular (en m) la altura del acantilado. A)^ B)5 C)3 D)3-73 E) Razones Trigonométricas en Situaciones Contextualizadas PP5 797
R.T. DE UN ÁNGULO ENP. NORMAL Razones Trigonométricas de Angulos en el Plano Cartesiano SITUACIONES GRÁFICAS Ol.-Si:O<0<a<27t y además: sen 6 + sec a = 0 „ , - a 0 2(a+6) Calcular: M = csc ~7 + cot 77 - cos--—- o 2 9 A) 1 B)5/2 C)2 D) 1/2 E)3/2 3ti 02.- Si: -y < x < 27t, hallar "tan x" sabiendo que: M = (3 cotx + 2 tan x)/2 toma su máximo valor. A)-V3 B)-V3/2 C)-¿6/2 D)-V3/4 E)-Vó/4 03.- Siendo 6 un arco positivo (0 < 0 < 27t), donde se cumple: Vexsec20 + -¿versa-1 < -¿2 sen - sen 7; 4 2 Ct Calcular: k = tan O.exsec y A)0 B)1 C)2 D)3 E)4 04.- Si: Veos2 6 + cos 6 = 0 y: ¿4-tanO + Vtan0-4 = tan a + 2 7 Siendo: a G IIC; determinar: E = (sen 0 cos a) A) 8/5 B) 16/85 C) 19/15 D) 19/85 E) 17/19 05.- A partir de la figura, determinar el área de la región triangular AOB, si el punto medio del segmento PQ es el baricentro del triángulo AOB. A) 3/4 B) 1/2 Q2/3 D)l/4 E)9/2 06.- En el siguiente gráfico, evaluar "tan a" a-1 B) — a C) a— b D) — a a-2 O'(-2;7) 07.- Sea a un ángulo positivo menor que una vuelta cuyo lado final no cae en el IC , y otro ángulo: 0 G (-180°; 0), con el cual se verifica que: 1 + V-cos20 = tan a Calcular: . , tana + sen0 M= ----------- ¿2 sena A)0 B)1 C)2 D)3 E)4 PP6 Problemas de Trigonometría y cómo resolverlos ¿tRACSO W1DITO11I
08.- A partir de la figura mostrada, calcular: E = tan a + cot a, si se sabe que "G" es el baricentro del triángulo ABC. A) -61/30 B)-67/30 C)- 71/30 D)-73/30 E) -74/30 D)21 12.- Del gráfico, determinar: N = 4 cot 0+3 A)0 09.- Del gráfico mostrado, calcular el valor de: cot 20; si se sabe que: PO = 41 B)-l A)-4/5 B)4/5 C)3/5 D)-3/5 E)3/4 C)-2 D)-3 E)-4 10.- En el gráfico mostrado, "O" es el centro de la semicircunferencia. Si BC = 0.2AC, eva- luar la expresión: A) 26 13.- Del gráfico siguiente. Determine el valor de: cot 20 - cot <J>. A)^ D)^ 14.- En la figura: AB = 4BC. Se pide encontrar: P = 16 tan 0+13 cot 0 donde "G” es el baricentro de la región trian- gular sombreada. A) 27 11.- En la figura mostrada las coordenadas del punto "R” son (6 43 ;8). Determinar la distan- cia del baricentro de la región "MON” al pun- to "R". B)28 C)29 D)30 E)32 Razones Trigonométricas de Ángulos en el Plano Cartesiano
PROBLEMAS CONDICIONALES [7t sen 6 ; n csc 6] y [0; 7t]. 15.- Se tiene un ángulo 6 en posición canóni- ca que verifica las siguientes condiciones: A)[pfl C*!?’71) i) |cos 6| = -cos 6 ii) |tan6| = tan6 iii) |sen 6| = Evaluar: M= 45 csc 6 + 9 cos 6 D) 71 271\ 5’ 3 / 19.- Siendo "a" y "6” ángulos agudos tal que: |3 + sen cc| -12 - cos6| — P; calcular el valor entero de "P". A)-11 B)-10 C) -9 D)-8 E) -6 A)-l B)0 C)1 D)2 E)3 16.- Sabiendo que: tan 0 = - — ; calcular el va- lor de "sec 6 - csc 6”, verificándose además 20.- Del gráfico mostrado, calcular el mínimo valor de E, si: cot 6. sen 6 < - |^sen6¡ .. -5VÍ3 -5JÍ3 A) “13~ B) 6 E)-13-/7 C)-13 V5 17.- Determinar la ecuación de la circunferencia que tiene como diámetro la porción de la recta: L:2t-3y + 12 = 0 comprendida en el segundo cuadrante por los ejes coordenados y la tangente del ángulo en posición normal cuyo lado final pasa por el centro de la circunferencia. A) (jc + 3)2 + (y - 2)2 = 13; -2/3 B) (x+ 3)2 + (y + 2)2= 13; 3/2 C)(x+3)2+(y-2)2= JÍ3 ;-2/3 D)(x-3)2 + (y-2)2 = 13;-3/2 E)(x-3)2 + (y-2)2= VÍ3;-1 18.-Si: |sen6-^| = | donde 6 es un ángulo agudo, determinar la intersección entre los in- tervalos: „ Area del AABC E = 7--T“¡—;MQ = 2MN Area del □ MNPQ Sugerencia: 2 sen 6 cos 6 = sen 26 21.- La abscisa, ordenada y radio vector de un punto P que pertenece al lado final de un án- gulo "a" en posición normal están en progre- sión aritmética, en el mismo orden. Calcular: csc a - cot ct. A) 1/2 • B) 1/3 C)1 D)2 E)3/2 22.- Si se cumple: sen x cos 26 = 1; siendo "x" y ''6'' ángulos no negativos y menores que una vuelta, determinar el mayor valor de "x + 6". A) 7t rad B) — rad D) 2tt rad E) 3ti reíd C)^rad PP6 800 Problemas de Trigonometría y cómo resolverlos RACSO
23.- Dado: |sec a + cos a| = 2,9 donde: "a" pertenece al tercer cuadrante, de- terminar el valor de: E = sen a. tan a A) -2 B)-2,l C)-2,4 D)-2,5 E)-2,7 SIGNOS DE LAS R.T. 24.- Si: se verifica: < a <-^ , 0 < B ; 2 4 4 evaluar el signo de: sen(ct + 0)sen a cos(30+a)sec3y r / vitan(^+2P) secl2a+0 + ^j cos(3a+0)sec^ + a) sen a. tan 20 esc3^c(f+a) A) (-);(-); (-) D) (+);(+); (+) B) (+);(-); (+) E) (-);(-); (+) C) (-);(+);(+) 25.- Sabiendo que "a" es la medida de un án- gulo trigonométrico, tal que su valor está com- prendido entre 210° y 300°; determinar el sig- no de las siguientes expresiones: I. tan —. csc a II. cot(ct - 30°). sen & IH. tan(ct+60°). |sen ct| A)+, + ,+ B) + , + ,- D)-,-,+ E) + ,-,+ C) + ,-,- 26.- Si "6" e IIIC, determinar el signo de: P = cos(sen 6).tan(sen 6) A)(+) B)(-) C)(±) D)(+)ó(-) E)Nulo 27.- Se sabe que: "a" y "6" son ángulos en posición normal, positivos, menores de una vuelta y ubicados en diferentes cuadrantes, tal que: a < 6. Si además: i) |tan ct| =-tan a ii) cos 6 > 0. Indicar el signo de: P =---------- csc^ 2 A)(+) B)() C)(+)ó(-) D) (+) y (-) E) nulo 28.- Siendo: tan a + sen a < 0 tan 6. csc a < 0 Identificar el cuadrante de "6" sabiendo que "6" y "a" pertenecen al mismo. A)ic B)nc c)inc D)IVC 1) IIC y IIIC R.T. DE ÁNGULOS CUADRANTALES _ _ sen2jr+sen4x-sen6x 29.- Dado: f(x) =--------------------- J cos2x+cos4x+tanx-4sec4x calcular: "/(7t/4) " A) 1 B)l/2 C)-l D)-l/2 E)2 30 .- Calcular: 3sen90°- 2cosl80°+ sen270° 4cos360°- 5cosl80°- 2sen90° A) 1/7 B)l/2 C)4/7 D)3 E)-l 31 .- Calcular: sen n + sec 0 - cot 270° - sec n csc90° - cos n+tan 18 0o - cos k/2 Razones Trigonométricas de Ángulos en el Plano Cartesiano
A) 1 B)-l C)2 D)-2 E)3 32 .- Si la suma de a, 0 y 6 está comprendida entre 2760° y 3180°, siendo a y 6 suplementa- rios, calcular "0" si se sabe que es un ángulo cuadrantal que toma su mayor valor. A) 2 070° B)2670° C)3240° A)-74/35 B)-15/24 Q-7/24 D)-21/10 E) 21/10 D)2970° E)3060° 38.- En el gráfico: tan a= 0,75; calcular tan 0. 33 .- Reducir la siguiente expresión: cos(4A: -1)^| +[tan(2A:+l)ít]4'1 + |sen(4t:+1)^ [secít]" +[cscy] -[senyj donde: neZ A)l+(-l)“ B)(-l)’n C)1 D)0 E)-l 34 .- Si :fíx)=sen 3x- cos 2n es una expresión nula; calcular la suma de valores que toma "cosa" sien- do "x" un arco positivo y menor que una vuelta. A) 1 B)-l C) 1/2 D)0 E)j3/2 ÁNGULOS COTERMINALES 35 .- Sabiendo que: "a" y "0" son suplementa- rios y coterminales, calcular el valor de: M = cot a - cos 6 A)0 B) 1 C)2 D)3 E)4 36 .- Determinar dos ángulos coterminales sa- biendo que el mayor es a la suma de ambos como 13 es a 17 y que la suma de estos es mayor que 2 300° pero mayor que 1900°. A)365°yl725° B)819°yl021° C) 460° y 1570° D) 455° y 595° E)480°y 1560° 37 .- De la figura mostrada, evaluar: tan 6 - cot 6, sabiendo que: cot 0 = 1,4.0 A) -3/4 B) -4/3 C) -7/12 D) -12/7 E) -14/13 39 .- La suma de las medidas de dos ángulos coterminales es 240° y la medida del ángulo mayor está comprendida entre 470° y 500°, calcular: sena N=^0+tan(a-0> siendo "a" y "0" las medidas del ángulo ma- yor y menor respectivamente. »- -J3 3 4 A)0 B)-V3 C)-Y P)-| E)-| 40 .- Se sabe que: "a” es coterminal con: "-2a" y ”0" es coterminal con ,,-30,,. Si además a y 0 son coterminales; calcular "a+ 0", siendo "a" y "0" positivos, diferentes y los menores posibles. A) 1080° B) 1 140° C) 820° D)835° E)840° 41 .- Sean "a” y ”0" dos ángulos trigonomé- tricos que se encuentran en la relación de 5 a 2 respectivamente. Si además se sabe que: "2a" y "0" son coterminales, determinar "a + 0" siendo "0" el mayor ángulo negativo posible. A)-24CT D)-315° B)-90“ E) -270° C)-135° VÍRACSO ^«TOWi PP6 802 Problemas de Trigonometría y cómo resolverlos
LÍNEA TRIGONOMÉTRICA SENO (Rozones Trigonom ^tricas de Números Reales) 01.- ¿Qué valores puede tomar "x" para que i « x-2 x+1 se cumpla: sen 0 = —— + —, siendo: 6 un arco del tercer cuadrante? A) (-2V3 -l;2>/3 -1) B) (-2^31)^(1; 2^3 ) C) (-2>^;-l;-3)'-’(l;2V3-l) 02.- Ordenar en forma creciente: sen 4, sen 5, señó, sen(-l). A) sen 5; sen(-1); sen 4; sen 6 B) sen(-l); sen 5; sen 4; sen 6 C)-sen 5; sen 4; sen(-l); sen 6 D) sen(-1); sen 4; sen 5; sen 6 E) sen 4; sen(-l); sen 5; sen 6 03.- Hallar el conjunto de valores que toma la expresión: sen2x + |senx| E = i i r sen x-|sen x| A)(-oo;-i] B)(-<=o;-l) C)(-°°;0) D) (-oo; 0] E) (-«>; 1) D) (-2a^;2a^) E)<-3; 1> O5.-Si:xie y 0e determinar el intervalo de: P = 2 sen 0 - y sabiendo que: sen X] = y . A> (M] B> [t;t) C> (? ; J7J] 06.- Hallar la extensión de "a" que cumple con: -77 <a<n< partir de 1 < 2sena+l < >/3 +1 2 2 2 [27t.57tl rix/27t.57tl „ /íT. 57t\ T’Tj B)\3’óJ C)\3'~6/ r2n.57i\ _/2n.57r\ D) LT’T/ E) \T'-ó) 04.-SÍ: % o 3 determinar el intervalo de: M = 4 sen x - 1 71 07.-Si: — < 3a <7t, calcular la variación de: P= Jl-sen2(273sena) Circunferencia Trigonométrica PP7
A) (cos 3 ; 0) C) (cos 3 ; cos ) B) [0; cosí) D) (-eos ; cos3) E)[0;-cos3> LÍNEA TRIGONOMÉTRICA COSENO / 7t 1171 II 08.- Si 0 c (—; —r~ \ determinar el intervalo de: \ j o J A)[-2;V3] C)[0;V3) M = 2cos6 + 1 B)(-l;>/3] D)[l;^3 + 2] 12.- En la C.T. mostrada, calcular la ordenada de "P". j A) 3 cos 0/( 1 + 2 sen 0) B) 3 cos 0/(1 - 2 sen 6) C) 3 sen 0/(1-2 cos 6) D) 3 sen 0/(2 cos 0-1) E) 3sen© (2cos© - 1) P 13.- Determinar los valores de "m" que hacen posible la siguiente igualdad: 2 2 eos 0 + 2/ncosO + m = 1 E)[- 1;a/3 +1] 09.- Si: x e —, evaluar el intervalo de: L 3 6/ /(x) = 2cos|2a¡ A)(-l;0> C) B) [-1 ;2] 1 2 10.- Hallar AT en términos de ”6". A)(-l; l) B)[-l;l> C)(-2;2> D)[-2;2] E)[-2;2> p 14.- Si: 0 e IVC,P> 0y cos 0 =-; enton- J x-n ces podemos afirmar que: A)x>P-n B)x<P-n C)x>p + n D)x + 2m>p E)x-2n<p 15.- De las siguientes proposiciones: I. |cosx|<l II. -1 < sen9x < 1 O. “7 — sen6x + cos6x < 1 4 son correctas: A)I B)lyll C)m D) I y III E) Todas son correctas J&) = 2 A eos* x-4cosx+5 ,Vxe 16.- Hallar la variación de cos (0 - x ]), sabien- do que: x1 < 0 < x2 y (x2 - X]) e • D) | 1 A) (cos(x2-Xj); 1) C)(cos(x2-X]); 1] E) (cos(x2 - X,); 0) B) [cos(x2-Xj); 1) D) [cos(x2-X]); 1] PP7 804 Problemas de Trigonometría y cómo resolverlos ARACSO
UN. TRIG.: TAN-COT- SEC - CSC 17.- Si se cumple que: sec x. csc y = 1 2 > _ tan x + cot~y = 0 y además: x e (3; 5) a y e (4; 6); obtener el valor de: 3x-2y. A)0 B)ti C)¿ D)y E)2n 18.- Si se verifica que: sen 0 > sen ct, además: o, 0 e \—’ anal|zar e> valor de ver- dad de las siguientes proposiciones: I. cos a > - |cos P| II. -|tan a| < - |tan 0| III. tan(a+ > cos(P+ A)VFF B)FFV QVFV D)WF E)VW calcular la extensión de “ -Jl tan a + cot P" A)(-V3;3) B)(-3;3> C)(-3;3] D)[-V3;3] E)[0;3] 20.- Del gráfico mostrado, calcular la suma de coordenadas del punto "P". A) (2; 3) 21.- Determinar los límites de "a” entre los cuales se cumple que: sec cotana (0<a<27t) C) (2 D)(2;7I)°(^;27r) \ ¿ 1 \ ¿ • 22.- Sabiendo que: 0 e (-1; 1), identificar la proposición correcta: A)sen0>O B)tan0>O C)cos6<0 D)cot6<0 E)sec6>0 23.- Del gráfico, evaluar el área de la región sombreada si MN // AB A) sen6 - cos6 B) sen26 - cos20 C) cos20 - sen20 0, eos2 0-sen2 0 ' 2 E) sen20 - tan20 24.- De la figura, hallar la ordenada de "P" A)-1/6 B)-l/5 C)-l/7 D)-^7 n E)->/5/5 25.- Si: 1 + exsec «Axe (-71/6;ti/6) hallar la extensión de "n" B)(l;2> C)(-2;2> D)(-l;2> E)(0;3> Circunferencia Trigonométrica PP7 805
26.-Si: seca - sen6 =1 y Se R a ae <-7t/2; 7t/2) determinar el intervalo de "a". determinar la extensión: de "seca" A) \ 3 3 _ /-2j3 2-J3\ ^3’3/ E)[°.^ B) -2j3 2V3 3 ’ 3 D)[J3,2] 28.- Determinar la extensión de "B" en la si- guiente igualdad: (B G IIIC). 2 sen2 A = tan B + 1 - V2 ; (K G Z) A) Z^ + Kt^-^ + KtA \ o o / B) p^ + 2Kn;^ + 2K7tl Lo o J D) ~+KtA E)^2K7t;^ + 2K^ 29.- Decir cuál es verdadero (V) o falso (F): I. El máximo valor de secx es-1; y <x<^ IX > K 371 II. senx+cosx> 0, para todo: - — <x< -r~ IH. 1 tan jX-, es negativo si: < x < ~ l+tan2x 4 4 A)WF B)VFV C)FFF D)VW E)FW 3j[ 30.- Ordenar en forma decreciente; tan -7-; 4 5ti „ . . tan —; tan 2; tan 4. ., . _ 571 . 371 A) tan 4 ; tan 2 ; tan -y ; tan — Syr 371 B) tan 4 ; tan -z- ; tan 2 ; tan -7- ’ 3 4 Q. 3tt _ 57t tan 4 ; tan -7- ; tan 2 ; tan — 4 3 „ 3tt 5tt D) tan 4 ; tan 2 ; tan . ; tan — 4 3 . 3tt 5tt _ E) tan 4 ; tan . ; tan -z- ; tan 2 ’ 4 3 31 .- Si: ti/2 < x2 < X] < ti; analizar la verdad o falsedad de las siguientes proposiciones. I. sen X] < sen x2 II. cos(-X[) > cos(-x2) IH. tan x1 < tan x2 • IV cot(-x1)<cot(-x2) A)WFV B)VFVF QVWF D)VFFV E)VFFF 32.- Identifica la proposición correcta. A)ae —>cosae [0; 1] B)ae ->cotae <-<=°;0] C)ae (7t; 2tt) —> sen a g [-l;0> D)ae (—\—>tanae <l;0] \4 3/ E)ae (‘2^’°) —>cosae [-1:0) ARACSO P*DITDBBB PP7 806 Problemas de Trigonometría y cómo resolverlos
ÁREASENLAC.T. 33.- Calcular el área de la región sombreada: A)<O;1> B)<O;1] C)[0; 1] D)<0;2> E)[0;2> 34.- Calcular el área de la región sombreada en términos de "a", siendo: OP = PB'. 37.- Si; 0 e ; nj, del gráfico hallar la varia- ción del área de la región sombreada. .. -sena ' 2cosa+l -senacosa B) 2 q 2 eos2 a 2sena+l D) -sen acosa 2sena+l E) l-2cos asena sena 35.- Del gráfico mostrado, calcular el área de la región sombreada. l-sen0 2 + cos6 B) -tan 6 1 + COS0 p. -sen0 9 2+cos0 tan^ +sen0 D)---~ñ------ 1 COS0 p, COS0 9 sen0-l 36.- Evaluar la variación del área de la región sombreada. 39.- Del gráfico, calcular el área de la región sombreada. Circunferencia Trigonométrica
DEMOSTRACIONES 01.- Demostrar las siguientes identidades: a) sen x. cot x — cos x b) cos x. csc x .tan x = 1 c) senz. cosx. tanx = sen2x d) cos x. tan x = sen x 02.- Demostrar las siguientes identidades: 2 a) (1 -cosx)(1 + cosx) = sen x 2 b) (1-senx) (1 + senx) = cos x c) (1-cos2x). csc2x= 1 d) (l-cos x)sec x = tan x 03.- Demostrar las siguientes identidades: a) sen x + cot x. cos x = csc x 2 b) l-2sen x = 2cos“x-l c) sen4 x - eos4 x = sen2 x - eos2 x .. 2 2 2 2 d) cot x - cos x = cot x cos x 04.- Demostrar las siguientes identidades: .4 - 2 4 2 a) sen x+cos x = cos x + sen x b) sen x (csc x - sen x) = eos2 x c) sec x - tan x. sen x = cos x , 2 2 2 2 d) tan x - sen x = tan x. sen x SIMPLIFICACIONES 05.- Reducir: 1 1 secx(l+senx) secx— tanx csc x - cot x senx A)sen x B)cos x C)tanx D)secx E)cscx 06.-Simplificar: 4 4 6 6 sen x + cos x sen x+cos x IV — f— - t— V2-2senxcosx V3 -3sen xcosx A) D)^ O o 73 B) e) + 2-J1 6 Ql/6 07.-Reducir: (I + tan 0 + sec 0 )(1 + cot 6 + csc 0) (l + tan0 + cotO + secO + csc0) A) 1/2 B) 1 C)2 D)4 E)8 08.- Simplificar: „ 2sen3ct—3senct+csca F=------3--------------- 2cos a—3cosa+seca A) tan a B)-cot a C)-tan a D) tan a E) cot a RACSO IDITOlll PP2 808 Problemas de Trigonometría y cómo resolverlos
PROBLEMIZACIÓN CONDICIONAL 09.-Si: sec x csc11 x + sec11 x csc x = sec11 x csc11 x; calcular el valor de «n» para que la igualdad sea una identidad. A) 1 B)2 C)3 D)4 E)5 10.- Calcular a, b y c para que la siguiente expresión sea una identidad: 4 sen x + 3 cos x + 5 sec x + 7 tan-* = = a sen2* + b tan2* + c A)a = b = c = 2 B)a = b = c=l D)« = 1; fe = 2; c = 4 E)« = 1 ;fe = 12;c=4 C)a = 1 ;fe = 12;c = 8 11.- Calcular: P = csc(x) + cot (*), si: sec (*) + tan (*) = n A) ^4 n-1 C) n-1 2 , n +1 B)^ n — 1 n + 2 5 D) E)^£2 n-1 12.- Calcular el valor de «P» para que la si- guiente igualdad sea una identidad: •>7 2'* (sec~*-2) -(csc *-2)“ = p = (tan * + cot *). (tan * - cot *) A)1 B)2 C)3 D)4 E)5 13.- Calcular «A» en términos de 6, si: A( 1 + tan 6) A) -2 sec 6 D) 3 sec 0 sec6 —1 sec6 + tan6 B) 2 sec 6 E) 4sec 6 secO + 1 sec6-tan6 C) -3 sec 0 14.- Sabiendo que fe < a, ¿entre qué valores debe variar «c» para que se cumpla la igual- dad: (fe<o) 2 2 a cot * + fe = c csc * A)(fe;a> B)R-{a} C)(fe;+o°> D)R-(fe;«) E)(-oo;fe> 15 .- ¿Cuál es el máximo valor que puede to- man N = (1 - c) (fe - c) en la siguiente igualdad: 2 7 sen * + 8 sen * cos v + fe eos- * = c A) 13 B)14 Q15 D) 16 E)17 sen*-l 1 . 2—sen* 16 .-Si: ------+ -r- > -=----- senr—2 2 3-sen* para qué valores de v esta desigualdad se trans- forma en igualdad, (k e. Z) A)(2fe+l)n/2 B)(4fe+l)n/2 C)krc D)7i(2fe+1) E)(4fe-l)7t/2 17 .- Si: cot" 6 = 3 sen u, donde: a e . calcular la extensión de: /(6) = 1 -2^3 |cos6| A)[-2;l] B)(-2;l> C)(-2;2] D)(-2;2> E)[-3;2> 18 .-Si: k sen 6 + cos 6=1, calcular «E» en términos de «fe», si: E = [(1 - fe2) tan 6 + (1 + fe2) sen 61'/2 A)2Vfe B)3Vfe C)4jfe D)5-Jfe E)6jfe 19 .- Si: fe sen 6 + cos 6=1 .donde: 6 e ^6; , calcular: E = [(1 - fe2) cos 6 sec‘6 + (1 + fe') sen 6]1,2 A)fe-2 B)fe + 2 C)fe-1 D)fe+1 E)fe2 Identidades trigonométricas PP2 809
20.- Sabiendo que: A) 1 B)2 03 D)4 E)5 sec x - cos x = Ja ; csc x - sen x = Jb calcular: M = Ja cos x + Jb sen x A) 1 B)2 Q3 D)4 E)5 sen O(sen O + cos O -1) *' secO + tanO(cosO-l)-l es equivalente a : a sen O + b cos O + c, calcular:/? + b + c A)1 B)2 03 D)4 E)5 22.- Si tan x = 1 - sen x, calcular: sen x cos x A) + 1 B) J2 -1 O Jí Ji D) -1 E) + 1 23.- Sabiendo que: a sec x + b cos x = b, calcular: M = 1 + cos x - eos2 x * \ a *4" b \ i a "4" b A) —B) a - b O 2 D)^p E)2fl-¿ 24 .- Si sec x = 1 - sen x, 3 calcular: M= * 1 + sen x A)1 B)2 Q3 D)4 E)5 3 *7 25 .- Sabiendo que: tan x + tan*x + tan x = k 3 Q calcular: m = Acot x - cot*x - cot x A)-2 B)2 OI * D)-l E)0 26.- Si: -Jsena-1 = tan20 + cos20 calcular: M = sen2 0 + sec2 O + eos2 a 27 .- Si aumenta en 3 la secante de un ángulo se obtiene la tangente de su suplemento au- mentado en 5. ¿Cuánto vale la cosecante de dicho ángulo? A) 13/5 B)5/3 0-5/3 D)-13/5 E)-5/4 i— 7 28 .-Si: V2 +senx+sen x- 1=0 calcular: N = cos3 4x + cos2x + J1 senx A) 1 B)V2 Q2 D)2V2 E)4 29 .- Si: tan a + cot a = 4; calcular: 3 3 71 N = tan a - cot a; siendo: O < a < -r 4 A)-15-/3 B)-30V3 C)-25V3 D)-24V3 E)-12-/3 3 30 .- Si: sen"x+ senx - % ~ O 2 3 2 calcular: D = cos x - tan x 4 A) 1/4 B)0 03 D)2 E)-5/4 31 .- Si: cos O - cot 0=1 calcular: M = cos 0 + tan O A)J2 B)-1+J2 C)2J2 D)2+V2 E)-J2 32 .- Calcular: tanx + tan"O; si: . sec20.sen2x—cos2x tan O =---:---------------- 1—sen xcosx A) I B)0 0-1 D)2 E)-2 33.-Siendo: P = l+2sec20tan20—tan4O l+2csc2 6cot2 O—cot4 0 y además: sec4O - p = A + B tan20 Problemasde Trigonometría y cómo resolverlos RACSO BDITOtlI
calcular: A + B A)1 B)2 C)3 D)4 E)-2 M-Si:xe^-.2/.y Icos3 x(l+cosx)— sen3 x(l+senx) V cosx—senx = a + b sen x + c cos x 39.-Si: sen46+cos46 sen60+cos60 m 2 2 evaluar: E = sec 0. csc 0 2-3m 2-3in A) — B) ----------- 1+m 1-m D) 1 + m E) 1 - m ELIMINACIÓN DE ARCOS C)2-3ni calcular:---r— a+b+c A) 1/6 B)3/4 Q5/4 D)7/4 E)9/4 35.- Si: a sen6x + b cos4x + a cos6x + b sen4x es independiente de "x", hallar cos 6, si: cot 6 = 4b2—2a2 Jo jo A) B) O ±fí •J2 16 36.-Si: 3 4 <x<7t y A = sec2x+csc2x-4 l-4sen2xcos2x A determinar el valor de:------- cscxsecx A)-l B)1 Q-2 D)2 E)3 37.- Si: tan a=---¿ 9-1 evaluar: E = sen2a + p sen a cos a + q cos2a A) \!q B)2/q C)q D)2q E)-q 38.- Si secx. csc x = 2-J2 ; calcular: ,,8 8 V = sen x + cos x A) 17/32 B) 32/17 C) 15/71 D) 23/17 E) 71/32 40.- Si:tanO + (p + l)cotO=p.(I) (sec20+csc20) (l-cot0)2 =q..............(II) al eliminar ”0” se obtiene: A)p2-9 = 0 B)p-<? = 0 C)p + g2=l D)2p2-9 = 0 E)p+q=2 41.- Si: csc 0 vers 0 = m , sec 0 . cov 0 = n identificar,/, cuál de las siguientes es una ex- presión independiente de 0? A) m + n = 1 + mn B)m-n=i- mn 7 9 C) m + n = 1 - mn D) ni - n~ = 1 r-. 2 2 , E) m +n =1 42.- En base a las siguientes igualdades, se pide determinar ¿cuál de las siguientes corres- ponde a una expresión obtenida de estas e independiente de "a y 0": 2 2 sec 0 - sec a = p cos'O + cos a — n sen20 - sen2a = m A) 16 m = (n2 - m2)p2 B) 16mn = p\m' - n2)2 C) 4 mn = p2(m - n) D) 4mn = p'{m' - mn') E) 4 mnp = m2 - n2 Identidades trigonométricas PP8 811
SENO, COSENO DE ARCOS COMPUESTOS 5V3 01.- Si: tan a = 4-J3 y tan b = -pp, siendo a y b ángulos agudos, calcular: P — sen(« + b) cos(« + b) B)-l/2 Q--J3/4 D) V3/3 E)-V2/4 02.- Si: J3 sen 0 = 2 sen(0 + A) - cos 0 cos 0 = 2 cos(0 + B) + V3 sen 0; A, B e IC Calcular: sen(B - A) A)-l/2 B)l/2 C)1 D)-l E)0 03.- Si: x-y = ^ f senx+cosy+cscy Hallar: M = -----------r.-----eos y I versx—cov v+ tanx—cot y l J A) 1/2 B)1 C)-l D)2/3 E)-l/2 04.- Dada la expresión: (3 + sen 84°)sen ^-2o) + cos 6o = 1 Hallar: l+tan23° ^cscO-tan® A) 2 tan 20 B) 1 - cos 20 C) 1 + cos 20 D) -2 tan 20 E) 2 sen 20 05.- Si: 9 sen 0-40 cos 0 = 41, Evaluar: cos(0-127°) .. 156 158 161 A) 205 B>205 C) 205 163 r-x 164 D> 205 E) 205 A 06.- Calcular aproximadamente: A = cosí 11° B = tan2° 231 A)-W m 233 B)’ 25 C)- — ' 25 D)-^1 > 25 243 E>-^¡ 07.- Sabiendo que M e R . Además: senl45°cos65+senl75°cos85 M > cos700sen400+senl700cos800 Hallar el mínimo valor entero que toma "M". A) 1 B)2 C)3 D)4 E)5 08.- Calcular aproximadamente el valor de la siguiente expresión: sen2°tan3°+cos5°csc87°T0™ L cos2°cot4<>-cos60csc4<> J A) 101 A) B) 102 C) 103 D) 104 E) 105 09.- Si a + P + 0 = 37t; además sec2 0- tana.tanP.sec2 0 -----------------------> A sena.secp.sec0 si RACSO PIDITOKEB PP9 812 Problemas de Trigonometría y cómo resolverlos
Hallar el mayor valor entero que toma "A". A)-3 B)-2 C)-l D)0 E) 1 10.-Si:M = ) - cos(x+y) sen x A) 2 B)-2 C)3 D)-l E)1 15.-Si: tana+tanP = «............(1) y; cot a + cot P = fe.......(2) Reducir: 2/ además: cos y = ; y e ^0; Hallar: N= 5^3 M A) 5 B)10 C)15 D)25 E)30 11.- Si: sen 77° + sen 7° = n Hallar en función de n. P = cos 24° + cos 46° + cos 50° - cos 60° A) y B) | C) y D) y E) y TAN, COT DE ARCOS COMPUESTOS 12.- Si: Vl+sen26 = -y- 6 g IC Hallar:c°t2^y) - 1 2 1 A) y B)2 C)1 D)y E)0 2 2 13 .- Si: cos x tan(y + z) = sen x cot(y - z). Hallar: tan z A) tan(y - x) tan(y + x) B) cot(y - x) cot(y + z) C) tan(y+x) cot(y - x) D) cot(y + x) tanfy - x) E) tan(y - x) cot(y - x) 14 .-Reducir: tany^tan-^+ tan-^ + tan^ p _____1 o_1 o_____lo____1 o (2+V3)tan^ 2 2 M = b(tan a + tan P) - afe(tan a + tan P) + 2a A)2a B)fe C)0 D)-l E)-2 16 .- Si: tan a + tan P=tan(a + P)- tan a tan P y tan a - tan P = tan(a - P) 4y tan a tan P Calcular: P = tan 2a - tan 2P A) 1 B)| Q-y D) g E)-| 17 .- Si: sen(a + 3P) sen(a + P) = tan2(a+ 2P) sen2Pcsc2(a+2P) Hallar: E= “Í+’sec(a+2P) +«ec(«+2P) A)-l B)1 C)2 D)3 E)4 IDENTIDADES ESPECIALES 18 .- Si se cumple: 3-2 sen x = J5 cos x además: x 6 (0;—) Calcular: L = csc x - sen x A) 2/5 B)3/4 C)3/5 D) 1/6 E)5/6 19 .- Si: /(x) = sen x + 4 cos x. Hallar la variación de/(x), si además: x G ^0;^ A)(1;V7) B)[2;4] C)[3;5] D)(l;Vñ] E)[2;5] 20 .- Si: sen a + sen 6 = a a cos a + cos 6 = fe Hallar: tan(a + 0) Identidades Trigonométricas de Arcos Compuestos PP9 813
B) a2+b2 a2-b2 _ lab C)7^ 25.-Siendo: («*«-«** \cosx cotx, 2 2 = cot a - cot b a+b b) b2-a2 21 .- En un triángulo ABC, reducir: K=(cot A+cot B) (cot A + cot C) (cot B + cot c) A) sen A sen B sen C B) cos A cos B cos C C) tan A tan B tan C D) cot A cot B cot C E) csc A csc B csc C 22 .- Sabiendo que: 7t<P<27t ; <0<7t ; a-0 = n Además: tan 6 + tan 0 = tan a + tan <J> Hallar: M = cos(a + P)-cos(0 + <|>). A)0 B) 1 C)-l D)2 E)l/2 23 .- Calcular el valor de ”6" si 0 e [270°; 360°] para que la expresión: E = Vó (sen 6 - cos 6) - 3 cos 45°(sen 6 + cos 6) tome su mínimo valor. A) 280° B)240° C)300° D)320° E)345° 24 .- Hallar la extensión de: 2 E = tan 4x - tan 8x + tan 4x tan 8x faz ít\ s,xG\4’2/ A) <0; 1) B)[0;l] C) [0; 1) D)<1;2> E)R-{0} Hallar: sen x B) tan b cot a D) tan a cot b A) tan a tan b C) tan a + tan b E) tan a - tan b 26.- Dado: seca cosfy + 6) = 1 + tan y tan 0 sec0 cosCy - 0) = 1 - tan y tan 0 Halar: cos (y+ 0) 2cosPJcosa Jcosa+^cosP 2sencc-JsenP Vsena+^/senP 2cosa.Jcosp Vcosa+^cosP 2sen0jsena Jsena+^senp Ideosa. cos0 Jcosa+^cosP SITUACIONES GRÁFICAS 27.- De la figura que se muestra se tiene que: tan a = «; tan 0 = b, luego tan x vale: a\ b a a + b a-b D) H E) ' a-b a+b 28 .- Del gráfico mostrado, hallar: tan a cot P RACSO BD1TOKCS PP9 814 Problemas de Trigonometría y cómo resolverlos
14/9 B)14/11 C)17/9 D) 17/11 E) 19/11 29 .- Determina cot a en la siguiente figura, si BD=lyBC = 6. a)3¿ B)^ qU£ D)l^ E)Jig 30 .- Si: ABCD es un cuadrado. Hallar el máxi- mo valor de tan 6. A) V3/3 B) V3 C)1 D)3/4 E)4/3 31 .- De la figura hallar el equivalente de: tanP+ tanO P = tanP-tanB en términcs de "*" A) 2 cos 2x •J D) 4sen“jr B) 2 secx 4 csc2x 2 J3 E) - esc 2x IDENT.TRIG. DESUMA DE3 ARCOS 32 .- Sabiendo que: a sen x + b sen y + c sen z = 0 a cos x + b cos y + c cos z = 0 Hallar: M = sen(x - z) csc(y - z) A) afb B) tía C) -alb D) -tía E) ale ,, tan<z-cot<z-2cot2¿> 33 .- Reducir: P - +tan¿,)(cotfl+cot¿) A) tan(¿? + b) B) -cot(« + b) C) cot(« + b) D) -tan(« + b) E) -cot(« - b) 34 .-Si: 2 tan x + tan 2x + tan 3a tan x(tan 2x + tan 4r) senA(x +v) + tan 4y es idéntico a: “sBvcosBy Hallar: A + B A)8ó0 B)±8 C)8 D)0 E) 4 35 .- Sabiendo que: tan 0p tan 0-> y tan 03 son las raíces de la ecuación: ax~ + bx~ + cx+d=0 Calcular: P = tan(6, + 6, + 03) A) — B) C)^^ a-c a-c a+c ™ db a+b+c+d ac abed Identidades Trigonométricas de Arcos Compuestos PP9 815
CASOS DE REDUCCIÓN 01.- Simplifica: T= tan(7t+x)cos ^3^+xj tan(270°+x) csc(360°-x) A) -sen x B) sen x C) -tan x D) tan x E) 1 02.-Simplificar: N=cot +x j sec(2ít- x) tan ~ 2 j cos(4it +x) A)-2 B)-l C)0 D)1 E)2 03.- Reduzca; cot(360°-x)-tan(450°—x) P= tan(270°—x)—cot(L 80°—x) A)-2 B)-l C)0 D) 1 E)2 04.-Calcular Wsi 0= sen(l 80°+0)cos(0-90o)tan(1260<>+0) W " cos(270°—0)sen(54Oo+O)tan(45O°+O) A)-4j3 B)-4 Q-3-J3 D)3 E)-l 05.- Si: tana= V2 ; calcular: sen(-a)cot(270°+a)sec(l 80°+a) M= cos(360o-a)tan(a-270<,)csc(a-180°) A)1 B)3 Q4 D)-3 E)-4 06.- Reducir: J=tan 181°+tan 183°+tan 185°+... +tan 357° + tan 359° + tan 361° A) 1 B) tan Io C)-tan Io D) -l E)0 07.- Si x+y = 4ít, calcule: K = tan(x +10°) + sen(y + 40°) + tan(y - 10°) + sen(x - 40°) A)0 B) 1 C)2 D)-l E)-2 08.- Calcular: A = 2 sen 330° + 4 cos 120° - csc 1050° A)-5 B)-4 C)-3 D)-2 E)-l 09.- Hallar el valor de: _ l(lt\ 3pít\ 3Í47t\ 3Í6ít\ E=cos lyl + cos lyl + cos ryj+cos lyj A) 1 B)2 C)4 D)-3 E)0 10.- Simplificar: (x->')2secl620“+(x+v)2senl890p-Zvycosl260p ---——-----------— -----------—-------------- (x+^secSKT+íx2+y¿ )cos900°+3xycscl350f> A)-2 B)-3 C)-4 D)-5 E)-6 11.-Si a + b = 2ít, calcular: R = tan a + sen a + tan b + sen b A) 0 B) 2 tan a C) -2 sen a D) 2 sen a E) -2 tan a 12.-Si: tan 36 Io tan 363°.... (10 términos) = K Calcular en términos de K. N = cot2 181°cot2cot2°cot2 183°...cot2 190° A ACSO Weditciei PP10 816 Problemas de Trigonometría y cómo resolverlos
A)K2 B)¿- C)2K2 E)^ í\. Z.XX. xx 13.- Simplificar: cos(6+ 20°)sen(a+P)ctg(2a+20+30) N4 = ---------------------—--------- sen(70°—0)sen0tan(342O°—0) si: ct+ 0 + 0= 180° A) 1 B)-l C)sen20 7 7 D) -eos 0 E) -cor© 14.- Hallar "a" si se cumple Jse^a+^)cos(a-l 947^)] ^cos<330’t+x)+sen(x_2Z92) (Ke Z) A)Kn B)Kn/2 C)(2K+1)^ D)Kít/4 E)(4K+l)-j 15.-Reducir: Sen( 2 ~1) cqs(1998ti+2) tan(nsec60°+3) K-cos(67t-l)+ Een^> 3ti) +cot(ítver60°+3) A)tanl B)tan2 C)-cot3 D) 1 E)-l 18 .- Calcular el valor numérico de la siguiente expresión: cos^tani^+cot^i^ E=_______2_____3______6 sec-y^ - tan í ^^lsen2 6 \ 3 / 4 A) 1 B)2 Q7/3 D)3/7 E)2/5 19 .-Reducir: sen-^+sen^pp-cos^é^ 11 11 i • f • TJtan-^cos^ A) 1 B)2 C)3 D)-l E)-2 20 .-Reducir: 2 tan (y+©jsen - ©jsec^+©j cos(7ti+ 0)csc(57t-0)ctg(l 7ti+0) A)-2 B)2 Q1 D)-l E)0 21 .- Indicar verdadero (V) o falso (F), las si- guientes proposiciones (n G Z). I. sen(n7t + 0) = (- l)n sen 0 EL tan(nrt - 0) = -tan 0 ni. sec(n7t-0) = (-l)n+1sec0 sen(148ít+0)cos(Í^-0)cosÍ0+^i^) 16.-Si:-----------2--------í---------U tan2 (19^j+sen1997t = Ksen3(2B0) Es una identidad, hallar: "K + B”. A) 1/4 B)±l/6 C) 1/6 D)-l/6 E)±l/4 17.- Reducir la siguiente expresión: tan(y- x-7t)tan( z- y+7t)tan(x- z+ 2ti) tan(x-z+ti)- tan(x-_v+ti)-cot(z - y + — j A) 1 B)-l C)2 D)-2 E) 3 A)WV B)FFF QVFV D)WF E)FW 22.- Si: cot 0 =-r¿- a 0 G 15 \ 2 j Calcular: E= ¿ 2(sen|cot| A)V2 B) D)-V2 C)-l Reducción al Primer Cuadrante PP10 817
CASOS PARTICULARES DE REDUCCIÓN 23.- Si: sec(a- P) = sen810° „ cos[(csc300)Aít+a] cos[(csc2 315°)k ít+0] A) sen 180° B) sen 270° Q sen 450° D) sen 0° E) sen 190° 24.-Si: a 6 UC Simplificar: í |sen(900'>-a)|+|csc(270°-a)| A P = cosa^|senq2600+a)+|cos(6300+a)| J A) cos a B) -csc a C) sen a D) -cos a E) csc a 25.- Si a es complementario y coterminal con x e y respectivamente Calcular el mayor y el menor valor de: E = 2sena+(-l)psen x senasenp(a-y+(2n+l)^ ; n e Z A)3;l B)3;0 C)3;-l D)3;-3 E)0;-l 26.- Calcular: M= £sení^p+2A:ít+a) k=l '4 ' A) 1 B)0 Q-l D)-2 E) *£ 27 .- Si A y B son complementarios, simplificar: 3sen(2A+3B)tan(4A+3B) p = ---------------------- tan(2 A+ 3B)cos(3 A+ 2B) A)-2 B)-l Q2 D)-3 E)3 28 .- Reducir: 6 E= £(-l)n n=l A)2 B) 1 Q0 D) 1 E)-2 29 .- Si 0 6 IIC, hallar el signo y la expresión simplificada de: csc(7ít-0)tan(o-i^j.sec(O-72ít) cot(0-19ít)sec2^y^-oj.cosn A) + ; tan 0 B) - ; tan 0 C) + ; cos 0 D) - ; cos 0 E) + ; cot 0 sen(l 38o+0)tan(298o-0) 30-' S1: 2sen(42o-0)cot(0-28o) = sena Además "a" pertenece al segundo cuadrante. Calcular el valor de: E = tana + seca J3 r- A) - B)-V3 C)-3 ... 2^3 „ V3 D)_r E)v 31.- Calcular: sen^sen^sen^ 2cos~ E=________4___5____7_+______3 sen^sen^sen” cos^ A) 1 B)l/2 Q2 D)3 E)3/2 32.- Reducir: 2tan(4A:+l)ít+3cot(4A -l)y S=------------------------— 3sen^+2cosAít O Si: (ke Z) A)2 B) 1 C)0 D)-l E)2 21ít 33.- Si: x - y=, calcular el valor de: E = csc(tanx) - csc(cot(-y)) A) tanx B) 2csc(tany) C) 2csc(cotx) D)cot(-y) E)0 PP10 818 Problemas de Trigonometría y cómo resolverlos -a RACSO P*DITO*BI
SITUACIONES GRÁFICAS 38.- Del gráfico siguiente, calcular: 35.- Del gráfico, calcular "tanO" si PQ = QO y mZ. CBQ = wiZAOQ. Además O': centro de la semicircunferencia. A) V2 B)-l C) V3 D)2 E) 4$ L = 8 tan 0 - 21 cot 0 39.- Dada la figura, hallar: tan 36.- Del gráfico mostrado hallar: cota + tana B) C) — 7 b D)^ ab A)-3 B)3 «I D) 3 E)2 40.- Del gráfico, hallar M = tan a + cot a. F. a2+b2 37.- Del gráfico que se muestra el triángu- lo ABO es isósceles. (BO = AO = 8) y la altura relativa a un lado igual es 6. Calcu- lar: M = cota-cotp. A) 13/6 B) 17/6 C) 19/6 D)23/6 E)35/6 Reducción al Primer Cuadrante
PROBLEMAS CONDICIONALES RELACIONESFUNDAMENTALES . sen2acosa 01.-om1Fl.Ged1. E- (1+cos2a)(i+Cosa) A) tana B)tany C)cota D)coty E)1 02.- Determina el máximo valor de: 07.- Si tanO = |; 0 e ^7t;^ Calcular: E = 125 cos (24) sen^® A) 36x5 B) 36 VÍ0 C) 12-^0 D)20VÍ0 E)40V3 l-cos2x_____ 2+-j2(l+cos2x) A) 2 B)1 C) 1/2 D) 1/3 E)0 03.- Si: tan2x = 2tan2 y + 1 2 calcule: E = cos 2x + sen y A)0 B)1 C)-l D)l/2 E)-l/2 04.-Si: secO 2 cscO , , —.calcular: 3csc20+2sec20 sec20.csc20 A) 1 B)2 C)3 D)4 E)5 2 05.- Si se cumple: tan x + tanx = cos 2x Calcular: sen 2x - cos 2x - tan3x A) 1 B)2 C)3 D)-2 E)0 08.- Si: tan -10°) = 2; calcular el valor de: E = tan(25° + 0) + tan(0 - 65°) ; 90<0<180° a x 48 .j. 48 50 . 50 y—* _ A) y B) y C) y D) y E)7 09.- Si: 2sena = senP + cosp, calcular: ^_cos2(45°+P) cos2a A) 1/2 B) 1/3 C)1 D)2 E) 1/4 10.-Si0e UC, tal que: cos4(—+Ateos4 \4 / \4 / -------=------------= sen 6 3-cos4x 06.- Reducir: N = sen(a-P) A) sen(a+P) B) cos(a - P) C) cos(a + P) D) tanfa + P) E) sen(a - P) 1-^sen2 2a—sen2 P-cos4 a Hallar: sen 20 A) 1/4 B)^ VÜ5 1-x V13 D> ‘ 8 E)' 4 RACSO • DITOBll PP11 820 Problemas de Trigonometría y cómo resolverlos
11.- Si se verifica que: cos6-sen6 + l-cos46 cos6+sen6 sen 46 — Hallar cos 40 A) 1/2 B)-l/2 C)1 D)-l/5 E) 1/3 2 3 12 .- Si: senx + sen 2x + sen x es equivalente x. ascnx+bsencx-dcos4x+d Calcular: a + b + c + d A)2 B)3 C)4 D)5 E)6 13 .-Reducir: A = (3 - 4 sen20)(4 cos20 - 3) - 2 cos 40 A)1 B)2 C)3 D)4 E)5 14 .-Si: tan 2x cot x+tan4xcot2x = a tan 2x tan x + tan 4x tan 2x = b calcular una relación independiente de x entre ay b. A)a = b B)a-h=l C)n-h=4 D)a2 = h2-1 E)a + h=l 15 .- Si la expresión: (tan 2x + tan 8x)(csc 8x - cot 8x) es equivalente a: sec ax + sec bx + c, además: b<a <0. Calcular: a + b + c. A)-16 B)-15 Q-14 D)-13 E)-12 16 .-Si: sen4x + cos4x - tn ¡ \ xe (77.it) 6 6 \2 / sen x+ cos x = n 17.-Si: cov(nx) sen4 eos4 (^) — a; calcular: sen(nx) l+cos(nx) A)l/« B) l-« C)^ ' 1+n D>¿ 2(1^ 1-a 18.-Si:tanx = 1 Isen2xcos2x 1-1 lsen2x . calcular: 2cos2x-l l+tan2x A, 11 dx 2 „ £ n. 10 1 A) y B)~5 O-jo D) [i ^)"9 19.- Reducir la expresión. r- senxcos3 x-cosxsen3 x sec4x A) sen 4x B) J sen 4x C) 4 sen 8x 4 o D) sen 8x E) .eos 8x 20 .- Si se verifica: cos30 t sen30 _ £ csc30 sec30 ” 4 Hallar el valor de "cos 80". A) 1/7 B)3/4 C)l/9 D)8/9 E)7/9 21 .-Si: Calcular sen2x en términos de m y n. A)-2^y^ B)-2. l2+m-n 1 2 COs6(^+B) +cos6(^—®) = m + n sen(p6) C)-2^^^ D)-3. l2+m-n 1 2 considerando: p > 0, entonces el valor de: m + n+p. E)-5 D)5 E) 6 A) 2 B)3 C)4 3-m-n 3 identidades Trigonométricas del Arco Doble PP11 821
f 4 tan x “i-1 . r xx 22 .- Si: :-----= tan (x + <p) 4 - tan x Calcular: tan 2<¡>, (<¡>: agudo) A) 8/15 B)4/15 C) 15/16 D)4/7 E) 1/2 23 .- Si se verifica: tn csc x = n csc y . , , 371 Ademas: x - y = - Expresar: E = csc(2x + 2y) + cot(2x + 2y) en términos de "tn" y "n". nzn tnn 2tnn A) 2 2 tn n B) m+n tn-n 2tnn _ 1 1 D) i 2 tn -n E) 2+ 2 tn n VARIACIÓN DE EXPRESIONES 24 .- Calcular el máximo valor de: 3 3 cos a. sen a - sen a. cos a A) 3/4 B)2/5 C)l/3 D) 1/4 E)l/2 25 .- Calcular el máximo valor de la siguiente expresión: E = sen3x+ cos2x+ 2senx(l - cos2x) A) 17/8 B) 17/2 C) 17/4 D)15/8 E)15/2 26 .- Hallar la suma de los valores máximos y mínimos de la siguiente expresión: E = M eos2 j + N cos x. M, N son constan- tes reales. A) N B)M C) N/2 D) M/2 E) 0 IDENTIDAD!^ AUXILIARES sec2 6(csc40+cot46) 27 .-Simplificar: W=------------------ A) csc 20 B) sen 20 C) tan 20 D)cot 20 E)sec 20 28 .- Reducir: R = 3 tan220 tan 60 + 3 tan 20 - tan320 para0= A)0 B)1 C)2 D)3 E)4 29 .- Reducir: E = sen6x+sen3x cos2^ 9x .csc~2 A)cos l J 3x B) sen C)tan^ D)sec^y rx x E)CSC 30.- Reducir la expresión: E= (cscrx-tan^|-2sen2y)cos2^| 1 2 1 2 2 A) 2 eos ct B) 2 sen a C)cos a D) sen a E) sen y 31.- Calcular "tan 2x" de modo que: 2 2 sen x - sen x cos x - 2 cos x = - 0,5 A)-l B)-2 C)-3 D)-4 E)-5 4 4 sen r+cov x 32.-La expresión:-----7----?— se puede ex- sen +cos x presar de la forma (a + i>cos4x)/ (tn - ncos4x) y RACSO BDITOBB1 PP11 822 Problemas de Trigonometría y cómo resolverlos
se cumple que: sec <¡) = J—+^ - ¿Qué valor V m+n /Q'TT \ tiene <¡>), si <¡)G x 771 n\ 1 571 yx\ 161T 17tt A) V B) O -f D) ~9“ K) 9 __ „ , , , „ sec2z-sec2y 33 .- Hallar el valor de: E=-=-- secz»v-sec2x si se verifica: sen(y + z) ,csc(»v + x) = sen(w - a) . csc(z - y) cos(w + x).sen(z - y) - cos(z + y).sec(w - a) A)-l B)1 Qcero D)2 E)-2 34 .- Sabiendo que: tan x tan 2x + tan 2x tan 3x = n obtener "tan 3x cot x". A) ^+3 B)n+3 C)4n +12 D)2n + 8 E) ~ +3 35 .- Siendo "x" e "y" ángulo agudos, tal que: 2[tanx - tan(x - y)] = sec x. sec y Calcular: sen(x+y) A)| B)1 C)^ D)^ E)j 36 .- Hallar la suma de los "n" primeros térmi- nos de la sene: S = tan 0 + 2 tan 26 + 4 tan 46 + 8 tan 86 + ... A) cot 6 - 2nlcot 2nI 6 B)tan6-2ncot2n0 C)cot6-2ncotn6 D) cot 0 + 2" 1cot2D0 E)cot6 + 2Dcotn6 37 .- Reducir la siguiente expresión: _ 3csc36°-2cot36° L, _ ---1--------- 5tanl8°+cotl8° A) 1 B)-2 Ql/2 D)-l/2 E)2 SITUACIONES GRÁFICAS 38 .- La parte alta de una estatua es vista des- de un punto en tierra con una ángula de eleva- ción a. Cuando el observador se ubica en el punto medio de la línea que une el pie de la estatua con el primer punto de observación esta vez es (90° - 2a) el ángulo de elevación del mismo punto de vista de la estatua. Calcu- lar "cot a". A)V2 B)y Q-J3 D)J4 E)J5 39 .- En la figura mostrada, calcular la varia- AB ción de sabiendo que AD = DC y 0 < 6 < 3(T. A)(0; 1> D) (0,^) C) ) B) (°^ E) (O; Jí) Identidades Trigonométricas del Arco Doble PP11 823
REIACTONES FUNDAMENTALES i 6 i 01.-¿Cuántos valores admitecosl — I?; a partir A) 2 sen — 2 B)2cos — C) -2 sen N> I Q de: 5 sen 0 = sen y D)-2cos — 2 E) tan - A) 1 B)1 C)3 D)4 E)5 05.- Hallar el valor de cos 247° 30’. 119 ___ 371 02.- Si: cos 20 = - ; Tt < 0 < —, calcule el valor de k, siendo: 2 2 k - 7 sen 0 2 -4 cos o 2 2 03.- Reducir: sen jc + cosjc—1 R=-----. vjl—sen x A) 2 sen — 2 D) - 2 cos — 2 04.- Reducir: W = Vl+senx Vxe D)V11 E)V13 —<J2 + V2 2 06.-Si :0< x <7t/2,además: ; re 7171 4’2d 2 Obtener: sec — 2 B) 2 cos — C) -2sen - A) 1/2 B) 1/4 E) tan - 41 - tan — 2 Ql/6 D)l/8 E) 1 07.- Sabiendo que: 3n/2 < 0 < 271 Hallar :K= +cosjr + i 1 _ e e —+—cosO + sen — + cos — 2 2 4 4 1 1 — + — 2 2 11 —+—cosO - 2 2 PP12 824 Problemas de Trigonometría y cómo resolverlos
11.- Reducir: A) sen— B)sen— C)2sen — 4 2-4 e D) 2 sen — E) sen 6 08,-Si: 180°<6< 270°ycos26 = 16/25, calcular: csc + sec e —+ 71 2 tan x sec 2x—csc 2x+cot 2x R. — tan 2a— tan x A) 2 sen2x B)2 senx C) 2cos2x 2 D) tan x E) tan x sen x FÓRMULAS RACIONALIZADAS 12.-SixG (0;7t/4),reducir: B)- 3 7 °2 18 D)-y 4JÍ0 E) 3 R = a 09.- Si: cos x -- —- b+c b A) 2^2 B)-2/2 Q1 ; cos y =--- a+c D)3 E)2 , entonces: 13.- Determina el intervalo de «E», si: , 2 tan COS Z = ---- a + b es igual a: A) (o + b + c )- 1 x 2X E = cosx cot — -2 cosx cos — .cotx 2 2 A) [-1; 1/2] B)[-l/2;0] C)(-I/2;l> D)(0;l> E) [-1/2; 1/2]-{0} B) a + b + c 14.- Calcule el máximo valor de: C) (2a + b + c)(a + b + c) -1 D) (a + 2b- c)(a + b + c) - 1 E = sen E)1 10.-Si:7t<e<37i/2 y cos26= 16/25 cot--tan— 2 2 cot —+ tan— l 2 2) + cos (cos x) e ~ 6 Calcular: csc — +sec I —+ 7t 2 2 2 A) y¡2 B)1 02J2 E) 1/2 B)- 3 7 C)I 18 D)? 4J10 E>— 15.- Calcular el máximo valor de: E = cot<¡>-cot<]>/2 ; <¡)G(0;7t) A)-l B)-2 C)-3 D)-4 E)-5 Identidades Trigonométricas del Arco Mitad PP12 825
2 x 2 y 16.- Si: (1 - a)tan - = (1 + «)tan ; 2 calcule: N = (l + acosx)(l -acosy) + a A)0 B)1 C)2 D)3 E)4 17.- Reducir: Vi + sen40° + Vi-sen40° K“ Vl + cos50° -Vi—cos50° A) tan 10° B) tan 20° C) cot 10° D) cot 120° E)1 18.-Si: sec a = cot p cot 0, calcular el máximo valor de: E = cos (p + 0) sec (p - 0) /n 3n si además a G \2 4 A)V2+2 B)3+2V2 C)2+ 4Í D)2+2-^2 E)3 + V2 2 2 (71 19.- Si:/(0) = csc 0 + csc 20 evaluar:/I 2 22 .- Si : (1 + n cos Ct)(l - n cos P) = l- n ; Determinar A en: A)-l B)1 C)2 D)-2 E)0 23 .- Reducir: E = 3 cot 0 - csc 20-5 cot 20 A) tan 0 B) 2 tan 0 C) 3 tan 0 D) cot 0 E) 2 cot 0 24 .- Si se cumple que: 1 - a cos x l+o 1-a ~ 1-ocosy 2 x 2 y Obtener: tan — .tan — 2 2 A) 3a B) 1 + a C) 1 - a A)2 B)3 C)4 D)5 E)7 20 .- Calcular el valor de: 2 1971 2 2071 2 2271 E = sec sec sec —— A) 160 B)62 C)64 D)65 E)68 C- .2 > ,2 Z 21 .- Si: cot — = cot — cot — 2 2 2 Reducir la siguiente expresión: cos y + cosz—cosx F=------------------ cos x.cos y.cos z A)-l B)1 C)cero D)2 E)-2 ( a + 6^ 25 .- Un valor de tanl Icos a para que 6 3 a tan — sea tan — es: 2 2 A) 2 B)13 C)0,5 D)0,3 E) 0,2 PROBLEMAS CONDICIONALES 26.- Obtener “ sen 40” en términos de “u” si se cumple: 2 csc 40 = 1 + n cos 0 + cos20-cot0 + cot40 ARACSO wpt DiroiBi PP12 8261 Problemas de Trigonometría y cómo resolverlos
4 A) — n 2 b>t n2 +2n O — 4 4 D) 2 n + 2w 4 E) 9 n — 2 27.-Si: 0 « 2 6 cote 1 cos 6 cot — - 2 cos 6 cos ~ — — 2 2 4 Hallar el valor de: E = 2 sen 26- 1 A)-l B)1 C)2 D)cero E)-2 28.-SÍ se verifica: 1-secysecz seca- sec(Jt+z)_sec(Jt_.y) • / 3tt) ademásx, y a z pertenece al intervalo (n’ Hallar “cotí T I” en términos de “z” e “y”. y z A) - cot — tan — 2 2 „ y z B)tan— tan — 2 2 C) tan y tan z D) - tan y tan z 30.- Siendo: csc x - cot x = 3 f 45° x A Calcular: tan —--- , indica un valor. 12 4 1 41 <— A)41 -2 B) -— C)V2 -1 4 D)V5+2 E)^ 31 .-En la ecuación: Gsenjr + ¿>cosjr = c ; tan a/2 admite un solo valor: (a + b)2 —c2 Hallar: 2---------- ab A) 1 B)2 C)3 D)4 E)5 32 .- Si: a; <¡) y 6 son ángulos, que cumplen: 2 2 2 eos a-cos 6 = cos <¡), además: 4x~l y[x -Jx + 1 senct=-------;senó=------;sen6 =------- 2 2 2 .. y li) - tan — tan z 29.-Si: sena = sen p+sen 6 1+sen Psen 6 (n fu Hallar: E = cot —+— cot —+— ^4 2) ^4 2) en términos de “( a”: (n ct'l A)'CO\4 + 2j B)cot -+— (4 2J fn a'' 1 (n a) D)tan — + — l4 2) 1 l4 2 J (n a") E)±cot|j+-J Calcular: tan — +2 tan — + 3 tan — 2 2 2 A) 42 B)2V2 C)3j2 D) V3 E)2-j3 Íx + a] íx—a A jb —-— tan —— =tan — 2 I I 2 I 2 COSA entonces------ es igual a: cosa A) cos b B) sen b C) sen a D) cos 2b E) cos a Identidades Trigonométricas del Arco Mitad PP12 827
34.- Si: x + y = z; Hallar E en función de z: 2 2 2 2 E = 1 - sen x - cos y + 2 sen x cos y + 2 sen x sen y cos x cos y 2 9 2 A)cos z B)tanz C)sec z 7 2 D) cot z E) sen z 2 35.-Siaestalque7i/2<a<7tysen a= 16/25; hallare! valor de: I-- 7 x P = 7v 65 + 130 sen — A) 61 B)62 C)63 D)64 E)65 39 .- De la figura. Hallar sen a, sabiendo que los segmentos AD y CD están en la relación de2a5. (sen 2a)(sen ot/2) 48 24 24 f- A) 125 B) 125 C) 25 A) 1/2 B) 1/3 C) 1/4 D) 1/5 E) 1/6 48 í* E»-ISV5 40 .- En gráfico, el AABC es isósceles. Si la longitud de AD es tn y la longitud BD es h, calcular tan a. 36 .- En un triángulo rectángulo ABC, donde C A es el ángulo recto, calcular: sen en función de los lados del triángulo A) B) C) 37 .- Calcular: "A' y "B" en la siguiente identidad: A cot 20 = cot 0 + B tan 0 D) 2/¡ tn2-h2 2mh E) 2 i2 ni —h A) 2 y 1 B)-2 y 1 C) 2 y -1 41.- Del gráfico hallar: tan2 0/2 cot ct. 16 R D)----V5 ’ 125 D) -2 y -1 E) 2 y -2 SITUACIONES GRÁFICAS 38.- Dado un triángulo rectángulo ABC recto en “B”, se ubican dos puntos “M” y “N” en los catetos; “M” es punto medio de AB, tal que MA = MB = CN - 2 y tn Z_ BAC = 37° y tn Z NMC = x, calcular: Problemas de Trigonometría y cómo resolverlos A)1 B)2 C)3 D)4 E)5
RELACIONES FUNDAMENTALES 01.- Si: tan a = 1/3, calcular: 3 tan 3a — tana F=-------------- 3tana-tan3a A)-10 B)-9 C)+4 D)-8 C)-2 r 271 1 02.- Si: cos a = —, calcular cos 3a k 3 J 3 23 23 J3 A) 27 b)-27 C)v J3 1 D)v 03.- ¿A qué equivale: tan x (2 cos x + cos 3x)? A)sen 4x B) sen 2x C) sen 5x D) sen x E) sen 3x 04.- Calcular tan 3a, sabiendo que: tan(o + 15°) =-^- A)9/13 B)3/10 C)7/9 D)5/2 E)l/6 05.- Hallar n para que la siguiente expresión sea una identidad: cos 3x.tan x + 2 sen x = sen(n - l)x A) 7 B)2 C)1 D)4 E)5 06.- Reducir: 3 3 V = sen (60° + a) sec a + sen (60 - «Isec a 3j2 1>[3 „ 5^7 A)^- B)— O — D)^ E)íf 07.- Hallar n para que la siguiente expresión sea una identidad: sen 3x cot x-2 cos x = cos nx A) 3 B)5 C)8 D)7 E)4 08.- Simplificar: cos3x—cos3x sen3x + sen3x A) tan 3x/2 B) tan x C) tan x/5 D) tan x/2 E) tan x /4 71 09.- Si: a+p = ~ calcular el mínimo valor de: 6 3 3 E = sen a cos p + cos a sen p A) 1 B) 1/2 C) 1/4 D) 1/8 E) 1/16 RELACIONES AUXILIARES 10.- ¿A qué se es igual? 8cot3a y cot(a + 60°)+cot(a-60°) A) 3 - cofa B) 4 - cofa C) 3 - cot2a D) 7 - cot a E) 2 - cot a Identidades Trigonométricas del Arco Triple PP13 829
11.- Simplificar: cos 36sen6 + sen 26 l + 2cos26 A) 2/4 sen 36 B) 5/6 sen 56 C) 8/7 sen 46 D) 3/4 sen 46 E) 1/2 sen 26 12.- Calcular: V = 4 sec 18° tan 72° +cot 72° 18.- Si: sen2A = 1/2 (cos 6A - cos 4 A), calcular: E = sen A + sen 3A A) 1 B)6 Q3 D)7 E)5 19.- Hallar (A+ B), en la expresión identidad: 2 2 sen 3x(3 sen 3x - sen 9x) + sen 6x = A sen Bx A)5 B)8 Q-3 D)4 E)7 A) V2 +1 B) V2 -1 C) yJ5 -1 20.- Simplificar: D) <7 +3 E)V6 -5 13.- Calcular: eos310°+Visen310° cosí 6o + Visen 10° A) 2/3 B)5/3 C)6/7 D)8/3 E)3/4 14.- Sabiendo: sen (45° - a)=1/3, calcular: sen (45°+3a) A) 20/25 B) 15/10 C) 23/27 D) 13/11 E)9/8 15.- Calcular: 2cos320°+2cos340° cos 20° +cos 40° A) 4/6 B)2/5 Q7/2 D)3/2 E)6/5 4sen 2 3ó — sen 2 6ó 16.-Simplificar: R— A) sen420 B)sen43<¡) C)sen34<i) D)sen40 E)sen470 17.- Sabiendo que: sen36 sen 36 + cos36 cos 36 =1/27, determinar: cos 66 A) 23/24 B)-23/24 C) 25/17 D)-25/17 E)-23/27 1 1 y—------------~-------------- tan 3a+tan a cot 3a+cot a A) cot 2a B) cot 7a C) cot 5a D) cot 4a E) cot 3a 21.- Si: sen a - cos a - 1 = 6. Hallar: cos (3a -135°) Si: (a-45°)e IC A)-VÍ/2 B)-VÍ/2 C)VÍ/3 D)-VÍ/4 E)Vó/4 22.- Si: sen a + sen P = 3(cos P - cos a), hallar: P = cos 3a + cos 3P A)1 B)4 Q3 D)0 E)5 23.- Calcular: E = sen320° + sen3140° + sen 3280° 3VÍ 3VÍ 3 B)-— C)-7 3 1 D)‘i E*'i 24.- Calcular el valor de: 4 eos3 24° + 6sen212° — 3 -------------------- 4cos3 8° + 6sen2 4o - 3 RACSO DITOtBI PP13 830 Problemas de Trigonometría y cómo resolverlos
Si: cos 48° = n A) 2a B)2a + 1 C)2a-1 D) 1 - 2a E) 2 - a PROBLEMAS CONDICIONALES 25.-Si:tanl x---=2, calcular: l * 3J sen3xcot x - 2 cos x 2senx+cos 3x tan x A) 11/2 B)10/3 Q9/8 D)7/2 E)8/ll 2 26.- Calcular “tan 0” si se cumple: 3 /— eos 0 + cos 0 = V 2 cos a sen 0 + sen 0 = V 2 sen a B) cos 3x cos 3y cos 3z C) sen 3x cos 3y cos3z D) cos 3x sen 3y sen 3z E) tan 3x tan 3y tan 3z 30.- Si se verifica que: sen 30 cos 20 = sen 0, hallar el valor de la expresión: E = sen 50 - sen 0 A)-l B) 1 C) 2 D)-2 E)cero 31.-Simplificar: í 2lt A f 471 I ~+ct I +cscl -^- + a A) 2 csc 2a B) csc 3a C) 3 csc 3a D)4csc2a E)5csca A)0,5 B)0,4 C)Oa D)0,3 E)O,1 27.- Si: sen 60 = tan2© cot2(30° - 0)tan(24O° - 0) Calcular «tan 30» si se sabe que es diferente a cero A)1 B)2 Q3 D)2/3 E)3Z2 28.- Si se verifica: 32,-Si: cos2a=x... (1) cos3a = y... (2) Eliminara: A) 3y2 = 3x3-4x+2 B)2y2=4x3-3x+ 1 C)4y2 = 2x3-3x+3 D)5y2 = 6x3-2x + 5 E)7y2=4x3-2x + 4 V sen30sen30 + cos3 0cos30 = — Además 0 e IIC, calcular “6 cos 0” A)--Jó B)-2Vó C)-l D)-2 E)-3-Jó 29.- Cumpliendo que: x + y+ z = n factorizar: 3 3 3 sen3xcos (y-z)+sen3ycos (z- vj+sen3zcos (x- y) A) sen 3x sen 3y sen 3c 3 r~ 33.- Resolver: x - 3x + V 3 =0, luego, una de las soluciones es: A) -2 sen 10° B) -2 sen 40° C) -2 sen 40° D)-2 sen 50° E) -2 sen 80° 34.- Eli minar “0” de: x cos 20 + y cos 0 = 2(x - y) cos"0 y sen 30 - x sen 0 = 2(x - y) sen‘0 Identidades Trigonométricas del Arco Triple PP13 831
A) 3(x2+y2) = lOxy C)x2 + y2 = 4xy •> 9 E)3(x"+yj=4xy B)3(x2-y2) = 10 xy D)x2-y2 = 4xy 39.- Hallar el equivalente de “zn” en la siguien- te identidad: 35.- Evaluar: f 71 15 si: /(x) = (cos x - sen x)(2 sen 2x + 1) - (cos x -¡ sen x)(2 sen 2x - 1) 5-1 45-1 B>— C)^ 45 + 1 D) — E) 45 + 1 sen5a 2 36.-Si:------=x +x-1, hallar el equivalen- sena te de : 3x - x A)2 cos 3a B) - 2 cos 3a C) 2 cos 6a D)- 2 cos 6a E) 2 cos 12 a cot 30 sen60 37.-Si:----— - a, el equivalente de-— ,es: cot 0 sen20 2(1 +a2) A) □ (!-«)“ 2(1+a2) (l + «) 2(1—a ) C)-------F (l-«)2 X sen — 2 X + 2cosjcsen— a 2 9x cos 4 3x 2 —cos—cos3x 4 1 3x 1 3x 1 3x A)I sec — 4 B) j esc — C)—sec — 1 3x 3x D)7 CSC — 4 E)sec — 4 SITUACIONES GRÁFICAS 40.- De la figura mostrada, obtener el valor de “x”. A) 13° B)23° C)27° D)30° E) 37° 41.- En la figura, si AE = EC = a y 1 IC = b, el valor de BC en términos de a y b es: A>^ 5 + 1 4a D)------7 (!-«)“ 4a (l + «)2 38.- Simplificar la expresión: sen3x „ -------2cosx tanx________ cos3x „ -------1- 2senx cotx A) tanx B)tan2x C)tan3x D)cot2v E)cot3x A) ^h2 B) 3/^ C) 3/^1 D) E) 3/^2 42.- Hallar "x". A) 10° B) 15° C) 20° D) 25° E) 30° ÍJéRACSO W^BDITOK» PP13 832 Problemas de Trigonometría y cómo resolverlos
insfóHr DifeU TRANSFORMACIONES A PRODUCTO 01.-Simplificar: senO+ senÁ0 + sen(2£-1)0 COS 0 + COS A0 + cos(2fc —1)0 A) tan A0 B) tan 0 C) cot kG D) cot 0 E) 1 02.- Transformar a producto: E = sen 24° + sen 16° - sen 8o A) 4 cos 4°sen 8°.cos 12° B) 4 sen 4°.sen 8°.sen 12o C) 4 cos 4°.cos 8°.sen 12° D) 4 sen 4°.cos 8°.sen 12° E) 4 cos 4°.cos 8°.cos 12° 03.- Transformar a producto: E = cos 2x + cos"a + 2 cos 2a - sen 3a 3 3 3 A) 4 sen"3x sen 2a D) 8 sen"3A sen "2a 2 a 2 3 B) 8 cos 4a cos "2a E) 4 cos 4a cos"a C) 4 cos22a cos 2a 04.- Si: a = 5o, hallar: sen2« + sen3« + sen4o E=--------------------- cos 2a + coi 3a + cos 4« A)2+VI B)2-V2 C)2+V2 D)l+j3 E)2- 4?> 05.- Simplificar la expresión: cos 4 a + cos 8a + 2 - 4sen "a E = ------------------ seriv + sen3A + sen5A + sen?A A) cos"3a . csc 4a . sec a B) sen"3A. cos 4a . sec 2a C) sen 2a . csc 3a D) csc 3a . sec 2x. cos 2a E) sen 3a . csc 2x 06.- Reducir a monomio: sen2« sen4fi sen4fi sen3« sen3« sen5« A) sen 6a. csc 5a D) tan a . cot 5a B) sen 3a . cos 4ri E) cot a . tan 5a C) sen 2a. csc 3a 07.-Simplificar: 1 + cos a + cos 2a + cos 3a E=----------------5----- l + cosa-2sen a A) csc a B)seca C)tana D) 2 cos a E) 2 sen a 08.- Si: A + B = ti/4, hallar el valor de: A) y/3 + 1 D) V2 -1 sen A + senB cos A + cos B B) 42 +1 E)2- 43 C)V3-1 Transformaciones de Sumas o Diferencias a Productos PP14 833
09.- Transformar a producto: S = exsec x - vers x A) 2sen2.l — I secx 2 D) 4 eos4! 2 .senx B) 4 sen4 x 2 . sec x 2 I JC E) 2 cos x. eos — 2 sen 2A + sen 2B + sen 2C=4 cos A.cos B.sen C A) Obtusángulo D) Isósceles B) Rectángulo E) Equilátero C) Acutángulo 14.- Factorizar: y = sen a + sen 3a + sen 4« C) 2 eos" x 2 . cscx a 3a A) 4 cos —. csc — sen 2a 2 2 10.- Calcular: sen3 70° + sen350° + sen3 20° + eos310o y= cos 40° +sen 80° a a B) 2 sen —. sec —. tan a 2 2 a a C) 4 cos a . sec — . csc — 2 2 a a a D) 2 tan — . sec — . csc — 2 2 2 a 3a E) 2 cos — . csc— . sen 2a 2 2 ll.-Si:xG Tt Tt 18’18 hallar la extensión de: 15.- Calcular: E = sen410° + sen450° + sen470° senx(cos2x + cos 4x + cos 6x) A=-------------------------- cos4x A)<0;l> B)<-1;1> C)<0;l> / 1 1\ / 1 1 D)\ 2’2/ E)\ 2’2 A) 9/8 B)9/4 C) 1/8 D)ll/8 E) 1 16.- Calcular: A + B + C 4^71 4^71 sen —i- x + sen-------x l6 J l6 Acos 4x + B cos 2x + C 12.- De qué naturaleza es el triángulo ABC si se cumple: 2 2 1 sen A + sen B + sen"C = 2 A) Isósceles D) Obtusángulo B) Equilátero E) Acutángulo C) Rectángulo 13.- De qué naturaleza es el triángulo ABC, en el que se cumple: A)2 B)4 C) 1/8 D)8 E)10 17.- Si: sec x + sec y = a ... (l) tan x + tan y = b ... (2) a - i a (*-*) <A + F) ¿A que es igual: A = cos —~. csc —-— A)a2¿2 B)ab C)al? D)a/b E)b/a -4 RACSO ^tSDITOBEB PP14 834 Problemas de Trigonometría y cómo resolverlos
PROBLEMAS CONDICIONALES A) 15° B)36° C)45ü D)53* E)75‘ 18.- Sabiendo: a + y + z = 366° Simplificar: 23.- Factorizar; 3A a y z cos—eos—cos— 2 2 2 A E = sen - + senA + sen-+ sen 2A + sen 2 2 5A 2 L = 1 + cos a + cos y + cos z 3A A A) sen --- . sen A . csc — ’ 2 4 x . y z , COS-Feos—+ COS-1 2'2 2 A y Z sen—sen —sen — 4 4 4 A) 17/4 B)3/4 C)5/3 D)5/4 E)9/4 19.- Calcular “a” a partir de la siguiente igua- lando sabiendo que: A + B + C = 186° 5A B) sen — 4 3A A . sen — . csc — 2 4 3A C) tan---- 2 5A . tan---. csc A 4 3A A D) eos . cos A . sec — A(sen A + senB + senC) senAsenBsenC ABC cos—cos—cos— 2 2 2 7A 5A 3A E) sen--. sen----. csc-- ’ 4 2 4 24.- Calcular el máximo valor de: sen2 A + sen2B + sen2C E = sen (50° + a) - sen (10° - a) A) 1/8 B) 1/16 C) 1/4 D)16 E) 4 20.- De qué naturaleza es el A ABC si se cumple: A) 1/2 B) 1 C)V3 D)V5 E)2 cos 2A + cos 2B + cos 2C + 1 - 0 25.-Si: A) Obtusángulo D) Equilátero ¿Cuál es el valor de _v que verifica la igualdad? B) Acutángulo E) Isósceles ABC sen A + senB-sen C= a sen —.sen —. eos — 2 2 2 C) Rectángulo 21.- Simplificar: A) 1 B)2 C) 3 D)4 E)5 a sen.v — 2/xsen3A + a sen5 a V---------------------------- a cos r — 2A cos 3a + o cos 5 a 26.- Transformar a producto: E - sen A’ + sen y + sen z A) tan a B) tan 2a C) tan 3a A.) 4 cos(a + v).cos (y + z). cos(a + z) D) tan4v E) tan 5a B) 4 sen(A + y).s en (y + z). sen (a + z) 22.- De la siguiente identidad: C) 4 eos . cos 2sen 60 sen 6 = sen 96 + sen 36, 2 . cos 2 la medida de 6, es: Transformaciones de Sumas o Diferencias a Productos PP14 835
27.- Calcular: E = cos"6° + cosZ42° + cos_66° + cos278° A) 7/2 B)7/4 C)7/8 D)7/16 E)9/4 28,-Si: senx + seny = a ... (1) cos a + cos y = b ... (2) ¿A qué es igual sen (a + y)? cosx cos(x+y) cos(x+2y) cos(x+3y) a b c d A) c(« + d) = b(b + c) B) c(a + c) = b(b + d) C) b(a + d) = c(b + c) D) c(a -d~) = b(b - c) E) c(a -c) — b(b - d) 271 47t 871 32.- Calcular: E = sen ~ + sen ~ + sen — 7 7 7 A) a2—b2 a2+b2 2flfc B)-T-7T a —b C) 2ab a2+b2 D) ab a2—b2 ab E) 2 (2 a +b 33.- Calcular “n” si: 12 cos x + cos l Ix = n cos 0. cos <J> A) 10 B) 11 C)12 D) 13 E) 14 29.- Calcular el menor entero par que adopta “Á” en la igualdad: 71 471 Ó7T 71 sen(A - l)y = cos~y. tan — + 2 seny A)2 B)4 C)6 D)8 E)10 30.- Calcular una relación entre x, y z indepen- diente de “0” sen6 sen30 sen56 x y z A) x(c - x) = y(y + x) D)y(z-x) =z(y-x) B) a(z + x) = y(y + x) E) y(x + y) = z(z + x) C)x(z + x)=y(y-x) 31 .- Calcular una relación entera a, b, c y d independiente de x e y: 34.- Simplificar: cos5x + 3cos3x+4cosx sen5x - 3sen3x + 4senx A) - cot3x B) tan3r C) - tan3x D) cot3x E) 1 EXPRESIONES EQUIVALENTES 35 .- Calcular el valor de: senl090° + sen790° + sen460° K 2cos5°cos35°cos50° A)-l B)-2 C) 1/4 D)05 E)2 36 .- ¿En qué tipo de triángulo ABC, se cumple? senB + senC sen A =----“-----~ cos B + cosC A so B^bditobes püL 8361 Problemas de Trigonometría y cómo resolverían
A) Equilátero B) Isósceles C Rectángulo D) Obtusángulo E) Acutángulo 37 .- Si: sen(y + z + x); sen(x + z - y); sen (x+ y - z) forman en ese orden una progresión arit- mética, ¿a qué es igual tan y? A)tanx+tanz B)tanx-tanz C) 1/2 (tanx + tan z) D) 1/2( tanx + tan z) E) l/2(tanx-tanz) 38.- De la siguiente relación: C) 4/h sen x sen 3x sen 5x D) 4m cos x cos 2x sen 4x E) 4m cosx cos 3x cos 5x cos(x - y) cos(x + y) sen2y a b c ¡ A qué es igual cosx ? sen y a-b a+b a + c A) B) C) c c b a—c b+c D) —- E) b a 41.- En la figura, calcule sec x. (cot 2x+cot x) (sen 3x+sen x) A) 72 6)2-72 C)73 D)2 E)7ó/2 42.- De la figura, encontrar la medida de 6.- SITUACIONES GRÁFICAS 39.- Si: AB = DC, calcular E = tan 6 - 2 cos 46 + tan 36 A)-2 B)-I C)6 D)1 E)2 40.- En la figura, calcula el equivalente en for- ma de producto de la siguiente expresión: A — AE + BE + CE + DE A) 10° B)36° C)60° D)50° E)35° 43.- De la figura determina el valor de "P" A) 4/n sen x sen 2x sen 4x A) 30° B)45° C)53° D)«F E)75° B) 4m sen x sen 2x sen 3x Transformaciones de Sumas o Diferencias a Productos PP14 837
05.- Transformar a producto: APLICACIONES DIRECTAS 01.- Transformar a producto E = sen 5.r sen 4a + sen 4x sen 3x- sen 2x sen x A) 2 sen 5x cos 3x sen x B) 2 cos 5x sen 3x cos x C) 2 sen 5x sen 3x cos x D) 2 cos 5x cos 3x sen x E) 2 cos 5x cos 3x cos x 02.- Expresar en forma de sumas o diferencias: E = 4 sen x sen 2x sen 3x A) sen 2x + sen 4x - sen 6x B) cos 2x + cos 4x - cos 6x C) sec 2x + sec 4x - sec 6x D)tan4x E) sec 6x 03.- Calcular: 9 ? E — sen 36° - sen 36° cos 6o + eos 6o 3 13 13 A)- B) — O- D)- E)~ 04.- Reducir: E= ll cos óx.cos 3x+sen5xsen4x + cos x A) 2sen x B) y[2 sen x C) 2 cos x D) V2 cos x E) cos x 271 E = 2 sen — + tan “¡tj- 7t 11 71 271 571 A) 8 sen — . sen —. sen — 7 11 11 11 71 271 571 B) 6 sen —. sen —. sen — 7 11 11 11 71 2ti 571 C) 4 sen —. sen —. sen — 7 11 11 11 71 2 71 571 D) 2 sen —. sen u . sen — 1 71 2 71 571 E) — sen — . sen — . sen — 7 4 11 11 11 2 71 571 871 06.- Reducir: CSC + CSC - CSC — 11 11 11 4ti 271 471 271 A) 2 cos —. .CSC u B) 2 sen —. sec 11 471 271 471 271 C) 2 sen —. CSC — D)4cos — .csc 11 471 271 E)4sen — + sec — 11 11 07.- Expresar como diferencia de senos: P = 4sencc sen —+ a 4 . sen PP15 838 Problemas de Trigonometría y cómo resolverlos RACSO DITOBRB
A) sen 3a - sen 2a B) sen 4a - sen 2a C) sen 5a - sen 3a D) sen 3a - sen a E) sen 4a - sen a 08.- Reducir la expresión: senx sen3x v= —— - —— sen2.x sen4x A) sen x. csc 4x B) cos x. sec 4.x C) -sen x. csc 4.x D) - cos x. sec 4x E) sen 2x csc 4x 09.- Transformar a producto: 4 sen 2x. cos 4x. cos 6x - 2 sen x. cos 3x A) 2 cos 2x cos 15x cos 5x B) 2 sen 2x sen 15x csc 5x C) cos 2x cos 15.x cos 5x D) sen 2x sen 15x csc 5x E) cos 2x cos 15x csc 5x PROBLEMAS CONDICIONALES 11.- Si: k—(cos 3x sen 2x - cos 4.x sen x) sec 2x, calcular la extensión de “k" A)<-1;1) B)<0’;l> C)[0;l> D)[-i;i>-{-72/2} E)[-l; li- li.- Calcular la medida del ángulo “x”. Si: x e ( 0o, 90°). Además se cumple: 4cos40° 3 cos 20° + 2 cos 80° (cos 10° +V3 j = tanx A)30° B)40° Q45° D)50° E)20“ 13.- Calcular el máximo de: E = sen (x + 20°).sen (x + 80°) 1 2 3 4 1 a)7 B)I C)7 D)? E)’4 14.- Si: tan a. tan ( a + b ) = k, ¿a qué es igual? E = cos (2a + b) sec b 10.-Si:xe 77171 30’4 mo y mínimo valor de: , calcular la suma del máxi- i—k 1 + * k-1 A) 1-F* C)7+T k + 1 E)Jl M = 2 cos22x cos x - cos x - 0,5 cos 3x 15 .- Transformar al producto: E = sen 18°.csc 12° + cos 36°.sec 6o A) 4 sen 36° - cos 12° B) 4 cos 36°-sen 12° C) 4 cos 18° - sen 6o D) 2 sen 36o- cos 12° E) 2 cos 36° - sen 12° 16 .- Si: sen x + sen y = sen x.sen y Calcular: U-y) E = cos—~—- sen 2 (*+>’) 2 A) 1/2 B)-l/2 C)±l D)±l/4 E)±l/2 Transformaciones de Producto a Sumas o Diferencias PP15 839
17 .- En un triángulo ABC se cumple que: A 2 sen B sen C = sen A. cot — 2 ¿Qué tipo de triángulo es? A) Rectángulo B) Isósceles C) Equilátero D) Escaleno E) No se puede afirmar nada 18 .- ¿A qué es igual? E= cot 20°-4 cos 20° A) V3 B) sen 20° C) cot 20° D) cos 20° E)1 19 .-Calcular el máximo valor de: 2ti ti E = sen(x + — ).cos(x + ~) Cuando: 0<x<ti 3+V2 D> 2 V3+2 *>— C) ^3 + 2 2 20 .- Si: tan(n + b) = 3 tan a sen2(fl + b) + sen2a Calcular: E=-------------—--------- sen2í> 2 2 2 22 .- Si: sen x + sen y + sen z = 1 x+y+z=0 Calcular: cos x. cos y . cos z A) 3/2 B)2/3 C)1Z2 D)2 E)1 2 tanx 1 + cos x 23 .-Si:---=-------- tany 1 + sen x Calcular: sen (3x + y), csc (x - y) A) 1 B)3 C)5 D)7 E)8 24 .- ¿A qué es igual? “0” cot 0 = — sec 80° - 2 sen 70° 2 A) 15° B)30° C)45° D)54° E)60° 25 .- Simplifican sec46° - sec 14° + sec 74° A)sec42° B) 3sec42° C)-3sec42 D) 3 sec 14° E) - 3 sec 14° 26 .-Si: V3 cos 17°-cos 13° = wi ¿A qué es igual? E= V3 cosl7°- sen 17° A)/n B)2/n C)mf2 D)m/4 ~E)y[3m A) 1 B)2 C)3 D)4 E)5 sen3 3x - sen4x.sen2x 21.- Si: ---5-----------------= k eos 4x + sen5x.sen3x Calcular: cos 2x 1-k2 2 A) , 1 + Á:2 B) 1 + * Q 1 + k2 D)2Á:V1-Á:2 E)k 5n . 4ti tan----4sen— 27--Simplificar: “ tan — + 4sen— 11 11 A)V11 B)^y- C)1 D)^y- E) 2 28.- Si: 1 - cosx; 1 - cosy; 1 - cos z Están en progresión armónica PP15 840 Problemas de Trigonometría y cómo resolverlos RACSO IDITO1I1
Calcular: E= 2sen xsen z senx+senz ”sen^ A)0 B)1Z2 C)1 D)2 E) 4 29.-Si: A)V3 B)V2 C)2 D)-2 E)1 2n 4n 6ti 35.-Calcular: E= sec — + sec— + sec~y A)-l B)-2 C)-l D)2 E)-4 cos Ix sec x = Acos 6x + B cos4x+Ccos2x +D 36.-Calcular: Calcular: A + B + C + D A) 1 B)2 Q3 D)4 E)5 7x 3x 9x 5x 30.-SÍ: cos—. cos — = cos — .cos —, 2 2 2 2 hallar: E = tan 6x. sen x A)-l B)1 C)0 D) 1/2 E) -1/2 VALORNUMÉRICO 31.-Calcular: í. 271 'I í. 471^1 6ti^ I 1—cos 1 1—cos 1-cos— l 1 ) l 1)' l 7 J 7 7 7 7 7 a>7 b>- q- d>- e>- 37.- ¿A qué es igual? cos 12°. cos 60“. sec 72° - cos 24° r- ti 2n 3n n 2n L= V7 cot ycot~cot —- sec y sec — 3n sec — A)-7 B)7 Q 1/7 D)-l/7 E)9 32.-Calcular: 2n 4n 6n E = tan — + tan — - tan — 14 14 14 A)-l D)-V5 B) -V2 C) 3 33.- ¿A qué es igual “0”? tan0 = l + 4cos20° A)cos12° B) cos 60“ C) cos 36° D)cos 48° E) cos 24° 38.-Calcular: 7t tan—. tan 14 3ti 5ti — . tan — 14 14 V7 A,’V B’v O- 41 D) y[7 E) -2y[7 39.-Calcular: 5 371 5 571 5 ti sen — + sen — - sen ~ V7 3j7 B) 16 5^7 C) 16 D)^ 7 16 J7 E’v A) 10° B)20“ C)80“ D)70° E)30° 2 71 2 71 2 27t o 371 40.- Calcular: E = csc y + csc ~y + csc~— 7 34.-Calcular: tan 5o+cot 5o - 8cos20° 4 A)4 B)8 Q12 D)16 E)20 Transformaciones de Producto a Sumas o Diferencias PP15 841
01.- La suma de los senos de 5 ángulos en progresión aritmética es cero. Calcular la ra- zón de la progresión, sabiendo que es positi- vo y menor de 90°. A)72° B)75° 060° D)45° E) 30° 02.- Calcular el valor de: E = (cos 10° + cos 20° + cos 30° +... sen5° + cos 240°)----- cos55° V3 V3 r- i- A)-y- C) <3 D)-V3 E) 1/2 03.-Calcular: 71 71 71 R=sen—~ + sen +sen “+... +sen 7i loU >u oU A)tan¿5 71 B) ,an Isó 71 C)tan 360 71 71 D)cot — jou E)OTisó 04.- Evaluar la siguiente expresión: 71 271 471 E = tan — + tan ~ + tan— A) V7 B) V2 C) V3 D)V5 E)-V7 05.- Calcular el valor de S, si: 7t 2ti 3ti 4ti S = cosy+cos— + cos~ +cos~ + 5ti 671 COS— +cos~ 06. En ontrar el valor que le corresponde a S en la siguiente expresión trigonométrica: 2 71 2 271 2 371 S = cos ~ + cos ~ + COS — 7 7 7 A) 1/4 B) 1/2 Q 3/4 D)5/4 E)1 07.- Calcular: Tt 3n 5ti 7ti 97t S=COS — +COS — +COS — +COS — + COS — A)-1/2 B)1 C)1Z2 D)-3Z2 E)3/2 08.- Evaluar: 7 7 7 S = cos~ 10° + cos220° + cos j0°+ - - - + cos2170° A) 7 B)8 C)6 D)9 E)17 09.- Hallar el equivalente de: S = senx+sen3x+sen5x+ ... + senl5x 2 o A) sen 8x. csc x B) sen~8x sec x C) cos28x . sec x D) cos 8x csc x E) cos 8x. csc2 x 10 .- Si la expresión: es idéntica a: A + B csc 1°; se pide calcular: V = 2A-4B A)46 B)44 C)42 D)40 E) 80 PP16 842 Problemas de Trigonometría y cómo resolverlos RACSO IDITO1II
11 .- Calcular el valor de la suma de la serie: ti 3ti 5ti E = cos + cos —— + cos + 2n +1 2n +1 2n +1 f 2n-l'| .. .+ COS I „ , 71 ^2n + l j A)n B)2n C)2n + 1 D)-l/2 E) 1/2 12 .-Calcular: 271 471 671 E = cos ----- + cos --7 + eos ---7 + 2n + l 2n + l 2n + l 2n7i ... + cos~--7 2n + l A)n B)2n C)2n+1 D)-l/2 E) 1/2 13 .- Evaluar P en la siguiente expresión: •7 7t 2 2rt 3ti 2 4tt P=sen“ — +sen ~ + cos“— +cos ~ A) 1/4 B)2/4 Q3/4 D)5/4 E)9/4 14.-Calcular: 2ti 4ti 4ti 6ti R = COS~COS~ +COS“COS~ + 2ti 671 eos— eos— 7 7 A)-l/2 B)l/2 Ql/4 D)-l/4 E) 1/8 15 .- Al reducir la siguiente expresión, se obtiene: 7 7 7 R = sen“(x - 120°) + sen x + sen2 (x + 120°) A) 1/2 B)1 C) 3/2 D)3/4 E)5/4 2 71 2 27t 2 3rt 16 .- Calcular: R = tan ~ + tan — + tan — 7 7 7 A) 7 B)9 C)ll D)17 E)21 17.- Se pide evaluar P, si se sabe que: A) 19/16 B)17/8 C)13/4 D) 1/8 E)l/16 18.- Sabiendo que: 10 6 = Tt, y además: 7 7 7 7 sen~0 + sen“20 + sen 30 + .. sen“90 + + sen“110 = A+ Bcos(C0) ; calcular: A + B + C A) 5 B)3 C)6 D)7 E)8 19.-Determinar: K . 2n+l 2L 2n+l 271 2n+l 371 M = COS ~ + COS — +COS — +- - - 6ter minos Sabiendo que: n e Z A)0 B)3 C)6 D)7 E)8 20.- Hallar los valores de “a", para los cuales existan soluciones de la ecuación: 2 21 71 ] '>( Tt 1 sen x + sen x + — + sen x + — =a l 6J l 3J r~ 1 5 A) 0<n< V2 D) — <n<— 1 1 1 B)0<n<- E)-<n<- 4 4 2 C)0<n<^ 21 .- Calcular la suma de los n primeros térmi- nos de la serie: 1x1 x 1 x S = — tan — + — tan “T + tan v + - 2 2 2“ 22 23 23 1 x 1 x A) cot— - cotx B) — cot— +cotx 1 X 1 X C) — tan— -tanx D) — tan— + tanx 2n 2 2 2 1 1 x E) - cotx Sucesionesy Series Trigonométricas PP16 843
22 .- Calcular: 2ti 4n 6ti 20ti S=cos—+2cos—+3cos — +...+10 cos -yj- A)0 B)45 C)11Z2 D)-11Z2 E)-9/2 23 .- Calcular la suma de los “n" primeros tér- minos de la serie: S = sen I.sec3 + sen3. sec9 + sen9. sec27+... A) 1/2 (cot 1 - cot 3n) B) 1/2 (cot 3n- cot 1) C) 1/2 (tan 1 - tan 3n) D) 1/2 (tan 3”- tan 1) E) 1/2 (tan 3n'1 - tan 1) 24 .- Determinar la suma de los “n” primeros términos de la serie: S = sec2.tan 1 + sec A) tan 2n - tan 1 C) cot 2n - cot 1 E) cot 1 -cot2n+1 25,-Si se sabe que: tan2x tan22x tanx tan2x ¿a qué es igual: ^_tan2x tan22x cotx cot2x A) m - 2n B) m D)m-2n E) 2¡ 4. tan2 + sec 8. tan4 +... B) tan2n+1 - tan 1 D) cot 1 - cot 2n tan23x ' 5 + tan 2 x tan2”x • -+---^“7“ = m ; tan2 *x tan23x tan2nx cot22x cot 2 *x r2n C)2n-m i-m 26.- Calcular la suma de la serie de 10 térmi- 71 nos, cuando x = —, 44 S = sen x sen3x + sen3x sen 5x + sen5xsen7x+.. 71 71 71 A) 10 sen — B) 20 sen — C) 10 sen— 2 22 11 2 11 71 71 D) 20 sen— E)5cos— 22 2 22 2 2 2 2 27 .- Si: cot 1 + cot 2 + cot 3+...+ cot n = m ¿a qué es igual: cotí cot 2 cot 3 cotn S = ~ + T + ~ + . . . + ? tan 2 tan4 taño tan2n A)l/2(m-n) B)l/2(nz + n) C) 1/2(2 m-rí) D)l/2(2n-nz) E) l/2(rwí-l) 28 .- Simplificar la expresión: senx + sen3x + sen5x + ... + sen(2n —l)x cos x+cos 3x+cos 5x+... + cos(2n - l)x A) tan (nx) B) cot (nx) C) 1 D) tan x E) n tan (nx) 29 .- Calcular la suma: S = eos21°+ cos22° + cos23° +..+ cos290° A)0 B)45 C)90 D)90,5 E)44,5 30 .- Evaluar la suma para n = 12 ti 2ti 3ti (n-l)7t S = sen — + sen—+sen — +...+sen----- n n n ti A)-j6+V2+2 + V3 B)Vó +42-2- 4?> C) 4^ -42+2- 4Í D)Vó-V2-2-V3 E) 46 -42-2+ 41 PRODUCTORIAS 31 .- La expresión: cos 20° cos 40° cos 80° cos 160° cos 320° sen 10° es equivalente a: ^iRACSO A^SDITOaiBfl PP16 844 Problemas de Trigonometría y cómo resolverlos
A) 1/2 B)l/4 C)-l/8 D)l/16 E)-l/64 32.-Calcular: ti 2ti 3ti 4ti 12ti P=tan—.un — .tan — tan—...un— 36.- Si: (2n + 1) x=ti, calcular: P = sen x. sen22x. sen33x... sen2n2nx A) (2n + l)Zp+l 2n(2n+l) .2n+I 2n(2n+l) A) 1 B)2 C)3 D) V5 E)5 33.-Calcular: 71 271 3ti M = sen y sen~ sen — (2n + l)n 2n(n+l) D) -S-:------- ' 2n<n+,> Q n V7 V7 V7 V7 A)— B)— C)— D) — E) 1 8 4 6 7 E) O 37.-Calcular: 34.-Calcular: P = tan x tan 2x tan 3x tan 4x... tan 16x E= sen Io. sen 2o. sen 3o... sen 88°. sen 89°. sen 80° sabiendo que: 271 sen 32x = sen x, para: 0 < x < V33 r— V33 A) ~ B)-V33 Q--^— 33 ' 33 A)0 B) 1 Q-y5o 90 V89 1 D) „89 E) 90 38.- Calcular el producto de los “n” primeros factores: 7 7 “7 P = (1 - tan 1)(1 - tan 2)(1 - tan~4)... D) V31 E) V 33 35.- Calcular el producto de los n primeros números factores: 2ntanl A) tan 2” 2ntan2n 2ntanl P=(2cos — - l)(2cos — -l)(2cos~ -1)... 2 4 8 2” D)----- tanl 2»+! E)^í 2 cos— + 1 2 ) 2cos2xnx +1 2cos2nx+l B)---------- 2 eos—+1 2 39.- Calcular el producto de los “n” primeros factores: P = cos x. cos 2x. cos 4x. cos 8x. .. 2 cos x + 1 O-----x 2 cos— + 1 2n 2cos---+ 1 2” D) ---- ---- 2cosx+l sen2n x 2nsenx sen2n+l x B) 2n+1senx sen4“x Q —----- 4 senx 2 cosx—1 E)-----~ 2 eos—-1 2” 2n D)----- sen2 x sen2 2n-,senx Sucesiones y Series Trigonométricas
DOMINIO DE LAS F.T. Jversx—covx , si x G [0; 2n] 01.- Determine el dominio de la función/defí- nida por: I ex—secx 11 ex—secx+21 fix^= |secx+l| I 71 A)R-^xór=(2p + l)7f Ux=(2m+ l)p,pe Z; me Z a \ - r ti - 5ti i n\ ¡tí . 5ti i A)xg 2’~F B)xg y»-y- Qxe |p~yj D)xg [4.-47J i tt . 5tc i E)xe Ls’tJ 03.- Determine el dominio de la función/defí- nida por: B)R-lx/x=(2p+ 1)-^ Ux=(2m+ l)p,pG Z; Z QR-<x/x=(2p + l)y Ux=(2m+ l)p,pG Z; m 6 Z II 71 D)R--jx/x=(2p+ l)y Ujr=(2m+ l)p,pe Z; tnG Z I 71 E)R-jjcóic=(2/>+1)^ \ux=(2m+l)p,pG Z; mG Z • 02.- Encontrar el dominio de la función/defí- nida por: 71 A) R- -j xJx = (2k + 1) -j vj x—2 ntit, kG ZjnG Z {TT. x/x=(2k+ x=2nm,kG'^,mG'Z r {TE . II x/x=(2L+1) —VJx=2nm, Á:GZ,mGZ » {TE II x/x=(2A+ l)y G)x=2rmt, kGZ^nGZ r {TE II x!x= (2k+ 1) y ux= 2 mn, kG Z,mGZ r 04.- Si el rango de la función: F(x) = csc^2x—es (-2; - -J2 ) U ( -J2 ; 2) determinar el dominio en el recorrido: 41*RACSO iDITDHI PP17 846 Problemas de Trigonometría y cómo resolverlos
7ti\ /7ti 5ti — ) ;— 4 / \ 6 4 07.- Dada la función definida por: fix)- Vi - tanx + 71+tanx, / 71 hallar eldominiodelafunciónsixE D, / 3n 517i\ /3ti 1971 \ 4 4 / \ 4 24 / 1 Iti 5n\ /3ti 19ti\ \ 24 12/ \ 4 24 / D) B) 7171 4*4 Q 71 71 3*2 71 71 4’4 E) 71 71 / 17ti 1 Iti\ / 5tt 31ti\ D) (-----;----) U (—;-----) \ 12 8/ \ 4 4 / E) 5ti 4 2971 \ /25ti 13ti\ 24 / \ 24 12 / 05.- Sea la función “ff" definida por la siguien- te regla de correspondencia: sen2x + cos— fix) =---------—; (n e Z), hallar el domi- x cos— n niode/ „ ti rm A)R-(2n+l)- B)R C)R-— 2 2 D) R - (2n + 1)ti E) R- 2«n 06.- Hallar el dominio de la función definida por: 08.- Para que valores dex, la función/defini- I tanx+cotx—4 71 da por: f(x) = J-----------, 0 < x < —, V tanx+cotx 2 tiene valores reales? /_ ti\ / ti 5ti\ / ti\ /ti ti\ A) \ ’2/ \12’12/ B) \ ’2/ \6’4/ C)/o;-\ D)/o;-V/— \ 4/ \ 2/ \12 4 E) /o;-\ - \ \ 2/ \4 12/ RANGO DE LAS F.T. 09.- Determine el rango de la función/defini- da por: fix) = 2sen(7i. tan x), si x e 377t 537t~ 180’180; f(x) ----------- cotx—tanx ;(nE Z) A)í-V2;>^] B)[-V5; V8] C)[-V7; 74] D)[-V3; 72] . „ _ 71 «71 A)R-,m B)R-(2n+l)- C)R- — tm ti D)R-— E)R-(4«+l)y E) [ -79; V3] 10.- Encontrar el rango de la función/definida por: fix) = senx.cotx+cosx cotx 1 Funciones Trigonométricas PP14 847
A)(-2;0>o(0;2> B)<-2;7>o<0;7) C)(-5;0>u(0;5> D)(-4;0>u(0;0> E)(-l;O>u(O; 1> 11 .- Delimitar el rango de la función f definida por: /x) = 2sec2^j +2,sixe IC A) (2; 3) B)(9;4> C)(5;6) D)(7:7> E)(4;6> 12 .- Determine el rango de la función f defini- da por: /¡jc) = sec‘x + 6|secjc|+ 11 A)[14; + oo> B)[18; + o°> C)[16; + «>> D) [9; + E) [5; + ») 13 .- Evaluar el rango de la función/definida por; _ 2 fix) — x . secx.sucG n n 1’7 A)[o;^ D) 0;^ E) 0,2?- 14 .- Encontrar el rango de la función /definida por: fix) =1 + 2 csc A)[3;5> B)[3;2> C)[4;2> D)[7;9) E)[4;5> 15 .- Sea/una función definida por: fix) = sec x + csc x, determine el rango de la función: A)[6;oo) B)[5;oo) C)[4;~> D)[3;oo> E)[2;o°> ^6.- Determine el rango de la función/defini- da por: f(x) = Jsec2x+csc2x A) [6; +“) B) [4; +<») C) [2; +°°) D)[3;+°o> E)[7;+°°) 17.- Delimitar el rango de la función /definida por: o 7 fix) — csc~x + 2 sec"x A) [5 v/2 + 3; + °°) B)[2>/5 + 3;+oo> C)[7^ + 3; + oo) D)[3v/2 + 3; + <*>) E) [2-J2 + 3; + ») 18 .- Encontrar el rango de la función /definida por: |sec.v|-l fix)='-----— J * tan-x A) <0; l/3> B) (0; l/2> C) (0; l/5> D)(0;l/4> E)(0;l/6> 19 .- Sea/una función definida por: /x) = sec~v + 4 tan x, evaluar el rango de la función. A)[-3;+oo) B)[-7;+°o> C)[-2;+°°> D)[-5;+°°) E)[-9;+oo) 20 .- Hallar el dominio y rango de: F(x) = (cos 3x + sen 3x tan x)tan 2x; k 6 Z Problemas de Trigonometría y cómo resolverlos •iiRACSO Pbditdbbb
A) D/: R-{2*+l)^ o(2*+1)| 23.- Hallar el rango de la función: H/: R-{-2; V2 } flx) = sen 4 2 1 X 4-COS X -5n 6 —2n 3 B) Df. R-(2*+l)^ 4 Rf: R-<-2;2)-{-V2 , Jz | 13 13 A)\4’16 B) 4 3 15 16 -3 1 C) 4’2, „ 71 Q D/:R-{2*+l|- Pf: R-{2} {71 7t (2*4-1)- 0(2* + 1)- K/: R-{-2; , y[z ,2} {71 I (2*4-1- 1 4 J Rf: R-{-2;2-/7} D) 2.22* 4’16 3 E) 4 7 8 24.-Si:/(x) = cosx ---------y su rango pertene- cosx+senx 3-1.1 2 intervalo de: (O; 2n). ce a A) n n 4’Í calcular su dominio en el 5n 4n T’T B) nn 4’3 21.- Hallar los puntos de discontinuidad de: cos(tanx+cotx) F(x) =----;*g Z I I x ] 1 sen sen neos— 2 II E) n n A)(2*+l)y B)(2* + l) — n Q*- *n n D) — E) (2*4-1)— 2 4 22.- Dada la función: F(x) = sen x 4- cos 3x; x g n 3n 2’~2* hallar la suma de valores de “x” para el cual la función es nula. 3n A)4 5n 5n 5n B) — Q4n D) —- E) — Z 4 o 5n 4n T’T n n 6’3 D) n n 5¡’Í 5n 4n T’T 5n 4n ~9~’~ 25.- Hallar el dominio y rango de la siguiente función: 2 cot- fix)---------— , (hg Z) cscx-cotx . mt A) nn; R B) nn; R C) — , Rq «7t mt . D)y;R E)~R+ 26.- El dominio y rango de la función: fa) = eos2 X tanx sen2x cotx , (* G Z) ; son: Funciones Trigonométricas PP14 849
A) D/R-Atc B) D/:R- — R^R-{0} R/=R O kn D/R- — D) D/= R - kn R/=R-{0} Rf=R 3) kn D/=R- — R/=R 27.- Hallar dominio y rango de /para: senx tan x+cotx fí,x) = , conÁ:e Z n 7 1-cosx secx kit A) R-jbr B) R- — Q R-2fcrc kTt 71 D) R- — E)R-(2£+l)- 29.- El rango de la función definida por: f(x) = (sec x - cos x)(csc x - sen x) es: A) B) O D) 7t_7t 4’4 D/= D/ = tan2x ,si: /(x) =--~ J ’ tan4x 7t_7t 7’4 7t 7t 4’4 / 71 71 D/=\_T;T J \ 4 4 R/= D/= A)[-l;l]-{0} B)(-l;l> C) 2’2 -{0}n n n 8’8 71 71 7’7 R/= (0; + o°; R/= <o»; 0> D) —— 2 2 {0} E) -1;1 cosxJ —1 30.- Hallar el rango de/(x) =--5---- eos x—1 1 r —- ’2 -{0} A)<-1;1> B)(0;l> C) E) \2 / L2 J VALORESMÁXIMOSYMÍNIMOSDELASET. ti n 4’4 7t 7t 8’8 31.- Calcule el valor mínimo de la función / definida por: „ . 4 4 j(x) = sec x+ csc x A) 8 B)7 C)6 D)5 E)4 R/= 2 32.- Si el máximo valor de la expresión: / 71 7t\ 71 71 E> \ 4’4/ ’ 8*8 R/ = <—1>-{0} 28.-El Df n R/ para: 1 7 2 , M=2sena+ ~ sen 2P+ ~ cos /B,es“P”y el mínimo valor de la expresión: N = (2sec0+ l)(2sec0- l),es“Q”; calcule: W= ^Q + 4P RACSO DITOKBB PP17 850 Problemas de Trigonometría y cómo resolverlos
A)s/19 D)#H B)s/15 E)-s/17 C)>/13 12 4 1 71 /(jc)=Jcos x-cos x-~ es(2n+l) — ,G Z 33.- Determine el mínimo valor de la expresión: 7777 W=6csc o+5sec"<¡>+4cot 6 + 3tan 6 - 2cosP+scna si todos los arcos son diferentes. II. El valor mínimo que toma la función fi r) = 2cos ~ x + 3 sen x , donde: A)6 B)7 C)8 D)9 E)10 34.- Hallar los valores de x para el cual la fun- ción g alcanza su máximo valor: 2 x #(x) = 2 sen x - cos x + 2 sen — -1; k g Z arc sen — < x < — es 3 l4J 2 III. El periodo de/pe) = sec( Tt sec x) mas el periodo de g(x) = csc4x es 2ti A) WF B)FFV OVFV D)FVF E)VW A)(2*+l)^ B)(2*+1)J C)(8Á + 3)| D)A^ E) (8A + 3)^ PERÍODO DE F.T. 35.- Sean:/(x) = sen43x, g(x) = cos3Zx y h(x) = J[x) - g(x) cuyos períodos son: T,. T2 y T3 res- pectivamente. Calcular ^3T,T2T3. 71 71 371 A)- B)y C)7t D) — E)2tt 36.- Determinar el período de: M(x) = cos(tan x - cot x) + 2 N(x) = cosftan 3x) + cos (cot 3x) + 1, respecti- 38.- Al graficar las funciones/(x) = |4 sen x|, g (x) = sec x, calcular la suma de todas las abscisas de los puntos donde:/(x) = ,§(x) - Tt < X<7t. A)4n B)cero D)20n E)32it Q87t 39.- Analizar la verdad (V) o falsedad (F), res- pecto a la función: fix) = 2 cos(sen x - cos x) I. Dominio: [2 cos V 2 ; 2] n. Rango: R ni. Máximo valor: 2 IV Período: Tt vamente. 7t Tt Tt Tt .71 A)7t; - B)? ’6 C)4 Tt Tt D) 2'3 Tt E)I . 71 ’12 A)FFW B)VWV QWFF D)VFW E)FFVF 40.- De la función definida por: /(x) = sen 37.- Indicar verdadero (V) o falso (F) en I. El dominio máximo de la función Hallar su amplitud y período respectivamente: Funciones Trigonométricas PP14 851
5ti 5ti A) 2 sen— ;ti B) 2 sen — ; 2ti 5ti 7ti C)2sen~ ;ti 10 D)2sen— ;2n lo 5ti E) 2 sen — ; 2ti lo 41.- Hallar la regla de correspondencia de la siguiente sinusoide: A)y = 4sení 2x+— [-2 I 3 I A)y = 4cos 9jc ti A T 6) + 2 B) y = 4 cos ( _ Tt' |+2 ~4 ( 9x 71 C)y = 4cos l5 5J (9x 71? + 2 D)y = 4cos 15 'Qr ti 'l 1 + 2 E)y = 4cos 1 1 + 2 B) v = 4sen í 2jc + — [+2 3 I C)y = 4sen 71 i + 2 D)y = 4 sen í 2jc + y j+ 2 E)y = 4sen | 3x + — | + 2 l 3 J 42.- Hallar la regla de correspondencia de la siguiente cosinusoide; 44.- Para la función: F(jc) =-~----5—, cos 2jr—cos x hallar el rango y el período de F(jr) A) R/:<-3; 11 - {0} B) R/:R-{0;-3} T = 7t T=7t ** RACSO JP DITORSI PP17 852 Problema de Trigonometría y cómo resolverlos
Q Rf.R- T=7t D) T=7t E) R/R-[-l;3) T=7t 47.- Graficar la función definida por: GRÁFICAS DE LAS F.T. 45.- Siendo: F(x) = sen 2x + 2 sfl cosí — — x ^4 Obtener la gráfica de: -JF(jc) + 1 cos jrlog, (senx) H«=-------7 \ covl x---1-1 2 46.- Graficar la siguiente función: 48.- Graficar la función definida por la siguien- te regla de correspondencia: Funciones Trigonométricas PP14 853
51.- Graficar la función: 3 2 49.- Si el área de la región sombreada es — p , hallar la ordenada del punto “P”. A)-1/3 B)-l/2 C)-l/4 D)-l/7 52.- Esbozar la gráfica de: P(jc) = x - cos jr 50.- Graficar la función definida por: cosxcos2x fíx)=------------ cos3jc + cosjc PP17 854 Problemas de Trigonometría y cómo resolverlos RACSO
MISCELÁNEA 53.- Si la función/está definida por: fix) = cosí xh— I senx-sen x--cosx l 6 J ( 6 J ¿para qué los valores deje del intervalo [0; 7i] la función es negativa?. /ti 3ti\ /ti 5ti\ /ti 2ti\ A)\4’T/ B)\4’T/ C)\6’3/ /n 5n\ /ti 7ti\ D) \6’ 6 / E) \3* 8 / 54.- Hallar los puntos donde la función H (x) no está definida: cot x +senx H (x) ----------:—¡-------- ; n G Z cot x( senx — cos x ) nn 7t A) — B)nn C)(2n+1)- «71 D) — E)2/m 4 55.- ¿Cuántas de las siguientes funciones son pares? I. fix) = tanx sec |x| 56.- ¿Cuál o cuáles de las siguientes funcio- nes trigonométricas son inyectivas?. I. fix) = sen3x ;xG senx EL g(x) = cot2x(l+sec2x)+tanx;xG jX X 2 ID. /i(x) = sen — sen — + cos x; xg (ti; 2ti} A) Sólo I B) Sólo I y II C) Sólo II D)SóloIyni E)i,nyin 57.- Calcular el área de la región encerrada por las gráficas de las funciones: cosx + sen2x f(x) = ~ ~ ~ ; g (x) = 1 a h (x) - -1 l + senx-cos2x para x G (0; 2 7t) 71 7 7 371 n A) -u2 B)nu2 7 57t 7 D)2tiu E) — u EL 2 /(x) = sen(x )cos ID- /(x) = |sec x| csc 3x IV fix) — csc í — I + |x| I ti J V. fix)=X CSC 9 A) 3 B)5 C)4 D)1 E)2 Funciones Trigonométricas PP14 855
DOMINIO DE LAS F.T. INVERSAS 01.- ¿Cuáles de las siguientes funciones tri- gonométricas son inyectivas? I. F(jc) = 2 sen 4jc; jc G /ti 3ti \8’~8~ árceos 6 —arc sen— 3 Calcular su dominio fa) = — — 2 3 1 4 4 /n,2n II. G(jc)= — {eos jr-sen jc};jre 4jc 4jc IILH(jr) = csc— + cot~;xG ( 4’4 A) IaII B) Sólol C)IlAm a,H;í]101 / 1 Vz\ B)\“ I;V/ D) Sólo IB E) IaHI 02.- Si se verifica que: I tt /---z------- A(jr) = J —-arcsenx + yjn-¿arccosx calcular su dominio, para: jc G 2’4/ A) ( 0;— \ 4, B) C) RANGO DE LAS F. T. INVERSAS 04.- Sea la función: P>v4 / 1 1 e) 03.- Dada la función: 2 arc cosx--- ta---------4 ----arc sen jc 4 hallar el rango de dicha función: PP18 856 Problemas de Trigonometría y cómo resolverlos RACSO DIYOB.BS
A)l=? B) -kj Q -li— 5 A) T-' B) -3 r 2 ’2 O y-1 D) -1;2 D) -3 ¿ 4 ’2 05.- Determinar el rango de la función: DOMINIO Y RANGO DE LAS F.T. INVERSAS 1 08.- Hallar el dominio y rango de la función: fíx)=------------------—----- | arc tan x | —21 ore cot jr | 1 I flx) = — ere sen A) ¿.2 71 ’ 37t 1 D)------, 71 2ti 3 2x —1 3 71 i B) 1--A 71 ’ 471 E) _1 ,__3_ 71 ’ 2ti A) Df = [-!;!] 771 Rf= -ti;— 12 Q \ 71 2ti 06.-Dado: F(jc) = arc cos (sen6 + cosSc) su rango está dado por: B)D/= -|;2 C)D/= -1;1 3 D)Df= -!;- © Df=[-1;2] -7ti -ti Rj<=[ 12 ’17. -3ti -ti 4 ’ 4 Rf= R/- 3 ’ 12 12 12 A) (fyarccos— \ 3 „ 1 1 , B) — creeos—;1 2 4 09.- A partir de la siguiente función: C) |0;- 4 D) 0; arc eos— 4 g(x) = 2 arc cos I sen — , I 2 J hallar el período y rango respectivamente: E) creeos—;! 3 A)T=^,[-7t;7t] B)T = ti, [-2; 2] 07.- Sea la función: 71 aresenx---- fix) —-----------, J ’ 71 creeos jc + — 4 C)T = 2ti, [0;2ti] E) T=47t,(0; 271] PROPIEDADES DE LAS F.T. INVERSAS D)T = 471,[0;2ti] 10.- Calcular el valor de: hallar el rango de dicha función: Funciones Trigonométricas Inversas PP18 857
14.- Calcular el valor de: árceos arc tan A) 1 B)-l/4 Q-l/2 D) 1/2 E) 1/4 «cesen I — Torceos 11.- Reducir: orcsen(sen5) — orccos(cos6) K=-------------------------------- «rccos(cos 3) — arc tanftan 4) +1 Tomar; 71=22/7 A)2/7 B)7/2 05/3 D)ti+1 E)-l/2 12.- De las siguientes proposiciones: I. Si: «rc cos x = fc => - 1 <x< 1 O. Si: Q = arc cos (sen 2x + cos 2x) => x puede tomar cualquier valor para que exista Q. 71 III Si: a = arc sen(sen 6) => cos 0 = — v 0 = 37t — existe a, son correctas: 2 A)FVF B) FFV QVFV D)FFV E)FFF 13.-Calcular: A) 5/2 B)7/8 C)8/9 D)3/4 E)l/2 15 .- Evaluar la expresión: F = arc sen(cos 2) + arc cos (sen 3) A) 1 B)25 03 D)7t-5 E)5-ti 16 .- Calcular el valor de: r— ( ti 1 12 A A= v 13 eos-------creeos— l4 2 13 J A)0 B)1 C)2 D)3 E)4 17 .- Determinar la verdad ó falsedad de las siguientes proposiciones: I. arc sen x = arc cos 1 —x2 ; V x e [. 1; 1] II. cos(arc senx) = -Jl-x2 ; VxG [.1; 1] /j_ 2 III. tan(arccosx)=----—; VxG [.1; 1]- [OJ x A)WV B)FFF QFW D)FFV E)FVF 2 arc cot (2 + V3 ) 18.- Simplificar: A)71/2 B)2 Q-ti/6 D)-2 E)0 0 = arc tan —---- [x2-!, para todo x e [O; 1) A) 2 arc tan x C) ti - 2 arc tanx E)7t/2 -arc tanx + arc cos -- IxM B) - 2 arc tanx D) 7t-4«rctanx Problemas de Trigonometría y cómo resolverlos RACSO UPSDITOKIB
19.-Calcular: E = cos árceos---- 3 A) 20.- Calcular: 0= árceos A)n/6 B)7t/3 C)2n¿3 D)5n/6 E)n/4 23.- Graficar: F(jr) = | csc (are csc a2)- 3)| SITUACIONES GRÁFICAS 21.- Del gráfico, calcular las coordenadas de "P": 24.- Hallar la regla de correspondencia de la D) A) r A.B.C gráfica mostrada, e indicar el valor de: ——— /3 n 2~*6 22.-Graficar: A) 2 B)l/27t C)-l/7T D)4 E) 1/2 Funciones Trigonométricas Inversas 25.- Hallar la regla de correspondencia de la
(71 A)y = 3 arc sen I l + y (*+Q n B)y = 2arcsenl I + — ( 2x-Q n C)y=3arcsenl I + — ( 2x + l^ n D) y = 3 arc sen I _ + — í X + 1 I " E) y = 3 arc sen c + — j ) 4 26.-Graficar: coscare cos x) + sen(arcsenx) M =---------------j; x < 1 tan(arc cot(x - x“ )) y A)7i/4 B) 571/12 C) 771/12 D)7i/6 E) 71/12 28.- Hallar los valores de “x” e “y”; que satis- facen la igualdad: 2 are senx = 7t csc y (ríe Z) A)x =±1 y = (2m 71 B)x=l y=(2rn 71 C)x=-1 y=(2rn 71 D)x=±l y = «7t E)x=±l y=(2rn -1)71 29.- Resolver: (arc tan x)(arc cos 2x)- (arc cot x)(arc sen 7I2 2x) = 71 arc cos 2x - — 4 30.- Resolver: 1 C)±I MISCELÁNEA 27.- Calcular: y dar como respuesta la suma de soluciones. 0 = 2 arc tan (V3+2V2)+ A)4 B)3 C)5 D)2 E)6 arccsc(- )3arccot- (V7-4V3) 31.- Resolver: 2 arc tan(cos x) = are tan (2 csc x) ^ÍKACsO P*DITOKBJ PP18 860 Problemas de Trigonometría y cómo resolverlos
A)7i/3 B)7t/6 C)7i/4 D)57i/12 E)7i/12 32.- Dado: F(x) = arc cos (sen6 + cos6x) su rango está dado por: A) (O;«rccos— B) —árceos—;1 2 4 Q 0;— 4 D) 0; «re eos— 4 36.-Resolver: /— n arc cos x V 3 + arc cos = — J3 A) 1/3 B) 1/2 Q -y- D)-y- E)±l/2 37.- Resolver la ecuación: 2x x 71 árceos ~ - árceos ~ = y A)-1/2 B)-3/2 Q-2/3 D)3/4 E)1 38.- Resolver para «x» en: E) creeos—;1 3 x n «retan — -arccot(x + 2) = — 33.- Resolver: arc sen x - arc sen(x - 1) = arc cos x y dar como respuesta la suma de soluciones A) 1/3 B)0 C) 1/2 D) 1 E)5/8 34.- Calcular “x” de: 2Í zf 3xA cos «resen — =0,5+sen árceos— l 4J l 4 J A)2/3 B)±V3/3 C)-4/9 D)+l E)±4/3 2 2 35.-Calcular: tan 0 + cot 0,si: arc sec (cot 0) -arc csc (4 tan 0) = tan kn.- keZ, dar como respuesta un valor: A) V2 B) V3 C)2 D)2/V3 E)4 39.- Determinar el valor de «x» que satisface a la ecuación: 2 cos(2 arc sen x) = e indicar la suma de los valores absolutos de las soluciones: 34-15^3 4 B)4 + 2-j2 C)l±3-j3 E)6-2>/3 Funciones Trigonométricas Inversas
ECUACIONES TRIGONOMÉTRICAS 01.- Determinar la suma de soluciones de: 2senx+cscx=3,en:O<x<7i A) 71/6 B) 7i C)37t/2 D) 571/6 E)2n 02.- Indicar el número de soluciones positi- vas menores de 1 vuelta en: sen2x+senx = cos(2n+l)y VneZ A)1 B) 2 C)3 D)4 E)5 03.- Identificar una solución de: 2 2tan x- secx= 1 2 n 7i A) ore eos — B) — C)t 3 4 3 1 D)2n E) arc cos — 04.- Al resolver la ecuación trigonométrica: sen 7x sen 5x = cos 3x cos x, dar como respuesta la menor solución positiva: n 7i n 2ti n A) ~ B) - C) - D) — E) — o 4 o o lo 05.- Sea la ecuación: 2f n A cos2x-2sen------x =0, l4 ') indicar el número de soluciones comprendi- das en;0<x<7i A) 1 B)2 C)3 D)4 E)5 cosx senx 06.-Resolver:------ + ——~- =O;(á:g Z) cos2x sen2x n nk 7i A)2áti± ~ B) — C)nk± ~ n tú., D)tU± - E) —' 6 4 07.- Al resolver: tan cos x^ - cot(7t sen x) = 0; se obtiene: A) arc cos +kn 2 y/5 B) 2 árceos—+ 2An y¡5 C) arc cos----+ 2x71 3 y/5 D) arc cos —— + (2k + l)n E) 2¿tt - arc cos 08.- Resolver: n n . cos x- sen x=l ; en el intervalo — ro entero natural. n 3tt\ 2'~2/ siendo: n un núme- ^LRACSO W^IDITOKBa PP19 862 Problemas de Trigonometría y cómo resolverlos
n 71 71 71 A)0; — ;ti B)--;0;7i C)--;0;-;7t 71 D) -;n E) O;ti 09.- Para qué valores de “a" la ecuación: (sen jc + cos jc) sen 2x = a (sen3x + cos3a) tiene soluciones entre: 7t/2 y n. ti 3ti ti 5ti ti 4n A)0;-;y B)0;-;y C)0;-;y n Zn 4ti 5n D’0;rT E)0:t;t 14. Resolver la ecuación: 3 sen 2 a = tan v - sen 4a ; (k G Z) B) (~T:0) 2 1\ C)|_ 3 3/ kn A)V kn n C)(2^+l)- kn D)T kn E)t 15.- Calcular la suma de soluciones de la ecuación: 10.- Determinar el número de soluciones de la ecuación: sen x + sen 3a = cos x + cos 3x Vl + sen2x - -Jz. cos 3a = 0, en. 0<a<2ti comprendidas entre (ti; 2ti). 117t 7n 9n 5n 11ti A)— B)— C)— D)— E) — 2 2 2 2 o A) 2 B)3 C) 4 D)5 E) 6 11.- Encontrar las soluciones de: 3 3 3 sen jc + cos x = — sen 2a +1 2 comprendidas en el intervalo como respuesta la suma. 5ti\ —7t;— ) y dar 2 / 16.- Resolver: 4 sen 3a sen r = sen 4a csc 2a ; (k G Z) kn n A)T^ kn n kn n C) T ± i k n D)t kn E)t A) — B)ti C)27t D) — E)37t 12.- Hallar la suma de soluciones de: 5 sen 2x + cos 2a = — en: x G ( 0o; 360°) A) 180° B)540° Q750° D)450° E)720° 13.- Resolver la ecuación trigonométrica: x sen — + cos a = 1 17.- Determinar la suma de soluciones positi- vas y menores de 2ti, de la ecuación: sen 2a - sen x - 1 + cos 2 a - cos a 3ti 571 7n A) T C)T 9ti 1171 D) T E)v Ecuaciones e Inecuaciones Trigonométricas
SISTEMAS DEECUACIONES X = Á7I + 7t/2 Q 18.- Encontrar el número de soluciones com- prendidas en el intervalo <-7t;7i) de la ecuación: y=kn-n!2 , 271 x = kn + y sen2xcos3x 1 ------------ = — secx sen4x—sen2x 4 D) , 2n y = A7t-T A) 6 B)5 C)4 D)3 E)2 E) X=Á'7t+7t/4 19.- Resolver el sistema de ecuaciones: y = kn-nlA 1 - tan x --------= tan v 21.- Siendo: x, y los menores valores positi- vos que verifican el sistema: n 6 5ti kn n kn ’ V 571 71 kn ' ~2 571 71 C)x=-+ín; - + kn 71 571 0),= - + ^; - kn ’ ~2~ 71 5ti kn E)x=—-Air; y=~^ + T 20.- Hallar las soluciones que satisfacen el siguiente sistema de ecuaciones: tanx = 3 tany senx cosy = sen^cos(x- y) sen3 x = y sen y eos3 x =—cosy 2 Í3 y ----2x 2 A)V5-3 B) - -22^3- t- v2—2 D)l-V2 E)-^— 22.- Si: x, y, z son ángulos positivos que perte- necen al intervalo [0; ti/2], evaluar: (x + y + z), si además: V2 y/2 sen x sen y =— ;senysenz=---- 4 2 1 sen x sen z = — siendo:x + y*kn,keZ .. . 271 A) x = kn+ — v=kn-n]Q> A)7t/12 D) 1171/12 B)5tt/2 C) 771/12 E) 1371/12 23.- Resolver el sistema: B) x=A-7t+7t/3 i n y = kn- j *+y=?;•?<* 7 2 2 tanx—tany =0, 71 2 PP19 864 Problemas de Trigonometría y cómo resolverlos
dar como respuesta: x- y A)0 B)7t C) 37t/4 D)7t/2 E)3tt/2 INECUACIONES 24.- Resolver el sistema: \ / 271 571 —;n) B)(—;~ 4 / \ 3 6 , ¡7n 3>n\ /117l \ D) \ (-;7t) \12 4/ \12 ¡ /5n C) (—;7t \ O 1 E) 371 571 4 ’ 6 *+y=-y cos 2x + cos2y = 0 27.- Resolver: •Jsenx + V cosx > 0; k G Z Donde: «G Z nn 7ti ti nn A)*=t+t;,=h-t nn n n nn B,x=T+3 3 ’T nn 7n n nn C)X= 2 + 12 ’J=12 ’ T 7n n D)jt = 2/m+ — ;v= —-2nn 4 12 nn n n ¡tn E)x=T+ 3;V=3’T 25.- Hallar los valores de “x” comprendidos en ( 0; n) para que: F(jc) = sen 3jc - sen 2x + sen x, tome solo valores positivos: kn n A)—<x<2kn B)2nk<x<2kn+ — kn n C) — <x<2kn D)kn<x<(2k+ 1) — kn ~6~ 28.- Resolver la inecuación: sen n 3 ln I7n para:xG ( 9 7t_47t 2’T _ 2n . ' E) —;2ti 3 ¡ / 7t\ /n \ 7 D) 7t\ n 2n > 6/ \4 3 29.- Resolver: sen3x-V3 cos2x + 3cosx + 2 E) ' 2ti\ ¡2n 4ti o;—)vj(—;— i 3/^\3 3 sí:xg (0;n) 26.- Resolver: 2 sen 4x < 3 tan 2x, A) 271 471 T’T B) n 2n 2’T para “x” en el recorrido de: 'ti \ —;7t) >2 / / 4ti D) ( E) 271 471 3 ’ 3 Ecuaciones e Inecuaciones Trigonométricas PP19 865
30.- Resolver la inecuación: 1 2 4 — - 8cos x + 8 cos x < 0 2 A) (----;arctan — \ 2 3, para: x G / 71 571 A)\3’3 0;— 2 / 71 5ti C) \12’T_ 71 571 E) [12’12. 31.- Resolver: B) 0;— 12, D) 0:— ’ 12 5ti tt 12’12 71 . 71 4’2 B) (----;orctan—)(—;7t-orctan — \ 12 3/ \2 3, / Tt 1 \ C) l—-,arctan—) \2 3/ /ti 5ti\ Í3ti1 } \2’~6/~ 1 4 J E) (---;ti—arctan—) \ 2 3/ 33.- Resolver las siguientes desigualdades: sen 4x > 4 senx sen 2x sen 3x; k G Z, 71 2 arc tan (x - 2) < — 4 dar como respuesta un intervalo solución: A) (— o°; 1- 2] B)(-o°; 2 -1] .. /, 5ti , 7n A.) lkit +— ;kn +— \ 4 4 , C)<-~, D)<O°; 1] E)<-~;1> B) (2kit;2kit + - \ 4, 34.- Resolver: 1 1 arc cos — < árceos (sen x) < Tt - árceos ~, / Tt C) (kn,kn+~ \ 4, para:xG / 371, Tt D) < kn + — ;k7t+ — \ 8 2, A) (0,arc sen— \ 3 1 ’ Ti—arcsen —;ti 3 , /, 5ti , 17ti E) < kn +—;k7t+--- \ 3 6 , / „ 71 B) \ ; 2 32.- Resolver: 5 + 2 cos 2x < 3|2 senx -11; 3ti —;ti 4 , para:xG Tt —;ti , 2 . * 1 D) ¿zresen —; 7t—¿z resen — 3 3 Problemas de Trigonometría y cómo resolverlos ’ÍIRACSO Pbditokbb
E) arasen 2V2 ---;Tt—aresen 3--3 A) ¿ _r 2’4 B) £ 2' 5 5 C) (0;- \ 3 35.- Resolver la inecuación: senx> 1- cos2x. D) 1 2’ 5 5 para:O<x<Tt / Tt A)(0; 7 3tt —;tt , 4 D) Tt 6 5rt T B>(»¿ C> °*6 5tt —;tt 6 , 5tt ----’.Tt 6 2n E) Tt 3’ 3 36.- Si: x e (0; Tt), resolver la inecuación: tan x < sen 2x A) ¿O;— \ 6 , B) (O;— \ 3 , C) (»,- \ 6 D) 0; — 3 37.- Resolver: Tt E) (0;--)u(-;Tt \ 4/ \2 , 4 tan~x-(V3 + l)tanx + considere: 0 < x < Tt A) Tt 3 Tt 2 B) Tt 2tt i’T C) (0;- \ 3 D) Tt 4*3 Tt E) Tt Tt 4’2 38.- Resolver: arc sen 2x < arc cos x 1 E) 39.- Resolver: V 0 e ( 0; Tt) la inecuación: 1 1 < 3 - — sec “6 < 2 2 A) B) C) D) E) Tt Tt 4’3 Tt Tt 4 3 2tt 3tt T’T 2tt 3tt 3 ’ 4 Tt 4 3ti 4 5tt —;?t , 6 , 40.- Resolver: — í Tt 1 2 sen 4x cos 2x - eos----------2x < 0 l 4 I A) (0 ; Tt/16) U [n/8 ; 5Tt/16] B) <0; Tt/8) cj < rt/4: it/3) C) (0, tt/12) (n/6; ti/4) D) (0; n/12) U <Tt/4; Tt/3) E) <Tt/8; 5Tt/16) O <Tt/6; n/4) Ecuaciones e Inecuaciones Trigonométricas PP19 867
LEY DESENOS 01.- Si AM es la mediana relativa al lado BC de un triángulo ABC y m Z ABC = 30°, m Z ACB = 15°, calcular m Z MAC. A) 15° B)20° C)30° D)45° E)60° 02.- En un A ABC, de lados a, b, c y circunradio (R); la expresión equivalente de: C W = 4 eos—.eos 2 A-B 2 es: fl2+í>2 A)--------- B) (o + Í>)-R Q 03.- En la figura mostrada, rnZBAD = 3a, rnZDBC-rnZDCB = 2a, AD-BC, calcule: W = (cot 3a + cot 4a).sec 2a.sen 4a A) 1/2 B)1 C) 42 D) 4$ E) 2 04.- En un AABC de lados a, b, y c, m Z A = ti 2ti 4ti —, m Z B — , m Z C = ~~. 7 7 7 Determine el equivalente de: W = b l + c'1 A) - B) — C) D) - E) — a 2a b+c a b-c 05- En un AABC, de lados a.byc simplificar irnos B + c.cosC - a la expresión W = —:--~ — psenB-c.senC ( B— A) 2 tan(B - C) B) -2 tan I —-— I f B—C^ _ íB+C'l C)-tan ------ D)cot ------------ l 2 J l 2 J (B-C'Á E)-cot ------ l 2 J 06.- En un triángulo ABC de lados a, b y c ; eos2 A eos2 B eos2 C ------+---------+-------= H, calcular: abe _________________R _______________ cos A.cot A + cos B.cot B + cos C.cot C ’ circunradio A) — B) — C)— D)-— E) — H 2H H H H 07.- Si en un triángulo ABC de lados a, b y c; b(b + c) - 9 y mZN = 2/mZB. Calculara. A)9 B)4 C)3 D)5 E) 1 08.- Las medidas de los lados de un triángulo ABC están en progresión aritmética, calcular H = sen C - 2 sen B + sen A, siendo a > b > c. A)0 B)l/2 C)4 D)1 E) 2 RACSO D1TO11I PP20 868 Problemas de Trigonometría y cómo resolverlos
09.- En el A ABC mostrado en la figura: A) 30" B)50" C)20" D) 10" E) 40" AD = BC, hallar mZ[l. 10.- En un AABC de lados a, b,yc.a sen(A + B) = fe.sen (A + C), simplifique: a. cosB-fe. cosC W=------------------- fe.senC — fl-senB A) tan(B + C) B) cot(B + C) C) tan(B - C) 11 .- En un AABC de lados a, b y c; BM es mediana (M en AC) si mZB AC = 2,«ZBCA y mZAMB - 45". Hallar mZACB. A) 5° B) 10" C) 15" D) 25" E)35" 12 .- En un triángulo ABC de lados a, b y c circunradio R se cumple que: cosB fe.cosC+c.cosB a.cosC + c.cosA cosA a.cosB + fe.cos A cosC = 2R Hallar: tan A tan B tan C + tan A +tan B + tan C A) 1 B)2 C)3 D)4 E)5 LEY DE COSENOS 13 .- En un A ABC de lados a, bye, si — 2rnZ.C, cos C = 3/4, c = 4, con (b > c), hallar la medida de lado a. A) 1 B)2 C)4 D)6 E)8 14 .- En un triángulo ABC de lados a, b y c si mZB = 4znZA, entonces cos A + cos 3A es: A) - B) — C) — D) — E) — a 2a 3a 4a 5a LEYDEPROYECCIONES 15 . En un AABC de lados a,b,cy circunradio R; al simplificar: (a+c) cos B+(a+b) cos C+(b+c) cos A H = “ “ + 1 2R(senA+senB+senC) se obtiene: A)2 B)1 Q-l D)-2 E)-3 16 .- En un A ABC, de lados a, b ye: 2 C 3b 2 a a. cos — - — =c.sen — -c 2 2 2 2b calcule:--- a + c A) 1 B)2 C)3 D) 4 E)5 17 .- En un A ABC de lados a, b y c: 7 7 7 7 7 24S = a(l> + c) cos A+ fe(«" + c") cos B + c(a + b) cos C S —> área de la región triangular ABC. Determine la medida del circunradio del AABC A)6 B)3 C)2 D) 1 E)4 18 .- En un AABC de lados a, b y c, de semiperímctro p, reducir: 2 A 2 B , 2 C bc.cos — + ac.cos — + «fe. cos — 2 2 2 xv------------------------------— P2 A) 1/p B)p2 C)p D) 1 E)2 ÁREA DEUNA REGIÓNTRIANGULAR 19.- En un triángulo ABC, de lados a, b, c, se cumple: a + b+ c = 9, abe = 24, S es el área de la región triangular ABC, halle en términos de Resolución de Triángulos Oblicuángulos PP20 869
S, la expresión equivalente de: ABC W = COS — .COS — -COS — 2 2 2 7S 5S 3S 3S S A) 77 B) — C) — D) — E) - lü lü lü o o 20 .- En un A ABC de lados a, b, y c, donde «S» área de la región triangular ABC. 2(b2—a2) la expresión: E =--------— , es igual a: cot A—cot B A)S B)2S C)3S D)4S E)5S 21 .- En un AABC de lados a, b y c; reducir: A B W = c + p. tan —. tan —, 2 2 siendo p el semiperímetro de dicho triángulo. A)p-fc B)p C) c + a D) p/2 E)2p 22 .- En un triángulo ABC de lados a, b y c; se verifica la siguiente relación: 4R2sen3B + 8S cos B =16 sen B, calcule la longitud de la mediana (en u) relati- va al lado b, (R: circunradio), S: área de la re- gión triangular ABC. A)2 B)5 C)4 D)6 E) 8 23 .- El área de la región triangular ABC es S, el lado BC tiene como longitud a y los ángulos MNB y CNP tienen la misma medida. Siendo M, N y P los puntos medios de los lados del triángulo ABC: (N en BC y P en AC). Halle la tangejite del ángulo ABC en términos de a y S (en rZ) 4S 4S 4S 4S S A) 2 B) 2 C) 2 D) o E) 2 a la 5a 5a a 24 .- Al simplificar la expresión siguiente don- de S es el valor del área de la región triangular ABC. b2 —c2 senB.senC E=--------.-----------, se obtiene: 2 sen(B—C) A)5S B)4S C)3S D)2S E)S 25 .- En un triángulo ABC de ladosa,byc si: r —> inradio • R —> circunradio p —> semiperimetro > hallar: E = abe A)2prR B)2p/rR C)prR D)4prR E)4p/rR 26.- En un triángulo ABC, de lados a,b,cy semiperímetro p se cumple: AS = {p-b)(p-c)+p{p-a) , S: área AABC. Hallar la medida del ángulo A: A) 45° B)60° C)30° D) 75° E)15° 27 .- En un triángulo ABC de lados a, b, c y área S la expresión: „ r? A B C E = 2 J abepsen—sen — sen y , es igual a: A)S B)2S C)3S D)4S E) 5S 28 .- En un triángulo ABC de lados a,by c, m¿ A = 30°, S = , (/nZB) > (»nZC) y S es el 4 área de la región triangular ABC. Halle »iZB. A) 30° B)60° C)120° D) 150° E) 160° LÍNEAS NOTABLES 29 .- En un AABC de lados a, b, c circunradio R^, /n^A =45°. La expresión E = fc‘cos*B + c cos“C, es igual a: A)R2 B)2R2 C)3R2 D)4R2 E)5R2 30 .- En la figura mostrada, halle la medida del ángulo a si EC = 2AB PP20 870 Problemas de Trigonometría y cómo resolverlos RACSO WlDITDlII
A) 11°15’ B)22°30’ C) 33°3O’ D)25°30’ E)22°20’ 31.- La longitud de los lados de un triángulo ABC son BC = a = 8 tn, AC = b = 8 tn y AB = c = 13 m. Halle la longitud (en m) de la bisectriz interior relativa al lado b. A)2ó/2Í 35.- En un cuadrilátero circunscriptible ABCD, AB = a, BC = b, CD = d y AD — d, además el ángulo que forman las diagonales mide 150°, determine el área de la región cuadrangular ABCD. A) ac B) bd C) ac + bd (ac + db\ „.(ac + bd —2—/ E)\-----4 36.- En un cuadrilátero circunscriptible ABCD, AB = a, BC = c, CD = dy AD = d, se cumple: a ad -c = b-d. determine: M - be A) tany B)2tany C)tan"y D)coty E)cot"y 32.- En un AABC, la bisectriz exterior relativa al lado b mide 576 m. Calcule la medida de la bisectriz interior relativa al mismo lado (en ni), además se tiene mZC - mZA. = 32° 37.- En un cuadrilátero inscriptible ABCD, AB = a, BC = b, CD = c. Si: a - c — b - d. A) 187 B)144 C)198 D) 188 E) 168 33.- En un A ABC de lados a.byc donde ha es la altura relativa al lado A, si a = /i„, enton- ces: sen(B + C) E senBsenC ’eS’ A) 2 B)3/2 C)1 D) 1/2 E)l/4 CUADRILÁTEROS 34.- Se tiene un cuadrilátero circunscriptible ABCD de lados AB — a, BC = ¿, CD = J y AD = d, se cumple: >(C cof — I 2 determine: M —-----/— be A) ad B) -Mf C)y^ 7 (bc)~ be a<! (be)2 ™*(bc? E) ad 38.- En un cuadrilátero inscriptible ABCD, AB = a, BC - b, CD = c, AD = d, se cumple: a - c = b + d, además: 4sen A + 3 cos A = 5, íbe A e (0; 71/2), determine: M= J—~ A) 0,5 B)1 C)4 D)0,25 E)2 B ÍB+D| D seny . senl —— I = seny, determine “ ab" si el área de la región cuadrangular es 5 2 cm . A) 10 B)5 015 D)2,5 E)20 Resolución de Triángulos Oblicuángulos
FORMA POLAR 01.- Luego de expresar en su forma trigono- métrica, indique el argumento del siguiente nú- mero complejo. tan" 0 i 7t ------- +---ñ-----Z ;0<6< — l + tan-0 taae + cote 2 04.- Siendo “k" un número par, reducir las si- guiente expresión: I 7t 27t ] cos + ÍCOS— l 10 5 J kn A) (-1) 2 2sen — 3n n 'Y sen---isen— 5 10J k-2 kn D) (-1) 2 2/cos — A) 6 Tt D)-+e Tt B)6- - E) 7t -0 7t c)--e -ir. líTL B)(-l)2k2isen — k-2 kn C) (-1) 2 2isen — k-2 ¡íTL E) (-1) 2 2cos— 02.- Dado: Z = (1 - sen 6 + i cos 6)4; hallar la variación de “6” de tal manera que el argu- mento principal de Z este comprendida en el intervalo [0; 2n). 05.- Utilizando propiedades y definiciones de números complejos, determine tan 4x como una función racional de tan a. A)6e C) Ge Tt Tt 2'2 B)6e Tt _ Tt 2’2 Tt _ Tt 2’2 E) Ge [-Tt;Tt] D)6e Tt _ Tt 2’2 03.- Indique un equivalente de la siguiente ex- presión: ( cota+íY1 f cota-i M= ------- + ------- ^cota-i I l cota+i A) sen 2a B) cos 2a C) 2 sen 2a 4 tanx — 4tan3x 16tan2x + tan4x tanx—tan3 r l-6tan2x + tan4x 4tanx + 4tan3 x 1—6tan2 x + tan4 x 4tan3 x—4tanx 1—6tan2x + tan4x 2tanx-2tan3 x l-6tan2 x + tan4 x D) 3 sen 2na E) 2 cos 2«a Problemas de Trigonometría y cómo resolverlos -Sa RACSO WlDITOkll
06.-Halle la paite imaginaria del número complejo: i + senx + i cos x 10.- El complejo. W = . ; Vx l + cos3x + isenx W= , 1 + cosx+isenx i + senx—icosx 16 e R, se puede expresar de la forma: re , en- tonces el valor de: r - tan 6 es A) sen 2x - cos 2x B) sen 2x - cos x A) senx B)cscx C)cosx/2 C) cos2x-cosx+1 D) senx- sen 2x D)secx/2 E)6 E) sen2x - senx 11.- Sea el complejo z/z = re,n/36 obtener el 571 O7.-Si:z = cis6- — <6<-27t, valor de: , - \2 . . 2, .1 z- z 1 M-sec (nrgz)+ 1 z + - 1 l-(z)2 calcule: . 2 1 + z A)0 B)-l C)1 D)i E)-i 12.- Calcular el valor de: sen (iLni) A) tan 6 B) - tan 6 C) cot 6 A)l/e B)-l/e C) 1/rt D)-l/n E)-l D) - cot 6 E) - tan(6/2) 08.- Utilizando complejos degradar: cos5G. 13.- Representar el número complejo: Z = sen 2a - i cos 2a, en la forma exponencial. 5 5 1 A) ~ cos 6 -7~ cos 36 + — cos 56 8 16 16 A)-ci(2a-n) B)ei(2a+7t) cj^ ¡(20+45) 5 5 1 B) ~ cos G + 77 cos 36+7“ cos 56 8 16 16 D)el 2' E)e'2 ' 14.- Siendo z un número complejo, calcular: 5 1 5 C) ~ cos 6+77 cos 36-77 cos 56 8 16 16 cos 2z, sabiendo que: cos z = 2 A)4 B)5 C)6 D)7 8 5 5 1 D) ~ cos 6+77 cos 36 + 77 cos 56 0 lo lo Tt Tt 15.-Si:Z = cos— +isen ,W= 1 + iZ, hallar 4 4 el argumento principal de W°. 5 5 1 E) -7 cos G-77cos3G + 77 cos5G 8 16 16 Tt Tt Tt Tt Tt A) 7 B)37 C)97 D)ll— E)137 FORMA EXPONENCIAL 09.- Resolver: 16.- Sabiendo que V n e N se cumple: í e' Ln(l + cosG + -isenG) = 1+Ln| 2cos— l 2, A) 2 B)i C)1 D)2í E)-i x~,1+l - i=(x-l)7l [(x - xk)(x - xk )], donde: k=l . 2kn 1 xk= e 2,1+1 y xk : conjugada de xk ¿qué suce- de cuando x toma el valor de 1? Estudio de la Trigonometría con Números Complejos 873
í kn \ V2n + 1 k=o l2n + lj 2 ( kn ] V2n + 1 k=i l2n + lj 2 n ( kn } V2n + 1 c)isenl.^J=^_ n f 2kn 'j -j2n + \ k=i t2n + 1J 2 ( 2kn -J2n + 1 E) 71 sen 7 = z— k=i V2n + 1J 2 17.- Expresar el numero complejo: 18.- Encontrar las raíces cuadradas de: -8í - 15, indicar uno de ellos. A)4í-l;l-4i B)3i-l;l-3t C)2í-2;2-2i D)5i-l;l-4í E)4i-2;2-4i 19.- Hallar el módulo del número complejo (1 +i)i,kc Z. 71 „ 71 ,71 --+ 2kn 2kn+— kn— A) e 4 B) e 4 C) e 4 2kn+— +2krt D) e 2 E) e 2 20 .- Dado el número complejo: indicar el gráfico que mejor representa al nú- mero: W = Ze' con0<tx< — ; J>0. ' 'o 21 .- Al simplificar: (1+i tana)11 (1-i tana)” -2nai , rji . e A) 2i sen(2na) B) 2 tan(na) C)-2í D) tan(na) E)0 22 .-Calcular: cot — l 21 J i- r A)-l B)-V3 C)1 D)— E)V3 23 .- Calcular la parte maginaria del complejo cosix+isenix “z”, si: z=------------- 2i A)e B)-e C)l/2e D)-l/2e E)2e 24 .- Resolver: sec(ix) + i csc (ir) = 4 ícsc(2ix) A)-Ln2B)Lw4 C)Ln(-2) D)2Ln2 E) Ln2 Z = Je” , 0<e< - ; 4 ^Mracso wy iDiTcm PP21 874 Problemas de Trigonometría y cómo resolverlos
cot6 + í 25 .- Dado el número complejo z = ~ , 1-icote halle Re(z6) A) cos 1206 B) sen 126 C) cot 66 D) tan 66 E)-1 26 .- Dado el número complejo: z = x + iy, deter- minar el área de la región: R= (ze C/|z|< 1 Almz<cos(Rez)} A)ti/8 B)7t/5 C)ti/3 D)7t/2 E)n 27 .- Simplificar la expresión: l-Ze-^+e-** K =-------------r +2 (cos6-ísen6) A) 2 cos26 B)cos 26 C) -2 cos 26 D) - cos26 E) sen 26 a+bi 28.-Si: — a-bi valor de tan a. =eia; a, b, a e R, determine el ab 2ab A) 2 » 2 a +b B) 2 .2 a + b 2 A2 a —b 2ab 2 í2 a —b 2 » 2 a — b E) 2ab 29 .- Hallar la parte ral de: z = A) C) E) 3x cos— 2_ cosx cos2x B) 3x cos— 2_ cosx 3x sen— 2_ senx X .eos— 2 D) sen3x senx 3x cos— 2_ cosx X sen— 2 eos — 2 eos— 2 30 .-Determine la parte real del complejo: 1+icoteY2 n 1-icoteJ ,s*®_ 15 A) 4$ +1 B) 7? - 1 C) 42 +1 D) 41 - 1 E) 41 + 2 ?3G-1 31 .- Para la siguiente expresión W = —- se pide hallar Re(W) A)-l/2 B)-1/4 C) 1 D) 1/4 E) 1/2 32 .- Hallar el complejo equivalente a: A) i tan 6 B) 2/ cot 6 C) i cot 6 D) rsenfi E) i cos 6 REGIONES SOMBREADAS 33 .- La región sombreada: i3x e 7^ Se puede expresar como: 71 2ti A) l<|z|<2 a — argz< Tt 2rt B) 1 < | z | < 2 a y < arg z < "y Tt 271 C) 1<|z|<2a — < argz< — Estudio de la Trigonometría con Números Complejos PP21 875
71 2ti D) 1 <|z|<2a — <argz< — 2tc E)1<|c|<2a< argz< — 34 .- Siendo Zk un número complejo de la forma: Zk = cos(AG) + i cos 2(á:6). Hallar la región correspondiente para Z,, de modo tal que |Z( | < 1. 35 .- Hallar el conjunto de los números com- plejos que describen la región mostrada, sa- biendo además que el área de dicha región sombreada (S) es óTtp A) S = |z G C/2 < |z| <4 a 3— < arg z < 7t B)S= |zG C/2<|z|<4a3— <argz<n Ín ze C/4<|z|<8 a3- <argz<n 171 zg C/4<|z <8a3— <argz<n> 4 IJS= |zg C/2<|z|<8a3— <íirgz<7t| 36 .- Calcular el área de la región sombreada por: í . , 3rt 1 R = |Zg C7Inl(Z)<sen[Re(Z)] a|Z|< — | 9 , •, 9 •, 3 9 2 , A) 77tX B)~7tp 4 4 8 9 3 3 9 , , D)-?fp3 E)— tQT o lo 37. ¿Cuál de las gráficas corresponde al si- guiente conjunto de números complejos: R={Ze C/l^+Re^^OvlZl^ l;Re(Z)<0) 38.- Sea la región R definida por: R={z=reiee c/arg(z3)e |7t/4,57t/4] y 0<R1<r<R2} halle el área de la región R. A)(7tf6)(R22-R12) B)(ti/4)(2R22-Ri2) Q(71/3XR22-Ri2) D)(2t73)(R22-R|2) E)(7tf7)(R22+RI2) RACSO Pbditokb* PP21 876 Problemas de Trigonometría y cómo resolverlos
; i i i i i 1 । । i i । Limites y Derivadas: zTrigohpmétiJcos LIMITES TRIGONOMÉTRICOS 01.- Determinar el valor de la siguiente expresión: A)-12 B)-6 C)-18 D)24V3 E)-24 06.- Cuando “x” se aproxima a cero, la expresión: cosx 3n ;cuandoxse aproxima a: — cos7x-cos5x ------2-----; es igual a. sen—reos— 2 2 A)-l B)-l/2 C)-12 D)-6 E)-18 A)-V B)—y2 C)~V 07.- El siguiente límite: lim x—»y cos2x-cos2y D)-2-j2 E)V2 02.- Calcular: A) -sen P í cosot-cosB'l senP lim <x-»P B) senf a-P ? C)-2 D) 2 sen P E) cos P 03.-Evaluar: lim tan2x.cot x + — n 1 4 1 *-»- k k )) 4 A) 1/4 B) 1/3 C) 1/5 D) 1/2 E) 4 04.-Determinar: lim 71 X »— 3 A) 1 B) y[3 C)-yl r l-2cos senl x — 3 D)2 X 3 1 J E)-l es equivalente a: A) -2 sen 2y B) -3 sen Iy C) -sen 2y D) -2 sen y E) -sen y 08.- Dada la siguiente expresión, calcular: 2x— ----—2 + lim (crcsen3x cosx x-»— x—— x~>0 senx 1-x 14 14 A) 1 B) 2 C) 3 D) 4 E) 5 09.- Determinar (a + P), si : a= lim n 05.- Encontrar: cosx A R-iüp sen3x.sen5x x_n x+0 (x-x3)2 < 2j A) 16 B)15 C)14 D)13 E)0 Límites y Derivadas Trigonométricos PP22
10.- Calcular: DERIVADAS lim x—>2 tan (2nx)+cos (0, 5tix) + tan (0, 37tx) x~ + 4x-12 71 A)li 3?t 5?t 7ti 9tí B) — C) — D) — E) — ' 12 7 12 ’ 12 7 12 cosx 11.- Si:/(x) = —~, ¿cuál de las alternativas es correcta? A) lim fíx) = 1 x—>0 B) lim /(x) = +<=o x->0 C) lim fix) = +<=o x—>0“ D) lim y(x) = +°o x—>0 E) lim fix) = I 71 X—»— S7 15.- Si :/(x) = senx, hallar/ (x), (57 ésima derivada) A) senx B)cosx C)-senx D) - cos x E) 0 2 16.- Dada la función:/(x) = arc cos (x ) ; cal- cular : /'(O) A)0 B) 1 C)-l D)0.5 5)0,5 17. Si: /(x) - "I tan3x - tanx + x. Calcula:/'(x) 2 3 A) tanx B) tan x C) tan x D) tan4x E) 2tan*x 12.- Determinar el verdadero valor de ‘M” cuan- í 271^1 do “O” se aproxima a I I, si: 18.- Calcular el valor de: Tt 16 sabiendo que:/(x) = sen2x tanx z, 27t) 2sen 6 l 3 -sen 3 ) Ivl — z tan 0 7t 2 3 A) V2 + V2 B) V2-V2 C)--J2 + V2 A)-2 B) cero C)2 D)1 E)-1 13.- ¿A qué valor se aproxima “M”, cuando **x” se aproxima a cero, si: 19.- Calcular la derivada enésima de: y = cos3x 3 A)i 31 I -> 1m I I eos 3x+— +cos I 2 I H7t 2 vl + xsenx-cosx M=-----------------? > x sen“ — 2 A)4 B)2 C)1 D)-l E)-3 14.- Calcular el valor de “E” cuando “x” se aproxima a cero. 2—-Jcosx -cosx 3 B)I 3’ I -> nn 1 I sen 3x + —- +sen 2 I I H7t T 3 -in-i í •> «ni í nn ± 3 sen 3x + — +sen x + — 4 l 2 J I 2 3. 3n 'cosí 3x + —| + cosí x+ — 2 L l 2 I I 2 A) 3/2 B) 1/2 C) 1/4 D)3/4 E)2/3 3 _n_¡ í _ nn ] f nn _ 3 eos 3x+— +cos x + — 4 L l 2 J I 2 RACSO DITOKBI PP22 878 Problemas de Trigonometría y cómo resolverlos
REGLA DEL’HOSPITAL A)0 B)1 C)2 D)3 E)4 20.- Calculan lim n x—>— 2 sen(senx + cos x -1) sen(cos x) A)-l B) 1 C)2 D) 1/2 E)-2 21.- Calcular el valor de: lim sen3x + cos—cot-----— sen — 2 A 4 4) 2 senx—1 A)-6 B)-9 C)-l/9 D)-8 E)-l/6 22.- Calcular el valor de: APLICACIONES DE LA DBRI1 'ADA 27.- ¿Para cuáles de los siguientes intervalos la función/» es decreciente, si: sen2r /x) = senr+—— ? ln \ / 5tt _ \ / 71 A)^-^y b)\T'27 c)\» /tc 5ti\ ln 5n\ D)\3’T/ E)\4’T/ 28.- Calcular el máximo valor que toma la función: lim x—>0 senx—x cosx x(l-cosx) y = 2 cos — + 3 cos ~ A) 1 B)5 C)4 D)2 E)3 A) 2/3 3 B)3/2 C) 1/3 D) 1/2 E)4/ 23.- Evaluar: nrcsen((/i + l)x) 3x 29.- Al graficar la función: y = 2 sen 2x + sen 4x; ¿para cuál de los siguientes valores de “x” la función toma su máximo valor? A) ti/3 B)7tc/6 C) 4ti/3 D)11ti/6 E) tt/6 A) — n b)3 1 C)3 1 D,I 4 E)i 30.- Determine sobre la curva: y = arc tan x, un ín o 711 puntoqueesté más próximo al punto B;2 + — 24.- Calcular: e2x — 1 arcsen3x A) 1/4 B) 1/5 C) 2/5 D) 1/3 E) 2/3 7C *6 7 25.- Calcular el siguiente límite: lim x—>0 xsenx 1—cosx 31.- Las diagonales de un cuadrilátero son: A)1 B)2 C)3 D)4 E)5 26.- Evaluar: lim 71 2 2sen(cos x) + sen(2cos x) cosx 71 4 los cuales forman el ángulo (tc - a). Calcular el valor de mínimo de dicha región cuadrangular. 2 sen y cos a - sen a, V6 A)v V6 B)--- 18 C) n/6 7 Límites y Derivadas Trigonométricos PP22 879
2^3 D)~?~ 3^2 H 9 intervalo ;0 3 se caracteriza por ser: 32.- Calcular el máximo valor de la función (x) definida por la regla de correspondencia: 7 fix) = sen"x sen 2x B)^V3 A) No creciente B) No decreciente C) Cóncavo hacia arriba D) Cóncavo hacia abajo D)v 3 E) 16 E) Estrictamente decreciente 33.- Hallar el ángulo, bajo el cual se cortan las curvas dadas por:/(x) = sen x, g(x) = cos x, xe [O;2tt] A) - arc tan( V2 + 2) 37.- Un estudiante quiere conseguir la tabla (AB) más corta posible a fin de llegar al edifi- cio tal como se muestra en la figura (la tabla está apoyada en un muro de altura lí). Calcular la longitud de dicha tabla. B) - arc tan(2-Jl ) C) - arc tan(3 -Jz ) A) [h2 + d2)y2 B) (/i3 + r/3)1/3 ch/^W-V2 D) (/i3/2 + d372)273 E) (A273 + d273)172 34.- Encontrar el mínimo valor de la función: fix) = arc tan 1-x 1+x ; xe IP;1] 38.- Un ingeniero diseña un estanque cuyo corte de sección tiene la forma mostrada en la figura, donde: AB = BC — CD = L. ¿Qué valor le debe asignar a “0" para que en el estanque se pueda depositar el mayor volu- men posible de agua? A)-ti/4 B) -7t/2 O-ti/8 D) O E) 71/4 35.- Dos móviles parten simultáneamente des- de un mismo punto y en direcciones opues- tas, sobre una pista circular de radio 140 m con rapidez constante de 3ti m/s y 4n m!s ¿Con qué rapidez se están alejando el uno del otro al cabo de 10 5 de haber partido? A) 37° B)45° C)53° D)60“ E)75° 39.- Los lados de un triángulo son: (sen 0 + cos 0) y . Sn-Jz A) —-—hi/v B)37tniA O47twi/$ 2 eos los cuales forman el ángulo “0”. Calcular el mínimo valor de dicha región triangular. D)—-—in/s E) 5n>n/s 36.- La función:/(x) = cos x -1/2 cos. 2x en el V6 A)v V3 V6 B)1Í C)IT PP22 880 V3 V6 D)---- E)------- Problemas de Trigonometría y cónfb resolverldfi -j» RACSO
CLAVES DE RESPUESTAS CAPÍTULO 1: SISTEMAS DE MEDIDA ANGULAR 01.- C 02.- C 03.- A 04.- B 05.- B 06.- B 07- C 08- C 09- B 10- D 11- C 12- B 13- E 14- C 15- C 16- D 17- C 18- B 19- B 20- D 21- D 22- D 23- D 24- A 25- C 26- A 27- A 28- C 29- A 30- C 31- C 32- B 33- C 34- C 35- D 36- D 37- E 38- D 39- E 40- C CAPÍTULO 2: LONGITUD DE ARCO 01-A 02-C 03-B 04-C 05-C 06-C 07-D 08-B 09-C 10-C 11-C 12-B 13-D 14-A 15-B 16-B 17-B 18-E 19-B 20-E 21-A 22-E 23-E 24-C 25-E 26-B 27-A 28-C 29-C 30-B 31-C 32-B 33-D 34-A 35-C 36-C 37-B 38-B 39-B CAPÍTULO 3: RAZONES TRIGONOMETRICAS DE ÁNGULOS AGUDOS 01- A 02- D 03- B 04- C 05- B 06- C 07- D 08- E 09- C 10- C 11- B 12- A 13- E 14- D 15- D 16-E 17-E 18-A 19-A 20-C 21-C 22-C 23-B 24-D 25-B 26-B 27-C 28-C 29-B 30-A 31-D 32-C 33-B 34-D 35-C 36-B 37-A 38-D CAPÍTULO 4: RESOLUCIÓN DE TRIÁNGULOS RECTÁNGULOS 01- A 02- C 03- B 04- D 05- B 06- C 07- A 08- B 09- B 10- A 11- D 12- B 13- D 14- B 15- C 16- B 17- B 18- C 19- B 20- C 21- E 22- B 23- E 24- A 25- C 26- D 27- C 28- B 29- C 30- C 31- A 32- D 33- C 34- C 35- C 36- D 37- C 38- B 39- C CAPITULO 5: RAZONES TRIGONOMÉTRICAS EN SITUACIONES CONTEXTUALIZADAS 01-D 02-C 03-E 04-B 05-B 06-B 07-B 08-C 09-C 10-E 11-E 12-C 13-A 14-A 15-B 16- B 17- D 18- B 19- E 20- A 21- C 22- D 23- A 24- E 25- B 26- A 27- E 28- C 29- A 30- C 31-C 32-C 33-C CAPÍTULO 6: RAZONES TRIGONOMÉTRICAS DE ÁNGULOS EN EL PLANO CARTESIANO 01-B 02-C 03-A 04-B 05-E 06-E 07-A 08-A 09-B 10-A 11-B 12-C 13-C 14-E 15-C 16-A 17-A 18-B 19-D 20-A 21-A 22-E 23-B 24-A 25-B 26-B 27-B 28-B 29-B 30-C 31-A 32-D 33-B 34-D 35-A 36-A 37-E 38-D 39-B 40-A 41-D CAPÍTULO 7: CIRCUNFERENCIA TRIGONOMÉTRICA (RAZONES TRIGONOMÉTRICAS DE NÚMEROS REALES) 01-C 02-A 03-E 04-C 05-C 06-D 07-D 08-E 09-B 10-A II-C 12-D 13-D 14-C 15-E 16- D 17- A 18- D 19- A 20- C 21- E 22- E 23- D 24- C 25- B 26- C 27- E 28- A 29- B 30- E 31- E 32- C 33- A 34- D 35- C 36- A 37- A 38- D 39- E CAPITULO 8: IDENTIDADES TRIGONOMÉTRICAS 05-E 06-A 07-C 08-B 09-C 10-C II-A 12-C 13-A 14-A 15-A 16-A 17-A 18-A 19-D 20-A 21-C 22-B 23-A 24-A 25-C 26-B 27-C 28-D 29-B 30-D 31-A 32-C 33-C 34-A 35-B 36-A 37-C 38-A 39-B 40-A 41-C 42-B CAPÍTULO 9: IDENTIDADES TRIGONOMÉTRICAS DE ARCOS COMPUESTOS 01-C 02-B 03-E 04-B 05-A 06-A 07-B 08-C 09-B 10-D 11-A 12-C 13-A 14-E 15-C 16- B 17- B 18- E 19- D 20- E 21- E 22- A 23- E 24- E 25- D 26- E 27- C 28- A 29- A 30- E 31- A 32- D 33- B 34- A 35- A CAPÍTULO 10: REDUCCIÓN AL PRIMER CUADRANTE 01- E 02- D 03- B 04- D 05- E 06- B 07- A 08- E 09- E 10- E II- D 12- B 13- E 14- B 15- E 16- B 17- A 18- D 19- B 20- B 21- D 22- A 23- C 24- B 25- D 26- B 27- D 28- C 29- B 30- B 31-D 32-C 33-E 34-A 35-A 36-E 37-C 38-B 39-A 40-A CAPITULO 11: IDENTIDADES TRIGONOMÉTRICAS DEL ARCO DOBLE 01-B 02- A 03-A 04-C 05-E 06-A 07-C 08-B 09-C 10-D 11-B 12-D 13-A 14-C 15-C 16- A 17- B 18- D 19- C 20- E 21- D 22- A 23- D 24- D 25- A 26- B 27- A 28- B 29- D 30- A 31-C 32-C 33-B 34-B 35-B 36-C 37-A 38-C 39-B
CAPÍTULO 12: IDENTIDADES TRIGONOMÉTRICAS DEL ARCO MITAD 01.-E 02.-E 03.-A 04.-D 05.-B 06.-A 07.-C 08.-E 09.-E 10.-E 11.-A 12.-E 13.-E 14.-A 15.-A 16.-B 17.-B 18-B 19-C 20-C 21-B 22-A 23-B 24-D 25-C 26-E 27-D 28-A 29-E 30-D 31- B 32- B 33- A 34- E 35- E 36- B 37- C 38- E 39- B 40- E 41- A CAPÍTULO 13: IDENTIDADES TRIGONOMÉTRICAS DEL ARCO TRIPLE 01- B 02- B 03- E 04- A 05- D 06- D 07- A 08- B 09- D 10- A 11- E 12- C 13- E 14- C 15- D 16-B 17-E 18-B 19-E 20-D 21-A 22-D 23-B 24-C 25-A 26-A 27-A 28-B 29-A 30-E 31- C 32- B 33- E 34- A 35- D 36- D 37- D 38- E 39- B 40- D 41- A 42- E CAPÍTULO 14: TRANSFORMACIONES DE SUMAS O DIFERENCIAS A PRODUCTOS 01-A 02-A 03-C 04-A 05-A 06-A 07-D 08-D 09-B 10-A II-D 12-C 13-B 14-A 15-A 16-C 17-D 18-D 19-B 20-C 21-C 22 - B 23-B 24-C 25-D 26-D 27-B 28-C 29-B 30-C 31-B 32-C 33-E 34-A 35-E 36-C 37-D 38-B 39-E 40-D 41-C 42-C 43-E CAPÍTULO 15: TRANSFORMACIONES DE PRODUCTO A SUMAS O DIFERENCIAS 01- C 02- A 03- A 04- D 05- A 06- A 07- D 08- C 09- D 10- A 11- E 12- D 13- C 14- A 15- A 16-C 17-B 18-E 19-E 20-B 21-C 22-C 23-D 24-C 25-C 26-B 27-E 28-A 29-D 30-C 31- A 32- E 33- D 34- E 35- E 36- D 37- D 38- B 39- B 40- B CAPÍTULO 16: SUCESIONES Y SERIES TRIGONOMÉTRICAS 01-A 02-B 03-D 04-E 05-B 06-D 07-C 08-B 09-A 10-A II-E 12-D 13-E 14-A 15-C 16-E 17-A 18-D 19-A 20-D 21-A 22-D 23-D 24-A 25-D 26-E 27-A 28-A 29-E 30-A 31-E 32-E 33-A 34-E 35-D 36-A 37-C 38-A 39-A CAPÍTULO 17: FUNCIONES TRIGONOMÉTRICAS 01-A 02-D 03-B 04-E 05-D 06-D 07-D 08-A 09-D 10-A 11-E 12-B 13-D 14-A 15-C 16-C 17-E 18-B 19-A 20-D 21-D 22-C 23-D 24-A 25-B 26-B 27-C 28-B 29-D 30-A 31-A 32-B 33.-C 34-E 35-C 36-B 37-C 38-B 39-A 40-E 41-C 42-C 43-C 44 A 45-A 46-B 47-A 48-A 49-B 50-C 51-A 52-D 53-A 54-D 55-E 56-D 57-D CAPÍTULO 18: FUNCIONES TRIGONOMÉTRICAS INVERSAS 01- C 02- C 03- C 04- E 05- A 06- D 07- C 08- E 09- D 10- B 11- E 12- D 13- D 14- B 15- A 16-C 17-C 18-D 19-D 20-C 21-B 22-D 23-C 24-C 25-A 26-A 27-E 28-A 29-B 30-D 31-C 32-D 33-C 34-D 35-D 36-E 37-B 38-B 39-C CAPITULO 19: ECUACIONES E INECUACIONES TRIGONOMÉTRICAS 01-C 02-C 03-A 04-E 05-C 06-B 07-B 08-B 09-E 10-B II-D 12-D 13-B 14-D 15-A 16-C 17-C 18-C 19-A 20-A 21-D 22-D 23-B 24-D 25-C 26-D 27-B 28-A 29-E 30-E 31- D 32- B 33- C 34- A 35- B 36- E 37- D 38- D 39- B 40- A CAPÍTULO 20: RESOLUCIÓN DE TRIÁNGULOS OBI ICUÁNGULOS 01- C 02- C 03- E 04- D 05- C 06- B 07- C 08- A 09- C 10- E 11- C 12- B 13- D 14- B 15- A 16-A 17-C 18-D 19-C 20-D 21-B 22-A 23-A 24-E 25-D 26-C 27-B 28-C 29-A 30-B 31- D 32- E 33- C 34- B 35- E 36- C 37- B 38- E CAPÍTULO 21: ESTUDIO DE LA TRIGONOMETRÍA CON NÚMEROS COMPLEJOS 01-C 02-B 03-E 04-B 05-A 06-A 07-B 08-D 09-D 10-B 11-C 12-E 13-D 14-D 15-A 16-B 17-D 18-A 19-A 20-D 21-E 22-C 23-D 24-E 25-E 26-E 27-D 28-D 29-B 30-B 31- E 32- A 33- C 34- A 35- D 36- C 37- E 38- A 38- E 39- C CAPÍTULO 22: LÍMITES Y DERIVADAS TRIGONOMÉTRICOS 01-B 02-A 03-D 04-B 05. E 06-C 07-A 08-E 09-C 10-C 11-D 12-B 13-A 14-D 15-B 16-A 17-D 18-D 19-E 20-B 21-B 22-A 23-A 24-E 25-B 26-E 27-D 28-B 29-E 30-E 31- B 32- A 33- B 34- D 35- D 36- A 37- C 38- D 39- C
BIBLIOGRAFÍA 01.- Trigonometría James Gehrmann y Thomas Lester Carbajal S.A. - 1988 02.- Análisis Matemático L.S. Pontriaguin Paraninfo S.A. Madrid - 1983 03.- Funciones Numéricas Edgar de Alencar Filho Luraria Novel S.A. 1985 04.- Análisis Matemático Segunda Edición T.M. Aposto! Editorial Reverté S.A. - 1993 05.- Trigonometría Contemporánea H. E. Taylor T.L. Wade Editorial Limusa- 1976 06.- Trigonometría Plana heineman. Richard E. Libros Me Graw - Hill de México - 1973 07.- Trigonometría Moderna TOMO I C.W. Lucas y R.T. James Editorial Hispano - Americana - 1973 08.- Trigonometría con Aplicaciones Patrick J. Boyle Editorial Mexicana - 1983 09.- The Circular Fuctions Clayton W. Sodge. Prentice - Hall, Inc - 1996 10.- Plañe and Advanced Trigonometry E. W. Hobson. DIRECCIONES WEB 1) maestro.ucsc.cl/educacion/especiales/documen- tos/LJnidad%202Geometr%EDa.doc - ...medición angular 2) http//www.pntic.mec.es/Descartes/Bach_CNST_ 1/ Razones_tngonometri-cas_operaciones_identidades/ angcual3.htm#Circunferencia 3) http://www.hp49gcompetition.com/uk/calculus/ 4) dcc.puc.cl/gente/usuarios/beto/yud9.pdf - Longitud de Arco. 5) w w w.j untadeandal ucia.es/.. ./iesarroyo/ matematicas/materiales/4eso/geometria/ trigonometria/trigonometria.htm - 32k - ... Razo- nes trigonométricas de un ángulo cualquiera. 6) thales.cica.es/cadiz/edomac/trigonometria.pdf - ... Calculo de las razones trigonométricas 7) personal5.iddeo.es/ztt/For/F7_Triangulos.htm - 14k- ... Resolución de triángulos 8) descartes.cnice.mecd.es/Geometria/Triangulos/ - 86k- Resolución de Triángulos Rectángulos. 9) usuarios.lycos.es/arquillos/tema01 BC.htm - 25k - Relación entre las razones trigonométricas de án- gulos de diferente cuadrante... 10) www.ucm.es/info/Geofis/practicas/ trigonometria.htm - 24k - Circunferencia Trigonométrica 11) usuarios.lycos.es/calculo2 lZid352.htm - 32k -... Funciones Trigonométricas de los números reales... 12) www.cnice.mecd.es/Descartes/ach_CNST_ 1 / Razpnes_trigonomctricas_operaciones_identidades/ identid.htm - 8k - Identidades trigonométricas 13) usuarios.lycos.es/calculo21/id355.htm - 14k - Números complejos. Ecuaciones trigonométricas. Misceláneas 14) www.sectormatematica.cl/proyectos/ reduccion.htm - 18k - Reducción de las Funciones Trigonométricas. Al Primer Cuadrante. 15) usuanos.lycos.es/arquillos/lema02BC.htm - 18k Transformaciones de sumas y diferencias en pro- ductos. 16) www.monografias.com/trabajosl 1/traaprox/ traaprox.shtml - 64k - Series trigonométricas 17) mal .eii.us.es/miembros/rogodi/calculo/ lema2_04-05.pdf - ... Sucesiones de funciones.
Impreso en los talleres gráficos de MAQUETI E.I.R.L, Jr. Carlos Arrieta 1319, Santa Beatriz, Lima 1
NUEVA COLECCION RACSO Problemas de Lazcnamientt Hatematicc Probwmab de PBOBLtMAB DE PRDMl r MAS DE «ética PROBLEMAS DE tuír.aca PJJOBLEMAB DE i!<*eóra Más de una década aportando cultura PRuhlem»u DE Iristnciiietiia Calle Pira 650 - Los Olivos Telf.: 522-1634 E-mail: racsoeditores@peru.com